Mécanique - fondements et applications [7 ed.] 978-2-10-072189-4

2,417 306 16MB

French Pages 827 Year 2014

Report DMCA / Copyright

DOWNLOAD FILE

Polecaj historie

Mécanique - fondements et applications [7 ed.]
 978-2-10-072189-4

Citation preview

Mécanique Fondements et applications Avec 320 exercices et problèmes résolus José-Philippe PÉREZ Professeur émérite de l’Université Paul-Sabatier de Toulouse

Avec la participation de Olivier PUJOL Maître de conférences à l’Université de Lille

7e édition

4 7 2 3

2 7

© Dunod, 2001, 2006, 2014 5 rue Laromiguière, 75005 Paris www.dunod.com © Masson, 1984, 1989, 1991, 1995, 1997 ISBN 978-2-10-072189-4

Table des matières

Avant-propos . . . . . . . . . . . . . . . . . . . . . . . . . . . . . . . . . . . . . . . . . . . . . .

xii

Les grands noms de la mécanique . . . . . . . . . . . . . . . . . . . . . . . . . . . . . . . . . . . .

xiv

Constantes physiques, notations et symboles . . . . . . . . . . . . . . . . . . . . . . . . . . . . . .

xviii

. . . . . . . . . . . . . . . . . . . . . . . . . . . . . . . . . .

xviii

II . — Constantes physiques . . . . . . . . . . . . . . . . . . . . . . . . . . . . . . . . . . . . .

I . — Constantes fondamentales

xviii

III . — Constantes du système solaire . . . . . . . . . . . . . . . . . . . . . . . . . . . . . . . . .

xviii

IV . — Notations . . . . . . . . . . . . . . . . . . . . . . . . . . . . . . . . . . . . . . . . . . .

xx

V . — Alphabet grec . . . . . . . . . . . . . . . . . . . . . . . . . . . . . . . . . . . . . . . . .

xxii

VI . — Multiples et sous-multiples en notation scientifique . . . . . . . . . . . . . . . . . . . . . .

xxii

Description de l’ouvrage . . . . . . . . . . . . . . . . . . . . . . . . . . . . . . . . . . . . . . . .

xxiii

La mécanique en vingt questions . . . . . . . . . . . . . . . . . . . . . . . . . . . . . . . . . . . . xxvii

1. Calcul vectoriel. Torseurs. Analyse dimensionnelle I . — Espace vectoriel . . . . . . . . . . . . . . . . . . . . . . . . . . . . . . . . . . . . . . .

1

II . — Espace affine . . . . . . . . . . . . . . . . . . . . . . . . . . . . . . . . . . . . . . . . .

2

III . — Opérations sur les vecteurs . . . . . . . . . . . . . . . . . . . . . . . . . . . . . . . . . .

3

IV . — Vecteur lié et système de vecteurs liés . . . . . . . . . . . . . . . . . . . . . . . . . . . . .

8

V . — Champ de vecteurs antisymétrique . . . . . . . . . . . . . . . . . . . . . . . . . . . . . .

11

VI . — Torseurs . . . . . . . . . . . . . . . . . . . . . . . . . . . . . . . . . . . . . . . . . . .

12

VII . — Analyse dimensionnelle . . . . . . . . . . . . . . . . . . . . . . . . . . . . . . . . . . . . Exercices et problèmes . . . . . . . . . . . . . . . . . . . . . . . . . . . . . . . . . . . . . . . . . .

15 20

2. Cinématique du point mobile. Vitesse de rotation d’un repère I . — Cadre spatiotemporel de la cinématique galiléenne . . . . . . . . . . . . . . . . . . . . . .

22

II . — Cinématique du point mobile . . . . . . . . . . . . . . . . . . . . . . . . . . . . . . . . .

26

III . — Différents mouvements d’un repère . . . . . . . . . . . . . . . . . . . . . . . . . . . . . . Exercices et problèmes . . . . . . . . . . . . . . . . . . . . . . . . . . . . . . . . . . . . . . . . . .

32 36

3. Changement de référentiel I . — Relativité du mouvement . . . . . . . . . . . . . . . . . . . . . . . . . . . . . . . . . . .

40

II . — Composition des vitesses . . . . . . . . . . . . . . . . . . . . . . . . . . . . . . . . . . .

43

III . — Composition des accélérations . . . . . . . . . . . . . . . . . . . . . . . . . . . . . . . . Exercices et problèmes . . . . . . . . . . . . . . . . . . . . . . . . . . . . . . . . . . . . . . . . . .

47 49

vi

Table des matières

4. Dynamique du corpuscule I . — Masse et quantité de mouvement d’un corpuscule . . . . . . . . . . . . . . . . . . . . . . .

53

II . — Loi fondamentale de la dynamique . . . . . . . . . . . . . . . . . . . . . . . . . . . . . . III . — Principe de l’inertie . . . . . . . . . . . . . . . . . . . . . . . . . . . . . . . . . . . . . .

54 59

IV . — Moment cinétique

. . . . . . . . . . . . . . . . . . . . . . . . . . . . . . . . . . . . . .

61

V . — Mouvements rectilignes et mouvements plans . . . . . . . . . . . . . . . . . . . . . . . . .

62

VI . — Troisième loi de Newton . . . . . . . . . . . . . . . . . . . . . . . . . . . . . . . . . . . Exercices et problèmes . . . . . . . . . . . . . . . . . . . . . . . . . . . . . . . . . . . . . . . . . .

65 67

5. Énergétique du corpuscule I . — Puissance et travail d’une force . . . . . . . . . . . . . . . . . . . . . . . . . . . . . . . .

69

II . — Énergie cinétique . . . . . . . . . . . . . . . . . . . . . . . . . . . . . . . . . . . . . . .

71

III . — Énergie potentielle . . . . . . . . . . . . . . . . . . . . . . . . . . . . . . . . . . . . . .

73

IV . — Énergie mécanique . . . . . . . . . . . . . . . . . . . . . . . . . . . . . . . . . . . . . . Exercices et problèmes . . . . . . . . . . . . . . . . . . . . . . . . . . . . . . . . . . . . . . . . . .

76 80

6. Gravitation. Propriétés du champ de gravitation I . — Force de gravitation . . . . . . . . . . . . . . . . . . . . . . . . . . . . . . . . . . . . . .

82

II . — Champ et potentiel de gravitation . . . . . . . . . . . . . . . . . . . . . . . . . . . . . . .

83

III . — Propriétés du champ de gravitation . . . . . . . . . . . . . . . . . . . . . . . . . . . . . .

84

IV . — Calculs de champs et de potentiels . . . . . . . . . . . . . . . . . . . . . . . . . . . . . .

87

V . — Énergie potentielle gravitationnelle . . . . . . . . . . . . . . . . . . . . . . . . . . . . . . Exercices et problèmes . . . . . . . . . . . . . . . . . . . . . . . . . . . . . . . . . . . . . . . . . .

93 96

7. Référentiels non galiléens. Dynamique terrestre I . — Référentiels non galiléens . . . . . . . . . . . . . . . . . . . . . . . . . . . . . . . . . . .

99

II . — Dynamique terrestre . . . . . . . . . . . . . . . . . . . . . . . . . . . . . . . . . . . . . III . — Marées océaniques . . . . . . . . . . . . . . . . . . . . . . . . . . . . . . . . . . . . . .

101 106

IV . — La force de Coriolis terrestre . . . . . . . . . . . . . . . . . . . . . . . . . . . . . . . . .

107

V . — Référentiels accélérés par rapport à la Terre . . . . . . . . . . . . . . . . . . . . . . . . . .

113

VI . — Référentiels inertiels . . . . . . . . . . . . . . . . . . . . . . . . . . . . . . . . . . . . . Exercices et problèmes . . . . . . . . . . . . . . . . . . . . . . . . . . . . . . . . . . . . . . . . . .

116 119

8. Particule chargée dans un champ électromagnétique stationnaire I . — Force de Lorentz . . . . . . . . . . . . . . . . . . . . . . . . . . . . . . . . . . . . . . .

123

II . — Particule chargée dans un champ électrique . . . . . . . . . . . . . . . . . . . . . . . . . .

124

III . — Particule chargée dans un champ magnétique . . . . . . . . . . . . . . . . . . . . . . . . .

127

IV . — Champs électrique et magnétique simultanés . . . . . . . . . . . . . . . . . . . . . . . . .

130

V . — Applications . . . . . . . . . . . . . . . . . . . . . . . . . . . . . . . . . . . . . . . . . Exercices et problèmes . . . . . . . . . . . . . . . . . . . . . . . . . . . . . . . . . . . . . . . . . .

132 135

9. Mouvement d’un corpuscule guidé I . — Corpuscule guidé sur une courbe . . . . . . . . . . . . . . . . . . . . . . . . . . . . . . . II . — Pendule circulaire

138

. . . . . . . . . . . . . . . . . . . . . . . . . . . . . . . . . . . . . .

141

III . — Corpuscule guidé sur une surface . . . . . . . . . . . . . . . . . . . . . . . . . . . . . . . Exercices et problèmes . . . . . . . . . . . . . . . . . . . . . . . . . . . . . . . . . . . . . . . . . .

145 149

vii

Table des matières 10. Oscillateurs harmoniques. Oscillateurs amortis I . — Oscillateurs harmoniques . . . . . . . . . . . . . . . . . . . . . . . . . . . . . . . . . . .

152

II . — Oscillateurs amortis par frottement visqueux . . . . . . . . . . . . . . . . . . . . . . . . .

158

III . — Analogie électrique . . . . . . . . . . . . . . . . . . . . . . . . . . . . . . . . . . . . . .

162

IV . — Oscillateur amorti par frottement solide . . . . . . . . . . . . . . . . . . . . . . . . . . . .

163

V . — Représentation dans l’espace des phases . . . . . . . . . . . . . . . . . . . . . . . . . . .

165

VI . — Oscillateurs paramétriques . . . . . . . . . . . . . . . . . . . . . . . . . . . . . . . . . .

166

VII . — Effets de termes non linéaires . . . . . . . . . . . . . . . . . . . . . . . . . . . . . . . . . Exercices et problèmes . . . . . . . . . . . . . . . . . . . . . . . . . . . . . . . . . . . . . . . . . .

170 174

11. Oscillations forcées. Résonance I . — Exemples physiques d’oscillations forcées . . . . . . . . . . . . . . . . . . . . . . . . . .

178

II . — Oscillations forcées. Résonance . . . . . . . . . . . . . . . . . . . . . . . . . . . . . . . .

181

III . — Excitation maximale indépendante de la pulsation . . . . . . . . . . . . . . . . . . . . . . .

184

IV . — Applications . . . . . . . . . . . . . . . . . . . . . . . . . . . . . . . . . . . . . . . . . Exercices et problèmes . . . . . . . . . . . . . . . . . . . . . . . . . . . . . . . . . . . . . . . . . .

189 193

12. Corps ponctuel soumis à une force centrale conservative I . — Mouvements à force centrale conservative

. . . . . . . . . . . . . . . . . . . . . . . . . .

197

II . — Problème de Kepler . . . . . . . . . . . . . . . . . . . . . . . . . . . . . . . . . . . . . .

199

III . — Lois de Kepler. vitesse d’évasion . . . . . . . . . . . . . . . . . . . . . . . . . . . . . . .

206

IV . — Satellites de la Terre . . . . . . . . . . . . . . . . . . . . . . . . . . . . . . . . . . . . . Exercices et problèmes . . . . . . . . . . . . . . . . . . . . . . . . . . . . . . . . . . . . . . . . . .

208 213

 c Dunod – Toute reproduction non autorisée est un délit

13. Système de corps ponctuels en interaction. Problème à deux corps I . — Éléments cinétiques d’un système de corpuscules . . . . . . . . . . . . . . . . . . . . . . .

217

II . — Problème à deux corps . . . . . . . . . . . . . . . . . . . . . . . . . . . . . . . . . . . .

221

III . — Dynamique des systèmes de corps ponctuels . . . . . . . . . . . . . . . . . . . . . . . . .

227

IV . — Énergétique des systèmes de N corps ponctuels . . . . . . . . . . . . . . . . . . . . . . .

231

V . — Interaction gravitationnelle à N corps . . . . . . . . . . . . . . . . . . . . . . . . . . . . Exercices et problèmes . . . . . . . . . . . . . . . . . . . . . . . . . . . . . . . . . . . . . . . . . .

237 239

14. Collision de deux particules I . — Définition et propriétés des collisions de particules . . . . . . . . . . . . . . . . . . . . . .

245

II . — Collision élastique directe de deux particules . . . . . . . . . . . . . . . . . . . . . . . . .

249

III . — Diffusion élastique par une cible immobile . . . . . . . . . . . . . . . . . . . . . . . . . .

250

IV . — Collisions inélastiques . . . . . . . . . . . . . . . . . . . . . . . . . . . . . . . . . . . . Exercices et problèmes . . . . . . . . . . . . . . . . . . . . . . . . . . . . . . . . . . . . . . . . . .

253 255

15. Diffusion de Rutherford. Notion de section efficace I . — Diffusion de Rutherford

. . . . . . . . . . . . . . . . . . . . . . . . . . . . . . . . . . .

257

II . — Sections efficaces de diffusion . . . . . . . . . . . . . . . . . . . . . . . . . . . . . . . .

262

III . — Libre parcours moyen . . . . . . . . . . . . . . . . . . . . . . . . . . . . . . . . . . . . .

266

IV . — Application au pouvoir d’arrêt des matériaux . . . . . . . . . . . . . . . . . . . . . . . . . Exercices et problèmes . . . . . . . . . . . . . . . . . . . . . . . . . . . . . . . . . . . . . . . . . .

268 269

viii

Table des matières

16. Cinématique du solide et des solides en contact I . — Cinématique du solide . . . . . . . . . . . . . . . . . . . . . . . . . . . . . . . . . . . .

272

II . — Cinématique des solides en contact . . . . . . . . . . . . . . . . . . . . . . . . . . . . . .

277

III . — Mouvements plans d’un solide . . . . . . . . . . . . . . . . . . . . . . . . . . . . . . . .

281

Exercices et problèmes . . . . . . . . . . . . . . . . . . . . . . . . . . . . . . . . . . . . . . . . . .

283

17. Éléments cinétiques des systèmes matériels I . — Centre de masse d’un système matériel . . . . . . . . . . . . . . . . . . . . . . . . . . . .

286

II . — Moments d’inertie . . . . . . . . . . . . . . . . . . . . . . . . . . . . . . . . . . . . . .

291

III . — Méthodes de calcul des moments d’inertie . . . . . . . . . . . . . . . . . . . . . . . . . .

296

IV . — Quantité de mouvement et moment cinétique . . . . . . . . . . . . . . . . . . . . . . . . .

298

Exercices et problèmes . . . . . . . . . . . . . . . . . . . . . . . . . . . . . . . . . . . . . . . . . .

305

18. Dynamique des systèmes matériels I . — Forces appliquées à un système . . . . . . . . . . . . . . . . . . . . . . . . . . . . . . . .

311

II . — Principe fondamental de la dynamique . . . . . . . . . . . . . . . . . . . . . . . . . . . .

313

III . — Théorèmes généraux de la dynamique . . . . . . . . . . . . . . . . . . . . . . . . . . . . .

314

IV . — Applications des théorèmes généraux . . . . . . . . . . . . . . . . . . . . . . . . . . . . .

318

V . — Lois de conservation . . . . . . . . . . . . . . . . . . . . . . . . . . . . . . . . . . . . .

323

Exercices et problèmes . . . . . . . . . . . . . . . . . . . . . . . . . . . . . . . . . . . . . . . . . .

326

19. Lois de Coulomb sur le frottement solide I . — Actions de contact . . . . . . . . . . . . . . . . . . . . . . . . . . . . . . . . . . . . . . II . — Lois sur le frottement solide

332

. . . . . . . . . . . . . . . . . . . . . . . . . . . . . . . . .

332

III . — Applications . . . . . . . . . . . . . . . . . . . . . . . . . . . . . . . . . . . . . . . . .

336

Exercices et problèmes . . . . . . . . . . . . . . . . . . . . . . . . . . . . . . . . . . . . . . . . . .

341

20. Énergétique des systèmes matériels I . — Travail des forces qui s’exercent sur un système . . . . . . . . . . . . . . . . . . . . . . . .

344

II . — Travail des forces qui s’exercent sur un solide . . . . . . . . . . . . . . . . . . . . . . . . .

347

III . — Travail total des actions de contact entre solides . . . . . . . . . . . . . . . . . . . . . . . .

349

IV . — Théorèmes de l’énergie . . . . . . . . . . . . . . . . . . . . . . . . . . . . . . . . . . . .

352

V . — Conservation de l’énergie mécanique . . . . . . . . . . . . . . . . . . . . . . . . . . . . .

355

Exercices et problèmes . . . . . . . . . . . . . . . . . . . . . . . . . . . . . . . . . . . . . . . . . .

359

21. Mécanique des chocs I . — Dynamique des chocs . . . . . . . . . . . . . . . . . . . . . . . . . . . . . . . . . . . . .

365

II . — Énergétique des chocs . . . . . . . . . . . . . . . . . . . . . . . . . . . . . . . . . . . .

367

III . — Applications . . . . . . . . . . . . . . . . . . . . . . . . . . . . . . . . . . . . . . . . .

369

Exercices et problèmes . . . . . . . . . . . . . . . . . . . . . . . . . . . . . . . . . . . . . . . . . .

372

Table des matières

ix

22. Mécanique des systèmes ouverts. Théorèmes d’Euler I . — Exemples de systèmes ouverts . . . . . . . . . . . . . . . . . . . . . . . . . . . . . . . .

375

II . — Caractère conservatif de la masse . . . . . . . . . . . . . . . . . . . . . . . . . . . . . . .

378

III . — Théorème de la quantité de mouvement . . . . . . . . . . . . . . . . . . . . . . . . . . . .

379

IV . — Théorème du moment cinétique . . . . . . . . . . . . . . . . . . . . . . . . . . . . . . . .

382

V . — Théorème de l’énergie cinétique . . . . . . . . . . . . . . . . . . . . . . . . . . . . . . . Exercices et problèmes . . . . . . . . . . . . . . . . . . . . . . . . . . . . . . . . . . . . . . . . . .

385 387

23. Statique I . — Statique des corps ponctuels et des solides . . . . . . . . . . . . . . . . . . . . . . . . . .

389

II . — Statique des fils . . . . . . . . . . . . . . . . . . . . . . . . . . . . . . . . . . . . . . . .

393

III . — Méthode des travaux virtuels . . . . . . . . . . . . . . . . . . . . . . . . . . . . . . . . . Exercices et problèmes . . . . . . . . . . . . . . . . . . . . . . . . . . . . . . . . . . . . . . . . . .

396 400

24. Lagrangien et hamiltonien I . — Équations de lagrange et lagrangien . . . . . . . . . . . . . . . . . . . . . . . . . . . . . .

404

II . — Hamiltonien et équations canoniques . . . . . . . . . . . . . . . . . . . . . . . . . . . . .

408

III . — Lagrangien et hamiltonien d’une particule chargée . . . . . . . . . . . . . . . . . . . . . .

412

IV . — Déterminisme, imprédictibilité et chaos . . . . . . . . . . . . . . . . . . . . . . . . . . . . Exercices et problèmes . . . . . . . . . . . . . . . . . . . . . . . . . . . . . . . . . . . . . . . . . .

413 417

25. Mouvement d’un solide autour d’un axe fixe I . — La liaison pivot . . . . . . . . . . . . . . . . . . . . . . . . . . . . . . . . . . . . . . . .

422

II . — Mouvement d’un solide autour d’un axe fixe . . . . . . . . . . . . . . . . . . . . . . . . .

422

III . — Machines tournantes . . . . . . . . . . . . . . . . . . . . . . . . . . . . . . . . . . . . .

427

IV . — Équilibrage des machines tournantes . . . . . . . . . . . . . . . . . . . . . . . . . . . . .

428

Exercices et problèmes . . . . . . . . . . . . . . . . . . . . . . . . . . . . . . . . . . . . . . . . . .

431

 c Dunod – Toute reproduction non autorisée est un délit

26. Gyroscope. Mouvement d’un solide autour d’un point. Effets microscopiques I . — La liaison sphérique . . . . . . . . . . . . . . . . . . . . . . . . . . . . . . . . . . . . .

435

II . — Gyroscope . . . . . . . . . . . . . . . . . . . . . . . . . . . . . . . . . . . . . . . . . .

436

III . — Approximation gyroscopique . . . . . . . . . . . . . . . . . . . . . . . . . . . . . . . . .

440

IV . — L’approximation gyroscopique en magnétisme . . . . . . . . . . . . . . . . . . . . . . . .

443

V . — Mouvement de Poinsot . . . . . . . . . . . . . . . . . . . . . . . . . . . . . . . . . . . .

447

VI . — Mouvement de Lagrange et Poisson . . . . . . . . . . . . . . . . . . . . . . . . . . . . . . Exercices et problèmes . . . . . . . . . . . . . . . . . . . . . . . . . . . . . . . . . . . . . . . . . .

451 456

27. Oscillateurs couplés. Cas de N oscillateurs identiques I . — Système de deux oscillateurs couplés . . . . . . . . . . . . . . . . . . . . . . . . . . . . .

460

II . — Coordonnées normales . . . . . . . . . . . . . . . . . . . . . . . . . . . . . . . . . . . .

469

III . — Modes de couplage de deux oscillateurs . . . . . . . . . . . . . . . . . . . . . . . . . . . .

473

IV . — Mouvement forcé d’un système de deux oscillateurs . . . . . . . . . . . . . . . . . . . . .

474

V . — Couplage de N oscillateurs identiques . . . . . . . . . . . . . . . . . . . . . . . . . . . . Exercices et problèmes . . . . . . . . . . . . . . . . . . . . . . . . . . . . . . . . . . . . . . . . . .

475 479

x

Table des matières

28. Introduction à la mécanique des fluides. Statique des fluides I . — Définitions et grandeurs caractéristiques . . . . . . . . . . . . . . . . . . . . . . . . . . .

483

II . — Pression . . . . . . . . . . . . . . . . . . . . . . . . . . . . . . . . . . . . . . . . . . .

485

III . — Équation d’état d’un fluide . . . . . . . . . . . . . . . . . . . . . . . . . . . . . . . . . .

488

IV . — Statique des fluides . . . . . . . . . . . . . . . . . . . . . . . . . . . . . . . . . . . . . .

489

V . — Théorème d’archimède et corps flottants . . . . . . . . . . . . . . . . . . . . . . . . . . . Exercices et problèmes . . . . . . . . . . . . . . . . . . . . . . . . . . . . . . . . . . . . . . . . . .

496 498

29. Cinématique des fluides I . — Champ des vitesses dans un fluide . . . . . . . . . . . . . . . . . . . . . . . . . . . . . .

504

II . — Accélération d’une particule de fluide . . . . . . . . . . . . . . . . . . . . . . . . . . . . .

508

III . — Bilan de masse. Débit-masse . . . . . . . . . . . . . . . . . . . . . . . . . . . . . . . . .

510

IV . — Différents types d’écoulements . . . . . . . . . . . . . . . . . . . . . . . . . . . . . . . .

512

V . — Exemples de champs de vitesse irrotationnels . . . . . . . . . . . . . . . . . . . . . . . . .

517

VI . — Écoulement d’un fluide autour d’un obstacle . . . . . . . . . . . . . . . . . . . . . . . . . Exercices et problèmes . . . . . . . . . . . . . . . . . . . . . . . . . . . . . . . . . . . . . . . . . .

519 523

30. Équation d’Euler et relation de Bernoulli I . — Équation d’Euler . . . . . . . . . . . . . . . . . . . . . . . . . . . . . . . . . . . . . . .

526

II . — Relation de Bernoulli . . . . . . . . . . . . . . . . . . . . . . . . . . . . . . . . . . . . .

530

III . — Applications de la relation de Bernoulli . . . . . . . . . . . . . . . . . . . . . . . . . . . .

533

IV . — Équations-bilans dans les systèmes ouverts fluides . . . . . . . . . . . . . . . . . . . . . . Exercices et problèmes . . . . . . . . . . . . . . . . . . . . . . . . . . . . . . . . . . . . . . . . . .

539 546

31. Fluides visqueux I . — Viscosité . . . . . . . . . . . . . . . . . . . . . . . . . . . . . . . . . . . . . . . . . . .

551

II . — Dynamique des fluides visqueux . . . . . . . . . . . . . . . . . . . . . . . . . . . . . . .

554

III . — Forces exercées par les fluides en mouvement . . . . . . . . . . . . . . . . . . . . . . . . . Exercices et problèmes . . . . . . . . . . . . . . . . . . . . . . . . . . . . . . . . . . . . . . . . . .

561 567

32. Ondes mécaniques dans un milieu continu I . — Propagation d’ondes mécaniques . . . . . . . . . . . . . . . . . . . . . . . . . . . . . . .

571

II . — Aspect énergétique . . . . . . . . . . . . . . . . . . . . . . . . . . . . . . . . . . . . . .

577

III . — Réflexion et transmission des ondes mécaniques . . . . . . . . . . . . . . . . . . . . . . .

580

IV . — Acoustique physiologique

. . . . . . . . . . . . . . . . . . . . . . . . . . . . . . . . . .

587

V . — Propriétés ondulatoires des ondes acoustiques . . . . . . . . . . . . . . . . . . . . . . . . .

588

Exercices et problèmes . . . . . . . . . . . . . . . . . . . . . . . . . . . . . . . . . . . . . . . . . .

595

Annexe 1. Les coniques . . . . . . . . . . . . . . . . . . . . . . . . . . . . . . . . . . . . . . . . .

599

I . — Définition

. . . . . . . . . . . . . . . . . . . . . . . . . . . . . . . . . . . . . . . . . .

599

II . — Équation polaire . . . . . . . . . . . . . . . . . . . . . . . . . . . . . . . . . . . . . . .

600

III . — Équation cartésienne . . . . . . . . . . . . . . . . . . . . . . . . . . . . . . . . . . . . .

600

IV . — Propriétés fondamentales des coniques . . . . . . . . . . . . . . . . . . . . . . . . . . . .

602

Annexe 2. Dérivées et différentielles

. . . . . . . . . . . . . . . . . . . . . . . . . . . . . . . . . .

604

Table des matières

xi

I . — Dérivées . . . . . . . . . . . . . . . . . . . . . . . . . . . . . . . . . . . . . . . . . . .

604

II . — Différentielles . . . . . . . . . . . . . . . . . . . . . . . . . . . . . . . . . . . . . . . .

607

III . — Systèmes de coordonnées . . . . . . . . . . . . . . . . . . . . . . . . . . . . . . . . . . .

609

IV . — Formes différentielles . . . . . . . . . . . . . . . . . . . . . . . . . . . . . . . . . . . . .

611

Annexe 3. Équations différentielles . . . . . . . . . . . . . . . . . . . . . . . . . . . . . . . . . . .

613

I . — Équations différentielles linéaires . . . . . . . . . . . . . . . . . . . . . . . . . . . . . . .

613

II . — Équation différentielle non linéaire . . . . . . . . . . . . . . . . . . . . . . . . . . . . . .

615

Annexe 4. Flux et circulation de vecteur . . . . . . . . . . . . . . . . . . . . . . . . . . . . . . . .

616

I . — Flux d’un champ de vecteur . . . . . . . . . . . . . . . . . . . . . . . . . . . . . . . . . .

616

II . — Circulation d’un champ de vecteur . . . . . . . . . . . . . . . . . . . . . . . . . . . . . .

621

III . — Opérateurs différentiels du second ordre . . . . . . . . . . . . . . . . . . . . . . . . . . .

623

IV . — Relations d’analyse vectorielle . . . . . . . . . . . . . . . . . . . . . . . . . . . . . . . .

624

Annexe 5. Simulations en mécanique . . . . . . . . . . . . . . . . . . . . . . . . . . . . . . . . . .

627

I . — Similitude et loi d’échelle . . . . . . . . . . . . . . . . . . . . . . . . . . . . . . . . . . .

627

II . — Interaction forte entre nucléons . . . . . . . . . . . . . . . . . . . . . . . . . . . . . . . .

633

III . — Satellites dans l’atmosphère . . . . . . . . . . . . . . . . . . . . . . . . . . . . . . . . . .

638

IV . — Portrait de phase d’oscillateurs . . . . . . . . . . . . . . . . . . . . . . . . . . . . . . . .

640

V . — Machine d’Atwood dansante (MAD) . . . . . . . . . . . . . . . . . . . . . . . . . . . . .

645

Réponses aux vingt questions . . . . . . . . . . . . . . . . . . . . . . . . . . . . . . . . . . . . . .

651

Solutions des exercices et problèmes . . . . . . . . . . . . . . . . . . . . . . . . . . . . . . . . . .

653

Bibliographie . . . . . . . . . . . . . . . . . . . . . . . . . . . . . . . . . . . . . . . . . . . . . .

791

Index . . . . . . . . . . . . . . . . . . . . . . . . . . . . . . . . . . . . . . . . . . . . . . . . . .

793

Les Anglais enseignent la mécanique comme une science expérimentale ; sur le continent, on l’expose toujours plus ou moins comme une science déductive et a priori. Ce sont les anglais qui ont raison, cela va sans dire ; [...] Ce n’est pas tout, il n’y a pas d’espace absolu [...]. Henri Poincaré La science et l’hypothèse, p. 113, 1902

Avant-propos Ce cours de mécanique newtonienne correspond globalement à l’enseignement donné dans les trois années des licences de physique (mécanique incluse) des universités. Il nous a paru pédagogiquement intéressant de le découper en leçons quasi autonomes. Un tel découpage a entraîné quelques redites, qu’on voudra bien considérer comme des points importants toujours utiles de rappeler. La première partie concerne d’abord la mécanique du corpuscule en interaction avec des systèmes que l’on schématise uniquement par les forces qu’ils exercent : force de pesanteur, force de Lorentz, force de contact, force élastique, etc. Afin de rompre avec un exposé dépassé de la mécanique, qui tend à réduire cette partie de la physique à un problème de projections de vecteurs et à une résolution d’équations différentielles, nous avons privilégié l’énergie dans la résolution des problèmes unidimensionnels et largement utilisé les possibilités qu’offrent, à des fins de discussions qualitatives, les concepts d’énergie potentielle et d’énergie mécanique. Dans la deuxième partie, on présente la mécanique des N corpuscules en interaction et on établit les théorèmes de la quantité de mouvement, du moment cinétique et de l’énergie. On souligne alors la complexité du problème et donc tout l’intérêt de l’approximation du problème à deux corps. On termine cette partie en développant successivement les collisions newtoniennes de deux particules, et la diffusion de particules. Cet ensemble constitue ce qui est en général traité en première année de licence ou des classes préparatoires. Dans cette partie, on ne manque de rappeler que les lois de la mécanique s’appliquent aussi aux systèmes vivants. Avec la troisième partie, plus technique, la séparation est suffisamment nette. On commence par la cinématique des corps indéformables (solides) et celle, très utile dans la pratique, des solides en contact. Dans tout le développement, on s’appuie modérément sur le concept de torseur d’intérêt limité. Ainsi, afin de compenser ce qui pourrait paraître comme un excès de formalisme, on insiste sur de nombreux thèmes physiques : les systèmes ouverts (fusées, etc.), la statique, la rotation d’un solide autour d’un axe avec l’équilibrage des machines tournantes, le mouvement d’un solide autour d’un point, notamment le gyroscope, le mouvement de Lagrange et Poisson dans l’approximation gyroscopique et son extension au magnétisme, précisément la Résonance Magnétique Nucléaire, enfin les oscillateurs couplés, en particulier les N oscillateurs identiques en interaction qui schématisent un solide déformable unidimensionnel.

Avant-propos

xiii

Enfin, on aborde alors la quatrième et dernière partie sur les fluides : la statique, la cinématique des écoulements, la dynamique des fluides non visqueux, la viscosité et la propagation des ondes acoustiques dans les milieux continus. En physiciens, nous sommes très sensibles aux lois de conservation et donc à l’interprétation des théorèmes généraux de la mécanique en termes de bilans de grandeurs. Aussi avons-nous volontairement subordonné le torseur dynamique au torseur cinétique et souligné l’importance des théorèmes d’Euler en mécanique des systèmes ouverts, notamment dans le cas des fluides. Dans ce contexte, nous avons tenu à donner un aperçu de la mécanique analytique, équations de Lagrange et équations canoniques d’Hamilton, ces dernières étant en marge des programmes mais au cœur de la physique moderne. Cet ouvrage s’adresse d’abord aux étudiants : il doit donc être clair et efficace. Aussi la typographie est-elle volontairement aérée, le renvoi à des formules éloignées pratiquement inexistant, les appendices et les mathématiques juste nécessaires. Des exercices et des problèmes, concrets et en nombre suffisant (environ 320), sont rassemblés à la suite des différents chapitres et leurs solutions réunies à la fin de l’ouvrage. Ces dernières permettront à l’étudiant, et plus largement à l’autodidacte, de tester sa propre compréhension du cours, d’enrichir sa réflexion sur le contenu et surtout de développer sa capacité de travail autonome. Nous pensons avoir rassemblé, dans un seul ouvrage, tous les éléments indispensables à une acquisition solide des connaissances fondamentales en mécanique. Cette nouvelle édition s’est enrichie de nombreux points d’actualisation, de précisions historiques, de plusieurs problèmes originaux et d’une annexe supplémentaire centrée sur l’illustration et la gestion informatique de plusieurs problèmes concrets de mécanique. Ce livre prend en compte toutes les discussions intéressantes que j’ai eues avec des collègues des universités et des classes préparatoires. Il s’inspire aussi des réactions exprimées par des étudiants de la licence et de l’agrégation de physique. Nous les remercions pour leurs remarques et commentaires constructifs.

 c Dunod – Toute reproduction non autorisée est un délit

Toulouse, juin 2014

Les grands noms de la mécanique Archimède Mathématicien et physicien grec, né à Syracuse en 287 av. J-C. et mort en 242. Il fut à la fois grand théoricien et habile expérimentateur. Sa contribution en physique la plus connue est le célèbre théorème d’hydrostatique qui porte son nom. Daniel Bernoulli Mathématicien et physicien suisse, né à Groningue en 1700 et mort à Bâle en 1782. C’est à SaintPétersbourg qu’il mène des recherches en mécanique des fluides ; il publie sa contribution majeure Hydrodynamica, en 1738, dans laquelle on peut reconnaître la célèbre équation de conservation de l’énergie en mécanique des fluides et les éléments de la théorie cinétique des gaz. Jacques Binet Mathématicien et astronome français, né à Rennes en 1786 et mort à Paris en mai 1856. À sa sortie de l’École polytechnique il devient répétiteur de géométrie descriptive dans cet établissement, puis professeur de mécanique en remplacement de Simon Denis Poisson. Il est surtout connu pour ses travaux dans le domaine de l’astronomie ; ses formules de cinématique donnent l’expression en coordonnées polaires de la vitesse et de l’accélération des corps soumis à une accélération centrale, telles les planètes du système solaire. Tycho Brahe Astronome danois, né à Knudsrup en 1546 et mort à Prague en 1601. En 1576, Frédéric II du Danemark le charge de construire un observatoire dans l’île de Hveen. Grand observateur, il accumule de nombreuses données astronomiques, pendant une trentaine d’années. Il poursuit ses travaux à Prague, assisté, peu avant sa mort, par un jeune astronome J. Kepler. Il reste cependant attaché aux idées géocentriques de Ptolémée. Nicolas Copernic Astronome polonais, né à Thorn en 1473 et mort à Frauenburg en 1543. Copernic, qui était chanoine, attendit la fin de sa vie pour publier son célèbre traité De revolutionibus. Son œuvre fut moins inspirée par les observations astronomiques que par sa ferme conviction que le système de Ptolémée manquait de simplicité, d’élégance, bref de symétrie. Ce dernier point l’incita à tort à supposer que les trajectoires des planètes étaient nécessairement circulaires, ce que Kepler corrigea un siècle plus tard grâce aux observations de Brahe. Gaspard Coriolis Ingénieur français, né à Paris en 1792 et mort à Paris en 1843. Devenu professeur de mécanique, il met en évidence l’existence, dans tout référentiel en mouvement accéléré par rapport au référentiel terrestre, d’une force d’inertie proportionnelle à la vitesse. Charles de Coulomb Ingénieur et physicien français, né à Angoulême en 1736 et mort à Paris en 1806. Initialement ingénieur de l’armée, il abandonne cette activité à 36 ans pour se consacrer à la recherche scientifique. Il publie en 1779 un traité sur la théorie des machines simples dans lequel il donne les lois du frottement solide. Mais son œuvre principale concerne l’électromagnétisme, notamment la force d’interaction entre charges électriques, proportionnelle à 1/r2 , et le magnétisme terrestre.

Les grands noms de la mécanique

xv

Leonhard Euler Mathématicien suisse, né à Bâle en 1707 et mort à Saint-Pétersbourg en 1783. Il applique à la mécanique les résultats importants qu’il établit sur l’intégration des équations différentielles. Il publie en 1736 un Traité complet de mécanique. Léon Foucault Physicien français, né à Paris en 1819 et mort à Paris en 1868. Autodidacte et inventeur de talent, il apporte des contributions importantes, notamment en optique pour corriger les miroirs de leurs aberrations géométriques et pour mesurer avec précision la vitesse de la lumière. Il est surtout connu pour ses études en mécanique ; il met en évidence, de façon spectaculaire, l’influence de la rotation de la Terre sur le comportement d’un pendule simple (pendule de Foucault) et invente le gyroscope. Galileo Galilei dit Galilée Astronome et physicien italien, né à Pise en 1564 et mort à Arcetri (près de Florence) en 1642. Il est considéré comme le précurseur de la science moderne, d’une part en utilisant les mathématiques pour décrire les lois de la physique, et d’autre part en testant la validité de ces lois par l’expérimentation. Ses principales contributions en astronomie sont l’observation de la surface de la Lune et la découverte des satellites de Jupiter à l’aide d’instruments d’optique qu’il a lui-même mis au point. L’ensemble est publié en 1610 dans Le messager des étoiles. Cependant, c’est sa contribution en mécanique qui est la plus célèbre. En étudiant le mouvement des corps, il découvre les lois de la chute libre, notamment l’indépendance de la masse lorsque la résistance de l’air est négligée, l’isochronisme des petites oscillations d’un pendule ; il perçoit aussi le principe de l’inertie et se rallie aux idées héliocentriques de Copernic. Il publie alors le Dialogue sur les deux principaux systèmes du monde en 1632, ce qui lui vaut une condamnation par l’Inquisition, en raison de sa vision copernicienne du monde (mobilité de la Terre). C’est à la fin de ce procès qu’il aurait prononcé la célèbre phrase « eppur si muove » (Et pourtant, elle tourne).

 c Dunod – Toute reproduction non autorisée est un délit

William Rowan Hamilton Mathématicien et physicien irlandais, né en 1806 à Dublin et mort en 1865 près de Dublin. Enfant prodige puis étudiant génial, il impressionne à 22 ans l’Académie Royale d’Irlande en présentant un exposé moderne sur la théorie des rayons lumineux. Ce travail est le point de départ d’une contribution capitale en dynamique qui s’achèvera en 1833 sur une remarquable analogie entre l’optique et la mécanique. Cette synthèse débouchera sur la relation de L. de Broglie et sur l’équation de Schrödinger en mécanique ondulatoire (cf. Quantique). Christian Huygens Mathématicien, astronome et physicien hollandais, né à La Haye en 1629 et mort en 1695. Il fut à l’origine de la théorie ondulatoire de la lumière et, à ce titre, s’opposa à la théorie corpusculaire de Newton. Il interpréta la propagation rectiligne de la lumière dans les milieux homognes et isotropes en introduisant le concept de surface d’onde. Expérimentateur confirmé, il découvrit, avec les instruments d’optique qu’il mit lui-même au point (oculaire d’une lunette astronomique), les anneaux de Saturne ainsi que son satellite Titan. En mécanique, on lui attribue l’invention du pendule cycloïdal. Johannes Kepler Astronome allemand, né à Weil en 1571 et mort à Ratisbonne en 1630. D’origine modeste, Kepler se révèle vite très bon théoricien et adepte des idées héliocentriques de Copernic. Chassé de Graz où il enseignait les mathématiques, il se réfugie à Prague dans l’observatoire de Brahe. Les observations de ce dernier lui permettent de découvrir les célèbres lois qui portent son nom.

xvi

Les grands noms de la mécanique

Johann Kœnig Mécanicien allemand, né à Büdinger en 1712 et mort à Amerongen aux Pays-Bas en 1757. Sa contribution importante en mécanique est connue sous le nom de théorèmes de Kœnig, à ne pas confondre avec le mécanicien français Gabriel Kœnigs. Joseph-Louis de Lagrange Mathématicien italo-français, né à Turin en 1736 et mort à Paris en 1813. Il est considéré comme l’un des plus grands mathématiciens de son époque. Sa contribution majeure en mécanique, a été publiée en 1788 sous le titre Mécanique analytique ; on y trouve les célèbres équations qui permettent d’étudier le mouvement des corps sans utiliser la géométrie et le calcul vectoriel. À la mort de Frédéric II de Prusse, qui l’avait chargé de diriger la section mathématique de l’Académie des Sciences de Berlin, il est accueilli à Paris pour poursuivre ses travaux. Pierre-Simon de Laplace Astronome, mathématicien et physicien français, né à Beaumont-en-Auge en 1749 et mort à Paris en 1827. Bien que professeur de mathématiques et homme politique, ses travaux en physique sont nombreux. Il signe diverses contributions sur la capillarité, la propagation du son dans l’air, l’évolution adiabatique des gaz et le travail des forces électromagnétiques. Cependant, c’est sa publication sur la mécanique céleste, Exposition du système du monde, qui est la plus remarquée. On y trouve développées les conditions d’un déterminisme rigoureux à la base d’une physique totalement prédictive. Pierre Louis de Maupertuis Philosphe et scientifique français, né à Saint-Malo en 1698 et mort à Bâle en 1759. Il est nommé membre de l’Académie des sciences dès 1723. Il publie alors divers travaux de mécanique, d’astronomie et de biologie. En 1728, il se rend en Angleterre et est élu membre de la Royal Society ; il découvre là les idées de Newton, en particulier la gravitation universelle. Il devient un ardent défenseur en France de ces idées, aboutissant à l’aplatissement de la Terre aux pôles, et donc un opposant à J. Cassini, qui affirmait au contraire que la Terre était allongée aux pôles. Les mesures directes auxquelles il participa activement, en se rendant en Laponie, confirme la thèse de Newton. Il publie en 1746 un article sur le principe de moindre action, dont l’argumentation finaliste est critiquée, notamment par Diderot. Isaac Newton Mathématicien, astronome et physicien anglais, né en 1642 à Woolsthorpe et mort en 1727 à Kensington. Il est considéré avec Einstein comme le plus grand physicien de tous les temps. Curieusement et comme ce dernier, il fut un élève moyen qui ne révéla au collège aucune capacité exceptionnelle. C’est à Cambridge que l’on remarqua ses grandes possibilités. Plus influencé par Descartes que par Aristote, il publie en 1687 une synthèse magistrale de la mécanique Principia mathematica ; il y énonce les fondements de la dynamique et relie, en introduisant la loi de la gravitation universelle, le mouvement des planètes et la chute des corps. En optique, il s’opposa à C. Huygens et R. Hooke, adeptes d’une théorie ondulatoire de la lumière. Cependant, sa contribution faite dans “Optics” est elle aussi exceptionnelle : il interprète la décomposition spectrale de la lumière, à partir d’expériences conues avec des prismes, et montre que la couleur blanche est un mélange des différentes couleurs spectrales ; il explique aussi la formation des images par des miroirs et suggère même la possibilité d’échange entre lumière et matière. Blaise Pascal Mathématicien, physicien et philosophe français, né à Clermont-Ferrand en 1623 et mort à Paris en 1662. Enfant et adolescent génial, Pascal est vite reconnu par toute la communauté scientifique comme un mathématicien de tout premier plan. Sa contribution la plus importante en physique concerne la statique des fluides.

Les grands noms de la mécanique

xvii

Henri Pitot Ingénieur français, né à Aramon (Gard) en 1695 et mort en 1771. Après des études en mathématiques et en astronomie, il devient assistant du physicien Réaumur en 1723. Spécialisé en hydraulique, il devient surintendant du Canal du Midi et construit un aqueduc pour l’alimentation en eau de Montpellier. Il participe notamment à la restauration du pont du Gard par la réalisation d’un second ouvrage d’art, accolé à ce dernier, et réalise de nombreuses digues de protection, notamment sur le cours du fleuve Vidourle. Il est surtout connu pour être l’inventeur du tube qui permet de mesurer la vitesse d’écoulement d’un fluide. Jean-Louis Poiseuille Médecin et physicien français, né à Paris en 1799 et mort à Paris en 1869. Son étude de la circulation sanguine le conduit à analyser avec soin l’écoulement laminaire des fluides visqueux dans les tuyaux cylindriques. Henri Resal Ingénieur et mathématicien français, né à Plombières en 1828 et mort à Anemasse en 1896. Professeur de mécanique à l’École polytechnique, il résout de façon élégante, en projetant dans un système d’axes adapté, le mouvement des solides de révolution en rotation autour d’un point. Osborne Reynolds Ingénieur anglais, né à Belfast en 1842 et mort à Watchet en 1912. Il apporta en 1883 une contribution importante dans la distinction quantitative entre les régimes d’écoulement laminaire et turbulent.

 c Dunod – Toute reproduction non autorisée est un délit

Ernest Rutherford Physicien britannique, né à Brightwater, près de Nelson, en Nouvelle Zélande en 1871 et mort à Cambridge en 1937. Même si très jeune, en classe, il excellait sur tous les sujets, Rutherford fut intéressé surtout par les sciences et les mathématiques. Boursier, il poursuit ses études en Angleterre, à Cambridge, où il devint l’élève de J.J. Thomson. Après avoir été professeur à Montréal, à Manchester, puis à Cambridge en 1909, il prit la direction du Laboratoire Cavendish en 1919. Il est surtout connu pour ses travaux sur la radioactivité, notamment l’étude des rayons a , qu’il identifie à des noyaux d’hélium, et de leur diffusion, ce qui lui permet d’établir le modèle actuel de la structure atomique. Il reçut, en 1908, le prix Nobel de chimie. Giovanni Battista Venturi Physicien italien, né à Reggio d’Émilie en 1746 et mort en 1822 dans sa ville natale. En 1773, il devient professeur de géométrie et philosophie à l’Université de Modène et en 1776 professeur de physique à l’École du génie militaire de Modène. Établi à Paris en 1796, c’est suite à ses nombreux travaux en dynamique des fluides qu’il décrit dans un ouvrage ce qui deviendra l’effet Venturi, précisément la relation entre la vitesse d’un fluide et sa pression. Evangelista Torricelli Physicien italien, né à Faenza en 1608 et mort à Florence en 1647. Il est surtout connu pour ses travaux sur la pression atmosphérique, effectués à la demande des fontainiers de Florence ; ces derniers souhaitaient élever l’eau à plus de 10 m à l’aide d’une pompe aspirante.

Constantes physiques, notations et symboles Les symboles utilisés sont généralement ceux recommandés par l’AFNOR.

I . — CONSTANTES PHYSIQUES G = 6,673 84(80) × 10−11 m3 .kg−1 .s −2 c = 299 792 458 m.s −1 h = 6, 626 069 57(29) × 10−34 J.s me = 0, 910 938 291(40) × 10 −30 kg mp = 1, 672 621 777(74) × 10 −27 kg NA = 6, 022 141 29(27) × 10 23 mol−1 R = NAkB = 8, 314 462(75) J.mol−1.K −1 kB = R/NA = 1, 380 648 8(13) × 10 −23 J.K −1 e = 1, 602 176 565(35) × 10−19 C F = NAe = 96 485, 339 9(24) C.mol−1 m0 = 4p × 10−7 N.A−2 ε0 = (m 0 c2 )−1 = 8, 854 187 817... × 10−12 F.m −1 e2 q2e = 4pε 0

Constante newtonienne de gravitation G = (6,673 84 ± 0,000 80) × 10−11 m3 .kg−1 .s−2 Constante d’Einstein ou vitesse de la lumière dans le vide (valeur exacte) Constante de Planck Masse de l’électron Masse du proton Nombre d’Avogadro Constante molaire des gaz parfaits Constante de Boltzmann Charge élémentaire (charge de l’électron : −e) Constante de Faraday Perméabilité du vide (valeur exacte) Permittivité du vide (valeur exacte) q2e = 230, 707 705 6 × 10 −30 S.I

II . — CONSTANTES DU SYSTÈME SOLAIRE II . 1 . — Caractéristiques du Soleil Masse Rayon Diamètre apparent

M S = 1, 989 1 × 10 30 kg ≈ 2 × 1030 kg RS = 696 000 km ≈ 0, 7 × 10 9 m u S = 32 ≈ 0, 5 ◦

xix

Constantes physiques, notations et symboles II . 2 . — Caractéristiques de la Terre Masse Rayons polaire équatorial moyen Demi-grand axe de l’orbite elliptique Excentricité Vitesse orbitale moyenne

M T = 5, 972 × 10 24 kg ≈ 6 × 1024 kg

R T ,p = 6 357 km = 6, 357 × 10 6 m R T ,e = 6 378 km = 6, 378 × 10 6 m R T ≈ 6 371 km ≈ 6, 4 × 106 m a ≈ 149, 6 × 10 9 m e ≈ 0, 016 71 v T ≈ 29, 8 km.s−1 .

Le plan de la trajectoire de la Terre autour du Soleil est le plan de l’écliptique. Il fait avec le plan équatorial l’angle pratiquement constant u = 23◦ 26  (Fig. 1a). Le parsec, mot condensant « parallaxe d’une seconde », est la distance à laquelle le rayon de l’orbite terrestre autour du Soleil est vu sous un angle de 1 seconde d’arc (5 mrad) (Fig. 1b) : 1 pc ≈ 3, 085 677 × 1016 m soit

1 pc ≈ 30, 85 Pm

On utilise souvent en astrophysique l’année-lumière (al) qui est la distance parcourue par la lumière, dans le vide, pendant une année julienne de 365,25 jours : 1 al = 299 792 458 × 365, 25 × 24 × 3 600 ≈ 9, 460 730 ≈ 9, 46 Pm On a donc 1 pc ≈ 3, 26 al . 23◦ 26 Axe polaire E Soleil

P H

Terre Plan de l'écliptique

O 100 1 pc

Plan e´ quatorial T

A

 c Dunod – Toute reproduction non autorisée est un délit

a) F IG . 1. — a) Plan de l’écliptique et plan équatorial

S

Plan de l'écliptique

b) b) Parsec

II . 3 . — Caractéristiques des planètes Les planètes Mercure, Vénus, Mars, Jupiter et Saturne sont connues depuis l’Antiquité ; elles sont à l’origine des noms des jours de la semaine, respectivement : Mercredi, Vendredi, Mardi, Jeudi, Samedi. Lundi vient de Lune comme « Monday » vient de « Moon » en anglais ; quant au dimanche, c’est le jour du Soleil en anglais « Sunday ». La planète Uranus fut découverte en 1781 par l’astronome allemand William Herschel. En 1846, le Français Urbain Le Verrier et séparément l’Anglais John Adams « inventèrent » Neptune à partir des écarts observés entre la trajectoire théorique d’Uranus et sa trajectoire réelle, ce qui permit sa découverte par l’astronome berlinois Johann Galle quelques mois après. Les caractéristiques des huit planètes du système solaire sont rassemblées dans le tableau 1. Les unités de ces grandeurs sont relatives à celles de la Terre.

xx

Constantes physiques, notations et symboles Planètes

Masse

Rayon

Distance au Soleil

Période sidérale de rotation propre

Période sidérale de révolution autour du Soleil

Excentricité

Mercure

0,555

0,382

0,387

58,6 j

0,24

0,206

Vénus

0,815

0,949

0,723

243 j

0,615

0,007

Terre

1

1

1

1j

1

0,017

Mars

0,107

0,533

1,52

24 h 37 min

1,88

0,093

Jupiter

318

11,2

5,20

9 h 55 min

11,86

0,048

Saturne

95,1

9,45

9,55

10 h 39 min

29,46

0,056

Uranus

14,5

4,01

19,2

17 h 14 min

84,07

0,046

Neptune

17,1

3,88

30,1

16 h 7 min

164,81

0,009

TAB . 1. — Caractéristiques principales des huit planètes du système solaire

II . 4 . — Caractéristiques de la Lune Masse Rayon Distance Terre-Lune Diamètre apparent Vitesse orbitale moyenne

ML ≈ 7, 35 × 10 22 kg ≈ MT /81 RL ≈ 1737 km = 1, 7 × 10 6 m DL ≈ 383 400 km = 383, 4 × 106 m uL ≈ 33 ≈ 0, 5◦ vL ≈ 1 km.s−1

Période sidérale de révolution autour de la Terre : 27 j 7 h 3 min Lunaison (durée entre deux nouvelles Lunes) : 29 j 12 h 44 min Angle d’inclinaison du plan de l’orbite lunaire par rapport au plan de l’écliptique : 5, 14 ◦

III . — NOTATIONS R r = OA (d U/ d t)R v, v a, a S Sd p, P LO P W Ek Ep

référentiel (repère d’espace et de temps) vecteur position d’un point courant A dérivée du vecteur U par rapport au temps, relativement à la base de R vecteur vitesse de A par rapport à R et sa norme vecteur accélération de A par rapport à R et sa norme solide (indéformable) système matériel (déformable) quantité de mouvement d’un point ou d’un système moment cinétique au point O d’un point ou d’un système puissance travail énergie cinétique énergie potentielle

Constantes physiques, notations et symboles

 c Dunod – Toute reproduction non autorisée est un délit

Em VS/R C, G R0 Rg R∗ g, g G, F E, V B [P]O ou [P, L O] [D]O ou [D, NO] [I]O ex , ey, e z x, y, z r, w, z r, u, w c, u, f ˙x x(t) = x m cos(vt + fx ) x x(t) = x m exp[j(vt + fx )] T, f , v xm = xm exp(jfx ) x |x| x∗ Re{x}, Im{x} P(t) ε F(r) C(r) L H p r Jm qm qv kT kS

xxi

énergie mécanique vecteur vitesse de rotation du solide S par rapport à R centre de masse ou centre d’inertie (≈ centre de gravité) référentiel de Copernic référentiel géocentrique référentiel du centre de masse ou référentiel barycentrique champ de pesanteur et sa norme champ et potentiel de gravitation champ et potentiel électriques champ magnétique torseur cinétique au point O torseur dynamique au point O opérateur ou tenseur d’inertie en O base orthonormée directe de R coordonnées cartésiennes de r coordonnées cylindriques coordonnées sphériques angles d’Euler (précession, nutation, rotation propre) fonction dérivée de x par rapport au temps élongation sinusoïdale valeur complexe de x expression complexe de l’élongation sinusoïdale période, fréquence, pulsation amplitude complexe valeur complexe de x module de x complexe conjugué de x parties réelle et imaginaire du signal x puissance instantanée facteur de restitution en énergie cinétique fonction potentiel des vitesses dans le plan Oxy (v = − grad F) fonction courant dans le plan Oxy (v = rot(−Ce z )) lagrangien hamiltonien pression masse volumique vecteur courant de matière débit-masse débit-volume coefficient de compressibilité isotherme coefficient de compressibilité isentropique

xxii

Constantes physiques, notations et symboles Zc h n = h/r

impédance acoustique caractéristique coefficient de viscosité viscosité cinématique

ln lg lb exp ≈ ∼

logarithme népérien logarithme décimal logarithme binaire exponentielle sensiblement égal à de l’ordre de

IV . — ALPHABET GREC alpha

A

a

eta

H

h

nu

N

n

tau

T

t

beta

B

b

theta

Q

u

xi

J

j

upsilon

Y

y

gamma

G

g

iota

I

i

omicron

O

o

phi

F

f

delta

D

d

kappa

K

k

pi

P

p

chi

X

x

epsilon

E

e

lambda

L

l

rho

P

r

psi

C

c

zeta

Z

z

mu

M

m

sigma

S

s

omega

V

v

V . — MULTIPLES ET SOUS-MULTIPLES EN NOTATION SCIENTIFIQUE Nom yotta zetta exa péta téra giga méga kilo milli micro nano pico femto atto zepto yocto

Facteur 24

10 1021 1018 1015 10 12 109 106 103 10∗ 3 10∗ 6 10∗ 9 10∗ 12 10∗ 15 10∗ 18 10∗ 21 10∗ 24

Origine

Signification

Année d’adoption

Symbole

grec (októ) latin (septem) grec (hex) grec (pente) grec (teras) grec (gigas) grec (megas) grec (chiloi) latin (mille) grec (mikros) latin (nanus) italien (piccolo) danois (femtem) danois (atten) latin (septem) grec (októ)

huit sept six cinq monstre géant grand mille mille petit nain petit quinze dix-huit sept huit

1991 1991 1991 1975 1960 1960 1960 1795 1960 1960 1960 1960 1964 1964 1991 1991

Y Z E P T G M k m m n p f a z y

Description de l’ouvrage L’ouvrage comporte plusieurs parties qui correspondent aux différentes étapes de l’enseignement de la mécanique dans les universités ou dans les classes préparatoires aux Grandes Écoles scientifiques. En dehors de la leçon 1 qui rassemble les rappels et les compléments sur le calcul vectoriel, le déroulement du cours est le suivant : i) Partie I, Licence 1, semestre I Leçons 2 à 12 : cinématique, dynamique et énergétique du point matériel, libre ou gêné, par rapport à un référentiel galiléen ou non. ii) Partie II, Licence 1, semestre II Leçons 13 à 15 : Dynamique et énergétique des systèmes de N points matériels (problème à deux corps, collisions et diffusion de particules). iii) Partie III, Licence 2, semestre I Leçons 16 à 21 : Dynamique et énergétique du solide et des systèmes de solides.

 c Dunod – Toute reproduction non autorisée est un délit

iv) Partie IV, Licence 2, semestre II, et licence 3 Leçons 22 à 32 : Applications de la mécanique des solides et introduction à la mécanique des fluides. Les leçons 1, 2, 3, 4, 13, 16, 17, 18, 24, 28 ont un rôle central car elles contiennent les éléments indispensables (définitions, lois et principes) à l’étude des leçons qui suivent. Il faut donc les étudier avant d’aborder les suivantes. Même si ces dernières sont présentées dans un certain ordre, il est possible de les lire dans un ordre différent qui tienne compte des préoccupations et des intérêts du lecteur ; en effet, les leçons sont quasi autonomes et le renvoi à des formules éloignées pratiquement inexistant. Par exemple, si l’on souhaite étudier la mécanique des solides, il est conseillé de lire les leçons 16, 17 et 18 avant ; de même, si l’on ne s’intéresse qu’à la mécanique des fluides, mieux vaut lire d’abord la leçon 28.

Méthode de travail Lecture des leçons Dans une première phase, une leçon doit être lue une première fois, en insistant sur l’introduction, laquelle situe la leçon dans le cours, et sur la conclusion qui répertorie l’ensemble des résultats essentiels. Dans une deuxième phase, l’étudiant doit refaire avec soin tous les calculs intermédiaires. Enfin, une dernière lecture devrait permettre d’appréhender complètement la leçon, notamment les résultats essentiels, les exemples significatifs et les ordres de grandeur.

xxiv

Description de l’ouvrage

Exercices et problèmes Une fois la lecture de la leçon achevée, l’étudiant doit passer à la phase d’application en faisant des exercices simples et courts, directement liés au contenu de la leçon ; il doit essayer de résoudre ces exercices avec le seul support que constitue le cours. En cas de difficultés, un coup d’œil rapide sur la solution, proposée en fin d’ouvrage, devrait l’aider. Éviter cependant la simple lecture de la solution proposée et la mémorisation de la démonstration : mieux vaut revenir sur la leçon pour résoudre l’exercice. En cas de difficulté majeure, lire la solution et tenter de la refaire sans aucune aide un ou deux jours plus tard. Une fois ces exercices rédigés, tenter de résoudre des problèmes d’examens et concours généralement plus longs. Révision Pour réviser, une ultime lecture devrait conforter l’apprentissage. Ne pas hésiter à souligner au crayon les parties essentielles et à porter en marge des remarques personnelles suggérées par d’autres ouvrages ou documents annexes, tels que des revues grand public (La Recherche, Science et Vie, Ciel et Espace, etc.).

Comment résoudre un problème de mécanique On résout correctement un problème de mécanique si l’on s’astreint à répondre successivement à plusieurs questions, même lorsque le texte n’invite pas à y répondre explicitement. Quel est le système étudié ? Il faut définir le système dont on veut étudier le mouvement, c’est-à-dire le délimiter en précisant sa frontière commune avec l’extérieur. Quel est le référentiel ? Le temps étant un paramètre universel en mécanique newtonienne, définir le référentiel revient à préciser le repère d’espace par rapport auquel on se propose d’étudier le mouvement d’un système. Il est commode de lui associer une base orthonormée ( ex , e y , e z ). On désigne généralement ce référentiel par son origine O et une base orthonormée, ce que l’on écrit sommairement R = Oxyz . i) Ce référentiel est-il galiléen ou non ? ii) Est-il terrestre ou non ? iii) Si le référentiel est terrestre, on peut le considérer comme galiléen à condition de substituer à la force de gravitation le poids et d’exclure les mouvements très précis (déviation vers l’est. . .) ou très rapides (pour v > 700 m . s−1 , le terme de Coriolis est supérieur à 1 % du poids). iv) Si le référentiel a un mouvement accéléré par rapport au référentiel terrestre, il faut ajouter les forces d’inertie d’entraînement et de Coriolis. Dans le cas où ce mouvement se limite à une translation, la force de Coriolis est nulle. Quel est le bilan des forces ? Par forces, on entend plus largement forces et moments. On doit distinguer les forces extérieures et les forces intérieures au système considéré, les forces données et les forces inconnues, les forces qui

xxv

Description de l’ouvrage

dérivent d’une énergie potentielle et les autres. Dans le tableau 2, on énumère les principales forces qui apparaissent dans les problèmes concrets et on donne l’expression des énergies potentielles associées. Quel est le nombre de degrés de liberté ? Les degrés de liberté sont les paramètres {q i } dont dépend la position d’un système : a priori, un point en a trois, un solide six, un ensemble de n points et de s solides en a 3n + 6s . Les contraintes limitent généralement ce nombre ; un solide, dont un point est fixé et qui tourne autour d’un axe déterminé, n’a plus qu’un seul degré de liberté. La connaissance de l’état du système exige en plus un nombre identique de données sur les dérivées q˙ i Remarque : Une condition de roulement sans glissement intégrable diminue le nombre de degrés de liberté. Force

Énergie potentielle

Force de gravitation (−Gm 1m2/r 2) er

Ep = −Gm1 m 2/r + Cte Ep = −mg · OA + Cte = mgz + Cte

Force de pesanteur mg

(l’axe Oz est vertical ascendant) Force électrostatique qE

E p = qV + Cte

Force magnétique qv × B

ne travaille pas

Force de rappel d’un ressort −K (x − l 0) e x

Ep = K (x − l0 )2/2 + Cte

Couple de torsion d’un fil −C(u − u0 ) ez

Ep = C(u − u 0 )2/2 + Cte

Force de frottement visqueux −av ou −bv 2v/v

pas d’énergie potentielle

Force de frottement solide (R, G O)

pas d’énergie potentielle

 c Dunod – Toute reproduction non autorisée est un délit

Force d’inertie d’entraînement de translation uniforme : −m ae = −m aO  Force d’inertie d’entraînement de rotation uniforme : −m ae =

mV 2

HA

Force d’inertie de Coriolis −m a C = 2mV × v

Ep = m aO · OA + Cte E p = −mV2HA 2/2 + Cte ne travaille pas

TAB . 2.

Quelles sont les équations du mouvement ? i) Si le système n’a qu’un seul degré de liberté et si les seules forces de puissances non nulles dérivent d’une énergie potentielle, appliquer la conservation de l’énergie (intégrale première de l’éner-

xxvi

Description de l’ouvrage

gie) : E m = Ek + Ep = Cte

Cette équation suffit pour connaître le mouvement. ii) Si le système a plusieurs degrés de liberté, rechercher en priorité les intégrales premières : conservation de l’énergie, conservation du moment cinétique en projection suivant un axe si le système tourne sans frottement autour de cet axe, etc. iii) Exploiter d’abord le caractère vectoriel des théorèmes généraux : dP = MaC = Sex et dt

d LO = MO, ex dt

iv) Si le système est soumis à une réaction inconnue passant par un point, appliquer le théorème du moment cinétique en ce point de telle sorte que la contribution de cette force inconnue disparaisse. Quelles sont les expressions des éléments cinétiques P , LO et E k ?

Bien que technique, cette question joue un rôle important. Il faut : i) décomposer le système en éléments simples et utiliser les propriétés d’additivité, ii) appliquer les théorèmes de Kœnig, lorsque le système n’a pas de point fixe dans le référentiel d’étude, iii) utiliser les bases principales d’inertie, ce qui exige d’adopter une base de projection différente de la base du référentiel, et donc d’utiliser la formule de Bour : dL dL = + VR /R × L dt R d t R iv) Choisir les bases de projection les plus commodes : si le mouvement est rectiligne, projeter les vecteurs suivant l’axe du mouvement ; si la trajectoire d’un point est connue, utiliser les coordonnées intrinsèques ; s’il s’agit du mouvement d’un solide de révolution autour d’un point, utiliser la base de Resal avec les angles d’Euler, etc. Comment résoudre les équations différentielles ? C’est encore un problème technique. Les équations différentielles le plus souvent rencontrées en mécanique sont les suivantes : ¨ x+

x˙ + v20 x = a t

(caractéristique des oscillations)

v = a (caractéristique de la mise en rotation d’une machine) t La résolution de ces équations linéaires ne présente pas de difficulté. La conservation de l’énergie permet généralement d’analyser qualitativement le mouvement et donc de discuter des divers mouvements possibles d’un système. Cette analyse est très intéressante, notamment lorsque l’intégration de l’équation différentielle est délicate. v˙ +

Quelle est l’interprétation des résultats obtenus ? Cette dernière phase est capitale, car elle permet de vérifier les calculs et donc de revenir sur des erreurs de signes décisives ; par exemple, dans l’équation oscillante ¨ x + v 20 x = 0 , l’erreur de signe exclut toute oscillation, ce qui doit pouvoir être décelé par une analyse préliminaire du mouvement, à la fois qualitative et intuitive.

La mécanique en vingt questions

1. Pourquoi tire-t-on généralement les fusées vers l’est à partir d’une base de lancement proche de l’équateur ? 2. Un corps abandonné au sommet du mât vertical d’un bateau, qui est animé d’un mouvement de translation rectiligne uniforme par rapport à un référentiel terrestre, tombe au pied du mât. Pourquoi ? 3. Alors qu’une énergie cinétique est toujours non négative, pourquoi l’énergie mécanique d’un système peut-elle être négative ? 4. Pourquoi la force de gravitation, qui est de loin la plus faible des forces fondamentales, est-elle, à l’échelon de l’Univers, la plus influente ? 5. Pourquoi la force de Coriolis terrestre intervient-elle dans l’expression de la loi fondamentale dans un référentiel terrestre et non la force d’inertie d’entraînement terrestre ? 6. On dit que la verticale ne passerait pas par le centre de la Terre, même si cette dernière était sphérique. Pourquoi ? 7. Pourquoi la chute identique de deux corps différents dans le vide est-elle un fait exceptionnel ? 8. Pourquoi y-a-t-il impesanteur (ou microgravité) dans un satellite artificiel ? 9. Les marées sont attribuées à la non-uniformité du champ de gravitation G a dû aux astres autres que la Terre. Pourquoi la contribution de ces astres est-elle proportionnelle à M /D3 , M étant leur masse et D leur distance à la Terre ? 10. Pourquoi la Terre a-t-elle un mouvement plan autour du Soleil (plan de l’écliptique) ? 11. Pourquoi faut-il freiner les satellites provenant de la Terre lorsqu’on veut qu’ils aient une trajectoire circulaire autour de la Lune ? 12. Pourquoi la planète la plus éloignée du Soleil a-t-elle la période de révolution la plus grande ? 13. Pourquoi un satellite soumis à une force de frottement a-t-il sa vitesse qui augmente ? 14. Pourquoi l’avance du périhélie de Mercure n’est-elle pas interprétée comme celle d’Uranus par la perturbation due à une autre planète ? 15. Un champ magnétique stationnaire n’augmente pas la norme de la vitesse d’une particule chargée. Pourquoi est-il si largement utilisé dans les accélérateurs de particules ? 16. Pourquoi l’énergie mécanique d’un système isolé ne se conserve-t-elle pas dans tous les cas ? 17. Pourquoi le roulement sans glissement présente-t-il un intérêt énergétique ? 18. Lorsqu’on effectue le bilan des forces qui agissent sur un véhicule ou sur une personne qui marche, le long d’un trajet horizontal, on est amené à conclure que c’est la force de frottement qui permet le déplacement. Est-ce paradoxal ? Pourquoi ? 19. Pourquoi notre signe zodiacal (Bélier, Taureau, . . ., Poissons) ne coïncide-t-il pas avec la constellation dans laquelle se trouvait le Soleil le jour de notre naissance ? 20. Pourquoi un joueur de football peut-il, seul, dans un tir de corner, marquer un but ?

1 Calcul vectoriel. Torseurs Analyse dimensionnelle

Même si l’addition vectorielle date des Grecs pour les vitesses et du XVI e siècle pour les forces (règle du parallélogramme), la forme actuelle du calcul vectoriel est attribuée aux mathématiens irlandais W. Hamilton en 1843 et allemand H. Grassmann en 1855. Comme outil mathématique en physique, il joue un rôle considérable car de nombreuses grandeurs physiques (vitesse, accélération, quantité de mouvement, etc.) sont représentées par des vecteurs. Il apparaît donc naturel de rappeler, avant tout développement, les propriétés des vecteurs et notamment les opérations telles que l’addition vectorielle, le produit scalaire et le produit vectoriel. On introduit ensuite la notion de vecteur lié et celle de torseur associé à un champ de vecteurs antisymétrique. Ce dernier concept est techniquement commode lorsqu’on est conduit à considérer des ensembles de vecteurs liés.

 c Dunod – Toute reproduction non autorisée est un délit

Enfin, on présente l’analyse dimensionnelle en soulignant son intérêt majeur en physique, pas uniquement en mécanique.

I . — ESPACE VECTORIEL I . 1 . — Définition On appelle espace vectoriel E sur un corps commutatif K un ensemble d’éléments, appelés vecteurs, qui satisfait aux propriétés suivantes : i) l’ensemble E est muni d’une structure de groupe commutatif pour une loi de composition interne, l’addition vectorielle, notée simplement +. ii) Pour deux vecteurs U et V appartenant à E, on a, si l et m appartiennent à K : l(U + V) = lU + lV

(l + m)U = lU + mU

l(mU) = (lm)U

1U = U

2

1. Calcul vectoriel. Torseurs. Analyse dimensionnelle

I . 2 . — Espace vectoriel euclidien Un espace vectoriel E est euclidien s’il est muni d’un produit scalaire f qui, à deux vecteurs U et V de E, fait correspondre le nombre réel f (U, V) tel que : f (U, V) = f (V, U)

f (U, lV) = lf (U, V)

f (U, V + W) = f (U, V) + f (U, W) f (U, U) > 0

U = 0

si

et f (U, U) = 0 si U = 0.

La quantité f (U, U) est appelée le carré de la norme de U, laquelle est notée U ou plus brièvement U. Le plus souvent, on note le produit scalaire des deux vecteurs U et V simplement U · V. I . 3 . — Base d’un espace vectoriel On appelle base d’un espace vectoriel un système de n vecteurs de E, indépendants, permettant d’exprimer linéairement tout vecteur de E. On a donc : n

U=

xi ei i=1

Les différents coefficients xi sont appelés composantes de U dans la base considérée. La base est orthonormée si, quels que soient i et j différents, on a : ei · e i = 1 et ei · ej = 0.

II . — ESPACE AFFINE II . 1 . — Définition On appelle espace affine E un ensemble d’éléments, appelés points, tel qu’à tout couple ordonné (AB) de deux points A et B (bipoint), on puisse faire correspondre un vecteur AB d’un espace vectoriel E ; si A, B, C désignent trois points de E, on doit avoir : i) AB = −BA ii) AC = AB + BC iii) O étant un point quelconque de E et V un vecteur appartenant à E, il existe un point A et un seul de E défini par OA = V. II . 2 . — Espace métrique Un espace métrique est un espace affine auquel on a associé un espace vectoriel euclidien. On peut alors définir une norme pour tout vecteur associé aux points A et A  : la norme est la distance de ces points. En coordonnées cartésiennes, c’est-à-dire dans une base orthonormée (e x, e y , ez) de E, on a : OA =

x i ei

OA =

i

xi ei

d’où

i

AA = OA  − OA =

On en déduit la norme au carré de AA  selon : AA2 = i

(xi − xi ) ei ·

j

(xj − xj ) ej =

i

(xi − xi )2

i

(x i − xi ) ei

3

Calcul vectoriel. Torseurs. Analyse dimensionnelle

puisque ei · ei = 1 et ei · ej = 0. Dans l’espace affine physique à trois dimensions, la norme a pour expression : AA = [(x − x)2 + (y − y)2 + (z − z)2]1/2

soit

AA = d s = (d x 2 + d y2 + d z 2 )1/2

si les deux points A et A  sont infiniment voisins. II . 3 . — Bases directe et inverse Trois vecteurs U, V et W forment une base directe si un observateur placé sur OA = U, les pieds en O et regardant vers OB = V, voit OC = W à sa gauche (Fig. 1.1a). Si OC est à sa droite, la base est inverse. Cette règle est appelée la règle du bonhomme d’Ampère ; elle est connue aussi sous le nom de règle du tire-bouchon ou règle de Maxwell : pour une base directe, toute rotation qui amène U vers V est accompagnée d’une translation suivant W. L’intérêt de cette distinction entre les bases directe et inverse est lié à l’existence de deux grandes catégories de grandeurs physiques : celles qui sont indépendantes de l’orientation de l’espace et celles qui en dépendent. Les bases généralement utilisées en physique sont orthonormées et directes, comme la base dite cartésienne {ex, ey , ez } (cf. chapitre 2) (Fig. 1.1b). z

A U B

V

O

W O

C

y

x

a) Repère direct

b) Repère orthonormé direct F IG . 1.1.

III . — OPÉRATIONS SUR LES VECTEURS  c Dunod – Toute reproduction non autorisée est un délit

III . 1 . — Produit scalaire a) Expression analytique Soient U = i Ui ei et V = i Vi ei deux vecteurs de E exprimés dans une base orthonormée directe. D’après les propriétés du produit scalaire, on a : U·V =

Ui Vi i

b) Représentation géométrique Soient deux points A et B tels que OA = U et OB = V (Fig. 1.2). Dans le repère d’espace Oxyz tel que OA est colinéaire à ex et OB contenu dans le plan Oxy, Ux = U et Vx = V cos u, projection de V suivant la direction de U. Or U · V = Ux V x. Par conséquent : U · V = UV cos u

4

1. Calcul vectoriel. Torseurs. Analyse dimensionnelle

Remarque : D’après cette relation, on voit que le produit scalaire est un nombre indépendant de la base d’où son nom : c’est une grandeur intrinsèque. y B

V

θ U

O

A

x

F IG . 1.2.

III . 2 . — Produit vectoriel a) Définition Considérant deux vecteurs U et V, dont les composantes respectives sont (U 1, U 2, U 3) et (V1, V2, V3 ) dans une base orthonormée directe (e1, e 2, e3), on appelle produit vectoriel de U et V, noté W = U × V, le vecteur suivant : W = (U 2V3 − U 3V2 ) e1 + (U3 V 1 − U1V3 ) e2 + (U1V 2 − U 2V 1) e 3 Dans la pratique, on explicitera souvent U × V selon : U1 V1 U 2V 3 − U3 V2 U 2 × V 2 = U 3V 1 − U1 V3 U3 V3 U 1V 2 − U2 V1 Notons que les différentes composantes s’obtiennent à partir de l’une d’entre elles par une simple permutation circulaire. En outre, l’écriture des vecteurs en colonnes permet de mémoriser facilement les composantes du produit vectoriel ; ainsi, la première composante du vecteur W se calcule à partir des deuxièmes et troisièmes composantes des vecteurs U et V selon la règle simple en croix suivante : U2V 3 − U3 V2 .

La définition de W = U × V est telle que la base (U, V, W) a la même orientation que (e 1, e 2, e3). Pour l’illustrer, plaçons-nous dans le cas particulier où U est dirigé suivant e 1 et V suivant e 2 (U1 > 0, U 2 = 0, U3 = 0, V1 = 0, V2 > 0, V3 = 0). On trouve : W = U 1 V 2 e3

avec

U1 V2 > 0

Ainsi, lorsque U est orienté suivant e1 et V suivant e2 , U × V l’est suivant e3. Le sens du vecteur obtenu dépend donc de la convention d’orientation. De tels vecteurs sont appelés vecteurs axiaux. On les signale parfois par une flèche courbe, réservant la flèche droite aux vecteurs habituels dits polaires car définis indépendamment de l’orientation de la base. Remarque : Cette distinction entre vecteurs polaire et axial, qui dépasse largement le cadre de la mécanique, disparaît techniquement si les bases utilisées sont directes. Cependant, elle ne doit pas être sous-estimée ; en électromagnétisme par exemple, elle joue un rôle capital : le champ électrique E est un vecteur polaire alors que le champ magnétique B est un vecteur axial ; il en résulte que la nature profonde de ces deux champs et donc leurs propriétés de symétrie, diffèrent fondamentalement (cf. Électromagnétisme).

5

Calcul vectoriel. Torseurs. Analyse dimensionnelle b) Propriétés du produit vectoriel

i) D’après la définition précédente, W = 0 si U = 0, V = 0 ou si U et V sont parallèles. En effet, dans ce dernier cas, on a U 1/V1 = U 2/V2 = U3 /V3 . ii) À partir des composantes, il est facile de voir que U · W = 0 et V · W = 0. Donc W est perpendiculaire à U et V. iii) La norme de W est UV| sin(U, V)|. En effet : W 2 = (U 2V 3 − U3 V 2 )2 + (U3 V1 − U1 V3)2 + (U 1V2 − U2V1) 2 = U 2V 2[1 − cos 2(U, V)] = U 2V 2 sin2(U, V)

c) Signification géométrique de U × V

La signification géométrique du produit vectoriel découle de ce qui précède. La norme de W, qui vaut UV| sin(U, V)| s’identifie à l’aire du parallélogramme construit avec les vecteurs U et V (Fig. 1.3). Donc, W est un vecteur normal à U et V, orienté de telle façon que la base (U, V, W) soit directe, et de norme égale à l’aire du parallélogramme construit avec U et V. W W

U3 h

V

U2

U F IG . 1.3.

U1

F IG . 1.4.

d) Règles de calcul D’après la définition, on a : U × V = −V × U

(lU) × (mV) = lmU × V

U × (V1 + · · · + V n) = U × V1 + · · · + U × Vn

U × lU = 0

 c Dunod – Toute reproduction non autorisée est un délit

III . 3 . — Produit mixte de trois vecteurs dans un espace à trois dimensions Par définition, le produit mixte de trois vecteurs U 1 , U2 , U 3 est la quantité scalaire : (U1 × U2 ) · U3 a) Signification géométrique Sur la figure 1.4, on peut voir que la valeur absolue du produit mixte s’identifie au volume limité par le parallélépipède construit avec les trois vecteurs U1 , U2 , U 3 : la norme de W = U 1 × U2 est l’aire de la base et la norme de U3 cos(W, U 3) est la hauteur h. b) Permutation circulaire Si, dans le produit mixte (U 1 × U 2 ) · U3, on effectue une permutation circulaire sur les indices 1, 2, 3, l’interprétation géométrique précédente permet d’établir que : (U1 × U 2) · U 3 = (U2 × U3 ) · U1 = (U 3 × U1 ) · U2

6

1. Calcul vectoriel. Torseurs. Analyse dimensionnelle

III . 4 . — Double produit vectoriel On appelle double produit vectoriel de trois vecteurs U 1 , U2 , U3 , le vecteur U1 × (U2 × U3 ).

D’après les propriétés du produit vectoriel, ce vecteur est normal au vecteur U 2 × U3. Il est donc contenu dans le plan défini par U2 et U 3 et peut se mettre sous la forme d’une combinaison linéaire de U2 et U3 . À partir des composantes dans une base orthonormée, on montre aisément que (cf. Exercices) : U1 × (U 2 × U 3) = U2(U1 · U3 ) − U 3(U 1 · U2 ) Notons que dans le cas où deux des vecteurs sont égaux, par exemple U 1 = U 2, on a (Fig. 1.5) : U 1 × (U1 × U3) = U 1(U 1 · U 3) − U 3(U12) = −U 21[U 3 − e 1 (e 1 · U3 )] = −U12 U3,⊥ U 3,⊥ étant le vecteur projection de U 3 dans le plan perpendiculaire à U 1. Ce résultat sera utilisé pour établir l’expression de la force d’inertie d’entraînement dans un mouvement de rotation uniforme (cf. chapitre 7). U1

U3

e1 U3 ,? F IG . 1.5.

Plus généralement, on retiendra que le double produit vectoriel de trois vecteurs, dont deux sont identiques à e, de norme 1, permet d’obtenir le vecteur projection du troisième vecteur dans un plan perpendiculaire à e. En effet : e × (e × U) = −U + e(U · e) = −U ⊥ III . 5 . — Détermination des composantes d’un vecteur Bien que purement technique, la détermination des composantes d’un vecteur est capitale pour traduire les lois physiques de nature vectorielle. Dans la suite, pour la commodité des calculs, nous ne considérerons que des bases orthonormées directes. a) Technique de projection Il arrive très souvent que l’on doive projeter un vecteur unitaire e situé dans un plan orienté. Il convient alors de localiser d’abord l’axe polaire, c’est-à-dire l’axe qui, par une rotation de +p/2, se confond avec le second axe du plan, puis d’évaluer algébriquement l’angle que fait le vecteur unitaire e avec cet axe. Ainsi, dans le cas de la figure 1.6, il n’est pas inutile de rappeler que les composantes de e, situé dans le plan Oxy, sont + cos u et + sin u dans tous les cas ; dans le premier, l’angle u est aigu et positif, dans le second il est obtus et positif, dans le troisième, il est aigu et négatif. y

y

y

e

e

µ

µ Oz

x

Oz F IG . 1.6.

x

Oz e

µ

x

7

Calcul vectoriel. Torseurs. Analyse dimensionnelle

Exemple : Soit à exprimer, en fonction de l’angle u sur la figure 1 . 7, les composantes du vecteur CA, de norme r. Comme (Cx, CA) = (Cx, CI) + (CI, CA) = −p/2 + u, les composantes de CA suivant les axes x et y sont respectivement : r cos(−p/2 + u) = r sin u

et

r sin(−p/2 + u) = −r cos u

Pour retrouver ce résultat à l’aide de la figure, il suffit de remarquer que l’angle u est obtus et négatif. En faisant u = −2p/3, on retrouve bien r sin u < 0 et −r cos u > 0. y z

A C

r

g

T = Tz ez

θ

π/2

O

x

z

O

x

I

O

F IG . 1.7.

F IG . 1.8.

Remarques : (1) Lorsqu’un vecteur a un sens connu et déterminé, il est judicieux d’en tenir compte directement dans la projection. Ainsi, pour le vecteur g, orienté suivant une verticale descendante, on écrira, g = g étant sa norme, g = −g e z, plutôt que g = g z e z avec gz = −g (Fig. 1.8). Dans le cas contraire, où un vecteur T, dirigé suivant Oz, est de sens inconnu, on écrira T = Tz ez , Tz étant la mesure algébrique de T suivant l’axe. (2) Le choix d’un trièdre orthonormé définit le sens positif d’une rotation dans un des plans du trièdre. b) Coordonnées cylindriques Les coordonnées cylindriques sont évidemment adaptées pour décrire des systèmes physiques qui ont une symétrie cylindrique. Désignons par P la projection perpendiculaire du point A dans le plan Oxy et par H sa projection parallèle à ce plan suivant l’axe Oz (Fig. 1.9). Par définition, les coordonnées cylindriques sont :

 c Dunod – Toute reproduction non autorisée est un délit

r = OP

w = (Ox, OP)

et z = OH z

z

H

A

ez H

θ



A

O x

ϕ

ρ

P

r eθ

O



ϕ y

F IG . 1.9.

θ

x

P

F IG . 1.10.

On voit aisément, à l’aide de la figure, que : x = r cos w

er eϕ

et y = r sin w

y

8

1. Calcul vectoriel. Torseurs. Analyse dimensionnelle

La base orthonormée directe cylindrique est formée de trois vecteurs (er , ew , ez ) respectivement définis par le déplacement vectoriel de A lorsque r, w, z varient séparément. On obtient er colinéaire à OP, ew normal au plan w = Cte et ez colinéaire à Oz (cf. annexe 2). c) Coordonnées sphériques Les coordonnées sphériques sont, elles, naturelles pour décrire des systèmes physiques qui ont une symétrie sphérique. Par définition, on appelle ainsi les coordonnées : u = (Oz, OA)

r = OA

w = (Ox, OP)

On appelle u la colatitude et w l’azimut (Fig. 1.10) : x = r sin u cos w

y = r sin u sin w

z = r cos u

La base orthonormée directe sphérique est formée des trois vecteurs (er, eu , ew ) définis respectivement par le déplacement vectoriel de A lorsque r, u et w varient séparément. On obtient e r colinéaire à OA, e u normal à OA dans le plan w = Cte et e w normal au plan w = Cte. Dans cette base, OA = r er . Remarque : On se reportera à l’annexe 2 pour connaître les expressions des surfaces élémentaires et du volume élémentaire en coordonnées cylindriques et sphériques.

IV . — VECTEUR LIÉ ET SYSTÈME DE VECTEURS LIÉS IV . 1 . — Définition d’un vecteur lié On appelle vecteur lié le couple d’un vecteur V de E et d’un point A de E associé à E. On le note (A, V). Par exemple, la force qui s’exerce sur un point matériel A peut être représentée par un vecteur lié (A, F). IV . 2 . — Moment en un point d’un vecteur lié Par définition, le moment en un point O d’un vecteur lié (A, V) est le vecteur : M O = OA × V On voit aisément, à l’aide de la figure 1.11, que M O  = |OA V sin u| = OH V. Établissons la relation entre MO et le moment en un autre point O , MO  : Retenons donc :

MO = OA × V = (OO  + O A) × V = OO × V + MO MO = M O + OO × V

IV . 3 . — Moment d’un vecteur lié par rapport à un axe Par définition, O étant un point d’un axe D de vecteur unitaire e D , le moment du vecteur lié (A, V) par rapport à D est le produit scalaire (Fig. 1.12) : MD = eD · M O = e D · (OA × V)

9

Calcul vectoriel. Torseurs. Analyse dimensionnelle O Δ

MO

MO



O

A

V

θ

H

V

O A

F IG . 1.11.

F IG . 1.12.

Cette notion a bien un sens, puisqu’elle ne dépend pas du point O choisi sur l’axe. Montrons-le en introduisant un second point O de l’axe : MD = eD · (OA × V) = e D · (OO  + OA) × V = eD · (OO  × V) + eD · (O A × V)

Comme eD et OO sont colinéaires, on a bien : eD · (OA × V) = eD · (O A × V). IV . 4 . — Vecteur et moment d’un système de vecteurs liés

Soit un ensemble de vecteurs liés {(A i , V i)}. Le vecteur S et le vecteur moment M O, en un point O arbitraire, de ce système de vecteurs liés, sont définis selon : S=

Vi i

et MO = i

OA i × Vi

On les appelle les éléments de réduction du système de vecteurs liés. Exprimons, en O, le moment résultant MO , en introduisant un autre point arbitraire O  : MO = i

OAi × V i =

i

(OO  + O A i) × Vi = M O + OO ×

Vi i

Par conséquent : MO = MO  + OO × S Notons que les moments, en un même point O, des deux vecteurs liés (A, V) et (A , V), tels que AA  soit colinéaire à V, sont égaux, puisque :

 c Dunod – Toute reproduction non autorisée est un délit

OA × V = (OA  + A A) × V = OA  × V

Il en résulte que le nouveau système de vecteurs liés, obtenu en faisant glisser les vecteurs sur leurs supports, a même vecteur et même moment que le système initial. Aussi certains auteurs introduisent-ils le concept de vecteur glissant, sans grand intérêt. IV . 5 . — Couple On appelle couple tout système de vecteurs dont le vecteur résultant est nul et le moment résultant non nul. Il en résulte que le moment d’un couple est indépendant du point où on le calcule : MA = M B = M L’origine du mot couple vient des propriétés du système formé par l’ensemble de deux vecteurs liés (A1, F1) et (A 2, −F 1) dont la somme est un vecteur nul : S = F 1 + F2 = 0. Le moment est alors : M A = AA1 × F 1 + AA2 × (−F1 ) = A2 A 1 × F1 = F1 d e z

d étant la distance entre les supports des deux vecteurs liés (Fig. 1.13).

10

1. Calcul vectoriel. Torseurs. Analyse dimensionnelle

d −F 1

F1

A2

A1

F IG . 1.13.

IV . 6 . — Systèmes de vecteurs liés concourants Considérons un système de vecteurs liés concourants et désignons par C le point de concours de droites définies par les vecteurs liés. On a (Fig. 1.14) : MC = i

CA i × Vi = 0

MO = OC × S

d’où

en un point quelconque O . Ainsi, le moment M O en un point O d’un système de vecteurs liés concourants en un point C est le même que celui d’un vecteur lié (C, S) ou de tout autre vecteur lié obtenu en faisant glisser ce dernier sur son support. Comme exemple, citons les forces de gravitation qu’exercent les éléments d’une distribution de masse sur un point matériel extérieur. S

Vi

C

Ai

An

V2

V1

A1 V1

A3

An

S Vi

A2

A1 Vn

V3

A2

Vn

K Ai

V2

F IG . 1.14.

F IG . 1.15.

IV . 7 . — Système de vecteurs liés parallèles Désignons par e le vecteur unitaire commun à tous ces vecteurs liés (Fig. 1.15). Il vient S = ( i Vi )e et, au point arbitraire O, MO = i OAi × Vi = ( i Vi OA i ) × e. Posant vient :

i

Vi OAi = S OK, K étant le barycentre des Ai affectés des mesures algébriques V i, il M O = S OK × e

soit

M O = OK × S

Ainsi, le moment MO en un point O d’un système de vecteurs liés parallèles est le même que celui du vecteur lié (K, S) ou de tout autre vecteur lié obtenu en faisant glisser ce dernier sur son support. Exemple : Le vecteur lié associé à la force de pesanteur (C, M g), représentant l’action locale de la Terre sur un système matériel de centre de masse C et de masse M, a même moment que l’ensemble des vecteurs liés {Ai , m ig}. Comme il en est de même pour tous les vecteurs liés obtenus en faisant glisser (C, Mg) sur son support, on dit que cette force est bien représentée par un vecteur glissant.

11

Calcul vectoriel. Torseurs. Analyse dimensionnelle

V . — CHAMP DE VECTEURS ANTISYMÉTRIQUE V . 1 . — Champ de vecteurs On appelle champ de vecteurs l’application qui fait correspondre, à tout point A de E, un vecteur U d’un espace vectoriel E de même dimension que E. Par exemple, le champ électrique et le champ magnétique produits en tout point de l’espace physique par des charges électriques sont des champs de vecteurs. V . 2 . — Champ antisymétrique a) Définition Un champ de vecteurs M(A) est antisymétrique s’il existe un vecteur S tel que, quels que soient les points A et B, on ait : M(A) = M(B) + AB × S Le vecteur S est le vecteur du champ antisymétrique. Remarque : Compte tenu de la définition précédente, on voit que S et M(A) ont des caractères polaire et axial différents. b) Équiprojectivité Si on multiplie scalairement les deux membres de l’équation précédente par AB, on a : AB · M(A) = AB · [M(B) + AB × S] = AB · M(B)

puisque

AB · (AB × S) = 0

L’égalité AB · M(A) = AB · M(B) est la propriété d’équiprojectivité d’un champ antisymétrique (Fig. 1.16) : AB · M(A) = AB eAB · MA et AB · M(B) = AB e AB · MB

 c Dunod – Toute reproduction non autorisée est un délit

eAB étant le vecteur unitaire défini par AB ; il en résulte que : eAB · MA = eAB · MB

soit

AH = BK

M(B) M(A) A

H

B

K

F IG . 1.16.

Réciproquement, tout champ équiprojectif est un champ antisymétrique. Pour le montrer, introduisons un point arbitraire O et le champ M(O) en ce point. En retranchant AB · M(O) à AB · M(A), il vient : (OB − OA) · [M(A) − M(O)] = (OB − OA) · [M(B) − M(O)] puisque AB · M(A) = AB · M(B), d’après l’équiprojectivité du champ.

12

1. Calcul vectoriel. Torseurs. Analyse dimensionnelle Il en résulte, puisque OA · [M(A) − M(O)] = 0 et OB · [M(B) − M(O)] = 0 : OB · [M(A) − M(O)] = −OA · [M(B) − M(O)]

Le champ W(A) = M(A) − M(O) satisfait donc à la propriété : OB · W(A) = −OA · W(B)

d’où

OA · W(A) = −OA · W(A)

et donc

OA · W(A) = 0

De même, OB · W(B) = 0. Comme W(A) est perpendiculaire à OA, on peut l’écrire, S étant un vecteur fixé : W(A) = AO × S soit M(A) − M(O) = AO × S

VI . — TORSEURS Comme nous le verrons ultérieurement (cf. chapitre 18), les théorèmes généraux de la dynamique des systèmes matériels font apparaître deux ensembles de vecteurs liés : celui des quantités de mouvement et celui des forces. Cependant, ces ensembles n’interviennent dans ces théorèmes que par leurs éléments de réduction, d’où une impossibilité de distinguer les effets sur le mouvement de deux systèmes de vecteurs liés qui admettent les mêmes éléments de réduction : seuls importent, relativement à ces théorèmes, le champ antisymétrique et le vecteur associé à ces ensembles de vecteurs. On comprend, dans ce contexte, l’intérêt de mettre en avant les éléments de réduction d’un ensemble de vecteurs liés et par conséquent le concept de torseur construit à partir de ces seuls éléments de réduction. Historiquement, c’est l’ingénieur britannique R. Ball qui l’a introduit au milieu du XXe siècle, et le mathématicien français P. Appel qui a contribué à son usage dans l’enseignement français de la mécanique. VI . 1 . — Définition On appelle torseur [T ] l’ensemble d’un champ antisymétrique M(A) et de son vecteur S ; M(A) et S sont appelés respectivement moment et vecteur du torseur [T ]. M(A) = M(B) + AB × S Remarque : Cette définition dépasse le cadre initialement fixé par les théorèmes généraux, puisque, même en l’absence de systèmes de vecteurs liés, on peut associer un torseur à un champ de vecteurs antisymétrique. Par exemple en cinématique du solide, le torseur cinématique n’est pas construit à partir d’un ensemble de vecteurs liés, mais à partir du seul champ antisymétrique des vitesses (cf. chapitre 16) : v(A) = v(B) + AB × V pour lequel le vecteur V, vitesse angulaire de rotation, n’est identifié à aucune somme de vecteurs élémentaires. VI . 2 . — Propriétés des torseurs a) Égalité Deux torseurs [T 1 ] et [T2] sont égaux si et seulement si leurs éléments de réduction sont égaux : S 1 = S2

et M1 (A) = M2 (A)

Calcul vectoriel. Torseurs. Analyse dimensionnelle

13

b) Somme La somme de deux torseurs [T 1] et [T2] est le torseur [T ] dont les éléments de réduction sont la somme des éléments de réduction de chacun des deux torseurs : S = S1 + S2

et M(A) = M1(A) + M2 (A)

c) Multiplication par un scalaire Le torseur l[T ], l étant un scalaire, admet lS comme vecteur et lM(A) comme moment. d) Torseur nul Un torseur [T ] est nul si ses éléments de réduction sont nuls : S = 0 et M(A) = 0. e) Produit scalaire de deux torseurs On appelle produit scalaire de deux torseurs [F] = {F, M(A)} et [v] = {V, v(A)} la quantité suivante : [F][v] = F · v(A) + M(A) · V Cette définition est suggérée par l’expression de la puissance qu’exerce un système de forces sur un solide en mouvement. f) Invariants d’un torseur D’après la relation de définition d’un torseur, on établit aisément, en projetant le vecteur M sur S et sur AB, que : i) S · M(A) = S · M(B)

ii) AB · M(A) = AB · M(B) (équiprojectivité).

iii) Le produit scalaire de deux torseurs est aussi un invariant. Montrons que [F][v] est indépendant du point A en lequel on exprime les moments. Comme :

 c Dunod – Toute reproduction non autorisée est un délit

on a :

v(A) = v(B) + AB × V et M(A) = M(B) + AB × F

F · v(A) + M(A) · V = F · [v(B) + AB × V] + [M(B) + AB × F] · V = F · v(B) + M(B) · V g) Axe central d’un torseur L’ensemble des points H de l’espace en lesquels le moment d’un champ antisymétrique M(A) est parallèle à son vecteur S est une droite appelée axe central. On établit ce résultat à partir de l’équation (cf. Exercices) : M(H) = lS d’où lS = M(A) + HA × S Il est facile de montrer qu’en des points de l’axe central, la norme du champ antisymétrique est minimale : en effet, si l’on désigne par u l’angle que font entre eux les vecteurs S et M(A), on a : S · M(A) = S M(A) cos u La norme S étant invariante, la norme de M(A) est minimale si cos u est maximal, c’est-à-dire aux points H de l’axe central pour lesquels u = 0 puisque S et M(A) sont parallèles.

14

1. Calcul vectoriel. Torseurs. Analyse dimensionnelle

VI . 3 . — Torseur associé à un ensemble de vecteurs liés Comme le moment d’un ensemble de vecteurs liés {(A i , V i )} a la forme d’un champ antisymétrique, on lui associe le torseur caractérisé par les éléments de réduction : S=

Vi

(vecteur)

et

M(O) =

i

i

OA i × Vi

(moment)

a) Équivalence torsorielle de deux systèmes de vecteurs liés Deux systèmes de vecteurs liés sont torsoriellement équivalents si leurs torseurs associés sont égaux. Cette équivalence n’a de sens que relativement à la théorie des torseurs, ce qui traduit une équivalence physique précise et limitée. Par exemple, les torseurs associés aux systèmes de vecteurs représentés sur la figure 1.17 : {(A1 , F), (A2, −F)} et {(A 1 , −F), (A2 , F)} sont tous les deux nuls. Ces deux systèmes de vecteurs liés sont donc torsoriellement équivalents. Sur un corps indéformable tel qu’un solide idéal, les effets sur le mouvement sont identiques. En revanche, sur un corps déformable tel qu’un ressort, bien que le mouvement d’ensemble du ressort soit le même (centre de masse immobile et absence de rotation), les effets sont différents : dans le premier cas le ressort est dilaté, alors que dans le second il est comprimé.

−F A 2 A2 F

A1 F − F A1 F IG . 1.17.

b) Systèmes de vecteurs équivalents à un ensemble de vecteurs liés concourants ou parallèles Un système de vecteurs liés concourants en C et de somme S est torsoriellement équivalent au vecteur lié (C, S). De même, un système de vecteurs liés parallèles et de somme S est torsoriellement équivalent au vecteur lié (K, S), K étant le barycentre associé.

VI . 4 . — Système de trois vecteurs liés de torseur nul Cherchons les conditions dans lesquelles le torseur associé à un système de trois vecteurs liés (A1, V 1) (A 2 , V2 ) et (A3, V 3 ) est nul. Ce cas est intéressant en statique (cf. chapitre 23) lorsqu’un solide, soumis à trois forces, est immobile. Le torseur étant nul, la somme des moments des trois vecteurs liés par rapport à toute droite D, s’appuyant sur les supports de deux d’entre eux, doit être nulle (Fig. 1.18). Par conséquent, le moment par rapport à D du troisième vecteur lié est nul. Il en résulte que le support de ce vecteur lié passe par D et, donc que les trois vecteurs liés doivent être dans le même plan défini par les points A1 , A2 et A3.

15

Calcul vectoriel. Torseurs. Analyse dimensionnelle

A1

V1

D

A2

C V2

V3 A3

F IG . 1.18.

En outre, les trois vecteurs liés sont concourants puisque le moment d’un vecteur lié au point d’intersection des supports des deux autres doit être nul. Finalement, les conditions dans lesquelles le torseur associé à trois vecteurs liés est nul sont les suivantes : i) les trois vecteurs liés sont coplanaires, ii) ils sont concourants, iii) ils satisfont à l’équation V 1 + V2 + V3 = 0. Remarque : Dans le cas singulier banal où les trois vecteurs liés sont parallèles, le point d’intersection est rejeté à l’infini.

VII . — ANALYSE DIMENSIONNELLE En physique et donc en mécanique, on ne se limite pas à l’observation et à l’analyse qualitative des phénomènes qui nous entourent. On étudie aussi la nature en utilisant des lois quantitatives empiriques (issues de l’expérience) entre des grandeurs physiques. Ces lois sont ensuite réunies en un petit nombre de lois fondamentales, supposées universelles tant que les faits n’en montrent pas la limite. Un tel ensemble diminue donc le nombre de lois : on réalise ainsi une synthèse. On peut ainsi ramener la physique à la quête de l’universalité à travers la réalisation de synthèses. Ce mode de pensée diffère fondamentalement du relativisme (philosophique) où on multiplie les lois et particularise les faits. VII . 1 . — Unités des grandeurs physiques

 c Dunod – Toute reproduction non autorisée est un délit

a) Définition La mesure d’une grandeur physique X est la comparaison de X à une autre grandeur, de même nature, X0 , choisie arbitrairement comme unité ; précisément, on effectue le rapport : mX =

X X0

Le résultat de la mesure est un nombre mX , valeur de X , suivi de son unité. b) Unités de base du Système International et unités dérivées Dans la suite, on utilisera essentiellement trois unités du Système International (SI), la seconde pour les durées, le mètre pour les longueurs et le kilogramme pour les masses. Toutes les autres unités, qui découlent de ces unités de base à partir de relations de définition, sont des unités dérivées. Par exemple, l’unité de vitesse m.s −1 est une unité dérivée, car, par définition, la vitesse d’un point est le rapport entre la longueur d’un trajet et la durée de parcours de ce trajet.

16

1. Calcul vectoriel. Torseurs. Analyse dimensionnelle

c) Incertitudes expérimentales En physique, toute mesure d’une grandeur X doit s’accompagner de son incertitude expérimentale DX , c’est-à-dire qu’il faut donner l’intervalle de valeurs de X , autour de la valeur mesurée Xm , qui est censé encadrer la valeur vraie Xv de X : X = X m ± DX

avec

DX > 0

ce qui signifie que Xm −DX  Xv  Xm +DX . Notons que cet intervalle n’est jamais nul puisqu’aucune grandeur physique ne peut être mesurée avec certitude. Une estimation raisonnable de l’incertitude expérimentale est donnée par l’instrument utilisé pour la mesure. Ainsi, une règle graduée en millimètres donne, sur la mesure d’une longueur, une incertitude de l’ordre de 1 mm . Par exemple, pour mesurer la longueur l d’un pendule constitué d’un fil inextensible et d’une masselotte sphérique, on doit relever deux positions, chacune ayant une incertitude de l’ordre de celle de la règle : celle du point de fixation et celle du point d’attache de la masselotte. On trouve, par exemple : l = (0, 492 ± 0, 002) m ce qui signifie que la longueur du pendule est comprise entre 0, 490 m et 0, 494 m .

De même, un chronomètre permettant d’accéder au dixième de seconde donne, pour la mesure d’une durée, des mesures incertaines en ordre de grandeur à 0,1 s . Par exemple, pour la période T 0 du pendule précédent, on mesure dans la pratique dix périodes afin de réduire les incertitudes sur la mesure d’une seule période ; en outre l’incertitude de 0, 1 s se retrouve lors du déclenchement du chronomètre et lors de son arrêt. Par conséquent : 10 T0 = (14, 1 ± 0, 2) s d’où T 0 = (1, 41 ± 0, 02) s ce qui signifie que la période du pendule est comprise entre 1, 39 s et 1, 43 s . VII . 2 . — Dimension des grandeurs physiques La dimension physique est la propriété que l’on peut associer à un ensemble de grandeurs physique de même nature. Par exemple, la taille d’une règle et celle d’un être vivant possèdent la dimension d’une longueur. On dit que ces grandeurs ont même dimension, ou encore qu’elles sont homogènes. a) Absurdité dimensionnelle En physique, il est absurde d’écrire une relation d’égalité entre deux grandeurs de dimensions physiques différentes. Par exemple, le nombre qui exprime, au cours de l’évolution d’un système, un instant t , et celui qui représente sa position x le long d’un axe sont mathématiquement deux réels dont il est physiquement absurde de concevoir l’égalité. En effet, le premier a la dimension physique d’une durée alors que le second est homogène à une longueur. Les lois physiques prennent généralement la forme de relations entre plusieurs grandeurs de dimensions physiques différentes. Pour éviter toute absurdité, il est nécessaire que les deux membres de l’équation qui exprime une relation soient, eux, de même dimension physique. Ainsi, on peut écrire une loi d’évolution de la position x de la façon suivante : x = at + b à condition que a ait la dimension d’une longueur par unité de temps et b celle d’une longueur.

Calcul vectoriel. Torseurs. Analyse dimensionnelle

17

b) Absurdité vectorielle On peut rencontrer un deuxième type d’absurdité physique si on relie des vecteurs de natures différentes : certains, comme les vecteurs position r , vitesse v , accélération a et force F , sont définis indépendamment de toute convention d’orientation de l’espace, contrairement à d’autres, tel que le champ magnétique B qui est indissociablement lié à une telle convention. Les premiers sont appelés vecteurs polaires alors que les seconds sont qualifiés d’axiaux. La convention d’orientation est la règle du bonhomme d’Ampère ou du tire-bouchon de Maxwell : le sens de progression d’un tire-bouchon qui tourne dans le sens trigonométrique est positif. Dans ce contexte, il est absurde en physique d’égaler un vecteur polaire et un vecteur axial. Ainsi, l’égalité entre une vitesse et une vitesse angulaire de rotation est physiquement doublement absurde : d’une part la dimension physique n’est pas la même, d’autre part la nature de ces vecteurs est différente ; en revanche la relation donnant le champ des vitesses d’un solide : v(A) = v(B) + AB × V est correcte à la fois dimensionnellement et du point de vue strictement vectoriel, car V est certes un vecteur axial mais la multiplication vectorielle prend en compte cette spécificité. c) Absurdité structurelle Un troisième type d’absurdité physique peut se manifester en manipulant des grandeurs physiques de structures mathématiques différentes : égaler un nombre et un vecteur, ou encore une différentielle, qui peut être infiniment petite, et une quantité finie, n’a pas de sens. VII . 3 . — Équations aux dimensions On appelle équation aux dimensions une équation qui relie la dimension d’une grandeur à celles des grandeurs de base. a) Exemple de la force de gravitation

 c Dunod – Toute reproduction non autorisée est un délit

La force de gravitation, attractive, entre deux corpuscules A 1 et A2, distants de r , a pour expression : m m  A 2A1 r r1 − r2 OA 1 − OA 2 = = = F2→1 = −G 1 2 2 e r avec er = r A 2A 1 r r r Cette loi fait apparaître les masses graves, m 1 et m 2 , lesquelles traduisent la capacité de ces corps à s’attirer mutuellement. Notons la présence de la constante de gravitation, G , ou constante de Newton, qui vaut en unité SI environ 6, 67 × 10 −11 . Déterminons la dimension physique de G . Il vient, en désignant par des crochets la dimension de toute grandeur : [F] = [G][M] 2[L]−2 d’où [G] = [F][L] 2[M]−2 [M] et [L] représentant respectivement les dimensions de masse et de longueur. Comme, d’après la loi fondamentale de la dynamique ou deuxième loi de Newton (cf. chapitre 4), [F] = [M ][L][T ] −2, on en déduit, T désignant le temps : [G] = [M][L][T ] −2[L] 2[M]−2 = [L]3 [M]−1 [T ]−2 Ainsi, G est homogène au produit d’une longueur au cube par l’inverse d’une masse et par l’inverse du carré d’un temps. Par conséquent, G ≈ 6, 67 × 10−11 m 3.kg−1.s −2 .

18

1. Calcul vectoriel. Torseurs. Analyse dimensionnelle

VII . 4 . — Analyse dimensionnelle L’analyse dimensionnelle est une méthode qualitative qui permet d’identifier les paramètres pertinents d’un phénomène physique et d’en déduire la dépendance d’une grandeur en fonction de ces paramètres. Évidemment, les facteurs numériques, sans dimension par définition, restent inaccessibles avec cette méthode ; leur détermination relève d’une étude quantitative. Exemples 1) Montrons, à l’aide de l’analyse dimensionnelle, que la période des petites oscillations d’un pendule simple est proportionnelle à (l/g)1/2 , l étant la longueur du pendule et g l’intensité du champ de pesanteur terrestre. A priori, l et g constituent, avec la masse m de la bille, les paramètres pertinents du système. La période T peut donc s’écrire dimensionnellement : [T] = [L] a[g]b[M]g

avec [g] = [L][T ] −2

puisque g est une accélération, celle due à la pesanteur. Il en résulte : [T] = [L] a+b [T ]−2b [M]g

soit

g=0

b = −a = −

1 2

On en déduit T = Cte × (l/g)1/2 . On retrouve ainsi, par la seule voie de l’analyse dimensionnelle, l’absence de la masse dans l’expression de la période d’un tel pendule, absence que l’on doit attribuer physiquement à l’égalité de la masse grave m∗ et de la masse inerte m . L’étude quantitative approfondie montre que cette constante vaut 2p . 2) Vitesse d’évasion v d’un objet dans l’environnement terrestre La vitesse d’évasion ou de libération d’un objet dans l’environnement terrestre est la vitesse que l’on doit lui communiquer pour que son énergie mécanique soit nulle (cf. chapitre 12). Les paramètres pertinents sont la masse MT de la Terre, la distance r de l’objet au centre de la Terre et la constante de Newton G . Par conséquent, on a : [v] = [G]a[M]b [L]g

avec

[G] = [L] 3 [M]−1 [T ]−2

Il en résulte : [v] = [L] 3a [M]−a[T ] −2a [M]b[L] g = [L]3a+g[M] b−a[T ] −2a avec Ainsi, on obtient : 3a + g = 1

b − a = 0 et

a = b = −g =

1 2

[v] = [L][T ]−1

2a = 1 d’où : et v =

GMT r

1/2

En réalité, la vitesse obtenue dans cet exemple est la vitesse de satellisation et non celle d’évasion, que √ l’on obtient en introduisant le facteur 2 . Cet exemple illustre bien la différence entre une analyse dimensionnelle, par définition qualitative, et une analyse quantitative. VII . 5 . — Similitude en physique Il est souvent instructif, avant de tenter de résoudre les équations fournies par les lois de la physique, d’extraire de ces lois des propriétés générales, issues de la seule analyse dimensionnelle.

Calcul vectoriel. Torseurs. Analyse dimensionnelle

19

a) Méthode des similitudes La méthode des similitudes consiste à multiplier chacune des grandeurs physiques qui interviennent dans une loi (longueur, temps, masse, etc.) par un facteur de telle sorte que la loi garde sa forme initiale. Cette technique permet d’une part d’établir des propriétés universelles, et, d’autre part, de diminuer considérablement les coûts d’expérimentation en travaillant sur des systèmes réduits, ou maquettes. On l’utilise largement en mécanique des fluides, en raison notamment de la difficulté de résolution des équations différentielles, mais aussi pour étudier expérimentalement, en laboratoire, des écoulements sur des reproductions réduites de la réalité (cf. annexe 5). b) Lois d’échelle Si toutes les dimensions géométriques d’un système sont multipliées par un facteur déterminé l , on obtient un nouveau système homothétique du précédent. Le volume initial est donc multiplié par l 3 , et donc son poids si la masse volumique est inchangée. Pour que le système homothétique ait le même comportement que le système initial, il importe que les autres forces, par exemple les forces intérieures, soient multipliées par l 3 elles-aussi. En réalité, il n’en est rien comme le montre l’analyse des conséquences d’une homothétie sur les systèmes déformables que sont les êtres vivants. En effet, on constate que les grands animaux sont généralement plus trapus que les petits. On explique cela par la nécessité pour les grands animaux de compenser leur poids important par une musculature adaptée.

CONCLUSION Retenons, au-delà des définitions et des opérations simples sur les vecteurs, les points essentiels suivants. (1) Le calcul technique des composantes d’un vecteur, notamment dans les systèmes de coordonnées cartésiennes, cylindriques et sphériques, doit être mené avec soin. (2) Un vecteur lié (A, V) est l’ensemble d’un point A et d’un vecteur V.

 c Dunod – Toute reproduction non autorisée est un délit

(3) La relation caractéristique d’un champ de vecteurs antisymétrique est : M(A) = M(B) + AB × S (4) Un torseur est l’ensemble de M(A) et de son vecteur S. (5) Les ensembles de vecteurs liés concourants ou parallèles sont torsoriellement équivalents à des vecteurs liés simples : le premier ensemble est équivalent à un vecteur dont le support passe par le point de concours, le second est équivalent à un vecteur parallèle à tous les vecteurs liés et tel que le support passe par le barycentre des points d’application. (6) Enfin, l’analyse dimensionnelle, qui consiste à combiner plusieurs paramètres physiques pertinents d’un système pour en dégager une grandeur caractéristique et l’estimer, constitue une méthode d’analyse qualitative préalable efficace. Elle joue un rôle essentiel chaque fois que les lois quantitatives sont difficilement exploitables ; c’est le cas notamment en mécanique des fluides (cf. chapitres 30 et 31).

20

1. Calcul vectoriel. Torseurs. Analyse dimensionnelle

EXERCICES ET PROBLÈMES P1– 1. Équation d’un plan On considère un vecteur a de composantes (2, 1, −2) dans la base (e x , ey , e z) et un point B de coordonnées (1, 2, 3). 1. Trouver les cosinus directeurs de a. 2. Quelle est l’équation d’un plan passant par B et perpendiculaire à a ? En déduire la distance de O au plan. P1– 2. Moment en un point et moment par rapport à un axe 1. Calculer le moment du vecteur lié (A, a) au point B. On donne, dans la base (e x, ey , e z) : OA = (0, 5, 2)

a = (−1, 2, 0)

et OB = (3, 1, 4)

2. Calculer le moment de (A, a) par rapport à l’axe D passant par B et parallèle au vecteur c de composantes (−1, 2, 2). P1– 3. Volume d’un parallélépipède Calculer le volume du parallélépipède clinorhombique (prisme oblique à base losange) défini par les trois vecteurs suivants : a = 3 ex + 2 ey, b = 3 ex − 2 e y , c = 2 ex + 7 e z l’unité étant le nanomètre. P1– 4. Double produit vectoriel Établir la formule du double produit vectoriel entre les trois vecteurs U, V, W : U × (V × W) = V(U · W) − W(U · V) On explicitera la première composante en coordonnées cartésiennes et on fera une permutation circulaire. Étudier le cas où U = V. P1– 5. Vecteurs réciproques On définit les vecteurs réciproques a ∗ , b∗, c∗ , associés aux trois vecteurs a, b, c, par les relations : a∗ = 2p

b×c

b ∗ = 2p

c×a

c ∗ = 2p

a×b

1. Effectuer les produits scalaires du type a · a ∗ et a · b∗ . 2. Établir la relation entre

et



= a ∗ · (b∗ × c∗ ).



= a·b×c

21

Calcul vectoriel. Torseurs. Analyse dimensionnelle P1– 6. Axe central d’un torseur

On considère, au point A de coordonnées cartésiennes (1, 3, 1), le torseur suivant dont les éléments de réduction sont : S = 2 ez et M(A) = 4 ex + 7 ey + 2 e z. 1. Trouver l’équation de l’axe central du torseur. 2. Quel est le vecteur moment en tout point H de l’axe central ? P1– 7. Torseur associé à trois vecteurs liés On considère, dans un repère d’espace Oxyz, les trois vecteurs : V 1(0, 0, 1), V2 (−1, 2, 0) et V3(a, b, g) , et les trois points A 1 (1, 0, 0), A2 (0, 1, 0) et A 3(0, 0, 1). On construit les vecteurs liés (Ai , Vi ). Déterminer les nombres réels a, b, g, tels que le système de ces trois vecteurs liés soit torsoriellement équivalent à un couple dont on calculera le moment. P1– 8. Mur énergétique de Planck En physique, il existe trois constantes fondamentales : la constante de Newton G , la constante d’Einstein c , ou vitesse de lumière dans le vide, et la constante de Planck barrée  , homogène au produit d’une énergie par une durée. Ces trois constantes valent, en unités SI, respectivement : G ≈ 6, 67 × 10−11 m3.kg−1 .s−2

c ≈ 2, 997 × 10 8 m.s −1

et  ≈ 1, 054 × 10−34 kg.m2.s −1

 c Dunod – Toute reproduction non autorisée est un délit

À l’aide d’une analyse dimensionnelle, combinant G , c et  , trouver la hauteur du mur énergétique de Planck mPc 2 en gigaélectronvolt ( 1 GeV ≈ 1, 6 × 10−10 J ).

2 Cinématique du point mobile. Vitesse de rotation d’un repère La cinématique est l’étude des mouvements des corps indépendamment des causes qui les produisent. Elle s’appuie uniquement sur les notions d’espace et de temps que tout observateur possède intuitivement. Pour étudier le mouvement des corps, l’observateur doit rendre ces notions objectives en utilisant d’autres corps pris comme repères d’espace et de temps. Il convient d’abord de préciser le cadre spatiotemporel de la cinématique dite galiléenne ou newtonienne, en hommage aux apports décisifs du physicien italien G. Galilée en 1638 et du physicien anglais I. Newton en 1687. Bien plus tard, en 1905, A. Einstein montra le caractère approché de cette cinématique (cf. Relativité et invariance). Cependant, pour la presque totalité des problèmes de mécanique qui se posent en physique macroscopique, la cinématique newtonienne constitue une excellente approximation qu’il convient de développer.

I . — CADRE SPATIOTEMPOREL DE LA CINÉMATIQUE GALILÉENNE I . 1 . — Notion d’événement Les phénomènes physiques peuvent être considérés comme des ensembles d’événements, c’està-dire de phénomènes élémentaires que l’on caractérise en associant, à l’espace ambiant et au temps, respectivement, deux espaces métriques, l’un à 3 dimensions et l’autre à 1 dimension. Le physicien rend objectif ce double repérage en introduisant la mesure de ces grandeurs, ce qui revient à choisir une origine et une norme dans chacun des deux espaces précédents. I . 2 . — Mesure du temps La mesure du temps suppose implicitement une orientation, du passé vers le futur ; cette orientation s’appuie sur l’irréversibilité de l’évolution de tous les phénomènes physiques réels, c’est-à-dire sur le deuxième principe de la thermodynamique (cf. Thermodynamique). Pour être complet, la mesure exige le choix d’une origine ; celle-ci sera prise conventionnellement à un instant donné de l’évolution du phénomène étudié. Il est naturel d’adopter comme instant initial l’instant pour lequel l’état du système est connu ; les instants ultérieurs correspondent alors à l’évolution du système vers le futur où le mouvement est encore inconnu.

Cinématique du point mobile. Vitesse de rotation d’un repère

23

On mesure le temps à l’aide d’horloges qui sont des systèmes physiques dont on se donne la loi d’évolution en fonction du temps. On définit ainsi une échelle de temps qui dépend de la nature de la loi adoptée ; aussi existe-t-il plusieurs échelles de temps. a) L’échelle de temps universel (T.U.) Admettant l’uniformité de la vitesse de rotation de la Terre autour de son axe des pôles, l’angle de rotation permet de mesurer le temps. Le temps ainsi défini est appelé le temps universel (T.U.). On tient compte des irrégularités de cette rotation en apportant de légères corrections, ce qui donne T.U1. b) L’échelle de temps des éphémérides (T.E.) Le paramètre d’évolution est ici la position du centre de la Terre sur son orbite autour du Soleil. Le temps ainsi défini est le temps des éphémérides (T.E.). c) L’échelle de temps atomique internationale (T.A.I.) Là, on compte le nombre de périodes d’un système physique qui oscille tel un pendule mécanique, un circuit électrique, un système quantique à deux états. Dans ce dernier cas, on peut mesurer des durées très courtes avec une précision remarquable. Le temps est alors appelé temps atomique international (T.A.I). L’unité de durée, dans le système international (SI), est la seconde, c’est-à-dire la durée de 9 192 631 770 périodes de radiation correspondant à la transition entre les deux niveaux hyperfins de l’état fondamental de l’atome de césium 133 (cf. Quantique). d) L’échelle de temps géologique Pour la mesure de durées très longues, on utilise la loi de désintégration de corps radioactifs, comme le carbone 14, isotope du carbone naturel 12. e) L’échelle de temps légal En raison, notamment, des besoins de la navigation maritime, l’échelle de temps légal en France est un compromis entre l’échelle de temps universel (T.U.) et l’échelle de temps atomique (T.A.I.). On l’appelle échelle de temps universel coordonné (T.U.C.).

 c Dunod – Toute reproduction non autorisée est un délit

Les horloges publiques diffusent, en France, T.U.C. + 1 h en hiver et T.U.C + 2 h en été. Remarque : Il convient de ne pas confondre l’instant initial, choisi conventionnellement pour une expérience déterminée de cinématique, et un hypothétique instant initial de l’Univers. Le premier a un sens physique précis puisque des événements réels se produisent dans le présent, alors que le second est de nature purement spéculative. Retenons, après l’analyse du temps et de sa mesure faite par Einstein dans la théorie de la relativité (cf. Relativité et invariance) que le temps est un concept physique qui n’a de sens que relativement à des phénomènes physiques. I . 3 . — Repère d’espace On appelle repère d’espace R un ensemble de points dont les distances mutuelles sont invariables au cours du temps ; un tel ensemble est aussi appelé solide de référence. Le repérage spatial de la position à chaque instant d’un point quelconque A de l’espace exige d’abord que l’on se donne conventionnellement une origine O à partir de laquelle les longueurs sont comptées. Dans l’espace physique à trois dimensions, cette position est déterminée par le vecteur OA.

24

2. Cinématique du point mobile. Vitesse de rotation d’un repère

Il est souvent commode d’expliciter ce vecteur dans une base généralement orthonormée, formée de trois vecteurs unitaires ex , ey , ez fixes par rapport à R. On écrira par exemple : OA = x ex + y e y + z ez

x, y, z étant les composantes cartésiennes du vecteur OA ou les coordonnées du point A dans la base ex , ey, ez . Notons que l’on peut associer à un repère d’espace plusieurs bases différentes mais fixes les unes par rapport aux autres. Si la base est directe le repère est direct, si elle est inverse le repère est inverse. Rappelons que cette orientation de l’espace est liée à l’existence de deux catégories de grandeurs physiques : celles qui dépendent de l’orientation de l’espace et celles qui n’en dépendent pas. Dans ce contexte, l’opération produit vectoriel de deux vecteurs exige pour sa définition une orientation préalable de l’espace, laquelle s’appuie sur la règle physique du bonhomme d’Ampère ou du tire-bouchon de Maxwell (cf. chapitre 1). En général, on munit le repère d’une base directe orthonormée. Dans la suite, on écrira brièvement R = Oxyz pour désigner le repère R, muni de son origine O et de la base directe formée des trois vecteurs unitaires e x , ey et ez fixes par rapport R.

L’unité SI de longueur est le mètre, c’est-à-dire, depuis la conférence internationale des poids et mesures de 1983, la distance parcourue par la lumière dans le vide en une durée égale à 1/c seconde, la vitesse c ayant la valeur exacte suivante : c = 2, 997 924 58 × 10 8 m . s −1 I . 4 . — Référentiel On peut associer à un repère d’espace R un temps unique en synchronisant les horloges de ce repère. La synchronisation se fait en deux étapes (Fig. 2.1) : z H2 H1 O x

A1

A2 y

F IG . 2.1.

i) Contrôle de la même marche On considère deux horloges H 1 et H 2 situées respectivement aux points fixes A 1 et A 2. À l’instant t1 de H 1 , A1 envoie un signal vers A 2 qui le reçoit au temps t 2 de H2 . À l’instant t1 + T de H1, A 1 envoie un nouveau signal que A 2 doit recevoir au temps t2 + T. ii) Contrôle de la même origine À l’instant t1 de H1 , A1 envoie un signal vers A 2 qui le réfléchit vers A 1. Ce dernier le reçoit au temps t1 + T. L’origine des deux horloges est la même si A2 a reçu le signal au temps t 1 + T/2. Grâce à ce signal, le plus souvent électromagnétique en raison de sa grande vitesse de propagation dans le vide, on peut synchroniser toutes les horloges d’un repère d’espace. Ce temps unique, donné par toutes ces horloges, est le temps du repère. On définit alors la simultanéité :

25

Cinématique du point mobile. Vitesse de rotation d’un repère

Deux événements sont simultanés si les horloges, préalablement synchronisées, placées aux points où se produisent ces phénomènes, donnent la même indication. L’ensemble d’un repère d’espace et d’un repère de temps forme un référentiel, c’est-à-dire une référence spatiale et une référence temporelle, toutes deux indispensables dans l’étude de tout mouvement. I . 5 . — Durée et longueur de Planck Concernant le temps et l’espace, une question se pose : existe-t-il une durée et une longueur minimales en dessous desquelles toute durée ou toute longueur n’auraient pas de signification physique. On admet actuellement que la réponse est oui, et que ces valeurs minimales sont celles introduites par M. Planck en 1914. Ces grandeurs sont construites à partir des trois constantes fondamentales de la physique, la constante de gravitation G , la constante d’Einstein c ou vitesse limite et la constante de Planck h , précisément  = h/(2p) (prononcer h barre). a) Analyse dimensionnelle Pour déterminer ces constantes, on s’appuie sur une analyse dimensionnelle, qui est une méthode qualitative d’étude permettant d’identifier les paramètres pertinents d’un phénomène physique et d’en déduire la dépendance d’une grandeur en fonction de ces paramètres. Les facteurs numériques, sans dimension par définition, restent évidemment inaccessibles, leur détermination relevant de la seule analyse quantitative. Montrons, à l’aide de l’analyse dimensionnelle, que la période des petites oscillations d’un pendule simple est proportionnelle à (l/g)1/2 , l étant la longueur du pendule et g l’intensité du champ de pesanteur terrestre. A priori, l et g constituent, avec la masse m de la bille, les paramètres pertinents du système. La période T peut donc s’écrire dimensionnellement : [T] = [L] a[g]b[M]g

avec [g] = [L][T ] −2

puisque g est une accélération, celle due à la pesanteur. Il en résulte :

 c Dunod – Toute reproduction non autorisée est un délit

[T] = [L] a+b [T ]−2b [M]g

soit

g=0

b = −a = −

1 2

On en déduit T = Cte × (l/g)1/2 . On retrouve ainsi, par la seule voie de l’analyse dimensionnelle, l’absence de la masse dans l’expression de la période d’un tel pendule, absence que l’on doit attribuer physiquement à l’égalité de la masse grave et de la masse inerte (cf. chapitre 7). L’étude quantitative approfondie montre que cette constante vaut 2p (cf. chapitres 5 et 10). b) Durée de Planck Cherchons à déterminer une durée fondamentale t P à partir de de c , G et  . Pour cela, écrivons a priori cette durée sous la forme suivante : [t P] = [G] a [c] b []g Or les quantités G , c et  s’expriment aisément en fonction d’une durée [T ] , d’une longueur [L] et d’une masse [M ] ou d’une combinaison de ces dernières comme l’énergie [E] = [M][L] 2[T ] −2 (cf. constantes fondamentales) : [G] = [L] 3[M] −1 [T ]−2

[c] = [L][T ]−1

et [] = [E][T] = [M][L] 2[T ]−1

26

2. Cinématique du point mobile. Vitesse de rotation d’un repère

Il en résulte : [tP ] = [L]3a [M]−a [T ] −2a [L]b [T ]−b [M]g [L]2g [T ]−g = [L] 3a+b+2g[M]−a+g [T ]−2a−b−g Il en résulte en identifiant le système d’équations suivantes : 3a + b + 2g = 0

− a + g = 0 et

− 2a − b − g = 1

On en déduit aisément a = g = 1/2 et b = −5/2 , d’où : G c5

1/2

= 0, 54 × 10 −43 s

c) Longueur de Planck On l’obtient rapidement à partir de la précédente en multipliant t P par c : lP = ct P =

G c3

1/2

= 1, 61 × 10−35 m

II . — CINÉMATIQUE DU POINT MOBILE II . 1 . — Vitesse d’un point mobile par rapport à un référentiel R a) Définition Notant OA le vecteur position d’un point A compté à partir de l’origine O d’un référentiel R, la vitesse de A par rapport à R est le vecteur suivant, noté vA/R ou plus brièvement v : v A/R =

d OA dt

R

Précisons que l’expression par rapport à R signifie pour un observateur lié au référentiel R, pour lequel les vecteurs de base de R, ex , e y et e z sont fixes et donc indépendants du temps. b) Composantes cartésiennes de vA/R Comme OA = x e x + y e y + z e z , il vient (Fig. 2.2) : vA/R =

dx dy dz ex + ey + ez dt dt dt

soit

vA/R = ˙x ex + y˙ e y + z˙ ez

en utilisant la notation pointée, internationalement admise depuis son introduction par I. Newton en 1671, pour désigner la dérivée par rapport au temps d’une fonction, ici x(t), y(t) ou z(t). Indiquons que la notation d x/ d t a été introduite par G. Leibniz en 1682, et que celle x  , attribuée à J. Lagrange, date de 1756.

27

Cinématique du point mobile. Vitesse de rotation d’un repère z H

z H

A

O

ϕ

O

y P

x

A

ϕ

x

F IG . 2.2.

y P ρ

F IG . 2.3.

c) Composantes cylindriques de vA/R Il est souvent commode d’exprimer la vitesse précédente v A/R à l’aide des coordonnées cylindriques r, w, et z (Fig. 2.3). Comme OA = r er + z e z, il vient : vA/R = r˙ er + r

d er + ˙z ez avec dt

d er dt

= R

d er dw

w˙ = w˙ e w R

ew étant le vecteur unitaire perpendiculaire à er et parallèle au plan Oxy. Il en résulte que : vA/R = r˙ er + rw˙ e w + ˙z e z Soulignons que nous avons ainsi exprimé, en b et c, le même vecteur dans deux bases différentes. En b, la base utilisée était celle du référentiel R, en c, c’est la base cylindrique. On distinguera donc soigneusement le référentiel R, qui est une donnée fondamentale pour définir la vitesse puisqu’il représente l’observateur, de la base, parfois mobile, qui est une donnée technique et donc secondaire mais nécessaire à l’exploitation d’une égalité vectorielle. Dans la suite, on sera conduit à écrire un même vecteur dans deux bases différentes. Un indice rappellera la base utilisée. Par exemple, on écrira en colonne les composantes cartésiennes et cylindriques du vecteur vA/R ; l’indice R indique qu’il s’agit d’une base invariablement liée au repère R, et l’indice Rc que la base utilisée est cylindrique :

 c Dunod – Toute reproduction non autorisée est un délit

x˙ y˙ R z˙

et Rc

r˙ rw˙ z˙

Remarque : Certains auteurs distinguent, dans l’écriture des colonnes, la base B = (ex , ey , ez) du repère R. Cette distinction systématique nous a paru superflue. d) Composantes de Frenet ou composantes intrinsèques de la vitesse Les composantes de Frenet (du nom du mathématicien français J.F. Frenet) sont relatives à la base définie localement (en un point de la trajectoire C) par les trois vecteurs unitaires suivants : et , en et eb = et × en. Le vecteur et est tangent à la trajectoire alors que en est, lui, normal à la trajectoire conformément à la relation : d et e = n ds R dans laquelle s représente l’abscisse curviligne sur C orienté et R le rayon de courbure locale de la trajectoire. Le plan Aet en est appelé plan osculateur. Ces composantes sont généralement les plus

28

2. Cinématique du point mobile. Vitesse de rotation d’un repère

adaptées à la résolution des problèmes qui se posent en mécanique du point matériel lorsqu’on connaît la trajectoire (Fig. 2.4). et

eb

z

A A0

C

en

A "

C

C

s

O

C0

y

I A

R0 "

I J

C

x F IG . 2.4.

F IG . 2.5.

D’après la définition de v A/R , on peut écrire : vA/R =

d OA dt

= R

d OA ds

ds R dt

La signification du vecteur (d OA/ d s)R s’obtient aisément : A étant un point tendant vers A, ce vecteur est la limite du vecteur AA /AA , dont la norme tend vers l’unité et dont le support tend vers la tangente à C. Il s’identifie donc à et . Quant à d s/ d t, appelé vitesse scalaire v, c’est au signe près, la norme de la vitesse suivant que s est orienté dans le sens du mouvement ou non : d s/ d t = AA / d t = v. D’où : vA/R = v e t Pour préciser la signification géométrique du rayon de courbure R, considérons le point A voisin de A sur la trajectoire. Ces deux points appartiennent à un même cercle C 0 . Évaluons, à l’aide de la figure 2.5, le vecteur De t : De t = A I − AI = AJ − AI = IJ

avec

IJ = 2 sin

Ds ε ≈ε= 2 R0

Lorsque A tend vers A, Det /ε tend vers le vecteur unitaire e n orienté vers le centre C. On peut donc écrire : d et d et d et en R 0 = en soit = = ds ds R0 ε En identifiant avec la relation de définition du rayon de courbure, on en déduit que R est le rayon R0 du cercle intérieurement tangent à la trajectoire ; 1/R est sa courbure. II . 2 . — Accélération d’un point mobile par rapport à un référentiel R a) Définition L’accélération d’un point A par rapport à un référentiel R est le vecteur suivant, noté a A/R ou plus brièvement a : aA/R =

d v A/R dt

= R

d2 OA d t2

R

29

Cinématique du point mobile. Vitesse de rotation d’un repère b) Composantes cartésiennes de l’accélération aA/R

On obtient immédiatement les composantes cartésiennes de a A/R à partir de celles de la vitesse puisque les vecteurs de base sont fixes dans R : aA/R = ¨ x ex + ¨ y ey + ¨ z ez

noté aussi R

¨ x y ¨ z ¨

c) Composantes cylindriques de l’accélération aA/R En dérivant l’expression de la vitesse en coordonnées cylindriques, on trouve : d er d ew + ¨z ez aA/R = r ¨ er + r˙ + r˙ w˙ ew + rw ¨ ew + rw˙ dt R dt R Comme : d er d er d ew d ew = w˙ = w˙ = w˙ ew et = −w˙ er dt R dw R dt R dw R l’expression de aA/R en coordonnées cylindriques est la suivante : 2

aA/R = (¨ w + 2r˙ w) ˙ e w + ¨z ez r − rw˙ ) e r + (r¨

soit Rc

r ¨ − rw˙ 2 r¨ w + 2r˙ w˙ = (1/r) d(r2w)/ ˙ dt ¨ z

Les deux premières composantes sont qualifiées, respectivement, de radiale et d’orthoradiale. d) Composantes de Frenet ou composantes intrinsèques de l’accélération On déduit de v A/R exprimé dans la base de Frenet : d(v et ) dt

aA/R =

= R

de dv et + v t dt dt

avec

d et de ds e = t = nv dt ds dt R

Donc :

 c Dunod – Toute reproduction non autorisée est un délit

aA/R = at + a n avec

at =

dv et dt

et a n =

v2 en R

Ainsi, l’accélération présente deux termes : le premier est l’accélération tangentielle, le second est l’accélération normale. Cette écriture permet de distinguer deux grandes catégories de mouvements d’un point : i) les mouvements rectilignes, pour lesquels le rayon de courbure R est infini, ii) les mouvements uniformes pour lesquels la norme de la vitesse est une constante (d v/ d t = 0). Si R est infini et v constant, le mouvement est rectiligne et uniforme. II . 3 . — Exemples a) Mouvement hélicoïdal Le mouvement hélicoïdal est celui d’un point A dont la projection dans le plan Oxy décrit un cercle. C’est le mouvement en hélice décrit par un point d’un objet que l’on visse ; c’est aussi le mouvement d’une particule chargée dans un champ magnétique (cf. chapitre 8). En coordonnées cartésiennes, les équations paramétriques caractéristiques de la trajectoire sont, v et h étant des constantes appelées respectivement la pulsation et le pas de l’hélice (Fig. 2.6a) : OA = r sin(vt) e x + r[1 − cos(vt)] ey + hvt ez

30

2. Cinématique du point mobile. Vitesse de rotation d’un repère z

y 2r

h C

A C

O

!t y

' = !t=2 x

Oz

P

x

P

a)

b) F IG . 2.6.

La trajectoire de la projection P de A dans le plan Oxy est bien un cercle, de rayon r, puisque : x r

2

+

2

y−r r

=1

Le rayon du cercle est r et son centre se trouve sur l’axe Oy à la distance r (Fig. 2.6b). En dérivant, on en déduit les composantes cartésiennes de la vitesse et de l’accélération, respectivement : −rv2 sin(vt) rv2 cos(vt) 0 R

rv cos(vt) rv sin(vt) hv R

Les composantes cylindriques s’obtiennent aisément. En effet : r = (x2 + y 2)1/2 = 2r sin

vt 2

w = arctan

y vt = x 2

z = hvt

On a respectivement pour vA/R et a A/R : r ¨ − rw˙ 2 = −rv 2sin (vt/2)

r˙ = rv cos (vt/2)

r¨ w + 2r˙ w˙ = rv 2 cos (vt/2)

rw˙ = rv sin (vt/2) Rc

z˙ = hv

¨z = 0

Rc

Quant aux composantes de Frenet, on les détermine en calculant les normes de la vitesse et de l’accélération : v = v(r2 + h2) 1/2 et a = rv 2 d’où : at =

dv et = 0 dt

an = rv 2 en

et

R=

v2 =r a

1+

h2 r2

La courbe n’étant pas contenue dans un plan, le vecteur binormal eb = e t × e n n’est pas constant puisque sa direction change. Aussi définit-on la torsion T de la courbe par l’équation : d eb e =− n ds T Le calcul de T ne présente pas de difficulté particulière (cf. Exercices).

31

Cinématique du point mobile. Vitesse de rotation d’un repère b) Mouvement cycloïdal

Considérons le mouvement d’une particule A caractérisé par les équations cartésiennes paramétriques suivantes, v étant une constante : OA = r[vt − sin(vt)] e x + r[1 − cos(vt)] ey

On montre que c’est le mouvement d’un point situé sur la roue d’un vélo qui avance en roulant sans glisser sur la chaussée (cf. Exercice P3.1) y 2r

x

O

πr

2πr

F IG . 2.7.

La trajectoire est périodique puisque y est inchangé lorsque vt varie de 2p, x variant de 2pr. Sur le tableau 2.1, on a rassemblé quelques valeurs caractéristiques de x et y. vt

0

p/2

p

3p/2

2p

x

0

pr

r(p/2 + 1)

2pr

y

0

r(p/2 − 1)

2r

r

0

r

TAB . 2.1.

 c Dunod – Toute reproduction non autorisée est un délit

Les composantes cartésiennes de v et a permettent de préciser la nature de la trajectoire dans le plan Oxy (Fig. 2.7) : rv 2 sin(vt) rv[1 − cos(vt)] rv 2 cos(vt) rv sin(vt) 0 0 R R Notons qu’aux points de rebroussement (vt = 0, 2p, etc.), la vitesse s’annule contrairement à l’accélération dont la valeur se réduit à celle de la composante tangentielle. La pente de la cycloïde aux points de rebroussement est ±∞ ; en effet : dy sin(vt) 2 sin(vt/2) cos(vt/2) 1 y˙ = = = = 2 dx 1 − cos(vt) tan(vt/2) x˙ 2 sin (vt/2) soit, pour vt = 0 et vt = 2p respectivement : dy dx

0

= ∞ et

dy dx

2p

= −∞

Exprimons les composantes de v et a dans la base de Frenet. Comme v = 2rv sin(vt/2) : vt vt vA/R = 2rv sin e t et a t = rv 2 cos et 2 2 On obtient alors aisément an et le rayon de courbure R : a n = (a2 − a2t )1/2 en = rv 2 sin

vt v2 = 4r sin e n et R = 2 an

vt 2

32

2. Cinématique du point mobile. Vitesse de rotation d’un repère

III . — DIFFÉRENTS MOUVEMENTS D’UN REPÈRE Rappelons que les repères d’espace associés aux référentiels R et R sont des ensembles de points dont les distances mutuelles sont invariables. On dit aussi qu’ils forment des solides parfaits de référence. III . 1 . — Mouvements particuliers d’un repère R par rapport à un référentiel R a) Translation du repère R par rapport au référentiel R Le repère d’espace R  a un mouvement de translation par rapport à un référentiel R si, au cours du mouvement, A et B étant deux points quelconques fixes de R , le vecteur AB est constant par rapport à R au cours du mouvement (Fig. 2.8) : d AB dt

=0

d’où

vA/R = vB/R

R

Il en est évidemment ainsi pour les vecteurs de la base orthonormée directe (e x , ey  , ez ) : d ex  dt

d ey dt

=0 R

d ez dt

=0 R

=0 R

Tous les points de R ont donc même vitesse et même accélération par rapport à R ; notons que, cette même vitesse et cette même accélération peuvent être quelconques. z0

z z R t1

H

R t2

R O

Ω O

y

A

vA=R y0 y

R0

ϕ

P R F IG . 2.9.

x

x F IG . 2.8.

x0

b) Rotation du repère R autour d’un axe du référentiel R Si le repère R  tourne autour d’un axe du référentiel R, par exemple Oz, un point A, fixe dans R, décrit, par rapport à R, un cercle de centre H et de rayon HA, H étant la projection de A sur l’axe de rotation (Fig. 2.9). Sa vitesse : vA/R =

ds e t = HA dt

dw e t = HA w˙ et dt

est donc portée par le vecteur unitaire et de la tangente à ce cercle. Notons que les points situés sur l’axe Oz, commun à R  et R, ont une vitesse nulle. Les points O et A appartenant au même repère R , la vitesse du point A par rapport à R peut s’écrire aussi : HA 0 0 0 0 HA w˙ = × vA/R = V × OA puisque ˙ R 0 R w R  OH

33

Cinématique du point mobile. Vitesse de rotation d’un repère

La grandeur V se présente comme un vecteur porté par l’axe de rotation Oz ; sa mesure algébrique est égale à w, ˙ si l’orientation du plan et celle de la normale au plan sont reliées par la règle du bonhomme d’Ampère ou du tire-bouchon de Maxwell : V = w˙ e z. i) Si w˙ > 0, la rotation a lieu dans le sens direct Ox → Oy.

ii) Si w˙ < 0, la rotation a lieu dans le sens inverse Oy → Ox.

Ainsi, la distribution des vitesses de R , à la date t, est complètement déterminée par le vecteur V appelé vitesse angulaire ou de rotation du repère R par rapport au référentiel R : vA/R = V × OA

avec

V = w˙ e z

On écrira souvent, pour éviter toute ambiguïté, V R /R de façon explicite au lieu de V. c) Mouvement hélicoïdal Considérons le mouvement du repère R  par rapport au référentiel R, tel que les axes Oz et O  z  coïncident et qu’un point A de R , en dehors de O  z , décrive une hélice. On sait que la trajectoire de la projection P de A dans un plan perpendiculaire à Oz est un cercle. Comme OA = OH + OP, H étant la projection de A sur Oz, on a : vA/R = vH/R + vP/R

avec

vH/R = v H/R ez

et v P/R = V × OP

Par conséquent, puisque OP = HA : v A/R = v H/R + V × HA La distribution des vitesses des points du repère R, dans un tel mouvement, est donc caractérisée par la relation vectorielle : v A/R = v H/R + V × HA

 c Dunod – Toute reproduction non autorisée est un délit

Ce mouvement est qualifié d’hélicoïdal car les points situés en dehors de l’axe décrivent des hélices. Pour un autre point B, on a une relation analogue : vB/R = vK/R + V × KB K étant la projection de B suivant l’axe Oz. On en déduit, en retranchant les deux équations précédentes, puisque H et K ont même vitesse : vA/R = v B/R + V × (HA − KB)

Comme HA − KB = HA − KH − HB = BA − KH et que les vecteurs KH et V sont colinéaires, la relation précédente s’écrit : vA/R = v B/R + V × BA On retrouve le cas de la translation en faisant V = 0. Remarque : Le plus souvent, on réserve le nom de mouvement hélicoïdal au cas particulier où les vecteurs OH et V sont proportionnels.

34

2. Cinématique du point mobile. Vitesse de rotation d’un repère

III . 2 . — Mouvement le plus général du repère R  par rapport au référentiel R a) Champ des vitesses du repère R Les mouvements, par rapport au référentiel R , de deux points quelconques A et B, appartenant au repère R , sont tels que le carré de la distance AB qui sépare ces deux points est une constante (Fig. 2.10a). On a donc AB · AB = Cte, soit, en dérivant par rapport au temps : 2AB ·

d AB = 2AB · dt

d OB dt

R



d OA dt

R

= 2AB · vB/R − v A/R = 0

Ainsi (Fig. 2.10b) : AB · v A/R = AB · v B/R On montre que cette propriété d’équiprojectivité des vitesses sur AB implique la relation suivante entre les vitesses des points A et B (cf. chapitre 1) : vA/R = vB/R + AB × V ou

vA/R = v B/R + V × BA

Contentons-nous de rappeler la réciproque en multipliant scalairement les deux membres de cette dernière équation par AB : v A/R · AB = v B/R · AB + (AB × V) · AB = vB/R · AB

puisque (AB × V) · AB = 0.

Ainsi, d’après l’équation vectorielle, qui donne le champ des vitesses de R  par rapport à R, le mouvement le plus général de R  par rapport à R peut être considéré, à chaque instant, comme un mouvement hélicoïdal dont le vecteur vitesse de rotation est V. z A

z

R O

R

x

O

y B

vB/R

vA/R

y

A

x a)

B

H

K

b) F IG . 2.10.

b) Degrés de liberté du mouvement de R On appelle degrés de liberté du mouvement du repère R  par rapport au référentiel R le nombre de paramètres indépendants qu’il faut se donner pour connaître, de manière non ambiguë, la position de R par rapport à R . Pour déterminer cette position il suffit de se donner trois points non alignés, A, B, C de R : d’abord trois paramètres pour la position de A, par exemple x A , yA, zA ; ensuite la position de B est déterminée par deux nouvelles coordonnées, puisque B évolue sur la surface de centre A et de rayon AB ; enfin la position de C est fixée par une dernière coordonnée, car ce point se déplace sur la courbe intersection de deux sphères, l’une de centre A et de rayon AC, l’autre de centre B et de rayon BC. On a donc besoin, en tout, de six paramètres pour situer R par rapport à R.

35

Cinématique du point mobile. Vitesse de rotation d’un repère III . 3 . — Caractère axial du vecteur vitesse de rotation

On aura observé que la définition de V s’appuie sur celle du produit vectoriel, laquelle exige la convention du bonhomme d’Ampère. La vitesse de rotation est donc un vecteur axial (cf. chapitre 1). Sur le plan de la représentation mathématique, cette quantité, que l’on peut caractériser par trois composantes, comme les vecteurs, mais reliées par une convention supplémentaire, est un être mathématique plus élaboré ; on montre que c’est un tenseur antisymétrique d’ordre deux [M ] qui s’exprime dans la base orthonormée de R par une matrice carrée antisymétrique à neuf éléments de la forme : m12 0 −m 23

0 −m12 −m13

m 13 m 23 0

En explicitant dans la base de R l’égalité suivante, [M ]OA = V × O A, dans laquelle O  est l’origine de R et A l’un de ses points, de coordonnées (x , y, z  ), on peut exprimer les composantes du vecteur vitesse de rotation en fonction des élements de la matrice représentant le tenseur antisymétrique. En effet, on a :

R

ce qui donne, en effectuant :

0 −m12 −m13

R

On en déduit, en identifiant :

m12 0 −m23

m 13 m23 0

R

x vx   y = vy  ×  z R vz 

R

x y z

vy z − v z y m12 y + m13 z     vz x  − v x z −m12 x + m23 z =   −m13 x + m23 y R  vx y − v y x V = −m 23 ex + m13 ey − m12 ez

 c Dunod – Toute reproduction non autorisée est un délit

CONCLUSION Retenons les résultats essentiels de la cinématique qui permet de décrire le mouvement de points par rapport à un référentiel R, lequel est un repère d’espace et de temps associé à l’observateur. (1) On introduit naturellement pour un point A, en mouvement par rapport à R, les concepts vectoriels de vitesse et d’accélération que l’on peut exprimer souvent par leurs composantes dans des bases variées : d OA vA/R = = v e t de composantes cartésiennes x˙ y˙ ˙z dt R aA/R =

d2 OA d t2

= R

dv v2 e t + en R dt

de composantes cartésiennes

x ¨ y ¨z ¨

Soulignons la distinction entre le référentiel R, par rapport auquel on étudie les mouvements, par l’intermédiaire des vecteurs vitesse et accélération, et toute base de projection dans laquelle il est commode d’exprimer les relations entre les différents vecteurs. Dans ce contexte, on retiendra aussi les expressions de la vitesse et de l’accélération d’un point en coordonnées cylindriques, respectivement :

Rc

r˙ rw˙ z˙

et Rc

r ¨ − rw˙ 2 r¨ w + 2r˙ w˙ = (1/r) d(r 2w)/ ˙ dt ¨z

36

2. Cinématique du point mobile. Vitesse de rotation d’un repère

(2) Le mouvement de rotation d’un repère d’espace R  par rapport au référentiel R peut être caractérisé par un vecteur VR /R , appelé vecteur vitesse de rotation de R  par rapport à R. Il est nécessaire pour cela que toutes les bases orthonormées choisies soient directes. (3) Le mouvement le plus général de R  par rapport à R est un mouvement hélicoïdal ; on l’exprime à l’aide de la relation suivante entre les vitesses de deux points quelconques A et B de R par rapport à R : vA/R = v B/R + VR/R × BA VR  /R étant la vitesse de rotation de R par rapport à R. On écrit souvent la relation précédente sous la forme réduite : d AB = V × AB dt R Pour un mouvement de translation de R  par rapport à R , V = 0 .

EXERCICES ET PROBLÈMES P2– 1. Mouvement hélicoïdal Un point A se déplace, relativement à un référentiel R = Oxyz, le long d’une courbe d’équations paramétriques : x = 0, 3 cos(vt) y = 0, 3 sin(vt) z = 0, 1 vt où v = 2p rad . s −1. L’unité de longueur est le cm. 1. Exprimer, dans la base de R, les composantes de sa vitesse et de son accélération. En déduire la norme de ces vecteurs. 2. Trouver la vitesse et l’accélération en coordonnées intrinsèques. En déduire le rayon de courbure de la trajectoire. 3. Calculer la torsion T de la trajectoire. P2– 2. Mouvement dans le champ de pesanteur uniforme On considère, dans un plan Oxy, les trajectoires des points ayant une accélération constante a = −g ey et une vitesse initiale v0 faisant l’angle a avec l’axe Ox (Fig. 2.11).

1. Montrer qu’il existe deux valeurs de a pour lesquelles ces trajectoires, issues de l’origine O, atteignent une même cible C. 2. En déduire l’équation de la courbe enveloppe de toutes les trajectoires correspondant à une même valeur de la norme de v0. y Came

y g

H r v0

O

α

A w

C

Oz x

F IG . 2.11.

F IG . 2.12.

x

Cinématique du point mobile. Vitesse de rotation d’un repère

37

P2– 3. Déplacement d’un point matériel le long d’une came Un point matériel A est astreint à se déplacer, dans le plan Oxy d’un référentiel R = Oxyz, le long du pourtour d’une came, reliant r et w fixe dans R (Fig. 2.12). L’équation polaire de la came est r = b − c cos w. La tige, qui permet le mouvement et réalise à l’aide d’un ressort le contact sur la came, tourne uniformément autour de l’axe Oz, avec une vitesse angulaire V = w. ˙ 1. Exprimer, en coordonnées polaires (r, w), la vitesse v et l’accélération a de A par rapport au référentiel R.

2. Calculer v et a pour V = 30 tr . min −1 , lorsque A atteint le point H de la came (w = p/2), sachant que b = 2 cm et c = 1 cm. P2– 4. Mouvement en spirale Un point A décrit une courbe plane d’équations polaires paramétriques : r = b exp(−t/t) et w = vt, les quantités b, t et v étant des constantes. 1. Calculer les composantes radiale et orthoradiale de la vitesse et de l’accélération. En déduire les normes de ces vecteurs ainsi que l’angle que fait la vitesse avec le rayon vecteur. 2. Trouver les composantes intrinsèques de l’accélération. 3. Quel est le rayon de courbure R de la trajectoire ? P2– 5. Mouvement sinusoïdal Un point en mouvement suivant un axe Ox possède une accélération proportionnelle à sa distance au point O. Trouver son mouvement dans les deux cas suivants : 1. le coefficient de proportionnalité est négatif et s’écrit −v 20. On suppose qu’initialement la particule est en O , avec une vitesse v0 = v 0 ex . 2. Même question si le coefficient de proportionnalité est positif et s’écrit a 2 .

 c Dunod – Toute reproduction non autorisée est un délit

P2– 6. Combinaison de deux mouvements sinusoïdaux Un point évolue dans un plan de telle sorte que ses mouvements suivant deux axes rectangulaires soient sinusoïdaux : x = a cos(vt) et y = b cos(vt + f). Trouver l’équation de la trajectoire et déterminer ses caractéristiques. Étudier les cas où f = 0, f = p. À quelle condition la trajectoire est-elle un cercle ? Citer un exemple physique d’un tel mouvement. P2– 7. Mouvement à accélération de la forme V × v

Un point a une accélération a reliée à la vitesse par la relation : a = V × v, V étant un vecteur constant. 1. Que peut-on dire de la norme de la vitesse et de sa composante suivant le vecteur V ? 2. Montrer que, si initialement v est perpendiculaire à V, la trajectoire est un cercle dont on déterminera le rayon R.

38

2. Cinématique du point mobile. Vitesse de rotation d’un repère

P2– 8. La passe en-avant au rugby Au rugby, les arbitres semblent juger la passe en-avant à partir du mouvement des mains des joueurs, alors que les spectateurs considèrent, eux, le mouvement du ballon par rapport au terrain de jeu. Les discussions passionnantes qui en résultent incitent à analyser cinématiquement le problème. Pour cela, considérons deux joueurs partenaires A et B (Fig. 2.13), situés sur une ligne parallèle à la ligne d’en-but, et se déplaçant avec la même vitesse v e par rapport au référentiel R du terrain. Rappelons que ce dernier est délimité par les lignes de touches et les lignes d’en-but. Supposons qu’à un instant, A fasse une passe vers B en communiquant au ballon une vitesse v  parallèlement à la ligne d’en-but, dans le référentiel R que leurs centres de masses définissent, en translation par rapport à R . Ligne d’en-but

ve

ve

A v B

F IG . 2.13.

1. Quelle est la vitesse v du centre C du ballon par rapport à R , dans la situation de la passe précédente de A vers B ? Calculer l’angle a que fait v avec la ligne d’en-but pour v e = 7 m.s−1 et v  = 14 m.s −1 . 2. Sachant que l’on néglige toute modification de la vitesse de C avant réception du ballon par B , situé à 10 m de son coéquipier, calculer la longueur l parcourue par B entre l’envoi du ballon et sa réception. Conclure et commenter.

3 Changement de référentiel

Jusqu’à maintenant nous avons décrit le mouvement d’un mobile ponctuel A par rapport à un référentiel R en introduisant sa vitesse et son accélération par rapport à ce référentiel.

La question à laquelle nous souhaitons répondre maintenant est d’une autre nature : R et R  étant deux référentiels en mouvement quelconque l’un par rapport à l’autre, quelles relations existe-t-il entre les caractéristiques cinématiques, vitesse et accélération, d’un même mobile A, relatives à R et R  (Fig. 3.1a) ?

 c Dunod – Toute reproduction non autorisée est un délit

Contrairement à un usage malheureusement persistant, nous n’emploierons pas les qualificatifs « absolu »et « relatif »généralement attribués respectivement à R et R  . En effet, d’une part, toutes les vitesses et toutes les accélérations sont des grandeurs relatives à un référentiel, et d’autre part, le qualificatif « absolu » rappelle l’erreur historique de Newton, celle d’avoir introduit le concept inutile et infondé d’espace absolu (cf. chapitre 4). Remarque : Précisons que R et R sont quelconques au sens où la dynamique privilégie certains référentiels dits galiléens (cf. chapitre 4). En revanche, en cinématique de la relativité restreinte, R et R doivent être tous deux galiléens (cf. Relativité). z

z

A

R

y

O

R

O

R

x

O

z

z

O Index

y

H x

x a)

b) F IG . 3.1.

x y

R

y H

40

3. Changement de référentiel

I . — RELATIVITÉ DU MOUVEMENT I . 1 . — Hypothèse de l’universalité du temps ou quatrième loi de Newton Considérons deux référentiels R et R  en mouvement quelconque l’un par rapport à l’autre et un phénomène physique se produisant au point O , par exemple le passage devant un index de l’extrémité d’un pendule dont le bâti est lié à R (Fig. 3.1b). L’hypothèse de l’universalité du temps, admise par Newton, suppose que les périodes du pendule mesurées par les horloges H et H  sont égales : le temps est un paramètre universel. Cette hypothèse, qui est à la base de la cinématique dite galiléenne ou newtonienne, est confirmée par l’expérience avec une excellente approximation, mais infirmée dès que les vitesses considérées ne sont plus négligeables devant la constante d’Einstein c (cf. Relativité et invariance ).

Dans le cadre galiléen, deux référentiels R et R  ne se distinguent donc que par leur repère d’espace, lequel est défini par l’ensemble d’un point origine et de trois vecteurs linéairement indépendants qui forment la base du référentiel. I . 2 . — Hypothèse sur l’espace en cinématique galiléenne Entre les positions r et r  que donnent deux observateurs différents, en mouvement l’un par rapport à l’autre, lorsqu’ils localisent spatialement un même événement en un point A , on admet que l’on puisse choisir des étalons de mesure identiques de sorte que (Fig. 3.1b) : r = r + OO

ou

OA = OA + OO

Selon cette hypothèse, l’espace n’est pas universel mais relatif (r = r) : la position de A n’est pas invariante par changement de référentiel. Remarque : Cette relation, qui s’écrit aussi OA = OA + OO, ne doit pas être confondue avec la relation d’addition vectorielle, dite de Chasles, dans un même référentiel, OA = OO  + OA, laquelle se justifie par l’appartenance des points O, O et A à un même référentiel R. a) Cas d’un mouvement de translation rectiligne uniforme de R  par rapport à R

Dans ce cas particulier, on a, en désignant par v e la vitesse constante de O par rapport à R , en choisissant l’axe Ox selon cette vitesse et en supposant que O coïncide avec O à l’instant pris comme origine (Fig. 3.2) : OA = O A + OO  = O A + ve t ce qui donne, en explicitant : x = x  + vet

y = y

y

y

z = z R A

R O

x

x O F IG . 3.2.

ve

41

Changement de référentiel b) Transformation de Galilée

Pour une famille particulière de référentiels privilégiés dits galiléens, qui sont en translation les uns par rapport aux autres (cf. chapitre 4), les formules de transformation précédentes sur le temps et sur l’espace entre deux référentiels R et R forment la transformation de Galilée : t = t

x = x  + v et  = xe + vet

y = y

z = z

Vectoriellement, la transformation de Galilée sur les coordonnées spatiales s’écrit aussi, en utilisant les symboles  et ⊥ pour désigner respectivement les directions parallèles et perpendiculaires à la vitesse ve : OA = O A + OO

et OA ⊥ = O A ⊥ d’où

OA = OO  + OA

En cinématique galiléenne, on retrouve le même résultat que celui donné par la relation de Chasles, d’où la confusion fréquente entre les deux relations. c) Limites de la cinématique galiléenne : la transformation de Lorentz-Poincaré Historiquement, les hypothèses fondamentales précédentes ont donné d’excellents résultats pendant les quelques trois cents ans qui ont suivi la première synthèse de Newton entre pesanteur et gravitation (cf. chapitre 7), au point que, confrontés à ses insuffisances en physique moderne, précisément dès que la vitesse des corps n’est plus négligeable devant la constante d’Einstein c , les physiciens on a dû se défaire de ce que l’on supposait immuable. Cet épisode rappelle que la physique (et donc la mécanique) s’appuie sur un corps de doctrines conforté ou rejeté par l’expérience, comme toute théorie scientifique ! Même si la cinématique galiléenne constitue encore aujourd’hui, pour beaucoup de phénomènes quotidiens, une excellente approximation, l’analyse la plus satisfaisante aujourd’hui est celle que proposa Einstein en 1905 (relativité restreinte), puis en 1916 (relativité générale) ; c’est cette dernière qui permet une localisation spatiale précise des objets (cf. Relativité et invariance). La transformation de Lorentz est la généralisation suivante de la transformation de Galilée : ve t = g e t + 2 x  x = ge (x + ve t) y = y z = z c avec ge = (1 − ve2/c 2) −1/2 . Comme on a aussi : x = ge (x − vet)

soit

x=

x + vet ge

 c Dunod – Toute reproduction non autorisée est un délit

on en déduit l’écriture vectorielle suivante de la transformation spatiale de Lorentz-Poincaré : O A OA = + OO ge

et OA⊥ = O  A⊥ d’où

OA = O A + OO 

Ainsi, dans le cas de la transformation de Lorentz-Poincaré, la relation entre les coordonnées d’un même événement par rapport à deux référentiels galiléens ne coïncide pas avec la relation géométrique de Chasles, ce qui souligne la différence de nature entre ces deux relations. I . 3 . — Dérivées d’un vecteur C’est un fait expérimental bien connu que les concepts de vitesse et d’accélération sont relatifs au référentiel par rapport auquel on analyse le mouvement. Nous avons vu que, dans le calcul, cela se traduisait par la nécessité de dériver, par rapport au temps et relativement à une base, le vecteur reliant le point A considéré à l’origine choisie du référentiel. Rappelons que l’expression relativement à une base veut simplement dire que l’on considre que les vecteurs de base du référentiel ne varient pas au cours du mouvement.

42

3. Changement de référentiel

Il importe donc, avant tout développement, d’établir la relation entre les dérivées d’un même vecteur, par rapport au temps, mais relativement à des bases de référentiels différents. Considérons un vecteur U de composantes (X , Y , Z ) dans la base de R et (X  , Y, Z  ) dans celle de R : U = X eX + Y eY + Z e Z = X eX  + Y  eY  + Z e Z Par définition, la dérivée d’un vecteur par rapport au temps, relativement à une base, est le vecteur obtenu en dérivant ses composantes dans cette base et en considérant ses vecteurs de base fixes. Ainsi : dU dt

= X˙ eX + Y˙ eY + Z˙ eZ

dU dt

et

R

= X˙  eX + Y˙  eY  + Z˙  eZ  R

À partir de U = X e X + Y eY  + Z eZ  , exprimons (d U/ d t) R : dU dt

= X˙ e X + X  R

d eX dt

d eY  dt

+ Y˙  e Y + Y  R

+ ˙Z eZ + Z R

d eZ dt

R

Or, V étant le vecteur vitesse de rotation de R , par rapport à R, on sait que : d eX dt

R

= V × eX · · ·

d’où

dU dt

dU dt

= R

R

+ V × (X eX + Ye Y + Z eZ )

Il en résulte que : dU dt

dU dt

= R

R

+V×U

Retenons que le terme complémentaire provient de la modification de la direction des vecteurs unitaires de la base de R , du fait du mouvement de rotation de R par rapport à R.

Cette égalité vectorielle a été établie au XIX e siècle par l’ingénieur français E. Bour. Elle est très utile dans la pratique, car elle permet, connaissant les composantes d’un vecteur U dans la base de R , de déterminer les composantes dans R de sa dérivée par rapport au temps, relativement à une autre base. Notons que V × U est nul si R et R sont en translation l’un par rapport à l’autre (V = 0) : dU dt

= R

dU dt

R

Remarque : (1) Dans le cas où U est un vecteur fixe par rapport à R , ce qui est le cas des vecteurs unitaires de R, cette formule se réduit à : dU dt

R

=V×U

(2) Le vecteur V admet les mêmes dérivées par rapport à t, relativement à R et R , quel que soit le mouvement de R  par rapport à R : dV dt

= R

dV dt

R

43

Changement de référentiel I . 4 . — Composition des vecteurs vitesses de rotation

Soient deux points A et B appartenant à un référentiel R 2, en rotation par rapport aux référentiels R 0 et R1 . Notant V2/0 et V 2/1 les vecteurs vitesses de rotation de R 2 par rapport à R 0 et R 1 respectivement, et V1/0 le vecteur vitesse de rotation de R 1 par rapport à R0, il vient : d AB dt

0

= V 2/0 × AB

Comme, d’autre part :

d AB dt

et

d AB dt

0

1

= V2/1 × AB

d AB dt

=

1

avec

d AB dt

=0 2

+ V1/0 × AB

nous en déduisons l’égalité : V2/0 × AB = V2/1 × AB + V 1/0 × AB, soit, puisque les points A et B de R2 sont quelconques : V 2/0 = V2/1 + V1/0

Cette addition vectorielle des vitesses de rotation se généralise à un nombre quelconque de repères. Exemple : Le repère R2 d’origine O, et de vecteurs de base e r, e w et eu (Fig. 3.3), s’obtient à partir du repère R = Oxyz en procédant à deux rotations successives, l’une d’angle w autour de l’axe O e z et l’autre d’angle u autour de l’axe O ew. L’axe Ou est l’intersection des plans O e x ey et O e r e u . V R2 /R = VR 2/Ouwz + VOuwz/R = u˙ e w + w˙ ez z

r θ

eϕ y

O ϕ

x

θ u



 c Dunod – Toute reproduction non autorisée est un délit

F IG . 3.3.

II . — COMPOSITION DES VITESSES II . 1 . — Loi de composition des vitesses Les vitesses d’un même point A en mouvement par rapport aux référentiels R et R  , en mouvement quelconque par rapport à R , s’écrivent, respectivement : vA/R =

d OA dt

et vA/R  = R

d O A dt

R

Afin d’établir la relation entre ces deux vitesses, introduisons dans la première expression la relation vectorielle OA = OO + OA , O étant l’origine du repère R . Il vient : d OA dt

= R

d OO dt

+ R

d O A dt

R

44

3. Changement de référentiel

Le premier terme du membre de droite représente la vitesse de O par rapport à R, v O /R . Quant au second, il s’écrit, d’après la formule de Bour : d O A dt

d O A dt

= R

R

+ VR /R × O A

où VR /R est le vecteur vitesse de rotation de R  par rapport à R . Il en résulte : vA/R = v O /R +

d OA dt

R

+ VR /R × O A

soit la relation suivante, entre les vitesses vA/R et vA/R : vA/R = vA/R + v e/R

ve/R = v O/R + V × O  A

avec

en simplifiant l’écriture de VR /R en l’absence d’ambiguité. La vitesse ve est appelée la vitesse d’entraînement. Notons que c’est la vitesse par rapport à R du point A , invariablement lié à R , et qui à l’instant considéré coïncide avec A ; aussi dit-on que c’est la vitesse du point A  coïncidant avec A. Dans le cas particulier de la translation, V = 0, la vitesse d’entraînement se réduit à v O /R = v e (Fig. 3.2) ; elle est alors indépendante de la position du point A : on dit, exceptionnellement, que A est entraîné avec la « vitesse du référentiel R ». Remarque : Curieusement, le concept de vitesse d’entraînement n’est pas introduit dans les ouvrages anglo-saxons. II . 2 . — Exemples a) Problème du parapluie Comment une personne, se déplaçant suivant l’axe Ox d’un référentiel R, avec une vitesse u, doit-elle incliner son parapluie pour se protéger au mieux de la pluie qui tombe suivant −Oy avec la vitesse v (Fig. 3.4) ? y

R0

y0 u v0 R

O

u

v ® O0

x

x0

F IG . 3.4.

L’axe du parapluie doit être dirigé suivant la direction de la vitesse v des gouttes de pluie par rapport à R lié à la personne en mouvement : v  = v − u, u étant la vitesse d’entraînement, d’où l’angle d’inclinaison a = (v, v ) tel que tan a = u/v . b) Théorie newtonienne de l’aberration des étoiles La composition des vitesses précédente a permis à l’astronome anglais J. Bradley de montrer, en 1728, que la lumière se propageait avec une vitesse finie de l’ordre de 3 × 108 m . s −1 , conformément aux conclusions tirées en 1676 par l’astronome allemand O. Roemer, à partir de l’observation du satellite Io de Jupiter. C’est le phénomène connu sous le nom de l’aberration des étoiles, que l’on doit étudier en toute rigueur dans le cadre de la relativité.

45

Changement de référentiel

Bradley observa que l’étoile g du Dragon (constellation entre la Grande et la Petite Ourse), située approximativement sur un axe perpendiculaire au plan de l’écliptique (plan du mouvement de la Terre autour du Soleil) décrivait, en une année, une trajectoire apparente P E  A H , homothétique de celle PEAH de la Terre (Fig. 3.5a). Le diamètre apparent de cette trajectoire vaut 2a ≈ 2u/v. À partir de la mesure de 2a ≈ 40 et de u ≈ 30 km . s−1 , Bradley en déduisit v ≈ 3 × 108 m . s −1 (Fig. 3.5b). Remarques : (1) l’aberration stellaire diffère de la parallaxe, car elle concerne toutes les étoiles de la même manière. (2) Cette expérience est la première qui montra le mouvement quasi circulaire du centre de la Terre autour du Soleil. H0 ° A0

P0

°

A0

E0

(Printemps) P

® Plan de l' écliptique Terre

Soleil

E (Été)

(Hiver) H

v0

(Automne) A u a)

u

A v

® b)

F IG . 3.5.

c) Lancement vers l’est des satellites artificiels La composition des vitesses permet de montrer qu’on tire avantage, au mieux, de la rotation de la Terre en installant les champs de tirs de satellites artificiels le plus près possible de l’équateur et en les lançant vers l’est (Fig. 3.6a). Sur la figure, on a représenté le référentiel terrestre R = Txyz invariablement lié à la Terre, le référentiel géocentrique Rg = Tx0y0 z0 , qui a pour origine le centre T de la Terre et des axes parallèles à ceux du référentiel de Copernic R0 = C0 x0y 0z0 d’origine le centre de masse C du système solaire et dont les axes sont définis par trois étoiles éloignées. z z0  c Dunod – Toute reproduction non autorisée est un délit

T

Ouest

Est N

z0 C0 x0

H

O µ

y0

v O=Rs y vA=R 0

T

R0

x0

a)

v A=R

y0 x

Rg

O

S

ve

b) F IG . 3.6.

est :

La vitesse d’un point O de la surface de la Terre, de latitude l = p/2 − u, par rapport à R g , v O/R g = V T RT sin u ≈ 465, 4 × cos l (m . s −1)

46

3. Changement de référentiel

puisque le rayon terrestre et la vitesse de rotation de la Terre autour de son axe de révolution valent respectivement : R T ≈ 6, 4 × 106 m et V T =

2p ≈ 7, 3 × 10 −5 rad . s −1 3 600 × 24

D’après la composition des vitesses, une fusée A lancée en O, de la Terre R = Txyz, avec une vitesse vA/R a, par rapport à R g, la vitesse vA/Rg = v A/R + v O/R g (Fig. 3.6b). Pour que v A/Rg soit maximal, il faut que : i) v O/Rg soit maximal, c’est-à-dire l = 0, ii) vA/R soit colinéaire et de même sens que vO/R g, c’est-à-dire dirigé vers l’est. Remarque : (1) Parmi les bases de lancement de satellites, Cap Canaveral aux États-Unis (l = 28, 5 ◦), Pletsek en Russie (l = 63 ◦ ), Tyuratam (Baïkonour) dans le Kazakhstan (l = 46, 25 ◦), Tanegashima au Japon (l = 30, 5 ◦ ), Kourou en Guyane française (l = 5, 23 ◦), c’est cette dernière qui réalise le mieux la première condition. En 1999, une fusée a été lancée depuis l’équateur (l = 0), à partir d’une ancienne plateforme de forage en mer transformée en base de lancement. Cependant l’intérêt technique et économique d’une telle mise en œuvre s’avère limité. (2) Pour que le lancement soit effectué en toute sécurité, notamment en cas d’explosion de la fusée, les régions situées à l’est des bases de lancement sont généralement inhabitées. C’est un autre avantage du site de Kourou. d) Composition des vitesses en coordonnées sphériques Une tige T , dont une extrémité O est fixe dans un référentiel R = Oxyz, a par rapport à R un mouvement quelconque caractérisé par les paramètres u et w. Un point mobile A se déplace sur la tige. Sa position sur T est définie par r = OA (Fig. 3.7). Comme OA = r er , on en déduit : vA/T = ˙r er. D’autre part, ve = V×OA avec V = u˙ e w +w˙ ez, d’où : vA/R = v A/T + v e avec ve = (u˙ ew + w˙ ez ) × r er Dans la base (er, e u, e w) liée à T , les vecteurs v A/T , ve et vA/R s’explicitent selon :

T

r˙ 0 0

T

w˙ cos u 0 r −w˙ sin u × 0 = r u˙ ˙ sin u u˙ T 0 T rw

et T

z H T

A θ r

R



O P

ϕ x eθ

F IG . 3.7.

y

˙r ru˙ rw˙ sin u

47

Changement de référentiel

III . — COMPOSITION DES ACCÉLÉRATIONS III . 1 . — Loi de composition des accélérations Dérivons v A/R dans l’équation traduisant la loi de composition des vitesses : d v A/R dt

= R

d vA/R dt

d v O/R dt

+ R

On a, d’après la formule de Bour : d v A/R d v A/R = + V × vA/R dt dt R R

+ R

dV dt

R

d O A dt

et

× OA + V ×

= R

d O A dt

d O A dt

R

R

+ V × O A

Comme : aA/R =

d vA/R dt

d v A/R  dt

aA/R = R

et a O/R = R

d v O/R dt

R

on obtient : aA/R = a O /R + a A/R +

dV dt

R

× OA + V × (V × O A) + 2V × vA/R 

ce que l’on écrit sous la forme : aA/R = aA/R  + ae/R + a C avec : ae/R = aO  /R +

dV × OA + V × (V × O  A) dt

et

aC = 2V × v A/R 

a) Accélération d’entraînement

 c Dunod – Toute reproduction non autorisée est un délit

Le terme a e est appelé l’accélération d’entraînement. Il s’interprète comme ve/R : c’est l’accélération d’un point A invariablement lié à R puisque, d’après la formule de composition des accélérations, on voit que aA /R = a e/R si aA /R et vA /R  sont nuls. On dit aussi que ae/R est l’accélération du point A coïncidant avec A. Dans le cas de la translation (V = 0), a e se réduit à ae/R = aO /R . L’accélération d’entraînement est la même pour tout point A. C’est le cas exceptionnel où l’on peut parler d’« accélération d’un référentiel par rapport à un autre référentiel ». Si O et O  sont confondus et V = Cte (Fig. 3.8) : ae/R = V × (V × OA) = −V 2 HA,

HA étant le vecteur projection de OA dans un plan perpendiculaire à V. C’est ce terme qui est à l’origine de la force centrifuge, comme nous le verrons ultérieurement (cf. chapitre 7). z z ae/R

H

A O x

y y



R x R

F IG . 3.8.

P

48

3. Changement de référentiel

Remarque : La relation entre ae et ve n’est pas simple : ae/R =

d ve/R dt

R

− V R /R × vA/R  =

d ve/R dt

R

sauf évidemment dans le cas de la translation. De même que pour v e/R , le concept d’accélération d’entraînement est ignoré dans les ouvrages anglo-saxons, alors qu’il joue un rôle essentiel dans l’expression des forces d’inertie (cf. chapitre 7). b) Accélération de Coriolis Le terme a C = 2V × vA/R  est l’accélération de Coriolis introduite par le physicien français G. Coriolis en 1835. C’est ce terme qui est à l’origine de la célèbre force de Coriolis (cf. chapitre 7). Notons que cette accélération disparaît dans les deux cas particuliers suivants : i) pour V = 0, c’est-à-dire pour un mouvement de translation, ii) si A est invariablement lié à R , c’est-à-dire en équilibre par rapport à R  . III . 2 . — Exemples a) Cinématique terrestre Proposons-nous d’exprimer l’accélération d’un point O de la surface de la Terre par rapport au référentiel de Copernic R0 (Fig. 3.6). Introduisons le référentiel géocentrique R g = Tx 0y0z0 . Il vient, d’après la composition des accélérations : a O/R0 = a O/R g + aT/R 0

puisque

VR g /R0 = 0

Comme aO/R g = −V 2T HO, V T étant la vitesse angulaire de rotation de la Terre autour de son axe de rotation, on a : aO/R0 = −V 2T HO + aT/R 0 . Concrètement : V2T HO = V2T R cos l = (7, 3 × 10 −5 )2 × 6, 4 × 106 × cos l ≈ 3, 41 × 10 −2 × cos l (m . s−2 ) et aT/R 0 =

v 2T/R0 ST



(30 × 10 3) 2 = 6 × 10 −3 m . s−2 149 × 109

b) Composition des accélérations en coordonnées sphériques Reprenons l’exemple du point A sur la tige T (Fig. 3.7). On a : aA/T = ¨r er

ae =

dV × OA + V × (V × OA) dt

et a C = 2V × vA/T

L’explicitation de ae et a C dans la base T , associée aux coordonnées sphériques, donne respectivement :

T

−rw˙ 2 sin 2 u − ru˙2 w˙ cos u 0 w ¨ cos u − w˙ u˙ sin u r −w˙ sin u × w sin u − w˙ u˙ cos u × 0 = u r ˙u + −¨ −rw˙ 2 sin u cos u + r ¨ ¨ ˙ r w ˙ sin u 0 u˙ ˙ u cos u + rw ¨ sin u u T T T T +2rw

49

Changement de référentiel et 2 T

w˙ cos u 0 r˙ −w˙ sin u × 0 = 2˙r u˙ ˙u ˙ sin u T 0 T 2r˙w

On en déduit aA/R = aA/T + a e + ac , de composantes :

T

¨r − r w˙ 2 sin2 u − r u˙ 2 r¨ u − r w˙ 2 sin u cos u + 2˙r ˙u r¨ w sin u + 2rw˙ u˙ cos u + 2˙rw˙ sin u

CONCLUSION Retenons : (1) La loi de composition des vecteurs vitesses angulaires : V 2/0 = V2/1 + V1/0 (2) La loi de composition des vitesses : vA/R = vA/R  + ve

avec

ve/R = v O /R + V × OA

où V est le vecteur vitesse de rotation de R par rapport à R. (3) La loi de composition des accélérations : aA/R = a A/R + ae + aC avec pour l’accélération d’entraînement et l’accélération de Coriolis, respectivement : ae/R = aO /R +

dV × O A + V × (V × OA) dt

et a C/R = 2V × v A/R

Ces relations possèdent un champ d’application très vaste, puisque, d’une part, elles préparent toute l’étude de la dynamique et, d’autre part, elles possèdent un intérêt pratique, par exemple dans l’étude de la cinématique des solides en contact (cf. chapitre 16).

 c Dunod – Toute reproduction non autorisée est un délit

EXERCICES ET PROBLÈMES P3– 1. Mouvement d’un point du périmètre d’un disque Un disque de rayon r tourne uniformément autour de son axe, à la vitesse angulaire V, dans le sens indiqué sur la figure 3.9. Son centre C se déplace sur la droite horizontale z = r du plan vertical Ozx du référentiel R = Oxyz. On appelle R∗ le référentiel Cxyz, en translation par rapport à R, d’origine C, et on désigne par u l’angle que fait un rayon CA du disque avec Cz, A étant un point de la périphérie. 1. Exprimer, dans la base de R, la vitesse et l’accélération de A par rapport à R ∗ .

2. Quelle vitesse, par rapport à R, doit-on donner à C pour que la vitesse v B/R du point le plus bas du disque soit nulle ? 3. Trouver les équations x = x(u) et z = z(u) de A, sachant que, pour u = 0, x = 0 et z = 2r.

50

3. Changement de référentiel y h z

z R

µ C B

r



AR Ω

O

R

D

H R1

B A

θ

C

x

O

x

F IG . 3.9.

F IG . 3.10.

P3– 2. Mouvement elliptique d’un point appartenant à une tige Les extrémités d’une tige, de longueur l, se déplacent, respectivement, le long de l’axe Ox d’un référentiel R = Oxyz et le long d’une droite D parallèle à l’axe Oy. La distance qui sépare D de Oy est OH = h (Fig. 3.10). La position, dans le plan Oxy, d’un point quelconque A de la tige BC est caractérisée par l’angle u = (−Oy, BC). On note b la distance AB. 1. Exprimer, en fonction de u, les coordonnées de C et A dans la base de R. 2. Quelles sont, dans la base de R, les composantes de v A/R et de v A/R 1 , R1 étant le référentiel, d’origine B, en translation par rapport à R ? Trouver la vitesse d’entraînement de R1 par rapport à R. 3. Quelles sont, dans la base de R, les composantes de a A/R et de a A/R1 ? Trouver l’accélération d’entraînement de R1 par rapport à R. 4. Montrer que la trajectoire de A est une ellipse et déterminer ses caractéristiques. P3– 3. Mouvement d’un mobile sur un cercle en rotation Un mobile A se déplace sur un cercle, de rayon R, qui tourne uniformément autour d’un diamètre vertical, comme le montre la figure 3.11. On note u l’angle (Oz, OA) qui situe A sur le cercle et a˙ = V la vitesse angulaire du repère R = Ox y z lié au cercle. 1. Exprimer, en fonction de u, la vitesse et l’accélération de A par rapport à R  , dans la base de Frenet. 2. Écrire, dans la base de R  , la vitesse d’entraînement, l’accélération d’entraînement et l’accélération de Coriolis. 3. En déduire les expressions, dans la base de R , de la vitesse et de l’accélération de A par rapport à R. Retrouver ces résultats directement à partir des composantes de OA dans la base de R . z V

A µ

A

y

O x

b y

x F IG . 3.11.

b −u

Ω B F IG . 3.12.

u b

51

Changement de référentiel P3– 4. Vitesse d’éjection de l’eau dans un tourniquet hydraulique

Un tourniquet hydraulique, destiné à l’arrosage des jardins, est constitué de deux bras symétriques, de longueur b, qui tournent autour d’un axe vertical avec la vitesse angulaire V. L’eau est éjectée par les extrémités des bras avec une vitesse u, par rapport aux bras, qui fait un angle b avec la normale aux bras (Fig. 3.12). 1. Trouver, en fonction de u, b et V, la vitesse de l’eau par rapport au référentiel terrestre R au moment de l’éjection. 2. Quelles sont ses composantes radiale et orthoradiale ? Étudier le cas où V = u cos b/b. P3– 5. Mouvement apparent d’un satellite artificiel Dans le référentiel géocentrique Rg , la trajectoire d’un satellite artificiel S de la Terre est un cercle de rayon r = 42 000 km contenu dans le plan TXY, l’axe TX coïncidant avec Tx 0 et l’axe TY faisant l’angle u avec Ty 0 (Fig. 3.13). La trajectoire circulaire est décrite uniformément avec la vitesse v. 1. Quelles sont les composantes de TS dans la base de TXYZ et dans celle de R = Txyz 0 lié à la Terre ? 2. Le rapport v/r est égal à la vitesse de rotation propre de la Terre autour de l’axe des pôles. Quelle est la trajectoire apparente de A pour un observateur terrestre (lié à R) ? À quelle condition le satellite paraît-il fixe pour cet observateur ? z0 Z

θ

S

Y

y

θ

T

y

B O1

y0 x

y

Ωt Oz

2Ω t x x

 c Dunod – Toute reproduction non autorisée est un délit

X , x0 F IG . 3.13.

F IG . 3.14.

P3– 6. Système bielle-biellette Une bielle OO 1, de longueur l, tourne uniformément autour d’un axe Oz avec une vitesse angulaire V par rapport au référentiel R = Oxyz. Une biellette O1B, de longueur moitié, tourne dans le même plan que la biellette. Son mouvement est tel que (ex , O1B) = 2(e x , OO1 ) à chaque instant (Fig. 3.14). Initialement les points O, O1 et B sont alignés sur l’axe Ox. 1. Exprimer, en fonction du temps, les composantes de OO 1 et de O 1 B dans la base de R  = Ox y z lié à la bielle. 2. Trouver la vitesse v B/R dans la base de R. Dans cette même base, écrire la vitesse d’entraînement ve de B dans le mouvement de R  par rapport à R. En déduire vB/R projeté dans R .

52

3. Changement de référentiel

P3– 7. Durée du jour solaire moyen et durée du jour sidéral La durée du jour solaire moyen est la durée T m qui sépare en moyenne deux positions successives du Soleil au zénith dans son mouvement apparent. La durée du jour sidéral est la durée Ts qui sépare deux positions successives d’une étoile éloignée dans son mouvement apparent. Montrer que T m − T s = T2m /(Ta + T m), Ta étant la période de révolution de la Terre autour du Soleil. Calculer Tm − Ts sachant que Tm = 24 × 3 600 = 86 400 s et T a = 365, 25 Tm . P3– 8. Lunaison synodique et lunaison sidérale La lunaison synodique est la durée T n qui sépare deux positions successives de la nouvelle Lune. La lunaison sidérale est la durée Tl de révolution de la Lune autour de la Terre dans le référentiel géocentrique. Montrer que T n − T l = Tl2 /(Ta − Tl ), Ta étant la période de révolution de la Terre autour du Soleil. Calculer Tn − Tl en jour, sachant que Tl = 27, 3 j et Ta = 365, 25 j. P3– 9. Mouvement d’un point sur un disque lié à une tige en mouvement Une tige T = OO 1, de longueur l = 40 cm, tourne dans le plan Oxy du référentiel R = Oxyz avec la vitesse angulaire w. ˙ Son extrémité O 1 est le centre d’un disque D de rayon r = 10 cm. On souhaite analyser le mouvement d’un point A du pourtour de D. On repère sa position par l’angle u que fait O 1A avec OO1. On désigne par R  = O1x y z le référentiel lié à la tige, de centre O1 et tel que l’axe Ox soit défini par OO1 (Fig. 3.15). 1. Donner les expressions vectorielles des vitesses vO 1 /R et vA/R . Exprimer ces vecteurs dans la base de R .

2. En déduire, en fonction des angles et de leurs dérivées, les composantes de v A/R dans la base de R. 3. Que deviennent ces composantes si u = 0 et u = p alors que u˙ =  0 ? Comparer les expressions obtenues à celle de la vitesse de O 1 par rapport à R. 4. Un mécanisme impose une vitesse nulle au point A si u = p. Établir la relation qui en résulte ˙ En déduire la vitesse angulaire du disque par entre u˙ et w. ˙ Sachant que w˙ = 2 tr . min−1 , calculer u. rapport à R. y

y1

A u

O1 D

T w O

x F IG . 3.15.

x1

4 Dynamique du corpuscule La dynamique est l’étude des mouvements des corps en relation avec les causes, appelées forces, qui les produisent. Les lois physiques sur lesquelles elle s’appuie ont été énoncées partiellement par G. Galilée en 1632 et complètement par I. Newton en 1687 dans le célèbre Principia mathematica. On peut dire que la science, telle que nous la concevons aujourd’hui, commence avec ces lois physiques de la mécanique énoncées par ces deux grands physiciens. Dans ce chapitre, nous considérons d’abord les lois relatives au mouvement d’un corpuscule (corps de petite dimension), connues sous les noms de 1 re et 2e lois de Newton. La première, ou principe de l’inertie, a été énoncée pour la première fois par Galilée ; dans la deuxième, Newton introduit déjà le concept de quantité de mouvement qui regroupe les notions de vitesse et de masse. La 3e loi de Newton, ou loi d’opposition des actions réciproques entre deux corpuscules, joue un rôle essentiel dans l’étude des systèmes de N corpuscules ou points matériels (cf. chapitre 13).

 c Dunod – Toute reproduction non autorisée est un délit

Remarque : On s’accorde souvent pour définir la science comme le mode de pensée qui permet de comprendre rationnellement le comportement de la nature à partir d’un petit nombre de principes réfutables par les faits. La rationnalité de la pensée et la réfutabilité des principes par les faits sont les caractéristiques qui singularisent l’activité scientifique par rapport aux autres activités humaines (artistique, religieuse, etc.). Ces dernières ont cependant une influence non négligeable dans les phases de création ou de transmission de la science.

I . — MASSE ET QUANTITÉ DE MOUVEMENT D’UN CORPUSCULE I . 1 . — Masse inerte L’Univers peut être considéré comme un ensemble de corpuscules. De tels éléments sont caractérisés : – i) cinématiquement, par un seul vecteur vitesse et un seul vecteur accélération ; on ne distingue donc pas les mouvements des différents points géométriques qui peuvent constituer de tels éléments ; – ii) dynamiquement, par un scalaire positif appelé masse inerte, brièvement masse, qui est constant (au cours du temps) et invariant (par changement de référentiel). On mesure la masse d’un corpuscule en la comparant à celle d’un autre corpuscule prise comme unité : la comparaison se fait naturellement à partir des lois physiques où les masses interviennent. Comme nous le verrons plus loin, dans la loi fondamentale de la dynamique, la masse inerte exprime une propriété particulière de la matière, l’inertie.

54

4. Dynamique du corpuscule L’unité de masse dans le Système International est le kilogramme (kg) : c’est la masse d’un certain cylindre en platine iridié, appelé prototype international du kilogramme.

Remarques : (1) Historiquement, les grands physiciens, Galilée, Newton, Euler, Maxwell, Boltzmann, ont utilisé respectivement les mots projectile, corpuscule, corps ponctuel, particule, molécule, pour désigner le point matériel, lequel fut introduit par le mathématicien suisse L. Euler en 1736 et en même temps la difficulté soulevée par une masse volumique infinie ! Dans la suite, on utilisera indistinctement ces différentes expressions. (2) En mécanique newtonienne, m = 0 exprime l’absence de matière. Il n’en est pas ainsi dans la théorie plus générale de la relativité où des corpuscules, de masse nulle, peuvent avoir une énergie et une quantité de mouvement non nulles ; la masse perd alors son statut de concept primaire (cf. Relativité et invariance). (3) Le mot inerte exclut, pour le corpuscule, la présence de forces intérieures caractéristiques des systèmes matériels formés d’un ensemble de corpuscules, tels que les systèmes vivants, par exemple. I . 2 . — Quantité de mouvement d’un corpuscule La quantité de mouvement est, pour un corpuscule A, de masse m et de vitesse v A/R par rapport à un référentiel R, le vecteur : p = mv A/R Tout comme la vitesse, ce vecteur est défini par rapport à un référentiel. Remarques : (1) Il n’est pas sans intérêt de noter que ce concept a été introduit initialement par R. Descartes en 1645 et repris par Newton en 1687 dès les premières pages de son traité Principia mathematica. Curieusement, dans l’enseignement français de la mécanique, on substitue à tort à ce concept, à la fois moderne et historiquement important, celui moins fondamental de quantité d’accélération ma A/R ! (2) En physique, on désigne souvent la quantité de mouvement par moment linéaire. Le mot moment vient du latin momentum, qui est une contraction de movimentum (mouvement) et de movere (déplacer) ; on le qualifie de linéaire par opposition à angulaire. Ce choix est naturel dans le contexte de la formulation lagrangienne ou hamiltonienne de la mécanique où toutes les grandeurs traduisant un mouvement selon un axe ou autour d’un axe sont appelées moments (cf. chapitre 24). On l’appelle aussi parfois impulsion, probablement en raison du rôle essentiel que joue p dans les collisions (cf. chapitre 14).

II . — LOI FONDAMENTALE DE LA DYNAMIQUE II . 1 . — Référentiels galiléens Pour formuler la loi fondamentale de la dynamique, il est nécessaire de considérer un référentiel, c’est-à-dire un repère d’espace et un repère de temps. Le temps étant universel en mécanique de Newton, il est judicieux de choisir un repère d’espace dans lequel la formulation de cette loi soit la plus simple possible. Un tel référentiel est qualifié de galiléen. Le référentiel du laboratoire, par rapport auquel les expériences quotidiennes sont conduites, peut être considéré comme une bonne réalisation d’un référentiel galiléen. Dans la suite proche, nous nous contenterons de ce résultat dont la justification est d’abord expérimentale. Nous verrons ultérieurement que certains désaccords irréductibles entre théorie et expérience ont conduit à substituer au référen-

55

Dynamique du corpuscule

tiel du laboratoire d’autres référentiels réalisant une meilleure approximation d’un référentiel galiléen (cf. chapitre 7). Remarque : En cinématique galiléenne, on n’a attribué aucun caractère privilégié au référentiel d’étude (cf. chapitre 3). Ce n’est que dans la recherche des causes du mouvement, c’est-à-dire dans sa relation aux forces, que l’on est conduit à donner à ce référentiel un statut particulier, galiléen ou non. Dans la théorie de la relativité d’Einstein, qui généralise celle de Galilée et de Newton, il est nécessaire au contraire de préciser la nature physique des référentiels dès la cinématique ; aussi dit-on que la relativité a fait entrer, en 1905, la cinématique dans le domaine des sciences physiques. II . 2 . — Forces ou interactions Tout système matériel de l’Univers exerce sur un corps ponctuel A, non inclus dans ce système, une force représentée, dans un référentiel galiléen R, par l’ensemble d’un point A et d’un vecteur F. Exemples Force de gravitation universelle Un corps ponctuel A 1 subit de la part d’un autre corps ponctuel A 2 une force d’expression :

 c Dunod – Toute reproduction non autorisée est un délit

F2→1 = −G

m ∗1m ∗2 er r2



er =

r et r = A 2 A1 = r 1 − r 2 r

La constante G = 6, 67 × 10−11 SI est la constante de Newton et les quantités scalaires positives m ∗1 et m∗2 sont les masses de gravitation de ces corpuscules (cf. chapitre 6). On montre, après analyse, que ces masses peuvent être identifiées aux masses inertes m1 et m2 de A1 et A2 (cf. chapitre 7). Cette force fut introduite par Newton pour interpréter le mouvement des planètes du système solaire (cf. chapitre 13). Son expression est parfois appelée la cinquième loi de Newton. La force de pesanteur ou poids d’un corps est une manifestation locale de la force de gravitation (cf. chapitre 7). Pour l’instant, on admettra qu’on peut l’écrire sous la forme mg , où g est le champ de pesanteur terrestre. La norme de g vaut, à Toulouse, g ≈ 9, 80 m . s−2 et sa direction définit la verticale. Force électromagnétique de Lorentz Un système matériel constitué de charges électriques au repos ou en mouvement par rapport à un référentiel galiléen R exerce, sur un corpuscule chargé électriquement A, une force : F = q(E + v × B)

dans laquelle q est la charge électrique de A, E et B les champs électrique et magnétique produits par le système et v la vitesse de A par rapport à R. Cette force est appelée la force de Lorentz (cf. chapitre 8). Elle est bien plus intense que la précédente, comme le montre l’exemple suivant de l’atome d’hydrogène ; le rayon de l’atome de Bohr étant aB ≈ 50 pm, on a : F elec =

mpm e 1 e2 −8 = G ≈ 8 × 10 N et F ≈ 4 × 10−47 N grav 2 2 4pε0 aB aB

Forces nucléaires Les forces nucléaires, fortes et faibles, permettent d’expliquer la cohésion des nucléons qui composent le noyau. Retenons simplement qu’elles sont 100 fois plus intenses que les forces électromagnétiques et de très courte portée (∼ 1 fm = 10−15 m), au point que leur domaine d’application se limite au noyau.

56

4. Dynamique du corpuscule

Forces de contact L’expérience courante montre que l’on décrit bien le mouvement d’un corpuscule, à l’échelle macroscopique, en tenant compte de forces supplémentaires de contact : les forces de frottement visqueux, exercées par les fluides sur les corps en mouvement, de la forme −av ou −bv 2 (cf. chapitres 10 et 31) et les forces de frottement entre solides (cf. chapitres 9 et 19). Ces forces, qui jouent un rôle important dans la vie pratique, sont dites non fondamentales, car elles n’apparaissent pas à l’échelon microscopique. II . 3 . — Loi fondamentale de la dynamique ou deuxième loi de Newton a) Énoncés Énoncé historique Le changement de mouvement est proportionnel à la force imprimée et s’effectue suivant la droite par laquelle cette force est imprimée. Énoncé actuel Par rapport à un référentiel galiléen R, le mouvement d’un corpuscule A, soumis à plusieurs forces, dont la somme est F, satisfait à la relation : dp = dt

F

soit

m aA/R =

F

où m est sa masse inerte et p = mvA/R la quantité de mouvement de A par rapport à ce référentiel. Remarque : La dérivée par rapport au temps est relative à la base du référentiel galiléen R. Si les composantes de la quantité de mouvement p sont relatives à une autre base R  , il est indispensable d’utiliser la formule de Bour (cf. chapitre 3) : dp dt

= R

dp dt

R

+ VR /R × p

b) Inertie de la masse La propriété qu’a la matière d’intervenir dans la loi fondamentale par sa masse s’appelle l’inertie, car si deux corpuscules, initialement au repos dans R, sont soumis à la même force F, celui dont la masse est la plus grande acquiert l’accélération la plus faible : son aptitude à s’opposer à l’effet de F est donc plus grande ; on dit que son inertie est plus grande. Remarques : (1) Il convient de distinguer la matière, qui caractérise les objets qui nous entourent, de sa masse inerte, qui, elle, traduit l’une de ses propriétés, son inertie. (2) Contrairement à ce que l’on pensait, avant la découverte en 2012 du boson H dit de Higgs, la masse d’une particule apparaît désormais comme le résultat de son interaction avec un champ particulier appelé champ H. II . 4 . — Relativité galiléenne Montrons que la loi fondamentale de la dynamique est invariante par changement de référentiel galiléen. Pour cela, considérons le mouvement d’un corpuscule A par rapport à deux référentiels galiléens R et R . Comme les accélérations d’entraînement et de Coriolis, dans la composition des mouvements entre R et R, sont nulles, on a aA/R = a A/R , d’où, en admettant l’invariance des forces par chan-

57

Dynamique du corpuscule gement de référentiel : F = maA/R

et

F = ma A/R

Ainsi, aucune expérience de mécanique ne permet de distinguer deux référentiels galiléens entre eux. C’est bien ce que l’expérience courante nous apprend ; dans un véhicule (automobile, train, avion, bateau), se déplaçant à vitesse vectorielle constante par rapport au sol terrestre, il est impossible de déceler notre état de mouvement, si l’on ne regarde pas à travers une fenêtre évidemment. Seules les modifications occasionnelles de cette vitesse, dans un virage ou au cours d’un freinage, sont détectables. Ce résultat essentiel a été très tôt perçu par Galilée, en 1632, et résumé par la phrase bien connue : « le mouvement (rectiligne et uniforme) n’est rien ». Cette invariance de la loi fondamentale en changeant de référentiel galiléen est appelée la relativité galiléenne. Dans ce contexte, le référentiel de Copernic, souvent utilisé comme référentiel galiléen, n’est qu’une excellente réalisation d’un référentiel galiléen parmi d’autres.

 c Dunod – Toute reproduction non autorisée est un délit

Remarques : (1) La relativité galiléenne montre clairement que le concept d’espace absolu, introduit initialement par Newton, n’a aucune justification scientifique. Il est surprenant qu’on le trouve encore, dans certains ouvrages ou publications de mécanique newtonienne, alors que, dans cette approximation, si le temps est absolu, l’espace est, lui, relatif, au sens où la vitesse d’un corpuscule dépend du référentiel d’étude. La seule vitesse qui soit absolue, au sens de l’invariance par changement de référentiel galiléen est la vitesse de la lumière dans le vide, laquelle sort précisément du champ de la mécanique newtonienne. (2) L’invariance galiléenne dépasse le cadre de la mécanique newtonienne. En effet, Einstein la généralisa en 1905 à tous les phénomènes physiques, ce qui permit de résoudre l’important problème posé par la constatation expérimentale de l’invariance de la vitesse de la lumière dans le vide (cf. Relativité et invariance). Exemple : chute libre La chute libre des corps dans le voisinage de la surface de la Terre a été étudiée complètement par Galilée. C’est lui qui le premier a montré que deux corps, suffisamment denses pour rendre négligeable l’influence de l’air, que l’on abandonne sans vitesse de la même hauteur, devaient atteindre le sol en même temps. C’est lui aussi qui montra l’influence des conditions initiales sur la nature de la trajectoire du mouvement d’un corps : elle est rectiligne si le corps est abandonné sans vitesse et parabolique s’il est lancé avec une vitesse initiale perpendiculaire à la verticale du lieu. Pour établir ce résultat, appliquons par rapport au référentiel terrestre la loi fondamentale de la dynamique à un corps ponctuel soumis à la seule force de pesanteur m g : ma = mg

d’où

a=g

ce qui s’explicite selon : ¨ x = 0 et ¨ y = −g. Dans le cas de la chute libre d’une hauteur h , on a :

gt2 +h 2 Dans ce contexte, il est instructif de citer le célèbre problème posé par Galilée dans le Dialogue des deux mondes : quel est le point de chute d’un boulet abandonné au sommet du mât vertical d’un voilier, qui se déplace à vitesse (vectorielle) constante par rapport au référentiel terrestre, si on néglige l’influence de l’air (Fig. 4.1) ? Le point situé au bas du mât, en avant ou en arrière ? Comme le référentiel du voilier possède les mêmes propriétés que le référentiel terrestre et que, dans ce dernier, tout corps abandonné sans vitesse a une trajectoire verticale, la réponse est le point situé au bas du mât. x˙ = Cte = 0

d’où

x = Cte = 0 et

˙y = −gt + Cte = −gt

d’où

y=−

58

4. Dynamique du corpuscule

Remarques : (1) Le résultat précédent n’est pas intuitif, comme le prouvent les réponses erronnées données même par des personnes instruites. (2) On montre qu’en réalité on doit ajouter une force supplémentaire appelée la force de Coriolis terrestre dont l’influence peut être ici négligée (cf. chapitre 7). (3) Soulignons bien que cette chute, indépendante de la masse, n’est vraie que si l’influence de l’air peut être négligée ; en dehors de cette hypothèse, la chute dépend de la masse, car la force de frottement n’est pas, elle, proportionnelle à la masse. y R (Voilier)

y

u

R (Terre) O

O

x

(Mer)

x

F IG . 4.1.

II . 5 . — Application à la mesure des forces La preuve de l’existence d’une force est donnée par la non-observation du principe de l’inertie. On constate qu’en présence de corps étrangers à un corpuscule A, la quantité de mouvement de ce dernier n’est pas constante. La relation d p/ d t = F devient alors une relation permettant de déterminer la force inconnue F. C’est ainsi que Newton a pu établir, à partir de données observationnelles en astrophysique, que la force de gravitation était inversement proportionnelle au carré de la distance des corps ponctuels en interaction. Très souvent, on mesure F par la variation de la déformation d’un système, tel qu’un ressort dont on a fixé une extrémité. Il est nécessaire dans ce cas d’étalonner préalablement le ressort. On réalise ainsi un dynamomètre. L’unité SI de force est le newton (N). II . 6 . — Contenu physique des deux premières lois de Newton Les deux premières lois de Newton font intervenir simultanément plusieurs concepts : l’espace, le temps, des référentiels privilégiés, la masse inerte d’un corpuscule et la somme des forces qu’exerce son environnement sur lui. Elles peuvent donc servir à déterminer l’une de ces grandeurs si les autres sont connues. Cependant, la deuxième loi exprime dans le cas général l’égalité entre le taux de variation de la quantité de mouvement d’un corpuscule et la somme des forces qui s’exercent sur lui ; au préalable, chacune de ces forces a pu être étudiée avec soin indépendamment des autres, en considérant un autre corps ponctuel dont la quantité de mouvement s’est trouvée modifiée à la suite de son intervention. Lorsque l’étude expérimentale du mouvement ne coïncide pas avec les prévisions théoriques, comment doit-on modifier la loi fondamentale ? Seule une analyse physique du problème permet de répondre. On doit se demander : i) si le système étudié est bien défini, car on a pu oublier une interaction, ii) si l’échelle de temps est suffisamment précise, iii) si le repère utilisé est une bonne réalisation d’un repère galiléen, iv) si l’expression des forces est correcte.

59

Dynamique du corpuscule

Compte tenu de la précision des mesures, le temps fourni par les physiciens n’est pas en cause. D’autre part, on hésite à modifier les expressions des forces que des considérations de symétrie et de simplicité, voire d’élégance, ont permis d’établir, et que des expériences ont par ailleurs confirmées. L’erreur la plus fréquente est celle due à un mauvais choix de référentiel galiléen. C’est historiquement ce qui s’est passé : on s’est aperçu que certains désaccords disparaissaient lorsqu’on adoptait comme référentiel galiléen un référentiel lié au système stellaire et non un référentiel lié à la Terre ; c’est ainsi qu’on a pu expliquer la déviation vers l’est, le comportement du célèbre pendule de Foucault, l’existence de deux marées par jour, etc. (cf. chapitre 7). Certaines divergences entre les résultats expérimentaux et les prévisions n’ont pas pu être attribuées aux causes précédentes, notamment quand les forces d’interaction étaient d’origine électromagnétique ou lorsque les vitesses n’étaient pas négligeables devant la constante d’Einstein (vitesse de la lumière dans le vide) qui vaut environ 300 000 km . s−1. Elles n’ont pu être expliquées que par la théorie de la relativité restreinte élaborée par Einstein en 1905 dans laquelle les bases de la cinématique, et par conséquent la loi fondamentale, sont différentes. On a toujours : dp = dt

F

mais p = gmv



v2 g= 1− 2 c

−1/2

est le facteur relativiste (cf. Relativité et invariance).

III . — PRINCIPE DE L’INERTIE Ce principe de l’inertie, connu aussi sous le nom de première loi de Newton, a été énoncé en réalité pour la première fois par Descartes en 1644 (Newton n’avait que deux ans). III . 1 . — Énoncés historique et actuel a) Énoncé historique

 c Dunod – Toute reproduction non autorisée est un délit

On peut lire dans les premières pages du traité de Newton : « Tout corps persévère dans son état de repos ou de mouvement rectiligne uniforme, sauf si des forces imprimées le contraignent d’en changer. » b) Énoncé actuel (

Par rapport à tout référentiel galiléen R , tout corps ponctuel A , éloigné de tout autre corps, F = 0) , a un mouvement rectiligne uniforme : v = Cte . En effet, d’après la deuxième loi de Newton, si dp =0 dt

d’où

p = Cte

F = 0 , alors : et

v = Cte

Notons que la vitesse et la quantité de mouvement sont alors des constantes vectorielles : le mouvement de A est rectiligne et uniforme, ce qui caractérise un corpuscule soumis à une force nulle. Le repos correspond évidemment à une valeur nulle de la vitesse.

60

4. Dynamique du corpuscule

Remarques : 1) C’est dans Principe des choses matérielles que l’on trouve l’énoncé de Descartes du principe de l’inertie sous sa forme définitive, avec mouvement rectiligne et uniforme. Ajoutons, pour l’anecdote, que, dans son énoncé original, Descartes associait explicitement la conservation de la vitesse d’un corps ponctuel isolé à l’immuabilité de Dieu ! Galilée, lui, n’avait, comme ses prédécesseurs, considéré que les mouvements circulaires uniformes en omettant le caractère rectiligne pourtant essentiel dans le principe de l’inertie. 2) Dans ce contexte, il n’est pas inutile de rappeler l’erreur historique d’Aristote, selon lequel « Il n’y a pas de mouvement (vitesse) sans moteur (force) ». 3) En dernière analyse, comme l’a fait remarquer le physicien autrichien Ernst Mach (prononcez « mar »), cette première loi de Newton n’est qu’une conséquence de la loi fondamentale de la dynamique, lorsque le corps ponctuel n’est soumis à aucune force. c) Corpuscule isolé et corpuscule pseudo-isolé On distingue parfois un corpuscule isolé, car soumis à aucune force ( F = 0 ), d’un corps ponctuel pseudo-isolé, soumis lui à un ensemble de forces dont la somme est nulle : F=0. d) Équilibre mécanique Un corpuscule, isolé ou pseudo-isolé, est dit en équilibre ou au repos par rapport à un référentiel galiléen si sa vitesse est nulle, ce qui suppose que sa vitesse initiale soit nulle. En effet : F=0

d’où

v = Cte = 0

Par exemple, une masselotte A , soumise à son poids mg et à la tension T d’un fil, est en équilibre si : mg + T = 0

et v = 0

III . 2 . — Référentiel inertiel Un référentiel est qualifié d’inertiel, si on peut y réaliser le principe de l’inertie. C’est le cas, pour le référentiel du laboratoire, si la pesanteur n’est pas prise en compte, parce que négligeable dans ses effets, ou compensée par une autre force ; référentiel du laboratoire et référentiel inertiel coïncident alors. Une table à coussin d’air, qui permet de compenser la pesanteur par la réaction normale qu’exerce de l’air soufflé par la table, est un référentiel galiléen, qui est aussi inertiel à deux dimensions. Nous verrons ultérieurement (cf. chapitre 7) qu’une cabine d’ascenseur en chute libre ou un vaisseau spatial sans propulsion réalisent tous deux un référentiel inertiel à trois dimensions, mais non galiléen. De façon générale, le référentiel du laboratoire est galiléen, avec une excellente approximation, mais il n’est pas inertiel, pour tout mouvement à trois dimensions, précisément en raison de la pesanteur qui empêche d’y réaliser le principe de l’inertie. Remarque : La plupart des auteurs ne font pas de distinction entre les référentiels inertiels et les référentiels galiléens.

61

Dynamique du corpuscule

IV . — MOMENT CINÉTIQUE IV . 1 . — Définition On appelle moment cinétique d’un corpuscule A, par rapport à un référentiel R, en un point O de R, le moment de sa quantité de mouvement : LO = OA × p = OA × mvA Remarques : (1) Le moment cinétique est parfois appelé moment angulaire, ce qui est naturel dans le contexte de la formulation lagrangienne ou hamiltonienne (cf. chapitre 24). (2) En France, le moment cinétique est encore souvent noté par la lettre s ; en adoptant L, nous nous sommes conformés à une recommandation internationale. IV . 2 . — Théorème du moment cinétique Si on dérive L O/R par rapport au temps, relativement à R, supposé galiléen, il vient : d LO d OA dp dp = × p + OA × = OA × = OA × dt dt dt dt

F

d’où : d LO = dt

MO

Ce résultat, qui pour un corpuscule est une conséquence de la loi fondamentale, s’énonce ainsi : La dérivée par rapport au temps du moment cinétique d’un corpuscule, en un point fixe O d’un référentiel galiléen, est égale à la somme des moments des forces qui s’exercent sur lui. Remarque : Évidemment, comme pour la loi fondamentale, la dérivée par rapport au temps est relative à la base du référentiel galiléen R. Si les composantes du moment cinétique sont relatives à une autre base R , il est indispensable d’utiliser la formule de Bour (cf. chapitre 3) :

 c Dunod – Toute reproduction non autorisée est un délit

d LO dt

= R

d LO dt

R

+ V R/R × LO

Ce théorème est très commode lorsque le moment des forces est nul. On obtient alors immédiatement une constante vectorielle du mouvement : L O/R = Cte (cf. chapitre 12). Nous avons supposé que le point O était fixe par rapport à R. Étudions le cas où le moment cinétique est calculé en un point O mobile dans R galiléen. Comme L O/R = O A × mv A/R , il vient, en dérivant par rapport au temps, relativement à R : d L O /R dp = vA/R − vO/R × mvA/R + OA × dt dt

En un point mobile O de R, le théorème du moment cinétique s’écrit donc : d LO/R + vO/R × m vA/R = dt

M O

Retenons que si le point où l’on applique le théorème du moment cinétique est mobile, il faut ajouter à la dérivée du moment cinétique le terme complémentaire v O /R × m vA/R .

62

4. Dynamique du corpuscule

Remarque : Le premier membre de l’équation précédente s’identifie au moment dynamique au point mobile O (cf. chapitre 18).

V . — MOUVEMENTS RECTILIGNES ET MOUVEMENTS PLANS A priori, la position d’un corpuscule dans l’espace dépend de trois paramètres indépendants, par exemple les coordonnées cartésiennes (x, y, z), les coordonnées cylindriques (r, w, z), etc. On dit que le corpuscule considéré a trois degrés de liberté. Nous verrons ultérieurement, lorsque le corps est guidé (cf. chapitre 9), que ce nombre diminue sous l’effet de liaisons qui lui sont imposées : contact sur une surface, évolution sur une ligne matérielle, etc. En l’absence de liaison, le mouvement d’un corpuscule peut être rectiligne ou plan si certaines conditions sont réalisées. C’est ce que nous nous proposons d’analyser. V . 1 . — Mouvements rectilignes a) Somme des forces et vitesse initiale parallèles F des forces qui s’exercent sur le corpuscule considéré est constamment parallèle Si la somme à une direction fixe D et si la vitesse initiale v 0 est parallèle à D ou nulle, la trajectoire est parallèle à D. Pour établir ce résultat, il suffit de projeter la loi fondamentale dans une base du référentiel telle que ez coïncide avec D. Suivant les autres axes, on trouve : ¨ x=0

¨ y = 0 d’où

x˙ = Cte = 0

˙y = Cte = 0

Il en résulte en intégrant que x = Cte et y = Cte. Dans le cas singulier où la vitesse v 0 est nulle, on obtient le même résultat puisque seule change la vitesse initiale suivant l’axe des z. b) Somme des forces et vitesse initiale centrales F et la vitesse initiale v 0 sont centrales, c’est-à-dire passent par un point O fixe, Si la somme la trajectoire est la droite définie par ce point et la vitesse initiale v 0 . En effet, le moment de F étant nul, le moment cinétique L O = OA × mv est une constante nulle, puisqu’initialement vitesse et vecteur position sont colinéaires. Il en résulte qu’à tout instant ultérieur cette propriété demeure : la trajectoire est donc rectiligne. V . 2 . — Mouvements plans a) Somme des forces et vitesse initiale parallèles à un plan fixe F des forces qui s’exercent sur le point matériel et la vitesse initiale v 0 sont Si la somme parallèles à un plan fixe du référentiel, le mouvement est plan. En effet, en projetant la loi fondamentale sur un axe OX perpendiculaire à ce plan, on trouve : mX¨ = 0 d’où

X˙ = Cte = 0 et X = Cte

63

Dynamique du corpuscule b) Force centrale

Si la somme F passe par un pôle O, autrement dit si la somme des forces est centrale, le moment cinétique est un vecteur constant. En effet : d LO = OA × F = 0 dt

d’où

LO = OA × mv A = Cte

Il en résulte que la vitesse et le vecteur position sont constamment perpendiculaires au vecteur L O : v A · LO = 0 et

OA · L O = 0

Le mouvement de A est donc contenu dans un plan. V . 3 . — Méthode d’application de la loi fondamentale Pour appliquer la loi fondamentale de la dynamique en minimisant le risque d’erreur, il est préférable de procéder selon une démarche déterminée que nous allons illustrer sur l’exemple d’un point matériel lancé, à la surface de la Terre, avec une vitesse initiale v 0 (Fig. 4.2). y v0

Oz

g θ0 xl

x

F IG . 4.2.

 c Dunod – Toute reproduction non autorisée est un délit

a) Définition du système Définissons le système en le distinguant de son environnement par une surface fermée qui le contient. Ici le système est simple puisqu’il se réduit au corpuscule considéré. Notons que nous avons évité l’expression « Isolons le système », laquelle pourrait laisser supposer que l’on procède à une suppression des forces auxquelles le point matériel est soumis de la part de son environnement. b) Nature galiléenne du référentiel R Le référentiel d’analyse R est le référentiel terrestre. Comme nous le verrons ultérieurement (cf. chapitre 7), ce référentiel est une bonne réalisation d’un référentiel galiléen, pourvu que l’on introduise la notion de poids et que la vitesse ne soit pas trop grande afin de négliger la force de Coriolis terrestre. c) Bilan des forces Le poids mg (cf. chapitre 7) et la résistance de l’air de la forme : −a v avec a > 0 ; cette dernière est une force de frottement visqueux proportionnelle à la vitesse (cf. chapitres 10 et 31).

64

4. Dynamique du corpuscule

Remarque : Dans le bilan des forces, on doit prendre en compte les seules forces qui apparaissent dans la phase du mouvement considérée. Ainsi les forces éventuelles, qui sont à l’origine de la position et de la vitesse du point à l’instant initial, ne sont pas à prendre en compte. Les informations contenues à l’instant initial jouent cependant un rôle décisif, comme nous le verrons lorsque nous utiliserons les conditions initiales pour déterminer de façon précise la nature de la trajectoire. d) Expression vectorielle de la loi fondamentale L’application de la loi fondamentale donne : ma A/R = mg − av soit a A/R = g − v/t en introduisant le coefficient t = m/a, homogène à une durée. Notons que nous avons évité une écriture répandue, bien que malheureuse, constituant à écrire d’emblée la force de frottement sous la forme −mv/t , ce qui laisserait penser qu’elle serait proportionnelle à la masse, alors que cette proportionnalité est la propriété exceptionnelle de la force de gravitation (cf. chapitre 7). e) Projections sur une base Explicitons, dans une base orthonormée directe, généralement la plus commode (ici, celle liée à R), l’équation vectorielle précédente. Il vient, puisque l’axe Oy est vertical ascendant : 0 x ¨x 1 ˙ ¨y = −g − y˙ t ¨z 0 z˙

˙ =0 d’où les trois équations : ¨ x + ˙x/t = 0, ¨y + ˙y/t = −g et ¨ z + z/t f) Résolution des équations différentielles Les équations suivant x et z sont identiques. La première s’écrit : vx x˙ = 0 soit v˙ x + =0 t t En cherchant une solution de la forme vx = Cte × exp(rt), on trouve r + 1/t = 0. Par conséquent : t v x = v0 cos u0 exp − t car, pour t = 0, vx = v0 cos u 0. En intégrant, on obtient : t t + Cte = v0t cos u 0 1 − exp − x = −v 0t cos u0 exp − t t puisque, pour t = 0, x = 0. Comme à l’instant initial, on a : ˙z0 = 0 et z0 = 0, alors z = 0. Le mouvement est donc contenu dans le plan Oxy. Quant à l’équation en y , il suffit, pour la résoudre, d’ajouter à la solution générale précédente la solution particulière évidente : vy = −gt. Par conséquent : ¨x +

v y = Cte × exp −

t − gt t

À t = 0, vy = v 0 sin u0 = Cte − gt. D’où :

t − gt t En intégrant une nouvelle fois et en faisant y0 = 0, on obtient : v y = (v 0 sin u 0 + gt) exp −

y = −gtt + t(v0 sin u0 + gt) 1 − exp −

t t

65

Dynamique du corpuscule g) Analyse physique du résultat

Lorsque t devient très grand, v x et vy tendent vers des valeurs limites : (vx )l = 0 et (vy )l = −gt Quant à x et y, ils valent respectivement : (x) l = v 0t cos u 0 et (y)l = −gt t Enfin, si t  t, on retrouve les résultats bien connus : x ≈ v0 t cos u0 1 − 1 + et y ≈ −gt t + t(v 0 sin u 0 + gt) 1 − 1 +

t t

= v0 cos u0 t

t t2 − 2 t 2t

≈−

gt 2 + v 0 sin u0 t 2

La trajectoire peut changer considérablement d’allure, selon les conditions initiales : i) Dans le cas général, c’est une courbe, précisément une parabole si la force de frottement est nulle (a = 0 ou t = ∞.) ii) Pour v 0 = 0, u0 = p/2 ou u0 = −p/2, c’est une droite verticale.

Remarque : On aura noté la nécessité d’avoir deux types de constantes pour intégrer les équations différentielles issues de l’application de la loi fondamentale : les unes concernent la posi˙ Physiquement, cela correspond à la définition de tion (x, y), les autres la vitesse (˙x, y). l’état mécanique du point à l’instant initial, lequel exige la connaissance de la position (où est le point ?) et de la quantité de mouvement (où va-t-il ?). L’espace des états mécaniques d’une particule, ou espace des phases, est donc a priori à 6 dimensions (cf. chapitres 10 et 24).

VI . — TROISIÈME LOI DE NEWTON

 c Dunod – Toute reproduction non autorisée est un délit

La troisième loi de Newton, ou principe de l’opposition des actions réciproques entre deux corpuscules, permet de passer de la mécanique du corpuscule à celle des systèmes (cf. chapitre 13). Nous verrons plus tard (cf. chapitre 18) qu’il est possible de bâtir la mécanique des systèmes à partir d’une axiomatique plus forte qui restitue cette troisième loi sous forme de théorème. VI . 1 . — Énoncés historique et actuel Énoncé historique La réaction est toujours contraire à l’action, ou encore les actions que deux corps exercent l’un sur l’autre sont toujours égales (en norme) et dirigées en sens contraires. Énoncé actuel Si un corpuscule A1 exerce sur un autre corpuscule A 2 une force F1→2 , A2 exerce sur A1 la force opposée : F2→1 = −F 1→2 Exemple : Dans le cas de la gravitation, on a : F2→1 = −F1→2 = −

Gm ∗1m∗2 er avec r2

er =

r r

et r = A2 A1

66

4. Dynamique du corpuscule

VI . 2 . — Application à la comparaison des masses L’opposition des actions réciproques permet d’établir un résultat important relatif à un système de deux corpuscules soumis à leurs seules actions mutuelles. Des relations : d p1 = F2→1 et dt

d p2 = F 1→2 dt

on déduit :

d (p + p2 ) = 0 soit p1 + p2 = Cte dt 1 Donc, la quantité de mouvement d’un système isolé de deux corpuscules est une constante vectorielle. Ce résultat fournit une méthode de comparaison de masses. En effet, on peut écrire, relativement à R, la relation suivante entre deux instants t et t : p 1 + p2 = p1 + p2

soit

m1v1 + m2 v2 = m 1 v1 + m2v2

en utilisant les notations habituelles. On en déduit : m2 v − v1  = 1 m1 v2 − v2 

CONCLUSION (1) La loi fondamentale de la dynamique permet d’étudier tout problème de mécanique du corpuscule connaissant les forces qui s’exercent sur lui. Relativement à un référentiel galiléen, elle s’écrit : dp = dt

F

soit

ma =

F

puisque

p = mv

Son application fournit, dans le cas général, trois équations différentielles du deuxième ordre qu’il faut résoudre pour connaître le type de mouvement. La difficulté relève alors de la seule technique mathématique. Les constantes introduites par cette dernière résolution sont déterminées de manière non ambiguë par les conditions particulières du mouvement, le plus souvent initiales, qui portent sur la position et sur la vitesse. Une fois le mouvement déterminé, il convient de vérifier les prévisions théoriques en s’aidant de l’expérience ou à défaut de son intuition newtonienne. (2) Dans certains cas, il est commode d’appliquer le théorème du moment cinétique, lequel est directement issu de la loi fondamentale appliquée au corpuscule considéré : d LO = dt

MO

avec

LO = OA × p

L O et p étants relatifs au référentiel d’étude. (3) Rappelons que les dérivées par rapport au temps de la quantité de mouvement et du moment cinétique sont relatives au référentiel d’analyse R. Si ce n’est pas le cas, il faudra utiliser la composition des dérivations de Bour. (4) Le domaine d’application de la loi fondamentale est très large. Ses limitations sont définies par : i) les vitesses non négligeables devant c ≈ 3 × 10 8 m . s −1 (cf. Relativité et invariance), i) l’échelle atomique, 1 nm ∼ 10 −9 m, (cf. Quantique).

67

Dynamique du corpuscule

EXERCICES ET PROBLÈMES P4– 1. Mouvement d’une fusée Une fusée, assimilée à un corps ponctuel, de masse 1 200 t, est soumise à une force de propulsion de 32 kN et à une résistance de l’air atmosphérique égale à 9, 6 kN (Fig. 4.3). Sa vitesse est de 3 km . s −1 et le champ de gravitation G, à l’altitude considérée, est de 6 m . s −2 . L’angle que fait la vitesse avec ce champ est 30◦ . Calculer le rayon de courbure de la trajectoire. y

30◦

G O

x F IG . 4.3.

P4– 2. Influence de la taille d’un athlète au lancé du poids Un athlète, de hauteur h bras levé, lance le « poids » (masse égale à 7, 26 kg), le plus loin possible avec une vitesse initiale de valeur v0 fixée. 1. Établir la relation entre l’angle a m qui réalise le meilleur lancer, h et la coordonnée horizontale xm du point de chute. 2. En déduire l’expression de am en fonction de h, v0 et g, intensité du champ de pesanteur. 3. Étudier l’influence de h sur x m. Calculer x m pour : v0 = 13, 5 m . s −1 avec d’abord h = 2, 2 m, puis h = 2, 7 m.

 c Dunod – Toute reproduction non autorisée est un délit

P4– 3. Expérience de Millikan On observe le mouvement rectiligne et uniforme, suivant la verticale, d’une gouttelette de glycérine A, sphérique, de rayon r ; la gouttelette est soumise à son poids et à une force de frottement visqueux, donnée par la loi de Stokes (cf. chapitre 31) : Ff = −6p h r v



h = 1, 8 × 10 −5 SI

est la viscosité de l’air (cf. chapitre 31). En l’absence de champ électrique, A tombe à la vitesse vl = 0, 392 mm . s−1. En présence d’un champ électrique uniforme et constant E, de direction verticale, de norme E = 400 kV . m−1 , la gouttelette remonte. On donne la masse volumique de l’huile : r = 810 kg . m−3 . 1. Montrer que la vitesse suivant la verticale tend vers une vitesse limite en l’absence de E. On posera 1/t = 6phr/m. Calculer r. 2. Que devient la vitesse en présence de E ? 3. Une observation prolongée montre que la vitesse subit, par instant, des variations brusques. On mesure, en particulier, les valeurs suivantes exprimées en mm . s−1 : v1 = 0, 270

v 2 = 0, 080

v3 = 0, 175

v4 = 0, 363

v5 = 0, 458

68

4. Dynamique du corpuscule

Des mesures analogues ont permis au physicien américain R. Millikan de conclure, en 1911, à l’existence d’une charge élémentaire et de déterminer la valeur de cette dernière, ce qui lui valut le prix Nobel en 1923. Retrouver cette charge à partir des données précédentes. P4– 4. Mouvement d’un point soumis à une force résistante en v2 Une bille sphérique, de masse m = 30 g, de rayon r = 1 cm, est abandonnée dans le champ de pesanteur terrestre. Elle est soumise, de la part de l’air, à une force de freinage (la traînée), opposée à la vitesse, d’expression (cf. chapitre 31) : Ff = CxrSv 2/2, où S est la surface de la projection de la bille dans un plan perpendiculaire à la vitesse, r la masse volumique de l’air et Cx un facteur de résistance de l’air. 1. On introduit v l tel que mg = C x Srv2l /2. Écrire l’équation différentielle à laquelle satisfait la vitesse v. 2. Trouver comment évolue v. Quelle est la signification de v l ? Représenter graphiquement v(t). P4– 5. Mouvement d’un point soumis à une force résistante en v1/2 Une bille, de masse m, en mouvement suivant un axe horizontal Ox, est soumise à une force de frottement proportionnelle à la racine carrée de la vitesse : F = −b v 1/2 ex. Sa vitesse initiale est v 0 ex .

Quelle est l’équation différentielle à laquelle satisfait la vitesse v ? En déduire l’expression de v(t). À quel instant t1, en fonction de m, b et v0 , la vitesse s’annule-t-elle ? Représenter graphiquement v(t).

5 Énergétique du corpuscule Nous allons déduire de la loi fondamentale de la dynamique un théorème faisant intervenir des grandeurs scalaires, telles que la puissance d’une force, le travail, l’énergie cinétique, l’énergie potentielle et l’énergie mécanique d’un corpuscule. Le point de départ est la simple multiplication scalaire par le vecteur vitesse v des deux membres de l’égalité vectorielle qui exprime la deuxième loi de Newton : dp ·v= dt

F·v

soit

mv ·

dv = dt

F·v

I . — PUISSANCE ET TRAVAIL D’UNE FORCE Considérons, dans un référentiel R, un corpuscule A, de vitesse v, soumis à une force F (Fig. 5.1). z

v

A

dr F

 c Dunod – Toute reproduction non autorisée est un délit

O x

R

er

y O

F IG . 5.1.

A r

dr

F IG . 5.2.

I . 1 . — Puissance Par définition, la puissance d’une force F s’exerçant sur le corpuscule A, ou puissance reçue par A par l’intermédiaire de F, est : P = F·v

soit

P = Fx x˙ + F y y˙ + F z ˙z

en coordonnées cartésiennes. Comme la vitesse, la puissance d’une force F dépend du référentiel dans lequel elle a été calculée. Certaines forces ont une puissance nulle : i) la force magnétique, qui s’exerce sur une charge ponctuelle plongée dans un champ magnétique stationnaire B : P = q(v × B) · v = 0

70

5. Énergétique du corpuscule

ii) la force de Coriolis (cf. chapitre 7), qui s’exerce sur un corpuscule si le référentiel considéré n’est pas galiléen : P = −2m(V × v) · v = 0

Plus généralement, la puissance d’une force quelconque est nulle pour tout déplacement normal à la direction de cette force. Ainsi, la puissance du poids d’un corps est nulle pour tout déplacement dans un plan horizontal, puisque le poids est vertical (cf. chapitre 7). Si P > 0, la puissance est motrice ; si P < 0, elle est résistante. L’unité SI de puissance est le watt (W), du nom de l’ingénieur écossais J. Watt. L’ancienne unité, encore utilisée, le cheval-vapeur, vaut 736 W. Évidemment, si plusieurs forces sont appliquées au même corpuscule A, la puissance résultante est la somme des puissances de toutes les forces : P = (F 1 + F 2 + · · · ) · v = F 1 · v + F2 · v + · · · Remarque : Bien que relative à un référentiel, la puissance P = F · v peut être calculée à l’aide des composantes des vecteurs F et v dans n’importe quelle base vectorielle. Exemples i) Puissance du poids mg : P = mg · v = −mg ˙z, Oz étant l’axe vertical ascendant. Ainsi, la puissance du poids d’une masse de 1 kg, qui tombe en chute libre, avec une vitesse de 5 m . s −1 au niveau du sol, est : P = mg · v = mgv = 49 W ii) Puissance d’une force centrale (Fig. 5.2) Comme F = Fr e r où r = OA et e r = r/r, il vient : P = Fr

r dr dr = Fr = F rr˙ · dt r dt

car l’équation r2 = r2 donne r · d r + d r · r = 2r d r en différentiant, soit r · d r = r d r . iii) Puissance d’une force de frottement visqueux F = −a v avec a > 0 : P = −av · v = −av 2 < 0.

I . 2 . — Travail a) Expression élémentaire Par définition, le travail élémentaire de la force F qui s’exerce sur un corpuscule A est : dW = P d t = F · v d t = F · d OA puisque d OA = v d t est le déplacement élémentaire de A dans le référentiel R par rapport auquel la vitesse est comptée. La notation d indique que dW n’est pas a priori la différentielle d’une certaine fonction. Exprimons dW à l’aide de la base de R : dW = F x d x + Fy d y + F z d z En raison de la relation simple existant entre dW et P, les remarques faites sur la puissance sont valables aussi pour dW. L’unité SI de travail est le joule (J), du nom du physicien anglais J. Joule. Remarque : La lettre W, utilisée pour désigner le travail, vient du mot anglais work.

71

Énergétique du corpuscule b) Travail au cours d’un déplacement fini Le travail de F au cours d’un déplacement fini s’obtient en intégrant l’expression précédente : F x d x + F y d y + Fz d z

W=

i) Dans le cas général et peu fréquent où F dépend à la fois de la position du point A, de sa vitesse et du temps, l’intégrale n’est calculable qu’une fois x, y et z connus en fonction du temps, et donc qu’une fois le mouvement connu. On a alors, entre ti et t f les instants initial et final : tf

W= ti

[Fx (r, v, t) x˙ + Fy (r, v, t) ˙y + Fz(r, v, t) ˙z] d t

Exemple : Soit, en unité SI, F tel que F x = 3 x + ˙x t/2 + 1, Fy = −5 z + 2 t2 + ˙y t, Fz = 10 x + 3, avec x = t 2 + 1, y = 2t2, z = t 2 . Le travail, entre les instants t 1 = 0 s et t 2 = 1 s, se calcule en exprimant toutes les quantités en fonction du temps : 1

W= 0

2

3

1

2

[8 t(t + 1) − 12 t + 2t(10t + 13)] d t =

(16 t 3 + 34 t) d t = 4 t 4 + 17 t 2

0

1 0

= 21 J

ii) Si la force ne dépend que de la position r , le travail se présente sous la forme d’une intégrale curviligne que l’on calcule le long de la trajectoire C du point d’application de la force, entre ses positions initiale et finale Ai et A f : W= C

F(r) · d OA =

Fx(x, y, z) d x + Fy (x, y, z) d y + F z(x, y, z) d z C

Le travail est ici défini pour un parcours déterminé indépendamment de la manière dont ce parcours est effectué au cours du temps. Exemple : Soit F, tel que F x = 3 x, F y = −5 z, Fz = 10 x, en unités SI, et la trajectoire C définie par y = 2x 2 et z = 0. Entre x = 0 et x = 1, on obtient, puisque z = 0 : 1

W= 0

3x dx =

3x2 2

1

= 1, 5 J 0

 c Dunod – Toute reproduction non autorisée est un délit

iii) Un troisième cas encore plus particulier présente un intérêt majeur, puisqu’il est à l’origine du concept d’énergie potentielle associée à une force. Il sera étudié un peu plus loin. Là, le travail ne dépend pas de la trajectoire entre les positions Ai et A f , mais seulement de ses positions initiale et finale.

II . — ÉNERGIE CINÉTIQUE II . 1 . — Définition Le concept d’énergie cinétique s’introduit naturellement lorsqu’on effectue le produit scalaire F · v. En effet : d 1 2 dp dv mv + Cte F·v = ·v= m ·v= dt dt dt 2 Ainsi, ce produit scalaire se met sous la forme de la différentielle de la fonction mv 2 /2 + Cte. Par convention, la constante est choisie égale à 0 et on appelle énergie cinétique Ek/R , par rapport au référentiel R , la quantité scalaire non négative suivante, liée à la vitesse : Ek =

1 2 mv 2

72

5. Énergétique du corpuscule

Remarques : (1) Nous avons noté l’énergie cinétique par la lettre cursive E k et non par la lettre droite E k afin de distinguer l’énergie cinétique de la norme du champ électrique E. On pourra utiliser cette dernière lettre lorsqu’il n’y aura aucun conflit de notation. (2) Le mot énergie, qui vient de energia, mot grec signifiant force en action, a été introduit par T. Young (cf. Thermodynamique) en 1802. (3) Le double de l’énergie cinétique, mv 2 , a été introduit par Leibniz, en 1691, sous le nom de « force vive », à l’époque où la différence entre force et énergie n’était pas encore clarifiée. II . 2 . — Théorème de l’énergie cinétique Il vient, d’après la loi fondamentale : d Ek/R =P dt

soit

d Ek/R = dW

ou

DEk/R = W

après intégration. Ainsi : La dérivée par rapport au temps de l’énergie cinétique d’un corpuscule est égale à la puissance de toutes les forces appliquées, ou la variation d’énergie cinétique d’un corpuscule est égale au travail de toutes les forces appliquées. Remarques : (1) Comme nous le verrons au chapitre 7, si R n’est pas galiléen, il faut ajouter à la puissance des forces appliquées celle de la force d’inertie d’entraînement, celle de la force de Coriolis étant toujours nulle. (2) Le théorème de l’énergie cinétique est particulièrement adapté aux systèmes dont la position est définie par un seul degré de liberté, puisqu’il ne fournit qu’une seule équation scalaire. (3) En multipliant scalairement par v les deux membres de l’équation vectorielle qui exprime la loi fondamentale de la dynamique, on introduit naturellement la solution supplémentaire v = 0 qui n’existait pas nécessairement. Aussi cette solution ne présente-t-elle aucun intérêt. II . 3 . — Exemple d’application : le pendule simple Une masselotte A, de masse m, glisse sans frottement sur un guide circulaire vertical, de rayon l (Fig. 5.3). On repère sa position par l’angle u que fait OA avec la verticale descendante, O étant l’origine du référentiel terrestre R . g

y

Oz l θ

Guide circulaire

R x

A F IG . 5.3.

En vue d’appliquer le théorème précédent dans le référentiel d’analyse R = Oxyz, exprimons, en fonction du paramètre u, l’énergie cinétique et la puissance des différentes forces : Ek/R =

1 2˙2 ml u 2

Pmg = mg · v = −mglu˙ sin u

et P R = R · v = 0

73

Énergétique du corpuscule

puisque la réaction R du guide est, en l’absence de frottement, perpendiculaire à la tangente au guide (cf. chapitre 9). Le théorème de l’énergie cinétique donne donc : d(ml 2u˙ 2/2) = −mglu˙ sin u dt

ml 2u˙ ¨ u = −mglu˙ sin u

soit

et

g ¨ u + sin u = 0 l

˙ ce qui revient à exclure la solution parasite u ˙ = 0. La résolution de cette en simplifiant par ml2 u, équation n’est simple que dans le cas des petits mouvements ; on trouve alors, puisque sin u ≈ u : g ¨ u + v20 u = 0 avec v20 = l ce qui est caractéristique d’un mouvement sinusoïdal de période T0 = 2p/v 0 = 2p(l/g) 1/2 . Remarque : L’équation du mouvement peut être obtenue aussi à l’aide du théorème du moment cinétique, appliqué au point fixe O par lequel passe la réaction R. Il vient, le moment en O de cette force étant nul : d(ml2 u˙ ez) = OA × mg = −mgl sin u e z dt

III . — ÉNERGIE POTENTIELLE III . 1 . — Définition Il arrive fréquemment que le travail d’une force F, qui s’exerce sur un corpuscule A, ne dépende pas du chemin suivi par son point d’application A, mais seulement des positions initiale A i et finale Af de ce point. C’est le cas de toutes les forces fondamentales de la physique ! On écrit alors : W = Ep (ri ) − E p(rf )

 c Dunod – Toute reproduction non autorisée est un délit

où E p(r), avec r = OA, est une fonction de position, appelée l’énergie potentielle, selon la recommandation faite par l’ingénieur écossais J. Rankine (cf. Thermodynamique). Le travail élémentaire d’une telle force se met alors sous la forme de l’opposé de la différentielle (totale, exacte) de la fonction énergie potentielle : dW = F · d r = − d Ep (r) En coordonnées cartésiennes, d E p a pour expression, si Fx , Fy et F z sont les composantes de F : − d E p = F x d x + Fy d y + Fz d z

d’où

Fx = −

∂Ep ∂x

Fy = −

∂Ep ∂y

Fz = −

∂Ep ∂z

On condense souvent les trois équations précédentes en introduisant l’opérateur vectoriel grad appelé gradient (cf. annexe 3) : F = − grad E p L’énergie potentielle Ep est définie à une constante additive près sans intérêt. Dans les cas réels d’interaction du corpuscule A avec le système S qui exerce la force, l’origine naturelle de l’énergie potentielle est prise lorsque A et S sont infiniment éloignés.

74

5. Énergétique du corpuscule

Dans les modèles où il est souvent commode de considérer la force comme un vecteur uniforme, la constante additive ne peut être prise égale à 0 à l’infini. Aussi, dans ce cas, prend-on comme origine un point arbitraire. Remarques : (1) Le concept d’énergie potentielle, tel qu’il a été défini ici comme fonction explicite des seules variables de position, trouve sa pleine justification physique dans l’existence d’une grandeur, l’énergie mécanique, qui se conserve si le corps ponctuel n’est soumis qu’à la force associée à cette énergie potentielle. C’est précisement parce qu’on voulait faire apparaître l’énergie mécanique sous la forme d’une somme de contributions qu’on a préféré la forme dW = − d E p(r) avec le signe moins. (2) Compte tenu du choix de la constante, l’énergie potentielle peut être positive ou négative, alors que la convention admise pour l’énergie cinétique rend cette dernière non négative, une fois pour toutes.

III . 2 . — Définition opérationnelle de l’énergie potentielle On donne souvent une autre définition de l’énergie potentielle en faisant intervenir un opérateur. Ce dernier exerce la force Fop sur le point A, lequel est soumis en outre, de la part d’un système S, à la force F qui dérive de l’énergie potentielle Ep et à une force occasionnelle de frottement.

Appliquons le théorème de l’énergie cinétique à A, entre une position infiniment éloignée de S et la position considérée. Il vient, en adoptant comme origine des énergies potentielles celle correspondant à A et S infiniment éloignés : DEk = W (F) + Wop + W f = −DE p + W op + Wf = −(E p − 0) + Wop + Wf d’où : Ep = −DEk + W op + Wf

Wf désignant le travail des forces de frottement éventuelles. Ainsi : Ep = Wop

si

DEk = 0 et W f = 0

Pour que W f soit nul, il faut que la vitesse de A soit très faible et qu’il n’y ait pas de frottement solide (cf. chapitre 9). L’énergie potentielle associée à la force qu’exerce le système S sur A est donc aussi le travail que doit fournir un opérateur pour amener A de l’infini à la position considérée, au cours d’un déplacement sans frottement et sans variation d’énergie cinétique. En pratique, ces conditions sont réalisées si la norme de la force F op est infiniment voisine de celle de F et s’il n’y a pas de frottement solide. III . 3 . — Exemples d’énergie potentielle a) Énergie potentielle de pesanteur Dans le cas de la force de pesanteur qui est mg, on a (Fig. 5.4) : dW = mg · d r = d(mg · r) = − d E p

d’où

Ep = −mg · r + Cte

Si conventionnellement, Ep est nulle pour r = 0, la constante est nulle et : Ep = −mg · r

75

Énergétique du corpuscule

En général, on retient l’expression précédente sous sa forme explicitée dans la base cartésienne dans laquelle l’axe Oz désigne la verticale ascendante (cf. chapitre 7), c’est-à-dire l’axe dirigé et orienté selon −g. Il vient, puisque g = −g ez avec g = 9, 80 m . s−2 et r a pour composantes x, y, z : E p = mg ez · r soit Ep = mgz. L’énergie potentielle de pesanteur est positive si le point est au-dessus de O et négative sinon, ce qui est conforme à l’augmentation de l’énergie potentielle lorsque la distance qui sépare le point A de la Terre augmente. Dans l’exemple du pendule simple (Fig. 5.3), l’énergie potentielle de pesanteur a pour expression : Ep = mgz = −mgl cos u. z g

A mg

O

g

y

A O

x

x F IG . 5.4.

F IG . 5.5.

Remarque : Il n’est pas inutile de préciser que, si la verticale avait été prise descendante, on aurait eu Ep = −mgz. b) Énergie potentielle élastique due à un ressort de raideur K et de longueur à vide l0 Calculons le travail élémentaire de la force −K (x − l 0 ) ex (Fig. 5.5) : dW = −K (x − l0 ) d x = − d Ep

d’où

1 E p = K (x − l 0)2 + Cte 2

Si l’on convient que l’énergie potentielle est nulle pour x = l 0, alors Cte = 0 d’où, en introduisant l’allongement X = x − l 0 : 1 1 Ep = K (x − l0 )2 = KX2 2 2

 c Dunod – Toute reproduction non autorisée est un délit

c) Énergie potentielle newtonienne ou coulombienne Cette énergie potentielle est celle associée aux forces newtonienne ou coulombienne qui sont de la forme (Fig. 5.6), K étant la constante de l’interaction : OA K F = 2 er où er = r r Si K > 0, l’interaction est répulsive ; c’est le cas de la force de Coulomb entre des charges électriques de mêmes signes (cf. Électromagnétisme). En revanche, si K < 0, l’interaction est attractive ; il en est ainsi pour la force entre les charges électriques de signes opposés et la force de gravitation. Calculons le travail élémentaire de cette force. Il vient, puisque r · d r = r d r : dW = F ·d r =

Ke r K K K ·d r = 3 r ·d r = 3 r d r = 2 d r = d 2 r r r r

−K r

= − d Ep

Conventionnellement, on pose Ep(r) = 0 à l’infini ; alors, Cte = 0 et : Ep =

K r

avec

Ep (r) =

K +Cte r

76

5. Énergétique du corpuscule z z H ω

z O

er

A

F

x

y

R

x

A

y

O ωt

F IG . 5.6.

x

P R

y

F IG . 5.7.

d) Énergie potentielle centrifuge Si le référentiel d’analyse R  n’est pas galiléen et qu’il tourne uniformément autour d’un axe fixe d’un référentiel galiléen R (Fig. 5.7), on montre qu’il apparaît deux forces supplémentaires, la force d’inertie de Coriolis qui ne travaille pas et la force d’inertie d’entraînement centrifuge d’expression : mV2 HA, V étant le vecteur vitesse de rotation constant de R  par rapport à R (cf. chapitre 7). Le travail de cette dernière s’écrit : dW = mV2 HA · d OA = mV 2HA · d HA = − d Ep

avec E p = −

mV 2 2 HA + Cte 2

Si Ep est nulle sur l’axe, alors Cte = 0 et : Ep = −

mV2 2 HA 2

ce qui s’écrit aussi

Ep = −

m 2 (V × r) 2

puisque HA 2 = (e z × r)2 . Notons que cette expression suppose que la rotation soit uniforme. Remarque : La puissance de la force magnétique étant nulle, l’énergie potentielle associée peut être prise égale à 0. Il en est de même pour la force de Coriolis (cf. chapitre 7).

IV . — ÉNERGIE MÉCANIQUE Distinguons, dans les forces qui s’exercent sur un corps ponctuel A, celles F (c) qui dérivent d’une énergie potentielle Ep de celles F(nc) qui n’en dérivent pas. Le travail élémentaire de toutes les forces peut alors se mettre sous la forme : dW = dW (c) + dW (nc) = − d E p + dW(nc) dW (c) = − d Ep étant le travail des forces, dites conservatives, qui dérivent d’une énergie potentielle et dW(nc) le travail des forces, dites non conservatives, qui ne dérivent pas d’une énergie potentielle. On obtient, en appliquant le théorème de l’énergie cinétique : d E k = − d E p + dW (nc)

soit

d(Ek + E p) = dW (nc)

77

Énergétique du corpuscule IV . 1 . — Théorème de l’énergie mécanique

Le résultat précédent, issu du théorème de l’énergie cinétique, suggère d’introduire le concept d’énergie mécanique Em d’un corpuscule, somme de son énergie cinétique E k et de son énergie potentielle Ep : Em = Ek + E p

Il en résulte le théorème de l’énergie mécanique : d Em = P(nc) dt La dérivée par rapport au temps de l’énergie mécanique est égale à la puissance des forces qui ne dérivent pas d’une énergie potentielle. Ce théorème s’écrit aussi : d Em = dW (nc) d’où

DE m = W (nc)

en intégrant ; dW(nc) = P (nc) d t est le travail élémentaire correspondant et W (nc) le travail total associé.

 c Dunod – Toute reproduction non autorisée est un délit

Remarques : (1) On notera la différence entre énergie et travail : le premier concept est une fonction d’état, caractéristique de l’état du système, contrairement au second : la fonction travail W (nc) n’existe pas. (2) L’énergie potentielle E p caractérise l’interaction du corpuscule A avec le système S qui exerce la force F. Ce concept ne doit donc pas être associé au seul corpuscule A mais au système {S, A}. C’est probablement pour cette raison que certains auteurs n’associent une énergie potentielle qu’aux seules forces intérieures d’un système qui dérivent d’une énergie potentielle. Le point de vue adopté ici est moins restrictif : il s’appuie sur la seule justification du concept d’énergie qui est la recherche d’une grandeur dite conservative, c’est-à-dire qui ne peut être créée. Cette démarche historique s’est achevée avec le premier principe de la thermodynamique (cf. Thermodynamique). (3) On comprend pourquoi les physiciens ont préféré le concept d’énergie potentielle à celui de fonction de force U = −Ep introduit par les mathématiciens : l’énergie mécanique se met sous la forme d’une somme de deux contributions qui s’ajoutent. IV . 2 . — Cas où l’énergie mécanique se conserve De façon générale, l’énergie mécanique ne se conserve pas, puisque P (nc) = 0. Si, occasionnellement, les seules forces qui travaillent dérivent d’une énergie potentielle, P (nc) = 0, l’énergie mécanique se conserve : E m = Ek + E p = Cte

si

P(nc) = 0

La constante est déterminée par les conditions initiales du mouvement. Cette conservation occasionnelle de l’énergie mécanique fournit une équation différentielle du premier ordre, contrairement au théorème de l’énergie cinétique. Pour cette raison, on l’appelle parfois l’intégrale première de l’énergie. Remarque : Les forces qui dérivent d’une énergie potentielle sont appelées forces conservatives. On comprend désormais pourquoi : lorsqu’elles interviennent seules, l’énergie du corpuscule se conserve.

78

5. Énergétique du corpuscule

IV . 3 . — Discussion qualitative d’un mouvement conservatif à un degré de liberté L’équation de conservation de l’énergie mécanique d’un corpuscule, soumis uniquement à des forces conservatives qui travaillent, s’avère très intéressante dans la résolution des problèmes à un degré de liberté, notamment lorsque l’intégration des équations différentielles est délicate. En effet, cette équation se prête bien à une discussion qualitative souvent suffisante. Comme Em = E k + E p = Cte, les seuls mouvements possibles sont ceux pour lesquels : Ek = Em − Ep  0 On représente alors graphiquement Ep(q) en fonction du seul degré de liberté q (Fig. 5.8) ; les mouvements possibles sont ceux qui réalisent Em  Ep . Ep Em

État libre

O q1 E 0

q 2 q0

q3

q 0

q4 q

État lié

E0

État libre

F IG . 5.8.

Supposons que E p (q) ait l’allure représentée sur la figure 5.8 : pour q = q0, l’énergie potentielle passe par une valeur minimale E0 et, pour q = q0, elle passe par une valeur maximale E 0 . On distingue plusieurs types de mouvement : i) E m  E0 ; les seuls mouvements possibles sont ceux pour lesquels q > q1 , q1 étant défini par E m = Ep . Comme le corpuscule peut s’éloigner indéfiniment du centre de force, cet état est dit libre. ii) E 0 < Em < E0 ; suivant la valeur initiale q i de q, le mouvement est tel que q2  q  q 3 ou q > q4 . Dans le premier cas, l’état est dit lié ou confiné car le corpuscule reste toujours dans une région déterminée de l’espace. Dans le second, il est libre.

iii) E m = E 0 ou Em = E0 . Il en résulte que q = q 0 ou q = q0 ; le mouvement est stationnaire en q. L’état est ici aussi lié. On peut montrer que le premier mouvement stationnaire est stable, alors que le second est instable : dans le premier cas, toute légère perturbation du corpuscule provoque un mouvement d’oscillation autour de la position d’équilibre q0 ; dans le second, la perturbation provoque une accentuation de l’écart par rapport à q0. En effet, un développement autour de q 0 donne : Ep (q) = Ep(q0 ) +

q − q0 1!

d Ep dq

+ 0

(q − q 0)2 2!

en remarquant que (d Ep / d q) 0 = 0 et en posant : mouvement est donc, en première approximation :

d2 Ep d q2

0

+ · · · = E0 +

d 2 Ep/ d q2

0

K (q − q0 )2 + ··· 2

= K. L’équation différentielle du

1 2 1 A q˙ + E0 + K (q − q0 )2 = E m 2 2 Deux cas se présentent suivant le signe de K. i) K > 0. L’équation traduit l’oscillation de q autour de la valeur d’équilibre q 0 , avec la pulsation v0 = (K /m) 1/2 (cf. chapitre 10). En effet, en dérivant cette équation par rapport au temps, on obtient

79

Énergétique du corpuscule bien l’équation différentielle du deuxième ordre caractéristique : d2 Q + v 20Q = 0 avec Q = q − q 0 d t2

et v 0 =

K m

1/2

Ainsi, l’équilibre est stable si l’énergie potentielle est minimale. ii) K < 0. L’équation différentielle à laquelle satisfait Q = q − q 0 est : d2 Q − a2 Q = 0 d t2

avec

a=

|K | m

1/2

ce qui est caractéristique d’une augmentation de Q au cours du temps. L’équilibre est donc instable si l’énergie potentielle est maximale. Remarques : (1) Cette discussion était autrefois conduite à partir de l’équation q˙ 2 = f (q). La forme canonique qui s’appuie sur le concept fondamental d’énergie mécanique a évidemment la faveur des physiciens. (2) Notons que cette discussion qualitative, bien que très intéressante, reste insuffisante pour analyser l’état mécanique d’un corpuscule, lequel est déterminé par l’ensemble du degré de liberté q, qui donne sa position, et de sa dérivée q˙ ; précisément, l’état est défini par q et son moment conjugué p, proportionnel à q˙ (cf. chapitre 24).

CONCLUSION 1) Rappelons les différentes formes du théorème de l’énergie cinétique :

 c Dunod – Toute reproduction non autorisée est un délit

d Ek =P dt

d Ek = P d t = dW

et

DEk = W

2) Il est préférable d’écrire ce théorème sous des formes à la fois plus générales et plus efficaces en faisant apparaître l’énergie mécanique Em , somme de l’énergie mécanique Ek et de l’énergie potentielle Ep : d Em = P (nc) d Em = P (nc) d t = dW (nc) et DEm = W (nc) dt ( nc ) ( nc ) P et W étant la puissance et le travail des forces qui ne dérivent pas d’une énergie potentielle. (3) On détermine l’énergie potentielle associée à une force en calculant son travail élémentaire dW . Si, entre deux positions extrêmes Ai et Af , le travail W ne dépend pas du chemin suivi, alors, on peut écrire W = Ep(A i) − E p(A f). En d’autres termes, si dW est une différentielle (totale, exacte), on pose dW = − d Ep . (4) L’application du théorème de l’énergie pour un corpuscule présente un intérêt multiple : i) on manipule des quantités scalaires et non des vecteurs, lesquels exigent des bases d’expression, ii) si les seules forces qui travaillent dérivent d’une énergie potentielle, l’énergie mécanique se conserve. Cette conservation de l’énergie mécanique se prête bien à une discussion qualitative très instructive et très précieuse, notamment quand le système ne possède qu’un seul degré de liberté et que l’intégration s’avère techniquement délicate, voire impossible. Aux chapitres 13 et 20, nous généraliserons ces résultats au cas des systèmes de points et de solides. Le premier principe de la thermodynamique élargit ce concept en introduisant celui d’énergie totale d’un système (cf. Thermodynamique).

80

5. Énergétique du corpuscule

EXERCICES ET PROBLÈMES P5– 1. Forces et énergies potentielles On considère les trois champs de force suivants : F1 = ay e x

F 2 = ax e x + by e y et F3 = a(y ex + x e y)

a et b étant des constantes, et (x, y) les composantes du vecteur position dans un plan Oxy. Ces champs de forces dérivent-ils d’une énergie potentielle ? Si oui, laquelle ? P5– 2. Conservation de l’énergie dans un puits d’énergie potentielle radiale Une particule A, de masse m, est soumise à une force centrale de la forme F = (K/r n) e r, où K et n sont des constantes et e r le vecteur unitaire porté par le vecteur position r. Elle est initialement immobile pour r = r0 et sa vitesse en r = 0 est v 0. 1. Donner l’expression de son énergie potentielle. 2. Montrer, à partir de l’énergie, que l’on doit avoir n < 1 et K < 0. P5– 3. Mouvement d’une particule dans un champ de force en r −3 Une particule A (masse m) est soumise de la part d’un centre O à une force F = (K/r 3) er , K étant une constante, r = OA et er = r/r. 1. Donner l’expression de l’énergie potentielle E p(r), sachant que E p est nul pour r infini. 2. En calculant le moment cinétique en O, montrer que le mouvement est plan. On note Oxy ce plan. Exprimer en coordonnées polaires (r, w) le moment cinétique et l’énergie cinétique. 3. Montrer que le mouvement suivant r satisfait à l’équation différentielle : 1 2 m ˙r + Ep,ef = Em 2 Em étant l’énergie mécanique de A et Ep,ef un terme énergétique que l’on écrira sous la forme E p,ef = K  /r2, K étant une constante que l’on déterminera en fonction de m, K et du carré du moment cinétique. 4. Les conditions initiales sont r 0 = 0 et ˙r0 = 0. Pour K  = 0, quelle est la variation de r en fonction du temps ? 5. On considère le cas général où K  = 0. Exprimer, en fonction de s = r2, l’équation différentielle précédente. En déduire l’équation différentielle du deuxième ordre à laquelle satisfait s. Quelle est alors la relation entre r et t en fonction de Em , m, r0 et r˙0 ? 6. Représenter graphiquement r en fonction de t pour ˙r0 = 0 dans les deux états suivants : l’état lié défini par Em < 0 et l’état libre défini par Em > 0. Trouver, en fonction de m, r 0 et E , la durée au bout de laquelle A dans l’état lié atteint le point O. P5– 4. Mouvement d’une particule soumise à une force de puissance constante Une particule A (masse m) a un mouvement rectiligne suivant un axe horizontal Ox. Elle est soumise à l’action d’une force de puissance constante P et à une force due à la résistance de l’air dont la norme a pour expression bmv 2. Elle part du repos en x = 0 pour t = 0 dans le sens des x positifs. 1. Trouver l’expression de l’abscisse x en fonction de la vitesse v. Montrer que la vitesse tend vers une limite vl que l’on déterminera en fonction de P, b et m.

81

Énergétique du corpuscule

2. Étudier la limite de x lorsque b tend vers 0. 3. Application numérique : m = 900 kg, P = 60 kW et v l = 144 km . h−1 . Quelle est la valeur de b ? Au bout de quelle distance, A aura-t-il atteint la vitesse vl /2 ? P5– 5. Analyse énergétique d’un oscillateur harmonique dans le champ de pesanteur terrestre Un oscillateur mécanique est constitué d’une masselotte A, de masse m, accrochée à l’extrémité inférieure d’un ressort vertical, de raideur K et de longueur à vide l0. On néglige les forces de frottement. 1. Appliquer le théorème de l’énergie mécanique. On désignera par x la coordonnée fixant la position de A, le long de la verticale descendante, à partir de l’origine O prise à l’extrémité supérieure du ressort. 2. Exprimer l’énergie potentielle en fonction de la variable X = x − l 1 , où l1 est la position d’équilibre de A. P5– 6. Aspect énergétique de la force de frottement de Stokes Une particule (masse m) est soumise à l’action d’une force dérivant de l’énergie potentielle E p et de la force de frottement de Stokes FS = −av (cf. chapitre 31). 1. Établir la relation entre l’énergie cinétique, l’énergie potentielle et l’énergie D dissipée par la force de frottement, qui est l’opposée du travail de cette force.

2. La particule est une fine gouttelette d’eau, de masse m = 0, 9 × 10 −15 kg, abandonnée (sans vitesse initiale). Le coefficient de la force de frottement est a = 2 × 10−10 N . s . m−1 . a) Montrer que la particule atteint pratiquement une vitesse limite v l au bout d’une faible durée que l’on estimera. Calculer cette vitesse limite en m . s −1. b) Quelle est la perte d’énergie D de cette fine gouttelette d’eau lorsqu’elle est descendue d’une hauteur h = 1 km ? P5– 7. Fronde de longueur variable

 c Dunod – Toute reproduction non autorisée est un délit

La trajectoire d’une fronde, constituée d’une masselotte A (masse m = 100 g) attachée à un fil de longueur l , est un cercle horizontal, de centre O et de rayon l0 = 1 m , parcouru avec une vitesse uniforme v 0 ; le fil de la fronde passe dans un tube vertical de très faible diamètre, placé en O (Fig. 5.9). 1. a) La fronde effectue 5 tours par seconde. Comparer la tension du fil au poids de A . Commenter. b) Déterminer les valeurs de la vitesse, du moment cinétique en O et de l’énergie cinétique de A . 2. On fait varier l en tirant sur son extrémité inférieure qui émerge du tube. a) Que deviennent le moment cinétique et l’énergie cinétique ? b) Calculer la vitesse de A et son énergie cinétique lorsque la longueur du fil de la fronde est réduite de moitié. En déduire le travail de la force de traction en appliquant le théorème de l’énergie cinétique. Retrouver ce résultat à partir de la tension. O

l

A T

Tube vertical

g E

F op

F IG . 5.9.

6 Gravitation. Propriétés du champ de gravitation La force de gravitation est l’une des quatre forces fondamentales de la physique. Elle fut introduite en 1687 par Newton pour interpréter le mouvement des planètes (cf. chapitre 12), le mouvement de la Lune et le mouvement des corps dans le voisinage de la Terre. Malgré le formidable succès de cette interprétation, le sujet reste d’actualité en raison du rôle décisif joué par la gravitation en astrophysique, notamment depuis l’analyse de cette interaction faite par Einstein en 1916 dans le cadre de la relativité générale. Traditionnellement, on présente la gravitation en soulignant la ressemblance entre la force gravitationnelle et la force électrostatique entre deux particules chargées. C’est ce que nous ferons au cours du développement en nous appuyant sur les résultats établis en électrostatique (cf. Électromagnétisme).

I . — FORCE DE GRAVITATION Entre deux corpuscules A 1 et A 2, de masses inertes respectives m1 et m2, la force de gravitation qu’exerce A2 sur A 1 a pour expression : F(2g→) 1 = −G

m ∗1m ∗2 er r2



er =

r et r = A 2 A1 = r 1 − r 2 r

La constante de gravitation G vaut : G ≈ 6, 67 × 10 −11 SI (Fig. 6.1) ; sa valeur a été testée avec une précision relative de 10 −4. Les quantités scalaires m ∗1 et m ∗2 , qui traduisent la capacité de deux masses à s’attirer, sont les masses de gravitation ou masses graves. On montre (cf. chapitre 7) que les masses graves sont égales aux masses inertes m 1 et m 2, lesquelles, rappelons-le, traduisent une autre propriété, celle qu’ont les corps de résister à leur mise en mouvement par des forces (cf. chapitre 4). La force électrostatique, établie expérimentalement par le physicien français C. Coulomb en 1785, a une forme analogue ; une particule, de charge électrique q2 , exerce sur une seconde particule de charge q 1 la force (cf. Électromagnétisme) : ) F(2e→ 1 =

1 q1 q2 er 4pε 0 r 2

avec

1 ≈ 9 × 10 9 SI 4pe0

Les constantes G et 1/(4pe0) donnent une idée du rapport considérable de ces forces : la force électrostatique est, en ordre de grandeur, 1039 fois plus intense que la force de gravitation.

83

Gravitation. Propriétés du champ de gravitation

Notons une autre différence : alors que la force électrostatique peut être attractive ou répulsive, la force gravitationnelle est uniquement attractive. En raison de la même dépendance par rapport à r, ces forces présentent des propriétés identiques. Nous établissons ces propriétés dans le cas de la gravitation, celles du champ électrostatique étant bien connues (cf. Électromagnétisme). Pour retrouver ces dernières, il suffira de remplacer les masses par les charges électriques et la constante G par −1/(4pε0 ). Remarques : (1) Souvent le vecteur r est défini autrement : r = A1A 2 = r2 − r1 , ce qui ne change rien à l’analyse. (2) Les limites d’application de la forme en 1/r 2 de la force de gravitation concernent les faibles distances (inférieures à 50 mm ). z

z A2 r

r2

r1

O

Pi

A1

Ri

ri

y

r

O

x

A

y x F IG . 6.1.

F IG . 6.2.

II . — CHAMP ET POTENTIEL DE GRAVITATION Soit un système déformable S d de corpuscules {Pi } de masses graves {m ∗i } et de positions {ri = OPi }. Ce système exerce sur un autre corpuscule A, de masse grave m∗ , repéré par r = OA , une force F égale à la somme vectorielle des forces exercées par chacun des éléments Pi (Fig. 6.2) : F=  c Dunod – Toute reproduction non autorisée est un délit

i

FP i→A = −Gm∗

i

m ∗i Pi A2

P iA Pi A

= −Gm ∗

i

m∗i R 2i

Ri Ri

avec

R i = P iA = r − r i

II . 1 . — Champ de gravitation G Par définition, le champ de gravitation G produit par l’ensemble S des points matériels est la force de gravitation qu’exerce S sur A par unité de masse : G=

F = −G m∗

i

m ∗i P iA 2

Pi A = −G Pi A

i

m∗i R2i

Ri Ri

Si la distribution de masse est continue, on introduit la masse grave r ∗ d de l’élément de volume d entourant un point générique P, r ∗ étant la masse grave volumique. D’où l’intégrale sur le volume occupé par les masses : PA ∗ R ∗ r d = −G r d G = −G 3 3 V PA V R

84

6. Gravitation. Propriétés du champ de gravitation

II . 2 . — Potentiel de gravitation F Évaluons le travail élémentaire dW de la force F exercée sur le corpuscule A par l’ensemble {Pi } fixe, lorsque A se déplace de d r. Comme les vecteurs r i sont fixes : d r = d Ri

et dW = F · d r = F · d R i = −Gm∗

i

Ri Ri

m∗i R2i

· d R i = −m ∗G

i

m∗i d Ri R2i

puisque R i · d Ri = Ri d Ri . Il en résulte que : dW = −m ∗ G

1 m∗i d − Ri

i

=d

m∗ G i

m ∗i Ri

avec

R i = PiA

Ainsi, dW se présente sous la forme d’une différentielle (totale, exacte) : la force F dérive donc d’une énergie potentielle de gravitation Ep : dW = − d Ep

avec

Ep = −m∗ G

m∗i + Cte Ri

i

Par définition, le potentiel de gravitation F est l’énergie potentielle par unité de masse grave : F=

Ep = −G m∗

i

m ∗i + Cte Ri

Comme E p, F est défini à une constante additive près, que l’on fixe en choisissant une origine des potentiels. Il est naturel de poser Ep = 0 pour des masses infiniment éloignées les unes des autres ; la constante est alors nulle : F = −G

i

m ∗i Pi A

ou

F = −G

V

r∗ d PA

si la distribution de masse est continue, P étant un point courant de la distribution.

III . — PROPRIÉTÉS DU CHAMP DE GRAVITATION Les propriétés fondamentales du champ de gravitation sont au nombre de deux : la première exprime la relation entre le champ G et le potentiel F, la seconde la relation entre le G et les masses graves qui le créent. Ces propriétés peuvent s’exprimer globalement ou localement. III . 1 . — Formes globales a) Relation entre G et F La relation entre G et F découle de ce qui précède. Comme F = m ∗ G et Ep = m∗ F, l’équation F · d r = − d Ep entraîne G · d r = − d F. Donc, le long d’un chemin C : B

F=−

C

G · d r + Cte

ou

F B − FA = −

A

G · dr

On dit que la différence de potentiel entre deux points A et B est la circulation du champ G, entre ces deux points, changée de signe.

85

Gravitation. Propriétés du champ de gravitation

Les lignes de champ sont, en chaque point, tangentes au champ G et les surfaces équipotentielles sont les ensembles de points tels que le potentiel est uniforme : dx dy dz = = Gx Gy Gz

et

F(x, y, z) = Cte

Le champ G est normal aux surfaces équipotentielles ; en effet, si d r représente un déplacement élémentaire quelconque sur une telle surface : G · d r = − d F = 0. Le champ de gravitation G est normal à d r et par conséquent à la surface équipotentielle. En outre, il est toujours dirigé suivant les potentiels F décroissants. b) Théorème de Gauss Le théorème de Gauss (prononcer Gaos) s’appuie sur la notion de flux d’un vecteur (cf. annexe 4). Le flux du champ G à travers une surface fermée S est l’intégrale : F=

S

G · n dS

n étant le vecteur unitaire, normal à la surface et orienté vers l’extérieur, au point où le champ est G (Fig. 6.3a). n

z

S

P i dS

αi



d

dΩi

O x

y

A

n αi dS

Pi

a)

b) F IG . 6.3.

 c Dunod – Toute reproduction non autorisée est un délit

Considérons une surface fermée S entourant un système de masses graves {m ∗i } : F=

S

G · n d S = −G

S



m ∗i i

Pi A d S = −G P iA 3

m∗i i

S

n · Pi A dS P iA 3

en permutant les signes somme et intégration. Avec les notations de la figure 6.3b, on peut voir la signification de la quantité sous le signe somme : n · P iA d S cos ai dS dS = = d Vi = 3 2 PiA P iA P i A2 C’est une quantité sans dimension, caractéristique du cône de sommet Pi qui s’appuie sur l’élément de surface d S. On l’appelle l’angle solide et on l’exprime en stéradian (cf. annexe 4). L’intégration sur l’angle solide Vi est indépendante de la surface S qui entoure le point Pi . On trouve en considérant une surface sphérique S, de rayon R : Vi =

d Vi =

dS 1 = 2 2 R R

dS =

1 4pR 2 = 4p R2

86

6. Gravitation. Propriétés du champ de gravitation

Les masses graves extérieures, elles, délimitent des angles solides opposés : leur contribution est donc nulle. Il en résulte que :

S

G · n d S = −4pGM∗in

où M∗in représente la somme des masses graves intérieures à la surface fermée S : M ∗in =

m∗i,in

ou

Min∗ =

r∗ (r) d

i

si la distribution de masse grave est continue. Ce résultat constitue le théorème de Gauss (du nom du mathématicien allemand C. Gauss) : Le flux du champ G à travers une surface fermée est égal à la somme des masses graves intérieures M ∗in multipliée par la constante (−4pG). Remarque : Notons que, contrairement à son flux à travers une surface fermée qui dépend uniquement des masses intérieures, le champ de gravitation G, lui, dépend de toutes les masses, intérieures et extérieures. III . 2 . — Formes locales a) Relation entre G et F La relation G · d r = − d F s’écrit, en coordonnées cartésiennes : G x d x + G y d y + Gz d z = − d F d’où : ∂F ∂F ∂F Gx = − Gy = − Gz = − ∂x ∂y ∂z Sous forme condensée (cf. annexe 2), on écrit : G = − grad F On déduit des relations précédentes les équations suivantes : ∂Gz ∂Gy − =0 ∂y ∂z

∂Gx ∂Gz − =0 ∂z ∂x

∂Gy ∂Gx − =0 ∂x ∂y

lesquelles peuvent être réunies en une seule équation vectorielle : rot G = 0 si l’on introduit le rotationnel de G , rot G , de composantes cartésiennes (cf. annexe 4) : (rot G )x =

∂Gz ∂G y − ∂y ∂z

(rot G)y =

∂Gx ∂Gz − ∂z ∂x

Ainsi, l’équation G = − grad F entraîne rot G = 0.

et

(rot G) z =

∂Gy ∂Gx − ∂x ∂y

87

Gravitation. Propriétés du champ de gravitation b) Théorème de Gauss

On établit la forme locale du théorème de Gauss à partir d’une relation intégrale connue sous le nom de formule d’Ostrogradsky (cf. annexe 4). Pour le volume , limité par une surface fermée S , on a, par définition de la divergence d’un vecteur :

S

G · n dS =

div G d

où div G est la quantité scalaire d’expression, en coordonnées cartésiennes (x, y, z) : div G =

∂Gx ∂Gy ∂G z + + ∂x ∂y ∂z

Par conséquent, le théorème de Gauss, appliqué au cas d’une distribution volumique de masse grave, donne : F= puisque

S

G · n dS =

div G d

= −4pG

r∗ d

d’où

div G = −4pGr∗

est quelconque.

III . 3 . — Équation de Poisson et équation de Laplace L’équation de Poisson, du nom du mathématicien français P. Poisson du XVIIIe siècle, est l’équation à laquelle satisfait le potentiel de gravitation F. C’est une conséquence des deux propriétés du champ : G = − grad F et

div G = −4pGr∗

Ces deux équations entraînent : div(− grad F) = −4pGr∗

Donnons la signification de div(grad F), en utilisant les coordonnées cartésiennes :

 c Dunod – Toute reproduction non autorisée est un délit

div(grad F) =

∂ ∂x

∂F ∂x

+

∂ ∂y

∂F ∂y

+

∂ ∂F ∂z ∂z

=

∂ 2 F ∂ 2F ∂ 2 F + + 2 ∂x2 ∂y2 ∂z

Cette quantité est le laplacien de F (du nom du physicien français P. Laplace), noté DF. Finalement, on obtient l’équation de Poisson : DF = 4pGr∗ Dans le cas particulier où la région de l’espace considérée ne contient pas de masse grave, cette équation se réduit à l’équation de Laplace : DF = 0

IV . — CALCULS DE CHAMPS ET DE POTENTIELS Lorsque l’on veut déterminer le champ et le potentiel de gravitation produits par une distribution de masse grave, deux cas se présentent. i) La distribution de masse possède une symétrie suffisante pour que l’application du théorème de Gauss donne, sans difficulté, le champ G ; on en déduit alors le potentiel.

88

6. Gravitation. Propriétés du champ de gravitation

ii) Si la symétrie est insuffisante, il est préférable d’exprimer d’abord le potentiel qui est un scalaire et d’en déduire le champ de gravitation par G = − grad F.

Pour illustrer ces deux cas, nous proposons d’abord de calculer le champ et le potentiel produits par une sphère concentriquement homogène (r ∗ ne dépend que de la distance r au centre O). Une telle distribution joue un rôle capital en astrophysique où de multiples objets célestes présentent la symétrie sphérique. Nous établissons ensuite le développement multipolaire à grande distance d’une distribution de masse grave sans symétrie. IV . 1 . — Champ et potentiel produit par une distribution sphérique de masse Considérons une distribution sphérique de masse grave à l’intérieur d’une sphère de centre O et de rayon R (Fig. 6.4). Du fait de la symétrie des causes, le potentiel de gravitation ne dépend que de la norme r. Par conséquent toute sphère S, de centre O, est une surface équipotentielle ; G normal à S est donc radial. z

O

n

G y

x

F IG . 6.4.

a) En un point extérieur Le théorème de Gauss appliqué à une sphère équipotentielle, de rayon r > R, donne (Fig. 6.4) : F = 4pr2G r(r) = −4pGM ∗ On en déduit la composante radiale Gr et G : GM ∗ G r (r) = − 2 r

et

G(r) = −

GM∗ er r2

Le potentiel F(r) s’obtient à partir de l’équation d F = −G · d r. On a donc : d F = G r(r) d r = −

GM∗ dr r2

d’où

F=−

GM∗ + Cte r

Si l’on impose F(r) = 0 pour r infini, ce qui est toujours possible puisqu’il n’y a pas de masse à l’infini. On obtient : GM ∗ F=− r Ainsi, le potentiel et le champ de gravitation créés par une distribution de masse à symétrie sphérique, en un point extérieur, sont les mêmes que ceux relatifs à une masse ponctuelle, située en O et affectée de la masse grave totale. La dépendance radiale r ∗ (r) n’a alors aucune influence et les résultats sont encore valables pour une sphère non homogène mais concentriquement homogène.

89

Gravitation. Propriétés du champ de gravitation b) En un point intérieur Appliquons le théorème de Gauss à une surface sphérique de rayon r < R. Il vient : 4pr 2 Gr (r) = −4pGM ∗in

r



M∗in =

4pr 2r ∗ (r ) d r  0

est la masse grave située à l’intérieur de la sphère de rayon r. Donc : G(r) = −

GM∗in er r2

Si la distribution est homogène (r∗ = Cte), on a : Gr (r) = −

G 4pr 3 GM∗ r r = − r2 R3 3

d’où

G=−

GM∗ r R3

On en déduit le potentiel : −GM ∗ GM ∗ r 2 r d r + Cte = + Cte R3 R3 2

−GM ∗ r · d r + Cte = − R3

G · d r + Cte = −

F(r) = −

Comme le champ G est fini, le potentiel est continu. On a donc, de part et d’autre de la surface : F + (r)

R

= F− (r)

d’où

R



GM∗ GM ∗ = + Cte 2R R

et

Cte = −

3GM ∗ 2R

Finalement : F(r) =

GM ∗ 2R

r2 −3 R2

Sur la figure 6.5, on a représenté les graphes G r(r) et F(r) dans le cas d’une sphère homogène. Gr(r)

 c Dunod – Toute reproduction non autorisée est un délit

O



Φ(r) R

O

r

R r

G M* R 3 GM * − 2 R

GM* R2



a)

b) F IG . 6.5.

IV . 2 . — Potentiel produit à grande distance par une distribution quelconque de masse a) Développement multipolaire Montrons que le potentiel gravitationnel F , à grande distance, se met sous la forme d’une somme de potentiels. Avec les notations de la figure 6.6a, le potentiel s’écrit : F(r) = −G

i

m∗i Ri



Ri = Pi A = r − ri 

et ri  r

90

6. Gravitation. Propriétés du champ de gravitation

Comme R i = (r2 + r 2i − 2r · ri )1/2 = r 1 − 2r · ri /r 2 + r 2i /r2

1/2

, il vient, à grande distance :

r · ri 1 1 3 2r · r i r2 ≈ 1 + 2 − i2 + 2r 8 Ri r r r2

d’où :

F(r) = − G Pi

i

m ∗i r · ri 3 r · ri 1+ 2 + r r 2 r2

2

+ ··· 2



ri2 + ··· 2r2 z

Ri

ri

A

r

O

A

θ r

O

y

x a)

b) F IG . 6.6.

Le potentiel se met ainsi sous la forme d’une somme de potentiels : F(r) = F0 (r) + F1 (r) + F2 (r) + · · · dans laquelle on reconnaît d’abord : F0 (r) = −

GM∗ r

avec

M∗ =

m ∗i

qui est la contribution unipolaire ; c’est le potentiel produit par un point situé en O et affecté de la masse grave totale M∗ . Ensuite, F 1(r) représente la contribution dipolaire, en 1/r2 , caractérisée par le vecteur d, appelé moment dipolaire de la distribution : F1(r) = − G

d · er d·r = −G 3 2 r r

avec

d=

m ∗i ri i

Quant à F2 (r), on l’appelle la contribution quadrupolaire : F2(r) = −

G 2r 3

i

m∗i 3(er · r i)2 − ri2 = −

G 2r 5

i

m ∗i 3(r · r i )2 − r2 r2i

Il convient de noter que, contrairement au cas électrique, la somme des masses graves et donc le premier terme F0 du développement ne sont jamais nuls en présence de matière. Il en résulte que le terme dipolaire a moins d’importance en gravitation qu’en électromagnétisme (cf. Électromagnétisme). b) Cas où la distribution présente une symétrie de révolution Considérons le cas important car fréquent d’un astre dont la distribution de masse grave présente une symétrie de révolution (Fig. 6.6b). Choisissons l’origine O en son centre de masse grave C défini m∗ m∗ par i i CP i = 0 (cf. chapitre 13). Le moment dipolaire est nul puisque i i ri = 0. Il en résulte que : F(r) = F0(r) + F2 (r)

91

Gravitation. Propriétés du champ de gravitation

Explicitons F2 (r) dans la base (ex , ey, ez ), sachant que ez est porté par l’axe de symétrie. Il vient : F2 (r) = −

G 2r 5

i

m ∗i [3(xxi + yyi + zzi ) 2 − (x2i + y 2i + z 2i )(x2 + y 2 + z 2)]

En raison de la symétrie matérielle, les sommes comportant une puissance impaire de x i ou de y i s’annulent et les sommes i m∗i x2i et i m ∗i y2i sont égales. Il en résulte : F2 (r) = −

G 2r 5

=−

G 2r 5

i

m∗i [x 2 (2x2i − y2i − z 2i ) + · · · + 6 xy x i yi + · · · ] m∗i x2

i

x2i + y 2i − z 2i 2

+ y2

x2i + y2i − z 2i + 2z2 2

z2i −

x2i + y2i 2

Les points de suspension représentent des termes analogues que l’on obtient par permutation circulaire. Introduisons le moment quadrupolaire de la distribution : Q= i

m∗i (2z2i − x2i − y 2i )

soit

Q=2 i

m∗i (z2i − x 2i )

en raison de la symétrie de révolution. Il vient : F 2(r) = −

GQ 2 GQ (2z − x2 − y2) = − 5 (3z2 − r2) 4r 5 4r

ce que l’on écrit aussi, en fonction de l’angle u que fait OA avec l’axe de révolution (z = r cos u) : F 2(r, u) = − G

Q(3 cos 2 u − 1) 4r3

 c Dunod – Toute reproduction non autorisée est un délit

Remarque : Le moment quadrupolaire Q s’exprime en fonction des moments d’inertie du système ∗ 2 ∗ 2 2 2 par rapport aux axes Ox et Oz : I Ox = i mi (yi + zi ) et I Oz = i mi (xi + yi ) (cf. chapitre 17). On a : Q = 2(I Ox − IOz ). Finalement, le potentiel de gravitation d’une distribution de masse à symétrie de révolution a pour expression : M∗ Q(3 cos2 u − 1) F(r, u) = − G −G 4r3 r d’où l’on déduit les composantes radiale et orthoradiale du champ de gravitation : Gr = − G

M∗ 3Q(3 cos 2 u − 1) − G 4r4 r2

et G u = −G

3Q sin(2u) 4r4

IV . 3 . — Variation du champ de gravitation dû aux autres astres à la surface de la Terre Le calcul de la variation du champ de gravitation produit par tous les autres astres, à l’exclusion de la Terre, en un point de sa surface, est utile pour interpréter le phénomène des marées (cf. chapitre 7). Cette variation avec la position du point considéré A est donnée par la différence : G a (A) − G a(T ), T étant le centre de la Terre.

92

6. Gravitation. Propriétés du champ de gravitation

Plaçons-nous dans le cas simple où TK reliant T et le centre K d’un astre quelconque, est contenu dans le plan équatorial terrestre et admettons que les astres présentent la symétrie sphérique (Fig. 6.7a). y0

z0 A

GK(A)

T GK (T )

x0

A T z0

K

x0

DK a)

b) F IG . 6.7.

Le champ de gravitation produit en un point extérieur par l’astre de centre K est le même que celui produit par toute sa masse concentrée en son centre : AK TK − GK (A) − G K (T) = GM ∗K 3 AK TK 3 Comme AK = AT + TK, il vient : AK 2 = (AT + TK)2 = AT 2 + TK 2 + 2AT · TK = TK 2 1 + soit, en posant DK = TK, puisque AT  D K : 1 1 ≈ 3 3 AK DK Il en résulte : G K (A) − G K (T ) ≈ G

1+3

TA · TK M∗K AK 1 + 3 3 DK TK 2

AT · TK AT 2 +2 2 TK TK2

TA · TK TK 2

− TK = G

M∗K D3K

AT + 3

AK(TA · TK) TK 2

Le premier terme est porté par AT ; il s’ajoute donc à la gravitation terrestre sans trop la modifier, puisqu’il est proportionnel à D −3 K . Le second représente un champ pratiquement parallèle à l’axe TK reliant les centres de masse et proportionnel à la coordonnée x0 = TA · TK/TK du point A selon l’axe Tx0 perpendiculaire au plan méridien normal à TK : G

AK(TA · TK) M ∗K MK∗ AT + 3 = G D 3K TK2 D3K

AT + 3

x0 AK TK

Ainsi, tout se passe comme si A était soumis, de la part du plan méridien normal à la direction TK, à un champ de répulsion proportionnel à la distance à ce plan (Fig. 6.7b). Comme nous le verrons ultérieurement (cf. chapitre 7), ce résultat permet d’expliquer l’existence de deux marées par jour. Sur le tableau 6.1, on a porté les valeurs de la quantité 10 3 × M∗K /D3K pour quelques astres proches de la Terre ; on voit que les contributions lunaire et solaire sont les plus fortes, dans le rapport 2, 17 à l’avantage de la Lune. 10 3 × MK∗/D3K

Soleil

Lune

Vénus

Jupiter

0,595

1,296

7 × 10 −5

6 × 10−6

TAB . 6.1.

93

Gravitation. Propriétés du champ de gravitation

V . — ÉNERGIE POTENTIELLE GRAVITATIONNELLE L’énergie potentielle gravitationnelle d’un corpuscule de masse grave m ∗, plongé dans le potentiel (g) F créé par une certaine distribution de masse grave, a pour expression Ep = m∗ F. On se propose maintenant d’exprimer cette énergie en fonction de toutes les masses en présence, notamment celles qui créent le potentiel F . Pour cela, commençons par l’exemple simple de deux corpuscules A1 et A2 distants de r . V . 1 . — Énergie potentielle gravitationnelle de deux corpuscules D’après ce qui précède, l’énergie potentielle gravitationnelle de l’ensemble des deux corpuscules A1 et A2 est l’énergie de A 1 dans le potentiel F(A1) créé en A 1 par A2 ; c’est aussi l’énergie de A2 dans le potentiel V (A2 ) créé en A2 par A 1 : E (pg) = m ∗1 F(A1 ) = m∗1

−Gm ∗2 m∗ m ∗ = −G 1 2 r r

ou bien Ep(g) = m ∗2 F(A2) = m ∗2

−Gm∗1 m ∗ m∗ = −G 1 2 r r

ce que l’on écrit souvent sous la forme symétrisée suivante : Ep(g)

1 = [m∗1 F(A 1) + m∗2 F(A 2)] = 2

i=2

m ∗i F(Ai ) i=1

Notons que le facteur 1/2 n’est pas lié au nombre de corpuscules, mais au nombre de couples que l’on compterait deux fois si l’on sommait sur les deux corpuscules. V . 2 . — Énergie gravitationnelle de N corpuscules Avec un système de trois corpuscules A 1 , A2 et A 3 , on aurait, en considérant les trois couples différents {A1 , A2} , {A 1, A3 } et {A2 , A3 } (Fig. 6.8) : Ep(g) = −G

m ∗1 m ∗2 m∗ m∗ m∗ m∗ + 1 3 + 2 3 r12 r13 r23

 c Dunod – Toute reproduction non autorisée est un délit

ce qui se met aussi sous la forme symétrisée : Ep(g) = −G

1 m∗2 m∗ m ∗1 + 3 + m ∗2 2 r12 r 13

m∗1 m∗ + 3 + m ∗3 r12 r23

r12

A1

m 2 m∗1 + r23 r 13

A2

r13

r23 A3 F IG . 6.8.

En introduisant les potentiels F(A i ) aux points Ai , on trouve bien : E(pg) =

1 ∗ 1 [m 1F(A 1) + m ∗2F(A 2) + m∗3 F(A 3)] = 2 2

i=3

m∗i F(A i ) i=1

94

6. Gravitation. Propriétés du champ de gravitation

L’énergie gravitationnelle d’un ensemble de N corpuscules s’obtient alors aisément en sommant les énergies gravitationnelles des différents couples (Ai , Aj) : Ep(g)

= −G

m ∗i m ∗j couple (i, j)

soit

rij

Ep(g)

G =− 2

N

N

m∗i i=1

j=i

m∗j rij

r ij étant la distance A iA j ; le facteur 1/2 dans la seconde expression est indispensable car on somme sur toutes les particules et non sur les seuls couples. En introduisant le potentiel Fi en A i , on trouve : Ep(g) = −

G 2

m∗i i

j=i

m∗j 1 = 2 r ij

N

m ∗i Fi

avec

i=1

F i = −G

j=i

m ∗j rij

Retenons donc l’expression suivante de l’énergie potentielle de gravitation d’un système de N corpuscules : Ep(g)

1 = 2

N

m∗i Fi i=1

V . 3 . — Énergie gravitationnelle d’une distribution continue de masse grave a) Expression de l’énergie gravitationnelle en fonction du potentiel L’expression précédente peut être utilisée si la distribution est continue, encore faut-il remplacer la sommation discrète par une intégrale et la masse grave m∗i par r∗ d : E (pg) =

1 2

r∗F d

Notons que le potentiel F est celui créé par toute la distribution de masse sans exclusion de la masse au point considéré A, contrairement au cas d’une distribution discrète. Si la distribution est continue, la contribution de la masse au point où l’on calcule le potentiel est négligeable, ce que l’on peut montrer en évaluant le potentiel Fε (A) produit par les masses contenues dans un petit volume sphérique de rayon ε : Fε(A) est proportionnel à ce volume, donc à ε3 , et varie comme ε−1 ; il change donc comme ε 2 , ce qui le rend négligeable lorsque ε devient suffisamment faible. Il en résulte que l’expression de l’énergie gravitationnelle, relative à une distribution continue de masse, contient à la fois la contribution des énergies d’interaction entre les différents éléments de volume et celle des énergies propres de chacun de ses éléments. b) Énergie gravitationnelle d’une boule uniformément répartie en volume Considérons une boule de rayon R et de masse M ∗ répartie uniformément en volume. Sa masse volumique est donc r∗ = 3M∗ /(4pR3 ). D’après ce qui précède, le calcul de son énergie gravitationnelle nécessite la détermination préalable du potentiel qu’elle crée en son intérieur. Comme le système présente la symétrie sphérique, le potentiel ne dépend que de la distance r du centre au point intérieur considéré (cf. section IV). L’énergie gravitationnelle s’écrit donc : E (pg)

1 = 2

r ∗ Fin d

1 = r∗ 2

R 0

Fin 4pr 2 d r

95

Gravitation. Propriétés du champ de gravitation En remplaçant Fin par son expression établie précédemment, on obtient : E (pg)

1 = r∗ 2

R 0

GM ∗ − R

3 r2 − 2 2 2R

3GM∗2 4pr d r = − 2R4 2

3r3 r5 − 2 3 10R2

R 0

soit : 3 M ∗2 Ep(g) = − G 5 R Lorsque l’origine de l’énergie potentielle de gravitation est prise à l’infini, le signe moins traduit le caractère attractif de la force d’interaction. Exemple : La Terre peut être assimilée à une distribution sphérique uniforme de masse grave. Son énergie potentielle vaut donc, puisque M∗ = 6 × 1024 kg et R = 6, 4 × 106 m : Ep(g) = −

3 × 6, 67 × 10 −11 (6 × 10 24)2 = −2, 25 × 10 32 J × 5 6, 4 × 10 6

c) Expression de l’énergie gravitationnelle en fonction du champ de gravitation Afin de relier l’énergie E p au champ G, utilisons l’équation locale div G = (−4pG)r ∗ dans l’expression de Ep . On a : 1 1 F div G d r∗ F d = − Ep(g) = 2 8pG étant un volume quelconque englobant la distribution, puisqu’en dehors de cette distribution r ∗ = 0. Afin d’exprimer autrement F div G, explicitons ce terme en coordonnées cartésiennes : F div G = F

∂Gx ∂Gy ∂Gz + + ∂x ∂y ∂z

Or :

∂Gx ∂(FGx ) ∂F ∂(FGx ) − Gx = = + Gx2 ∂x ∂x ∂x ∂x Il en résulte, en combinant avec les deux autres termes de F div G :  c Dunod – Toute reproduction non autorisée est un délit

F

Ep(g) = −

1 8pG

div(FG ) + G 2 d

L’intégrale du premier terme s’écrit aussi, à l’aide du théorème d’Ostrogradsky :

S

FG · n dS

S étant la surface fermée entourant le volume considéré. Si le volume d’intégration est choisi infiniment grand et s’il n’y a pas de masse grave à l’infini, la distribution vue d’un point de la surface est assimilable à celle d’un corpuscule. Il en résulte que F varie comme r −1 , G comme r−2 et donc FG comme r−1 ×r −2 = r−3 ; la surface variant, elle, comme r2, l’intégrale précédente devient négligeable lorsque r augmente indéfiniment. Par conséquent : Ep(g) = −

1 4pG

espace

G2 d 2

96

6. Gravitation. Propriétés du champ de gravitation

l’intégration portant sur tout l’espace. Ainsi, tout se passe comme si l’énergie de gravitation d’une distribution continue de masse grave était répartie dans tout l’espace avec une densité volumique égale à −G2 /(8pG). Notons que l’énergie gravitationnelle est une grandeur toujours négative si l’origine est prise lorsque toutes les masses graves sont infiniment éloignées. On peut retrouver, à l’aide de cette dernière expression, l’énergie potentielle de gravitation d’une sphère de masse grave uniformément répartie.

CONCLUSION (1) Retenons les deux propriétés fondamentales du champ de gravitation G, exprimées sous leur forme globale : B G · dr et G · n d S = −4pGMin∗ FB − FA = − A

S

Localement, ces propriétés s’écrivent respectivement : G = − grad F (ou

rot G = 0)

et

div G = (−4pG)r ∗

Elles sont analogues à celles du champ électrostatique ; on passe d’un domaine à l’autre en remplaçant les charges électriques par les masses graves et 1/e0 par (−4pG).

(2) Les calculs de champ et de potentiel sont identiques à ceux du champ électrostatique (cf. Électromagnétisme). Cependant l’importance du dipôle en gravitation est moins grande qu’en électrostatique car la somme des masses graves n’est jamais nulle. En ce qui concerne l’énergie gravitationnelle d’une distribution continue de masse grave, rappelons les résultats essentiels : G2 1 1 Ep(g) = et E(pg) = − d r∗ F d 2 4pG espace 2

EXERCICES ET PROBLÈMES P6– 1. Force de gravitation exercée par une sphère Trouver la force d’attraction exercée par une sphère (rayon R) de masse grave M ∗ régulièrement répartie en surface, sur un corpuscule A (masse grave m∗ ), situé à la distance r de son centre (Fig. 6.9).

A R

z r

F IG . 6.9.

α A R F IG . 6.10.

97

Gravitation. Propriétés du champ de gravitation P6– 2. Force de gravitation exercée par un disque

Quelle est la force qu’exerce un disque uniforme (masse M ∗ , rayon R) sur un point A, de masse grave m∗, situé sur son axe à une distance z de son centre (Fig. 6.10) ? P6– 3. Comparaison de différents champs de gravitation Comparer les champs de gravitation produits, en un point de leur surface supposée sphérique, par la Terre, le Soleil, la Lune, Vénus, Mars, Jupiter et Saturne. On utilisera les caractéristiques de ces astres données dans le tableau des constantes. P6– 4. Point équigravitationnel entre la Lune et la Terre Un satellite évolue entre la Terre et la Lune. Déterminer le point où le champ de gravitation total est nul. On donne la distance Terre-Lune et le rapport des masses : 384 000 km et 81 respectivement. P6– 5. Champ de gravitation dans une cavité creusée dans une boule Calculer le champ de gravitation G dans une cavité sphérique que l’on a creusée dans une boule homogène de masse volumique r∗ . En déduire qu’il est uniforme. P6– 6. Énergies potentielles de gravitation Terre-Lune et Terre-Soleil Calculer l’énergie potentielle de gravitation du système Terre-Lune. Même question pour le système Terre-Soleil. P6– 7. Variation de l’énergie potentielle de gravitation en fonction de l’altitude Quelle est l’expression, en fonction de l’altitude, de l’énergie potentielle de gravitation du système Terre-satellite artificiel, si on prend comme origine l’énergie potentielle à la surface du sol ? P6– 8. Moment des forces gravitationnelles au centre de la Terre On assimile la Terre à une distribution volumique de masse présentant une symétrie de révolution d’axe Tz, T étant le centre de la Terre. Le champ de gravitation G produit par une telle distribution, de masse totale M ∗T, en fonction de la distance r = TK et de l’angle u = (Tz, TK), est :

 c Dunod – Toute reproduction non autorisée est un délit

G(r, u) = G r er + G u eu

avec

Dans ces expressions : Q = 2(I1 − I3)

Gr = −

avec

3GQ GM∗K − 4 (3 cos2 u − 1) 2 4r r m ∗i (y2i + z 2i )

I1 = i

et Gu = −

3GQ sin(2u) 4 r4

m∗i (x2i + y2i )

et I3 = i

1. Exprimer la force de gravitation qu’exerce la Terre sur l’astre ponctuel K, de masse M ∗K , ainsi que son moment en T, M T ,T →K . On introduira le vecteur unitaire e u = er × e u. 2. En déduire, à l’aide de l’opposition des actions réciproques, le moment en T, M T , des forces de gravitation qu’exercent la Lune et le Soleil sur la Terre en son centre. On admettra que la Lune est dans le plan de l’écliptique, c’est-à-dire dans le plan dans lequel évolue le Soleil dans son mouvement apparent. Comparer les actions de la Lune et du Soleil. P6– 9. Énergie gravitationnelle d’une boule homogène 1. Retrouver l’expression de l’énergie potentielle de gravitation d’une boule homogène, de masse M ∗ et de rayon R, à l’aide du champ de gravitation. Application au cas de la Terre. 2. Quel serait le rayon R 0 d’un astre, de même masse que le Soleil, dont l’énergie gravitationnelle serait égale à −M∗ c 2 (c ≈ 3 × 10 8 m . s −1) ? Application numérique.

98

6. Gravitation. Propriétés du champ de gravitation

P6– 10. Énergie gravitationnelle d’une sphère homogène 1. Quel est le potentiel de gravitation produit par une sphère (creuse) homogène, de masse M ∗ et de rayon R ? 2. Trouver son énergie potentielle. Calculer sa valeur pour M ∗ = 1 kg et R = 0, 1 m. P6– 11. Détermination de certaines caractéristiques physiques de la Terre La planète Terre est assimilée à une sphère de rayon R. La longueur de l’équateur est l = 40 000 km. On suppose que la seule force contribuant à la pesanteur est l’attraction newtonienne exercée par la Terre, qui produit au niveau du sol un champ de gravitation G0 ≈ 9, 80 m . s −2 . On néglige ainsi l’influence de la rotation terrestre. 1. Établir la formule donnant la masse grave volumique moyenne r ∗0 du globe terrestre en fonction de G0 et de l. Calculer r ∗0 en kg . m−3 , ainsi que la masse grave M ∗ de la Terre. 2. L’expérience montre que la masse volumique de la croûte terrestre est bien inférieure à la valeur précédemment calculée. Aussi suppose-t-on que la Terre est constituée d’un noyau central, sphérique, homogène, de rayon r 2 et de masse grave volumique r ∗2, entouré d’une couche de masse grave volumique r∗1 . a) Écrire la relation qui existe entre r 2, R, r∗0 , r∗1 et r∗2 . b) Calculer la valeur G du champ de gravitation en un point de la couche superficielle distant du centre de la Terre de r, ainsi que la variation de G pour une petite variation dr, près du sol, en fonction de dr, G0, R, r ∗0 et r∗1 .

c) Sachant que la période propre d’une horloge à pendule simple, descendue au fond d’un puits de profondeur h = 195 m, avance de DT0 = 1 s par jour, calculer la masse volumique r∗1 de l’écorce terrestre. d) L’étude de la propagation des secousses sismiques transversales montre que le noyau central commence à une profondeur de H = 1 300 km. Calculer la masse grave volumique de ce noyau. P6– 12. Oscillations dans le champ de gravitation à l’intérieur d’une boule On suppose que la Terre a la forme d’une boule (rayon R T ) et que sa masse M ∗T est uniformément répartie dans son volume. 1. Trouver la force de gravitation à laquelle est soumise une particule A, de masse m ∗ , placée à une distance r du centre. 2. Montrer qu’une telle particule A, libre de se déplacer selon le diamètre passant par sa position initiale, oscillerait avec une période T0 que l’on exprimera en fonction de R T et du champ de gravitation G0 à la surface de la Terre. Calculer T 0. P6– 13. Paradoxe sur le système Soleil-Terre-Lune On assimile le système Soleil-Terre-Lune à trois points matériels S, T et L dont les masses sont celles de ces astres. 1. Calculer, dans le référentiel de Copernic, les forces de gravitation qu’exercent sur la Lune respectivement le Soleil et la Terre. Le résultat semble paradoxal. Pourquoi ? 2. Quelle est la valeur du champ de gravitation produit par la Terre en L ? Comparer cette valeur à la différence du champ de gravitation solaire aux points T et L. Conclure.

7 Référentiels non galiléens. Dynamique terrestre Nous avons vu que la loi fondamentale de la dynamique du point matériel faisait apparaître des référentiels privilégiés, dits galiléens. Or il arrive souvent que l’on doive étudier le mouvement d’un corpuscule par rapport à un référentiel quelconque. La question immédiate qui se pose alors est la suivante : comment écrire la loi fondamentale dans un tel référentiel ? Cette question n’est pas seulement formelle, car c’est en y répondant que le physicien français L. Foucault put interpréter certains désaccords entre l’observation et les prédictions de la loi fondamentale dans un référentiel terrestre. Le célèbre Pendule de Foucault fut l’éclatante démonstration de la possibilité de mettre en évidence la rotation de la Terre à l’aide d’une expérience terrestre, sans référence au ciel, et ainsi du caractère approximativement galiléen du référentiel terrestre.

I . — RÉFÉRENTIELS NON GALILÉENS

 c Dunod – Toute reproduction non autorisée est un délit

I . 1 . — Définition Un référentiel non galiléen est un référentiel en mouvement accéléré par rapport à un référentiel galiléen (cf. chapitre 4). Désignons par R un tel référentiel en mouvement par rapport au référentiel galiléen R (Fig. 7.1). D’après la composition des accélérations, on a, pour un corpuscule A en mouvement par rapport à R et R : aA/R = a A/R + ae + aC avec, si V est le vecteur vitesse de rotation de R  par rapport à R (cf. chapitre 3) : dV a e = aO /R + × O A + V × (V × O A) et aC = 2V × v A/R dt z A

z y O y

O x

x

R F IG . 7.1.

R

100

7. Référentiels non galiléens. Dynamique terrestre

I . 2 . — Expression de la loi fondamentale dans un référentiel non galiléen Remplaçant l’accélération a A/R par l’expression précédente dans la relation vectorielle qui traduit la loi fondamentale de la dynamique, il vient : maA/R =

F

soit

m aA/R  + a e + aC =

F

Ainsi, en isolant le premier terme du premier membre, on obtient : maA/R  =

F − ma e − ma C

Cette équation reliant la quantité de mouvement de A, par rapport à R , à la somme des forces F montre que la loi fondamentale de la dynamique peut être appliquée par rapport à un référentiel quelconque, pourvu que l’on ajoute aux forces précédentes, dues à la présence des autres corps, deux forces supplémentaires la force d’inertie d’entraînement et la force d’inertie de Coriolis respectivement : Fie = −mae = −m aO /R +

dV × O A + V × (V × O A) dt

et FiC = −maC = −2mV × vA/R Notons que ces forces d’inertie, qui par définition sont proportionnelles aux masses, ne peuvent pas être attribuées à la présence de corps situés à proximité du corpuscule : elles sont dues uniquement au caractère non privilégié du référentiel choisi ; pour cette raison, on les appelle aussi forces de repère. Remarques : (1) On dit parfois que ces forces sont fictives car elles n’existent que dans les référentiels non galiléens. En raison de la réalité de leurs effets dans ces référentiels, il nous semble préférable d’éviter cette dernière dénomination. (2) Alors que l’expression « force d’inertie de Coriolis » est universellement utilisée, celle de « force d’inertie d’entraînement » est peu répandue. (3) Dans les premiers écrits sur la mécanique, on appelait force d’inertie la quantité −maA/R qui apparaissait dans l’expression SF −maA/R = O de la loi fondamentale, ramenée à une condition d’équilibre sur les forces. I . 3 . — Cas particuliers Deux cas particuliers méritent d’être analysés : la translation de R  par rapport à R et la rotation uniforme de R  autour d’un axe fixe dans R. a) Translation Dans un mouvement de translation, V R/R = 0 (Fig. 7.2) ; par conséquent, l’accélération d’entraînement a e se réduit à l’accélération d’un point quelconque de R , par exemple celle de son origine O , et l’accélération de Coriolis est nulle : ae = a O /R

et a C = 2V × vA/R  = 0

Il en résulte : Fie = −ma e = −ma O /R

et

F iC = −maC = 0

101

Référentiels non galiléens. Dynamique terrestre

z

z

O



O

Fie

x

x

A

A

R



y

R

x

F IG . 7.2.

Fie

y

Ω O

y

y R

z H

ae

x

P



R

F IG . 7.3.

b) Rotation uniforme autour d’un axe fixe Considérons un référentiel R  de même origine que R et ayant un mouvement de rotation uniforme d’axe Oz de R (Fig. 7.3). Dans ce cas, l’accélération d’entraînement se réduit à (cf. chapitre 3) : ae = −V 2HA H étant la projection de A sur l’axe de rotation. Il en résulte : F ie = −mae = mV2HA Cette force d’inertie est dirigée suivant le vecteur HA ; on dit qu’elle est centrifuge car elle « fuit » le centre de rotation H. Elle a été introduite par Huygens en 1673. I . 4 . — Corpuscule immobile dans un référentiel non galiléen Un corpuscule A est immobile dans un référentiel non galiléen R  si le vecteur position O  A , qui détermine sa position par rapport à R , est constant : O A = Cte

d’où

vA/R = 0

et

aA/R  = 0

 c Dunod – Toute reproduction non autorisée est un délit

Par conséquent, la force de Coriolis étant nulle, A est immobile si : F − ma e = 0

II . — DYNAMIQUE TERRESTRE L’existence des forces d’inertie permet de définir expérimentalement les référentiels galiléens. En effet, pour un corpuscule soumis à des actions déterminées, tout désaccord entre les prédictions de la loi fondamentale de la dynamique et les résultats expérimentaux peut être attribué à la nature non galiléenne du référentiel considéré. C’est ainsi qu’à la suite d’expériences fines, telles que le pendule de Foucault et la déviation vers l’est d’une bille au cours d’une chute, on a pu interpréter ces expériences en considérant que le référentiel terrestre ne constituait qu’une réalisation approchée d’un référentiel galiléen. On admet actuellement que le référentiel de Copernic, dont l’origine est le centre de masse du système solaire et dont les axes sont définis par trois étoiles éloignées, réalise lui une meilleure approximation d’un référentiel galiléen. Nous allons, dans la suite, développer ce point en appliquant la loi fondamentale de la mécanique dans les référentiels terrestres.

102

7. Référentiels non galiléens. Dynamique terrestre

Distinguons les trois référentiels suivants : le référentiel de Copernic R 0 = C0x 0y0 z0 , le référentiel géocentrique Rg = Tx 0y0z 0, en translation par rapport à R0 et d’origine le centre T de la Terre, et le référentiel terrestre local R = Oxyz avec Oy dirigé vers le nord et Ox vers l’est (Fig. 7.4). On introduit parfois le référentiel de Kepler R K = Sx 0y0z0 en translation par rapport à R0 mais d’origine le centre S du Soleil. z0 ΩT z0

¸ N y

R ΩT

H y0

C0 x0

µ

T

¸

O

z

t

R0 x0 Rg

z

x (est) y0

S

ΩT A O x (est)

F IG . 7.4.

¸

y (nord)

R F IG . 7.5.

II . 1 . — Définition expérimentale du poids d’un corps Le poids d’un corps, ou force de pesanteur terrestre qui s’exerce sur lui, est défini expérimentalement comme la force opposée à celle qui le maintient en équilibre dans un référentiel terrestre R (Fig. 7.5). Si l’on suspend une masselotte A à l’extrémité inférieure d’un ressort, son poids F p est l’opposé de la tension T du ressort à l’équilibre dans R. II . 2 . — Relation entre pesanteur, gravitation et inertie Pour déterminer les causes de la pesanteur, écrivons, par rapport à R non galiléen, la loi fondamentale de la dynamique. Les forces qui s’exercent sur le corpuscule A dans le référentiel terrestre sont : i) les forces de gravitation, qui s’écrivent m ∗ G(A), en notant m∗ la masse grave de A et G A le champ de gravitation produit par tous les astres en A ; ii) les forces d’inertie d’entraînement −ma e et de Coriolis −2mVT × vA/R , VT étant la vitesse de rotation de la Terre autour de l’axe des pôles, par rapport aux référentiels R 0 ou Rg ; iii) les forces occasionnelles telles que la tension T du ressort. Il vient : maA/R = m ∗ G(A) + T − mae − 2mVT × vA/R À l’équilibre dans R, aA/R = 0, vA/R = 0 et T = −Fp. Par conséquent : Fp = m∗ G(A) − mae Ainsi la pesanteur Fp est la somme vectorielle de la force de gravitation et de la force d’inertie d’entraînement terrestre.

103

Référentiels non galiléens. Dynamique terrestre II . 3 . — Expression de la loi fondamentale dans un référentiel terrestre

En introduisant le poids F p et en désignant par F oc la somme des forces supplémentaires occasionnelles, la loi fondamentale dans R s’écrit : maA/R = F p + Foc − 2mV T × vA/R À condition d’introduire le poids, la loi fondamentale s’écrit rigoureusement, sans adjonction de la force d’inertie d’entraînement terrestre, mais avec la force d’inertie de Coriolis terrestre. II . 4 . — Égalité de la masse grave et de la masse inerte L’égalité de la masse grave et de la masse inerte d’un corps a été établie expérimentalement par Newton, à l’aide de l’expérience dite du tube de Newton (Fig. 7.6). Dans un tube dans lequel on a fait le vide, on constate que deux corpuscules, de masses inertes différentes, acquièrent, au cours d’une chute, la même accélération.

g

Tube vidé d’air

Corpuscules Pompe à vide F IG . 7.6.

 c Dunod – Toute reproduction non autorisée est un délit

Appliquons, aux deux corpuscules A 1 et A2 , la loi fondamentale de la dynamique dans le référentiel terrestre R. On obtient : m1 a1 = F p,1 − 2m1 VT × v1

et m2 a2 = F p,2 − 2m2 VT × v2

On constate que deux corps, abandonnés dans les mêmes conditions, acquièrent la même accélération et la même vitesse : a 1 = a2 et v1 = v2 . Il en résulte que les poids sont proportionnels aux masses inertes : Fp,1 Fp,2 m∗1 m∗ soit G(A1 ) − ae,1 = 2 G(A 2 ) − ae,2 = m1 m2 m1 m2 Or les coprs A1 et A 2 sont proches l’un de l’autre de sorte que l’on peut confondre les valeurs des champs de gravitation G(A1 ) ≈ G (A2 ) et les accélérations a e,1 et ae,2. Il en résulte : m∗1 m∗ ≈ 2 m1 m2 Si on adopte la même unité, le kilogramme, pour la mesure des masses grave et inerte, ce qui revient à poser égale à 1 la constante de proportionnalité, les masses grave et inerte sont égales. Ce résultat a été confirmé par des expériences de R. Dicke avec une précision relative de 10−11 .

104

7. Référentiels non galiléens. Dynamique terrestre

Cette égalité des masses grave et inerte est à l’origine du principe d’équivalence affirmant, dans le cadre de la théorie de la relativité générale d’Einstein, qui date de 1915, que : m∗ = m Bien que connue et apparemment banale, l’égalité de la masse grave et de la masse inerte rend la force de gravitation singulière, car c’est la seule force fondamentale qui, à la manière des forces d’inertie, est proportionnelle à la masse inerte. Il existe donc une proximité de nature entre les forces de gravitation et les forces d’inertie. C’est pour cette raison que la force de gravitation n’est pas considérée comme une force véritable en relativité générale (cf. Relativité et invariance) ! II . 5 . — Champ de pesanteur terrestre Par définition, on appelle champ de pesanteur terrestre le poids d’un corps par unité de masse : g=

Fp = G (A) − a e m

Séparons dans G(A) la contribution terrestre de celle des autres astres : G (A) = GT (A) + G a(A) De même, distinguons dans ae, accélération du point coïncidant A  lié à R, l’accélération du centre T de la Terre et l’accélération due à sa rotation autour de son axe. Il vient, en introduisant le référentiel géocentrique Rg, d’après la composition des accélérations de A  entre R0 et R g : ae = aA /R 0 = aT/R 0 + V2T A H = a T/R0 + V2T AH H étant la projection de A suivant l’axe de rotation. Il en résulte que : g = GT (A) + V2T HA + G a(A) − aT/R 0 L’application de la loi fondamentale à la Terre considérée comme un point matériel T de masse M T donne : MT aT/R 0 = MT G a (T )

soit

a T/R0 = Ga(T )

On en déduit l’expression suivante du champ de pesanteur : g = G T (A) + V2T HA + Ga(A) − G a (T ) Par définition, la direction de g est la verticale et sa norme g l’intensité du champ de pesanteur, appelée aussi accélération due à la pesanteur. Tout plan normal à g est un plan horizontal. Remarque : Nous avons volontairement utilisé l’expression accélération due à la pesanteur plutôt que celle d’accélération de la pesanteur qui est incorrecte mais très souvent employée.

105

Référentiels non galiléens. Dynamique terrestre

Dans l’expression de g, le premier terme G T (A) dû à la Terre est prépondérant. Le deuxième, proportionnel au carré de la rotation de la Terre autour de l’axe des pôles, est la contribution centrifuge. Quant à la différence des deux derniers termes, elle représente la variation du champ de gravitation en des points de la surface de la Terre ; c’est cette différence qui permet d’expliquer les marées (cf. III). Généralement, on la néglige ; le champ de pesanteur apparaît alors comme la somme du champ de gravitation terrestre et d’un champ centrifuge : g = G T(A) + V2T HA À l’équateur où il est maximal, puisque HA est égal au rayon terrestre R T , le champ centrifuge vaut : GT GT = 2 289 17 Ce résultat explique pourquoi on dit parfois qu’il y aurait impesanteur à l’équateur, c’est-à-dire g = 0, si le solide-Terre tournait 17 fois plus vite ! Ainsi, même si la Terre était sphérique et concentriquement homogène, la direction de g ne passerait pas par le centre de la Terre, excepté évidemment aux pôles et à l’équateur (Fig. 7.7). On montre que cette direction fait avec le rayon terrestre local un angle a dont la valeur maximale de 6 minutes d’arc est atteinte pour une latitude voisine de 45 ◦ (cf. Exercices). V 2T RT = 3, 4 × 10−2 ≈

y

VT

z O

N G µ T ®

−ae

g

S F IG . 7.7.

Pour connaître les variations de g avec l’altitude et la colatitude u, on explicite ce champ suivant la direction radiale er = TO/TO et la direction orthoradiale eu (Fig. 7.7) :

 c Dunod – Toute reproduction non autorisée est un délit

g r ≈ g z = −G

MT + V2T r sin 2u r2

et gu ≈ −g y = V2T r cos u sin u

ce qui s’écrit en fonction de la latitude du lieu l = p/2 − u : gr ≈ g z = −G

MT + V2T r cos 2 l r2

et gu ≈ −gy = V 2Tr sin l cos l

Lorsque les déplacements ont lieu au voisinage d’un point de la surface de la Terre, le champ de pesanteur peut être considéré comme uniforme. On écrit alors g = −g ez avec g = g. Remarques : (1) Cette présentation, qui subordonne le champ de pesanteur au champ de gravitation et aux forces d’inertie d’entraînement, montre que le concept de masse pesante, que l’on introduisait autrefois, est superflu. En revanche, certains auteurs distinguent la masse grave active qui produit le champ de gravitation, de la masse grave passive qui le subit ; l’opposition des actions réciproques permet alors d’établir l’égalité de ces deux masses. (2) En réalité, la Terre n’est pas sphérique mais légèrement renflée à l’Équateur, comme l’avait prévu Newton, soutenu en France par P. Maupertuis, G. Marquise du Châtelet et Voltaire, contre J. Cassini.

106

7. Référentiels non galiléens. Dynamique terrestre

III . — MARÉES OCÉANIQUES Sur la Terre, les marées océaniques sont attribuées à la non-uniformité du champ de gravitation produit par tous les autres astres. Elles sont donc dues à la variation, avec la position du point A, à la surface de la Terre, de la différence des champs de gravitation : G a (A) − G a(T ).

Plaçons-nous dans le cas simple où le vecteur TK, reliant le centre de masse T de la Terre et celui K d’un astre quelconque, est contenu dans le plan équatorial terrestre et admettons que les astres présentent la symétrie sphérique (Fig. 7.8) : le champ de gravitation, produit en un point extérieur, est le même que celui produit par toute la masse concentrée au centre des astres (cf. chapitre 6). Comme la distance qui sépare les points A et T est faible devant D K = KT, la contribution de cet astre à la différence des champs de gravitation est proportionnelle à la différentielle du champ GK = GM K /D2K , c’est-à-dire à sa masse M K et à 1/D 3K . C’est bien ce que révèle une analyse plus fine (cf. chapitre 6) : G K (A) − G K(T ) ≈ G

MK x0 AT + 3 AK 3 TK DK

x0 étant la composante du vecteur TA selon la direction TK. Sur la figure 7.8, on a représenté le vecteur GK (A) − G K (T ) : tout se passe comme si A était soumis à un champ de répulsion de la part du plan méridien perpendiculaire à la direction de KT et proportionnel à la distance du point à ce plan. y0 Terre A

ΩT

K x0

T

F IG . 7.8.

Ce terme prévoit donc l’existence de deux marées quotidiennes que l’astre K produit sur la Terre. En effet, au cours d’une révolution autour de l’axe Tz 0, le point A de la surface de la Terre, représentant par exemple un élément de liquide océanique, est soumis deux fois à une force maximale de répulsion. C’est ce que l’on observe sensiblement sur la côte atlantique de la France. Les effets de la Lune et du Soleil, qui sont les plus importants, dans le rapport 2,18 à l’avantage de la Lune (cf. chapitre 6), peuvent s’ajouter ou au contraire s’opposer. Au cours d’une lunaison, ils s’ajoutent aux syzygies, lorsque le Soleil, la Terre et la Lune sont alignés, c’est-à-dire à la pleine et à la nouvelle Lune (marées de viveeau), et s’opposent aux quadratures lors des premier et dernier quartiers (marées de morte-eau). Sur la figure (Fig. 7.9), la configuration des astres correspond à la nouvelle Lune. D’une journée à l’autre, en raison du mouvement orbital de la Lune autour de la Terre, on observe un décalage des heures de marée. Pour que la Lune se retrouve dans la même position que la veille, il faut attendre que la Terre ait effectué une rotation complète augmentée de l’angle de déplacement de la Lune sur son orbite pendant une durée T j = 1 j : w = 2p +

2p Tj TL

avec

T L = 27, 3 j

107

Référentiels non galiléens. Dynamique terrestre 23 ◦ 26 Axe sud-nord Soleil

Lune

Terre

5◦

Plan de l'écliptique F IG . 7.9.

Pour effectuer cette rotation, la Terre mettra une durée égale à : T=

2p 2p Tj w = + = Tj VT V T V T TL

1+

Tj TL

= 24 h + 53 min

 c Dunod – Toute reproduction non autorisée est un délit

Le décalage horaire des marées est donc de l’ordre 50 min par jour. Au cours d’une année, l’action du Soleil varie puisque, par rapport à la Terre, cet astre évolue dans le plan de l’écliptique qui fait avec le plan équatorial un angle de 23 ◦ 26 . Deux fois dans l’année, au moment des équinoxes de printemps et d’automne, lorsque le Soleil traverse le plan équatorial, sa contribution aux marées est maximale (marées d’équinoxes). En France, on introduit, depuis le XIX e siècle, des coefficients de marées qui varient linéairement avec le marnage, c’est-à-dire la dénivellation verticale entre la pleine mer et la basse mer. Arbitrairement, on a affecté la valeur 20 à une marée de morte-eau exceptionnelle et la valeur 120 à une marée de vive-eau exceptionnelle. Par exemple, lors de la marée exceptionnelle du 10 mars 1997, le coefficient valait 119. L’amplitude des marées peut varier beaucoup d’un point à un autre du globe. On explique cette grande variation en faisant intervenir la résonance entre le terme significatif de la gravitation qui présente une période de l’ordre de 12 heures et les oscillations propres des masses fluides dans les fonds marins. Dans certains sites, l’influence des dimensions, de la forme et de la profondeur des fonds marins est telle que les marées sont négligeables (c’est le cas en Méditerranée) ou que leur période diffère sensiblement de 12 heures. Remarque : L’effet de marée qu’exerce la Lune entre la tête et les pieds d’une personne mesurant h = 1, 80 m étendue selon la direction de l’astre est négligeable ; en effet, on a, par kg : GML 3h = 4, 7 × 10−13 N.kg −1 D3L

IV . — LA FORCE DE CORIOLIS TERRESTRE La mise en évidence de la force de Coriolis terrestre repose sur l’expression vectorielle de la loi fondamentale de la mécanique appliquée à un point matériel A dans un référentiel terrestre. Notant F oc la somme des forces supplémentaires occasionnelles, il vient : maA/R = mg + Foc − 2mVT × vA/R La force de Coriolis terrestre est le plus souvent négligeable devant le poids. En effet : g ≈ 9, 8 m . s −2 alors que

2V T × v < 1, 5 × 10−4 × v

108

7. Référentiels non galiléens. Dynamique terrestre

Le rapport de ces deux forces est donc inférieur à 1 % si la vitesse est inférieure à 700 m . s−1 . Cependant, c’est ce terme généralement négligeable qui permet d’expliquer certains phénomènes observables et jouant parfois un rôle majeur (déviation des vents et des courants marins). IV . 1 . — Déviation de la trajectoire d’une particule Considérons une particule A astreinte, par une force de réaction R, à évoluer dans un plan horizontal et ayant, au point O, une vitesse v0 horizontale (Fig. 7.10a). z0

z

¸

O x (est)

v0

Ωh

y

ΩT

ΩT

Ωv

z Hémisphère nord

µ O

Ωv

y (nord)

y

T µ

Ωv O

a)

z

b)

Hémisphère sud

F IG . 7.10.

L’effet de la force de Coriolis, peut être étudié à partir de l’équation vectorielle donnée par la loi fondamentale de la dynamique dans le référentiel terrestre R = Oxyz : m

dv = mg + R − 2mVT × v dt

Comme la trajectoire est pratiquement contenue dans le plan horizontal Oxy, l’équation précédente se réduit dans ce plan à : m

dv = −2mV v × v dt

soit

dv = 2v × Vv dt

où V v est la contribution verticale de VT . On voit que le vecteur vitesse varie vectoriellement selon la direction et le sens de v × V v. Dans l’hémisphère sud, la déviation change de sens, comme on peut le vérifier sur une particule se déplaçant vers le nord (axe Oy de la figure 7.10b). L’équation réduite du mouvement est caractéristique d’un mouvement circulaire et uniforme : en effet, on peut la rapprocher de l’équation du mouvement d’une particule chargée dans un champ magnétique B (cf. chapitre 8) : m

dv = qv × B dt

soit

dv = v × vc dt

avec

vc =

qB m

L’équation précédente donne donc, dans le plan horizontal : v 02 = 2v0 Vv = 2v 0V T| sin l| R

d’où

R=

v0 2VT | sin l|

Remarque : La contribution horizontale Vh de V T intervient dans l’équation qui exprime que la somme des forces verticales doit être nulle : mg + R − 2mV h × v = 0. Sur la figure 7.10a, on a représenté le cercle décrit dans l’hémisphère nord : la particule est déviée vers la droite d’un observateur placé selon la verticale ascendante locale ; elle repasse par le même

109

Référentiels non galiléens. Dynamique terrestre

point au bout d’une durée égale à 12 h/ | sin l| ; sa valeur est maximale aux pôles. Cette déviation de la trajectoire des particules par la force de Coriolis, établie théoriquement par Poisson en 1837, joue un rôle important en géophysique, car elle permet d’expliquer l’origine des alizés dans l’hémisphère nord ; ces vents sont dus au déplacement des masses d’air venant du nord et se dirigeant vers l’équateur où elles remplacent les masses d’air chaudes en mouvement ascensionnel. En raison de la déviation vers la droite par la force de Coriolis, ces vents semblent provenir du nord-est (Fig. 7.11). On explique aussi, par la force de Coriolis, la déviation vers la droite du Gulf Stream ; comme son nom l’indique, c’est un courant marin, produit dans le golfe des Caraïbes par les vents du sud-ouest, qui se dirige vers le nord. Pendant longtemps, on a attribué à ce courant marin chaud la différence de température constatée entre la côte est des USA et la côte ouest européenne plus tempérée ; cette interprétation est actuellement constestée. En météorologie, à une échelle synoptique, la force de Coriolis permet de la même façon d’interpréter la structure en spirale des masses nuageuses autour de zones de basse pression. Dans l’hémisphère nord, la spirale est décrite dans le sens inverse des aiguilles d’une montre ; dans l’hémisphère sud, c’est le contraire. La figure 7.11b représente une photographie, prise par satellite en 2005, du cyclone Katrina sur la côte atlantique sud des USA. Le sens de description de la spirale nuageuse est donné par la variation du vecteur vitesse orientée selon v × Vv et peut être obtenu en s’aidant de la figure 7.11c. En effet, la vitesse v est globalement orientée des hautes pressions périphériques vers les basses pressions de la zone centrale. En raison du terme 2v × Vv , les trajectoires des particules contournent la zone centrale de faible pression du fait de la déviation vers la droite par rapport à la direction de v. Il en résulte une orientation de la spirale dans le sens inverse des aiguilles d’une montre. La justification complète de ce mouvement sera donnée en mécanique des fluides (cf. chapitre 30).

Pôle nord

ΩT

920 hPa D

Haute pression 1 100 hPa

Équateur

 c Dunod – Toute reproduction non autorisée est un délit

Pôle sud

Hémisphère nord a)

b) F IG . 7.11.

c)

Remarques : (1) Autour de zones de surpression (cas d’un anticyclone), le sens de la spirale est inversé. (2) On donne souvent comme exemple de cette déviation celle prise par l’eau lorsqu’on vidange un lavabo. En réalité, l’analyse montre que la précision sur l’état mécanique du fluide devrait être très grande pour que l’expérience soit significative (cf. chapitre 30). IV . 2 . — Pendule de Foucault a) Expérience historique Dans sa publication originale du 3 février 1851, à l’Académie des Sciences, Foucault fit remarquer qu’il était possible de mettre en évidence et de mesurer la vitesse de rotation VT de la Terre autour de l’axe des pôles, sans l’aide des étoiles, en étudiant seulement le mouvement d’un pendule simple par rapport au référentiel terrestre R.

110

7. Référentiels non galiléens. Dynamique terrestre

Le comportement d’un tel pendule s’interprète aisément au pôle nord, sans équation ( !) comme le fit remarquer Foucault lui-même. En effet, par rapport au référentiel géocentrique Rg , le pendule, écarté de sa position d’équilibre et abandonné à son poids et à la tension du fil, effectue un mouvement oscillatoire dans un plan fixe, déterminé par les conditions initiales. Aussi, dans R, qui tourne par rapport à Rg , dans le sens ouest-est, avec la vitesse angulaire VT , ce plan semble effectuer une rotation autour de l’axe polaire, dans le sens opposé, avec une période de révolution T de 24 heures. L’expérience a été faite par Foucault à Paris, dans sa cave, puis refaite sous la coupole du Panthéon avec les caractéristiques suivantes : masse m = 28 kg et longueur l = 67 m (Fig. 7.12a). Dans ce cas, seule la composante selon la verticale ascendante Oz du vecteur rotation VT joue un rôle effectif, précisément Vv = VT cos u ez = VT sin l ez, u étant la colatitude et l = p/2−u la latitude. Comme l = 48◦ 51 , la période de révolution du plan du pendule de Foucault vaut : 2p 2p 2p T= = = = 31 h 52 min Vv V T cos u VT sin l Remarques : (1) Notons qu’à l’équateur (l = 0◦ ) le plan d’oscillation du pendule dans R est immobile (T = ∞) ; il tourne donc par rapport à R g . (2) On peut simuler le comportement d’un tel pendule à l’aide d’une expérience simple dans laquelle un pendule simple oscille, alors que son support repose sur le plan d’un tabouret en rotation autour d’un axe vertical (Fig. 7.12b) : le référentiel terrestre simule le référentiel géocentrique et le référentiel du tabouret celui lié à la Terre. On constate que le plan d’oscillation du pendule n’est pas influencé par la rotation du tabouret ; dans le référentiel lié au tabouret, ce plan semble tourner en sens inverse. z0

z

O1

O1

z Vv A

VT

O

¸

y0

x

y (nord)

O

y

A x0

V

Tabouret

x (est) a)

b) F IG . 7.12.

b) Développement quantitatif Il n’est pas inutile de préciser que l’analyse quantitative, développée à partir des projections de l’équation vectorielle du mouvement, celle qui a la faveur dans l’enseignement français de la mécanique, ne fut pas effectuée par Foucault, probablement peu intéressé par ce développement technique, mais par le mathématicien astronome italien Giovanni Plana, plusieurs mois après l’expérience. L’application de la loi fondamentale de la dynamique, par rapport au référentiel terrestre R , aux petits mouvements du pendule autour de l’origine O de R , donne, en remplaçant la force de tension FT qu’exerce le fil par −(F T/l)SA :

FT SA − 2mV T × vA/R l Le dernier terme du second membre est la force d’inertie de Coriolis terrestre qui apparaît dans R , la force d’inertie d’entraînement étant contenue dans le poids. Il vient, en supprimant toutes les contribumaA/R = mg −

111

Référentiels non galiléens. Dynamique terrestre

tions verticales des trois forces, mg, FT = −(F T /l)SO et −2mV h × vA/R , V h étant la composante horizontale de VT : F maA/R = − T OA − 2mV v × vA/R l Comme FT ≈ mg, on obtient, en introduisant la pulsation d’oscillation du pendule v0 = (g/l)1/2 , laquelle, rappelons-le, ne dépend pas de la masse en raison de l’égalité de la masse grave et de la masse inerte : aA/R + v 20 OA = −2V v × v A/R Le premier membre est caractéristique d’un oscillateur horizontal bidimensionnel ; le second, dû à la force de Coriolis, provoque une lente rotation du plan d’oscillation, à la vitesse angulaire V v , dans le sens est-ouest. La projection de l’équation vectorielle dans la base cartésienne de R donne des équations différentielles dont la résolution montre que, dans un référentiel R  en rotation uniforme par rapport à R , avec la vitesse angulaire V v = VT cos u , la trajectoire est une ellipse très aplatie (cf. Exercice). La simple comparaison des termes proportionnels à OA et à 2Vv × v A/R , où la vitesse introduit un terme de la forme v0 OA déphasé de p/2 , en raison du caractère oscillant du pendule, permet d’en déduire le rapport des axes de l’ellipse (Fig. 7.13) : VT cos u v0 VT cos u b T0 cos u ≈ 10−4 = = = 2 a TJ v0 v0 1/2

puisque TJ ≈ 86 400 s et T0 = 2p (l/g)

≈ 16, 4 s .

y (Nord)

y

x (Est) ¡(Ω T cos µ)t

Ouest Sud

x

 c Dunod – Toute reproduction non autorisée est un délit

F IG . 7.13.

Remarque : De nos jours, on mesure la vitesse de rotation de la Terre V T par des méthodes interférométriques (cf. Optique). On étudie l’évolution de la différence de phase f entre les ondes centimétriques monochromatiques (de longueur d’onde l0 ) provenant d’une étoile et reçues par deux antennes radioélectriques distantes au sol de la base b. Si s est la position de l’étoile comptée à partir du plan médian des antennes et D la distance de l’étoile, on a (cf. Optique) : f=

2p bs l0 D

avec

s = D

VT d t

d’où

df 2p b VT = dt l0

Connaissant l 0, b et d f/ d t, on en déduit VT . c) Référentiel local On peut aussi éviter d’introduire la force supplémentaire de Coriolis en cherchant un référentiel local adapté, précisément celui R en rotation par rapport à R , tel que le vecteur vitesse de rotation de ce dernier par rapport à R g soit compensé par le vecteur vitesse de rotation de R  par rapport à R (Fig. 7.13). Il vient, d’après la composition des vecteurs vitesse de rotation : VR/R g = V R /R + VR/Rg = 0

d’où

VR /R = −VR/Rg = −V T

112

7. Référentiels non galiléens. Dynamique terrestre

Il en résulte que les vecteurs de base de R  coïncident avec ceux de R g : ces deux référentiels sont en translation l’un par rapport à l’autre, mais R n’est pas galiléen, car son origine O est en mouvement accéléré par rapport à R g . Localement, la compensation de la rotation de R par rapport à R g ne concerne que la composante verticale de cette rotation : VR /R · e z = −V T cos u e z = −V T sin l ez Bien que non galiléen, R présente localement des propriétés de simplicité, analogues à celles de R g , puisque la loi fondamentale de la dynamique dans ce référentiel s’écrit simplement : m a A/R  = m g + F T La solution générale de l’évolution de l’angle a , que fait la direction du pendule SA avec la verticale descendante, est alors : a(t) = C1 cos(v0t) + C 2 sin(v 0t) , C 1 et C2 étant deux constantes déterminées par les conditions initiales. Si x et y sont les coordonnées de A dans le plan horizontal, on a, en utilisant la notation complexe, la relation suivante, avec les coordonnées correspondantes x et y dans R : z = x + iy = exp(−iVT cos u t) z  avec z  = (x + iy ) d) Déphasage géométrique L’analyse précédente montre que, dans le référentiel terrestre R , apparaît un terme de phase, d’expression exp(−iVT cos u t) , qui ne s’annule pas après un tour complet de la Terre, mais vaut : exp −iVT cos u ×

2p VT

= exp (−i2p cos u)

Cette différence de phase est celle que l’on observerait entre deux pendules de Foucault synchronisés, excités au même endroit et dans les mêmes conditions, mais à 24 h d’intervalle de temps. Retenons qu’après 24 h , le pendule présente une différence de phase de nature géométrique égale à −2p cos u . En ajoutant 2p , ce qui est légitime puisque les angles sont définis à 2p près, on trouve 2p(1 − cos u) , c’est-à-dire l’angle solide défini par le mouvement de la verticale Oz au cours d’une rotation complète de la Terre (Fig. 7.14). Cet angle solide est le flux du champ r/r 3 , avec r = CO , lequel est proportionnel au champ de gravitation, à travers la calotte terrestre supposée sphérique que découpe ce vecteur au cours du mouvement de rotation de la Terre. ΩT N µ

O r

C Terre S F IG . 7.14.

113

Référentiels non galiléens. Dynamique terrestre IV . 3 . — Déviation vers l’est lors d’une chute libre

Au cours d’une chute libre, la force occasionnelle F oc, qui apparaît dans l’expression de la loi fondamentale, est nulle. Il vient donc : maA/R = mg − 2mVT × vA/R À l’aide de la figure 7.15a représentant le vecteur VT dans R, il est facile de voir, qualitativement, que la force de Coriolis est toujours dirigée vers l’est puisque VT est porté par l’axe Tz 0 et v est dirigé initialement suivant la verticale descendante −Oz. z0

Ω

N

T

z

y

z v

Déviation vers l'est (Ox)

θ

v

a)

ΩT

h

T

λ

y Déviation vers l'est (Ox)

O x (est)

z

y (nord)

R b)

F IG . 7.15.

Comme v A/R ≈ −gt ez, il vient, en substituant et en simplifiant : aA/R ≈ g + 2g Vh t ex

Pour obtenir le déplacement vers l’est, projetons cette relation vectorielle selon ex. On trouve (Fig. 7.15b). gt3 VT cos l ax ≈ 2gt V T cos l d’où vx ≈ gt2V T cos l et x ≈ 3 On trouve le déplacement x h en fin de chute, en remplaçant la durée de chute th par la valeur qu’en donne l’équation approchée h ≈ gt2h/2 :

 c Dunod – Toute reproduction non autorisée est un délit

xh ≈

8h3 9g

1/2

V T cos l

La première expérience sur la déviation vers l’est a été tentée sans succès par l’astronome français J. Cassini, dans un puits de l’Observatoire de Paris. Elle a été refaite de nombreuses fois, en 1831, par l’ingénieur allemand F. Reich, dans un puits de mine de Freiberg (l = 51◦ ), de profondeur 158 m ; alors que la théorie prévoyait une valeur de 27, 5 mm, les valeurs expérimentales étaient comprises entre 27, 9 mm et 28, 7 mm. En 1903, Flammarion l’a refaite avec des billes d’acier abandonnées du haut de la coupole du Panthéon (h = 68 m, l = 48 ◦ 51 ) ; la durée approximative de chute était de 3, 72 s. Expérimentalement, il a trouvé 7, 6 mm, alors que la valeur théorique attendue était x ≈ 8 mm.

V . — RÉFÉRENTIELS ACCÉLÉRÉS PAR RAPPORT À LA TERRE Nous avons vu que, pour tout corps situé au voisinage de la surface de la Terre, et ayant une vitesse inférieure à 700 m . s−1 , on pouvait considérer le référentiel terrestre comme un très bon référentiel galiléen, pourvu que l’on remplace le champ de gravitation G par le champ de pesanteur g.

114

7. Référentiels non galiléens. Dynamique terrestre

Désormais, nous nous placerons dans cette hypothèse et nous étudierons le mouvement d’un corpuscule par rapport au référentiel R = Ox  y z , en mouvement accéléré par rapport au référentiel terrestre R = Oxyz. S’introduisent alors de nouvelles forces d’inertie d’entraînement et de Coriolis, associées au mouvement accéléré de R par rapport à R. V . 1 . — Loi fondamentale dans le référentiel non galiléen R Par rapport à R non galiléen, la loi fondamentale appliquée à un point matériel A, de masse m, s’écrit : m aA/R = m g + F oc − m ae − m aC Foc désignant diverses forces occasionnelles, les forces d’inertie étant évidemment relatives au seul mouvement de R par rapport à R. Il convient donc de ne pas les confondre avec la force d’inertie d’entraînement terrestre, qui est contenue dans m g, et avec la force d’inertie de Coriolis terrestre, qui est négligeable. V . 2 . — Champ de pesanteur apparent On définit le poids apparent F p,a de A dans R par la force opposée à la force occasionnelle de tension FT qui maintient A immobile dans R . On a donc, en faisant a A/R = 0 et v A/R = 0 dans l’équation précédente : 0 = m g + FT − m a e d’où

Fp,a = −F T = m g − m a e

En divisant par la masse, on obtient le champ de pesanteur apparent ga : ga = g − ae V . 3 . — Exemples a) Accéléromètre dans un véhicule en mouvement horizontal La masselotte A est suspendue par un fil au plafond d’un véhicule R  en mouvement de translation accélérée selon l’axe horizontal Ox (Fig. 7.16a). Si ax est l’accélération de O selon l’axe du mouvement Ox et si (FT ,x , FT ,y ) désignent les composantes de F T dans la base de R , l’équation d’équilibre FT + m ga = 0 s’explicite selon : FT ,x − max = 0 et

F T ,y − mg = 0 d’où

F T ,x ax = − tan a e = F T ,y g

et a x = −g tan a e

a e étant l’angle d’inclinaison du fil immobile par rapport à la verticale ; dans le cas considéré sur la figure 7.16b, a e < 0. Il est donc possible de mesurer ax par la simple détermination de l’angle d’inclinaison ae, d’où le nom d’accéléromètre donné au pendule. Une autre méthode de détermination de l’accélération du véhicule consiste à mesurer dans R  la période d’oscillation T du pendule, de longueur l. Le pendule oscille dans le champ de pesanteur apparent ga , d’intensité ga = g/ cos a e (Fig. 7.16b). Par conséquent, la période a pour expression : T  = 2p

l cos ae g

1/2

115

Référentiels non galiléens. Dynamique terrestre y

O

y

R

ae

O

g

FT

x −ma e

®e

A

R O

®e A

®e mg

x a)

b) F IG . 7.16.

b) Champ de pesanteur apparent dans une cabine d’ascenseur Considérons une cabine d’ascenseur R  en mouvement accéléré selon la verticale par rapport au référentiel terrestre R (Fig. 7.17a). D’après l’expression du champ de pesanteur apparent, il vient, en projetant sur un axe vertical ascendant : ga · ez = g · e z − ae · ez

d’où

g a,z = −g − ae · e z

i) Si a e · ez > 0 (mouvement accéléré ascendant), alors |ga,z | > g : les corps sont plus pesants.

ii) Si a e · ez < 0 (mouvement accéléré descendant), alors |ga,z | < g : les corps sont moins pesants.

À titre d’exemple, un observateur sur une balance dans la cabine d’ascenseur constate que son poids apparent est supérieur à son poids, dans R, lorsque la cabine a un mouvement accéléré vers le haut. C’est le contraire pour un mouvement accéléré vers le bas. Dans le cas particulier où la cabine est en chute libre, a e · e z = −g. Le champ de pesanteur apparent est alors nul dans la cabine. On dit qu’il y a impesanteur. Cette propriété est utilisée pour entraîner les cosmonautes en impesanteur. Souvent, le laboratoire d’essai est un avion dont on coupe périodiquement les moteurs, une fois en vol (Fig. 7.17b). Le centre de masse de l’avion décrit des portions de paraboles caractéristiques de la chute libre avec une vitesse initiale horizontale ; les cosmonautes sont en impesanteur pendant quelques minutes.

 c Dunod – Toute reproduction non autorisée est un délit

z (km) z aO 

z

O

x A

10 R

g

y

Ascenseur y

O

8

az = 1,8 g az = 0

R 0

x

20 b)

a) F IG . 7.17.

az = -1,8 g 45

65

t (s)

116

7. Référentiels non galiléens. Dynamique terrestre

Remarque : Le mot impesanteur, construit à l’aide du préfixe privatif latin im, a été préféré à apesanteur, formé avec le privatif grec a, car il permet d’éviter la confusion phonétique entre l’apesanteur et la pesanteur. c) Champ de pesanteur apparent dans un référentiel tournant Si le référentiel R  tourne uniformément autour de l’axe Oz de R, avec la vitesse angulaire V, l’accélération d’entraînement a pour expression (Fig. 7.3) : −V2 HA. Par conséquent : g a = g + V2 HA Ce champ de pesanteur apparent permet d’interpréter la forme incurvée que prend la surface d’un liquide dans un flacon astreint à tourner autour d’un axe vertical (Fig. 7.18). On peut montrer en effet que la surface libre du liquide en rotation uniforme prend la forme d’un paraboloïde de révolution (cf. chapitre 28). Ce résultat a été utilisé pour construire de grands miroirs paraboliques destinés à l’observation astronomique (cf. Optique). z H

A

Ω 2 HA ga

g

V F IG . 7.18.

VI . — RÉFÉRENTIELS INERTIELS VI . 1 . — Définition Un référentiel est dit inertiel si on peut y réaliser le principe de l’inertie : tout point matériel soumis à aucune force occasionnelle conserve sa vitesse initiale. Dans ces référentiels R i , en translation rectiligne uniforme les uns par rapport aux autres, les trois directions de l’espace ont les mêmes propriétés ; on dit que, dans ces référentiels, l’espace est isotrope. Notons que le référentiel terrestre R est approximativement galiléen mais non inertiel, puisqu’un corps abandonné dans R tombe en chute libre. Remarque : Si les forces de gravitation peuvent être négligées, par exemple pour les particules chargées dans un champ électromagnétique (cf. chapitre 8), l’anisotropie provoquée par la présence de la Terre est négligeable et le référentiel galiléen peut être considéré comme inertiel. C’est probablement pour cette raison que certains auteurs emploient, dans ce cas, indifféremment les qualificatifs galiléen ou inertiel. VI . 2 . — Loi fondamentale de la dynamique dans un référentiel tombant en chute libre Appliquons la loi fondamentale de la dynamique à un corpuscule A, par rapport au référentiel non galiléen R , en translation et en chute libre par rapport à un référentiel galiléen R.

117

Référentiels non galiléens. Dynamique terrestre

Dans R , d’accélération aO = g, la force d’inertie de Coriolis est nulle, puisque V R /R = 0, et la force d’inertie d’entraînement se réduit à −maO  . En désignant par Foc une force supplémentaire occasionnelle, il vient : d p = mg − ma O + Foc dt

d p = Foc dt

d’où

puisque la force d’inertie d’entraînement compense le poids mg. Si Foc = 0, alors p = Cte. Ainsi, les référentiels tels que R  , en translation et en chute libre par rapport à un référentiel galiléen, sont inertiels. VI . 3 . — Loi fondamentale de la dynamique dans un vaisseau spatial Considérons un vaisseau spatial, en mouvement de translation accélérée par rapport à un référentiel galiléen, sous l’action des seules forces de gravitation exercées par les différents astres : le vaisseau évolue donc dans l’espace interplanétaire, moteurs coupés (Fig. 7.19). z0

z0 R

0

C y0 x0 T x0

y0 R

 c Dunod – Toute reproduction non autorisée est un délit

F IG . 7.19.

La loi fondamentale de la dynamique, appliquée à un corpuscule A, par rapport au référentiel R  , lié au vaisseau spatial, donne, en introduisant le champ de gravitation G produit par tous les astres : d p = mG (A) − mae + Foc dt la force d’inertie de Coriolis étant nulle du fait de la translation. Or, l’application de la loi fondamentale de la dynamique au vaisseau, assimilé à son centre de masse C, de masse M, fournit l’équation (cf. chapitre 13) : MaC = MG (C) soit a C = G (C) Comme, dans un mouvement de translation, l’accélération d’entraînement est celle de n’importe quel point du vaisseau, on a : a e = a C. Il en résulte : d p = m[G (A) − G (C)] + Foc dt Les points A et C étant très voisins, on a : G (A) ≈ G (C)

d’où

d p ≈ Foc . dt

On obtient un résultat analogue au précédent, mais plus général : le référentiel R  , associé à un vaisseau interplanétaire, en mouvement de translation par rapport au référentiel de Copernic R0, sous l’action des seules forces de gravitation, est un référentiel qui n’est pas galiléen mais inertiel. Dans R  , les seules forces à prendre en compte, pour étudier le mouvement d’un point matériel, sont les forces occasionnelles ; si ces dernières sont nulles, ce mouvement est rectiligne uniforme. C’est ce qui se passe dans une cabine spatiale : les objets abandonnés gardent leur état de repos ou de mouvement.

118

7. Référentiels non galiléens. Dynamique terrestre

Remarques : (1) Dans de tels référentiels, où il y a impesanteur, on ne détermine la masse m d’un corps qu’en mesurant sa masse inerte. Cette dernière s’obtient par exemple à l’aide de la période d’oscillation T 0 d’un pendule élastique dont on connaît la raideur : on a v 20 = K /m et T0 = 2p/v0, d’où m = KT02/(4p2 ) (cf. chapitre 10). (2) La différence entre les champs de gravitation en A et en C, G(A) − G(C), est un terme de marée à l’intérieur de la cabine spatiale, que l’on néglige généralement en première approximation. VI . 4 . — Point de vue de la relativité générale La propriété commune à la force de gravitation et aux forces d’inertie d’être proportionnelles à la masse a conduit Einstein, en 1915, à proposer, dans la théorie de la relativité générale, l’abandon de la prise en compte de la gravitation en tant que force, au profit d’une description en termes de structure géométrique de l’espace-temps (cf. Relativité et invariance) ; on dit alors que la gravitation se manifeste par une courbure de l’espace-temps.

CONCLUSION Rappelons les principaux résultats. (1) La formulation de la loi fondamentale de la dynamique du corpuscule est invariante par changement de référentiel, à condition d’ajouter, aux forces produites par les corps situés à proximité, les forces d’inertie d’entraînement et de Coriolis. Ainsi exprimée, cette loi ne privilégie plus les référentiels galiléens : F ie = −mae et FiC = −ma C = −2mV × vA/R V étant le vecteur vitesse de rotation de R  non galiléen par rapport au référentiel galiléen R. (2) Le poids terrestre d’un corps est une combinaison de la force de gravitation et de la force d’inertie d’entraînement : F p = m ∗G − ma e En tenant compte de l’identité des masses grave et inerte et en explicitant, on obtient l’expression suivante du champ de pesanteur terrestre : g = G T (A) + V2T HA + Ga(A) − G a (T ) Les deux derniers termes permettent d’expliquer l’existence de deux marées par jour sur la côte atlantique française. (3) La nature non galiléenne du référentiel terrestre permet d’expliquer la déviation des masses de fluide vers la droite dans l’hémisphère nord ainsi que quelques expériences précises faites au milieu du XIXe siècle. Rigoureusement la loi fondamentale dans un référentiel terrestre a pour expression : maA/R =

F oc + mg − 2mV T × v

C’est la force de Coriolis terrestre qui permet d’interpréter le comportement du pendule de Foucault et la déviation vers l’est des particules lors d’une chute. (4) Pour la plupart des besoins pratiques et techniques, on peut considérer que le référentiel terrestre constitue une très bonne approximation d’un référentiel galiléen, pourvu que l’on introduise le concept de poids et que les vitesses soient inférieures à 700 m . s−1 . (5) Dans un référentiel R , en mouvement accéléré par rapport au référentiel terrestre R, la combinaison naturelle du champ de pesanteur terrestre et de l’accélération d’entraînement de R  par rapport à R donne le champ de pesanteur apparent : ga = g − ae

Le cas singulier où ga = 0 correspond à l’impesanteur.

119

Référentiels non galiléens. Dynamique terrestre

(6) Les référentiels inertiels sont les référentiels dans lesquels on peut réaliser le principe d’inertie : un corpuscule abandonné garde son état de repos ou de mouvement rectiligne uniforme s’il est isolé ou pseudo-isolé. Une cabine spatiale en mouvement, moteurs coupés, constitue un référentiel inertiel, la force de gravitation étant compensée par la force d’inertie d’entraînement. Ce résultat souligne la singularité de l’interaction de gravitation par rapport aux autres forces fondamentales et sa relation étroite avec les forces d’inertie. Il est à la base de la théorie de la relativité générale.

EXERCICES ET PROBLÈMES P7– 1. Angle entre G et g Déterminer, en fonction de la latitude l, l’angle a que fait la verticale du lieu avec la direction du champ de gravitation. À quelle latitude la valeur de a est-elle maximale ? Calculer l’angle a à Toulouse (l = 43 ◦35 ). P7– 2. Influence de la force de Coriolis sur une balle de fusil Une balle de fusil est tirée, horizontalement, dans la direction du nord, depuis un point de la Terre, de latitude l = 43◦ . Sa vitesse initiale est de 1 000 m . s−1. 1. Calculer la position de l’impact sur une cible située à 100 m, en ne tenant compte que de la force de pesanteur. 2. Étudier qualitativement l’influence de la rotation de la Terre sur le mouvement du projectile. 3. Donner les équations différentielles du mouvement en négligeant la résistance de l’air mais en tenant compte de la rotation de la Terre. 4. Trouver, en cm, la déviation vers l’est due à la rotation de la Terre autour de l’axe des pôles ? P7– 3. Analyse quantitative du mouvement d’un pendule de Foucault

 c Dunod – Toute reproduction non autorisée est un délit

On considère un pendule simple constitué d’une masselotte A (masse m = 30 kg) suspendue à l’extrémité inférieure d’un filin, de longueur l = 67 m. L’autre extrémité est fixée en un point O1 placé à une hauteur égale à l sur la verticale du lieu de latitude l (Fig. 7.12). 1. Expliciter l’équation du mouvement dans la base du référentiel terrestre local R = Oxyz, Oz étant la verticale ascendante et Ox l’axe local orienté vers l’est. On exprimera la tension du fil sous la forme FT = −F T O 1A/l. 2. En négligeant les mouvements verticaux, trouver les équations différentielles suivantes qui régissent le mouvement dans le plan horizontal : ¨ x − 2V T sin l y˙ + v 20x = 0 et

¨ y + 2V T sin l x˙ + v20 y = 0

VT étant la vitesse de rotation de la Terre autour de l’axe des pôles et v20 = g/l. 3. Résoudre le système d’équations différentielles précédent, sachant qu’initialement x = x m , y = 0, ˙x = 0 et y˙ = 0. On utilisera la méthode complexe en posant z = x + iy avec i2 = −1. 4. Quelle est la forme de la solution dans le système d’axes tournant autour de la verticale avec la vitesse angulaire VT sin l dans le sens nord-est-sud-ouest ? En déduire la durée T d’un tour complet de ce système d’axes tournants. Calculer T pour différentes latitudes : l = 90 ◦ (pôles), l = 43◦ 35  (Toulouse) et l = 0◦ (équateur).

120

7. Référentiels non galiléens. Dynamique terrestre

P7– 4. Mouvement d’une masselotte dans un véhicule accéléré Un véhicule a un mouvement de translation uniforme de vitesse v sur une route curviligne d’équation cartésienne y = f (x). On lui associe un référentiel R1 = O 1xyz, en translation par rapport au référentiel terrestre R = Oxyz. Une masselotte A, de masse m, liée à l’origine O1 par un ressort de raideur K, de longueur naturelle l0, évolue le long de l’axe O1 y. 1. Montrer que la composante cartésienne, suivant la verticale ascendante Oy, de l’accélération de O1 dans R s’écrit v 2f  y1 (1 + f 2 )2 y A f  et f  désignant les dérivées première et seconde de f par rapport à x. 2. Écrire l’équation différentielle à laquelle satisfait le mouveK S x1 ment de A dans R1. O1 h 3. Calculer la tension T du ressort dans le cas où, en raison M M2 1 O d’une force supplémentaire de frottement visqueux, A acquiert rax 2l pidement une position d’équilibre dans R1 . Comparer alors T au poids, dans les cas où la route forme une bosse ou un creux. Conclure, F IG . 7.20. en utilisant la notion de poids apparent. 4. Le profil de la route forme une bosse assimilable à un arc de parabole, M 1SM 2, dont les caractéristiques sont données sur la figure 7.20. Pour quelle valeur de la vitesse y a-t-il impesanteur en S ? P7– 5. Pendule de Schüler On transporte, dans un véhicule, un pendule simple O  A (longueur l) dont le point de suspension est fixe par rapport au véhicule R . Par rapport au référentiel terrestre R, le véhicule a une trajectoire circulaire dont le centre est celui T de la Terre et dont le rayon est approximativement son rayon RT = 6 400 km. O

1. Écrire vectoriellement le théorème du moment cinétique pour le pendule, au point de suspension O . Quelle doit être la vitesse de rotation angulaire V de R  par rapport à R pour que la direction de O A soit fixe dans R ? 2. Quelle est, dans R, la période de ce pendule ? P7– 6. Influence de la force de Coriolis terrestre sur le lit d’un fleuve On souhaite estimer l’influence de la force de Coriolis terrestre sur l’inclinaison du lit d’un fleuve, de largeur L = 300 m , qui s’écoule vers le nord à la vitesse v 0 = 2 m.s−1 en un point de la surface de la Terre de latitude l = 43, 5 ◦ . On désigne par R le référentiel terrestre local dans lequel Oz est la verticale ascendante, Oy l’axe orienté vers le nord et Ox l’axe suivant l’est du lieu. 1. Écrire, sous sa forme vectorielle rigoureuse, la loi fondamentale de la dynamique appliquée à un point A , de masse m , en mouvement horizontal par rapport à R , sous l’action notamment de son poids et de la réaction R qu’exerce sur lui son environnement. Comment est orientée la force de Coriolis locale ? 2. Quelles sont les trois équations du mouvement ? Que deviennent-elles pour le problème considéré ? 3. Exprimer les composantes de la réaction R . En déduire la dénivellation entre les deux rives du fleuve.

121

Référentiels non galiléens. Dynamique terrestre P7– 7. Chute des corps et relativité générale

On étudie la chute des corps dans un référentiel terrestre, en analysant différentes expériences historiques, effectuées successivement par Galilée en 1602, Newton en 1687, Guglielmini en 1791 à Bologne ( lG = 44, 5◦ ), Benzenberg en 1802 à Hambourg ( l B = 53, 5◦ ), Reich en 1831 à Freiberg ( lR = 50, 9◦ ), Hall en 1902 à Harvard ( lH = 42, 5 ◦ ) et Flammarion en 1903 à Paris ( l F = 48, 8◦ ). Pour cela, on analyse la chute d’une bille, de masse m , abandonnée (sans vitesse initiale) d’une hauteur H , à un instant pris comme origine. Dans les expériences précédentes, les hauteurs de chute étaient respectivement : HG = 78, 3 m

H B = 76, 3 m

HR = 158, 5 m

HH = 23, 0 m

HF = 68, 0 m

1. a) Rappeler l’écriture rigoureuse de la loi fondamentale par rapport au référentiel terrestre ; b) Expliciter les quatre termes vectoriels qui composent le champ de pesanteur terrestre g , en donnant la signification physique de chacun d’entre eux, et rappeller la définition de la verticale. Dans la suite, on prendra g = 9, 80 m.s−2 . c) Retrouver, en la justifiant, l’équation simplifiée de la chute libre, telle qu’elle a été analysée par Galilée et Newton. En déduire vz (t) et z(t) . Quelle est la durée de chute la plus grande parmi les cinq expériences précédentes ? Calculer sa valeur. d) En comparant l’équation différentielle simplifiée précédente à celle que l’on obtiendrait en soumettant une bille, de même masse, mais de charge électrique q , à un champ électrique constant E , dites en quoi consiste la singularité fondamentale de la force de gravitation, à l’origine de la théorie de la Relativité Générale. 2. Les expériences de Guglielmini, Benzenberg, Reich, Hall et Flammarion ont établi que la chute des corps s’accompagnait d’une déviation par rapport à la verticale. a) Montrer que la trajectoire de la bille s’écarte légèrement de la verticale, dans un sens que l’on précisera, d’abord dans l’hémisphère nord, ensuite dans l’hémisphère sud. En quels endroits de la surface de la Terre cette déviation est-elle maximale ou minimale ? b) Établir l’expression vectorielle approchée suivante de l’accélération de la bille, que l’on obtient en négligeant les composantes de la vitesse dans un plan horizontal :

 c Dunod – Toute reproduction non autorisée est un délit

a = g + 2gt VT cos l ex ex étant un vecteur unitaire que l’on définira au lieu considéré. c) Quelle est la composante a x , suivant l’axe Ox , de l’accélération de A ? En déduire, selon ce même axe, la vitesse vx et la déviation x par rapport à la verticale. d) Trouver les expressions approchées suivantes de v x et x , après une chute de hauteur H : v x = 2HVT cos l

et

x=

8H 3 9g

1/2

VT cos l

e) Les valeurs expérimentales de x , mesurées par Gulielmini, Benzenberg, Reich, Hall et Flammarion, furent, respectivement : xG = 1, 9 cm

xB = 0, 9 cm

xR = 2, 8 cm

xH = 0, 15 cm

x F = 0, 63 cm

Selon vous, comment concrètement la déviation a-t-elle été mesurée ? Calculer les valeurs des déviations prédites par la théorie. Commenter l’écart entre expérience et prévision théorique.

122

7. Référentiels non galiléens. Dynamique terrestre

3. Appliquer la loi fondamentale à la bille par rapport au référentiel R  , en translation, rectiligne, mais en chute libre par rapport à R . Ce référentiel non galiléen est qualifié d’inertiel, ou de naturel par Einstein. Pourquoi ? Que peut-on dire, dans ce référentiel R , de la déviation étudiée précédemment dans R ? P7– 8. Effet Eötvös On appelle effet Eötvös, du nom du physicien hongrois Loránd Eötvös, la différence qui existe entre le champ de pesanteur gb,e mesuré à bord d’un bateau naviguant sur l’océan, vers l’est, avec une vitesse vb,e et le champ analogue g b,o mesuré le bateau se dirige vers l’ouest, à la même vitesse. Pour comparer ces champs au champ de pesanteur terrestre mesuré en un point fixe de la surface de la Terre, on accroche une masselotte, de masse m , à l’extrémité inférieure d’un ressort, l’extrémité supérieure étant liée à un point fixe du bateau. Dans l’expérience réalisée par Eötvös, en 1908, sur la Mer noire (latitude l = 45◦ ), m = 10 kg et v b,e = 25 m.s−1 . 1. Rappeler la définition expérimentale du poids terrestre d’un corpuscule A , ainsi que la relation entre le champ de pesanteur g , le champ de gravitation G et l’inertie, en négligeant l’influence des marées. 2. a) On introduit le référentiel R b = Txb ybz0 , lié au bateau B , tel que Tz0 coïncide avec l’axe de révolution de la Terre, Txb soit défini par la position B du bateau et Tyb orienté vers l’est (Fig. 7.21). Montrer que, dans Rb , la relation précédente devient : g b,e = G + V2b,e KB K étant la projection de B selon l’axe de révolution de la Terre et V b,e une vitesse angulaire que l’on déterminera en fonction de la vitesse de rotation de la Terre autour de son axe VT , de v b,e , du rayon terrestre RT et de la latitude l du lieu. b) Exprimer la différence g b,e − gb,o . En déduire la projection de cette différence vectorielle selon la verticale du lieu et calculer sa valeur. Quelle erreur relative fait-on en négligeant cet effet ? c) Cet effet dépend-il de l’hémisphère ? Qu’obtiendrait-on si le bateau se dirigeait vers le nord ou vers le sud ? d) Comparer cet effet à la force de Coriolis qui s’exerce sur l’unité de masse du bateau. Commenter. z0

ΩT K

B

v b,c y b

T ¸

Rb xb

x0 Rg

F IG . 7.21.

y0

8 Particule chargée dans un champ électromagnétique stationnaire L’étude du mouvement d’une particule chargée dans un champ électromagnétique présente un intérêt considérable en physique, puisque son champ d’application est celui de l’optique corpusculaire et des grands accélérateurs de particules. Nous allons centrer notre analyse principalement sur les champs uniformes et stationnaires ; en outre, ces champs seront supposés suffisamment faibles pour que le domaine de validité soit strictement newtonien, lequel est défini par des vitesses de particules très inférieures à c ≈ 3 × 108 m . s−1 . Le prolongement naturel de cette étude relève de la relativité restreinte (cf. Relativité et invariance). Le point de départ est la loi fondamentale de la mécanique newtonienne dans le référentiel galiléen du laboratoire : dp = ma = F dt où p = mv est la quantité de mouvement de la particule et F la force exercée par le champ électromagnétique (E, B) ; en effet, on sait que la pesanteur est négligeable devant l’interaction électromagnétique, ce qui rend R inertiel avec une excellente approximation (cf. chapitre 7).

 c Dunod – Toute reproduction non autorisée est un délit

Nous allons tout d’abord rappeler l’écriture de la force Lorentz qui s’exerce sur une particule chargée.

I . — FORCE DE LORENTZ Considérons un système de charge et de courant qui crée, en une région de l’espace, un champ électrique et un champ magnétique, tous deux uniformes et stationnaires. Sans entrer dans le détail d’une telle réalisation, rappelons seulement que le condensateur plan et le solénoïde long produisent, respectivement avec une bonne approximation, de tels champs électrique et magnétique (Fig. 8.1). B + + + + + + + + +

d

E − − − − − − − − −

+ −

U

I

+ R

a)

b) F IG . 8.1.



124

8. Particule chargée dans un champ électromagnétique stationnaire

Les champs obtenus s’écrivent (cf. Électromagnétisme) : i) E = U /d, U étant la différence de potentiel entre les deux plaques du condensateur distantes de d, ii) B = m 0nI où m 0 = 4p × 10−7 SI, I étant l’intensité du courant et n le nombre de spires par unité de longueur. La force de Lorentz qui s’exerce sur une particule A, de charge q et de vitesse v par rapport à un référentiel terrestre, a pour expression : F = q(E + v × B) On voit que, pour une vitesse de l’ordre de c/10, un champ magnétique de valeur 0, 01 T, ce que l’on réalise aisément, est aussi efficace qu’un champ électrique E de valeur B × v = 3 × 10 5 V . m−1 . Cette force est bien plus intense que le poids ; par exemple, alors que le poids d’un électron est meg ≈ 8, 8 × 10 −30 N, une valeur typique de la force électrostatique est celle produite par un champ E = 0, 3 MV . m−1 sur la charge élémentaire e : Fe = eE ≈ 4, 8 × 10 −14 N.

II . — PARTICULE CHARGÉE DANS UN CHAMP ÉLECTRIQUE II . 1 . — Équations du mouvement Considérons une particule, pénétrant avec une vitesse v 0 dans une région où règne une champ électrique E. Choisissons, pour la commodité, le plan Oxy du référentiel terrestre R tel que O coïncide avec la position de la particule à l’instant initial (Fig. 8.2) ; et les axes soient tels que le champ électrique E et la vitesse initiale v0 s’écrivent respectivement : E = E ex

avec

E > 0 et v0 = v0 cos a 0 ex + v0 sin a0 ey

Notons que ce choix ne restreint en rien la généralité de l’étude. L’équation fondamentale de la dynamique s’explicite comme suit dans la base (ex , e y, e z) de R : dv = qE m dt

donne

y

¨ x E m ¨ y = q 0 ¨ 0 z

soit

¨ x = a1 ¨ y=0 ¨ z=0 y

E

O

v+ v−

α0

O

x F IG . 8.2.

a1 =

E +

v0

avec



x

F IG . 8.3.

Compte tenu des conditions initiales (v 0 et r 0 = 0), il vient : x˙ = a1 t + v 0 cos a 0 y˙ = v0 sin a0 ˙z = 0

a1 2 t + (v0 cos a 0)t 2 y = (v 0 sin a0 ) t x=

d’où

z = Cte = 0

qE1 m

125

Particule chargée dans un champ électromagnétique stationnaire

La trajectoire, qui est contenue dans le plan Oxy, s’obtient en éliminant le temps entre les deux premières équations : x=

a1 y2 y + 2 v02 sin2 a 0 tan a 0

avec

a1 =

qE m

C’est une parabole passant pas O, puisque x = 0 pour y = 0. La figure 8.2 correspond à une particule de charge positive. II . 2 . — Cas où v0 est colinéaire à E Si l’angle a 0 est nul, on trouve évidemment : x=

a1 2 t + v0 t 2

˙x = a1 t + v 0

y=0

et

˙y = 0

Exemple : Un proton (charge e), initialement au repos, est accéléré par un champ de 30 kV . m −1 , pendant 1 ns. Il acquiert ainsi l’accélération et la vitesse suivantes (Fig. 8.3) : a+ =

eE eE = 2, 87 × 1012 m . s −2 et v + = t = 2, 87 × 103 m . s −1 mp mp

Pour un électron (charge −e), l’accélération et la vitesse acquises sont : eE eE a− = − = −5, 27 × 1015 m . s −2 et v − = − t = −5, 27 × 10 6 m . s −1 me me II . 3 . — Cas où v0 est normal au champ E. Déviation électrique de la trajectoire Si v 0 est normal à E, il vient, en faisant a 0 = p/2 (Fig. 8.4) :

 c Dunod – Toute reproduction non autorisée est un délit

x=

a1 t2 2

˙x = a1 t

y = v0 t

˙y = v 0

Supposons que la zone d’action du champ (E non nul) soit définie par : 0  y  Le. La trajectoire, qui est rectiligne pour y < 0, est déviée de l’angle ue . Évaluons cet angle : la particule sort de la zone à l’instant t = Le /v0 : a L2 a Le xs = 1 2e et x˙ s = 1 v0 2v 0 On en déduit l’angle de déviation (algébrique) ue relié aux coordonnées par l’équation : tan u e = −

a1L e ˙xs =− 2 ˙ys v0

d’où

tan u e = −

qEL e mv02

Sur la figure 8.4, la charge de la particule est positive et donc l’angle ue négatif. Pour un électron, de vitesse initiale v0 = 5, 27 × 10 6 m . s −1 et accéléré par un champ E = 3 kV . m −1 dans une région où L e = 10−2 m, on trouve : tan u e ≈ ue = 0, 19 ≈ 10◦.

126

8. Particule chargée dans un champ électromagnétique stationnaire y

µe

A

O

+++++++++

S

y Zone d'action de E Le E

¼=2 x

v0

E

F

A P v

O Électron

F IG . 8.4.

®0 x

d F IG . 8.5.

II . 4 . — Propriétés de focalisation du champ électrique Considérons une source qui émet des électrons en O, autour d’une direction moyenne avec une vitesse moyenne de v 0 (Fig. 8.5) : l’angle de (e x , v 0) s’écrit initialement a 0 + ε avec |ε|  a 0 . D’après ce qui précède, puisque a1 est négatif et a0 = p/2, on peut écrire : x=

−eE y2 y + 2 2m v 20 sin a0 tan a0

Cherchons le point d’intersection F de la trajectoire avec l’axe Oy : xF = 0 entraîne

1 −eEy F mv 20 = 0 soit y sin(2a0 ) + = F tan a0 eE 2mv20 sin 2 a0

On voit que yF est maximal pour a0 = p/4. Développons y F au voisinage de a 0 = p/4 : yF = (y F)0 +

d yF da

(a − a0 ) + 1!

d2 yF d a2

(a − a0 )2 2!

avec

(y F)0 = yF (a 0) =

mv20 eE

Comme (d yF / d a) 0 = 0, y F = (yF )0 quel que soit ε, au second ordre près. Il y a donc focalisation en F du faisceau issu de O. Ces propriétés sont utilisées dans les spectromètres électrostatiques pour électrons. Dans ce cas, on a, en F pour a = p/4 : 2Ek mv20 d = eE eU si U désigne la différence de potentiel entre les électrodes et d la distance qui les sépare. Le système précédent peut donc servir à séparer des électrons d’énergies cinétiques initiales différentes. On définit alors la dispersion en énergie : 2d Dy De = F = DEk eU Remarquons que xmax doit être inférieur à d/2 pour que les électrons issus de O n’atteignent pas les électrodes. Déterminons la position du sommet P de la parabole : yF=

dx dy

P

=0=−

1 mv20 yP + 2 eE v20 sin a0 tan a0

Par conséquent :

soit

yP =

mv 20 sin(2a 0) y F = 2 2 eE

mv02 d entraîne Ek < eU < 4eE 2 Ce spectromètre ne présente donc d’intérêt que pour des électrons d’énergie cinétique initiale assez faible. xP =

127

Particule chargée dans un champ électromagnétique stationnaire

III . — PARTICULE CHARGÉE DANS UN CHAMP MAGNÉTIQUE III . 1 . — Équation du mouvement Choisissons l’axe Oz du référentiel terrestre R = Oxyz suivant B et Ox dans le plan que fait B avec la vitesse initiale de la particule chargée A : v0 = v0 cos a0 ez + v 0 sin a 0 e x , ce qui simplifie les calculs sans restreindre la généralité du problème (Fig. 8.6). La trajectoire représentée est relative à un électron (q = −e < 0, R < 0). z Électron

A

B α0

h y

O

v0 x

F IG . 8.6.

D’après la loi fondamentale de la dynamique appliquée à la particule, on a : m

dv = qv ×B dt

dv = vc (v × e z) dt

soit

en posant

vc =

qB m

La quantité vc, homogène à l’inverse d’un temps, est appelée la pulsation cyclotron. Notons qu’ainsi définie c’est une quantité algébrique dont le signe dépend de celui de la charge. En explicitant dans la base de R, on obtient : x ¨ y ¨ ¨z

= vc

x˙ y˙ z˙

×

0 0 1

soit

x = vc ˙y ¨ y = −vc ˙x ¨ ¨ z=0

La troisième équation s’intègre sans difficulté, compte tenu des conditions initiales :

 c Dunod – Toute reproduction non autorisée est un délit

˙z = Cte = v0 cos a 0

d’où

z = (v0 cos a 0) t

Pour résoudre le système constitué par les deux premières équations différentielles, intégrons la deuxième équation : ˙y = −vc x + Cte = −vc x , car initialement ˙y = 0 lorsque x = 0. En remplaçant ˙y dans la première, on obtient : ¨ x + v2c x = 0 La solution de cette équation est bien connue (cf. annexe 3) : x = A cos(v ct + f). En tenant compte des conditions initiales, il vient : ˙ 0 = −vc A sin f = v 0 sin a0 (x)0 = A cos f = 0 et (x) ce qui donne f = p/2, A = (−v0 sin a0 )/vc et par conséquent x = (v0 sin a0 /v c) sin(vc t). On en déduit : ˙y = −(v 0 sin a0) sin(vc t). Il en résulte, en intégrant : y=

v0 sin a 0 cos(vc t) + Cte vc

soit

y=−

v 0 sin a0 [1 − cos(vc t)] vc

128

8. Particule chargée dans un champ électromagnétique stationnaire

puisqu’à t = 0, y = 0. Ainsi, les équations paramétriques de la trajectoire sont : x=

v 0 sin a0 sin(vc t) vc

y=−

v 0 sin a0 [1 − cos(v c t)] vc

z = (v0 cos a0 ) t

Les deux premières équations s’écrivent aussi : x = R sin(vc t)

et y = −R[1 − cos(vc t)]

en posant

R=

mv 0 sin a0 p⊥ v0 sin a 0 = = qB qB vc

Notons que la norme de la vitesse reste constante au cours du mouvement, ce que l’on établit rapidement à partir du théorème de l’énergie cinétique, puisque la force magnétique ne travaille pas. Remarque : Il existe une autre méthode d’intégration du système différentiel formé par les deux premières équations différentielles. Elle consiste à poser x = x + j y ( j 2 = −1) et à effectuer la somme des deux premières équations une fois la deuxième multipliée par j. On obtient alors : ¨ x = −jv c ˙x, ce qui donne, compte tenu des conditions initiales : ˙x = (v0 sin a0 ) exp(−jv c t)

et x =

jv 0 sin a0 [exp(−jvc t) − 1] vc

En séparant les parties réelle et imaginaire, on retrouve les résultats précédents. III . 2 . — Trajectoire Si on élimine t entre x et y, on obtient : x2 + (y + R) 2 = R

2

C’est l’équation d’un cercle de rayon R = |R| dont le centre C a pour coordonnées (0, −R). Ainsi, le mouvement d’une particule dans un champ magnétique est la composition d’un mouvement circulaire uniforme dans le plan normal à B et d’un mouvement rectiligne uniforme suivant B. C’est donc un mouvement hélicoïdal de pas : h=

p 2pv0 cos a0 = 2p vc qB

p étant la quantité de mouvement suivant la direction de B. La quantité B R = (mv0 sin a0)/q = p⊥ /q est appelée la rigidité magnétique. Elle est égale à la quantité de mouvement de la particule dans le plan perpendiculaire à B par unité de charge. Exemple : Calculons v c pour un électron dans un champ magnétique de 10 mT : vc ≈ −

1, 6 × 10 −19 × 10−2 ≈ −1, 76 × 10 9 rad . s−1 0, 91 × 10−30

Si cet électron a une vitesse de 107 m . s−1 , perpendiculaire au champ, la rigidité magnétique est : BR ≈

0, 9 × 10−30 × 10 7 ≈ 5, 6 × 10 −5 T . m = 5, 6 mT . cm 1, 6 × 10−19

Ainsi, avec un champ de 1 mT, la valeur de R est 5, 6 cm.

129

Particule chargée dans un champ électromagnétique stationnaire III . 3 . — Déviation magnétique de la trajectoire d’une particule chargée

Il s’agit d’évaluer l’efficacité d’un champ magnétique pour courber la trajectoire d’une particule chargée. Dans le cas d’une particule, de charge positive, de vitesse initiale v0 portée par Ox et perpendiculaire à B, la trajectoire est une portion de cercle OS limitée par la zone d’action de B (Fig. 8.7). Au-delà de S, la trajectoire est une droite. Notant L m la longueur de l’arc OS, il vient : u m = OS/R avec R = |mv0 /qB| et OS = L m. Par conséquent : um =

qBL m mv0

Notons que um augmente comme 1/v 0, en raison de la proportionnalité de la force magnétique et de la vitesse, alors que ue varie comme 1/v20 . y

Zone d'action de B θm

z A v0

B Lm O

α0

θm

B y

S

O

v0

x

x F IG . 8.7.

F IG . 8.8.

III . 4 . — Focalisation magnétique a) Focalisation axiale (suivant B)

 c Dunod – Toute reproduction non autorisée est un délit

Considérons un faisceau de particules, légèrement ouvert, issues d’un point source O et se dirigeant en moyenne suivant la direction z du champ magnétique B : l’angle maximal que font les trajectoires issues de O avec la direction de B est très petit : a 0  1 (Fig. 8.8). D’après son expression, le pas de l’hélice varie peu avec a 0 ; en effet : 2pmv0 2pmv 0 2 Dh = − sin a 0 Da 0 ≈ − a0 d’où Dh ≈ 0 qB qB au deuxième ordre près en a 20. Un pinceau de particules est donc focalisé par un champ magnétique parallèle à sa direction moyenne. b) Focalisation radiale (normale à B) Si le faisceau de particules entre dans une région où règne un champ magnétique B, avec une vitesse normale à B, il suffit d’imposer a 0 = p/2 dans les solutions précédentes. Le mouvement est alors circulaire. Sur la figure 8.9, les particules sont des électrons. En réalité, toutes les particules n’ont pas leur vitesse initiale suivant Ox ; seule la direction moyenne du faisceau coïncide avec Ox. y

O

εF B

2R

v0 O F IG . 8.9.

ε

x

130

8. Particule chargée dans un champ électromagnétique stationnaire

Considérons le point F, intersection avec l’axe Oy de la trajectoire circulaire d’une particule dont la vitesse initiale fait l’angle ε, avec l’axe Ox. Comme ε est petit, yF = 2R cos ε = 2R(1 − ε2 /2). La distance qui sépare F de Fm, défini par ε = 0, est donc : DyF = yF,m − yF ≈ Rε 2

d’où

Dy F ≈ 0

au second ordre près en ε. Ainsi, yF ne change pratiquement pas lorsque ε varie, ce qui prouve que le système a des propriétés de focalisation. On définit alors la dispersion en énergie d’un tel système, utilisé en spectromètre magnétique, par (cf. Optique) : De =

2m 1 DyF Dy F Dv 0 R = = = DEk Dv0 DEk |q|B mv0 Ek

puisque

yF ≈ 2R = 2

mv 0 |q|B

et Ek =

1 2 mv 2 0

IV . — CHAMPS ÉLECTRIQUE ET MAGNÉTIQUE SIMULTANÉS Une particule (masse m, charge q) pénètre en I, avec une vitesse v 0 , dans une région où règnent un champ électrique E et un champ magnétique B, normaux à v 0 (Fig. 8.10). IV . 1 . — Champs électrique et magnétique parallèles Si les champs E et B sont parallèles, les angles de déviation électrostatique (dans le plan Izx) et magnétique (dans le plan Iyz) ont pour expressions respectives : qEL e qBL m ue = et um = 2 mv 0 mv0 En éliminant v0, on trouve l’expression suivante de la charge massique de la particule : q E Le u2 = 2 2 m m B L m ue Pour déterminer q/m, il suffit donc de recevoir les particules sur un écran luminescent OXY parallèle au plan Ixy. Les coordonnées X et Y du point d’impact permettent de déterminer respectivement u m et ue et par conséquent le rapport q/m. Historiquement, cette méthode a contribué à dégager le concept d’électron. Zone d'action de E et B

x

I y

v0 Y

X

E

A

S

E

x

X

B

S O

z y

v0

z

F Fente

A

B

F IG . 8.10.

Y F IG . 8.11.

IV . 2 . — Champs électrique et magnétique perpendiculaires Une source S envoie des particules chargées avec une vitesse v 0 = v 0 ex , dans une région où règnent un champ électrique E = E ex et un champ magnétique B = B e y perpendiculaires entre eux

131

Particule chargée dans un champ électromagnétique stationnaire (Fig. 8.11). La loi fondamentale appliquée à une particule, de masse m et de charge q, donne : ma = q(E + v × B) d’où : a = a1 e x + vc v × e y avec a1 = qE/m et vc = qB/m. a) Solution générale Il vient, en explicitant dans une base associée au référentiel du laboratoire R : ¨ x ¨ y ¨z

= a1

1 0 + vc 0

x˙ y˙ × z˙

0 1 0

soit

¨ x = a1 − v c ˙z ¨ y=0 ¨z = v c x˙

La deuxième équation s’intègre sans difficulté, compte tenu des conditions initiales : y˙ = Cte = 0 d’où y = Cte = 0 Pour résoudre le système constitué par les deux autres équations différentielles, intégrons la troisième équation : ˙z = vc x + Cte = v cx + v 0 , car initialement z˙ = v 0 lorsque x = 0. En remplaçant ˙z dans la première, on obtient : ¨ x + v 2c x = a1 − v cv 0

La solution de cette équation est bien connue (cf. annexe 4) : x = A cos(vc t + f) + (a 1 − v cv 0)/v2c . Avec les conditions initiales, il vient : (x) 0 = A cos f +

a 1 − vc v0 ˙ 0 = −vcA sin f = 0 = 0 et (x) v 2c

ce qui donne f = 0, A = −(a1 − v cv 0)/v 2c . Ainsi : x=

a 1 − vc v 0 v2c

[1 − cos(v ct)]

On en déduit : ˙z =

a1 − v c v0 vc

[1 − cos(v ct)] + v0 =

a1 a1 − v cv0 − cos(v c t) vc vc

 c Dunod – Toute reproduction non autorisée est un délit

et donc z en intégrant, sachant qu’à t = 0, z = 0 : z=

a1 v2c

v ct −

a 1 − v c v0 v2c

sin(v ct)

Ainsi, les équations paramétriques de la trajectoire, contenue dans le plan Ozx sont : z=

a1 v2c

vc t −

a1 − vc v 0 v2c

sin(vc t)

et x =

a1 − vc v0 v2c

[1 − cos(vc t)]

Les courbes correspondantes sont des trochoïdes (cf. Exercices). b) Trajectoires cycloïdales Les courbes précédentes deviennent des cycloïdes si les coefficients qui affectent v ct et sin(vc t) sont égaux en valeur absolue : a1 = |a1 − v cv 0 | ce qui est réalisé pour v0 = 0 ou v 0 = 2a1 /vc = 2E/B.

132

8. Particule chargée dans un champ électromagnétique stationnaire i) Vitesse initiale v 0 nulle Les équations paramétriques se réduisent à : z=

E [v ct − sin(v c t)] Bv c

et x =

E Bv c

[1 − cos(vc t)]

Cette courbe a déjà été étudiée (cf. chapitre 2) ; c’est une cycloïde. Elle est représentée en trait plein sur la figure 8.12. x 2E Bv c 0

pE /Bvc

3pE /B vc z

− 2E Bv c F IG . 8.12.

ii) Vitesse initiale v0 égale à 2E/B Les équations paramétriques se simplifient selon : z=

E Bvc

[vc t + sin(vc t)]

et x = −

E Bv c

[1 − cos(vct)]

La trajectoire est semblable à la précédente (courbe en pointillés sur la figure 8.12). C’est aussi une cycloïde. Remarque : Les deux trajectoires se déduisent l’une de l’autre à l’aide d’une simple translation.

V . — APPLICATIONS V . 1 . — Mesure de la quantité de mouvement d’une particule L’équation donnant le rayon de courbure de la trajectoire circulaire d’une particule chargée, dans un champ magnétique perpendiculaire à sa vitesse initiale, permet de comparer les quantités de mouvements de particules de même charge électrique. En effet, on a : p1 p2 = qB = R1 R2 Cette propriété est utilisée dans l’étude des collisions de particules rapides (cf. Relativité et invariance). V . 2 . — Confinement magnétique Si le champ magnétique n’est pas uniforme, les particules chargées ont des trajectoires hélicoïdales dont le rayon et le pas varient d’une région à l’autre. Une analyse détaillée montre que, sous certaines conditions, une particule peut décrire une hélice dont le pas diminue autour de la direction moyenne du champ et rebrousser chemin (Fig. 8.13). Ces résultats sont utilisés pour confiner des plasmas à très haute température.

133

Particule chargée dans un champ électromagnétique stationnaire +−

Cathode

V Anode

x Objet

Ao Lentille magnétique

z

z = z0 F IG . 8.13.

Ai

Image

F IG . 8.14.

V . 3 . — Optique corpusculaire En raison des propriétés de focalisation des champs électrique et magnétique, l’optique corpusculaire constitue, à elle seule, un vaste champ d’application de la dynamique des particules chargées dans un champ électromagnétique non uniforme.

 c Dunod – Toute reproduction non autorisée est un délit

Dans un microscope électronique, les électrons sont émis par une cathode, avec une vitesse initiale négligeable, par effet thermoélectronique ou à l’aide d’un champ électrique extracteur. Ils sont alors accélérés vers une anode trouée, portée à un potentiel positif par rapport à la cathode (Fig. 8.14). Lorsque ce potentiel accélérateur est de 100 kV, ils émergent du canon du microscope avec une vitesse qui est de l’ordre de 0, 558 c, c étant la vitesse de la lumière dans le vide. À la sortie du canon les électrons sont soumis à l’action d’électro-aimants qui modifient leurs trajectoires, comme le font les lentilles de verre avec les rayons lumineux. Dans ces lentilles magnétiques, le champ magnétique n’est pas uniforme mais présente la symétrie de révolution ; il a principalement une composante axiale suivant l’axe optique et une composante radiale bien plus faible. Comme ce champ a des propriétés focalisatrices, on obtient une image Ai stigmatique d’un point objet Ao comme en optique. On établit alors une correspondance entre les positions de l’objet et de l’image semblable à celle donnée par les formules de Descartes (cf. Optique). Signalons aussi que la composante du champ magnétique suivant l’axe optique fait tourner le plan de l’image avec une vitesse angulaire égale à la pulsation de Larmor vL = v c/2 (cf. Exercices). Enfin, lorsqu’on souhaite analyser l’énergie du faisceau d’électrons dans le plan image, on utilise un spectromètre magnétique que l’on place à la sortie du microscope ; ce spectromètre est souvent un prisme magnétique avec des propriétés dispersives analogues à celles d’un prisme en verre (cf. Optique). V . 4 . — Filtre de Wien Le filtre de Wien est un instrument qui, grâce à l’action combinée de deux champs perpendiculaires, l’un électrique et l’autre magnétique, permet de filtrer des particules chargées de vitesse et donc d’énergie déterminées (Fig. 8.11). On peut choisir E et B de telle sorte que la force de Lorentz, qui s’exerce sur les particules dont la vitesse est v 0 = v 0 e z, soit nulle : F = −e(E + v 0 × B) = 0

134

8. Particule chargée dans un champ électromagnétique stationnaire

Il en résulte que seules les particules issues de S avec la vitesse v0 ont une trajectoire rectiligne. En interposant une fente dans le plan FXY, on filtre les particules qui ont l’énergie mv 20 /2. Ces filtres d’énergie équipent certains microscopes électroniques, ce qui permet d’obtenir des images filtrées, c’est-à-dire formées par des électrons qui ont subi une interaction déterminée avec l’objet observé. On peut alors en déduire des informations sur la constitution physico-chimique de l’objet. V . 5 . — Action du champ magnétique terrestre Le rayonnement cosmique reçu par la Terre est en partie constitué de particules chargées positives. Celles-ci sont soumises à l’action du champ magnétique terrestre, lequel ressemble à celui produit par un dipôle magnétique, situé au centre de la Terre (cf. Électromagnétisme). a) Les ceintures de van Allen Le champ magnétique terrestre retient dans l’espace un grand nombre de particules chargées provenant du Soleil. Les particules piégées par ce champ forment deux anneaux appelées les ceintures de van Allen, l’une interne à une altitude d’environ 10 000 km, l’autre externe à une altitude de 60 000 km (Fig. 8.15). Elles ont été décelées en 1958, par le physicien américain J. van Allen, à l’aide d’instruments placés à bord de satellites artificiels. Axe géomagnétique Ceinture intérieure

Ceinture extérieure

10

20 30

Distance à partir 40 du centre de la Terre (´106 m)

F IG . 8.15.

b) Les aurores polaires Les particules arrivant suivant l’axe géomagnétique de la Terre, qui fait un angle d’environ 11 ◦ avec l’axe de rotation terrestre, ne sont pratiquement pas déviées ; en revanche, les autres, dont la vitesse fait avec l’axe géomagnétique un angle non négligeable, sont très déviées, au point que seules les particules de forte énergie peuvent atteindre la surface du sol. Une analyse détaillée montre que les particules venant de l’espace atteignent le sol en s’accumulant dans deux calottes sphériques entourant les pôles géomagnétiques. Elles produisent alors, par ionisation de l’air, des effets lumineux caractéristiques des aurores polaires. L’étude expérimentale a permis d’établir que ces phénomènes dépendaient fortement de l’activité solaire, ce qui prouve que les particules chargées tombant du ciel proviennent principalement du Soleil (vent solaire).

CONCLUSION Rappelons les principaux résultats. (1) Dans un champ électrique uniforme, le mouvement d’une particule chargée est parabolique. (2) Alors que le champ électrique peut modifier la norme de la vitesse, le champ magnétique ne fait que changer sa direction. Dans un champ magnétique uniforme, le mouvement est la somme d’un

135

Particule chargée dans un champ électromagnétique stationnaire

mouvement rectiligne uniforme suivant la direction du champ et d’un mouvement circulaire uniforme dans un plan perpendiculaire au champ. Le rayon algébrique de la trajectoire a pour expression : v 0 sin a 0 p⊥ qB = avec v c = R= vc qB m (3) Les applications sont très nombreuses (confinement magnétique, microscopie corpusculaire, accélérateurs de particules, magnétosphère des planètes, etc.). Notons que la vitesse acquise par une particule, soumise à un champ électrique, est très vite grande au point que certaines expériences ont été considérées comme des tests de limite de validité de la mécanique newtonienne. Si la vitesse des particules n’est plus négligeable devant la vitesse de la lumière dans le vide, la loi fondamentale de la dynamique a même forme, mais la quantité de mouvement a une expression plus complexe (cf. Relativité).

EXERCICES ET PROBLÈMES P8– 1. Réfraction et réflexion en optique corpusculaire Une cathode K, dont le potentiel est pris égal à 0, émet des électrons avec une vitesse négligeable, vers deux grilles métalliques G1 et G2 , à mailles fines, planes, parallèles et situées à une distance d = 0, 07 m l’une de l’autre (Fig. 8.16). Elles sont respectivement portées aux potentiels V 1 = 1 600 V et V 2 = 900 V. L’électron provenant de K, traverse G1 dans le plan Oxy du référentiel du laboratoire Oxyz. Sa vitesse v1 est alors contenue dans le plan Ozx et fait l’angle i 1 avec l’axe Oz. On se propose d’étudier le mouvement d’un électron entre G 1 et G2 où règne un champ électrique uniforme et stationnaire E orienté suivant Oz. 1. On désigne par i l’angle que fait p avec l’axe des z. Trouver la relation liant i, i 1 , V et V1 . Quelle analogie optique vous suggère-t-elle ? 2. Étudier le cas où V 2 = V1 sin2 i 1. Que se passe-t-il si V2 < V1 sin 2 i1 ? Analogie optique ? y x

 c Dunod – Toute reproduction non autorisée est un délit

K

v1

O

i1

G1 + V1



ε

G2 V2

z

O

+

S

x



F IG . 8.16.

F IG . 8.17.

P8– 2. Focalisation électrique dans un condensateur cylindrique Une source ponctuelle S émet des électrons à l’intérieur d’un condensateur cylindrique (Fig. 8.17). Pour une vitesse d’émission v0 appropriée, un électron décrit une trajectoire circulaire, de rayon r0 , concentrique avec les armatures et située à mi-distance entre elles. On se propose d’étudier le mouvement d’un électron émis avec une vitesse voisine de v0 et faisant un angle ε petit avec v0 . 1. Écrire, en coordonnées polaires (r, w), les équations différentielles du mouvement dans un plan de section du condensateur. On précisera d’abord comment varie le champ électrique E avec r. À

136

8. Particule chargée dans un champ électromagnétique stationnaire

quelle condition sur r0 , w0 et E0 , valeur du champ électrique correspondant, la trajectoire est-elle circulaire ? 2. On pose Dr = r − r 0. Établir, à l’ordre un, que w˙ = w˙ 0 (1 − Dr/r 0) , w˙ 0 étant la valeur de w, ˙ lorsque la trajectoire est circulaire. 3. Quelle est l’équation différentielle à laquelle satisfait Dr ? Montrer que les trajectoires se recoupent après avoir décrit un certain angle que l’on déterminera. P8– 3. Canon de microscope électronique Un canon de microscope électronique est constitué d’une cathode chauffée qui émet des électrons avec une vitesse négligeable. Ces particules sont soumises, dans un vide poussé, à un champ accélérateur entre la cathode portée au potentiel −Va avec V a > 0 et une anode trouée portée au potentiel 0.

1. Établir, dans le cadre newtonien, l’expression de la vitesse des électrons, en fonction de V a , lorsqu’ils passent au centre de l’anode. 2. Calculer la valeur théorique de la vitesse pour V a = 80 kV et pour Va = 120 kV. Comparer cette vitesse à celle de la lumière dans le vide. Conclure. P8– 4. Filtre de Wien Une particule A de charge positive se trouve initialement à l’origine d’un référentiel Oxyz. Elle est soumise à l’action d’un champ électrique E et d’un champ magnétique B uniformes, stationnaires, et perpendiculaires entre eux. On choisit Ox parallèle à B et de même sens, et Oz parallèle à E et de même sens. 1. La vitesse initiale de A est nulle. Trouver x, y et z en fonction du temps. 2. La particule est lancée avec une vitesse initiale v 0 perpendiculaire à E et B, dans le sens des y positifs. Pour quelle valeur particulière vc de v0 , la particule décrit-elle une trajectoire rectiligne confondue avec Oy ? Expliquer comment ce dispositif peut servir de filtre d’énergie ? P8– 5. Précession de Larmor Une particule A (masse m, charge électrique q) est soumise à l’action d’un champ magnétique B uniforme et stationnaire dirigé suivant l’axe Oz du référentiel du laboratoire. À l’instant choisi comme origine, la particule est en O. On se propose d’étudier le mouvement de A dans le référentiel R  tournant autour de B de telle sorte que la projection P de A dans le plan Oxy soit fixe dans R  (Fig. 8.18). z

H

A

B

x

O ϕ R

y ρ

P

R

F IG . 8.18.

1. Écrire dans R  la loi fondamentale de la dynamique appliquée à A. 2. En déduire trois équations scalaires en projetant la relation précédente dans la base de R  . 3. Montrer que la coordonnée radiale r oscille avec une pulsation v L que l’on exprimera en fonction de la pulsation cyclotron vc = qB/m.

137

Particule chargée dans un champ électromagnétique stationnaire

P8– 6. Trajectoires trochoïdales d’électrons Un électron A (masse me , charge électrique −e) est soumis à l’action d’un champ électrique et d’un champ magnétique orthogonaux : E = E ex

avec

E = 10 3 V . m−1 et B = B e y

avec

B = 20 mT

Sa vitesse initiale v0 est perpendiculaire à ces deux champs. 1. Calculer les grandeurs suivantes, en précisant leurs unités SI, a 1 = −eE/me , vc = −eB/me et E/B. 2. On introduit la variable temporelle t = |v c|t. Quelles sont les équations paramétriques du mouvement, z(t) et x(t), lorsque v0 = 4 × 104 m . s−1 ? Représenter la trajectoire de l’électron.

 c Dunod – Toute reproduction non autorisée est un délit

3. Même question pour v 0 = 8 × 104 m . s −1 et pour v0 = 12 × 10 4 m . s −1.

9 Mouvement d’un corpuscule guidé Le mouvement d’un solide, en contact avec un autre solide, peut souvent être étudié, en première analyse, en considérant que ce solide se réduit à un corpuscule astreint à se déplacer sur une courbe ou sur une surface. Ainsi, le mouvement d’un chariot sur un rail est celui de son centre de masse (cf. chapitre 13) sur une courbe parallèle au rail (Fig. 9.1a). De même, le mouvement d’une sphère, en contact avec un plan horizontal, est d’abord celui de son centre de masse (cf. chapitre 13) se déplaçant dans un plan parallèle (Fig. 9.1b). Nous nous proposons d’étudier en détail de tels mouvements dont le nombre de degrés de liberté est inférieur à trois en raison des liaisons, mais dont l’analyse est rendue délicate par la méconnaissance partielle des actions de contact.

C

C a)

b) F IG . 9.1.

I . — CORPUSCULE GUIDÉ SUR UNE COURBE Considérons un corpuscule A, de masse m, évoluant sur une courbe quelconque indéformable C (Fig. 9.2). Il est soumis à la réaction R de contact qu’exerce sur lui le guide qui l’astreint à décrire cette courbe et à une force donnée F, par exemple son poids mg. La trajectoire C étant connue, il est naturel et commode d’adopter le système de coordonnées intrinsèques, ou de Frenet, formé localement par les vecteurs unitaires et , tangent à C , e n porté par la normale principale orientée vers le centre de courbure et eb défini par e b = et × en (cf. chapitre 2). R C

en A

eb F IG . 9.2.

et

139

Mouvement d’un corpuscule guidé I . 1 . — Étude générale du mouvement

La loi fondamentale de la dynamique appliquée à A, par rapport à un référentiel terrestre R dans lequel C est fixe, s’écrit en supposant R galiléen (cf. chapitre 7) : m a = F + R avec

R = R t + Rn + R b

R t , R n et Rb étant respectivement les contributions tangentielle, normale et binormale de la réaction. En projetant, dans la base de Frenet, on obtient : v2 dv 0 = Fb + R b m = Ft + Rt m = F n + Rn dt r où Ft , Fn et Fb sont les composantes de F et Rt , Rn et Rb celles de R ; r désigne le rayon de courbure et v la mesure algébrique de la vitesse suivant et . Les composantes R t , Rn et R b sont des inconnues supplémentaires, ce qui, avec le paramètre fixant la position du point sur la courbe, porte à quatre le nombre total d’inconnues, alors que la loi fondamentale projetée ne fournit que trois équations. Il est donc nécessaire de préciser les conditions du contact ; c’est ce que permettent les lois expérimentales du frottement solide, établies par C. Coulomb (cf. chapitre 19). À ce stade, nous nous limitons aux résultats expérimentaux simples suivants : (1) Si v = 0, Rt a même direction que v mais un sens opposé ; en outre : R t  = m(R 2n+R2b )1/2 . m étant un nombre sans dimension, appelé facteur de frottement ou adhérence, qui dépend de la nature des corps en contact. (2) Si v = 0, R t   ms(R 2n + R2b)1/2 , m s étant le facteur de frottement statique. L’expérience montre que ms est le plus souvent légèrement plus grand que m. On assimile parfois m s à m. Si le mouvement a lieu sans frottement, R t = 0. La première équation permet alors de trouver le mouvement ; avec les deux autres, on détermine R. Remarques : (1) Si le référentiel dans lequel la courbe C est fixe n’est pas galiléen, il faut ajouter aux forces précédentes les forces d’inertie d’entraînement et de Coriolis (cf. chapitre 7). (2) Si la courbe C est plane et si F est dans ce plan, la réaction y est aussi puisqu’alors F b = 0 et donc R b = 0. (3) Si R t = 0 et si F dérive d’une énergie potentielle, l’utilisation de la conservation de l’énergie facilite grandement la discussion du mouvement.

 c Dunod – Toute reproduction non autorisée est un délit

I . 2 . — Mouvement d’une masselotte guidée sur une tige mobile Une masselotte glisse sans frottement sur une tige qui est astreinte à tourner uniformément, autour d’un axe vertical, en faisant un angle constant u0 avec cet axe. La masselotte est soumise aussi à l’action d’un ressort de raideur K (Fig. 9.3). Étudions le mouvement de A par rapport au référentiel non galiléen R, lié à la tige et tournant avec la vitesse angulaire V = V e z . H

z

r

µ0 A

g

Ω

y

O x

R

'

P  x µ0 µ F IG . 9.3.

R

y

140

9. Mouvement d’un corpuscule guidé

Aux forces mg, T = −K (r − l 0) e r et R, il faut ajouter, puisque R n’est pas galiléen, la force d’inertie centrifuge et la force d’inertie de Coriolis, soit respectivement : mV2HA et −2mV × v . Dans le référentiel R , l’énergie mécanique de A se conserve car la réaction R qu’exerce la tige sur A ne travaille pas : 1 Ek + Ep = Cte avec Ek = m˙r2 et Ep = E pp + Epe + Epc 2 En effet, l’énergie potentielle est la somme de trois énergies potentielles : l’énergie potentielle de pesanteur Epp , l’énergie potentielle élastique Epe et l’énergie potentielle centrifuge Epc . À une constante additive près sans intérêt, on a : K (r − l0)2 mV 2 HA2 −mV 2r 2 sin2 u0 Epp = mgz = mgr cos u0 Epe = Epc = − = 2 2 2 On en déduit l’équation du mouvement : 1 2 K (r − l0 )2 mV 2r 2 sin2 u 0 = Cte m˙r + mgr cos u0 + − 2 2 2 En dérivant par rapport au temps et en simplifiant par ˙r, on trouve : Em =

r + mg cos u0 + K (r − l0 ) − mV2 r sin2 u0 = 0 m¨

La position d’équilibre re sur la tige s’obtient aisément en faisant ¨r = 0 : K (re − l0 ) − mV2r e sin2 u 0 = −mg cos u 0

d’où

re =

v20l 0 − g cos u0 v 20 − V2 sin 2 u 0

si on introduit le carré de la pulsation propre de l’oscillateur élastique v 20 = K /m. Il vient donc, en posant r = r − re : r ¨ + (v20 − V2 sin 2 u0 )r = 0

La discussion ne présente aucune difficulté. (1) Si V2 sin2 u0 < v20 , la masselotte oscille, avec la pulsation (v 20 − V 2 sin 2 u0 )1/2 , autour de sa position d’équilibre. (2) Si V 2 sin2 u0  v 20, l’amplitude du mouvement de la masselotte sur la tige amplifie jusqu’à modifier le comportement du ressort et même provoquer sa rupture. On peut obtenir l’équation différentielle du mouvement, d’une manière plus laborieuse, à partir de la loi fondamentale de la dynamique, projetée dans la base e r, e u, e y . En effet : donne :

m a A/R = m g + K (r − l 0) er + mV 2HA − 2m V × vA/R  m ¨r er = −mg e z + K (r − l 0) er + mV2 r sin u0 ey − 2mV˙r ez × e r

d’où, en explicitant :

r = −mg cos u0 − K (r − l0) + mV 2r sin 2 u0 m¨ 0 = mg sin u0 + mV2r cos u 0 sin u0 + Rn 0 = −2m˙rV sin u 0 + R b Les deux dernières équations permettent de déterminer les composantes de la réaction en fonction de la coordonnée r : Rn = −m sin u0(g + V 2r cos u0 ) et Rb = 2m˙rV sin u 0 Remarque : Notons que, dans le référentiel terrestre R, l’énergie de la masselotte ne se conserve pas car le travail de la réaction R n’y est pas nul. Physiquement, cette non conservation est due à l’énergie nécessaire pour astreindre la tige à un mouvement de rotation.

141

Mouvement d’un corpuscule guidé

II . — PENDULE CIRCULAIRE Un pendule circulaire est le système simple constitué par une masselotte A qui se déplace sans frottement sur un guide circulaire vertical (Fig. 9.4a). On peut réaliser aisément un tel pendule en accrochant la masselotte à l’extrémité d’un fil tendu de longueur l. E p Énergies H

g

mgl

O µ

G

y

l

∆¼ ∆¼=2

R

B

∆µ1 0

Em µ1 ¼=2

¼ µ

Em

A

∆mgl

x a)

b) F IG . 9.4.

II . 1 . — Étude générale

 c Dunod – Toute reproduction non autorisée est un délit

Les équations du mouvement issues de la projection dans la base de Frenet de la loi fondamentale sont les suivantes : v2 dv m = −mg sin u et m = −mg cos u + Rn avec v = l ˙u l dt Il en résulte que : ¨u + g sin u = 0 l La solution de cette équation différentielle du deuxième degré n’est simple que dans le cas des petits angles. Aussi est-il préférable, puisque R ne travaille pas, d’exprimer la conservation de l’énergie mécanique afin de faciliter l’analyse qualitative du mouvement. La seule force qui travaille est le poids dont l’énergie potentielle s’écrit : E p = −mgx = −mgl cos u, si l’axe des x est vertical descendant et l’origine des énergies potentielles prise en O. Quant à l’énergie cinétique par rapport à R, elle vaut Ek = mv2A/2 = ml2 u˙ 2/2. Il en résulte : Em =

1 2 2 ml u˙ − mgl cos u = Cte 2

Suivant la nature de la liaison, l’analyse diffère comme nous allons le voir. II . 2 . — La liaison est bilatérale Si la liaison est bilatérale, la masselotte A reste en contact avec le guide, quelle que soit la valeur de Rn . Par exemple une perle enfilée sur un guide matériel filiforme est soumise à une liaison bilatérale ; cette dernière se traduit par l’équation de contrainte suivante sur les coordonnées de position : x2 + y2 = l 2 = Cte a) Discussion à l’aide du graphe de l’énergie potentielle La discussion du mouvement est très simple à partir du graphe E p(u) (cf. chapitre 5) : les mouvements possibles sont ceux pour lesquels Em  Ep (Fig. 9.4b).

142

9. Mouvement d’un corpuscule guidé i) −mgl < E m < mgl. Le point oscille entre les valeurs symétriques −u 1 et u 1 définies par : Em cos u1 = − mgl

avec Em = mv02/2 − mgl cos u 0 , v0 et u0 étant les valeurs initiales de la vitesse et de l’angle que fait le fil avec la verticale descendante. Le mouvement est oscillatoire autour du point le plus bas B. Dans le cas particulier où E m ≈ −mgl, u 1  p/2 : A oscille dans le voisinage de B où l’énergie potentielle est minimale. En remplaçant sin u par u, on obtient l’équation caractéristique des petits mouvements d’oscillation (cf. chapitre 10) : g 1/2 ¨ u + v20 u = 0 avec v0 = l Ordre de grandeur : Pour l = 50 cm, v 0 = 4, 43 rad . s−1 , d’où la période propre : T 0 = 2p/v0 = 1, 42 s On vérifie expérimentalement ce résultat en mesurant la durée de dix oscillations de faible amplitude (u1 < 10◦ ). Remarque : Lorsque l’approximation précédente n’est plus satisfaite, l’analyse qualitative s’impose car l’équation différentielle n’est plus linéaire. L’effet de cette non-linéarité sera étudié ultérieurement (cf. chapitre 10). ii) E m  mgl. La masselotte atteint le point le plus haut H. Son mouvement est révolutif et périodique. Notons que, dans les deux cas, le mouvement réel diffère des prévisions précédentes en raison des forces de frottement qui n’ont pas été prises en compte. Dans le premier, l’oscillation s’amortit jusqu’à l’immobilité en B ; dans le second, le mouvement révolutif est freiné jusqu’à devenir oscillatoire et s’arrêter en B. b) Discussion dans l’espace des phases La discussion précédente a montré une différence de nature entre deux types de mouvements : l’un est oscillatoire, l’autre est révolutif. Ce changement qualitatif important apparaît nettement dans l’espace des phases, ou espace des états en mécanique introduit par J. Maxwell en 1879. On définit un tel espace en considérant, pour un système à un degré de liberté, le plan cartésien (u, L u ), Lu = ml2 u˙ étant le moment cinétique de A en O (Fig. 9.5).



Em > mgl

Em = mg l -π

O

π

θ

Em < mgl F IG . 9.5.

Un point de cet espace représente l’état du système, puisque dans ce diagramme, u et u˙ à la fois sont accessibles. La trajectoire du point représentatif du système est alors donnée par l’équation exprimant Lu en fonction de u : 1 Lu = ml2 u˙ avec Em = ml 2 ˙u2 − mgl cos u soit Lu = ±l [2m(E m + mgl cos u)]1/2 2

143

Mouvement d’un corpuscule guidé

Sur la figure on distingue aisément deux zones séparées par la courbe séparatrice correspondant à E m = mgl : i) l’une (en trait pointillé) pour laquelle le mouvement est oscillatoire (E m < mgl) , ii) l’autre (en trait plein) pour laquelle le mouvement est révolutif (E m > mgl) . On voit que les conditions initiales permettent de déterminer sans ambiguïté le type de mouvement du pendule dans l’espace des phases : mouvement oscillatoire ou mouvement révolutif. c) Cas où le guide tourne uniformément autour de son diamètre vertical Supposons que le guide soit astreint à tourner uniformément autour de son diamètre vertical BT, à la vitesse angulaire V = −V ex (V > 0) (Fig. 9.6a). Il faut tenir compte en plus de l’énergie potentielle centrifuge −mV 2HA 2/2, H étant la projection de A selon l’axe de rotation. L’énergie potentielle dans le référentiel tournant R = Oxy z lié au guide est : m Ep = −mgl cos u − V 2l 2 sin 2 u 2 Dans R  = Oxy z  non galiléen, l’équation différentielle du mouvement est fournie par la conservation de l’énergie, laquelle est satisfaite car la puissance de la réaction y est nulle, et que la puissance de la force de Coriolis est toujours nulle. Comme l’énergie cinétique par rapport à R vaut E k = ml2u˙2 /2 : E k + Ep = Em = Cte

soit

1 2 2 m ml u˙ − mgl cos u − V2l2 sin 2 u = Em 2 2 Ep

T

mgl

g

Epg

Ω G

O

y

µ l

H

A

eu

−¼

−¼/2

 E pc

¼

µ

-mgl

B

x a)

 c Dunod – Toute reproduction non autorisée est un délit

¼/2

O

b) F IG . 9.6.

Sur la figure 9.6b, on a représenté le graphe E p (u) pour une vitesse angulaire V suffisamment grande. On trouve une position d’équilibre stable pour une valeur de u comprise entre 0 et p/2, ce qui était qualitativement prévisible. En effet, pour V = 0, on doit restituer l’analyse précédente dans laquelle seule la pesanteur intervenait. En revanche, pour V très grand, la pesanteur est négligeable devant la force centrifuge qui maintient la masselotte à la distance maximale l de l’axe de rotation. Le calcul des positions d’équilibre par dérivation de E p par rapport à u permet de préciser ces prévisions : d Ep = ml sin u(g − V2l cos u) = 0 du

pour

u = 0 et u e = ± arccos

g V 2l

Si ces positions correspondent à des minima, c’est-à-dire si (d2 Ep / d u2 ) e > 0, les équilibres sont stables. Comme : d2 Ep = ml sin u(V 2l sin u) + (g − V2 l cos u)ml cos u d u2

144

9. Mouvement d’un corpuscule guidé

on trouve :

d2 Ep d u2

0

= (g − V 2l)ml > 0

si

V
v0 , d2 Ep d u2

= ml2V 2 sin 2 ue > 0 ue

d’où la stabilité de cette position. Dans le cas où l = 50 cm, la valeur critique V c de la vitesse angulaire V vaut : V c = v 0 = 4, 43 rad . s −1 soit Vc = 42, 3 tr . min−1 Ce changement qualitatif de position d’équilibre, sous l’action du paramètre V , est une bifurcation. Remarque : Les deux positions d’équilibre u e = 0 pour V < v 0 et ue = ± arccos(v20 /V2) diffèrent fondamentalement ; la première respecte la symétrie du système par rapport à l’axe vertical, la seconde non. On dit qu’il y a brisure de symétrie. II . 3 . — La liaison est unilatérale Revenons à l’étude initiale du pendule circulaire, en considérant le cas où la liaison est unilatérale ; la contrainte sur les coordonnées de position se traduit alors par une inégalité. Concrètement, c’est ce qui se passe lorsque, pour une raison quelconque, le fil n’est plus tendu (Fig. 9.7a). On a alors : x 2 + y 2  l2 C’est aussi le cas d’une bille qui évolue dans une gouttière à section semi-circulaire. 3 mg l /2 mgl Ep

H

g

O l θ B

∆θ 2 ∆θ1 O ∆π ∆π/2

y R

θ1

θ2 π/ 2 π

θ

Em

A

∆mg l

x a)

3 E p/2

b) F IG . 9.7.

L’analyse est plus délicate que précédemment car, à l’équation fournie par la conservation de l’énergie, on doit ajouter la condition pour que le fil exerce une tension non nulle. On doit donc prendre en compte l’inégalité suivante sur la réaction normale : Rn > 0. Il importe donc d’exprimer R n en fonction de l’angle u : 2Em Rn = mlu˙ 2 + mg cos u = + 3mg cos u l

d’où

2Em + 3mg cos u > 0 l

Afin de faciliter la discussion qualitative du mouvement, il est commode de traduire cette inégalité

145

Mouvement d’un corpuscule guidé

en termes d’énergie et d’utiliser un même graphe. La condition précédente s’écrit aussi : 3 3 E m > − mgl cos u soit Em > E p puisque E p = −mgl cos u 2 2 Ainsi, si la liaison est unilatérale, la masselotte quitte le guide circulaire pour une valeur u2 de u telle que : 3 3 E m = Ep (u2 ) = − mgl cos u2 2 2 Sur le graphe de la figure 9.7b, on a représenté la courbe E p(u) et la courbe 3Ep(u)/2. Trois possibilités doivent être envisagées : i) −mgl  E m  0 La valeur de u 2 n’est jamais atteinte puisque u 1 < u2 ; la masselotte demeure sur son guide en oscillant autour de sa position la plus basse. Les petits mouvements sont identiques à ceux qui existent lorsque la liaison est bilatérale. ii) 0 < E m < 3mgl/2 La réaction s’annule pour la valeur u 2 < u 1. Le mouvement comporte une seconde phase au cours de laquelle le point évolue sous la seule action de son poids. La trajectoire correspondante est la parabole osculatrice au guide circulaire. iii) E m  3mgl/2 Le mouvement est révolutif avec contact permanent de A sur le guide.

III . — CORPUSCULE GUIDÉ SUR UNE SURFACE La position d’un corpuscule A sur une surface S indéformable quelconque ne dépend que de deux paramètres. Ce corps, de masse m, est soumis à la réaction R qu’exerce le système qui maintient A sur S et à une force donnée F, par exemple son poids m g (Fig. 9.8). III . 1 . — Étude générale du mouvement

 c Dunod – Toute reproduction non autorisée est un délit

Si la surface S est fixe dans un référentiel galiléen R, la loi fondamentale de la dynamique s’écrit : m a = F + R où R = Rt + Rn R t et R n étant les contributions tangentielle et normale, dirigées respectivement suivant la tangente à la trajectoire et suivant la normale à la surface S. Dans ce cas, les lois de Coulomb sont les suivantes : i) Si v = 0, R t a même direction que v mais un sens opposé ; en outre, Rt  = mR n,

m étant le facteur de frottement. ii) Si v = 0, R t   ms Rn , ms étant le facteur de frottement statique. Dans ce type de problème, où l’on a des informations sur la trajectoire, il est naturel d’expliciter l’équation vectorielle du mouvement dans la base locale de Frenet. En l’absence de frottement (R t = 0), l’équation du mouvement se réduit à : m a = F + Rn La projection de cette équation sur la normale à S donne l’expression de R n ; les deux autres équations fournissent alors les équations différentielles du mouvement. En outre, comme c’est souvent le cas, si la force donnée F dérive d’une énergie potentielle, on a intérêt à substituer l’équation de conservation de l’énergie à l’une des équations précédentes.

146

9. Mouvement d’un corpuscule guidé

Remarque : Il n’est pas inutile de rappeler comment l’on obtient le vecteur en normal à une surface d’équation f (x, y, z) = 0. Le vecteur grad f , de composantes ∂f /∂x, ∂f /∂ y, ∂f /∂ z, est normal à cette surface, car : d f = 0 et

df =

∂f ∂f ∂f d z = grad f · d r dx + dy + ∂x ∂y ∂z

grad f · d r = 0

donnent

On en déduit en en divisant ces composantes par la norme de grad f . Par exemple, si la surface est un paraboloïde de révolution d’équation : x 2 + y 2 − lz = 0, il vient : ∂f = 2x ∂x

∂f = 2y ∂y

∂f = −l ∂z

On en déduit les composantes de e n : 2x 2y 2 2 2 1 / 2 2 [4(x + y ) + l ] [4(x + y 2 ) + l2]1/2

[4(x2

−l + y 2) + l 2]1/2

z z

R

Rn A

Rt R

O

H

S

A

Trajectoire

g

O

θ

R g

mg

y

ϕ

y

P

x x F IG . 9.8.

F IG . 9.9.

III . 2 . — Pendule sphérique Le pendule sphérique est constitué d’une masselotte qui évolue sans frottement sur une surface sphérique (Fig. 9.9). On peux le réaliser en accrochant la masselotte à l’extrémité d’un fil inextensible, auquel cas la liaison est unilatérale. Pour simplifier, nous limitons notre étude à la liaison bilatérale. a) Équations du mouvement Le mouvement ayant deux degrés de liberté, u et w, les deux équations du mouvement peuvent être obtenues à partir de la conservation de l’énergie et de celle de la projection du moment cinétique, en O, suivant la verticale ascendante Oz. En effet, comme le poids dérive de l’énergie potentielle E p = mgz = mgl cos u et que la réaction R ne travaille pas, on a : Em = Ek + Ep = Cte

soit

1 m(l 2u˙ 2 + l 2 w˙2 sin 2 u) + mgl cos u = E m 2

En outre, le théorème du moment cinétique, appliqué en O, fournit l’équation suivante, le moment en O de R étant nul : d LO = OA × mg dt avec : L O = OA × mvA = le r × m(lu˙ eu + lw˙ sin u e w ) = ml2 (u˙ ew − w˙ sin u eu)

147

Mouvement d’un corpuscule guidé

En projetant sur l’axe vertical l’équation vectorielle fournie par ce théorème, on trouve, puisque le moment du poids est un vecteur horizontal : d LO d(LO · e z) d Lz = = = 0 d’où Lz = Cte dt dt dt Or L z = LO · e z = ml2 ( ˙u ew − w˙ sin u e u) · ez = ml2(−w˙ sin u)(eu · ez ). Par conséquent : ez ·

Lz = ml2 (−w˙ sin u) cos(u +

p ) 2

Lz = ml 2 w˙ sin2 u = Cte

soit

b) Nature du mouvement Si on élimine w˙ entre les deux équations, on se ramène à un problème unidimensionnel ; l’équation de conservation de l’énergie peut alors se mettre sous la forme : 1 2˙2 ml u + E p,ef = Em 2 où l’énergie potentielle E p,ef est l’énergie potentielle effective de ce problème à une dimension :

L 2z ml 2 2 2 w˙ sin u = mgl cos u + 2 2ml2 sin 2 u Les seuls mouvements possibles sont ceux pour lesquels E m  Ep,ef . Traçons donc le graphe Ep,ef (u) (Fig. 9.10a) : Ep,ef prend des valeurs infinies, pour u = 0 et u = p, et passe par une valeur minimale E0 . Comme : d Ep,ef L2 cos u = −mgl sin u − z2 du ml sin 3 u les variations de Ep,ef (u) sont celles qui sont données dans le tableau 9.1 ; le minimum se produit donc pour p/2 < u < p. Pour une valeur suffisante de l’énergie (Em > E0 ), les mouvements possibles sont ceux pour lesquels u est compris entre deux valeurs extrêmes u1 et u 2 : u oscille donc entre deux parallèles (Fig. 9.10b). Ep,ef = mgl cos u +

0

u d Ep,ef du

−∞

Ep



p/2 −

−mgl L2 2ml2

u0 −

p

0

+

E0

∞ ∞

 c Dunod – Toute reproduction non autorisée est un délit

TAB . 9.1.

Remarque : La fonction Ep,ef (u) a une allure dissymétrique ; notons que la demi-somme des deux valeurs extrêmes est supérieure à p/2. E p,ef

z

L2z 2ml2 sin 2 θ

Em E0 O

O

θ y

ϕ A θ1 π /2 θ0 θ2 mgl cos θ a)

π

θ

x b) F IG . 9.10.

148

9. Mouvement d’un corpuscule guidé

c) Mouvements stationnaires Le mouvement en u est stationnaire (u = u 0 = 0) si les valeurs initiales satisfont, à la fois, aux deux conditions suivantes : d Ep,ef L2z cos u 0 ˙ 0 = 0 d’où Ep,ef = Em et = 0 d’où = −g ( u) 0 du 0 m 2l 3 sin 4 u0 Un tel mouvement est uniforme, puisque w˙ = Lz/(ml 2 sin 2 u0) = Cte, et n’est possible que sur un plan parallèle situé en dessous du plan horizontal car u0 > p/2 (Fig. 9.10b). Remarque : Pour Lz = 0, le mouvement en w est stationnaire puisque w˙ = 0. Le pendule décrit alors un méridien et se comporte comme un pendule circulaire. d) Calcul de la réaction On obtient l’expression de la réaction R en appliquant la loi fondamentale de la dynamique à la masselotte : ma = R+ mg Comme la composante radiale de l’accélération est −lw˙ 2sin 2 u − lu˙2 (cf. chapitre 3), il vient : −ml(w˙2 sin2 u + u˙ 2) = −R − mg cos u

d’où

R = −mg cos u + ml(w˙ 2 sin2 u + u˙2 )

Par conséquent, pour u > p/2, R > 0 : la réaction est dirigée vers l’intérieur de la sphère lorsque A évolue au-dessous du plan horizontal passant par O (Fig. 9.10b).

CONCLUSION Retenons que l’étude du mouvement d’un corpuscule guidé est nettement moins simple que celle d’un corpuscule soumis uniquement à des forces fondamentales telles que le poids, car les forces de contact ne sont que partiellement connues. (1) Il importe de prendre en compte les lois de Coulomb, notamment la relation supplémentaire entre les composantes tangentielle et normale de la réaction, lorsqu’il y a glissement sur une surface : Rt  = mR n , m étant le facteur de frottement. En l’absence de glissement, cette équation est remplacée par l’inégalité, Rt   ms Rn , ms étant le facteur de frottement statique. (2) En l’absence de frottement, la réaction est normale à la trajectoire. Le mouvement est alors facilement étudié à l’aide de la conservation de l’énergie ; laquelle fournit une intégrale première qui se prête bien à une discussion qualitative du mouvement.

(3) Lorsqu’une seconde équation du mouvement est nécessaire, comme dans le cas du pendule sphérique, qui a deux degrés de liberté, il est naturel d’appliquer en outre le théorème du moment cinétique au point O par lequel passe la réaction inconnue afin d’éliminer cette dernière. (4) Deux types de liaison du point matériel avec le guide doivent être considérés : i) la liaison bilatérale ; le contact est permanent, ce qui se traduit par une équation de contrainte sur les coordonnées, ii) la liaison unilatérale ; le contact peut cesser, ce qui rend l’analyse plus complexe puisqu’une inéquation sur la réaction normale doit être prise en compte. Il peut être commode d’exprimer cette inégalité sous une forme faisant apparaître l’énergie potentielle : la discussion qualitative peut alors être grandement facilitée. Si le référentiel R n’est pas galiléen, l’analyse énergétique doit prendre en compte l’influence de la force d’inertie d’entraînement, le travail de la force d’inertie de Coriolis étant, lui, toujours nul.

149

Mouvement d’un corpuscule guidé

EXERCICES ET PROBLÈMES P9– 1. Mouvement d’une masselotte guidée par une courbe circulaire Une masselotte A, de masse m, se déplace sur un rail situé dans un plan vertical. Le rail comporte une partie circulaire, de diamètre BC = 2l, que le mobile parcourt à l’intérieur du cercle (Fig. 9.11). La masselotte est abandonnée (sans vitesse initiale) en H à la hauteur h au-dessus de B, point le plus bas du cercle. On prend comme origine de l’énergie potentielle de pesanteur Ep sa valeur en B. La liaison est unilatérale et on néglige tous les frottements. 1. Exprimer, à tout instant, la vitesse v de A en fonction de l’angle u = (OB, OA). 2. Donner l’expression de la réaction R exercée par le guide circulaire en fonction de u. 3. Discuter les différents mouvements possibles suivant la valeur de h. On utilisera le graphe E p(u). 4. Si h = 3l, l = 10 m et m = 350 kg, calculer les vitesses de passage en B et T, point le plus haut du cercle, ainsi que les réactions du rail lors des passages en ces points. z g

H

y g

H

A

T

h

O µ x

O

A x

B

F IG . 9.11.

B

y

µ

F IG . 9.12.

P9– 2. Mouvement d’un corpuscule sur une courbe hélicoïdale

 c Dunod – Toute reproduction non autorisée est un délit

On considère un cylindre circulaire, droit, de rayon R et de hauteur 2pR, auquel est lié un repère orthonormé direct Oxyz : Oz est l’axe vertical du cylindre et Oxy est la base inférieure horizontale (Fig. 9.12). La face latérale porte un tube mince de forme hélicoïdale HB dans lequel se déplace un corpuscule A , de masse m. La définition paramétrique de la trajectoire de A est : x = R cos u

y = R sin u

z = R(2p − u)

avec

0  u  2p

Le point H correspond à u = 0 et le point B à u = 2p. 1. Le cylindre est maintenu fixe et A glisse sans frottement sous l’action de son poids m g. Il est abandonné sans vitesse initiale en H. a) Déterminer la loi horaire du mouvement, c’est-à-dire l’abscisse curviligne s(t) de A sur sa trajectoire en fonction du temps t. b) Calculer la durée mise par A pour atteindre le point B, ainsi que la vitesse en ce point. 2. On suppose que le glissement de A se fait avec frottement, conformément aux lois de Coulomb, le facteur de frottement étant constant et égal à m. a) Calculer, en fonction de u(t) et de ses dérivées , l’accélération de A. En donner les projections dans la base de Frenet. b) Exprimer, dans cette base, les projections de la force de pesanteur et celles de la réaction d’appui. c) Écrire les équations du mouvement et déduire de la loi de Coulomb l’équation différentielle relative à s(t). d) Montrer que u croît et tend vers une limite. Quelle est la vitesse limite de A ?

150

9. Mouvement d’un corpuscule guidé

P9– 3. Écart maximal d’un pendule sphérique avec la verticale Une bille A, de masse m, est suspendue à l’extrémité d’un fil inextensible, de longueur l, attaché à l’origine O d’un référentiel terrestre R = Oxyz. On l’écarte de l’axe vertical descendant Oz d’un angle u0 dans le plan Ozx et on lui communique une vitesse v0 dirigée suivant l’axe Oy. À tout instant, on caractérise la position de A sur la sphère, de centre O et de rayon l, par l’angle u = (Oz, OA) et w = (Ox, OH), angle que fait avec Ox la projection H de A dans le plan horizontal (Fig. 9.13). 1. Montrer que l’énergie mécanique de la bille se conserve. 2. a) Établir que la composante L z du moment cinétique en O suivant l’axe z se conserve. b) Exprimer, dans la base sphérique (e r , eu, e w), le vecteur OA, ainsi que la vitesse de A par rapport à R. c) Montrer que, dans cette base, la vitesse se réduit à une seule composante, à l’instant initial (u = u0) et à l’instant où u est maximal (u = u m ). d) Traduire la conservation de L z entre l’état initial, caractérisé par u0 et v0 , et l’état final caractérisé par um et vm . 3. Pour quelle vitesse v, en fonction de u 0 , la valeur de u m est-elle égale à p/2 ? g

O

ϕ H

y g

A F IG . 9.13.

z0

x x0

θ z



y

O

S

x

y0

A

F IG . 9.14.

P9– 4. Masselotte en contact sans frottement avec une demi-sphère en rotation uniforme Une masselotte A, de masse m, se déplace, sans frottement, sur la surface intérieure d’une demisphère creuse S. Cette surface est astreinte à tourner uniformément, à la vitesse angulaire V, autour de son axe de révolution vertical. Sur la figure 9.14, on a représenté le référentiel terrestre, noté R 0 = Ox0y0z 0 , Oz0 étant la verticale ascendante, et R = Oxyz0 un référentiel invariablement lié à S. On se propose d’étudier le mouvement de A par rapport à R. Pour cela, on utilise la base de R et on introduit v0 = (g/r0) 1/2, g étant l’intensité du champ de pesanteur terrestre et r 0 le rayon de la demi-sphère S. 1. Exprimer, en fonction des coordonnées x, y, z de A dans R, de leurs dérivées par rapport ˙ z˙ et de V, la vitesse d’entraînement de A, son accélération d’entraînement et son au temps x,˙ y, accélération de Coriolis.

2. Écrire vectoriellement la loi fondamentale de la mécanique pour A dans son mouvement par rapport à R. En déduire les équations différentielles auxquelles satisfont x, y et z ; on mettra la réaction R qu’exerce S sur A sous la forme suivante que l’on justifiera : R = −R r/r 0 où r = OA.

3. a) Quelle est, en fonction de z, l’énergie potentielle de pesanteur de A ? On prendra l’origine de l’énergie potentielle à z = 0. b) Montrer que la force d’inertie d’entraînement dérive aussi de l’énergie potentielle mV 2z 2/2, lorsque l’on prend l’origine à z = 0. c) En déduire l’énergie potentielle totale E p de A.

151

Mouvement d’un corpuscule guidé

4. a) Tracer le graphe de la fonction f (u) = u 2 + 2v 20u/V 2 pour v20 /V2 = 0, 4. Montrer que E p = mV2r 20 f (z/r 0 )/2. b) Discuter qualitativement la nature des différents mouvements en z , suivant la valeur de l’énergie mécanique totale Em de A dans R (on prendra v 20 /V2 = 0, 4). c) Pour quelle valeur Em de l’énergie, le point A évolue-t-il en contact avec S dans un plan horizontal ? Quelle est la cote zm correspondante en fonction de r0 ? 5. Écrire l’équation vectorielle traduisant l’équilibre de A par rapport à R. Interpréter cette condition en introduisant le champ de pesanteur apparent ga = g + V2HA, H étant la projection de A sur l’axe de rotation Oz0. En déduire la cote, à l’équilibre , en fonction de r 0. Comparer cette cote à z m et conclure. P9– 5. Mouvement sans frottement d’une masselotte sur une tige en mouvement Une masselotte A (masse m) se déplace sans frottement le long d’une tige T qui est astreinte à tourner uniformément autour de l’axe horizontal Oz dans le plan vertical Oxy du référentiel terrestre R supposé galiléen (Fig. 9.15). La liaison de A le long de T est bilatérale. On désigne par V la vitesse de rotation de la tige par rapport à R. 1. Appliquer la loi fondamentale de la dynamique par rapport à R. En déduire l’équation différentielle du mouvement à laquelle satisfait la coordonnée radiale r. Quelle est, en fonction de r ou de ses dérivées par rapport au temps, l’expression de la réaction R qu’exerce la tige sur A. 2. Montrer que l’énergie mécanique E m de A par rapport à R ne se conserve pas.

3. L’énergie mécanique E m par rapport au référentiel tournant R lié à la tige se conserve, elle. Pourquoi ? En déduire l’équation du mouvement en r. y A (m) g T Ωt Oz

 c Dunod – Toute reproduction non autorisée est un délit

F IG . 9.15.

R

x

10 Oscillateurs harmoniques. Oscillateurs amortis

L’expérience courante montre que tout système physique, en équilibre stable, soumis à une légère perturbation revient généralement vers sa position d’équilibre après une suite d’oscillations dont l’amplitude finit par s’annuler. Ce comportement de tout système perturbé justifie l’importance de l’étude des oscillateurs harmoniques et amortis.

I . — OSCILLATEURS HARMONIQUES I . 1 . — Définition On appelle oscillateur harmonique tout système dont le degré de liberté x(t) évolue suivant une loi sinusoïdale : x(t) = x m cos(v0 t + f) = x m cos(2pf0 t + f) Dans ces expressions, xm , v0 , f0 = v0/(2p), T0 = 1/f0 et f sont des constantes appelées respectivement l’amplitude, la pulsation, la fréquence, la période et la phase à l’origine des temps (Fig. 10.1). L’importance des oscillateurs harmoniques est liée à la possibilité de représenter un oscillateur quelconque par un groupe d’oscillateurs harmoniques de pulsations différentes (cf. annexe 2 dans Optique). x

xm

f=0

T0

0

F IG . 10.1.

t

153

Oscillateurs harmoniques. Oscillateurs amortis I . 2 . — Équations différentielles caractéristiques

L’application des lois de la physique à un oscillateur harmonique fournit généralement l’un des deux types d’équations différentielles suivants : ¨ x + v20 x = 0

v 2x2 x˙ 2 + 0 = Cte 2 2

ou bien

On passe de la première équation à la seconde en multipliant les deux membres par ˙x et en intégrant ; réciproquement, on passe de la seconde à la première en dérivant et en simplifiant par x˙ = 0. La première de ces équations est clairement linéaire, puisque toute combinaison linéaire de solutions est aussi une solution.

I . 3 . — Petites oscillations autour d’une position moyenne Considérons un corpuscule A, de masse m, soumis à l’action d’une force qui dérive de l’énergie potentielle E p(x), x étant l’abscisse donnant la position de A par rapport au repère unidimensionnel R, d’origine O et de base ex . L’énergie mécanique E m de A est constante (cf. chapitre 5) : 1 Em = E k + Ep = m ˙x2 + E p(x) = Cte 2 Supposons, comme cela arrive très souvent, que l’énergie potentielle Ep (x) passe par un minimum de valeur E0 pour x = x 0 (Fig. 10.2). Il est alors naturel d’exprimer E p(x) à l’aide d’un développement de Taylor autour de la valeur minimale E0 ; on obtient : 1 x − x0 Em = m x˙ 2 + E0 + 2 1!

d Ep dx

0

(x − x0) 2 + 2!

d2 E p d x2

0

+ ···

 c Dunod – Toute reproduction non autorisée est un délit

Comme (d Ep / d x)0 = 0, le premier terme non nul est le terme quadratique ; ce dernier est positif puisque Ep passe par un minimum. Dans ces conditions, après perturbation du système, les seuls mouvements possibles sont ceux pour lesquels x1  x  x 2 : x oscille entre x1 et x2 , ce qui traduit la stabilité de l’équilibre. Cette stabilité doit donc être associée au minimum de l’énergie potentielle à l’équilibre. Ep

F ( x)

E E0 0

0 x1 x0

x2

x0

x

F IG . 10.2.

x

F IG . 10.3.

Si la perturbation apportée au système est suffisamment faible, on peut négliger les termes d’ordre supérieur à deux dans le développement précédent : 1 Ep ≈ E 0 + K (x − x 0)2 2

avec

K=

d 2 Ep dx2

>0 0

154

10. Oscillateurs harmoniques. Oscillateurs amortis

Donc :

1 2 1 m x˙ + K (x − x0 )2 = Em − E 0 2 2

ce qui donne, en dérivant et en rejetant la solution parasite ˙x = 0 : ¨ x + v 20(x − x0 ) = 0

en posant

v 20 =

K m

Désignant par X = x − x0, la position comptée à partir de la position d’équilibre, on obtient l’équation différentielle du deuxième ordre linéaire caractéristique de l’oscillateur harmonique : ¨ + v20 X = 0 X

dont la solution est

X = X m cos(v0t + f)

où Xm et f, amplitude et phase à l’origine, sont déterminées par les conditions initiales. Notons que la pulsation propre v0 et donc la période T 0 = 2p/v 0 dépendent des caractéristiques propres de l’oscillateur (K , m) et non de la perturbation extérieure. La force qui s’exerce sur l’oscillateur se déduit aisément de l’approximation quadratique de l’éner(2) gie potentielle. En effet, comme E p = E0 + K (x − x0) 2/2 il vient : F=−

d Ep ex = −K (x − x0 ) ex dx

Cette force est donc proportionnelle à l’écart du paramètre x par rapport à la position d’équilibre x0 (Fig. 10.3). En raison du signe moins, elle est positive si x < x 0 et négative si x > x0 . Aussi dit-on que c’est une force de rappel. Remarque : Le simple résultat v0 = (K /m) 1/2 confirme le fait bien connu que les objets lourds vibrent en général avec une fréquence f 0 = v0 /(2p) plus faible que les objets légers. I . 4 . — Énergie mécanique d’un oscillateur harmonique À une constante additive près, sans intérêt en mécanique newtonienne, l’énergie mécanique s’écrit : Em =

mx˙ 2 K (x − x0)2 mv20X 2m KXm2 KX 2m mv20X 2m + = = sin 2(v0 t + f) + cos2 (v0 t + f) = 2 2 2 2 2 2

L’énergie d’un oscillateur harmonique est donc une constante qui est proportionnelle au carré v 20 de la pulsation et au carré X2m de l’amplitude. Le graphe de la figure 10.4 montre bien qu’au cours du temps, il y a transformation d’énergie cinétique en énergie potentielle et vice-versa. ´ Energies Em

Ep

0 T0

2T 0

F IG . 10.4.

Ek

t

155

Oscillateurs harmoniques. Oscillateurs amortis I . 5 . — Exemples a) Pendule élastique

i) Mouvement horizontal Considérons une masselotte A, de masse m, soumise à la force horizontale de rappel exercée par un ressort : −K (x − l0 ) ex , l0 étant sa longueur à vide (Fig. 10.5a). O

g l0

l1 K

X X

O K

A

A

x

x

a)

b)

F IG . 10.5.

Projetée suivant l’axe horizontal Ox, la loi fondamentale appliquée à A donne : ¨ + v20 X = 0 m ¨x = −K (x − l0 ) soit X

en divisant par m, en posant v 20 = K /m et en changeant de variable : X = x − l 0 . Par conséquent : X = X m cos(v 0t + f) et X˙ = −Xm v0 sin(v0t + f)

Supposons qu’initialement on écarte A de sa position d’équilibre de la distance d m et qu’on l’abandonne. Il vient : X(0) = dm = Xm cos f et X˙ (0) = 0 = −X mv0 sin f d’où f = 0, Xm = dm et X = dm cos(v0 t).

Remarque : Nous avons évité, ici aussi (cf. chapitre 4) d’écrire d’emblée la force élastique sous la forme −m v20 (x − l0 )e x , car cette force n’est pas proportionnelle à la masse (v20 = K /m) contrairement à ce qu’elle pourrait laisser penser à un lecteur débutant. Seule la force de gravitation présente cette singularité majeure (cf. chapitre 7).

 c Dunod – Toute reproduction non autorisée est un délit

ii) Mouvement vertical Dans ce cas, l’équation différentielle suivant la verticale descendante s’écrit (Fig. 10.5b) : m¨ x = −K(x − l0 ) + mg En introduisant la nouvelle position d’équilibre le = l0 + mg/K et la variable X = x − le , on obtient la même équation différentielle que précédemment : ¨ X + v20 X = 0. Ainsi, le rôle du poids, et plus généralement de toute force constante, est uniquement de modifier la position d’équilibre. On pourra, par conséquent, déterminer préalablement la position d’équilibre et écrire la force exercée par le ressort sous la forme −KX, X étant l’allongement compté à partir de cette position d’équilibre. Ordre de grandeur : Calculons la raideur K d’un ressort qui forme, avec une masse m = 0, 1 kg, un oscillateur harmonique de période T0 = 2p/v0 = 1 s : K = mv20 = 4p2 m/T02 ≈ 4 N . m−1 . b) Pendule simple Considérons un pendule simple constitué par une masselotte A, de masse m, astreinte par une liaison bilatérale à se déplacer sans frottement sur un guide circulaire C fixe, de rayon l (cf. chapitre 9).

156

10. Oscillateurs harmoniques. Oscillateurs amortis

En appliquant le théorème de l’énergie mécanique, on obtient, avec les notations habituelles (Fig. 10.6) : ¨u + g sin u = 0 l Si sin u ≈ u, l’oscillateur est harmonique puisque l’équation différentielle devient : ¨ u + v20 u = 0

v20 =

en posant

g l

Ce résultat fournit une méthode pour mesurer g à partir de la période T 0 = 2p/v 0 : g = lv20 = 4p2 l/T02. Notons que la longueur l d’un pendule simple qui bat la seconde, c’est-à-dire de période T0 = 2 s , est : g gT 20 l= 2 = ≈1m 4p 2 v0 Oz

y

y g

l

g

2r

C θ

θ x

0

−πr

A(m)

F IG . 10.6.

θ/2 A πr

x

F IG . 10.7.

c) Pendule cycloïdal Ce pendule, appelé aussi pendule d’Huygens, est un pendule simple dont la trajectoire non circulaire est telle qu’il présente la propriété d’isochronisme des oscillations, quelle que soit la valeur de l’amplitude, contrairement au pendule simple circulaire. Il est constitué par une masselotte A assujettie à glisser sans frottement sur une courbe cycloïdale, d’équations paramétriques (cf. chapitre 2) : x = r(u + sin u)

et y = r(1 − cos u)

u étant le paramètre angulaire (−Cy, CA) représenté sur la figure 10.7. La tangente à la courbe fait l’angle u/2 avec l’axe Ox ; en effet : dx = r(1 + cos u) du

et

dy = r sin u du

dy = dx

d’où

dy du

du dx

=

sin u 1 + cos u

ce qui s’écrit aussi :

dy 2 cos(u/2) sin(u/2) u = tan = 2 dx 2 cos (u/2) 2 L’axe Oy étant la verticale ascendante, l’équation du mouvement de A, de masse m, en projection suivant la tangente à la trajectoire, orientée dans le sens des abscisses curvilignes croissantes s, s’écrit : d2s m 2 = −mg sin dt

u 2

Exprimons s en fonction de u, sachant que s = 0 pour u = 0. Il vient : s=

ds =

2

2 1/2

(d x + d y )

u

2

2

(1 + cos a) + sin a

=r 0

puisque 2(1 + cos a) = 4 cos2(a/2). Ainsi : s = 4r sin(u/2).

1/2

u

d a = 2r

cos 0

a 2

da

157

Oscillateurs harmoniques. Oscillateurs amortis L’équation différentielle est donc : ¨s + v20 s = 0

avec

g 4r

v0 =

1/2

Le pendule d’Huygens est donc un oscillateur harmonique, quelle que soit l’amplitude des oscillations ; sa période s’écrit simplement en fonction du rayon de courbure R = 4r de la cycloïde en O : T0 =

2p 4r = 2p v0 g

1/2

= 2p

R g

1/2

On peut retrouver l’équation du mouvement à l’aide de l’énergie : Em = Ek + Ep = Cte

et

avec

Ek =

1 2 m˙s2 = mv 2 A 2

u s2 Ks 2 mg = 2mgr avec K = = 2 2 16r 2 4r On obtient bien une énergie potentielle quadratique et l’équation caractéristique suivante : E p = mgy A = mgr(1 − cos u) = 2mgr sin2

Em =

m ˙s2 Ks2 = Cte + 2 2

d) Oscillateur harmonique bidimensionnel On appelle ainsi un oscillateur à deux degrés de liberté, dont les deux équations du mouvement sont celles d’oscillateurs harmoniques isochrones, c’est-à-dire de même fréquence. Un corpuscule A, dont les positions suivant les axes Ox et Oy d’un référentiel R satisfont aux équations différentielles : ¨ x + v20 x = 0

et

¨ y + v20 y = 0

est un oscillateur harmonique bidimensionnel. On en déduit : x = A1 cos(v0 t + fx )

et y = A 2 cos(v0 t + fy )

ce qui s’écrit, en posant f = fx − f y :  c Dunod – Toute reproduction non autorisée est un délit

x = A1 cos(v0 t + f x )

et y = A2 cos(v 0t + fx − f) = A 2[cos(v 0t + f x ) cos f + sin(v0t + f x ) sin f]

En éliminant (v0 t + f x), on obtient l’équation cartésienne de la trajectoire : y x x2 cos f + 1 − 2 = A2 A1 A1

1/2

sin f

soit

2xy x2 y2 cos f = sin 2 f + 2 2 − A A A1 A2 1 2

La projection de la trajectoire de A dans le plan Oxy est une ellipse (Fig. 10.8). Concrètement, un tel mouvement est celui de la projection de l’extrémité d’un pendule simple qui n’est pas astreint à se déplacer dans un plan vertical. C’est aussi le mouvement de la trace de l’impact électronique sur un écran d’oscillographe cathodique, lorsque les tensions analysées suivant les axes de déviations horizontale et verticale varient sinusoïdalement à la même fréquence : l’ellipse est une courbe de Lissajous . Enfin, c’est le mouvement de l’extrémité du vecteur champ électrique d’une onde électromagnétique polarisée elliptiquement (cf. Électromagnétisme et Optique). Remarque : Lorsqu’il y a propagation d’onde (électromagnétisme ou optique), la phase telle que f x ou fy est conventionnellement définie par l’argument vt − fx et non vt + f x.

158

10. Oscillateurs harmoniques. Oscillateurs amortis y A 0

x

F IG . 10.8.

II . — OSCILLATEURS AMORTIS PAR FROTTEMENT VISQUEUX Au cours du mouvement de l’oscillateur, on observe généralement que l’amplitude des oscillations n’est pas constante, mais décroît constamment. On rend compte de cette décroissance en introduisant une force supplémentaire de frottement proportionnelle à la vitesse, −av (a > 0), appelée force de frottement visqueux de Stokes (cf. chapitre 31). Sans restreindre sa généralité, nous allons rendre l’étude concrète en considérant l’exemple du ressort élastique. II . 1 . — Équation différentielle du mouvement Désignons désormais par x la position de la masselotte, à partir de sa position d’équilibre. L’équation différentielle s’écrit : a K m¨ x = −Kx − a ˙x soit ¨ x + x˙ + x = 0 m m Le rapport a/m étant homogène à l’inverse d’une durée, posons a/m = 1/te . La signification de la durée te apparaîtra clairement une fois l’équation différentielle résolue. On a donc l’équation canonique suivante : x˙ K m ¨ + v20 x = 0 avec v20 = et te = x+ te a m ˙ cette équation différentielle est linéaire : toute comLe terme d’amortissement étant proportionnel à x, binaison linéaire de solutions est aussi solution. II . 2 . — Nature du mouvement En cherchant des solutions de l’équation différentielle en exp(rt), on trouve l’équation caractéristique du deuxième degré : r r2+ + v20 = 0 te dont les solutions sont : r1 = −

1 + v0 2te

1 −1 4v20 t2e

1/2

et r 2 = −

1 − v0 2te

1 −1 4v20t 2e

1/2

La solution x(t) la plus générale se met donc sous la forme d’une combinaison linéaire des deux solutions exp(r1t) et exp(r2 t) (cf. annexe 3) : x = C 1 exp(r 1t) + C2 exp(r2 t) Suivant les valeurs du produit sans dimension : Q = v0 te appelé facteur de qualité, on distingue trois types de mouvement.

159

Oscillateurs harmoniques. Oscillateurs amortis a) Oscillateur faiblement amorti (Q > 1/2) Si Q > 1/2, ce qui est le cas le plus fréquent, on a, en introduisant j 2 = −1 : r=−

1 ± jva 2te

avec

va = v0

1−

1/2

1 4Q2

On en déduit : x(t) = C exp −

t 2te

cos(va t + fa )

Sur la figure 10.9, on a représenté l’évolution de x. L’allure de la courbe est celle d’un mouvement oscillatoire amorti, d’où le nom de pseudo-pulsation donné à va et de pseudo-période donnée à la quantité : Ta =

2p 1 = T0 1 − 4Q2 va

x

−1/2

Q >1/2

v0 t exp − 2¿e !a 0

2¼=!a t −

v0 t exp − 2¿e !a

 c Dunod – Toute reproduction non autorisée est un délit

F IG . 10.9.

On détermine les constantes C et f a à l’aide des conditions initiales sur la position et sur la vitesse. Supposons qu’à l’instant t = 0 on ait x = 0 et ˙x = v0 ; il vient : 1 x(0) = 0 = C cos f a et ˙x(0) = v0 = C −va sin f a − cos fa 2te car x˙ = C exp [−t/(2te)] [−v a sin(v at + fa ) − 1/(2t e ) cos(vat + f a)] . Il en résulte : v0 t exp − sin(v at) fa = p/2, C = −v 0 /v a d’où x(t) = 2te va La pseudo-sinusoïde est en contact avec les courbes (v0/va) exp[−t/(2t e)] et −(v 0/v a) exp[−t/(2te )] aux instants tc définis par : sin(v at) t v0 ˙x(t) = exp − − + va cos(va t) = 0 2t e 2t e va Par conséquent : 1 nTa tan(va tc ) = 2vate d’où tc = arctan(2v ate ) + 2 va Ta = 2p/va étant la pseudo-période du mouvement et n un entier. Ainsi, te est la durée au bout de laquelle l’amplitude est divisée par e1/2 ≈ 1, 5. Comme l’amplitude est nulle après quelques valeurs de te, on dit que t e caractérise la durée de vie de ces oscillations amorties et on l’appelle la durée de relaxation en énergie (Fig. 10.9).

160

10. Oscillateurs harmoniques. Oscillateurs amortis

Remarques : (1) On introduit souvent une durée de relaxation t a relative à l’amplitude qui vaut le double de celle définie ici : t a = 2te . Le choix de te , qui revient à privilégier le concept d’énergie d’un oscillateur par rapport à celui d’amplitude, est fortement suggéré par les définitions adoptées en physique moderne. (2) Le facteur de qualité d’un oscillateur harmonique est infini. Dans le cas important où Q  1/2, on obtient les formules approchées suivantes : va ≈ v0

1−

1 8Q2

Ta ≈ T0 1 +

1 8Q2

Ta T +n a 4 2

et t c ≈

i) Décrément logarithmique On caractérise aussi la décroissance de l’amplitude d’un oscillateur amorti par le décrément logarithmique L : 1 x(t) L = ln n x(t + nT a) x(t) et x(t + nT a) étant les élongations à des instants séparés par un nombre entier de périodes. On a : L=

1 C exp(−t/2te ) cos(va t + fa ) ln n C exp[−(t + nT a)/2t e] cos(v a t + fa )

d’où

L=

Ta p = 2te vat e

Ainsi, L est le rapport de la durée de la pseudo-période du mouvement sur la durée de relaxation en amplitude. Exemple : La durée de relaxation t e d’un oscillateur mécanique, de pseudo-période T a = 1 s, dont le mouvement a une amplitude qui est divisée par quatre au bout de dix oscillations, est obtenue selon : te =

Ta 2L

avec

L=

1 ln 4 = 0, 14 10

d’où

t e ≈ 3, 6 s

ii) Facteur de qualité et perte d’énergie relative On caractérise souvent l’amortissement d’un oscillateur par le facteur de qualité Q. Montrons que Q est relié à la perte d’énergie relative d’un oscillateur très peu amorti. Il vient, si Q  1/2 et donc va ≈ v 0 : Em =

1 2 1 2 1 mx˙ + Kx = m 2 2 2

v 20 v2a

exp −

t te

va cos(v at) −

sin(vat) 2te

2

+ v20 sin 2(va t)

Or le terme entre accolades vaut pratiquement v2a . Par conséquent : Em ≈

1 m 2

v 20 v2a

exp −

t te

v2a = Em (0) exp −

t te

On en déduit la perte d’énergie relative par unité de temps : 1 d Em 1 ≈ te Em d t ce qui donne, pendant une durée égale à la pseudo-période T a : −

2p 2p −DEm Ta ≈ ≈ = et Em te vat e Q

Q≈

Em p ≈ 2p L −DE m

Ainsi, le facteur de qualité d’un oscillateur faiblement amorti par frottement visqueux est une mesure de l’inverse de sa perte d’énergie relative pendant une durée égale à une pseudo-période.

161

Oscillateurs harmoniques. Oscillateurs amortis

Ordre de grandeur : Le facteur de qualité d’un oscillateur mécanique, dont le décrément logarithmique est L = 0, 14, vaut Q ≈ p/L = 22, 4. Dans le cas d’oscillateurs mécaniques tels que ceux que l’on utilise en sismographie (cf. chapitre 11), le facteur de qualité dépasse facilement la valeur 1 000. Certains oscillateurs mécaniques tels que les quartz ont des facteurs de qualité qui atteignent 10 6 . b) Oscillateur très amorti (Q < 1/2) Dans le cas où Q < 1/2, la solution de l’équation caractéristique du deuxième degré est : r=−

1 ±b 2te

avec

1 −1 4Q2

b = v0

1/2

L’élongation s’écrit donc : x(t) = exp −

t 2te

[C 1 exp(bt) + C 2 exp(−bt)]

Avec les mêmes conditions initiales que précédemment, on trouve (Fig. 10.10) : x(t) =

t v0 exp − 2te b

x

sinh(bt) x

Q > 1/2

Q =1/2

Cas critique

0

0

t

t

F IG . 10.10.

F IG . 10.11.

 c Dunod – Toute reproduction non autorisée est un délit

c) Amortissement critique (Q = 1/2) Si Q = 1/2, l’équation caractéristique du deuxième degré admet une racine double r = −1/(2te ). Par conséquent C 1 exp[−t/(2t e)] est une première solution de l’équation différentielle. Or x(t) = C2 t exp[−t/(2t e )] est aussi une solution (cf. annexe 4) ; en effet, en injectant cette solution dans l’équation différentielle canonique ¨ x + ˙x/te + v 20x = 0 , on trouve bien, en utilisant v20 te = 1/2 : C2 exp −

t 2te



1 t + 1− 2t e 2te



1 2te

+

1 t + v 20 t = 0 1− 2te te

Il en résulte que x(t), qui est une combinaison linéaire des deux solutions, s’écrit : x(t) = exp −

t 2te

(C 1 + C 2t)

Dans les mêmes conditions initiales que précédemment, on trouve (Fig. 10.11) : x(t) = v0 t exp −

t 2te

Ce cas est qualifié de critique. Dans la pratique, on règle souvent l’amortissement près de sa valeur critique ; on évite ainsi une trop longue attente de la position d’équilibre.

162

10. Oscillateurs harmoniques. Oscillateurs amortis

III . — ANALOGIE ÉLECTRIQUE III . 1 . — Circuit RLC Considérons un circuit électrique fermé constitué d’un condensateur de capacité C, d’une bobine d’inductance L et d’une résistance R placés en série (Fig. 10.12a). Si le condensateur est initialement chargé, on constate qu’il se décharge dans le reste du circuit, de façon oscillante : la charge q du condensateur évolue suivant une loi sinusoïdale amortie. On met en évidence une telle évolution en visualisant sur un oscilloscope la tension uC = q/C aux bornes du condensateur (cf. Électronique). On rend possible cette visualisation en utilisant un générateur de signaux carrés qui reproduit, avec une fréquence suffisante, l’excitation initiale du condensateur.

L

i q

R

A

++ ++

q

C

R

L

i

+ −

A

++ ++

C R

N a)

Syst e`me actif

R1

M

R1

b) F IG . 10.12.

La loi des mailles, appliquée à un tel circuit, en régime quasi stationnaire, donne (cf. Électromagnétisme) :

di q = −L − Ri dt C

soit

L

di dq q = 0 avec i = + Ri + dt dt C

où q est la charge de l’armature A du condensateur vers laquelle est orienté le courant. Il en résulte l’équation différentielle canonique : ¨ q+

q˙ + v 20 q = 0 te

avec

v0 =

1 LC

1/2

et te =

L R

Ce système électrique est donc dynamiquement équivalent à l’oscillateur amorti : la correspondance entre les différentes grandeurs est donnée dans le tableau 10.1. Le rôle de la masse m est tenu par l’inductance L, celui de la raideur K par l’inverse 1/C de la capacité, enfin celui de l’amortissement a par la résistance R. Ce dernier résultat n’est guère surprenant lorsqu’on sait que tout résistor dissipe de l’énergie électrique (cf. Électromagnétisme). L’expression du facteur de qualité de ce circuit s’en déduit aisément : Lv 0 Q = v 0 te = R Mécanique

x

v

a

m

K

Électricité

q

i

R

L

C −1

TAB . 10.1.

Ordre de grandeur : Dans un circuit électrique typique, où L = 0, 1 H, C = 10 mF, R = 14 V, on trouve : v0 = 103 rad . s−1, f 0 = 159, 15 Hz, T0 = 6, 28 ms, te = 7, 1 ms, Q ≈ 7, 1 et

163

Oscillateurs harmoniques. Oscillateurs amortis

fa ≈ 158, 75 Hz pour la fréquence de la vibration amortie, laquelle est dans ce cas pratiquement égale à la fréquence de la vibration harmonique. III . 2 . — Adjonction d’une résistance négative Du seul point de vue de l’équation différentielle à laquelle satisfait la charge du condensateur, il suffirait d’ajouter une résistance « négative » égale à −R dans le circuit pour compenser le terme dissipatif. Évidemment, un tel résistor n’existe pas. Cependant certains systèmes, que l’on qualifie d’actifs parce qu’ils contiennent une source auxiliaire d’énergie, se comportent comme des résistances négatives (cf. Électronique et exercice). La figure 10.12b représente un tel système incluant un amplificateur opérationnel qui joue précisément le rôle d’une résistance négative égale à −R ; conformément à l’usage, la source auxiliaire d’énergie n’a pas été représentée sur le schéma. On obtient ainsi un oscillateur harmonique électrique dont on peut vérifier l’expression de la période T 0 = 2p/v0 = 2p(LC)1/2 .

IV . — OSCILLATEUR AMORTI PAR FROTTEMENT SOLIDE Si l’oscillateur est amorti par frottement solide, la force de frottement est opposée à la vitesse et sa norme est proportionnelle à celle de la réaction (cf. chapitre 9). IV . 1 . — Équation différentielle du mouvement Considérons une masselotte A, de masse m, accrochée à un ressort, qui peut glisser sur une tige horizontale (Fig. 10.13a). L’équation vectorielle du mouvement de A, par rapport au référentiel terrestre, s’écrit : m a = R + m g − K (x − l 0) e x Elle s’explicite suivant l’axe du mouvement Ox et un axe perpendiculaire Oy comme suit :

 c Dunod – Toute reproduction non autorisée est un délit

m¨x = Rx − K (x − l0)

0 = R y − mg

D’autre part, on sait que |Rx | = m|Ry |. Par conséquent, en introduisant v20 = K /m et X = x − l 0, on obtient : X¨ + v20 X = εmg où ε = +1 si X˙ < 0 et ε = −1 si X˙ > 0. X D0

D0  4D1 (t/ T0)

y g O K

A(m)

D1 0 − D1

x

− D0

a) F IG . 10.13.

t1

t2  D0  4D 1 (t/ T0) b)

t

164

10. Oscillateurs harmoniques. Oscillateurs amortis

IV . 2 . — Nature du mouvement La solution de l’équation différentielle précédente est la somme de la solution générale de l’équation sans second membre et d’une solution particulière de l’équation globale. Elle s’écrit donc : mg ˙ = −Cv0 sin(v0t + f x ) X(t) = C cos(v0 t + fx ) + ε 2 d’où X v0 On doit distinguer deux cas selon que la vitesse X˙ est négative ou positive. i) La vitesse X˙ est négative (ε = 1) ; on a : X(t) = C cos(v0t + f x ) + D 1

avec

D1 =

mg v20

Si les conditions initiales sont X = D0 et X˙ = 0, il vient : D0 = C cos f x + D 1 et 0 = −C sin fx . On en déduit fx = 0 et C = D0 − D1. Par conséquent : X(t) = (D 0 − D1) cos(v0 t) + D1 et X˙ (t) = −v0(D0 − D1) sin(v 0t) Le graphe X(t) est une demi-alternance, de pente négative, d’une sinusoïde, de période T0 = 2p/v0 et centrée sur l’axe X = D1 . Cette phase s’achève à l’instant t1 tel que : X˙ (t 1) = −v0 (D0 − D 1) sin(v0 t1) = 0

soit pour

t1 =

T0 p = 2 v0

et X(t1) = −(D 0 − 2D1 )

ii) La vitesse X˙ est positive (ε = −1) ; dans cette phase, qui débute à t 1 = T 0/2, il vient : X(t) = C cos(v0t + f x ) − D 1 ˙ = 0, il vient : Comme les conditions à t1 = T0/2 sont X (t1 ) = −(D0 − 2D1) et X −(D0 − 2D1 ) = C cos(p + fx) − D1

et 0 = −v0 C sin(p + fx )

Par conséquent, f x = 0 et C = D0 − 3D1 . Il en résulte : X(t) = (D 0 − 3D 1) cos(v 0t) − D1

et

X˙ = −v0(D 0 − 3D 1) sin(v 0t)

Le graphe X (t) est une demi-alternance, de pente positive, d’une sinusoïde, de période T0 = 2p/v0 , centrée sur l’axe X = −D1 . Cette phase s’achève à l’instant t2 pour lequel : ˙ (t2) = −v 0(D0 − 3D1) sin(v0 t2 ) = 0 X

soit pour

t2 =

2p = T0 v0

et X(t 2) = D0 − 4D 1

IV . 3 . — Mouvement général Pour obtenir le mouvement général, il suffit de répéter, sur toute période T 0, la même analyse en distinguant la première demi-période à pente négative et la seconde demi-période à pente positive. Le graphe X (t) est donc constitué d’une succession de demi-alternances de sinusoïdes, de période T 0 = 2p/v0 centrées autour de la valeur εD 1 : pour ˙X < 0 (ε = 1), l’arche descendante est centrée sur la position D1 , alors que pour X˙ > 0 (ε = −1), l’arche montante est centrée sur la position symétrique −D1 (Fig. 10.13b). On en tire deux résultats essentiels qui permettent de distinguer l’amortissement solide de l’amortissement visqueux : i) la pulsation des oscillations amorties est la même qu’en l’absence de frottement v a = v0,

165

Oscillateurs harmoniques. Oscillateurs amortis

ii) les élongations maximales varient selon une progression arithmétique : D 0, D0 −4D1 , D0 −8D1 etc. Les courbes reliant les maxima et les minima d’oscillation sont deux droites d’équations : t X(t) = ± D0 − 4D1 T0 Ordre de grandeur : Pour un pendule élastique, de raideur K = 4 N . m −1, de masse m = 0, 1 kg, dont le facteur de frottement est 0, 1, on trouve : mg mmg 0, 1 × 0, 1 × 9, 80 D1 = 2 = = = 24, 5 mm K 4 v0 IV . 4 . — Arrêt du mouvement Dès qu’un extrémum de X (t), qui se caractérise par une vitesse nulle, tombe dans la bande délimitée par les deux valeurs opposées D1 et −D 1, la masselotte s’immobilise. En effet, dans cette bande, la force de rappel du ressort étant, en valeur absolue, inférieure à la force de frottement, si la masselotte a une vitesse nulle, elle la garde définitivement. Cette zone, définie par −D1  X  D1 , correspond à une incertitude sur la position de la masselotte égale à 2D1. Expérimentalement, on doit la prendre en compte, après avoir tenté de la réduire en diminuant le facteur de frottement m par un choix convenable des matériaux en contact. Remarque : Cette analyse se transpose à n’importe quel oscillateur soumis à un frottement solide, tel qu’un pendule simple constitué d’une perle enfilée dans un guide circulaire matériel.

V . — REPRÉSENTATION DANS L’ESPACE DES PHASES Pour un oscillateur, à un degré de liberté linéique, l’espace des phases ou l’espace des états est le plan cartésien (x, px ) avec p x = m x˙ (cf. chapitre 24). Nous nous proposons de représenter, dans un tel espace, l’oscillateur harmonique et l’oscillateur amorti par frottement visqueux. V . 1 . — Oscillateur harmonique dans l’espace des phases

 c Dunod – Toute reproduction non autorisée est un délit

Dans l’espace des phases bidimensionnel, le point représentatif de l’oscillateur harmonique décrit une trajectoire elliptique, puisque, d’après l’équation de conservation de l’énergie, on a (Fig. 10.14) : p 2x Kx2 + = Em = Cte 2m 2

soit

x2 p2x + = 1 avec a = a2 b2 ! 02

px

a

x 0

F IG . 10.14.

1/2

et b = (2mE m) 1/2

Amortissement faible Amortissement critique (parabole)

b 0

2Em K

Amortissement fort 1=¿ e F IG . 10.15.

166

10. Oscillateurs harmoniques. Oscillateurs amortis

V . 2 . — Oscillateur amorti par frottement visqueux dans l’espace des phases Pour l’oscillateur amorti par frottement visqueux, la résolution de l’équation caractéristique du deuxième degré a donné les solutions suivantes : r=−

1 2te

±

1 − v 20 2 4te

1/2

On répertorie parfois les différents cas à l’aide du plan cartésien ( 1/te , v20 ) (Fig. 10.15). Comme 1/te et v 20 sont positifs, seul le premier quadrant convient ; dans ce plan, la courbe donnant v 20 en fonction de 1/te , à l’amortissement critique, est une parabole puisque : v 20 = (1/t e) 2 /4. Sur l’axe des ordonnées, pour lequel l’amortissement est nul, on retrouve le cas harmonique : r = ±jv0 . Les points situés entre l’axe des ordonnées et la parabole correspondent à des oscillateurs faiblement amortis. Ceux qui sont situés au-dessous de la parabole représentent les oscillateurs fortement amortis.

Il est instructif de représenter la courbe des points figuratifs dans l’espace des phases. C’est une spirale convergente si l’amortissement est faible (Fig. 10.16a) et un nœud lorsque l’amortissement est fort (Fig. 10.16b) ; le nœud est naturellement critique pour Q = v 0t e = 1/2. px

0

px

x

0

a)

x b)

F IG . 10.16.

VI . — OSCILLATEURS PARAMÉTRIQUES VI . 1 . — Définition Un oscillateur paramétrique est un oscillateur décrit par l’équation différentielle : x˙ ¨x + + v2 (t) x = 0 te 2 dans laquelle le paramètre v (t) dépend du temps. Ainsi défini, l’oscillateur paramétrique est linéaire, puisque toute combinaison linéaire de solutions est aussi une solution. Si l’on peut négliger l’amortissement t e , l’équation différentielle se réduit à : ¨x + v 2(t) x = 0 Fréquemment, on rencontre en physique une telle équation avec : v 2(t) = v20[1 − h cos(Vt)]

V étant une pulsation d’excitation. Cette équation différentielle est appelée équation de Mathieu.

167

Oscillateurs harmoniques. Oscillateurs amortis VI . 2 . — Exemples mécaniques a) Pendule simple excité par déplacement de son extrémité

L’extrémité supérieure S d’un pendule simple est soumise à un mouvement oscillatoire vertical (Fig. 10.17). Son équation différentielle s’obtient en appliquant le théorème du moment cinétique au point S fixe dans le référentiel non galiléen R = Sx y  z , en translation par rapport au référentiel terrestre R le long de son axe vertical descendant Oz : d LS = SA × (mg − mae − a ˙u eu ) dt la force de Coriolis étant nulle car VR /R = 0. On obtient, en explicitant : ml2 ¨ u = −ml(g − ¨ z) sin u − la u˙

Comme OS = dm cos(Vt + fs), il vient, en posant v 20 = g/l et a/(ml) = 1/te : V2d m g

˙u ¨ u+ + v 20 1 + te

cos(Vt + f s) sin u = 0

L’équation canonique d’un tel oscillateur peut donc s’écrire, dans l’approximation linéaire (sin u ≈ u), pour dm suffisament faible (V 2 dm /g  1) et en choisissant fs = p : ¨x +

x˙ + v 20 [1 − h cos(Vt)]x = 0 te L

i

R uRL

i x

S O

l

C

uC

Multiplieur

x

 c Dunod – Toute reproduction non autorisée est un délit

z

u s= Ku Cu e

ue

µ

S

A(m) F IG . 10.17.

F IG . 10.18.

VI . 3 . — Exemple électrique L’exemple électrique d’oscillateur paramétrique est fourni par le circuit circuit RLC dans lequel le paramètre C qu’est la capacité du condensateur évolue selon : C(t) =

C0 1 − h cos(v e t)

L’équation différentielle à laquelle satisfait la charge de l’armature considérée du condensateur s’écrit : 1 1 − h cos(v e t) q˙ ¨ = = v20 [1 − h cos(ve t)] q + + v 2 (t)q = 0 avec v 2(t) = te LC(t) LC 0 Cette variation de C(t) était autrefois obtenue en faisant varier la distance entre les armatures du condensateur. Actuellement, on préfère utiliser un montage multiplieur tel que celui représenté sur la fi-

168

10. Oscillateurs harmoniques. Oscillateurs amortis

gure 10.18, dans lequel on reconnaît aisément les composants électriques L , C , R caractéristiques de l’oscillateur amorti. Avec les notations indiquées sur la figure, on obtient l’équation différentielle : ¨ q+

q˙ + v 20(1 − Kue )q = 0 avec te

v 20 =

1 LC

et te =

L R

ue étant la tension aux bornes du générateur à l’entrée du multiplieur et K un coefficient du multiplieur (cf. Électronique). a) Botafumeiro et balançoire Considérons un pendule simple dont on peut faire varier la longueur périodiquement, par exemple en tirant brusquement sur le fil qui passe par le point de suspension O (Fig. 10.19a). Comme les forces de tension qui s’exercent sur la masselotte A passent par le point de suspension, leurs moments sont nuls. Le mouvement s’effectuant dans un plan vertical Oxy , le théorème du moment cinétique donne : dL = −mgl(t) sin u e z avec dt

L = ml(t) 2 u˙ e z

Pendant la très faible durée t , pendant laquelle on tire ou on lâche le fil, le moment cinétique ne change pas, puisque, entre les instants tc et tc + t , on a : tc +t tc

dL dt = − dt

tc +t tc

mgl(t) sin u d t e z

d’où

(DL)ttcc t = −mg

t c +t tc

l(t) sin u d t e z

Or le second membre est négligeable si u ≈ 0 (passage à la verticale, puisque, pendant la durée t , l(t) sin u ≈ 0 . Le moment cinétique ne varie donc pas, pendant cette durée : (DL)ttcc+t ≈ 0

d’où

ml2(t c) u˙ (tc ) = ml2 (t c + t) u˙ (tc + t)

Entre les instants tc et t c + t , où la verticale est atteinte, la variation d’énergie cinétique a pour expression : Ek(t c + t) − Ek (tc) = On en déduit :

u˙ 2(tc + t) 1 2 1 ml (tc + t) u˙ 2(tc + t) − ml 2(t c)u˙ 2(t c) = Ek (t c) −1 2 2 u˙ 2(tc ) Ek (tc + t) u˙ 2(tc + t) = E k(t c) u˙2 (tc)

Cette variation d’énergie cinétique est due au travail de la force nécessaire pour modifier la longueur du fil. Ce travail est positif lorsqu’on raccourcit le fil et négatif quand on l’allonge. On voit que l’on augmente efficacement l’énergie cinétique, en diminuant la longueur du fil, si cette dernière est maximale, c’est-à-dire lorsque le pendule passe par la verticale. En revanche, aux extrémités, où le pendule a une vitesse nulle, on restitue la longueur initiale. Un exemple célèbre de ce type d’oscillateur est l’encensoir géant de la cathédrale de Saint-Jacques de Compostelle, appelé « botafumeiro » : « botar » signifie lancer en castillan et « fumeiro » fumée en galicien (Fig. 10.19a). La masse de ce récipient, rempli de braises et d’encens, est de 50 kg et sa longueur vaut l1 = 20, 6 m ; cette dernière diminue de 3 m lorsque le pendule passe par la verticale. Sur la figure, on voit, en appliquant la conservation de l’énergie mécanique au pendule de longueur OA0 = l0 , entre l’instant initial t = 0 (u = u0 ) et l’instant t c du premier passage par la verticale (u = 0) : Ek(B0) = mgl0 (1 − cos u0)

169

Oscillateurs harmoniques. Oscillateurs amortis O g

g C C

A0

B1 B0

C

C

a)

b) F IG . 10.19.

La corde étant alors rapidement raccourcie jusqu’à la longueur OB1 = l 1 , lors de ce passage, l’application de la conservation de l’énergie, entre cet instant t c + t et l’instant t 1 où le pendule fait l’angle u1 avec la verticale, donne : Ek(B1) = mgl1 (1 − cos u1) Il en résulte :

ml2 ˙u2(tc + t)/2 u˙ (tc + t) l2 Ek (B1 ) = 1 2 2 = = 02 ˙ c) Ek (B0 ) l1 ml0u˙ (t c)/2 u(t

˙ c + t) = ml20u(t ˙ c). Ainsi : puisque ml 21u(t l1(1 − cos u1 ) l2 Ek (B1) = = 02 l0(1 − cos u0 ) Ek (B0) l1 d’où l’on déduit :

l 30 l31 sin2 (u1/2) 1 = = avec K = h. Déterminer les positions d’équilibre stable. 4. Calculer la valeur de la fréquence d’oscillation dans le cas où h = 10 cm et l 0 = 15 cm. y S2

A

F h

S3

K

O F IG . 10.26.

A(m) x

O

S4 F IG . 10.27.

S1

x

Oscillateurs harmoniques. Oscillateurs amortis

177

P10– 7. Pendule élastique bidimensionnel à quatre ressorts Une masselotte A, de masse m, est accrochée à quatre ressorts identiques, de raideur K 0 et de longueur à vide l0 (Fig. 10.27). Au repos la masselotte est au centre du carré horizontal formé par les autres extrémités des ressorts. Le poids de A est compensé par une force de réaction opposée. 1. Montrer, à l’aide d’un raisonnement simple, que l’énergie potentielle élastique a pour expression quadratique : Ep = K (x2 + y2 )/2, K étant un coefficient que l’on exprimera en fonction de la raideur K0 d’un ressort. 2. En déduire la fréquence d’oscillation autour de la position d’équilibre. 3. Quelles sont les différentes trajectoires de A ? P10– 8. Pulsation d’un plasma. Plasmon Un métal peut être considéré comme un ensemble d’ions positifs, fixes, régulièrement espacés et d’un ensemble d’électrons libres. 1. Montrer que le déplacement de l’ensemble des électrons, suivant une direction, d’une valeur x autour de la position d’équilibre, produit un champ électrique. Préciser le sens et la valeur de ce champ, en fonction de x et du nombre d’électrons par unité de volume ne . 2. Exprimer, en fonction de n e , la pulsation vp d’oscillation du plasma. On posera q 2e = e2/(4pε0 ) avec e ≈ 1, 6 × 10−19 C. 3. Calculer la quantité v p en eV pour l’aluminium, de masse molaire 27 g . mol −1 et de masse volumique r ≈ 2, 7 g . cm −3. On rappelle que  ≈ 1, 05 × 10 −34 J . s. P10– 9. Potentiel de Morse La fonction énergie potentielle de Morse, en un point de coordonnée radiale r par rapport à un centre de force O, a pour expression : Ep,ef (r) = E 0 {exp[−2a(r − r 0)] − 2 exp[−a(r − r0 )]}

 c Dunod – Toute reproduction non autorisée est un délit

avec E0 ≈ 7,37 eV et r 0 = 109 pm pour la molécule de diazote.

1. Montrer que cette fonction passe par un minimum que l’on calculera. En déduire la signification de r 0, E 0 et 2a2 .

2. Effectuer le développement de la fonction E p,ef (r) jusqu’à l’ordre trois inclus. Sachant que a ≈ 3,14 × 1010 m −1 calculer les coefficients K et A directement reliés aux dérivées d’ordre 2 et 3. P10– 10. Période du botafumeiro La longueur du pendule du botafumeiro vaut 20, 6 m lorsque le pendule atteint son amplitude maximale ; elle vaut 3 m de moins lorsqu’il passe par sa position d’équilibre. On a mesuré la période de ce pendule lorsque son amplitude angulaire u m était de 13 ◦ puis 82◦ ; on a trouvé respectivement : T1 = 9, 1 s et T2 = 10, 5 s. Comparer ces valeurs à celles obtenues à l’aide de la formule approchée donnant la période T en fonction de l et u m.

11 Oscillations forcées. Résonance Nous savons qu’un oscillateur sinusoïdal excité, soit par un déplacement par rapport à sa position d’équilibre, soit par une vitesse initiale non nulle, oscille et s’amortit du fait de la présence des forces de frottement qui dissipent son énergie. La question à laquelle nous nous proposons de répondre dans ce chapitre est la suivante : comment se comporte un tel oscillateur lorsqu’on lui applique une force excitatrice sinusoïdale ? Cette question est essentielle car, à l’aide de l’analyse de Fourier, on peut ramener le cas d’une excitation quelconque à celui d’une somme d’excitations sinusoïdales (cf. Optique). Avant tout, donnons quelques exemples simples de systèmes physiques soumis à de forces excitatrices sinusoïdales.

I . — EXEMPLES PHYSIQUES D’OSCILLATIONS FORCÉES I . 1 . — Pendule élastique On peut provoquer des oscillations forcées d’un pendule élastique en lui imposant un déplacement sinusoïdal. Ce cas fréquent dans la pratique peut être illustré par l’exemple qui suit. L’oscillateur est constitué par un pendule élastique vertical A (masse m, raideur K) dont l’extrémité supérieure S a un mouvement oscillatoire harmonique, de pulsation v, imposé par un moteur (Fig. 11.1). On détecte le mouvement, de A, par rapport au référentiel du laboratoire R, supposé galiléen, en mesurant la variation de la tension entre une borne de référence et l’extrémité d’une tige conductrice liée à la masselotte en mouvement dans l’électrolyte. Écrivons l’équation différentielle du mouvement du pendule, en introduisant une force de frottement de Stokes F S = −av, proportionnelle à la vitesse v (cf. chapitre 31). L’axe Ox étant orienté suivant la verticale descendante, il vient : m a = m g − K(x − x S − l0) ex − av

où xS = OS désigne la coordonnée de l’extrémité supérieure du ressort. Comme x S = d m cos(vt + fe ), on a : m¨x = −K(x − l 0 ) − a˙x + mg + Kdm cos(vt + f e ) soit, en introduisant le = l 0 + mg/K et en procédant au changement de variable X = x − l 1 : ˙ + KX = Kd m cos(vt + fe ) mX¨ + a X

179

Oscillations forcées. Résonance

En divisant par la masse, on fait apparaître la pulsation propre v 0 = (K /m)1/2 et la durée de relaxation en énergie te = m/a. L’équation précédente prend alors la forme canonique suivante : X˙ X¨ + + v 20X = e m cos(vt + fe) te

avec

em =

Kd m Fm = m m

en posant Fm = Kdm

O O

S ω Moteur

g

K

R

L A

Ai ++++++++

u(t)

q

C

x F IG . 11.1.

F IG . 11.2.

I . 2 . — Source de tension sinusoïdale aux bornes du dipôle électronique RLC

 c Dunod – Toute reproduction non autorisée est un délit

On réalise facilement un problème électriquement équivalent au précédent en maintenant, aux bornes d’un dipôle électronique RLC, une tension sinusoïdale u(t) = um cos(vt + fe) (Fig. 11.2). La loi des mailles appliquée au circuit donne, si q(t) désigne la charge de l’armature A du condensateur (cf. Électronique) : q di dq u(t) = L + Ri + avec i = C dt dt Il en résulte que : d2 q dq q L 2 +R + = u m cos(vt + fe ) dt dt C 2 soit, en introduisant v0 = 1/(LC) et t e = L/R : ¨ q+

q˙ + v20 q = em cos(vt + fe) te

avec

em =

um L

I . 3 . — Modèle de l’électron élastiquement lié Lorsqu’on étudie, d’un point de vue classique (non quantique), le comportement de la matière soumise à l’action d’un champ électromagnétique (E, B), on constate qu’un modèle efficace d’interprétation et de prédiction consiste à admettre que les électrons atomiques sont élastiquement liés à leur position moyenne (cf. Électromagnétisme) : tout se passe comme si les électrons étaient soumis à une force de rappel −Kr, r étant l’écart par rapport à une position moyenne. On tient compte du rayonnement émis par ces électrons accélérés en introduisant une force supplémentaire équivalente à une force de frottement visqueux −av. Enfin, le champ électromagnétique exerce sur ces particules, de charge −e, la force sinusoïdale suivante : −eE = Fm cos(vt + fe ) e r

180

11. Oscillations forcées. Résonance

L’équation différentielle du mouvement est donc : Fm r˙ ¨ cos(vt + fe ) e r + v20 r = r+ te m En projection sur l’axe de la coordonnée radiale r, cette équation s’écrit : ¨ r+

r˙ + v 20 r = em cos(vt + fe) te

avec

em =

Fm m

I . 4 . — Sismographe Comme son nom l’indique, le sismographe est un instrument chargé d’enregistrer les mouvements vibratoires de l’écorce terrestre par rapport à un référentiel galiléen R. Il peut être représenté par un pendule élastique dont l’extrémité supérieure S est fixée au boîtier de l’instrument qui repose sur le sol (Fig. 11.3). Le mouvement de A par rapport à R  , non galiléen, s’obtient en ajoutant aux forces habituelles (poids m g, tension du ressort −K (x − l0 ) ex , force de frottement visqueux a ˙x ex ), la force d’inertie d’entraînement −m aS, la force d’inertie de Coriolis étant nulle du fait de la translation. On a donc : x˙  m aA/R = m g − K (x − l0 ) ex − a ˙x ex − m aS soit ¨ x  = g − v20(x  − l0) − xS −¨ te En introduisant, ici aussi, le = l 0+ mg/K et X = x − l e , on obtient, puisque xS = OO = dm cos(vt + fe ) et donc ¨ xS = −v 2 dm cos(vt + fe ) : X˙  ¨ X + + v 20 X  = e m(v) cos(vt + fe) te

em(v) = d mv2

y

O O

avec

S

y K

g x

A Enregistreur

Entrée

R

F IG . 11.3.

Système

Sortie

F IG . 11.4.

I . 5 . — Réponse linéaire et résonance Les équations différentielles précédentes se présentent toutes sous la forme canonique suivante : ¨x +

x˙ + v 20 x = em (v) cos(vt + fe ) te

l’amplitude em (v) du terme d’excitation étant parfois fonction de la pulsation. C’est le cas dans un sismographe et plus généralement si l’excitation est une fonction e(t) quelconque du temps ; en effet, l’amplitude des différentes composantes sinusoïdales de e(t) dépend de v (cf. Optique).

181

Oscillations forcées. Résonance

En théorie des systèmes, on considère l’oscillateur non excité comme un système qui fait correspondre une réponse ou « sortie » à une excitation ou « entrée » (Fig. 11.4). La question posée en introduction est alors exprimée autrement sous la forme suivante : quelle est la réponse du système si on le soumet à une excitation sinusoïdale ? Pour y répondre, on teste d’abord la linéarité du système soumis à l’excitation. On sait que l’oscillateur amorti par frottement de Stokes est linéaire ; si on le soumet à une combinaison linéaire de deux excitations ou entrées e1(t) et e2 (t), de sorties respectives s1(t) et s2 (t), le système admet comme sortie la même combinaison linéaire des réponses : l1 e1(t) + l 2e 2 (t)



l1s 1(t) + l 2 s2(t)

Dans ce cas, l’intérêt de l’analyse ne se réduit pas à la seule détermination du mouvement de l’oscillateur sous l’action d’une excitation sinusoïdale. L’étude concerne toute la théorie de la réponse linéaire d’un circuit électrique ou d’un matériau si on les soumet à une excitation quelconque. Cette dernière est décomposée en signaux sinusoïdaux dont on étudie les réponses qu’en donne le système. En recomposant linéairement ces sorties élémentaires, on obtient la réponse à l’excitation initiale. Le système se comporte différemment selon la valeur de la pulsation v de la vibration excitatrice. Le phénomène d’exaltation de certaines grandeurs que l’on observe lorsque v = v0 est appelé résonance. On le retrouve dans la plupart des domaines de la physique, notamment en physique moderne où on le décrit naturellement à l’aide du concept d’énergie.

II . — OSCILLATIONS FORCÉES. RÉSONANCE II . 1 . — Régime transitoire et régime forcé La solution de l’équation différentielle précédente : ˙x ¨ + v20 x = em cos(vt + fe ) x+ te est la somme de deux termes :

 c Dunod – Toute reproduction non autorisée est un délit

i) la solution générale de l’équation sans second membre (cf. chapitre 10) : t x(t) = A exp − cos(va t + fa) 2te ii) la solution particulière de l’équation totale : B cos(vt + f x ). Entre l’instant initial et une certaine durée, qui dépend de te , au-delà de laquelle le premier terme est négligeable devant le second, le régime est transitoire : t x(t) = A exp − cos(va t + fa ) + B cos(vt + fx ) 2te Une fois le régime transitoire achevé, on observe le régime forcée ou établi de la forme : x(t) = x m cos(vt + f x ) Dans la suite, nous étudierons uniquement ce régime forcé. II . 2 . — Élongation de l’oscillateur Afin de déterminer la solution particulière de l’équation différentielle canonique, nous allons utiliser la méthode complexe qui consiste à associer, à l’équation différentielle précédente, l’équation diffé-

182

11. Oscillations forcées. Résonance

rentielle suivante à laquelle satisfait la grandeur complexe x = x + jy avec j2 = −1 : x˙ ¨ x + + v20x = e m exp[j(vt + fe )] te La solution réelle x(t) s’obtient en prenant la partie réelle de x(t). Cherchons une solution de la forme : x(t) = xm exp(jvt)



x m = xm exp(jfx)

est l’amplitude complexe de l’élongation. Comme ¨ x = −v2x et x˙ = jv x, on a : v −v2 + j + v20 xm = a m exp[j(vt + fe)] = em exp(jvt) te em = em exp(jfe ) étant l’amplitude complexe de l’excitation. On en déduit : xm =

em 2 (−v + v 20) + jv/te

soit

xm =

Qem/v20 u[Q(1/u − u) + j]

en introduisant le rapport des pulsations u = v/v 0. Il en résulte que x(t) = x m cos(vt + fx ) avec : xm =

em (v20 − v 2) 2 + v2 /t2e

1/2

=

Qem /v20 u [1 + Q 2(u − 1/u)2 ]

1/2

=

em/v20 [(a 2 − 1)2 + u2 /Q2 ]1/2

et tan(fx − fe ) =

v/te 1 = 2 Q(u − 1/u) v2 − v0

II . 3 . — Vitesse de l’oscillateur ˙ écrivons x˙ sous la forme : Comme la vitesse de l’oscillateur est donnée par x, dx ˙x = = v m exp(jvt) = v m exp[j(vt + f v)] dt v m étant l’amplitude complexe de la vitesse, vm son amplitude et f v sa phase à l’origine. Or, d’après ce qui précède : Qe m /v20 ˙x = jvxm exp(jvt) = jv exp(jvt) u[Q(1/u − u) + j] En identifiant, on obtient :

vm =

em te 1 + jQ(u − 1/u)

On en déduit : vm = vxm =

emte [1

+ Q2 (u

1/2 − 1/u)2]

et

tan(fv − fe ) = −Q u −

La vitesse x˙ est ainsi en avance de phase de p/2 par rapport à l’élongation x : p p soit fv − f e = fx − f e + fv = fx + 2 2

1 u

183

Oscillations forcées. Résonance II . 4 . — Admittance généralisée. Résonance a) Admittance généralisée

Par définition, on appelle admittance généralisée Y g le rapport de l’amplitude complexe de la « sortie-vitesse » sur celle de « l’entrée-excitation » : Yg =

vm em

soit

Yg =

te 1 + jQ(u − 1/u)

Ainsi définie, Yg a la dimension d’une durée. On en déduit le module et sa phase : te Yg = |Yg| exp(jfy) avec |Y g| = et f y = fv − fe 1/2 1 + Q 2 (u − 1/u) 2

Conventionnellement, on introduit la différence de phase, notée w, entre la phase de l’excitation f e et celle de la vitesse fv : avec

w = −f y = fe − f v

tan w = Q u −

1 u

b) Résonance Sur la figure 11.5, on a représenté |Y g| et w en fonction de u et Q. On voit que, pour u = 1, c’est-à-dire pour une pulsation de l’excitation égale à la pulsation propre du système, le module de l’admittance généralisée passe par un maximum, qui vaut te ; en outre, la vitesse est en phase avec l’excitation. Notons que l’admittance prend une valeur infinie dans le cas limite où il n’y a pas d’amortissement. Yg Yg

p 2

max

w = fe -fv

0

 c Dunod – Toute reproduction non autorisée est un délit

0

v u=v

1 a)

v u=v

1

0

-p 2

0

b) F IG . 11.5.

On appelle résonance le phénomène d’exaltation de l’admittance généralisée d’un système qui fait correspondre un signal de sortie à un signal d’entrée, que l’on observe pour : v = v0 On estime l’importance de la résonance par la finesse du pic représentant le graphe |Yg |(u). Pour cela, √ on calcule les valeurs de u pour lesquelles, conventionnellement, |Y g| = |Y g|max/ 2. On a donc : Q2 u −

1 u

2

=1

soit

u−

1 ε = u Q

avec

ε = ±1

On obtient donc l’équation u2 − εu/Q − 1 = 0, dont les racines positives sont : u1 = −

1 1 + 1 + 4Q2 2Q 2Q

1/2

et u 2 =

1 1 + 1 + 4Q2 2Q 2Q

1/2

184

11. Oscillations forcées. Résonance

Ainsi, u2 − u1 = 1/Q. En posant Dv1/2 = v 2 − v 1 = (u2 − u 1) v 0, il vient : Q=

v0 Dv1/2

ou

Q=

f0 Df 1/2

en fonction de la fréquence. Le facteur de qualité Q s’identifie donc au rapport de la pulsation propre v0 sur la largeur totale à mi-hauteur Dv1/2 du pic de l’admittance généralisée à la résonance. i) Si Q est grand, c’est-à-dire Dv 1/2 faible pour une valeur de v0 donnée, la résonance est dite aiguë. Dans certains oscillateurs mécaniques utilisés en sismographie, le facteur Q peut atteindre des valeurs de l’ordre de 1 000. Dans des systèmes oscillants électromagnétiques comportant un quartz piézoélectrique, Q atteint des valeurs encore plus élevées, de l’ordre de 106 . ii) Dans le cas contraire où Q est faible, la résonance est dite floue.

III . — EXCITATION MAXIMALE INDÉPENDANTE DE LA PULSATION Dans cette section, on admet que l’amplitude de l’excitation est indépendante de la pulsation v, ce qui est fréquemment réalisé. C’est en effet le cas de l’expérience du pendule élastique, du modèle de l’électron élastiquement lié ; c’est aussi celui d’un dipôle électronique si l’amplitude de la tension sinusoïdale u(t) à ses bornes est indépendante de v. On a alors : em =

Fm m

ou

em =

um L

III . 1 . — Impédance mécanique et impédance électrique a) Impédance mécanique On appelle impédance mécanique de l’oscillateur le rapport entre l’amplitude complexe F m de la force excitatrice et celle vm de la vitesse : Zm =

1 Fm m = = a 1 + jQ u − vm Yg u

Il vient, en explicitant dans le cas mécanique (Q = mv0 /a, u = v/v0 et mv20 = K ) : Z m = a + j mv −

K v

On en déduit le module et la phase :

Z m = |Zm| exp(jw)

avec

|Z m | = a2 + mv −

K v

et w = f e − fv

tel que

tan w =

mv − K /v a

2 1/2

185

Oscillations forcées. Résonance b) Impédance électrique du dipôle RLC

On appelle impédance électrique du dipôle RLC le rapport entre l’amplitude complexe u m de la tension excitatrice et celle im de l’intensité du courant : Ze =

1 um L = R 1 + jQ u − = im Yg u

Il vient, en explicitant dans le cas électrique (Q = Lv0 /R, u = v/v0 et Lv20 = 1/C) : Ze = R + j Lv −

1 Cv

On en déduit le module et la phase : Ze = |Ze| exp(jw)

1 |Z e | = R + Lv − Cv 2

avec

2 1/2

et w = fe − fi tel que

tan w =

Lv − 1/(Cv) R

Évidemment |Z m | ou |Ze | passent par un minimum pour v = v0, quel que soit l’amortissement. À la résonance, l’impédance que présente l’oscillateur au milieu excitateur est minimale. Sur la figure 11.6, on a représenté Z m et Ze dans le plan complexe ; cette dernière construction est traditionnellement appelée la représentation de Fresnel de l’impédance électrique. Remarque : Une convention internationale recommande de désigner par la lettre w la différence de phase f u − fi entre la phase de la tension et celle de l’intensité.

 c Dunod – Toute reproduction non autorisée est un délit

K=!

1=C!

m!

|Z m| ' ®

L!

|Ze| ' R

a)

b) F IG . 11.6.

III . 2 . — Vitesse ou intensité au voisinage de la résonance L’amplitude de la vitesse s’écrit, en fonction de u et Q : vm =

Fm/a 1 + Q 2 (u − 1/u)2

1/2

Pour u = 1, vm est maximal et vaut Fm/a (Fig. 11.7a). Ainsi, comme l’admittance généralisée, l’amplitude de la vitesse passe par un maximum pour v = v0 , quel que soit l’amortissement et donc Q. Il en résulte qu’un moyen d’analyser le phénomène de résonance est d’étudier la variation de la vitesse du système considéré en fonction de la pulsation v : on dit qu’il y a résonance de vitesse.

186

11. Oscillations forcées. Résonance

Fm ®1

vm Áv − Áe = −' ¼ 2

Q1 Fm ®2

0

Q2 < Q1 1

0

u=



! !0

1

u=

¼ 2

! !0

b)

a) F IG . 11.7.

Cette variation de la vitesse en fonction de la fréquence peut être mise en évidence dans l’expérience initiale faite avec le pendule élastique. Il suffit d’utiliser un circuit dérivateur qui permet d’avoir la dérivée par rapport au temps du signal électrique proportionnel à l’élongation. On constate bien que l’amplitude de la vitesse est maximale pour v = v0 , quel que soit l’amortissement. Quant à la différence entre la phase de la vitesse et celle de la force excitatrice, on l’obtient directement à partir de w puisque : fv − fe = −w Il en résulte que la vitesse et la force excitatrice sont en phase à la résonance. Lorsque u varie de 0 jusqu’à l’infini, la différence de phase passe de p/2 à −p/2 (Fig. 11.7b). Si l’amortissement est nul, l’amplitude de la vitesse est infinie ; la phase, elle, vaut p/2 pour v < v0 et −p/2 pour v > v0 . Ces résultats sur la vitesse se transposent directement à l’intensité dans le circuit série RLC excité par un générateur de tension sinusoïdale. On obtient alors la résonance d’intensité.

Remarque : Du point de vue de la théorie du filtrage fréquentiel d’une excitation par un système, on peut dire que l’oscillateur apparaît comme un filtre passe bande, puisqu’il transmet avec une efficacité maximale l’excitation, si celle-ci a une pulsation égale à sa pulsation propre. III . 3 . — Élongation au voisinage de la résonance L’amplitude réelle x m de l’élongation s’écrit, en fonction de u et Q : xm =

Qem /v20 u [1

+ Q 2(u

− 1/u) 2] 1/2

=

QFm /K [u 2

+ Q2(u 2 − 1)2]1/2

Pour étudier la variation de xm, étudions la fonction suivante ayant la signification d’un facteur de transmission |H(u)| : 1 Kxm |H(u)| = = 2 2 QF m [u + Q (u2 − 1)2]1/2 Il vient, en dérivant :

1 1/2 2Q 2 √ Ainsi, l’amplitude réelle xm vaut Fm /K si u = 0 et s’annule à l’infini ; si Q > 1/ 2, ce qui est souvent le cas, xm passe par un maximum pour u = um, On dit parfois qu’il y a résonance en d |H (u)| 1 + 2Q 2(u 2 − 1) =0 = −u 2 du [u + Q2 (u2 − 1)2 ]3/2

pour

u = 0 et u = um = 1 −

187

Oscillations forcées. Résonance

élongation, mais notons que ce maximum est obtenu pour une pulsation v m inférieure à la pulsation amortie va et à la pulsation propre v 0 : vm < va < v0

avec

vm = v 0 1 −

1 2Q 2

1/2

et va = v0 1 −

1/2

1 4Q2

Cependant um ≈ 1 si Q  1 (Fig. 11.8a). L’amplitude de l’élongation vaut alors x m ≈ QF m /K . On met en évidence expérimentalement ce maximum à l’aide du montage de la figure 11.1 : en faisant varier la vitesse de rotation du moteur, on observe aisément, pour un amortissement faible, la forte augmentation de xm . Remarque : Évidemment, dans le cas limite où il n’y a pas d’amortissement, l’amplitude xm de l’élongation devient infinie pour v = v0 . xm

QFm K

Á x − Áe = −' −

Q 1 = 100 0

Q2 = 5 Fm K



Q3 = 0;2 0

1

u=

¼ 2 u=

1

! !0

¼ 2

−¼

! !0

a)

b) F IG . 11.8.

En ce qui concerne la différence de phase f x − f e, on l’obtient immédiatement en retranchant p/2 à la différence de phase f v − f e :

 c Dunod – Toute reproduction non autorisée est un délit

fx − fe = f v − fe −

p p = −w − 2 2

Elle varie donc entre 0 et −p lorsque u passe de 0 à l’infini. Ainsi, l’élongation de l’oscillateur est toujours en retard sur l’excitateur et ce retard vaut p/2 à la résonance (Fig. 11.8b). Si l’amortissement est nul, le maximum est infini et se produit pour v = v0 ; la phase vaut alors 0 si v < v 0 et p si v > v0 . Les résultats relatifs à l’élongation se transposent aisément à la tension aux bornes du condensateur dans le circuit série RLC. Remarque : (1) Notons que, contrairement à l’admittance et à l’amplitude de√la vitesse, l’amplitude de l’élongation x m ne passe par un maximum que si Q > 1/ 2 et, qu’en outre, ce maximum, lorsqu’il existe, ne se produit pas rigoureusement pour v = v 0 . (2) Ici aussi, du point de vue de la théorie du filtrage fréquentiel d’une excitation par un système, l’oscillateur apparaît comme un filtre passe bande. Cependant, dans le cas de l’élongation, le filtrage √ peut être moins efficace si l’amortissement est trop important ; par exemple, si Q < 1/ 2, il n’y a pas de maximum dans le voisinage de la pulsation propre.

188

11. Oscillations forcées. Résonance

III . 4 . — Étude énergétique a) Puissance mécanique fournie par l’excitateur à l’oscillateur La puissance fournie par l’excitateur à l’oscillateur, par l’intermédiaire du terme F m cos(vt + f e), a pour expression : P = F · v = F m cos(vt + fe ) vm cos(vt + f v) =

F2m [cos(2vt + fe + f v) + cos w] 2|Zm |

puisque vm = F m/|Zm |. La puissance évolue donc sinusoïdalement autour de la valeur moyenne P suivante : v2 F2 F2 P = m cos w soit P = a m 2 = a m 2|Zm | 2|Zm | 2 d’après la relation cos w = a/|Zm | , que l’on établit aisément à l’aide de la représentation de Fresnel de l’impédance. En remplaçant vm par son expression, on trouve : P=

F2m 1 a v2F 2m = 2 2 2 2 2 2 2 2m (v0 − v ) + v /te 2a 1 + Q (u − 1/u)2

Cette puissance moyenne fournie par l’excitateur sert à compenser la puissance perdue par frottement ; en effet, calculons la moyenne de la puissance de la force de frottement : v2m 2 Ce résultat rappelle le rôle dissipatif bien connu joué par un résistor dans un circuit électrique : P f = −av · v = −a ˙x2 = −a vm2 cos 2(vt + fv )

Pf = −R

d’où

Pf = −a

i2m = −R I 2 2

√ en introduisant l’intensité efficace I = im/ 2. Le signe moins exprime que cette puissance est perdue par le système. b) Variation de la puissance mécanique fournie par l’excitateur en fonction de la pulsation Étudions, en fonction de u = v/v 0 , l’expression de la puissance moyenne P transférée par la force excitatrice à l’oscillateur. Cette puissance s’annule pour les valeurs extrêmes de u et passe par un maximum pour u = 1 (Fig. 11.9a) : P=

P max

1 + Q2 (u − 1/u)

2

avec

P max =

F 2m 2a

Si l’on représente cette puissance moyenne en fonction, non de u, mais de U = ln u, on obtient une courbe symétrique (Fig. 11.9b) d’équation : P=

P max

2

1 + Q2 [exp U − exp(−U)]

=

Pmax 1 + 4Q 2 sinh 2 U

On voit que le transfert maximal de la puissance moyenne de l’excitateur vers l’oscillateur est maximal à la résonance. C’est la raison pour laquelle, en physique moderne, où l’aspect énergétique s’exprime plus simplement, on définit souvent la résonance par le transfert maximal d’énergie moyenne entre l’excitateur et l’oscillateur.

189

Oscillations forcées. Résonance P Pm

P

Pm

Q2 > Q1 Q1

0

1

u=

a)

! !0

0

U = ln

! !0

b)

F IG . 11.9.

Remarque : Pour observer, dans le montage initial (Fig. 11.1), le pic de puissance transférée en moyenne à l’oscillateur, une méthode consiste à dériver, à l’aide d’un dérivateur électronique, le signal électrique détecté qui est proportionnel à l’élongation, ensuite à le multiplier par cette dérivée à l’aide d’un multiplieur, et enfin à filtrer le produit en ne laissant passer que la composante continue. III . 5 . — Effets de non-linéarités sur la résonance On obtient une équation différentielle typique d’un oscillateur non linéaire en régime forcé en ajoutant un terme d’excitation sinusoïdale à une équation différentielle non linéaire, par exemple : x˙ ¨x + + v20 x(1 − bx2) = e m cos(vt + fe) te b étant un coefficient positif. L’amplitude X m varie, comme le montre la figure 11.10, en fonction du rapport u = v/v 0. Pour te suffisamment faible (a fort), la courbe x m(u) ressemble aux courbes correspondantes dans le cas linéaire (Fig. 11.10). En revanche, si t e est fort (a faible), la courbe se déforme au point que trois valeurs de l’amplitude peuvent correspondre à une valeur déterminée de la pulsation. Une telle équation différentielle permet d’expliquer l’instabilité d’oscillations moléculaires anharmoniques.

 c Dunod – Toute reproduction non autorisée est un délit

xm

te fort

te faible 0

u =

! !0

F IG . 11.10.

IV . — APPLICATIONS Les applications pratiques de la résonance sont nombreuses en mécanique et en électricité : soit on souhaite amplifier la réponse que donne un oscillateur à une excitation extérieure et on se place à la résonance ou dans son voisinage, soit, au contraire, on veut étouffer cette réponse et on s’en éloigne. En mécanique, on l’utilise surtout dans le second cas, alors qu’en électricité l’application principale relève du premier : on sélectionne un signal électrique parmi d’autres à l’aide d’un circuit résonnant de grand facteur de qualité.

190

11. Oscillations forcées. Résonance

IV . 1 . — Réponse des sismographes Supposons pour simplifier que le facteur de qualité d’un sismographe soit suffisamment grand, ce qui est généralement le cas. L’équation différentielle établie en section I peut être réduite à : X¨ + v20 X = d mv 2 cos(vt + fe ) La recherche d’une solution de la forme X = Xm cos(vt + fx ) donne : (−v 2 + v20 )Xm cos(vt + f x) = d mv2 cos(vt + fe ) Il en résulte, en introduisant u = v/v0 :

u2 = dm 1 − u2 avec fx − fe = 0 pour u < 1 et fx − fe = p pour u > 1. Sur la figure 11.11a, on a représenté le graphe X m(u) : il part de la valeur 0, passe par un maximum et tend vers une valeur asymptotique égale à l’amplitude de l’excitation. La valeur infinie de ce maximum est due au modèle simplifié choisi, pour lequel on a négligé la force de frottement visqueux. Ainsi, pour u  1, X m ≈ dm : l’amplitude Xm est égale à celle de l’excitation. Le système restitue donc les vibrations excitatrices, ce qui est précieux en sismographie. Il en résulte que v 0  v soit T0  T. On est alors limité par la réalisation de pendules de périodes propres longues. X m

 Xm

V´ehicule dm 0

1

O

! u= !0

Route

O Axe galiléen b)

a) F IG . 11.11.

IV . 2 . — Amortissement des vibrations excitatrices. Accéléromètre Dans l’étude précédente, l’autre cas extrême u  1 présente aussi un intérêt, car l’amplitude X m est alors beaucoup plus faible que l’amplitude de l’excitation : X m ≈ d m u2  d m Le système amortit donc les vibrations excitatrices. Ce résultat est utilisé dans les véhicules destinés à transporter des passagers. Ces véhicules peuvent être assimilés à des oscillateurs élastiques dont l’extrémité basse, en contact avec la route ou le rail, est soumise à un déplacement fonction du temps (Fig. 11.11b). Comme on souhaite éviter que les passagers, situés sur l’extrémité haute, soient incommodés par les oscillations verticales imposées par la route, on se place dans la situation où u  1. Notons que X m est proportionnelle à l’accélération de translation du repère R = O xyz  par rapport à R : v2 aS Xm ≈ dm 2 = 2 v0 v0

191

Oscillations forcées. Résonance

aS étant la norme de l’accélération de R en translation par rapport à R. Ainsi, lorsque u  1, l’oscillateur peut être utilisé comme un accéléromètre, puisque la mesure de Xm permet d’accéder à l’accélération de R . IV . 3 . — Diffusion La diffusion d’un rayonnement incident par un électron atomique constitue un exemple physique important d’application des oscillations forcées d’un oscillateur par une excitation. a) Modèle Une telle diffusion est interprétée classiquement à l’aide du modèle de l’électron élastiquement lié . On montre que la puissance moyenne diffusée est proportionnelle au carré de l’accélération de la particule et donc à la quantité : 4

v x2m = u4

Q 2Fm2 /m2 e2E2m u4 = u2 + Q 2(u2 − 1)2 m 2 (u2 − 1)2 + u2 /Q2

b) Différents types de diffusion Représentons le graphe de la fonction D(u) caractérisant la diffusion (Fig. 11.12a) : D(u) =

D(u)

Diffusion résonnante

u4 (u 2 − 1)2 + u2/Q 2

Lentille de projection Soufre colloïdal

Condenseur

Diffusion Thomson

1

Source de lumière blanche

Diffusion Rayleigh

Diaphragmeobjet Rayonnement bleuté

u

1 a)

Rayonnement rougi

b)

 c Dunod – Toute reproduction non autorisée est un délit

F IG . 11.12.

Trois domaines sont généralement considérés : i) Diffusion Rayleigh (u  1) . Lorsque u  1 : D(u) ≈ u4 =

v v0

4

=

Cte l4

où l désigne la longueur d’onde du rayonnement excitateur. Ainsi, dans ce cas, la puissance diffusée varie comme 1/l4 . C’est cette diffusion, étudiée par le physicien anglais J. Rayleigh, qui permet d’interpréter la couleur bleue du ciel. On peut mettre en évidence la diffusion Rayleigh à l’aide du montage représenté sur la figure 11.12b. On interpose sur le trajet de lumière blanche, entre un diaphragme circulaire et son image dans un plan d’observation, une cuve contenant une solution de thiosulfate de sodium (2Na +, S 2 O 2− 3 ). En ajoutant de l’acide chlorhydrique (HCl) à la solution, on provoque la formation de soufre colloïdal, ce qui augmente l’intensité de la diffusion et donc le rayonnement bleuté observé dans une direction perpendiculaire à la direction moyenne du faisceau lumineux. En même temps, le faisceau transmis devient rouge sur l’écran. On réalise la même expérience en versant du lait dans de l’eau.

192

11. Oscillations forcées. Résonance

ii) Diffusion de Rayleigh résonnante (u ≈ 1) . Comme dans ce cas u 2 − 1 ≈ 2(u − 1), D(u) s’écrit : 1 D(u) ≈ 4(u − 1) 2 + 1/Q2

iii) Diffusion Thomson (u  1) . Lorsque u  1, la fonction de diffusion est indépendante de u : D(u) ≈ 1. C’est ce que l’on observe avec des rayons X durs, pour lesquels u  1 : la puissance diffusée est indépendante de la fréquence. IV . 4 . — Cas d’une excitation quelconque. Filtrage mécanique Si l’excitation (déplacement ou force) n’est pas une fonction sinusoïdale du temps mais une fonction quelconque, l’analyse est menée en remplaçant cette dernière fonction par son développement de Fourier (cf. Optique). Comme le système est linéaire, on cherche la réponse que donne l’oscillateur aux différents termes du développement, puis on recompose les différentes réponses obtenues avec leurs coefficients respectifs. Notons que l’amplitude de chaque composante de Fourier dépend alors de la pulsation. Par exemple, dans le cas d’une force d’excitation, d’expression complexe : t exp(jv0 t) ex F = F0 exp − t l’analyse de Fourier montre que F est la somme de termes sinusoïdaux d’expression : t0 Fm (v) exp(jvt) ex avec Fm (v) = F0 1 + j(v − v0)t Les résultats établis dans la section III devront être totalement reconsidérés en raison de l’influence de la pulsation dans l’amplitude complexe de la force.

CONCLUSION Retenons les points essentiels. (1) Lorsqu’une excitation sinusoïdale agit sur un oscillateur, l’équation canonique caractéristique est la suivante : ˙x x+ ¨ + v20x = e m (v) cos(vt + fe ) te (2) L’excitation impose sa fréquence v en raison des forces de frottement. Pour déterminer l’amplitude et la phase de l’oscillateur, il suffit de chercher une solution particulière de cette équation, sinusoïdale et de même pulsation que celle de l’excitation. (3) Si la pulsation de l’excitateur est égale à celle de l’oscillateur, on constate que le module de l’admittance complexe généralisée du système, définie comme le rapport de l’amplitude complexe de la réponse en vitesse sur celle de l’excitation, est maximal : te te d’où |Y g| = Yg = 2 1 + jQ(u − 1/u) [1 + Q (u − 1/u) 2]1/2 C’est la résonance. (4) La différence de phase entre la réponse en vitesse x˙ de l’oscillateur et l’excitation est : 1 fv − fe = −w avec tan w = Q u − u À la résonance, vitesse et excitation sont en phase. La différence de phase entre la réponse en élongation x de l’oscillateur et l’excitation s’en déduit : p p fx − fe = f v − fe − = −w − 2 2 À la résonance, l’élongation est en retard de p/2 sur l’excitation.

193

Oscillations forcées. Résonance

(5) Dans le cas où l’amplitude de l’excitation est indépendante de la pulsation, on observe à la résonance : i) une amplitude de la vitesse maximale, ii) une puissance moyenne transférée de l’excitateur vers l’oscillateur qui est maximale, iii) une amplitude de l’élongation√proche de sa valeur maximale, laquelle n’existe que pour des systèmes peu amortis (Q > 1/ 2 ). L’analogie entre les systèmes mécaniques et les circuits électriques est évidente : on définit, en régime sinusoïdal forcé, une impédance complexe analogue à l’impédance d’un circuit RLC : Zm = a + j mv −

K v

et Ze = R + j Lv −

1 Cv

(6) Les oscillations forcées sont souvent utilisées, en mécanique dans la construction des sismographes, des amortisseurs et des accéléromètres, en électricité dans la réalisation de filtres passe-bande. (7) En électromagnétisme, les oscillations forcées permettent d’expliquer classiquement la diffusion de la lumière par les électrons atomiques, selon le modèle de l’électron élastiquement lié. (8) La résonance peut être soit un inconvénient (on peut provoquer la rupture d’un appareil), soit un avantage lorsqu’on veut augmenter la sensibilité du système ; c’est le cas dans la réception des signaux hertziens (cf. Électronique). Par extension, le mot résonance est souvent employé lorsqu’un système excité admet une grandeur en sortie qui passe par un maximum pour une valeur déterminée d’un paramètre dont dépend l’excitation.

EXERCICES ET PROBLÈMES P11– 1. Pendule élastique horizontal soumis à l’action d’une force sinusoïdale

 c Dunod – Toute reproduction non autorisée est un délit

Un pendule élastique est constitué d’une masselotte de masse 100 g, reliée à un ressort de raideur 16 N . m−1. La masselotte coulisse sans frottement solide sur une tige horizontale fixe dans le référentiel du laboratoire R. L’expérience montre que le mouvement est amorti par frottement visqueux : le décrément logarithmique L vaut 0, 05. On impose à l’autre extrémité S du ressort un mouvement sinusoïdal dont la fréquence est variable. 1. Établir l’équation différentielle du mouvement de la masselotte. 2. Calculer la durée de relaxation en énergie te et le facteur de qualité Q. 3. Pour quelle valeur de la fréquence excitatrice l’amplitude de la vitesse est-elle maximale ? Calculer alors l’impédance de l’oscillateur. P11– 2. Oscillations forcées d’une particule chargée dans un champ électrique sinusoïdal Une particule de charge électrique q, de masse m, liée élastiquement à un point fixe O, oscille harmoniquement sur un axe Ox, avec la pulsation v 0. Elle est soumise à l’action d’un champ électrique sinusoïdal E dirigé suivant Ox et d’expression E = E0 cos(vt) e x. 1. Écrire l’équation différentielle du mouvement de la charge. 2. Trouver la solution de cette équation sachant qu’à l’instant t = 0, x = 0 et x˙ = 0. 3. Quelle est la nature du mouvement lorsque v est voisin de v 0 ? Donner alors l’expression de l’énergie E de l’oscillateur.

194

11. Oscillations forcées. Résonance

P11– 3. Pendule élastique sur une tige en mouvement horizontal Une masselotte A, de masse m, ayant la forme d’un anneau, coulisse le long d’une tige horizontale Tx avec un frottement solide négligeable. Un ressort de raideur K placé autour de la tige est fixé aux extrémités en T et A. On donne à la tige un mouvement sinusoïdal rectiligne, de pulsation v et d’amplitude d m (Fig. 11.13). On admet qu’une force supplémentaire de frottement visqueux, proportionnelle à la vitesse de la masselotte par rapport à la tige et caractérisée par une durée de relaxation en énergie te , s’exerce sur A. En outre, on se place dans le cas où Q = v0t e  1/2. 1. Établir l’équation différentielle du mouvement de l’anneau par rapport à la tige. 2. Trouver l’amplitude du mouvement forcé de l’anneau par rapport à la tige. 3. On définit la bande passante de ce système par Dv = v 2 − v 1 , v1 et v 2 étant les pulsations pour lesquelles l’amplitude des oscillations de la masselotte par rapport à la tige est l’amplitude √ maximale divisée par 2. Calculer le produit Dvt e . K O T

A

x

F IG . 11.13.

P11– 4. Isolement de vibration On se propose d’étudier les vibrations que transmet une machine tournante au support horizontal sur lequel elle repose. On peut représenter cette machine par une masse M placée sur un ressort, de raideur K, muni d’un amortisseur et soumise à une force sinusoïdale F = mv2r cos(Vt) ex , r étant une longueur constante et m une quantité homogène à une masse. On désigne par x et xS respectivement les coordonnées de la masse M et du support suivant la verticale dans le référentiel du laboratoire ; les origines de x et x S sont prises en leurs positions de repos. 1. Établir l’équation différentielle du mouvement à laquelle satisfait x. 2. On s’intéresse au mouvement forcé mais on néglige, dans les calculs, l’influence de l’amortissement. Sachant que l’amplitude de mouvement du support est négligeable devant celle de la masse M, trouver le facteur d’amplification de la force transmise au support par rapport à la force sinusoïdale. À quelle condition ce rapport est-il égal à 0, 001 ? P11– 5. Force sinusoïdale sur un oscillateur harmonique de pulsation différente Étudier le mouvement d’un pendule élastique (masse m = 2 kg, fréquence propre f 0 = 4 Hz), lorsqu’on exerce sur lui une force sinusoïdale, d’amplitude 80 N et de fréquence f = 6 Hz. Initialement, le degré de liberté de l’oscillateur ainsi que sa dérivée par rappport au temps sont nuls ; en outre, la force a sa valeur maximale. P11– 6. Force sinusoïdale sur un oscillateur harmonique de même pulsation On suspend à l’extrémité inférieure d’un ressort vertical (raideur 48 N . m −1 ) une masselotte A, de masse 3 kg, et on lui applique une force sinusoïdale d’expression, en newton : 36 cos(4t). 1. Sachant que les termes d’amortissement sont négligeables, établir l’équation différentielle du mouvement. 2. Chercher une solution du mouvement de la forme : x = t [A cos(4t) + B sin(4t)] 3. En déduire la nature du mouvement sachant qu’initialement x = 0 et x˙ = 0. Tracer le graphe x(t).

195

Oscillations forcées. Résonance P11– 7. Diffusion de la lumière

Les électrons (masse m e , charge −e) des couches périphériques des différents atomes qui composent l’air peuvent être considérés comme des oscillateurs harmoniques dont la pulsation propre correspond à une radiation ultraviolette du spectre électromagnétique. On excite de tels oscillateurs en envoyant une onde lumineuse caractérisée par le champ électrique de la forme E 0 cos(vt). 1. Écrire l’équation du mouvement de ces oscillateurs. On posera K = m e v20 et t e = me /a, a étant le coefficient de frottement proportionnel à la vitesse. 2. Trouver l’expression du module du facteur d’amplification H = Kx m/(eE0) en fonction de u = v/v0 et du facteur de qualité Q. 3. Sachant que Q = 10 et que u varie dans le voisinage de 0, 2, montrer que H est sensiblement indépendante de la pulsation v. 4. La puissance émise par ces oscillateurs excités est proportionnelle au carré de l’amplitude de l’accélération des électrons. Comment varie-t-elle avec v ? En déduire une interprétation de la couleur bleue du ciel. P11– 8. Capteur de vibration Un ensemble comprend un support rigide et une masselotte A reliés par un ressort vertical et un amortisseur visqueux en parallèle. On l’utilise pour capter les vibrations d’une machine sur laquelle il repose (Fig. 11.14). La masselotte a une masse m = 0, 2 kg et le ressort une raideur K = 20 N . m −1 ; en outre, le coefficient d’amortissement de Stokes est égal à la moitié de sa valeur critique. O

y

O Support

K x

A

 c Dunod – Toute reproduction non autorisée est un délit

Machine

F IG . 11.14.

1. Trouver les caractéristiques v 0 et t e de cet oscillateur. En déduire le facteur de qualité Q. 2. La machine, avec laquelle le capteur est en contact a un mouvement sinusoïdal vertical de pulsation v et d’amplitude dm. Établir l’équation différentielle à laquelle satisfait le mouvement de la masselotte A par rapport au capteur. On désignera par X  la coordonnée de position de A par rapport à sa position d’équilibre dans le référentiel R lié au support du capteur. 3. Trouver l’expression de l’amplitude complexe de l’élongation X  en fonction de u = v/v0 et de Q. En déduire l’amplitude réelle Xm et sa phase f x par rapport à la phase du déplacement de la machine. Représenter le graphe Xm(u). Quelle est la valeur du rapport X m/d m pour u = 10 ? 4. Comment varient, en fonction de u, l’amplitude réelle de la vitesse X˙  et sa phase f v ?

196

11. Oscillations forcées. Résonance

P11– 9. Pendule simple excité par des oscillations horizontales Un pendule simple est constitué d’une masselotte A qui glisse, sans frottement, le long d’un guide circulaire de rayon l = 40 cm . Ce dernier, de centre I , est astreint à un mouvement sinusoïdal, d’amplitude hm et de pulsation v , suivant la direction horizontale Oy : y I = hm sin(vt) (Fig. 11.15). On désigne par u l’angle que fait le vecteur IA avec la verticale descendante Ox ; on prendra pour valeur du champ de pesanteur terrestre g = 9, 80 m.s−2 . 1. En appliquant le théorème du moment cinétique au point I , mobile dans le référentiel R d’étude, montrer que l’équation différentielle du mouvement en u s’écrit : y˙I ¨yI ¨ u − 2 u˙ sin u + cos u = −v 20 sin u l l dans laquelle v0 est une grandeur dont on donnera la signification physique et que l’on calculera. 2. Que devient l’équation précédente pour de petits mouvements ? L’équation obtenue rappelle celle caractéristique d’un phénomène physique important. Lequel ? 3. Sachant qu’initialement le pendule est au repos (u = 0 et u˙ = 0) , trouver la solution u(t) . Quelle est la nature générale du mouvement ? 4. Analyser et commenter les deux cas suivants. a) v/v0  1 . Calculer le facteur d’amplification en amplitude pour la fréquence f = 0, 01 Hz . b) v/v 0  1 . Calculer le facteur d’amplification en amplitude pour la fréquence f = 100 Hz .

g

O

I

y l

R µ x

F IG . 11.15.

A

12 Corps ponctuel soumis à une force centrale conservative

Le mouvement d’un corps ponctuel A soumis à une force centrale est celui pour lequel la force passe constamment par un centre O fixe dans le référentiel R considéré. Si cette force F dérive d’une énergie potentielle Ep , cette dernière ne dépend que de la norme r du vecteur r = OA ; en effet, e r étant le vecteur unitaire r/r : dE F = − grad E p(r) = − p er dr L’intérêt de l’étude de ce mouvement est lié au problème à deux corps (cf. chapitre 13), lequel est d’une importance considérable puisque l’étude concerne toutes les interactions fondamentales gravitationnelle, électromagnétique, forte, faible.

 c Dunod – Toute reproduction non autorisée est un délit

Avant d’étudier le cas particulier du problème de Kepler, où E p = K /r avec K < 0, nous devons rappeler les principaux résultats des mouvements à force centrale conservative (cf. chapitres 4 et 5).

I . — MOUVEMENTS À FORCE CENTRALE CONSERVATIVE I . 1 . — Conservation du moment cinétique La force étant centrale, son moment au centre O est nul. Le moment cinétique en O du corpuscule A, de masse m, par rapport au référentiel galiléen R, est donc un vecteur constant L : LO = OA × mv A = Cte = L Comme OA et vA doivent être constamment perpendiculaires à L, la trajectoire de A est contenue dans le plan perpendiculaire à L passant par O : OA · L = 0

vA · L = 0

Ainsi, le mouvement d’un corps ponctuel soumis à une force centrale est plan.

198

12. Corps ponctuel soumis à une force centrale conservative

Dans la suite, nous choisirons l’axe Oz du référentiel R suivant L et nous appellerons Oxy le plan dans lequel s’effectue le mouvement. En coordonnées polaires, L s’écrit : L = re r × m( ˙r er + rw˙ ew ) = mr2 w˙ ez Donc : L = Lz ez = Cte

Lz = mr2 w˙ = Cte

avec

Remarquons que w˙ a le signe de Lz et le garde constamment. En outre : L2z = L2. Pour des raisons historiques, on exprime parfois la conservation du moment cinétique à l’aide de la constante des aires C = r2w, ˙ qui n’est autre que le moment cinétique par unité de masse, ou à l’aide de la vitesse aréolaire a = d A/ d t , qui est la vitesse avec laquelle le rayon vecteur balaie l’aire du plan Oxy (Fig. 12.1a) : C = r 2w˙ =

Lz m

et

a

=

dA 1 = dt dt

r ×rdw Lz C = = 2 2m 2

Sur la figure 12.1b, on a représenté les aires égales balayées pendant des durées égales ; on voit que la vitesse au point B est plus faible qu’en A qui est plus proche du centre attractif O. z y

L

A

y

O dw

w

A

A

a)

B x

Oz

A

x

B

b) F IG . 12.1.

I . 2 . — Conservation de l’énergie Exprimons, en coordonnées polaires, la conservation de l’énergie mécanique de ce corps ponctuel soumis à une force qui dérive de l’énergie potentielle Ep (r) : Em = Ek + Ep(r) = Cte

donne

1 m(r˙2 + r 2 w˙2 ) + Ep (r) = Em 2

soit, en introduisant le carré de la norme du moment cinétique L2 = m2r 4w˙ 2 et l’énergie potentielle effective : L2 1 2 m ˙r + Ep,ef (r) = E m avec E p,ef (r) = Ep(r) + 2 2mr 2

199

Corps ponctuel soumis à une force centrale conservative I . 3 . — Équation de la trajectoire

D’après ce qui précède, on peut déduire, au moins en principe, la relation entre w et r. En effet : dw Lz w˙ = = r˙ dr r2 [2m(E m − Ep,ef )]1/2

La trajectoire peut donc être obtenue en calculant l’intégrale : w=

Lz d r + w0 r 2[2m(Em − E p,ef )]1/2

dans laquelle on a remplacé E p,ef par son expression. La trajectoire n’est pas nécessairement fermée ; on montre qu’elle peut l’être uniquement dans les deux cas importants suivants : i) L’énergie potentielle est de la forme E p = K /r ; c’est le problème de Kepler si K < 0 que nous étudierons de façon détaillée dans la suite. ii) L’énergie potentielle est quadratique Ep = Kr2/2 ; c’est l’oscillateur bidimensionnel si K > 0 (cf. Exercices). Aussi est-il intéressant, avant d’effectuer le calcul, de discuter qualitativement le mouvement à partir du graphe donnant l’énergie potentielle effective Ep,ef en fonction de r. On peut alors en déduire la nature de l’état, lié ou libre, du corps ponctuel, selon les valeurs de son énergie.

II . — PROBLÈME DE KEPLER On désigne par problème de Kepler (du nom de l’astronome allemand J. Kepler) le cas où la force centrale est newtonienne (ou coulombienne), c’est-à-dire de la forme K /r 2, K étant positif ou négatif suivant qu’elle est répulsive ou attractive. II . 1 . — Expression de l’énergie potentielle La force s’écrivant : F = (K /r 2 ) er , on trouve l’expression de l’énergie potentielle associée à F en exprimant le travail élémentaire dW :

 c Dunod – Toute reproduction non autorisée est un délit

dW = F · d r =

K K K e · d r = d r = − d r r2 r2 r

= − d E p avec

Ep =

K r

si l’on adopte comme origine de l’énergie potentielle celle pour r infini. Dans la suite, on écrira : K = ε|K | avec ε = sgn(K ) = 1 dans le cas répulsif et ε = sgn(K ) = −1 dans le cas attractif, la fonction sgn(x) représentant le signe de x. II . 2 . — Discussion qualitative du mouvement L’énergie potentielle effective a pour expression : E p,ef (r) =

K L2 + 2mr2 r

Sur la figure 12.2, on a représenté le graphe Ep,ef (r) dans le cas répulsif (K > 0) et dans le cas attractif (K < 0). On voit que, si la force est répulsive (a), les états possibles sont tous libres, alors que, si la force est attractive (b), les états peuvent être libres ou liés suivant que l’énergie mécanique est positive ou négative.

200

12. Corps ponctuel soumis à une force centrale conservative Énergies

Énergies

Em

L2 2mr 2

Cas répulsif (K > 0 )

E p,ef

Em

K=r

Cas attractif (K < 0 ) E p,ef

0

Em

L2 2mr 2 0

r

K=r r a)

b) F IG . 12.2.

II . 3 . — Loi de conservation caractéristique du problème de Kepler Il existe une troisième loi de conservation, de nature vectorielle, caractéristique du problème de Kepler. a) Troisième loi de conservation D’après la loi fondamentale, on sait que : m

dv K = 2er dt r

ce qui s’écrit aussi

dv K d ew K d ew =− 2 =− 2 dt mr d w mr w˙ d t

puisque er = − d ew / d w. En introduisant le moment cinétique en O, L = L ze z, on obtient : dv K d ew =− dt Lz d t

d’où en intégrant

v=−

K ew + w Lz

w étant un vecteur constant homogène à une vitesse. Ainsi : w=v+

K ew = Cte Lz

homogène à une vitesse, est une constante vectorielle du mouvement. b) Vecteur de Runge-Lenz Le vecteur de Runge-Lenz, du nom des physiciens allemands Runge et Lenz, en raison de son intérêt dans les problèmes de diffusion de particules, où l’interaction de type keplérien joue un rôle essentiel, est le vecteur constant R , défini à partir du vecteur constant w et du moment cinétique L en O, lui aussi constant : R = w ×L

soit

R = v × L + Ker

car ew × L = Lz er . On voit qu’il est homogène à la constante d’interaction K . Remarque : On introduit parfois le vecteur suivant j défini selon : Lz Lz w= v + ew K K que l’on appelle vecteur excentricité car sa norme est égale à l’excentricité e d’une conique, comme nous le verrons un peu plus loin. j=

201

Corps ponctuel soumis à une force centrale conservative II . 4 . — Formules de Binet

Les formules établies par J. Binet donnent les expressions de la vitesse v A et de l’accélération a A du point A en fonction des variables u = 1/r et w. En coordonnées polaires, ces vecteurs s’explicitent respectivement selon : ¨r − r w˙ 2 ˙r et 1 d 2 rw˙ (r w) ˙ =0 r dt Comme : r˙ =

dr du

Lz d u w˙ d u du w˙ = − 2 =− u dw m dw dw

rw˙ 2 =

L 2z u3 1 L2z u4 = u m2 m2

L2u 2 d2 u d dr d dr d L du − z = w˙ = w˙ =− z2 dt dt dw dt dw m dw m d w2 il vient, pour vA et a A, respectivement, puisque L 2z = L2 : et ¨r =

du Lz − d w m u

L2 − 2 m

et

d2 u + u u2 d w2 0

II . 5 . — Équation de la trajectoire Il existe plusieurs méthodes pour établir l’équation de la trajectoire. Évidemment, toutes s’appuient sur la loi fondamentale de la dynamique ou ses conséquences. La plus efficace est celle qui s’appuie sur les équation de conservation du moment cinétique et de l’énergie mécanique. a) Méthode par l’énergie mécanique Comme l’énergie mécanique se conserve, il vient : 1 L2 Em = Ek + Ep = m 2 2 m

du dw

2

+ u 2 + Ku soit

Em =

L2 2m

du dw

2

+ u2 + Ku

 c Dunod – Toute reproduction non autorisée est un délit

En dérivant par rapport à w, et en simplifiant par w˙ et d u/ d w , on trouve : L2 0= m

d2 u +u +K d w2

d’où

ε d2 u +u=− 2 dw p

en introduisant ε = sgn(K ) et le paramètre suivant p, homogène à une longueur : L2 p= m|K | La résolution de cette équation ne présente pas de difficulté ; la solution est la somme de la solution générale de l’équation sans second membre et d’une solution particulière : u = A cos(w − w0 ) −

ε p

ce qui s’écrit aussi u =

e cos(w − w 0 ) − ε p

202

12. Corps ponctuel soumis à une force centrale conservative

en posant e = Ap. Par conséquent : r=

p −ε + e cos(w − w0 )

Cette équation représente, en coordonnées polaires, une conique dont le centre O est l’un des foyers ; p est le paramètre de la conique et e l’excentricité (cf. annexe 2). Pour connaître la relation entre l’excentricité e et l’énergie E m , il suffit de remplacer, dans l’expression de l’énergie, u et (d u/ d w) par leurs expressions. Puisque d u/ d w = −e sin(w − w 0 )/p, il vient : Em =

L2 2m

du dw

2

+ u 2 + Ku =

1 2eε e cos(w − w 0) − ε L 2 e2 + 2 − cos(w − w0 ) + K 2 2m p p p p

d’où, en simplifiant : Em = −

|K | (1 − e 2) 2p

avec

e = 1+

1/2

2pE m |K |

b) Méthode à partir de la loi fondamentale L’application de la loi fondamentale de la dynamique donne, en utilisant les formules de Binet relatives à l’accélération : −

L2 m

d2 u + u u2 = Ku2 d w2

mK εm|K | ε d2 u +u=− 2 =− 2 =− 2 L L p dw

d’où

On retrouve bien l’équation différentielle précédente. c) Méthode par l’énergie potentielle effective L’intégrale donnant w à partir de la coordonnée radiale r peut être aisément effectuée en procédant au changement de variable suivant : a2 =

m 2K2 + 2mE m L2

et

b=

mK Lz + Lz r

En effet, comme : 2m Em − il vient : w= Il en résulte :

K L2 − r 2mr2

= a2 − b 2

Lz 1/2

r 2 {2m [Em − K /r − L 2/(2mr 2 )]} w = arccos

ce qui s’écrit aussi : cos(w − w 0) =

mK Lz + Lz r m2K 2 + 2mE m L2

1/2

=

et

db = −

dr =



(a2

Lz d r r2

db + w0 − b2)1/2

b + w0 a K L2 + |K | m|K |r

2L 2E m 1+ mK 2

1/2

p r 2pEm 1+ |K | ε+

=

1/2

203

Corps ponctuel soumis à une force centrale conservative en introduisant le paramètre p. On obtient donc : p ε + = e cos(w − w0) soit r

r=

p −ε + e cos(w − w0 )

Remarque : On peut retrouver l’équation de la trajectoire à l’aide du vecteur excentricité j . En effet, en multipliant scalairement les deux membres de son équation de définition par le vecteur unitaire ew perpendiculaire à er , on obtient : j · ew =

Lz v + e w · e w d’où K

j · ew =

Lz K v w + 1 et vw = (ew · j − 1) K Lz

Comme vw = rw˙ est aussi égal à L z/(mr), on en déduit : Lz K = (e w · j − 1) mr Lz

1 1 mK m|K | = 2 (ew · j − 1) = 2 (−ε + ε ew ·j) = (ε ew · j − ε) r L L p

d’où

puisque L2z = L 2. On retrouve ainsi l’expression de r : r=

p p = −ε + e w · ε j −ε + e cos(w − w0)



e = |ε j|

et où (w − w0) est l’angle que fait ew avec le vecteur constant εj . II . 6 . — Nature de la trajectoire a) Cas répulsif (K > 0) L’équation de la trajectoire s’écrit, puisque ε = 1 : r=

p −1 + e cos(w − w 0)

avec e > 1 car Em = mv2/2 + |K |/r > 0. Le dénominateur peut s’annuler ; r peut donc atteindre une valeur infinie : la trajectoire de A est une hyperbole de foyer O (Fig. 12.3). Hyperbole

 c Dunod – Toute reproduction non autorisée est un délit

y A r O

w w0 x F IG . 12.3.

b) Cas attractif (K < 0) Si ε = −1, l’équation de la trajectoire en coordonnées polaires est : r=

p 1 + e cos(w − w 0)

Comme prévu, lors de la discussion qualitative, on distingue les états libres des états liés suivant la valeur de l’énergie.

204

12. Corps ponctuel soumis à une force centrale conservative

i) États libres pour lesquels E m  0 (e  1) ; la conique est une hyperbole (Fig. 12.4a) si Em > 0 ( e > 1 ) ; c’est une parabole dans le cas limite où Em = 0 (e = 1).

ii) États liés pour lesquels E m < 0 ( e < 1) ; la conique est une ellipse (Fig. 12.4b). Dans le cas particulier où l’énergie mécanique Em prend la valeur minimale E 0 de l’énergie potentielle effective, il n’y a pas de mouvement radial : l’excentricité atteint sa valeur minimale e = 0 ; la trajectoire est alors un cercle. y

Y y

A

Hyperbole

A

p b

X w0 x

c O ¡

w O

w

Am

a

w0 x

AM

a)

b) F IG . 12.4.

II . 7 . — Étude particulière de l’état lié Rappelons les conditions d’un mouvement elliptique : la force doit être attractive (K < 0) et l’énergie mécanique E m négative mais supérieure à une valeur minimale E 0 de l’énergie potentielle effective : 1 |K | 0, la conique est toujours une hyperbole. Si K < 0 la conique est, soit une hyperbole, soit une ellipse suivant que l’état est libre ou lié ; à la frontière de ces deux mouvements, il y a l’état libre singulier correspondant à une trajectoire parabolique.

Corps ponctuel soumis à une force centrale conservative

213

(4) En ce qui concerne le mouvement des planètes et des satellites, retenons les trois lois de Kepler ainsi que les expressions de la vitesse d’évasion et de la vitesse de satellisation d’un satellite artificiel de la Terre : 2GM T 1/2 √ GMT 1/2 et v l = vs = = 2 vs r r Cette étude concerne aussi le cas d’une force répulsive pour lequel les états possibles sont toujours libres. Nous étudierons ultérieurement ces états (cf. chapitre 15).

EXERCICES ET PROBLÈMES P12– 1. Oscillateur bidimensionnel Une particule (masse m = 0, 1 kg) est soumise à une seule force dont l’énergie potentielle est de la forme : E p = Kr2 /2, K étant une constante d’interaction qui vaut 40 N . m−1 . 1. Trouver l’expression de la force correspondante. En déduire que le mouvement est plan.

2. Écrire les lois de conservation du mouvement de la particule, en fonction des coordonnées polaires (r, w). Calculer la valeur du moment cinétique, sachant qu’à l’instant pris comme origine r = r0 = 1, 2 m, v0 = 10 m . s−1 et que l’angle a0 = (r0 , v0) vaut p/6. 3. Quelle est l’énergie potentielle effective en fonction de r ? Représenter avec soin son graphe. Trouver la valeur de r correspondant à un équilibre stable. 4. Calculer les valeurs minimale et maximale de r correspondant à une énergie mécanique de 100 J. P12– 2. Périodes des planètes du système solaire Calculer, en année terrestre, les périodes de révolution des planètes : Mercure, Vénus, Mars, Jupiter, Saturne et Uranus. On utilisera les caractéristiques du système solaire rassemblées dans l’annexe 1. P12– 3. Vitesses d’évasion

 c Dunod – Toute reproduction non autorisée est un délit

Calculer les vitesses d’évasion à la surface de la Lune, Vénus, Mars et Jupiter. Les rayons et les masses de ces astres, comparés à ceux de la Terre, sont donnés dans l’annexe 1. P12– 4. Satellite Arabsat, Spot et satellite d’observation à haute résolution On étudie le mouvement de divers satellites artificiels de la Terre par rapport au référentiel géocentrique Rg = Tx0y 0z 0. On ne prend en compte que l’interaction entre le satellite et la Terre et on suppose que cette dernière a la symétrie sphérique. 1. Un satellite artificiel S, de masse m, décrit autour de la Terre une orbite circulaire de rayon r. a) Expliquer pourquoi le mouvement est uniforme. b) Déterminer, en fonction de r, la vitesse du satellite ainsi que sa période de révolution. c) Trouver, en fonction de r, l’énergie cinétique Ek , l’énergie potentielle Ep et l’énergie mécanique Em du satellite ; on adoptera comme origine de Ep sa valeur lorsque r est infini. Tracer les graphes Ek (r), Ep (r) et Em (r). d) Quelle est l’énergie potentielle de S au sol ? Quelle est son énergie cinétique lorsqu’il repose sans vitesse en un point du sol de latitude l ?

214

12. Corps ponctuel soumis à une force centrale conservative

2. Le 8 février 1985, le lanceur européen Ariane III a mis sur une orbite géostationnaire le satellite Arabsat F1 de masse m. Ce satellite est fixe par rapport à un référentiel terrestre d’origine T. a) Montrer qu’un tel satellite doit être contenu dans le plan équatorial terrestre et trouver la valeur rs du rayon de l’orbite. En déduire sa vitesse. b) Donner, en fonction de r et l, l’expression de l’énergie à fournir pour placer ce satellite sur une telle orbite. 3. Le 22 février 1986, Ariane III plaçait SPOT (Satellite sPécialisé dans l’Observation de la Terre) sur orbite circulaire à une altitude de 832 km. a) Calculer la quantité (GM T /RT ) 1/2. En déduire la vitesse orbitale et la période de SPOT. b) L’image d’un carré, donnée par un système optique embarqué sur SPOT, dont chaque côté est vu sur la Terre sous un angle de 4 ◦, est constituée de 9 × 10 6 taches élémentaires appelées pixels (cf. Optique). Quelle est la distance sur la Terre qui correspond à l’écart entre deux pixels ? 4. Un satellite d’observation a une orbite circulaire très basse, ce qui permet de discerner les détails d’environ 1 m sur la Terre. a) Calculer la vitesse orbitale et la période d’un tel satellite pour la valeur h = 180 km. b) Par suite des collisions avec les molécules d’air des couches supérieures de l’atmosphère, le satellite est soumis à une force de frottement opposée à la vitesse v et de norme bv 2 /h, h étant l’altitude et b/m une constante égale à 10−8 SI. Comment varie l’énergie mécanique du satellite ? Montrer que la vitesse du satellite, ainsi freiné, augmente. c) En déduire, en fonction de h, l’expression approchée de la variation d’altitude Dh du satellite, après une révolution. Calculer Dh. P12– 5. Orbites de transfert d’Hohmann On réalise le transfert d’un satellite d’une orbite circulaire basse vers une orbite géostationnaire, en empruntant une orbite elliptique ayant son périgée Sp sur l’orbite basse et son apogée S a sur l’orbite géostationnaire. Pour cela, on communique une variation brusque de vitesse au satellite en éjectant du combustible pendant une durée très courte. 1. Dans quel plan doit être située l’orbite basse pour que les trois orbites soient coplanaires ? 2. Trouver l’énergie, par unité de masse, du satellite sur l’orbite de transfert ainsi que la durée du séjour sur cette orbite. P12– 6. Excentricité d’une trajectoire de satellite Un satellite artificiel de la Terre a été lancé d’un point A 0 , avec une vitesse v 0 perpendiculaire au rayon vecteur r0. Quelle est l’excentricité e de la trajectoire en fonction de v 0 et de la vitesse de satellisation vs ? Calculer e pour v0 = v s, v0 = 0, 5v s et v0 = 2vs . P12– 7. Mouvement hyperbolique d’un satellite artificiel Un satellite artificiel S, de masse m = 2 000 kg, est placé sur une orbite circulaire d’attente, de rayon r0 = R + h autour de la Terre (h = 180 km). En un point de sa trajectoire, on lui communique un excédent de vitesse. La nouvelle vitesse v1 est tangente à l’orbite circulaire et vaut 14 km . s −1. 1. Exprimer, en fonction de r 0, la valeur v0 de la vitesse lorsque le satellite est sur son orbite d’attente. Calculer numériquement v 0 ainsi que son énergie mécanique. On prendra comme origine de l’énergie potentielle celle correspondant à S infiniment éloigné de la Terre et pour le rayon de la Terre RT = 6 400 km.

Corps ponctuel soumis à une force centrale conservative

215

2. a) Montrer que la nouvelle trajectoire est contenue dans un plan que l’on déterminera et calculer la nouvelle valeur de l’énergie. b) Établir que l’équation de la trajectoire s’écrit dans ce plan r = p/(1 + e cos w), e et p étant deux constantes dont on donnera la signification, r la coordonnée radiale et w l’angle que fait le rayon vecteur avec le rayon vecteur initial. c) Exprimer p en fonction du moment cinétique et calculer sa valeur. En déduire e. P12– 8. Trajectoire de Télécom 2A Le satellite Télécom 2A a été placé, en décembre 1991, sur une trajectoire géostationnaire. Pour cela, on a utilisé une trajectoire de transfert entre une orbite circulaire basse, à une altitude de 300 km et une orbite circulaire haute à une altitude de 35 980 km. La trajectoire de transfert est une demi-ellipse dont l’un des foyers est occupé par la Terre ; son périgée Sp est situé sur l’orbite basse et son apogée Sa sur l’orbite géostationnaire. On a réalisé ce transfert en communiquant des variations brutales de vitesse en Sp et en S a par éjection convenable de masse. 1. Calculer les vitesses de satellisation v p et va , respectivement pour hp = 300 km et h a = 35 980 km. 2. Dans quel plan particulier est située la trajectoire géostationnaire ? Dans quel plan doit être contenue l’orbite de transfert ? Représenter les trois orbites sur un schéma. 3. Transfert d’Hohmann a) Quelle est, en fonction de v p, rp et r a, la variation de vitesse Dvp du satellite au périgée Sp , entre l’orbite initiale et l’orbite de transfert ? Application numérique. On prendra R T = 6 400 km pour rayon terrestre. b) Même question en S a, entre l’orbite de transfert et l’orbite finale ? Application numérique. P12– 9. Satellite COBE (COsmic Background Explorer) Le satellite COBE, lancé en novembre 1989 pour étudier le rayonnement millimétrique émis par le fond du ciel, a été placé à une altitude de 900 km. Calculer, en heure et minute, sa période de révolution autour de la Terre ?

 c Dunod – Toute reproduction non autorisée est un délit

P12– 10. Masse volumique de Jupiter Montrer qu’en comparant la période de révolution d’un satellite de Jupiter à celle de la Lune autour de la Terre, on peut déduire le rapport des masses volumiques de ces deux planètes, si l’on connaît le rayon des orbites supposées circulaires des satellites. Application : La période de révolution de Ganymède, situé à une distance du centre de Jupiter égale à 15 fois le rayon de Jupiter, est 7 j 3 h 40 min ; la période de révolution de la Lune, située à une distance égale à 60 fois le rayon de la Terre, est 27 j 7 h 40 min. En déduire la masse volumique de Jupiter sachant que celle de la Terre est 5, 52 g . cm−3 . P12– 11. Durée de passage d’une comète dans l’environnement terrestre Une comète du Soleil a une trajectoire parabolique dans le plan de l’écliptique ; on rappelle que le centre de la Terre a, dans ce plan, un mouvement pratiquement circulaire autour du Soleil. 1. Écrire, en coordonnées polaires (r, w), les deux équations relatives à la comète et à la Terre. On prendra l’origine de l’angle w au périhélie de la comète.

216

12. Corps ponctuel soumis à une force centrale conservative

2. Montrer que les trajectoires de la Terre et de la comète se coupent pour les valeurs w 0 et −w0 que l’on exprimera en fonction du rapport des paramètres des deux trajectoires. 3. En intégrant l’équation de conservation du moment cinétique relative à la comète, trouver, en fonction de w0 , l’expression de la durée de passage de la comète à l’intérieur de la trajectoire terrestre. P12– 12. Mise sur orbite elliptique d’un satellite Un satellite artificiel de la Terre S, de masse m, est placé sur une orbite elliptique définie par les conditions initiales suivantes : le vecteur position est r 0 = TS et le vecteur vitesse v0, de valeur égale à la vitesse de satellisation relative à r0 , fait l’angle a0 avec la normale à r 0 . On repère la position de S par les coordonnées polaires (r, w), w étant compté à partir de la position du périgée P. 1. Rappeler l’équation du mouvement, en coordonnées polaires, en précisant l’expression du paramètre p de la conique en fonction du carré L2 du moment cinétique. De même, exprimer l’excentricité e en fonction de p et de l’énergie mécanique E m.

2. Établir, en fonction de r 0 et a0, les expressions de p, e, r min, r max et du demi-grand axe a de l’ellipse. P12– 13. Comète Hale-Bopp La comète Hale-Bopp a, dans le référentiel de Copernic, une trajectoire conique, autour du centre S du Soleil, d’excentricité e = 0, 995 et de période de révolution T = 2 400 années. Pour analyser simplement un tel mouvement, on suppose que la comète est assimilée à un point matériel H, de masse mHB , soumis à la seule action du Soleil que l’on réduit à son centre S, de masse MS. 1. Quelle est l’expression vectorielle de la force qu’exerce le Soleil sur la comète en fonction du vecteur r = SH ? En déduire que le mouvement de H autour de S est plan. 2. Établir l’expression de l’énergie potentielle de la comète dans le champ gravitationnel du Soleil en fonction de r. On prendra comme origine des énergies potentielles la valeur pour r infini. 3. L’équation de la trajectoire de la comète en coordonnées polaires (r, w) s’écrit : p r= 1 + e cos w a) Donner l’allure de la trajectoire en précisant la position de S, celle de l’aphélie A (point le plus éloigné de S) et celle du périhélie P (point le plus proche de S). b) Établir la relation entre p, e et le demi grand axe a de l’ellipse. 4. a) À l’aide de la troisième loi de Kepler, appliquée au mouvement de la comète et à celui de la Terre autour du Soleil, calculer le demi grand axe de l’ellipse. En déduire p. b) Calculer, la distance r max de S à l’aphélie et la distance rmin de S au périhélie. Quelle est la distance qui sépare la Terre de la comète au périhélie de sa trajectoire ? 5. a) Que peut-on dire de la composante radiale de la vitesse de H au périhélie et à l’aphélie ? Justifier. b) Calculer la vitesse minimale v min de la comète sachant que sa vitesse maximale a la valeur vmax = 200 000 km . h −1. 6. Quelle est l’énergie mécanique de la comète dans son mouvement autour du Soleil ? Calculer sa valeur en joule, sachant que m HB = 2 × 1012 kg. Commenter le signe de l’énergie.

13

 c Dunod – Toute reproduction non autorisée est un délit

Système de corps ponctuels en interaction. Problème à deux corps En principe, l’étude d’un système de N corps ponctuels en interaction se pose de la même façon que pour un seul corps. Cependant, dans le cas d’un seul corps ponctuel, l’analyse était simple car les forces ne faisaient apparaître que les variables du point considéré : la loi fondamentale fournissait alors trois équations scalaires comportant, lorsque les forces sont connues, trois inconnues qui sont les trois degrés de liberté du corps dans l’espace. En revanche, dans le cas d’un système de N corps, l’analyse est très complexe car, outre le grand nombre de corps, les équations du mouvement de chaque corps font apparaître les coordonnées de tous les autres. Le système d’équations à résoudre est alors inextricable. Aussi, pour N très grand, comme c’est le cas en thermodynamique où N est de l’ordre du nombre d’Avogadro NA ≈ 6, 02 × 10 23 mol−1 , utilise-t-on des méthodes statistiques (cf. Thermodynamique). Si le nombre de corps ponctuels n’est pas très grand, comme dans le cas du système Soleil-planètes, on s’appuie sur la dissymétrie du problème : en raison de sa très grosse masse, le Soleil a un rôle particulier ; on ramène alors le problème initial à N − 1 problèmes à deux corps « Soleil-planète » en considérant que l’influence des autres planètes ne constitue qu’une perturbation. On comprend dès lors tout l’intérêt que présente l’étude du problème à deux corps. En vue d’exprimer les lois auxquelles satisfait un système de N corps ponctuels, il est nécessaire en premier lieu de définir les éléments cinétiques qui permettent de décrire son mouvement.

I . — ÉLÉMENTS CINÉTIQUES D’UN SYSTÈME DE CORPUSCULES I . 1 . — Schématisation du système Considérons un système déformable S d de N corpuscules, en mouvement par rapport à un référentiel R , sous l’action de leurs forces d’interaction et des forces qu’exerce le milieu extérieur sur lui (Fig. 13.1a). Le nombre d’inconnues dont dépend a priori le système est 3N puisque chaque corpuscule a trois degrés de liberté. On caractérise à chaque instant l’état mécanique de S d par l’ensemble des N vecteurs positions ri et des N vecteurs vitesses v i ; par conséquent 6N variables doivent être données pour connaître l’état de S d. En fait, comme nous le verrons ultérieurement (cf. chapitre 24), on préfère caractériser cet état par l’ensemble des N vecteurs positions r i et des N vecteurs quantités de mouvement pi , ces dernières grandeurs étant des concepts plus riches que les vitesses.

218

13. Système de corps ponctuels en interaction. Problème à deux corps z

z A2 An C A1

O x

x

R∗

A1 (m ) y

C

Sd

A2 (m ) y

T mG (A2 ) mG(A 1)

R a)

b) F IG . 13.1.

I . 2 . — Quantité de mouvement du système Considérons, par rapport à R, l’ensemble des vecteurs quantités de mouvement p i = m ivi associés aux points {Ai } d’un système matériel Sd de N corps ponctuels. La quantité de mouvement du système est la somme vectorielle des quantités de mouvement (cf. section III) : P=

pi = i

m i vi i

Comme vi = (d OA i/ d t) R, P s’écrit aussi, en permutant les opérateurs sommation et dérivation : P=

d dt

mi OAi i

Introduisons le centre de masse C, c’est-à-dire le barycentre des corps ponctuels A i affectés des masses mi . Comme : 1 m iOA i où M = mi OC = M i i désigne la masse totale de S d, il vient : P=M

d OC = MvC dt

La quantité de mouvement d’un système matériel est celle de son centre de masse affecté de la masse totale. Remarque : Le centre de masse C est souvent appelé centre de gravité G, car, en raison de l’égalité des masses inerte et grave, et de l’hypothèse raisonnable d’un champ de gravitation uniforme sur l’étendue du système, ces deux points sont pratiquement confondus. En le désignant par C, on a voulu souligner que le barycentre des masses inertes est défini indépendamment de la force de gravitation. On peut préciser cette distinction en considérant deux points matériels A1 et A2, de même masse m, à la surface de la Terre (Fig. 13.1b). Si ces points sont suffisamment éloignés, le centre de masse C se trouve au milieu de la corde A1 A2 , alors que les deux forces de gravitation, supposées concourantes au centre T de la Terre, sont équivalentes à une seule force dont le support passe par T (cf. chapitre 1). Notons que si les deux forces n’étaient pas concourantes, comme c’est le cas en général, on ne pourrait pas les remplacer par une force unique. Ajoutons que, dans une cabine spatiale, le système A1A2 n’a pas de centre de gravité G , alors qu’il a toujours un centre de masse C .

219

Système de corps ponctuels en interaction. Problème à deux corps I . 3 . — Référentiel du centre de masse

On appelle référentiel du centre de masse R ∗ , associé à R, le référentiel en translation par rapport à R et tel que P∗ = 0. Comme P∗ = Mv∗C , on en déduit que v ∗C = 0. Par conséquent, le centre de masse est fixe dans R∗ ; aussi choisit-on généralement le centre de masse comme origine de R∗ . Cette définition est celle que l’on adopte en relativité où le centre de masse d’un ensemble de particules ne peut être défini géométriquement (cf. Relativité et invariance). Aussi l’avons-nous préférée à la suivante donnée en mécanique newtonienne : R∗ est le référentiel en translation par rapport à R et d’origine le centre de masse C. Évidemment, dans l’approximation non relativiste, les deux définitions coïncident. I . 4 . — Moment cinétique du système a) Définition et propriétés Le moment cinétique d’un système de N corps ponctuels en un point O, par rapport au référentiel R, est la somme vectorielle des moments cinétiques en O de chaque élément (cf. section III) : LO = i

OA i × mi vi

Explicitement, on écrira L O/R et v i/R. Si O  désigne un autre point, on a : LO = i

O Ai ×mi vi =

(O O+OA i )×mi vi = i

i

OA i ×mi vi +OO×

i

mi v i = L O+O  O × P

On retiendra donc : LO = LO + OO  × P

ou

Il en résulte que, dans R∗ où P∗ = 0, on a :

LO = L C + OC × P

L∗O = L∗O = L∗C

 c Dunod – Toute reproduction non autorisée est un délit

Ainsi, dans R∗ , le moment cinétique est indépendant du point où on le calcule. On pourra donc écrire, dans R∗ , sans autre précision : L∗ =

CAi × mi v∗i

i

b) Théorème de Kœnig relatif au moment cinétique Ce théorème, établi par le mécanicien allemand J. Kœnig, permet de relier le moment cinétique L O relatif à R au moment cinétique L∗ : LO = i

OAi × mi vi

et

L∗ = i

CAi × mi v ∗i

Comme OA i = OC + CA i et vi = v∗i + v C , d’après la composition des vitesses entre les deux référentiels R et R∗ en translation, il vient : LO = i

(OC + CAi ) × mi (v∗i + vC ) = OC ×

i

m iv∗i + OC ×

mi i

vC + L∗ + i

CA i × mi vC

220 Or

13. Système de corps ponctuels en interaction. Problème à deux corps i mi

∗ i m iv i

= M,

= 0 et

i m i CA i =

0. Par conséquent :

LO = OC × Mv C + L ∗ = OC × P + L ∗ Le moment cinétique d’un système matériel, par rapport à R, est la somme du moment cinétique de son centre de masse affecté de la masse totale et de son moment cinétique par rapport à R ∗. En comparant le théorème de Kœnig à la relation L O = LC + OC × P, on obtient : LO = LC + OC × P = OC × P + L ∗

d’où

LC = L∗

Le moment cinétique par rapport à R∗ est égal au moment cinétique en C par rapport à R. c) Exemples 1) Moment cinétique de la Terre par rapport au centre du Soleil Le moment cinétique de la Terre par rapport au centre du Soleil s’écrit, L S = ST × MT vT + L∗T , où le vecteur ST × MT vT , qui est porté par un axe perpendiculaire au plan de l’écliptique, a pour valeur : ST MT vT ≈ 6 × 1024 × 1, 49 × 1011 × 3 × 10 4 ≈ 2, 68 × 1040 J . s Quant au moment cinétique de la Terre par rapport à R ∗ = R g , il est porté par l’axe sud-nord, qui fait l’angle 23 ◦26 avec l’axe précédent et vaut (cf. chapitre 17) : (2/5)MT R2 V = 7, 18 × 10 31 J . s . On voit que le moment cinétique de la Terre est principalement dû au mouvement de son centre de masse. 2) Moment cinétique de l’électron dans l’atome d’hydrogène Dans l’atome d’hydrogène, l’électron ne peut pas être assimilé à un simple corps ponctuel, car son moment cinétique au point où se trouve le noyau est la somme de deux termes : le moment cinétique orbital OA × me vC , et un moment cinétique interne R∗ qui n’est pas nul, appelé moment cinétique de spin, du verbe anglais « to spin » qui signifie tourner (cf. Quantique). Ces deux contributions sont du même ordre de grandeur  ∼ 10−34 J . s. I . 5 . — Énergie cinétique du système a) Définition L’énergie cinétique d’un système est la somme des énergies cinétiques des N points matériels qui le constituent (cf. section III) : Ek =

i

1 mi v2i 2

b) Théorème de Kœnig relatif à l’énergie cinétique Ce théorème relie l’énergie cinétique d’un système matériel S d , par rapport à un référentiel quelconque R, à l’énergie cinétique de ce même système par rapport au référentiel du centre de masse R ∗ associé à R. Comme vi = v∗i + vC , d’après la composition des vitesses entre R et R∗ , il vient : Ek =

i

1 m i (v ∗i + vC )2 = 2

i

1 m i v∗i 2 + ( 2

i

m i v∗i ) · vC +

1 ( 2

m i )v 2C i

Système de corps ponctuels en interaction. Problème à deux corps soit, puisque

∗ i m iv i

221

=0 : Ek =

1 2 P2 MvC + Ek∗ = + E k∗ 2 2M

L’énergie cinétique d’un système matériel, dans R, est la somme de l’énergie cinétique de son centre de masse affecté de la masse totale et de son énergie cinétique dans R∗ . On appelle souvent le premier terme, Mv2C /2, l’énergie cinétique de translation. c) Exemples (1) Énergie cinétique de la Lune dans le référentiel géocentrique R g

Cette énergie cinétique est la somme de deux termes : E k = M Lv 2L/2 + Ek∗ . Comme le centre de masse de la Lune se déplace à une vitesse d’environ 1 km . s−1 , le premier vaut : 1 × 73, 5 × 1021 × 10 6 ≈ 36, 5 × 10 27 J 2 Quant au second, on peut montrer (cf. chapitre 17) qu’il a pour expression, si VL est le vecteur vitesse de rotation autour de son axe de rotation : 1 2 1 × MLR 2L V2L = × 73, 5 × 10 21 × (1, 76 × 106) 2 × (2, 66 × 10−3 )2 ≈ 3, 22 × 1029 J 2 5 5 (2) Énergie cinétique d’une molécule d’un gaz L’énergie cinétique d’une molécule d’un gaz est la somme de celle de son centre de masse et de celle dans le référentiel du centre de masse de la molécule. Dans ce cas, on montre que ces énergies sont du même ordre de grandeur (cf. Thermodynamique). À la température T = 300 K, on a, pour la molécule diatomique de gaz chlorhydrique HCl : 1 3 MvC2 = k BT ≈ 37, 5 meV et E∗k = kBT ≈ 25 meV 2 2 kB ≈ 1, 38 × 10−23 J . K−1 étant la constante de Boltzmann.

 c Dunod – Toute reproduction non autorisée est un délit

II . — PROBLÈME À DEUX CORPS Soient deux corps ponctuels A 1 et A2, de masses respectives m 1 et m2 , en mouvement dans le référentiel galiléen R , sous l’action de leur seule force d’interaction (Fig. 13.2). Le système est donc isolé. z v1 r2 O

A2

r

A1 r1 y

v2

x F IG . 13.2.

222

13. Système de corps ponctuels en interaction. Problème à deux corps

II . 1 . — Éléments cinétiques du système a) Quantité de mouvement Par rapport à R, la quantité de mouvement de ce système est : P = p 1 + p 2 = m1 v1 + m2 v2 = (m1 + m2 )vC = MvC Dans le référentiel du centre de masse R∗ , on a : P ∗ = p∗1 + p∗2 = (m1 + m2)v∗C = 0 Il en résulte que, dans R∗, les quantités de mouvement de A 1 et A2 sont opposées. Évaluons p ∗1 et p ∗2. D’après la composition des vitesses, on a : p∗1 = m 1 v∗1 = m1 (v1 − vC ) = m 1 v1 − d’où : p∗1 = −p ∗2 =

m 1v1 + m2v 2 m1 + m2

m1 m 2 (v1 − v2 ) m1 + m 2

Cette dernière relation s’écrit aussi : p∗ = p∗1 = −p∗2 = mv

en posant m =

m1 m2 m1 + m2

et v = v 1 − v2

Ainsi, dans R∗ , la norme de la quantité de mouvement de chacune des particules est égale à celle d’une particule fictive A, de masse m et de vitesse v. La masse m est appelée la masse réduite du système des deux corps. Remarquons qu’elle est inférieure ou égale à la plus petite des deux masses. b) Moment cinétique D’après le théorème de Kœnig, on a, pour le système des deux corps ponctuels : LO = OC × (p 1 + p2) + L ∗ avec : L∗ = CA 1 × p∗1 + CA 2 × p∗2 = (CA1 − CA 2) × p ∗1 = A2 A 1 × p ∗1 = r × mv

Ainsi, par rapport au référentiel du centre de masse R∗ , le moment cinétique du système des deux corps ponctuels est égal au moment cinétique de la particule fictive A dont la position dans R ∗ est définie par CA = r : L∗ = CA × mv = r × p∗ c) Énergie cinétique D’après le théorème de Kœnig, relatif à l’énergie cinétique, on a : Ek/R =

1 (m1 + m2)v C2 + Ek∗ 2

avec

E ∗k =

p∗2 p∗2 p∗2 1 + 2 = 2m1 2m2 2

1 1 + m 1 m2

=

p∗2 2m

Ainsi, l’énergie cinétique Ek∗ est celle d’une particule fictive A, de masse m et de vitesse v = v1 −v2 : E ∗k =

1 p∗2 = mv 2 2m 2

Système de corps ponctuels en interaction. Problème à deux corps

223

II . 2 . — Système isolé de deux corps ponctuels Pour un système isolé, les seules forces qui apparaissent sont les forces d’interaction mutuelle lesquelles sont opposées selon la troisième loi de Newton (cf. chapitre 4). La quantité de mouvement du système est alors constante puisque : dP d p 1 d p2 = F2→1 + F1→2 = 0 = + dt dt dt Il en résulte, dans ce cas : P = M vC = Cte Le mouvement du centre de masse est donc rectiligne et uniforme. Si, en outre, ces forces d’interaction dérivent d’une énergie potentielle, ce qui est généralement le cas, il vient : dW = dW1 + dW2 = F 2→1 · d r1 + F1→2 · d r 2 = − d Ep L’application du théorème de l’énergie cinétique au système donne donc : d’où :

d Ek,1 = F2→1 · d OA 1

et

d Ek,2 = F 1→2 · d OA2

d E k = d Ek,1 + d E k,2 = F2→1 · d OA 1 + F 1→2 · d OA 2 = − d Ep Il en résulte pour l’énergie mécanique Em = Ek + E p : d(Ek + E p) d Em = = 0 soit dt dt

Em = Ek + E p = Cte

II . 3 . — Équations du mouvement dans le référentiel du centre de masse

 c Dunod – Toute reproduction non autorisée est un délit

Le système des deux corps ponctuels étant isolé, le mouvement de translation de R ∗ , par rapport à R galiléen, est rectiligne et uniforme. Par conséquent, R ∗ est aussi galiléen. D’après la loi fondamentale appliquée à A1, il vient : d p ∗1 = F 2→1 dt

soit

m

dv = F 2→1 dt

Cette dernière équation peut être considérée comme la loi fondamentale de la dynamique appliquée à la particule A, de masse m, de vitesse v = v1 − v2 et soumise à la force F2→1 qui est centrale puisqu’elle passe par C situé sur le vecteur A2 A1 . On a en effet : m 1CA1 + m2CA2 = 0 et : CA 1 =

soit

CA1 CA2 A 2A 1 CA =− = = m2 m1 m 1 + m2 m1 + m2

m2 CA m1 + m2

et

CA2 =

−m1 CA m 1 + m2

Les trajectoires de A1 et A 2 sont donc homothétiques de la trajectoire de A ; le centre d’homothétie est C et ses rapports sont m 2/(m 1 + m 2) et −m 1 /(m1 + m 2) respectivement. Notons que, si l’une des particules a une masse beaucoup plus faible que l’autre, elle peut être assimilée à la particule fictive A, la position de la seconde coïncidant avec le centre de masse C.

224

13. Système de corps ponctuels en interaction. Problème à deux corps

II . 4 . — Conséquences de la conservation du moment cinétique La force d’interaction qui s’exerce sur la particule fictive A étant centrale, car passant par le centre de masse C, le mouvement est plan (cf. chapitre 12). Si on explicite la conservation du moment cinétique dans R∗ en coordonnées polaires (r, w) dans le plan du mouvement Cxy (Fig. 13.3), on obtient : L∗ = L ∗z ez = r × mvA = rer × m(˙r er + rw˙ ew ) = mr 2w˙ ez = Cte Retenons donc : L∗z = mr 2 w˙ = mC = 2m

a

en introduisant la constante des aires C et la vitesse aréolaire

= Cte a

(cf. chapitre 12) :

z L∗

R∗ x

y

C

A2

w B

dw

A1 B A

A

F IG . 13.3.

II . 5 . — Conséquences de la conservation de l’énergie D’après la conservation de l’énergie mécanique dans R ∗, on a : 1 Em∗ = Ek∗ + E p = m( ˙r 2 + r 2 w˙ 2) + Ep = Cte 2 Cette équation de conservation, combinée à la précédente, fournit l’équation du mouvement radial. En effet, introduisant le carré du moment cinétique L 2 = m 2r 4 w˙ 2, on obtient : 1 2 L2 m˙r + + E p (r) = E ∗m 2 2mr2 soit : 1 2 L2 m˙r + E p,ef = E ∗m avec E p,ef = Ep (r) + 2 2mr2 Ainsi, tout se passe comme si la particule A était soumise à une énergie potentielle effective E p,ef dans le mouvement unidimensionnel radial. II . 6 . — Analyse qualitative du problème à deux corps On considère généralement deux types particuliers d’énergie potentielle effective : le puits et la barrière d’énergie potentielle.

225

Système de corps ponctuels en interaction. Problème à deux corps a) Puits d’énergie potentielle

Un cas fréquent est celui où l’énergie potentielle effective passe par un minimum puis tend vers 0 lorsque r devient très grand (Fig. 13.4). Il correspond par exemple à l’interaction gravitationnelle si le moment cinétique n’est pas nul ou à l’interaction entre deux atomes dans la formation d’une molécule. Comme m r˙2 /2 = E ∗m − Ep,ef  0, on doit distinguer deux types de mouvement suivant les valeurs de l’énergie mécanique Em∗ : i) Si E m∗  0, l’état est libre car r  r min peut prendre une valeur infinie.

ii) Si E m∗ < 0, l’état est lié car r oscille entre deux valeurs extrêmes appelées distances apsidales. Il faut noter qu’en dehors de cas simples, tels que le problème de Kepler où l’on sait que la trajectoire est une ellipse, l’oscillation de r n’implique pas que la trajectoire soit fermée. Lorsque Em∗ est égal à la valeur minimale E0 de l’énergie potentielle, la trajectoire est circulaire pour L∗ = 0, puisqu’alors r = r0 et w˙ = 0 ; L ∗ = 0 correspond à une position d’équilibre (dans R ∗). Un développement de Taylor de l’énergie potentielle autour de E0 montre que l’équilibre est stable puisque les mouvements dans le voisinage de r = r 0 sont sinusoïdaux (cf. chapitre 5). Ep,ef ∗ Em

0

∗ Em E0

r0 r

∗ Em

∗ Em

∗ Em

0

E0

∗ Em

F IG . 13.4.

Ep,ef

r0 r max

rmin

r

F IG . 13.5.

b) Barrière d’énergie potentielle

 c Dunod – Toute reproduction non autorisée est un délit

Si l’énergie potentielle effective passe par un maximum puis tend vers 0 lorsque r devient très grand, plusieurs cas se présentent (Fig. 13.5) : i) E ∗m  0 : r oscille entre 0 et rmax .

ii) 0 < Em∗ < E 0 : là, deux solutions exclusives peuvent convenir, 0  r  rmax et rmin  r < ∞. À la première correspond un état lié, à la seconde un état libre. iii) Em∗  E 0 : la coordonnée radiale r peut prendre toutes les valeurs comprises entre 0 et ∞. L’état est libre. L’égalité Em∗ = E0 correspond à une position d’équilibre instable car un développement de Taylor de l’énergie potentielle autour de E0 montrerait que tout écart à la position d’équilibre augmente (cf. chapitre 5). Remarque : En quantique, les deux solutions du cas ii) ne sont pas exclusives. L’objet physique ponctuel, d’énergie déterminée comprise entre 0 et E0 , peut alors « passer » d’un état lié à un état libre de même énergie. C’est l’effet tunnel dont le nom imagé traduit le franchissement d’une « montagne » d’énergie potentielle (cf. Quantique). Ainsi, dans les deux cas, puits et barrière d’énergie potentielle, peuvent exister des états libres et des états liés.

226

13. Système de corps ponctuels en interaction. Problème à deux corps

II . 7 . — Retour sur le problème de Kepler On simplifie considérablement l’analyse du mouvement de N corps en interaction, le Soleil et les N − 1 planètes, en le ramenant à celui de N − 1 problèmes indépendants à deux corps dont l’un est le Soleil ; l’influence des autres planètes se traite alors comme une simple perturbation. Rappelons l’expression de la force attractive de gravitation exercée par A 2 sur A1 : F1 =

K er r2

avec

er =

r A 2A1 = r r

la constante d’interaction étant négative en gravitation, K= −Gm1m 2, et positive ou négative en électrostatique, K = q1q2/(4pε 0). a) Interprétation des lois de Kepler On interprète aisément les trois lois de Kepler en assimilant, en première approximation, une planète à un corps ponctuel, de masse Mp, soumis à la force de gravitation solaire dans le référentiel de Copernic. i) Le centre de la planète décrit une ellipse dont l’un des foyers est occupé par le centre de masse. En raison du faible rapport M p  MS des masses planétaire et solaire (même Jupiter n’a qu’une masse égale à 0, 001 de la masse du Soleil), la masse réduite est, avec une excellente approximation, égale à la masse de la planète : Mp MS Mp ≈ Mp m= = 1 + Mp /MS M p + MS Le centre du Soleil coïncide pratiquement avec le centre de masse et le centre de la planète avec le point fictif A : −Mp MS CA 1 = CA ≈ CA et CA 2 = CA ≈ 0 M p + MS Mp + MS ii) La deuxième loi n’est que l’expression de la conservation du moment cinétique : mr 2w˙ = Cte

soit

r 2w˙ = Cte

iii) Quant à la troisième loi, elle découle directement de l’expression de la période (cf. chapitre 12) : 4p2m 4p2 T2 = = a3 |K | (M p + MS )G

soit

4p2 T2 ≈ a3 GMS

b) Vitesse d’évasion et vitesse de satellisation d’une planète Pour obtenir l’expression rigoureuse de la vitesse de libération ou d’évasion, il suffit de remplacer la masse Mp de la planète par la masse réduite m = M p MS /(Mp + M S) dans l’expression de l’énergie mécanique de la particule A (cf. chapitre 12). On trouve : 1 |K | =0 Em = mv 2l − 2 r

d’où

2G(MS + Mp ) vl = r

1/2



2GM S r

Calculons la vitesse d’évasion de la Terre sur son orbite : r = 1, 49 × 1011 m

Ms = 2 × 1030 kg

d’où

v l ≈ 42 km . s −1

1/2

227

Système de corps ponctuels en interaction. Problème à deux corps

De même, la vitesse de satellisation s’obtient en appliquant la loi fondamentale de la dynamique à la particule de masse m. On trouve aisément (cf. chapitre 12) : G(MS + Mp) vs = r

1/2

1/2

GMS r



La vitesse de satellisation de la Terre sur son orbite est donc : v s ≈ v l/1, 414 ≈ 30 km . s −1.

III . — DYNAMIQUE DES SYSTÈMES DE CORPS PONCTUELS III . 1 . — Forces extérieures et forces intérieures à un système Considérons un système déformable S d de N corps ponctuels, en mouvement par rapport à un référentiel R galiléen (Fig. 13.1). L’un de ces points A i est soumis à la force Fi exercée par l’ensemble des autres corps de Sd et par des corps extérieurs à Sd . Si le référentiel considéré n’est pas galiléen, on doit ajouter les forces d’inertie d’entraînement et de Coriolis (cf. chapitre 7). La force F i qui agit sur le corps ponctuel A i est la somme de deux contributions : i) l’une F ex→i due à tout corps étranger à S d ; c’est une force extérieure ; ii) l’autre

j

Fj→i due à tout corps de Sd distinct de Ai ; c’est une force intérieure.

III . 2 . — Théorème de la quantité de mouvement Appliquons, par rapport à un référentiel galiléen, la loi fondamentale à chaque corps ponctuel A i : d pi = F ex→i + Fj→i dt j=i

En sommant sur tous les points i de Sd , il vient :

 c Dunod – Toute reproduction non autorisée est un délit

où :

i

d pi = dt P=

pi

Fex→i + i

F j→i soit i

Sex =

i

j=i

Fex→i

dP = Sex + Sin dt

et Sin =

i

F j→i i

j=i

désignent respectivement la quantité de mouvement totale du système, la somme des forces extérieures et la somme des forces intérieures. Notons que la quantité de mouvement d’un système est une grandeur vectorielle additive : elle est égale à la somme des quantités de mouvement de ses parties, même en présence d’interaction. a) Propriété fondamentale de la somme des forces intérieures En raison de la troisième loi de Newton (cf. chapitre 4), on a : F i→j + Fj→i = 0

d’où

S in =

F j→i = 0 i

j=i

228

13. Système de corps ponctuels en interaction. Problème à deux corps

b) Énoncé du théorème La somme des forces intérieures étant nulle, la quantité de mouvement d’un système matériel, soumis aussi à des forces extérieures, satisfait au théorème de la quantité de mouvement : dP = S ex dt Par rapport à un référentiel galiléen, la dérivée par rapport au temps de la quantité de mouvement est égale à la somme des forces extérieures. Ce théorème, de portée générale, est connu sous d’autres noms, car la quantité de mouvement est aussi appelée moment linéaire en mécanique analytique, impulsion et même somme cinétique dans certains exposés formels. On le désigne souvent par théorème du centre de masse, en mécanique de Newton, car l’équation vectorielle fait apparaître explicitement le centre de masse C . En effet, puisque P = Mv C : MaC = S ex Précisons que, si les composantes du vecteur P sont relatives à une base non galiléenne, il sera nécessaire de procéder à une composition des dérivations de Bour (cf. chapitre 3). Ajoutons que la variation d P de la quantité de mouvement, entre les instants t et t +d t , concerne les mêmes corps appartenant à un système fermé, c’est-à-dire qui n’échange pas de matière avec le milieu extérieur (cf. chapitre 22). Remarque : Le centre de masse C n’est pas nécessairement matériel ; par exemple, celui associé à deux points A 1 et A2 , de même masse, se trouve à chaque instant au milieu du segment qui les joint. Son mouvement est celui d’un point géométrique, doté de la masse totale et soumis à une force égale à la somme des forces. c) Caractère conservatif de la quantité de mouvement Le théorème de la quantité de mouvement d’un système matériel fermé peut être exprimé en termes de bilan de la quantité de mouvement, comme on le fait couramment en thermodynamique pour des grandeurs extensives, c’est-à-dire des grandeurs qui dépendent de la taille des systèmes (cf. Thermodynamique). La variation d P de la quantité de mouvement du système, entre les instants voisins t et t + d t , est a priori la somme de deux contributions : la première, celle liée à l’échange avec le milieu extérieur, ou quantité de mouvement élémentaire reçue (algébriquement) dP(r) , la seconde liée à une éventuelle création ou production dP (c) . On a donc, entre deux instants infiniment voisins t et t + d t : d P = dP (r) + dP(c)

avec

dP r = Sex d t

et

dP(c) = 0

car la comparaison avec le théorème de la quantité de mouvement permet d’identifier l’échange à S ex d t ; le terme de création de la quantité de mouvement est toujours nul : on dit que la quantité de mouvement est une grandeur conservative. d) Condition de conservation de la quantité de mouvement En intégrant l’équation-bilan précédente entre deux instants quelconques t i et t f , on obtient : tf

DP = P (r) = ti

S ex d t

229

Système de corps ponctuels en interaction. Problème à deux corps

La quantité de mouvement P d’un système matériel se conserve si la somme des forces extérieures est nulle, c’est-à-dire : i) si le système est isolé (pas de forces extérieures), ii) ou si le système est pseudo-isolé (forces extérieures dont la somme vectorielle est nulle). e) Exemples (1) Problème à deux corps Dans le problème à deux corps, les deux éléments, qui forment un système isolé, ont des quantités de mouvement opposées dans le référentiel du centre de masse R∗ . Dans le référentiel du laboratoire R , la somme des quantités de mouvement est un vecteur constant. (2) Collisions

L’étude des collisions de particules, élastiques et inélastiques suivant que le nombre et la nature des particules sont inchangés ou non, fournit un autre exemple important de système pouvant être considéré comme isolé (cf. chapitre 14). III . 3 . — Théorème du moment cinétique a) Propriété fondamentale du moment des forces intérieures Appliquons, par rapport à un référentiel galiléen, en un point fixe O , le théorème du moment cinétique à chaque corps ponctuel Ai : d LO,i = OAi × Fex→i + OAi × dt

Fj→i j=i

En sommant sur tous les points i , il vient : d LO,i = dt

i

i

OAi × F ex→i +

i

OAi ×

F j→i

soit

j=i

d LO = MO,ex + MO,in dt

 c Dunod – Toute reproduction non autorisée est un délit

où : LO =

LO,i

MO,ex =

i

i

OAi × Fex→i

et M O,in = i

OA i ×

Fj→i j=i

désignent respectivement le moment cinétique total du système, la somme des moments des forces extérieures et la somme des moments des forces intérieures. Notons que, comme la quantité de mouvement, le moment cinétique est une grandeur vectorielle additive. En raison de l’opposition des actions réciproques (troisième loi de Newton), il vient : MO,in = ij

OAi × F j→i =

couple(ij)

OA i × F j→i + OAj × Fi→j =

(OAi − OAj ) × F j→i

couple(ij)

puisque Fi→j = −Fj→i . Comme la force d’interaction est portée par le vecteur A jA i = OAi − OAj , les différents produits vectoriels de la somme sont nuls. Il en résulte que : M O,in = 0

230

13. Système de corps ponctuels en interaction. Problème à deux corps

b) Énoncé du théorème Le moment des forces intérieures étant nul, le mouvement d’un système matériel, soumis à des forces extérieures, satisfait au théorème du moment cinétique : d LO = MO,ex dt Par rapport à un référentiel galiléen, la dérivée par rapport au temps du moment cinétique, en un point fixe O , est égale au moment en ce point des forces extérieures. Comme pour la quantité de mouvement, si le référentiel n’est pas galiléen, il faut ajouter les moments des forces d’inertie d’entraînement et de Coriolis : MO,ie = i

OAi × (−m iae,i)

et M O,iC = i

OAi × (−mi a C,i)

En outre, il faudra dériver le vecteur LO par rapport au temps, relativement au référentiel considéré. Si les composantes de ce vecteur sont relatives à une autre base, on utilisera la composition des dérivations de Bour (cf. chapitre 3). Enfin, comme le théorème de la quantité de mouvement, le théorème du moment cinétique suppose que les systèmes considérés soient fermés (cf. chapitre 22). c) Caractère conservatif du moment cinétique Comme le théorème de la quantité de mouvement, le théorème du moment cinétique d’un système matériel fermé peut être récrit en termes de bilan de moment cinétique. La variation d L O du moment cinétique du système, entre les instants voisins t et t +d t , est a priori la somme de deux contributions : (r) la première liée à l’échange avec le milieu extérieur, ou moment cinétique reçu (algébriquement) dL O , (c) la seconde liée à une éventuelle création dL O : d L O = dL (Or) + dL(Oc) avec

dL (Or) =

M0,ex d t

et

dL(Oc) = 0

car la comparaison avec le théorème du moment cinétique permet d’identifier l’échange à M O,ex d t et d’affirmer que le moment cinétique est une grandeur conservative. d) Condition de conservation du moment cinétique Si l’on intègre l’équation-bilan précédente du moment cinétique entre deux instants quelconques ti et tf , on obtient : DLO = L(Or) =

tf

ti

M O,ex d t

Le moment cinétique LO d’un système matériel se conserve si la somme des moments des forces extérieures est nulle, c’est-à-dire : i) si le système est isolé (pas de moments de forces extérieures), ii) si le système est pseudo-isolé (somme vectorielle des moments des forces extérieures nulle). e) Exemples (1) Problème à deux corps Le système formé par l’ensemble des deux corps ponctuels est isolé. Par rapport au référentiel du centre de masse, qui est galiléen, son moment cinétique est donc une constante vectorielle (Fig. 13.6a).

231

Système de corps ponctuels en interaction. Problème à deux corps

..

..

A

y

A

A1

Ωf > Ωi

w

C

A

.

Ωi

ri Ai

...

A2

A1

x Plan du mouvement

LC

2

a)

b) F IG . 13.6.

(2) Effondrement gravitationnel d’une étoile On peut assimiler une étoile à une distribution sphérique de matière formant un système isolé en rotation autour d’un axe ; par exemple le Soleil est une distribution de masse sphérique qui effectue une rotation complète autour de l’un de ses axes en 25 jours environ. Au fur et à mesure que l’étoile se contracte du fait de l’attraction gravitationnelle, la conservation du moment cinétique en son centre O provoque une augmentation de la vitesse de rotation V autour d’un axe (Fig. 13.6b). En effet : LO = i

OA i × m iv i =

i

ri er,i × mi (˙ri er,i + r iV ew,i ) =

mi r2i V e z i

2 r i étant la distance du point A i à l’axe de rotation. Comme la quantité i mi ri diminue au cours de l’effondrement, la rotation augmente. L’effondrement radial peut provoquer une très forte augmentation de la vitesse de rotation. C’est ce qui a été observé sur les pulsars, objets célestes caractérisés par un rayonnement électromagnétique, intense et fortement pulsé.

IV . — ÉNERGÉTIQUE DES SYSTÈMES DE N CORPS PONCTUELS

 c Dunod – Toute reproduction non autorisée est un délit

IV . 1 . — Théorème de l’énergie cinétique Appliquons le théorème de l’énergie cinétique à chaque corpuscule A i : d E k,i = Pex→i + dt

Pj→i

j=i

P ex→i étant la puissance des forces extérieures au système qui s’exercent sur le Ai et puissance des forces intérieures au système qui s’exercent sur ce corps.

j=i P j→i

la

En sommant sur tous les corps i , on obtient :

i

d Ek,i d i Ek,i d Ek = = = dt dt dt

i

Pex→i +

i

j=i

P j→i



Ek =

i

Ek,i

est l’énergie cinétique du système. Ainsi définie, l’énergie cinétique d’un système est une grandeur additive.

232

13. Système de corps ponctuels en interaction. Problème à deux corps

Désignons par Pex et Pin la puissance totale des forces extérieures et la puissance totale des forces intérieures, respectivement : Ek =

i

Ek,i

P ex =

i

P ex→i et P in =

i

j=i

P j→i

Cette puissance des forces intérieures s’explicite selon : Pin =

i,j

Fj→i · vi =

couple(ij)

Fj→i · vi + F i→j · v j =

couple(ij)

Fj→i · (v i − v j )

en tenant compte de l’opposition des actions réciproques. On en déduit le travail élémentaires des forces intérieures : dWin = P in d t = Fj→i · d(ri − rj ) = Fj→i · d rij couple(ij)

couple(ij)

en posant ri − r j = r ij . Or la force Fj→i est portée par le vecteur r ij . Par conséquent : Fj→i

dWin = couple(ij)

r ij · d rij r ij

soit

Fj→i d rij

dWin = couple(ij)

puisque rij d rij = r ij d r ij . Ainsi, le travail élémentaire et donc la puissance des forces intérieures sont en général non nuls à moins que les distances entre les différents points matériels demeurent constantes. a) Énoncé du théorème On déduit de ce qui précède : d Ek = Pex + P in dt Par rapport à un référentiel galiléen, la dérivée par rapport au temps de l’énergie cinétique d’un système est égale à la somme des puissances de toutes les forces, extérieures et intérieures. En introduisant les travaux élémentaires des forces extérieures et intérieures, dW ex = Pex d t et dWin = P in d t , l’équation précédente s’écrit aussi : d Ek = dW ex + dWin ce qui donne, en intégrant entre deux dates quelconques : DEk = W ex + W in La variation d’énergie cinétique est égale à la somme des travaux des forces extérieures et des forces intérieures. Si le référentiel n’est pas galiléen, il faut ajouter la contribution des forces d’inertie d’entraînement, −m i ae,i · vi , celle de la force de Coriolis étant nulle. En outre, ce théorème suppose que les systèmes i considérés sont fermés.

Système de corps ponctuels en interaction. Problème à deux corps

233

b) Caractère non conservatif de l’énergie cinétique Comme les deux précédents théorèmes de dynamique, le théorème de l’énergie cinétique d’un système matériel fermé peut être interprété en termes de bilan d’énergie cinétique. La variation d E k de l’énergie cinétique du système, entre les instants voisins t et t+d t , est la somme de deux contributions : (r) la première liée à l’échange avec le milieu extérieur, ou énergie cinétique reçue (algébriquement) dE k , (c) la seconde liée à la création ou production dEk : (r) (c) d Ek = dE k + dE k

La comparaison avec le théorème de l’énergie cinétique permet d’identifier l’échange à l’influence des forces extérieures par dWex , et la création à celle des forces intérieures par dW in . On peut ainsi affirmer que l’énergie cinétique n’est pas une grandeur conservative : dE (kc) = dW in (r) (c) A priori, dW in = 0 . En intégrant entre deux dates quelconques t i et t f , on obtient : DEk = Ek +Ek .

c) Cas d’un système isolé Dans le cas où le système est isolé, le théorème précédent se réduit, puisque dW ex = 0 , à : d E k = dWin

soit

DEk = Win

en intégrant. Ainsi, contrairement à la quantité de mouvement et au moment cinétique, l’énergie cinétique d’un système isolé ne se conserve pas. Elle ne se conserve que si le travail des forces intérieures est nul. IV . 2 . — Théorème de l’énergie mécanique

 c Dunod – Toute reproduction non autorisée est un délit

a) Énergie potentielle Certaines forces extérieures et intérieures sont telles que leurs travaux ne dépendent pas du chemin suivi par les points d’application. Ces travaux élémentaires peuvent alors s’écrire sous la forme de différentielles de fonctions appelées énergies potentielles (cf. chapitre 5) : dWex = − d Ep,ex et dWin = − d E p,in b) Énergie mécanique Par définition, on appelle énergie mécanique d’un système S d la somme de son énergie cinétique et de son énergie potentielle des forces extérieures et intérieures : Em = Ek + Ep

avec

Ep = Ep,ex + Ep,in

Notons que, contrairement à l’énergie cinétique, l’énergie mécanique n’est pas une grandeur additive, en raison de l’énergie potentielle d’interaction.

234

13. Système de corps ponctuels en interaction. Problème à deux corps

c) Théorème de l’énergie mécanique Distinguons, parmi les forces qui s’exercent sur un système, celles qui dérivent d’une énergie potentielle des autres. Les travaux élémentaires des forces extérieures et intérieures peuvent se mettre sous la forme : dW ex = − d Ep,ex + dW (exnc) et dWin = − d E p,in + dWin(nc) (nc)

(nc)

où dWex , dWin désignent les travaux élémentaires des forces qui ne dérivent pas d’une énergie potentielle. En remplaçant dans l’expression du théorème de l’énergie cinétique, il vient : d Ek d Ep,ex d E p,in =− + P (exnc) − + Pin(nc) soit dt dt dt On obtient ainsi le théorème de l’énergie mécanique : d Em = P(exnc) + P(innc) dt

ou

d (nc) (E k + Ep,ex + Ep,in) = P ex + P (innc) dt

d Em = dW (exnc) + dWin(nc)

en introduisant les travaux élémentaires correspondants. En intégrant entre deux dates quelconques, on trouve évidemment : DEm = W (exnc) + Win(nc) La variation d’énergie mécanique est égale à la somme des travaux des forces extérieures et intérieures qui ne dérivent pas d’une énergie potentielle. d) Caractère non conservatif de l’énergie mécanique Interprétons le théorème de l’énergie mécanique en terme de bilan. On a, comme pour l’énergie cinétique : d Em = dE (mr) + dE m(c) soit DEm = Em(r) + Em(c) (nc)

en intégrant. On identifie l’échange d’énergie mécanique au travail dW ex des forces extérieures non (nc) conservatives, et la création ou la production au travail dWin des forces intérieures non conservatives. Comme l’énergie cinétique, l’énergie mécanique n’est pas une grandeur conservative : dEm(c) = dWin(nc) C’est précisément la recherche d’une grandeur énergétique conservative qui a conduit à postuler le premier principe de la thermodynamique (cf. Thermodynamique). e) Cas d’un système isolé L’énergie mécanique d’un système isolé ne se conserve pas en général, puisque l’on a : DEm = W nc in = 0 f) Exemples i) Problème à deux corps Dans le problème à deux corps, l’énergie mécanique dans le référentiel du centre de masse se conserve, précisément parce que les forces d’interaction sont conservatives. ii) Système isolé avec forces intérieures de frottement Pour un système soumis à des forces intérieures de frottement, l’énergie mécanique diminue car le travail des forces de frottement est toujours négatif.

Système de corps ponctuels en interaction. Problème à deux corps

235

IV . 3 . — Théorème du viriel Ce théorème a été établi par le physicien allemand R. Clausius en 1870 (cf. Thermodynamique). Il fournit une expression de la valeur moyenne dans le temps de l’énergie cinétique d’un système déformable Sd de N particules. a) Définition du viriel Écrivons l’énergie cinétique de S d sous la forme suivante : Ek =

1 2

miv2i = i

soit : Ek = −

1 2 1 2

1 d ri = dt 2

i

pi ·

i

Fi · r i +

d(pi · r i ) d pi − ri · dt dt

i

1 d 2 dt

i

pi · r i

La valeur moyenne dans le temps de l’énergie cinétique, E k , fait apparaître deux termes : i) le premier appelé le viriel (de même origine que virilité qui signifie force) =−

1 2

i

Fi · ri

ii) le second 1 d 2 dt

p i · ri

i

Montrons que ce second terme est nul. Pour une durée T tendant vers l’infini, la moyenne de la fonction : dG pi · r i avec G = g= dt i a pour expression : g = lim  c Dunod – Toute reproduction non autorisée est un délit

T→∞

1 T

T 0

g d t = lim

T→∞

1 T

T 0

dG 1 d t = lim [G(T ) − G(0)] T→∞ T dt

Comme les valeurs de {r i} et {pi } sont finies, les premières parce que le système est localisé, les secondes parce que les vitesses sont inférieures à la vitesse de la lumière, la différence G(T ) − G(0) est bornée. Il en résulte : d p i · ri = 0 dt i b) Énoncé du théorème Finalement, on a : Ek =

avec

=−

1 2

i

Fi · ri

La valeur moyenne dans le temps de l’énergie cinétique d’un système de particules est égale au viriel de toutes les forces

236

13. Système de corps ponctuels en interaction. Problème à deux corps Si on distingue les forces extérieures au système des forces intérieures, on obtient évidemment : Ek =

ex

ex

+

in

avec

ex

=−

1 2

étant le viriel des forces extérieures et

Fex→i · r i

i in

et

in

=−

1 2

i

j=i

Fj→i · r i

le viriel des forces intérieures.

Remarque : Il existe une autre définition du viriel, largement répandue, dans laquelle ne figure pas le facteur (−1/2) . Nous avons préféré adopter celle proposée par Clausius lui-même. c) Exemples (1) Gaz parfait Dans le cas d’un gaz parfait, que l’on définit comme un ensemble de particules dont on peut négliger les interactions, en dehors des chocs (cf. Thermodynamique), le viriel se réduit à celui des forces extérieures, c’est-à-dire à celui des forces de pression : in

=0

ex

=−

1 2

i

Fex→i · ri

d’où

Ek =

ex

On montre que ex s’exprime simplement en fonction de la pression et du volume pour un gaz parfait (cf. Thermodynamique) : ex

=

3pV 2

d’où

pV =

2Ek 3

Pour un gaz réel, caractérisé par des forces d’interaction entre particules, il faut ajouter au viriel des forces extérieures un viriel non nul des forces intérieures. (2) Système de deux particules en interaction harmonique Pour deux particules en interaction harmonique, on a : F 2→1 = −Kr avec r = r 1 − r2 et F1→2 = Kr. D’après l’opposition des actions réciproques, F in = F2→1 + F 1→2 = 0 , il vient : Ek = −

F2→1 · r1 + F 1→2 · r2 F 2→1 · r Kr 2 =− = 2 2 2

La quantité Kr2/2 représente l’énergie potentielle associée à ces forces d’interaction, puisque : dW = F2→1 · d r 1 + F1→2 · d r 2 = F 2→1 · d r = −Kr · d r = −Kr d r = − d E p

avec

si on adopte la convention E p = 0 pour r = 0 . Il en résulte que : Ek = Ep (3) Système de deux particules en interaction newtonienne Dans ce cas, F 2→1 = (K /r 2 ) er avec K < 0 , r = r1 − r 2 et er = r/r . Donc : Ek = −

1 K K F2→1 · r =− 2r=− 2 2 r 2r

Ep =

Kr 2 2

237

Système de corps ponctuels en interaction. Problème à deux corps La quantité K /r représente l’énergie potentielle d’interaction puisque : dW = F2→1 · d r1 + F 1→2 · d r 2 = F 2→1 · d r =

K K er · d r = 2 d r = − d Ep 2 r r

avec E p =

K r

si on adopte la convention E p = 0 pour r infini. Il en résulte que : 2Ek + Ep = 0

d’où

Em = Ek + Ep =

1 Ep = −E k 2

Ce résultat se généralise au cas de N particules en interaction newtonienne.

V . — INTERACTION GRAVITATIONNELLE À N CORPS Le problème de Kepler à N corps, précisément à N corps ponctuels, a fait l’objet de très nombreuses publications depuis 1750, dont plusieurs signées par de très grands mathématiciens. V . 1 . — Équations du mouvement Pour un tel système déformable S d, supposé isolé, la quantité de mouvement P et le moment cinétique au centre de masse LC sont des constantes vectorielles ; en outre, comme les forces de gravitation sont conservatives, l’énergie mécanique Em est aussi une constante : P = Cte

LC = Cte

et

Em = E k + Ep = Cte

 c Dunod – Toute reproduction non autorisée est un délit

En explicitant, on obtient sept équations scalaires, trois par équation vectorielle et une par équation scalaire. Dans le cas de deux corps, le nombre de degrés de liberté est six ; aussi les équations précédentes permettent-elles de déterminer le mouvement, comme on l’a vu précédemment. En revanche, si le nombre de points est supérieur à deux, il n’y a pas de solution générale. On se ramène, lorsqu’on le peut, à un problème à deux corps perturbé par un troisième ; c’est ce que fit Le Verrier avec le mouvement d’Uranus perturbé par Neptune. Cependant, le problème à trois corps ponctuels, qui a neuf degrés de liberté, ne peut être résolu que s’il existe des équations supplémentaires traduisant des conditions particulières. C’est le cas si les trois corps sont alignés ou s’ils forment un triangle équilatéral. C’est ce dernier cas qui est le plus intéressant. V . 2 . — Points de Lagrange Considérons un système de trois corps ponctuels, A 1, A2 et A3 , de masses respectives m1 , m 2 et m3, formant un triangle rigide S , qui tourne à vitesse angulaire constante V , autour d’une direction normale au plan du triangle, par rapport au référentiel galiléen lié aux étoiles (Fig. 13.7a). Dans le référentiel du centre de masse R ∗ , galiléen aussi puisque le système est isolé, le mouvement de A 1 satisfait à la loi fondamentale. On a donc : m1a 1 = G

m1 m 2 m 1 m3 r12 + G 3 r13 3 r12 r13

238

13. Système de corps ponctuels en interaction. Problème à deux corps L 4 Achille y

A3

A1

ro C

A3

π/3

x

Soleil A2

S

Jupiter A1

ro ro

A2

A 3 L 5 Patrocle b)

a) F IG . 13.7.

avec r12 = r 1 − r2, r 13 = r 1 − r3 et a 1 = −V 2 r1, puisque le triangle rigide a un mouvement de rotation uniforme par rapport à R∗ . Comme m 1 r1 + m2 r2 + m3 r3 = 0, d’après la définition du centre de masse, l’équation du mouvement donne, en éliminant le vecteur r 3 : m2

1 r 312



1 r313

r2 =

m1 m2 m3 V 2 − + + 3 G r313 r312 r13

r1

Les vecteurs r1 et r 3 n’étant pas colinéaires, l’égalité n’est possible que si les deux membres de l’équation sont nuls, ce qui implique r12 = r 13. En procédant de la même façon avec A 2, on obtiendrait aussi r21 = r23 . Il en résulte que : r 12 = r 13 = r23 = r0 et V 2 =

GM r 30

avec

M = m1 + m2 + m3

Les deux positions L4 et L5 du corps A3 , de part et d’autre de la droite A 1A 2 et à la même distance de A1 et A 2, sont appelées depuis 1772, date de leur prévision par J. Lagrange, les points de Lagrange (Fig. 13.7b). On montre que ces points correspondent à des positions d’équilibre. Aussi a-t-on envisagé de les utiliser dans le système Terre-Lune pour y placer des stations orbitales. Dans le cas du système Soleil-Jupiter, on sait depuis 1906 que l’un des points de Lagrange est occupé par l’astéroïde Achille et l’autre par l’astéroïde Patrocle ; en réalité, de nombreux astéroïdes, appelées planètes troyennes, se partagent ces positions de Lagrange.

CONCLUSION Retenons les points importants. (1) Dans le référentiel du centre de masse R ∗ , l’étude du système à deux corps ponctuels se réduit à celle du mouvement d’une seule particule fictive A, de masse m = m1 m2/(m 1 + m2). Cette particule est soumise de la part du centre de masse du système à la force d’interaction F 2→1 . (2) Il est avantageux d’exploiter les conservations occasionnelles du moment cinétique et de l’énergie mécanique, non seulement pour établir l’équation différentielle du mouvement, mais aussi pour discuter qualitativement de la nature du mouvement. Cette discussion, indispensable si l’information sur l’interaction n’est que partielle, est physiquement enrichissante, même dans les problèmes connus tels que celui de Kepler (cf. chapitre 12).

239

Système de corps ponctuels en interaction. Problème à deux corps

(3) Le mouvement d’un système matériel déformable Sd satisfait aux trois théorèmes généraux de la mécanique suivants : dP = Sex dt

d LO = M O,ex dt

d Ek = P ex + P in dt

et

Notons que les deux premiers ne font intervenir que les actions extérieures, contrairement au troisième. Il est préférable, à la fois sur les plans conceptuel et technique, d’écrire ce dernier théorème sous une forme différentielle, en faisant apparaître l’énergie mécanique Em et le travail des forces non conservatives : d Em = dW (exnc) + dWin(nc)

avec

Em = E k + E p,ex + E p,in

(4) Alors que la quantité de mouvement et le moment cinétique de tout système sont des grandeurs conservatives, c’est-à-dire telles que le terme de création est toujours nul, les grandeurs énergétiques de la mécanique que sont l’énergie cinétique et l’énergie mécanique ne sont pas conservatives. (5) Si le système S d est isolé, on a : P = Cte

LO = Cte

DEm = W(innc) = 0

mais

La thermodynamique étendra le concept d’énergie mécanique, en postulant l’existence d’une grandeur énergétique conservative. C’est le premier principe de la thermodynamique (cf. Thermodynamique). (6) Enfin, rappelons le théorème du viriel donnant la valeur moyenne dans le temps de l’énergie cinétique d’un système Sd de particules : Ek =

avec

=−

1 2

i

Fi · ri

Ce théorème est surtout utile en thermodynamique et en astrophysique.

EXERCICES ET PROBLÈMES

 c Dunod – Toute reproduction non autorisée est un délit

P13– 1. Système de trois particules On considère un système constitué de trois particules A1 , A2 et A3 , de masses respectives en kilogramme 0, 2, 0, 3 et 0, 5 kg. Les positions en mètre de ces points par rapport à l’origine O d’un référentiel galiléen sont les suivantes : r1 = 2t ex − 3 e y + t2 ez

r2 = (t + 1) ex + 3t ey − 4 ez

et r3 = t 2 ex − te y + (2t − 1)ez

t désignant la durée en seconde. 1. Trouver la quantité de mouvement du système. Comment varie le centre de masse au cours du temps ? Le système est-il isolé ? Calculer la somme des forces extérieures qui s’exercent éventuellement sur un tel système. 2. Quel est, en fonction du temps, le moment cinétique du système en O ? En déduire le moment des forces extérieures en O. 3. Donner la variation de l’énergie cinétique du système au cours du temps. En déduire la puissance de toutes les forces qui s’exercent sur le système, ainsi que le travail total entre les dates 0 et 1 s.

240

13. Système de corps ponctuels en interaction. Problème à deux corps

P13– 2. Conservation de l’énergie et invariance temporelle On considère le système isolé constitué de deux particules A 1 et A 2. L’énergie potentielle d’interaction est une fonction Ep des positions de A1 et A2 et ne dépend pas explicitement du temps. 1. Exprimer, en fonction de E p, les forces F1 et F 2 qui s’exercent sur A1 et A 2 respectivement. 2. En déduire les expressions correspondantes des variations élémentaires d E k et d P de l’énergie cinétique totale et de la quantité de mouvement totale, pendant la durée infinitésimale d t. 3. La condition E p stationnaire se traduit par ∂E p /∂t = 0. Montrer que cette invariance de Ep , par changement de l’origine des temps, entraîne une conséquence physique importante pour la grandeur E m = Ek + Ep. P13– 3. Atome de positronium Le positronium est un atome constitué d’un électron A 1 (masse me , charge −e) en interaction avec un positron A 2, antiparticule de l’électron, dont la masse est m e et la charge e. Il a été découvert en 1949 par le physicien allemand M. Deutsch (cf. Quantique). On désigne par R le référentiel du laboratoire par rapport auquel on étudie le mouvement d’un tel système que l’on peut considérer comme isolé. 1. Définir le référentiel du centre de masse R ∗ associé au système. Ce référentiel est-il galiléen ? Justifier. 2. a) Montrer que l’énergie potentielle d’interaction électrostatique entre A 1 et A 2 a pour expression : Ep = K , r, où K est une constante que l’on exprimera en fonction des constantes physiques du système. On précisera la convention adoptée pour l’origine de l’énergie potentielle et on commenter le signe de K. b) Calculer, eV, l’énergie potentielle, pour r = 106 pm. c) Même question pour l’énergie potentielle d’interaction gravitationnelle entre les deux particules ? Conclure. 3. Dans R∗ , le mouvement de l’électron et du positron s’obtiennent à partir de celui d’une seule particule fictive A soumise à la force d’interaction. a) Quelle est la masse de A ? Pourquoi le mouvement de A est-il plan ? b) Comment déduit-on les mouvements de A 1 et A2 dans R∗ de celui de A dans le même référentiel ? 4. a) À l’aide de la loi fondamentale de la dynamique appliquée à la particule A, en mouvement circulaire autour du centre de masse, établir l’expression, en fonction de r, de l’énergie cinétique E k∗ du système dans R∗. b) Comparer cette énergie cinétique à l’énergie potentielle et à l’énergie totale du positronium dans R ∗. En déduire les valeurs de Ek∗ et E ∗m pour r = 106 pm.

c) Sachant que toute particule chargée accélérée rayonne de l’énergie sous forme électromagnétique, comment varient Em∗ , E p(r) et E k∗ (r) ? La variation de Ek∗ surprend. Pourquoi ? Celle de E m∗ a conduit à l’abandon de la physique newtonienne à l’échelle microscopique. Pourquoi ?

241

Système de corps ponctuels en interaction. Problème à deux corps P13– 4. Vibrations de la molécule HCL

On étudie les mouvements relatifs des atomes d’hydrogène et de chlore de la molécule de HCl dans le référentiel du centre de masse R∗ de cette molécule. On admet que les forces d’interaction dépendent de la seule énergie potentielle E p(r), r étant la distance entre les atomes H et Cl. 1. Établir l’équation du mouvement radial de cette molécule, c’est-à-dire suivant r la distance r entre les atomes H et Cl, à partir de deux intégrales premières. On introduira l’énergie potentielle effective Ep,ef = Ep (r) + L2 /(2mr2 ), L étant la norme du moment cinétique et m la masse réduite des deux atomes. 2. Sachant que Ep,ef passe par un minimum pour r = r0 = 0, 126 nm (Fig. 13.8), quelle est, au voisinage de la position d’équilibre, l’expression de la force d’interaction entre H et Cl ? En déduire la nature du mouvement. 3. Les mesures spectroscopiques permettent de déterminer la fréquence de vibration de la molécule de HCl : f0 = 0, 857×10 14 Hz. Sachant que les masses de H et Cl sont respectivement 1, 67 × 10 −27 kg et 59, 4 × 10−27 kg, calculer la constante K caractéristique de cette vibration. Ep,ef

O

Ep,ef

r0 r O

r

E0

F IG . 13.8.

F IG . 13.9.

 c Dunod – Toute reproduction non autorisée est un délit

P13– 5. Interaction nucléaire entre neutron et proton Dans la théorie des forces nucléaires proposée par Yukawa en 1935, l’interaction attractive entre le neutron et le proton dans le deutéron (noyau de deutérium, isotope de l’hydrogène) est caractérisée par l’énergie potentielle suivante, fonction de la distance r qui sépare les deux particules : K r E p(r) = exp − r a a étant une distance caractéristique de quelques fm (1 fm = 10−15 m) et K = −100 MeV . fm la constante d’interaction. 1. Justifier le signe de K . Trouver l’expression littérale de la force correspondante. 2. Quelle est l’expression de l’énergie potentielle effective E p,ef en fonction de la masse réduite m du système et de son moment cinétique L∗ dans le référentiel du centre de masse ? Le graphe E p,ef (r) a l’allure représentée sur la figure 13.9. Trouver l’équation à laquelle satisfont les valeurs r1 et r2 de r pour lesquelles E p,ef (r) = 0. Même question pour les valeurs finies rm et r M pour lesquelles d Ep,ef / d r = 0. 3. Discuter les différents mouvements possibles suivant la valeur de l’énergie E . 4. On se place dans le cas où r = r m . Sachant que rm = 1 fm et a = 5 fm, calculer, en MeV, E ∗, E k∗ et Ep. En déduire la valeur du moment cinétique L∗ . On rappelle que m p ≈ mn ≈ 1, 67 × 10−27 kg. P13– 6. Interaction harmonique entre deux corps L’énergie potentielle d’interaction de deux corps est harmonique : E p = Kr 2/2, r étant la distance qui sépare les deux points matériels et K une constante positive d’interaction. 1. Donner l’expression de la force d’interaction.

242

13. Système de corps ponctuels en interaction. Problème à deux corps 2. Discuter qualitativement la nature du mouvement suivant la valeur du moment cinétique. 3. Calculer l’énergie et le moment cinétique si la trajectoire est un cercle de rayon r 0.

P13– 7. Effondrement gravitationnel du Soleil On admet que le Soleil est une distribution sphérique uniforme de masse, de rayon R ≈ 0, 7 × 10 9 m, de masse M ≈ 2× 1030 kg et de diamètre apparent u ≈ 32  d’arc. Il rayonne de l’énergie dans tout l’espace de telle sorte que la puissance reçue par une surface de 1 m 2 sur la Terre est 1 kW. 1. En attribuant l’énergie rayonnée à un effondrement gravitationnel, calculer, en watt, le taux de variation de l’énergie potentielle de gravitation. 2. Si la puissance rayonnée par le Soleil était constante, quelle serait la durée de vie du Soleil ? Comparer ce résultat à la durée estimée de 5 Gan ( 1 Gan = 10 9 années). P13– 8. Effet de marée dans un satellite Un satellite terrestre peut être schématisé par deux corpuscules A 1 et A2 , de même masse m, à une distance constante l l’un de l’autre. Ces points sont soumis uniquement à l’action gravitationnelle de la Terre (Fig. 13.10). On désigne par u l’angle que fait A 1A 2 avec la direction TC définie par le centre T de la Terre et le centre de masse C du satellite. 1. Exprimer 1/r1 = 1/TA1 en fonction de r0 = TC, l et u. Même question pour 1/r 2 = 1/TA2 . En déduire une expression approchée de l’énergie potentielle d’interaction gravitationnelle du satellite avec la Terre sachant que l  r 0 . 2. Trouver les positions angulaires d’équilibre stable et instable du satellite. 3. Calculer la période d’oscillation autour des positions d’équilibre stable. A2

C

r0 A1

T

θ

l

F IG . 13.10.

P13– 9. Masse volumique de l’Univers On considère un ensemble de points matériels répartis uniformément dans un volume sphérique, de centre O et de rayon R. La vitesse v de chaque point de la distribution est radiale, orientée vers l’extérieur du volume et proportionnelle à sa distance r à O (loi d’Hubble) : v = Hr

avec

H−1 = 13,7 Gan

H étant la constante d’Hubble (1 Gan = 1 milliard d’années). 1. On rappelle l’expression de l’énergie potentielle électrostatique d’une distribution analogue de charges électriques : ε0 E 2 1 3 Q2 Ep = d = 4pε0 5 R espace 2

Système de corps ponctuels en interaction. Problème à deux corps

243

E désignant le champ électrostatique, Q la charge totale et le volume. Trouver, par analogie, l’énergie potentielle de gravitation de la distribution de masse considérée, en fonction de la masse totale M et de R. Commenter le signe de cette énergie. 2. Montrer que l’énergie cinétique de la distribution s’écrit : E k = aMH 2R 2, a étant un facteur numérique à déterminer. 3. Les observations les plus récentes en astrophysique montrent que l’énergie de l’ensemble de la distribution est nulle. Quelle est donc l’expression de la masse volumique r en fonction de H et G ? Calculer numériquement sa valeur en unité SI. À quel nombre de nucléons par unité de volume cette valeur correspond-elle ? P13– 10. Galaxie spirale Une galaxie spirale peut être schématisée par un disque et un noyau sphérique de masse M n contenant pratiquement toute la masse de la galaxie (99 %). Dans le disque, chaque constituant A (étoile, nuage d’hydrogène, etc.), de masse m, situé à la distance r du centre O, est animé d’un mouvement circulaire uniforme de vitesse v. On mesure cette vitesse par effet Doppler-Fizeau (cf. chapitre 32). 1. En appliquant la loi fondamentale de la dynamique au point A, montrer que l’on peut déduire la masse Mn de la mesure de v. 2. Calculer le rapport M n /MS de la masse de la galaxie sur la masse du Soleil, sachant que v = 325 km . s−1 à r = 4 kpc, kpc étant le kiloparsec et le parsec la distance à laquelle le diamètre de l’orbite terreste autour du Soleil est vu sous un angle de 1 seconde d’arc (cf. constante physiques). 3. Expérimentalement, on constate que, pour r < 4 kpc, la vitesse augmente avec r, ce qui est en contradiction évidente avec le modèle simplifié considéré ? Quelle l’hypothèse faut-il remettre en cause ?

 c Dunod – Toute reproduction non autorisée est un délit

P13– 11. Ordre de grandeur de la température du Soleil On admet que le Soleil est constitué d’un ensemble de particules identiques dont la masse est celle du proton (mp c2 ≈ 938 MeV). L’énergie cinétique moyenne des protons est 3kBT /2, si T est la température du Soleil et kB = 1, 38 × 10 −23 J . K−1 la constante de Boltzmann. 1. Établir la relation suivante : E k = −Ep /2. 2. Sachant que la masse du Soleil est 2 × 10 30 kg et que son rayon R = 0, 7 × 109 m, trouver un ordre de grandeur de sa température à l’aide du théorème du viriel. P13– 12. Viriel associé à une énergie potentielle en Ep = Kr n On considère une particule d’énergie potentielle E p = Kr n , r étant la distance de la particule à une origine O et n un entier positif ou négatif. Calculer la valeur moyenne E de l’énergie totale en fonction de celle de l’énergie cinétique et de celle de l’énergie potentielle. Étudier successivement les cas où n = 2 , n = −1 et n = −6 .

244

13. Système de corps ponctuels en interaction. Problème à deux corps

P13– 13. Système de deux corpuscules liés par un fil inextensible Deux corps ponctuels A1 et A2, de masses respectives m 1 et m 2, sont reliés par un fil inextensible de longueur l. Le premier se déplace sans frottement dans un plan horizontal, le second est suspendu verticalement à une extrémité du fil. Ce dernier relie les deux corps en passant par un trou O effectué dans le plan horizontal. 1. Pourquoi le moment cinétique de ce système en projection selon l’axe vertical ascendant Oz est-il constant ? 2. Montrer que le travail des forces de tension exercées par le fil sur A 1 et A2 est nul. En déduire que l’énergie mécanique du système se conserve. À quelle équation différentielle radiale le système satisfait-il ? 3. Initialement r = l/2, r˙ = 0 et ra˙ = v0. Quelle doit être la vitesse orthoradiale v 0 pour que A2 ne traverse pas le trou ? Application numérique : m 1 = 0, 2 kg, m 2 = 0, 4 kg et l = 1 m. P13– 14. Système de deux masselottes en interaction élastique et en chute libre Deux masselottes A 1 et A2 de même masse m = 0, 2 kg , reliées par un ressort (raideur K = 400 N.m−1 , longueur à vide l0 = 10 cm ), sont astreintes à se déplacer sans frottement, le long d’un axe vertical descendant Ox . L’ensemble étant maintenu au repos par un fil, on coupe ce dernier à l’instant initial (Fig. 13.11). 1. a) Appliquer le théorème du centre de masse au système. Trouver la nature du mouvement du centre de masse C , ainsi que la relation entre les accélérations des deux masselottes. b) L’énergie mécanique du système se conserve-t-elle ? En déduire une seconde équation du mouvement. 2. a) Appliquer la loi fondamentale de la dynamique à chacune des masselottes par rapport au référentiel du laboratoire R . En déduire l’équation différentielle à laquelle satisfait r = r1 − r2 .

b) Quel est le mouvement des masselottes par rapport au référentiel du centre de masse, sachant que, dans ce dernier, on comprime initialement le ressort de 1 cm sans vitesse ? Calculer la fréquence d’oscillation. c) Trouver le mouvement des masselottes par rapport à R .

3. Quelles sont les accélérations des masselottes à l’instant initial ?

A1 g A2 x F IG . 13.11.

14 Collision de deux particules Si la loi d’interaction entre deux particules n’est pas parfaitement connue, il est intéressant de considérer cette interaction comme une collision : c’est là une attitude qui consiste à comparer les états initial et final et à ignorer les états intermédiaires. L’efficacité et le succès de cette attitude ont été tels qu’à l’heure actuelle on l’élargit à tous les domaines de la physique. L’analyse systématique en termes de bilan et de loi de conservation, telle qu’on les rencontre en chimie notamment, relève de cette même attitude. En outre, les collisions sont en physique un moyen d’investigation considérable. Pour connaître la structure d’édifices atomiques ou nucléaires, on lance sur ces derniers des projectiles de caractéristiques connues, tels que des électrons ou des protons, et on analyse les produits de collisions. Nous limiterons ici notre analyse au seul cadre newtonien, en laissant de côté volontairement la très importante extension relativiste (cf. Relativité et invariance).

I . — DÉFINITION ET PROPRIÉTÉS DES COLLISIONS DE PARTICULES

 c Dunod – Toute reproduction non autorisée est un délit

I . 1 . — Définition Dans le langage habituel, on dit que deux systèmes matériels entrent en collision ou subissent un choc à un instant, lorsque les surfaces qui les délimitent sont en contact à cet instant sans l’être dans les instants voisins, antérieur et postérieur parfois. L’expérience courante permet d’établir aisément que le choc : i) dure très peu de temps, ii) est localisé dans l’espace, iii) implique une variation brutale de la vitesse des points qui constituent les deux systèmes matériels. Cette variation brutale de la vitesse est attribuée à une interaction de courte portée, dite de contact. Sa complexité est si grande que l’on préfère renoncer à étudier ce qui se passe au cours du choc et ne faire que le bilan des modifications de la quantité de mouvement et de l’énergie cinétique du système total. Nous n’étudions ici que la collision entre deux particules réservant à une étude ultérieure le choc entre deux solides (cf. chapitre 21). Dans le cas de deux particules, la définition précédente du choc macroscopique par le contact de surfaces doit être abandonnée. On doit la remplacer par la notion plus générale d’interaction.

246

14. Collision de deux particules

Considérons deux particules A 1 et A 2, de masses respectives m 1 et m2. Leur interaction de courte portée est définie par l’énergie potentielle Ep(r), fonction de la distance r qui les sépare. Si ces points sont à une distance suffisante l’un de l’autre, Ep (r) est pratiquement nul.

Supposons ces particules initialement à une distance telle que E p(r) = 0 et se dirigeant l’une vers l’autre avec des vitesses v 1 et v2 dans le référentiel du laboratoire R (Fig. 14.1). Lorsqu’elles parviennent au voisinage l’une de l’autre, E p(r) = 0 et les vitesses varient jusqu’à ce que la distance atteigne à nouveau une valeur suffisante pour que Ep (r) soit à nouveau nul. Les vitesses acquises v1 et v2 sont alors appelées vitesses finales de la collision. On définit ainsi un domaine en dehors duquel l’interaction est négligée. v1

A1 v1

A1 v 2

v2 A 2

A2

Domaine d'interaction F IG . 14.1.

I . 2 . — Lois générales Ces lois particulières s’inscrivent dans le cadre de la mécanique des systèmes matériels mais s’appuient sur les hypothèses de localisation et de brièveté des collisions. Elles méritent le qualificatif de générales, d’une part parce qu’elles concernent tous les systèmes dont le mouvement satisfait aux hypothèses précédentes, d’autre part en raison de l’influence négligeable de la nature précise de l’interaction. a) Conservation de la quantité de mouvement totale Le système des deux particules étant isolé, la quantité de mouvement de l’ensemble, par rapport au référentiel du laboratoire R, est une constante vectorielle du mouvement (cf. chapitre 13). Les quantités de mouvement avant et après la collision sont donc égales : P av = Pap . Il en résulte, avec des notations explicites, l’indice i désignant les particules initiales de la collision et l’indice f les particules finales : pi = i

pf f

Notons que, dans le cas de systèmes non isolés, soumis à des forces extérieures finies telles que la pesanteur, la conservation de la quantité de mouvement est encore satisfaite. En effet, une analyse attentive (cf. chapitre 21) montre que la variation de la quantité de mouvement totale fait apparaître les forces extérieures uniquement par leurs percussions : t1 +t t1

Fex→i d t

t étant la durée de la collision. En effet : dP = S ex = dt

Fex→i i

entraîne

dP =

Fex→i d t i

d’où

t 1 +t

r

DP = P = i

t1

Fex→i d t

247

Collision de deux particules

Comme t est très faible devant toute durée caractéristique de l’expérience, les forces finies ont des percussions négligeables : t1 +t t1

F ex→i d t  tF m ≈ 0

Fex→i  F m 

si

C’est là une simplification de taille qui explique l’intérêt de l’analyse en termes de collision. b) Conservation de l’énergie totale Si on suppose que les forces d’interaction intérieures au système des deux particules dérivent d’une énergie potentielle, l’énergie mécanique, somme de l’énergie cinétique et de l’énergie potentielle, est aussi une constante. Au cours d’une collision, les particules A 1 et A2 peuvent se scinder en deux : leur état peut donc fondamentalement être modifié. Afin de tenir compte d’une telle modification, nous devons étendre la conservation de l’énergie mécanique à celle de l’énergie totale E, somme des énergies cinétiques, de l’énergie potentielle extérieure Ep,ex , et d’une énergie interne U du système, laquelle est constituée de l’énergie potentielle d’interaction et des énergies internes Ui de chacune des particules. On a donc : avec

E av = Eap

et U = E p,in +

E = Ek + E p,ex + U

Ui i

Comme la variation de position du système est négligeable au cours de la collision, E p,ex et Ep,in ne varient pas. Il en résulte en explicitant :

i

Ek,i + Ui =

f

Ek,f + Uf

c) Collision élastique i) Définition Une collision est dite élastique si le nombre ou la nature des particules en interaction sont inchangés. On en déduit que, dans une telle collision, l’énergie cinétique totale se conserve : Ui =  c Dunod – Toute reproduction non autorisée est un délit

i

Uf

d’où i

f

Ek,i =

f

Ek,f

Si, après l’interaction, le nombre ou la nature des deux particules sont modifiés, la collision est inélastique. En physique, les exemples sont nombreux : citons l’interaction chimique de deux molécules, la fusion de deux particules, l’excitation d’une transition atomique à la suite d’une collision électron-atome. Ainsi, au cours d’une collision élastique entre deux particules, ce que l’on représente par :

on a :

A1 + A 2 −→ A1 + A2 p1 + p 2 = p 1 + p2

et

p21 p2 p2 p2 + 2 = 1 + 2 2m 1 2m2 2m 1 2m2

La loi de conservation de la quantité de mouvement d’un système de deux particules, au cours d’une collision élastique, fournit une méthode de comparaison des masses de deux particules. En effet, en primant les grandeurs après la collision, on a : m1v 1 + m2 v2 = m 1 v1 + m2v2

d’où

m2 v − v 1 = 1 m1 v2 − v 2

248

14. Collision de deux particules

ii) Propriété remarquable d’une collision élastique dans le référentiel du centre de masse Dans le référentiel R ∗ , qui a les mêmes propriétés que R, puisqu’il a un mouvement de translation et que le système est isolé, les relations de conservation précédentes s’écrivent, si la collision est élastique : p∗2 p∗2 p∗2 p∗2 ∗ 1 2 1 2 + = + p∗1 + p∗2 = p∗ + p = 0 et 1 2 2m1 2m2 2m 1 2m2 Les équations de conservation précédentes donnent : p∗2 1 2

1 1 + m1 m2

=

p∗2 1 2

1 1 + m1 m2

d’où

∗ ∗ p ∗1  =  p∗ 1  =  p2  =  p2 

Au cours d’une collision élastique, chacune des particules conserve la norme de sa quantité de mouvement et donc son énergie dans le référentiel du centre de masse R ∗ . Ce résultat est important, car il permet de déduire le caractère élastique ou non d’une collision de la seule observation de la conservation de la norme de la quantité de mouvement de l’une des deux particules. Par exemple, en microscopie électronique, où l’électron incident pénètre dans l’échantillon avec lequel il interagit, on peut aisément tirer de la conservation de l’énergie de l’électron incident le caractère élastique ou non de la collision. En effet, dans ce cas, en raison du rapport élevé des masses des particules en interaction (électron et atome), le référentiel du centre de masse est confondu avec le référentiel du laboratoire. Sur la figure 14.2a, on a représenté les vecteurs quantités de mouvement de chaque particule dans R ∗, avant et après la collision élastique. R∗

p∗ 1 p∗2

R

p∗1

p2

p ∗ 2

p 1 p1

a)

b) F IG . 14.2.

iii) Cas particulier important Si le projectile et la cible immobile ont même masse (Fig. 14.2b), l’angle que font entre eux les vecteurs quantités de mouvement après la collision élastique est droit, car les relations : p1 + 0 = p 1 + p2

et p 21 = p 21 + p22

impliquent que le triangle formé par p1 , p1 et p2 est rectangle. En effet, on reconnaît dans la deuxième équation la relation de Pythagore. d) Collision inélastique Si, après l’interaction, le nombre ou la nature des deux particules sont modifiés, la collision est inélastique. En physique, les exemples sont nombreux : citons l’interaction chimique de deux molécules, la fusion de deux particules, l’excitation d’une transition atomique à la suite d’une collision électronatome. Typiquement, on écrira : A 1 + A 2 −→ A3 + A 4 + A5 + · · ·

249

Collision de deux particules

II . — COLLISION ÉLASTIQUE DIRECTE DE DEUX PARTICULES Une collision élastique est directe si les vecteurs vitesses, avant et après la collision, sont colinéaires. II . 1 . — Vitesses après la collision Les lois de conservation de la quantité de mouvement totale et de l’énergie cinétique totale fournissent deux équations scalaires contenant les deux inconnues v1 et v 2 sur l’axe du mouvement. Il vient : 1 1 1 1 m1 v1 + m2v 2 = m 1v 1 + m2v 2 et m1v 12 + m2 v 22 = m 1v 21 + m 2 v22 2 2 2 2 soit : m 1(v 1 − v1 ) = m2 (v 2 − v2) et m1 (v21 − v21 ) = m2(v 22 − v22) En divisant cette dernière équation par la précédente, on trouve : v1 + v1 = v 2 + v 2. Il en résulte que : v1 =

2m2v 2 + (m1 − m2 )v1 m 1 + m2

et v 2 =

2m1 v1 − (m1 − m2 )v2 m 1 + m2

Dans le cas particulier important où la particule A2 est une cible immobile (v 2 = 0), ces relations deviennent : 2m1 m1− m2 v1 = v1 v 2 = v1 m1+ m2 m 1 + m2 Si m2 > m1, le choc fait rebrousser chemin à A1 , alors que si m2 < m1, A 1 garde le même sens de déplacement. Pour m2 = m 1, A1 et A2 échangent leurs vitesses. II . 2 . — Perte d’énergie cinétique du projectile lorsque la cible est immobile La perte d’énergie cinétique de la particule A 1 est, au cours d’une collision élastique directe :

 c Dunod – Toute reproduction non autorisée est un délit

1 4m 1 m2 Q = −DE k,1 = DEk,2 = m 2v 22 = Ek,1 2 (m1 + m2 )2 i) Si m1 = m2, c’est par exemple la collision électron-électron ou la collision proton-proton : Q = Ek,1 et E k,1 = 0 Le projectile A1 perd toute son énergie cinétique au profit de A 2 . ii) Si m 1  m 2, c’est par exemple la collision proton-électron : Q≈

4m2 E  Ek,1 m 1 k,1

L’énergie cédée par le projectile A 1 à la cible A 2 est très faible. iii) Si m 1  m2, c’est par exemple la collision électron-proton ou celle d’une masse ponctuelle sur une paroi fixe rigide (cf. chapitre 21) : Q≈ Là aussi, l’énergie fournie à A2 est faible.

4m1 E  Ek,1 m 2 k,1

250

14. Collision de deux particules

III . — DIFFUSION ÉLASTIQUE PAR UNE CIBLE IMMOBILE Cette étude, dont le paragraphe précédent n’est qu’un cas particulier, est très importante en physique des collisions : A1 est le projectile et A2 la cible immobile. Le problème posé ne peut être complètement résolu : en effet, comme le mouvement est plan, les équations de conservation de la quantité de mouvement et de l’énergie cinétique fournissent au   total trois équations, alors que le nombre d’inconnues scalaires est de quatre : v1x , v1y , v 2x, v2y. Cette indétermination est due à l’ignorance de l’énergie potentielle Ep(r). Cependant, cette étude présente un grand intérêt, car les résultats établis sont alors indépendants de la nature particulière de E p(r). III . 1 . — Représentation géométrique de p1 et p2 Dans le référentiel du centre de masse R ∗ , on sait que les quantités de mouvement des particules sont opposées et leurs normes conservées (cf. chapitre 13 et I.2c) : p∗1 = p∗2 = p∗1 = p∗2 = p ∗ = mv

avec

m=

m 1m2 m1 + m2

et v = v1 − v2

m étant la masse réduite du système des deux particules. Dans le cas considéré où la cible est au repos (v 2 = 0) : v = v 1 − v2 = v 1

d’où

p∗ = mv = mv1

En utilisant la loi de composition des vitesses de A1 , entre R et R∗ , on obtient p1 : ∗ p 1 = m 1v1 = m 1(v∗ 1 + v C) = p1 + m1 v C

avec

vC =

mv 1 m1v1 + m2 v2 m1 v1 = = m1 + m2 m1 + m2 m2

puisque v2 = 0. Si l’on désigne par n le vecteur unitaire porté par la direction de p ∗1 et par u∗ l’angle (v1 , n), p 1 s’écrit : p 1 = p ∗n +

m1 mv1 m2

On établit de même que : p 2 = m 2(v∗2 + vC ) = p ∗2 + m2v C = −p ∗1 + mv1 = −p∗ n + mv1

puisque

p ∗2 = −p ∗1 = −p∗ n

Il est commode de représenter géométriquement ces résultats en traçant un cercle, de centre une origine arbitraire O, de rayon p∗ = mv1, et en portant les vecteurs OB = mv1 et OA = −(m 1/m 2)mv1 (Fig. 14.3a). La valeur de l’angle de diffusion u ∗ dans R ∗ détermine la position d’un point C sur le cercle et donc p1 et p2 car : Notons que :

p 1 = AC = AO + OC et p 2 = CB = OB − OC AB = OB − OA = mv1

1+

m1 m2

= m1 v1 = p1

Le point B est sur le cercle ; il en est de même pour A si les masses des deux particules sont égales.

251

Collision de deux particules

K C p2 p1 ∗ µ A µ10 n p∗ B Direction m 1  H − ¹v O µ 2 incidente m2 0

0

A

u1,m m − 1 ¹v m2

a)

p1 µ0 1

C n O

p∗ µ



p 2 B

H

µ 2

b) F IG . 14.3.

III . 2 . — Expressions des angles de diffusion dans R À l’aide de la figure 14.3a, on voit que l’angle u 1 de diffusion dans R de la particule A 1 a pour expression : sin u ∗ HC p∗ sin u ∗ = = tan u 1 = AH m1 /m2 p ∗ + p∗ cos u∗ m1 /m2 + cos u∗ On en déduit u2 :

p u∗ − = 2 2  < m Notons que, si m1 2 , l’angle de diffusion du projectile u 1 peut être compris entre 0 et p ; cette dernière valeur correspond à la rétrodiffusion. En revanche, si m1 > m2, l’angle de diffusion du projectile u 1 est compris entre 0 et une valeur limite que l’on obtient aisément à l’aide de la représentation géométrique. Le point représentatif A est alors situé à l’extérieur du cercle (Fig. 14.3b) ; l’angle de diffusion maximale u 1,m est l’angle que fait la tangente menée de A au cercle avec la quantité de mouvement p1 = AB = m1 v1 . On voit aisément que : |u2|

sin u1,m =

OK m2 mv = = OA m1 mv/m2 m1

Ainsi, dans une collision élastique entre un proton projectile et un électron cible, la valeur maximale de l’angle de diffusion du proton n’est que : u1,m = 1/1 836 ≈ 0, 5 mrad.  c Dunod – Toute reproduction non autorisée est un délit

III . 3 . — Perte d’énergie du projectile Cette énergie est l’énergie cinétique acquise par la cible A 2 . En s’aidant de la figure 14.3a, on trouve : ∗ p22 4m 2v 21 sin 2(u∗ /2) 4m 1m2 2 u  Q = Ek,2 = Ek,1/R = = sin 2m2 2m2 2 (m 1 + m2) 2 On détermine u∗ à l’aide de loi d’interaction Ep(r). Par exemple, dans les cas newtonien ou coulombien pour lesquels Ep(r) = K /r, on montre que (cf. chapitre 15) : tan

u∗ 2

=

|K | 2bE k,1

b étant le paramètre d’impact de collision, c’est-à-dire la distance de la cible à la vitesse initiale du projectile A1 . Dans ces conditions, la perte d’énergie relative est : 4m1 m2 1 Q = 2 E k,1 (m1 + m2) 1 + (2bE k,1/K) 2

252

14. Collision de deux particules

Si Ek,1  |K |/2b, les expressions précédentes se réduisent à : u∗ ≈

|K |/b  1 et Ek,1

4m1m2 Q ≈ (m 1 + m 2 )2 Ek,1

K 2bEk,1

2

L’angle de diffusion et la perte d’énergie relative de la particule projectile sont alors très faibles. III . 4 . — Exemples a) Collision électron-électron Comme m1 = m 2 = m e , le point représentatif A est sur le cercle de centre O et de rayon mv1 (Fig. 14.4a). On retrouve la diffusion à angle droit. La perte d’énergie est maximale au cours du choc direct (u∗ = p) : Q max =1 E k,1 Ce type de collision est fréquent ; en microscopie électronique par exemple, l’électron incident provenant du canon du microscope entre en collision avec les électrons atomiques des édifices constituant le matériau observé. p1

C p1 A

θ1 O

C

p2 θ∗

θ∗

B θ2

a)

O

b)

C

p 1

p2 A

B θ2

O

θ1

p 2 B θ2

c)

F IG . 14.4.

b) Collision proton-électron Dans ce cas, m 1 = m p et m2 = me  mp. Le point représentatif A est à gauche de O, à très grande distance (Fig. 14.4b). L’angle de diffusion u1 est très faible : le proton n’est donc pratiquement pas dévié et : Qmax 4me ≈ Ek,1 mp c) Collision électron-proton Le point représentatif A est très voisin du centre O puisque m 1 = me, m2 = mp  m e (Fig. 14.4c). On voit que u 1 = u∗ , ce qui était prévisible puisque les référentiels du laboratoire et du centre de masse sont dans ce cas pratiquement confondus. Il en résulte que : 4me Qmax ≈ Ek,1 mp Notons que la norme de la quantité de mouvement de la particule diffusée (OC) est la même que celle de la particule incidente (OB). Ce résultat est celui d’une collision élastique d’une masse ponctuelle sur une paroi fixe.

253

Collision de deux particules

IV . — COLLISIONS INÉLASTIQUES IV . 1 . — Facteur énergétique d’une collision On caractérise une collision inélastique entre deux particules, l’une le projectile, l’autre la cible, par la variation d’énergie cinétique du système : DEk =

E k,f −

f

i

Ek,i =

i

Ui −

Uf f

Si DEk < 0, la collision inélastique est endoénergétique ; si DEk > 0, elle est exoénergétique. Il est naturel d’introduire le facteur de restitution en énergie ε : ε=

f i

Ek,f Ek,i

Pour ε = 0, la perte d’énergie cinétique est maximale : la collision est parfaitement inélastique. On retrouve évidemment la limite élastique en faisant ε = 1. Exemples : (1) La collision entre deux points matériels, dont l’un se brise en deux, est nécessairement endoénergétique, puisqu’une partie de l’énergie cinétique initiale a servi à séparer en deux parties l’une des particules. (2) Une réaction chimique exothermique entre deux molécules est une collision inélastique. (3) Une collision entre deux particules A 1 et A 2 qui restent soudées en formant une particule unique A3 est parfaitement inélastique. En effet, dans le référentiel R ∗, galiléen puisque le système peut être considéré comme isolé, on a : ∗ ∗ ∗ − (E k,1 + E ∗k,2) avec Ek,3 =0 DEk = DEk∗ = Ek,3 puisque la particule A3 est fixe dans R ∗ . Par conséquent : DEk = −

i

E ∗k,i

Ainsi, au cours d’une collision parfaitement inélastique, le système perd toute son énergie cinétique.

 c Dunod – Toute reproduction non autorisée est un délit

IV . 2 . — Énergie cinétique seuil d’une collision inélastique Considérons une collision inélastique entre un atome projectile, d’énergie cinétique E k,1 , et une molécule cible au repos. Après collision, la cible est dans un nouvel état caractérisé par une nouvelle valeur de l’énergie interne. L’équation de conservation de l’énergie donne, dans R ∗ : i

Ek∗,i + Ui =

f

Ek∗,f + Uf

Dans R∗ , les produits de la réaction peuvent être chacun au repos puisque l’on peut réaliser P ∗ = 0 en faisant séparément p∗f = 0, soit E ∗k,f = 0. On en déduit : i

Ek∗,i + Ui 

Uf f

soit i

Ek∗,i  DU

avec

DU = f

Uf −

Ui i

Comme l’énergie cinétique des deux particules dans R∗ est mv2 /2, v étant la vitesse relative, et que la cible est immobile dans R, on a : Ek∗ =

mv 2 = 2

m1m 2 m1 + m2

v 21 m2 Ek,1  DU = 2 m1 + m 2

d’où

E k,1 

m1 + m2 DU m2

254

14. Collision de deux particules

Ainsi, l’énergie cinétique seuil de la collision inélastique a pour expression : s Ek,1 =

m 1 + m2 DU m2

Exemple : L’énergie cinétique seuil d’un électron incident capable de produire une transition K d’un électron atomique dans l’aluminium, c’est-à-dire d’extraire totalement de l’atome cet électron de la couche K, est : m + mAl E sk,1 = e E K ≈ EK = 1 550 eV mAl puisque m e  m Al . Avec un proton incident, cette énergie serait : s Ek,1 =

m p + m Al 28 EK = × 1 550 ≈ 1 607 eV 27 m Al

Remarque : Ce concept d’énergie cinétique seuil joue un rôle majeur dans les collisions inélastiques relativistes car il permet de prévoir la création de particules nouvelles à l’issue d’une collision (cf. Relativité et invariance).

CONCLUSION Retenons, outre l’importance des collisions comme moyen d’investigation en physique, plusieurs points. (1) Au cours d’une collision élastique ou inélastique, la quantité de mouvement et l’énergie du système isolé constitué par les particules se conservent : pi = i

pf i

f

E k,i + Ui =

f

Ek,f + U f

(2) Si la collision est élastique, l’énergie cinétique totale se conserve : i

Ek,i =

f

Ek,f

(3) Dans le référentiel du centre de masse, l’énergie cinétique de chaque particule se conserve au cours d’une collision élastique. (4) L’énergie cinétique seuil d’une collision inélastique entre une particule projectile et une particule cible au repos a pour expression : m 1 + m2 s DU Ek,1 = m2 DU étant la variation d’énergie interne du système au cours de la collision inélastique. Pour étudier les collisions entre particules rapides, il est nécessaire de se placer dans le cadre relativiste. Les lois de conservation y sont les mêmes, mais les expressions de la quantité de mouvement et de l’énergie cinétique sont différentes ; plus important, le concept d’énergie de masse, qui s’introduit naturellement joue un rôle essentiel dans l’écriture de la conservation de l’énergie au cours des collisions inélastiques de type fission ou fusion de particules (cf. Relativité et invariance).

255

Collision de deux particules

EXERCICES ET PROBLÈMES P14– 1. Spectrométrie neutronique On considère un faisceau de neutrons dont on veut déterminer l’énergie cinétique à partir de l’étude de la collision élastique neutron-noyau d’hélium. Ce dernier joue le rôle de particule cible, au repos dans le référentiel du laboratoire R. On adopte les notations suivantes : m 1 est la masse du neutron et m2 celle du noyau, Ek,1 est l’énergie cinétique et p1 la quantité de mouvement du neutron incident, E k,1 est  l’énergie cinétique et p1 la quantité de mouvement du neutron diffusé, Ek,2 est l’énergie cinétique et p 2 la quantité de mouvement du noyau après la collision, u1 et u2 sont respectivement les angles (p1 , p1) et (p1, p 2). 1. Quelle est, en fonction de m 1, m 2, u2 et Ek,1 , l’énergie cinétique Q transférée au noyau ? Les noyaux cibles étant connus, montrer que la mesure de Q et de u 2 permet d’obtenir la valeur de E k,1.

2. On considère le cas où m2 > m 1. a) À partir des lois de conservation, montrer que |u 2| est compris entre 0 et p/2. b) Calculer la valeur maximale Q max de Q pour Ek,1 , m1 et m 2 fixés. Préciser les valeurs cor , p , p    respondantes de Ek,1 1 2 , u 2 et u1 .  , p c) Quelle est la valeur minimale Q min de Q. Préciser les valeurs correspondantes de Ek,1 1,    p 2, u2 et u 1.  en fonction de E  3. On considère le cas où m 1 = m 2. Exprimer E k,1 et Ek,2 k,1 et u 2. Que peut-on dire des directions de p1 et p 2 ?

P14– 2. Ralentissement de neutrons

 c Dunod – Toute reproduction non autorisée est un délit

1. Quelle est l’énergie maximale Q max transférée à un atome cible (masse ma) par un neutron (masse mn) d’énergie cinétique E k , au cours d’une collision élastique ? Représenter q max = Qmax /Ek en fonction du rapport des masses h = mn/ma. 2. Pour ralentir des neutrons, d’énergie cinétique initiale E k,i = 6 MeV, jusqu’à ce qu’ils deviennent des neutrons thermiques d’énergie cinétique finale E k,f = 3k BT /2, à T = 300 K, on leur fait subir des collisions avec des atomes de masse mq = 9mn . Sachant que la perte d’énergie relative est en moyenne égale à 0, 82 qmax/2 , à chaque collision, calculer le nombre moyen de collisions nécessaires à ce ralentissement. P14– 3. Angle maximal de diffusion dans une collision élastique Montrer que la diffusion élastique d’un projectile A 1 par une cible A2 , de masse m2 < m1 , sous un angle déterminé, peut être réalisée pour deux valeurs différentes de l’énergie cinétique transmise à la particule cible A2. On utilisera la représentation graphique des quantités de mouvement à l’aide du cercle de rayon mv, m étant la masse réduite et v la différence des vitesses des deux particules (Fig. 14.4). P14– 4. Énergie transférée au cours d’une collision inélastique Un projectile A 1, de masse m1 , de quantité de mouvement p 1 = m1 v1 et d’énergie cinétique E k,1 , est dévié par un atome cible A2 au repos, de masse m 2  m 1. L’atome acquiert une énergie cinétique négligeable et une énergie interne DE  E k,1.

256

14. Collision de deux particules

1. Écrire les relations vectorielle et scalaire relatives à la quantité de mouvement et à l’énergie. On désigne par p et p les quantités de mouvement de A1 avant et après la collision, et par P la quantité de mouvement acquise par A2 . 2. Exprimer P 2 en fonction de l’angle de diffusion u = (p, p ) et analyser le cas où u est petit. 3. Quelles sont les valeurs extrêmes de Q = P2 /(2m e ), me étant la masse de l’électron, particule projectile ? P14– 5. Énergie seuil d’une collision inélastique Un proton A 1 subit une collision inélastique avec un atome de carbone A 2 au repos (masse m2). Après collision, les particules A 1 et A ∗2 , issues de la réaction, s’écartent d’un angle u avec les énergies  cinétiques respectives Ek,1 et E k,2 . L’énergie interne de l’atome a augmenté de DE n ≈ 300 eV. 1. Calculer l’énergie seuil de la collision.

  et u. 2. Trouver DEn en fonction de Ek,1 , Ek,2

15 Diffusion de Rutherford. Notion de section efficace La diffusion de particules est un thème d’une importance considérable en physique moderne. En effet, la démarche habituelle dans la recherche des propriétés microscopiques des corps consiste souvent à bombarder une cible inconnue avec un faisceau de particules projectiles, de caractéristiques physiques bien connues. L’étude de la diffusion est donc celle d’une interaction entre N corps, N étant très grand, ce qui rend l’analyse inextricable. Cependant, on sait qu’on peut, dans la plupart des cas, ramener ce problème à celui d’une interaction entre deux corps : le projectile et la cible (cf. chapitre 13). Nous proposons d’abord de rappeler ce que l’on désigne habituellement par diffusion de particules en donnant l’exemple de la diffusion de Rutherford. Nous dégageons ensuite le concept de section efficace en nous appuyant sur le cas simple de la diffusion de particules par une sphère dure. Nous introduisons en outre la notion de libre parcours moyen, directement associée à celle de section efficace.

I . — DIFFUSION DE RUTHERFORD

 c Dunod – Toute reproduction non autorisée est un délit

I . 1 . — Aspect expérimental L’étude expérimentale de la diffusion de particules a (noyaux d’hélium) par des atomes lourds fut conduite par le physicien anglais E. Rutherford, à partir de 1909, afin de tester le modèle atomique proposé par les anglais W. Thomson (Lord Kelvin) et J. Thomson. Selon ce modèle, les atomes sont représentés par des distributions sphériques de charge positive dans lesquelles sont incrustées des particules de charges négatives. Avec ses collaborateurs, Rutherford utilisa une enceinte vidée d’air dans laquelle une source radiaoactive envoyait, sur une feuille mince d’or, des noyaux d’hélium appelées particules a (Fig. 15.1) ; les particules diffusées étaient détectées par un matériau capable d’émettre de la lumière visible sous l’impact d’une particule chargée (détecteur à scintillations). Rutherford constata, à sa grande surprise, que des particules a pouvaient être détectées sous des angles de diffusion x égaux ou supérieurs à p/2 et que certaines étaient même rétrodiffusées (x = p). En effet, le modèle de Thomson, avec ses charges positives et négatives réparties dans le volume de l’atome, ne permettait de prédire que de faibles angles de diffusion. En outre, avec son équipe, il put établir que le nombre de particules a variait avec l’angle de diffusion x selon une loi en sin−4(x/2) (Fig. 15.2).

258

15. Diffusion de Rutherford. Notion de section efficace Nombre de particules diffusées Feuille d’or

10 5 Lunette 10 4

Source radioactive

Points expérimentaux

10 3 10 2

Détecteur à scintillation

sin-4 (X/2)

10 1 0

Pompe à vide

100

F IG . 15.1.

x(degrés)

F IG . 15.2.

La facilité avec laquelle les particules a traversaient la feuille d’or et l’existence de grandes valeurs de l’angle de diffusion conduisirent Rutherford à attribuer la diffusion sous grand angle à des charges positives concentrées en un noyau central et la diffusion sous petit angle aux électrons placées sur la périphérie de l’atome. Il s’en suivit l’abandon du modèle atomique de Thomson au profit d’un modèle planétaire où les électrons gravitent autour d’un noyau central, chargé positivement. Ce modèle atomique que nous allons développer fut publié par Rutherford en 1911. Il est considérée comme l’un des acquis fondamentaux de la structure des atomes. y A S

χ

p0

w0 w O

b x

F IG . 15.3.

I . 2 . — Diffusion d’une particule par un centre de force On a vu que le problème de l’interaction entre deux particules A 1 et A2 pouvait se réduire, dans le référentiel du centre de masse R∗ , à un problème entre une particule fictive A et un centre de force O situé à l’origine C de R ∗, centre de masse des deux particules (cf. chapitre 13). Dans le cas fréquent où la particule incidente A1 a une masse négligeable devant celle de la cible A 2, A1 a, dans R∗ , un mouvement identique à celui de A, et A 2 est pratiquement immobile dans R∗ . Considérons une particule A qui s’approche d’un centre de force O avec une quantité de mouvement p0 . Elle passerait rectilignement à la distance b de O si l’interaction n’existait pas : b est appelé le paramètre d’impact (Fig. 15.3). Du fait de l’interaction, la particule est déviée et sa trajectoire devient à nouveau une droite lorsque A s’éloigne de O. On dit que la particule A a été diffusée par O sous l’angle de diffusion x = (p 0, p) que fait la direction émergente définie par la quantité de mouvement p avec la direction

259

Diffusion de Rutherford. Notion de section efficace

incidente. La figure est relative à la force électrostatique répulsive entre deux particules dont les charges électriques sont de même signe. I . 3 . — Diffusion de Rutherford a) Nature de la diffusion On appelle diffusion de Rutherford la diffusion associée à une interaction coulombienne entre deux particules chargées. L’énergie potentielle électrostatique est : Ep =

K r



K = Z 1 Z2

e2 est la constante d’interaction 4pε0

b) Trajectoire hyperbolique Établissons la relation entre l’angle de diffusion x, la vitesse v 0 de la particule incidente et le paramètre d’impact b. Rappelons d’abord que cette trajectoire est une hyperbole dans le cas répulsif (K > 0), ainsi que dans le cas attractif (K < 0) pourvu que v 0 soit suffisamment grand. Par conséquent la trajectoire est une hyperbole d’équation en coordonnées polaires (r, w) (cf. chapitre 12) : 2pEm 1/2 p L2 K ε= où p = et e = 1 + −ε + e cos(w − w0 ) m|K | |K | |K | w 0 étant l’angle correspondant à r minimal. Rappelons que m = m 1m 2/(m 1 +m2 ) est la masse réduite de A, p est le paramètre de la conique, e son excentricité, E m l’énergie totale dans R∗ qui est une constante égale à mv20 /2 et L son moment cinétique constant dans R∗ : L = −mv 0 b ez = −L e z avec L = bp 0 = mv0b L’angle de diffusion x est la valeur de w pour laquelle r est infini. On a donc : ε cos(x − w 0) = e Or, une autre relation entre x et l’angle w0 peut être facilement établie, à partir du schéma de diffusion (Fig. 15.3) : x p x = p − 2(p − w0) d’où w 0 = + 2 2 Il en résulte que : x ε 1 = sin soit e 2 = 1 + 2 e 2 tan (x/2) En égalant les deux expressions de l’excentricité, on trouve :

 c Dunod – Toute reproduction non autorisée est un délit

r=

tan2

x |K | mK 2 K2 = = = 2 2pEm 2EmL 2 4b2E 2m

puisque L2 = m2 v20 b2 = 2mb2 Em . On en déduit : tan

x 2

|K | r0 =± =± 2bE m 2b

avec

r0 =

|K | Em

La grandeur r0 , qui a les dimensions d’une longueur, est la distance pour laquelle l’énergie du système est égale à la valeur absolue de son énergie potentielle : Em =

|K | r0

260

15. Diffusion de Rutherford. Notion de section efficace

Dans le cas répulsif (K > 0), r0 est la distance minimale d’approche du projectile au cours d’une interaction à paramètre d’impact nul. Il est commode d’exprimer les caractéristiques de la trajectoire hyperbolique (p et e) en fonction de r 0 et b : mv 20b2 2b 2 L2 = = p= m|K | |K | r0

et e = 1 + tan

−2

x 2

1/2

1/2

4b2 = 1+ 2 r0

2 = r0

1/2

r2 b + 0 4 2

On peut déduire de ces expressions la distance minimale de la particule au centre de force : rmin =

p b2 = 2 = e−1 (b + r 20/4)1/2 − r0 /2

b2 +

1/2

r20 4

+

r0 2

Pour b = 0, on trouve évidemment rmin = r0 . Notons que l’angle de diffusion est positif pour K > 0 : la trajectoire est une hyperbole qui s’éloigne du centre répulsif (Fig. 15.4a). Il est en revanche négatif pour K < 0 : la trajectoire est une hyperbole qui se rapproche du centre attractif (Fig. 15.4b). On a respectivement : r=

y

p −1 + e cos(w − w0)

et r =

p 1 + e cos(w − w 0)

Diffusion répulsive ( K > 0 )

y

Diffusion attractive ( K < 0 )

A S

χ S

χ eχ

ed O

b

O

b

x A

x

a)

b) F IG . 15.4.

c) Conservation du vecteur de Runge-Lenz La relation entre l’angle de diffusion x et le paramètre d’impact b peut être établie autrement à partir de la conservation du vecteur de Runge-Lenz caractéristique de l’interaction keplérienne (cf. chapitre 12) : R = v × L + K er

En effet, exprimons l’égalité entre le vecteur R0 initial et le vecteur Rd final dirigé suivant la direction diffusée. Il vient, puisque L = −L ez avec L = mv 0b (Fig. 15.4a) : −v0L e x × ez − Kex = −v0 L ed × e z + K e d

soit

v0 L e y − Kex = v 0L e x + K e d

en posant ex = e z × ed. Projetons cette dernière équation suivant l’axe Ox défini par la direction incidente ; on obtient, si K > 0 et donc x > 0 : −K = −v 0L sin x + K cos x

261

Diffusion de Rutherford. Notion de section efficace soit, en remplaçant L par mv0 b : sin x K = v0L 1 + cos x

ce qui s’écrit aussi

tan

x 2

=

K |K | = 2b 2bE m mv0

puisque K > 0. Notons que le vecteur R est colinéaire au vecteur OS, S étant la position de A si sa distance au centre O est minimale ; en effet, lorsque A est en S, le vecteur v × L est aussi colinéaire à er . I . 4 . — Modèle atomique de Rutherford On peut résumer le modèle atomique de Rutherford en trois points : i) L’atome est constitué d’un noyau central de charge positive Z 2 e contenant la presque totalité de la masse, malgré un rayon de l’ordre de 1 fm très faible devant les dimensions de l’atome. C’est le noyau qui est responsable de la diffusion des particules à grand angle. ii) Le noyau produit dans son entourage un champ électrique coulombien d’expression : Z2 e E= er 4pε0 r iii) Les électrons, de charge négative et de masse bien plus faible, sont en mouvement autour du noyau sous l’action des forces d’interaction coulombienne. Ils contribuent à la diffusion à petit angle. Sur la figure 15.5, on a schématisé les différentes diffusions angulaires en fonction du paramètre d’impact.

Faisceau de particules

Atome

F IG . 15.5.

 c Dunod – Toute reproduction non autorisée est un délit

Dans l’expérience initiale de Rutherford, dans laquelle des particules a étaient diffusées par des atomes d’or, K était positif (Fig. 15.4a) : Z1 = 2

Z 2 = 79

d’où

K = 2 × 79 ×

e2 >0 4pε0

En outre, en raison du rapport des masses (ma/mAu = 4/197), la particule fictive A coïncide pratiquement avec le projectile A1 . Pour des particules a, d’énergie Em = 5 MeV, la relation reliant x, et b est la suivante : tan

2, 5 x ≈ 2 b(fm)

puisque

r0 =

K 158 × 9 × 109 × (1, 6 × 10−19 ) 2 = ≈ 50 fm Em 5 × 106 × 1, 6 × 10 −19

On voit que l’angle de diffusion x est faible pour des particules incidentes de grande énergie et pour de grandes valeurs du paramètre d’impact b. Si la diffusion se produit à petit angle x, le résultat précédent devient : |K | |Ep(b)| x≈ = bEm Em En revanche, il prend la valeur p/2 pour b ≈ 2, 5 fm.

262

15. Diffusion de Rutherford. Notion de section efficace

II . — SECTIONS EFFICACES DE DIFFUSION II . 1 . — Définition On fait apparaître naturellement le concept de section efficace de diffusion en considérant un faisceau de particules uniforme et parallèle, arrivant sur un centre de force O (Fig. 15.6). Le faisceau est caractérisé par le vecteur courant particulaire défini par : Jn = nv v nv étant le nombre de particules incidentes par unité de volume et v leur vitesse (cf. Thermodynamique). Le flux de ce vecteur à travers une surface donne l’intensité particulaire I n , c’est-à-dire le nombre de particules qui traversent cette surface par unité de temps : dN = In = Jn · n d S dt S Ainsi, Jn est le nombre de particules qui traversent, par unité de temps, l’unité de surface normale à la direction du faisceau incident. x A

ϕ

Détecteur

χ z

O

y

F IG . 15.6.

a) Section efficace différentielle Évaluons le nombre d 2N de particules diffusées par le centre de force O, pendant la durée élémentaire d t, dans un angle solide élémentaire d V, autour de la direction caractérisée par l’angle de diffusion x et l’angle azimuthal w. Cet angle solide est concrètement défini par le détecteur placé normalement à la direction du faisceau diffusé. Si la surface élémentaire du détecteur, placé normalement à la direction des particules, est d S, l’angle solide élémentaire a pour expression : dV =

dS r| sin x d w| × r| d x| = = | sin x d w d x| 2 r r2

car d V est une quantité positive et les angles des grandeurs algébriques. Le nombre d2 N est proportionnel à la durée d t, à l’angle solide d V et à Jn ; aussi s’intéresse-t-on à la quantité suivante, homogène à une surface par unité d’angle solide : d 2N ds = Jn d t d V dV

soit

In = Jn

ds ds dS d V = Jn dV d V r2

appelée section efficace différentielle ou section efficace par unité d’angle solide, en raison de sa dimension physique ; en unités SI, on l’exprime en m2 . sr −1. Soulignons qu’a priori d s/ d V dépend à la fois des angles x et w. Remarque : La section efficace différentielle est le rapport de deux différentielles de même degré ; ce n’est donc pas une différentielle au sens habituel en mathématiques.

263

Diffusion de Rutherford. Notion de section efficace b) Section efficace totale

On obtient la section efficace totale en sommant la section efficace différentielle sur tous les angles solides : ds dV st = V dV Comme s t est généralement très faible, on l’exprime non pas en m 2 mais en barn : 1 barn = 10−28 m2 . Si le système présente la symétrie de révolution, ce qui est souvent le cas, la section efficace de diffusion ne dépend que de l’angle x. L’intégration suivant w donne donc 2p et la section efficace totale a pour expression : ds st = 2p| sin x d x| x dV Dans la suite, nous nous placerons dans cette hypothèse. II . 2 . — Relation entre la section efficace différentielle et l’interaction Le nombre de particules dN diffusées, par le centre de force O, dans l’angle solide d V, défini par l’espace compris entre les cônes de demi-angle x et x + d x, est le nombre de particules qui ont traversé, pendant la même durée élémentaire d t, l’anneau de rayon intérieur b et de rayon extérieur b + d b. Il vaut d2 N = Jn 2pb | d b| d t. Par conséquent : ds 2pb | d b| d 2N = = dV d t d V dV Jn

soit

ds db b = dV | sin x| d x

Nous allons calculer les sections efficaces différentielles de diffusion que donnent deux modèles efficaces de diffusion : le modèle de la sphère dure et le modèle de Rutherford. II . 3 . — Sections efficaces associées au modèle de la sphère dure  c Dunod – Toute reproduction non autorisée est un délit

a) Modèle de la sphère dure Un modèle d’interaction particulièrement simple est celui de la « sphère dure », c’est-à-dire que Ep = 0 pour r > R et Ep est infini pour r < R (Fig. 15.7a). On comprend l’origine de l’expression : l’interaction entre une particule et une sphère impénétrable immobile est caractérisée par une telle énergie potentielle. Si le paramètre d’impact b est supérieur à R, il n’y a pas d’interaction. Si b est compris entre 0 et R la particule A est diffusé par la sphère ; le cas où b est nul correspond à la collision de plein fouet (Fig. 15.7b). Ep p0 b

O R a)

r

R b) F IG . 15.7.

264

15. Diffusion de Rutherford. Notion de section efficace

b) Section efficace différentielle Supposons que le faisceau de particules tombe sur une sphère dure immobile de rayon R. Si la collision est élastique, les valeurs absolues des angles incident et réfléchi sont égales (cf. chapitre 14). La relation entre l’angle de diffusion x et le paramètre d’impact b, défini à partir du centre de la sphère, est alors facile à établir ; en effet, on voit, à l’aide de la figure 15.8, que : p−x x x b R = sin = cos donne d b = − sin dx 2 2 2 2 R en différentiant. On en déduit : db x b R 2 cos(x/2) = sin | sin x| d x 2 sin x 2

d’où

ds dV

= sd

R2 4

Cette section efficace différentielle est donc indépendante de l’angle de diffusion : elle est isotrope. χ

χ + dχ

R

b + db

b

π −χ 2 F IG . 15.8.

c) Section efficace totale Comme (d s/ d V) sd = R2/4, il vient : R2 R2 dV = V soit st = pR2 4 4 Ainsi, la section efficace totale de collision est, dans le modèle de diffusion de la sphère dure, égale à l’aire de la projection de la sphère dans un plan perpendiculaire à la direction du faisceau incident. Cet exemple simple montre que la section efficace est reliée à la surface présentée par la cible. st =

ds =

Nous n’avons considéré que le cas de particules ponctuelles heurtant une sphère immobile de rayon R. Si la particule incidente est une sphère de rayon R 1 diffusée par une sphère immobile de rayon R2 , l’énergie potentielle d’interaction caractéristique a la même forme que précédemment, mais : Il en résulte que :

Ep = 0

pour

r > R1 + R 2 ds dV

et Ep = ∞ pour =

sd

r < R1 + R 2

(R 1 + R2 )2 4

La section efficace totale a alors pour expression : s t = p(R 1 + R 2) 2. II . 4 . — Section efficace de Rutherford a) Section efficace différentielle Le calcul de la section efficace de diffusion de Rutherford joue un rôle important puisque l’interaction coulombienne est celle qui prévaut dans le cas des particules chargées. Rappelons la relation entre

265

Diffusion de Rutherford. Notion de section efficace le paramètre d’impact et l’angle de diffusion x : x r0 = 2 2b

tan Par conséquent, en différentiant, on obtient :

db 1 b b3 = sin x d x r0 | sin x| cos2 x/2 R En remplaçant b par son expression en fonction de tan(x/2), on trouve finalement : dx −r 0 d b = 2 2 cos (x/2) 2b2

et

ds dV

ds dV

= R

=

r0 4

2

1 sin (x/2) 4

Cette section efficace différentielle varie fortement avec x. Notons que, pour les faibles valeurs de x, l’expression précédente ne convient pas puisqu’elle donne une valeur infinie ; ce résultat inacceptable doit être attribué à l’insuffisance du modèle coulombien pour décrire la réalité. Dans ce cas, on évite cette difficulté en remplaçant l’énergie potentielle en 1/r par une énergie potentielle d’interaction écrantée de la forme : Ep =

K r exp − r a

La distance a, appelée distance d’écran, décrit l’influence du nuage électronique qui entoure le noyau. Cette analyse plus fine conduit finalement à substituer, à l’expression précédente de la section efficace différentielle, une autre légèrement différente qui contient un facteur d’écran h lié à la distance a (Fig. 15.9) : ds dV

= R,e

r0 4

1

2

sin 2 (x/2 + h)

2

On l’appelle la section efficace différentielle écrantée de Rutherford.

 c Dunod – Toute reproduction non autorisée est un délit

d¾ dΩ

0

Â

F IG . 15.9.

b) Section efficace totale En intégrant (d s/ d V) R , on obtient : st =

r0 4

2

1 2p| sin x d x| 4 sin (x/2)

soit

s t = 4p

r0 4

2

cos(x/2) dx sin 3(x/2)

Évidemment, cette expression ne convient pas si l’intégration contient la valeur nulle de x. On doit dans ce cas lui substituer la valeur que l’on obtient de façon plus laborieuse à partir de la section efficace différentielle écrantée.

266

15. Diffusion de Rutherford. Notion de section efficace

III . — LIBRE PARCOURS MOYEN III . 1 . — Traversée d’un matériau par un faisceau de particules Lorsqu’une particule chargée traverse un matériau, elle subit des collisions successives élastiques et inélastiques. Les premières écartent la particule de sa direction initiale sans changer son énergie, les secondes la freinent en diminuant son énergie. Ce freinage est dû essentiellement à l’interaction entre particules incidentes et électrons atomiques et non à l’interaction avec les noyaux lourds qui provoquent seulement un changement de direction des particules incidentes. a) Aspect expérimental La traversée de divers matériaux par des particules chargées fut étudiée dès 1890 par le physicien hongrois P. Lenard, avec des électrons sur de l’aluminium. Lenard utilisa un dispositif tel que celui représenté sur la figure 15.11 : les électrons produits par une tension électrique de 10 kV aux extrémités d’un tube contenant un gaz raréfié traversent une feuille mince d’aluminium ( 5 mm d’épaisseur) puis sont détectés sur un écran fluorescent placé à quelques centimètres plus loin. Des versions plus précises et plus récentes de cette expérience, ont été conduites sur des microscopes électroniques par transmission. Feuille d’aluminium

Gaz raréfié

x

O − +

x + dx

Courant particulaire

Écran fluorescent

Détecteur

Lame

Pompe à vide F IG . 15.10.

F IG . 15.11.

b) Atténuation d’un flux de particules à la traversée d’un matériau Considérons un faisceau de particules tombant sur une lame d’un matériau, normalement à sa surface (Fig. 15.12). Du fait des collisions, seule une partie des particules incidentes est diffusée et donc déviée de la direction incidente Ox. Comparons les nombres de particules reçues, pendant la même durée élémentaire d t, par deux détecteurs identiques, de surface s faible, l’un placé en x et l’autre en x + d x. Il vient, en désignant par Fn (x) le flux de particules : d N (x) = Fn (x) d t

et

d N (x + d x) = Fn(x + d x) d t

La différence représente le nombre de particules d N d diffusées par le nombre de centres diffuseurs rencontrés, lequel vaut : nv,d s d x, n v,d étant le nombre de ces diffuseurs par unité de volume. Le nombre de particules diffusées est proportionnel à d t, à l’épaisseur d x, à n v,d, au flux F n(x) et à la section efficace totale st . On a donc : d N (x) − d N (x + d x) = d t nv,d d x Fn st

d’où



d Fn d x d t = d t nv,d d x F n st dx

On en déduit :

d Fn = −nv,d st d x Fn Supposons que la quantité L = (nv,d st ) −1 , homogène à une longueur, ne dépende pas de x, ce qui est réalisé si le milieu est homogène. L’intégration donne : ln

Fn Cte

=−

x L

soit

Fn = Cte × exp −

x L

267

Diffusion de Rutherford. Notion de section efficace Si F n(0) désigne le flux particulaire en x = 0, la relation précédente s’écrit : F n(x) = Fn (0) exp −

x L

avec

L=

1 nv,dst

À partir du graphe Fn (x) (Fig. 15.12) il est possible de déduire L et donc la section efficace de diffusion st , si l’on connaît n v,d : ln

x F n(0) = x nv,d st = F n(x) L

Fn (0)

Fn (x) Fn (x)

O

L F IG . 15.12.

x

Précisons que s t = (nv,d L)−1 ainsi déterminé représente la section efficace totale sommée sur tous les types de collisions se produisant dans le matériau. Remarque : Si la lame est épaisse, la mesure de Fn prend en compte la diffusion multiple, qui, après plusieurs collisions, peut ramener la particule incidente dans la direction Ox. Il convient donc, pour éviter cette diffusion multiple, de travailler avec des échantillons suffisamment minces et d’extrapoler la courbe donnant F n(x) pour les faibles valeurs de x.

 c Dunod – Toute reproduction non autorisée est un délit

III . 2 . — Libre parcours moyen On appelle libre parcours moyen  associé à la diffusion d’un faisceau de particules par un matériau, la distance qui sépare en moyenne deux collisions successives d’une particule du faisceau par les centres diffuseurs du matériau. D’après la loi de décroissance exponentielle précédente, les N(0) particules, que recevrait le détecteur placé en x = 0, subissent des collisions avec les centres diffuseurs du matériau, après des distances parcourues différentes comprises entre 0 et l’infini. Par définition du libre parcours moyen, tout se passe comme si toutes les particules présentes sur l’axe, en x = 0, subissaient une collision, avec un centre diffuseur, après avoir parcouru la distance . Le graphe correspondant Fn (x) est donc constitué de deux portions de droite (Fig. 15.12) : Fn(x) = F n (0)

pour

x <  et Fn (x) = 0

pour

x>

La valeur de  est alors obtenue en écrivant que le nombre de particules qui atteindraient le détecteur ne change pas : ∞ 0

F n(x) d x =

∞ 0





Fn (x) d x

d’où

0

F n(0) d x =

0

Fn (0) exp −

x L

dx

268

15. Diffusion de Rutherford. Notion de section efficace

On obtient, en simplifiant par Fn (0) et en intégrant : ∞

=L 0

exp −

dx x = L − exp − L L

x L

∞ 0

=L

Ainsi, le libre parcours moyen  est relié à la section efficace totale par l’équation : =

1 nv,d st

Dans le cas d’un élément de nombre de masse A, de masse volumique r, la relation entre r, n v,d et A est simple : nv,d = NA r/A, NA étant le nombre de Avogadro. Pour des électrons d’énergie cinétique 0, 1 MeV traversant une feuille d’aluminium (A = 27, r = 2 700 kg . m−3 ), on a trouvé expérimentalement  = 66 nm, ce qui permet d’en déduire la section efficace totale : st = 2, 5 × 10 6 barn. Remarque : La formule précédente n’est valable que pour des centres diffuseurs fixes. En thermodynamique statistique, dans l’étude des gaz, on introduit, de façon analogue, un libre parcours moyen de particules, mais dans ce cas les centres diffuseurs sont eux-aussi en mouvement ; la formule reliant le libre parcours moyen √ et la section efficace totale diffère alors légèrement par un facteur numérique égal à 2 (cf. Thermodynamique).

IV . — APPLICATION AU POUVOIR D’ARRÊT DES MATÉRIAUX L’énergie cédée aux électrons atomiques par une particule chargée qui traverse un matériau, durant un trajet élémentaire de longueur d x, est proportionnelle à la perte d’énergie Q relative à une collision et au nombre de collisions subies. Ce nombre est égal au nombre de centres diffuseurs contenus dans le cylindre droit, de longueur d x, et de section égale à la section efficace de diffusion ; il vaut donc n d d s d x. Il en résulte que la perte d’énergie − d E , au cours du déplacement élémentaire d x, s’obtient en sommant sur toutes les valeurs de Q : −dE =

Q nv,d d s d x

Pour calculer cette intégrale, il faut exprimer d s en fonction de Q : rappelons l’expression de l’énergie cédée par un projectile de masse m 1 à une particule cible de masse m2 (cf. chapitre 14) : Q=

4m1m2 sin 2 (m1 + m2)2

x 2

m 1v20 2

soit, puisque dans le cas considéré m 1 = m 2 = m e et m = m e /2 : Q=

1 m e v20 sin 2 2

Par conséquent : 1 x d Q = m e v02 sin 2 2 et d sR =

cos

x 2

x m ev 20 dx = sin x d x 2 4

1 2pK 2 d Q K2 2 p sin x d x = m e v 20 sin 4 (x/2) m e v02 Q2

269

Diffusion de Rutherford. Notion de section efficace Il en résulte, en introduisant q2e = e2/(4pε 0 ) : −

dE = n v,d dx

Q dsR =

2pq4e nv,d m ev 20

Q max Qmin

Q max d Q 2pq 4e nv,d = ln Q Qmin me v 20

Les bornes d’intégration, Qmax et Q min , représentent les pertes d’énergie maximale et minimale au cours d’une collision. La contribution des grandes valeurs de Q est moins forte que celle des petites valeurs, d’où le soin particulier qu’il convient d’apporter à la détermination de Q min . Alors que Q max s’obtient aisément en remplaçant, dans l’expression de Q, x/2 par p/2, la détermination de Q min exige que l’on abandonne le modèle simplifié d’une cible constituée par un électron atomique libre. Une analyse plus fine montrerait que Qmin doit être pris égal à I 2 /(2me v 2 ), I étant une énergie associée à l’ensemble des états des électrons atomiques appelée « potentiel moyen d’ionisation ».

CONCLUSION Résumons les point importants. (1) L’angle de diffusion de Rutherford et la section efficace différentielle correspondante ont pour expressions respectives : tan

x |K | |K | = = 2 2bE mv 02b

ds dV

et

= R

1 K2 4 4 2 4m v0 sin (x/2)

(2) La section efficace totale dans le cas du modèle des sphères dures est : s t = pR 2. (3) Enfin, le libre parcours moyen associé à un processus de diffusion par des centres diffuseurs fixes est relié à la section efficace par l’équation : =

1 nv,d st

 c Dunod – Toute reproduction non autorisée est un délit

où nv,d est le nombre de centres diffuseurs par unité de volume. Les exemples donnés et l’application au pouvoir d’arrêt des matériaux soulignent l’actualité et l’intérêt d’un tel sujet. Ces notions se développent et se généralisent naturellement en relativité et en quantique. On les retrouve d’ailleurs en thermodynamique dans l’interprétation microscopique des propriétés macroscopiques des gaz (cf. Thermodynamique).

EXERCICES ET PROBLÈMES P15– 1. Diffusion d’un proton par un noyau lourd Un proton A 1 (masse m1 ) venant de l’infini, s’approche d’un noyau lourd immobile A 2 (masse m2, nombre de protons Z) avec un paramètre d’impact b. 1. La quantité de mouvement du système est-elle conservée ? Commenter. 2. Exprimer le moment cinétique du système ainsi que son énergie, lorsque le proton est à l’infini (vitesse v0) et lorsqu’il est en S, point où la distance au noyau est minimale (vitesse v s). 3. En déduire l’expression de la distance minimale s entre le proton et le noyau, en fonction du rapport r0 de b sur l’énergie E m du proton. Application au cas où b = 1 pm, Em = 0, 1 MeV et Z = 82.

270

15. Diffusion de Rutherford. Notion de section efficace

P15– 2. Diffusion d’un noyau d’hélium par un noyau de cuivre Un noyau d’hélium, de vitesse v 0 = 106 m . s −1 , est diffusé par un noyau de cuivre immobile, sous un angle x = p/2 dans le référentiel du centre de masse R ∗ . Calculer la constante d’interaction K et le paramètre d’impact b, en pm, en supposant que la diffusion est de type Rutherford. P15– 3. Diffusion de particules a par une feuille d’argent Dans l’expérience de diffusion de Rutherford, les particules a ont une énergie cinétique E k,1 = 10 MeV. Calculer les paramètres d’impact correspondant aux angles de diffusion suivants : 2 ◦, 20◦ et 85 ◦. P15– 4. Sections efficaces associées à différentes excitations Les libres parcours moyens associés à la diffusion d’électrons incidents, à la suite de créations de plasmons et d’excitations de transitions atomiques individuelles K, ont été déterminés expérimentalement dans de l’aluminium. Les résultats ont été les suivants : p = 314 nm, K = 100 mm. En déduire les valeurs en barn des sections efficaces correspondantes, sachant que le nombre d’électrons participant à l’excitation est de trois par atome pour la création des plasmons et de deux par atome pour l’excitation individuelle K. P15– 5. Expression de la section de Rutherford On considère la diffusion d’électrons d’énergie E 1 par des atomes cibles.

1. Donner l’expression de la section efficace différentielle de diffusion de Rutherford, par les noyaux, en fonction de l’angle de diffusion u1 des électrons incidents. En déduire le rapport du flux d’électrons diffusés, dans l’angle solide élémentaire 2p sin u 1 d u1 , sur la densité surfacique du flux d’électrons incidents. 2. Même question si la diffusion est produite par les électrons atomiques. P15– 6. Détection de particules a diffusées par de l’aluminium Un faisceau de particules a, de densité surfacique de flux J n = 5 × 108 m −2 . s−1 , est diffusé sous un angle droit par une cible en aluminium, de masse 30 mg. Les particules a, d’énergie E m = 12, 5 keV, sont, après diffusion, reçues par un détecteur placé à 50 cm de la cible. 1. Calculer le paramètre d’impact et la distance minimale d’approche des noyaux par les particules. 2. Quel est le flux des particules qui devraient atteindre la section du détecteur, d’aire A = 2, 5 cm 2 , selon le modèle de sphères dures de rayon égal à s ? On rappelle que le nombre de nucléons de l’aluminium est 27 et que la masse d’un nucléon vaut mn ≈ 1, 67 × 10−27 kg. P15– 7. Détection d’électrons diffusés par une feuille métallique On bombarde une feuille mince métallique à l’aide d’un faisceau d’électrons uniforme, parallèle et monoénergétique d’énergie cinétique E1 (Fig. 15.13). Les particules incidentes subissent une diffusion de type Rutherford avec les électrons atomiques de la feuille. On désigne par In l’intensité particulaire du faisceau incident ; nv,d est le nombre d’électrons diffuseurs par unité de volume, e l’épaisseur uniforme de la feuille et S sa section.

271

Diffusion de Rutherford. Notion de section efficace

Quel est, par unité de temps, le nombre d’électrons diffusés que reçoit un détecteur annulaire, de rayon r, de largeur faible w et tel que chacun de ses points est situé à une distance L du centre de diffusion ? w L

F IG . 15.13.

P15– 8. Atténuation d’un faisceau d’électrons Un faisceau d’électrons traverse une cible contenant n v = 8 × 1028 atomes . m−3 . On suppose que chaque atome se comporte comme une sphère dure de rayon R. Sachant que, pour une épaisseur x1 = 15 mm de la cible, seulement 1% des électrons n’ont pas été diffusés, trouver la valeur de R en pm. En déduire la section efficace totale et le libre parcours moyen correspondant ? P15– 9. Diffusion et rayonnement d’une particule chargée Une particule chargée A (masse m) est soumise, de la part d’un centre de force O, situé à l’origine d’un référentiel galiléen R = Oxyz, à une force attractive de la forme F = (K/r 2) e r, K étant une constante négative et er le vecteur unitaire défini par le vecteur position OA. 1. Quelles sont les trois constantes du mouvement ?

 c Dunod – Toute reproduction non autorisée est un délit

2. La particule est, en outre, soumise à une force de freinage de la forme F f = mt d a/ d t due au rayonnement électromagnétique qu’elle émet, t étant une durée caractéristique. Quelle est l’équation différentielle vectorielle à laquelle doit satisfaire le moment cinétique LO, en O, si F f  F ? Dans le cas où, en l’absence de F f , la trajectoire serait un cercle de rayon r 0, trouver r(t) sachant que r = r0 à t = 0.

16 Cinématique du solide et des solides en contact La cinématique du solide, précisément du corps rigide, c’est-à-dire d’un système indéformable, tient une place importante dans les applications quotidiennes de la mécanique, en raison de la large validité de l’approximation que constitue le solide rigide. Son intérêt dépasse largement le cadre du seul solide puisque la plupart des mécanismes utilisés dans la vie courante sont constitués de solides en contact.

I . — CINÉMATIQUE DU SOLIDE Dans la suite, on désignera brièvement par solide S un ensemble de points dont les distances mutuelles ne varient pas au cours du temps ; S est donc un corps rigide ou indéformable. Ajoutons qu’il importe peu qu’un tel solide soit rempli ou non de matière : trois points quelconques, situés à des distances mutuelles invariables, définissent le solide S . Évidemment, cette définition géométrique est réductrice, car le solide est d’abord un état de la matière qui ne se réduit pas à la schématisation précédente ; il se déforme s’il est soumis à une contrainte ou si sa température augmente. On le représente plus précisément par un ensemble d’oscillateurs dont les positions moyennes restent fixes les unes par rapport aux autres et sont régulièrement disposés dans l’espace. Cependant, pour étudier le mouvement de S par rapport à un référentiel R, on lui substituera un repère d’espace R = O  x yz , invariablement lié à S (Fig. 16.1). Comme trois points A , B , C de S ou de R  , non alignés, suffisent pour déterminer la position du solide, S possède 6 degrés de liberté, c’est-à-dire 6 paramètres de position indépendants qu’il est nécessaire de connaître pour déterminer sa position par rapport à R (cf. chapitre 2). z

z S

O x

y

O  R y x

R F IG . 16.1.

273

Cinématique du solide et des solides en contact I . 1 . — Champ des vitesses d’un solide

Considérons deux points quelconques A et B invariablement liés à un solide S. La norme du vecteur AB étant une constante au cours du mouvement, il vient, relativement à R : d(AB2 ) d AB d(AB2 ) = 0 soit AB · = AB · (vB/R − vA/R ) = 0 = dt dt dt Ainsi, les vitesses de deux points d’un solide satisfont à la propriété d’équiprojectivité des champs de vecteurs antisymétriques (cf. chapitre 1) : AB · v A/R = AB · v B/R On montre que cette propriété implique un champ des vitesses de solide de la forme : v B/R = v A/R + BA × VS/R

ou

vB/R = vA/R + V S/R × AB

VS/R étant le vecteur vitesse de rotation du solide S par rapport à R, noté parfois simplement V s’il n’y a aucune confusion possible. Le champ des vitesses d’un solide est donc le moment du torseur [V, vA/R] (cf. chapitre 1). Remarque : La relation précédente s’écrit aussi, pour deux points voisins A et A : v(A ) − v(A) = V × AA

soit

dv = V ×dr

L’opérateur V× qui agit sur la différentielle d r est un tenseur, d’ordre 2, car d v et d r ne sont pas colinéaires ; il s’explicite dans une base orthonormée selon une matrice carrée à 9 éléments dont la diagonale est nulle (cf. chapitre 2). Cet opérateur est parfois appelé tenseur-gradient du vecteur v , par analogie avec le gradient d’une fonction scalaire f : d v = t-grad v · d r

d f = grad f · d r

comme

I . 2 . — Différents mouvements d’un solide

 c Dunod – Toute reproduction non autorisée est un délit

a) Translation d’un solide Un solide S a un mouvement de translation par rapport à un référentiel R si le vecteur construit à partir de tout couple ordonné de deux points A et B de S reste équipollent à lui-même au cours du mouvement (Fig. 16.2). Le vecteur AB est donc constant dans R. Par conséquent : d AB dt

=0

d’où

vB/R = vA/R

et

VS/R = 0

R

Ainsi, tous les points de S ont même vitesse et donc même accélération par rapport à R ; cependant, cette même vitesse et cette même accélération peuvent évoluer de façon quelconque. b) Rotation d’un solide autour d’un axe Dans le mouvement de rotation d’un solide S autour de l’axe Oz de R, les points situés sur cet axe ont une vitesse nulle (Fig. 16.3). La vitesse d’un point A de S s’écrit donc : vA/R = AO × VS/R

ou

v A/R = V × OA

274

16. Cinématique du solide et des solides en contact z

t2

t1 B S A

O

z

B S A y

t3 B

S

H

vA et

Ω

A

y

O '

x

A

R

x

F IG . 16.2.

P R

F IG . 16.3.

D’autre part, comme A décrit un cercle de rayon AH, H étant la projection de A sur Oz, sa vitesse est v A/R = (d s/ d t) et = AH w˙ e t . La comparaison des deux expressions de vA/R montre que V est un vecteur parallèle à l’axe de rotation Oz dont l’orientation est donnée par la règle du tire-bouchon (cf. chapitre 2) ; sa mesure suivant Oz est w˙ : V = w˙ e z Si w˙ > 0, la rotation a lieu dans le sens direct Ox → Oy ; si w˙ < 0, la rotation a lieu dans le sens inverse Oy → Ox.

Exemple : La Lune peut être considérée comme un solide ayant un mouvement de rotation, autour de son axe de révolution, caractérisé par le vecteur vitesse angulaire de rotation VL (Fig. 16.4). En outre, son centre décrit une trajectoire sensiblement circulaire autour de la Terre, avec une période sidérale de TL = 27, 3 jours (cf. Exercices, chapitre 3). Ce mouvement est remarquable car cette période de révolution TL coïncide avec la période de rotation 2p/V L, de sorte que la Lune présente toujours la même face à la Terre. z ΩL

S

Lune

Terre

vH

vA

H

vP

A

Ω O x

F IG . 16.4.

P

y

F IG . 16.5.

c) Mouvement hélicoïdal d’un solide Considérons un solide S dont le mouvement est tel que tout point A de S tourne autour d’un axe Oz et, en même temps, se déplace suivant cet axe. Comme OA = OH + OP (Fig. 16.5), on a, par rapport à R : vA/R = vH/R + vP/R avec vH/R = v H ez Or la vitesse de P, projection de A sur Oxy, s’écrit : v P/R = VS/R × OP = V × HA. Il en résulte : vA/R = v H/R + AH × V S/R

275

Cinématique du solide et des solides en contact d) Mouvement le plus général d’un solide

On a vu que, quels que soient les points A et B d’un solide S, on avait toujours, par rapport au référentiel R : vB/R = v A/R + BA × V Cherchons l’ensemble des points H tels qu’à un instant donné vH/R soit parallèle à V : vH/R = lV = vA/R + HA × V Pour simplifier, explicitons cette relation dans une base, tel que V soit porté par l’axe Oz. Il vient : 0 0 lV

0 x˙A xA − x H = y˙A + yA − y H × 0 V z˙A zA − z H

soit

yH = y A + x˙ A/v xH = x A − y˙ A/v V = ˙z A/l

On obtient donc une droite parallèle à VS/R , appelée axe central. Il en résulte, H étant un point de l’axe central D : vA/R = lV + AH × V i) Si, à un instant t, V = 0, le mouvement est dit tangent à un mouvement de translation. ii) Si, à un instant t, V = 0 et v H/R = 0, il est dit tangent à une rotation d’axe D. iii) Si, à un instant t, V = 0 et v H/R = 0, il est dit tangent à un mouvement hélicoïdal d’axe instantané de rotation D. e) Angles d’Euler

 c Dunod – Toute reproduction non autorisée est un délit

Si un solide S a un point fixe O dans R, sa position par rapport à R dépend de trois paramètres indépendants qui sont ses degrés de liberté. Rappelons que l’on appelle ainsi le nombre de paramètres qu’il faut se donner pour déterminer la position d’un système. Il est commode d’associer au solide le repère R = Ox y z. Il est commode de prendre comme paramètres les angles d’Euler c, u, f, définis comme suit (Fig. 16.6). Notant Ou l’intersection des plans Oxy et Ox y , appelée ligne des nœuds, c = (Ox, Ou) est l’angle de précession, u = (Oz, Oz) l’angle de nutation et f = (Ou, Ox ) l’angle de rotation propre. z θ

w

y

z

φ θ

S

v ψ

O x

ψ

φ u

y

x

F IG . 16.6.

Remarque : Dans les ouvrages anglo-saxons, l’angle de précession est généralement noté f et l’angle de rotation propre c.

276

16. Cinématique du solide et des solides en contact

Afin de faciliter l’interprétation physique de ces angles c, u, f, représentons le solide S par une toupie. Le vecteur vitesse de rotation de S par rapport à R est la somme vectorielle des trois vecteurs vitesses de rotation. On sait en effet que les vecteurs vitesses de rotation s’ajoutent vectoriellement (cf. chapitre 3) : V = c˙ e z + u˙ e u + f˙ ez L’obtention des expressions de V S/R , en fonction des angles d’Euler, dans les bases liées à R et R  , n’est pas immédiate. En effet, dans les deux cas, un seul des trois vecteurs unitaires e u, e z ou e z est vecteur de base de R et R. Exprimons e u et e z , en fonction des vecteurs de base de R (Fig. 16.7) : eu = e x cos c + ey sin c

et ez = ez cos u − ev sin u

avec

ev = −e x sin c + ey cos c

On en déduit les composantes du vecteur V dans la base de R selon :

R

u˙ cos c + f˙ sin c sin u u˙ sin c − f˙ cos c sin u f˙ cos u + c˙

y

z ez

eu

ev ψ

θ

y

ψ O

O

x

O

v

x

F IG . 16.7.

Pour avoir les composantes de V dans la base de R  , exprimons eu et e z en fonction des vecteurs de base de R (Fig. 16.8) : eu = e x  cos f − ey  sin f

et e z = e w sin u + ez  cos u

On obtient :

R

y O

ew = ex sin f + ey  cos f

˙ sin u sin f u˙ cos f + c − ˙u sin f +c˙ sin u cos f ˙ c˙ cos u + f z

φ

avec

θ

y

ez

O

φ

ew

O

x

w

eu

x

F IG . 16.8.

Si le solide présente la symétrie de révolution, autour de l’axe Oz  par exemple, il est commode d’utiliser la base (eu , ew , ez ), dite base de Resal Re (du nom de l’ingénieur français H. Resal). Il suffit d’exprimer e z dans cette base. Comme on a : e z = ew sin u + e z cos u, V s’explicite dans la base de Resal selon : u˙ c˙ sin u ˙ cos u + f ˙ Re c

277

Cinématique du solide et des solides en contact

Remarque : On aura besoin ultérieurement de distinguer VS/R et le vecteur de rotation V Re/R de la base de Resal par rapport à R, lequel vaut : ˙ ez VRe /R = u˙ eu + c˙ ez = VS/R − f De ce qui précède, on déduit aisément les différentes projections de V Re /R dans les différentes bases de R, R  et R e . Il suffit de supprimer le terme en f˙ = 0 dans VS/R .

II . — CINÉMATIQUE DES SOLIDES EN CONTACT II . 1 . — Vitesse de glissement Considérons deux solides S 1 et S2 en mouvement par rapport à un référentiel R de telle sorte que leurs surfaces soient en contact. Pour simplifier, supposons que le contact soit ponctuel (Fig. 16.9a). Nous devons distinguer au contact de S1 et S 2 trois points : i) le point géométrique I de contact,

ii) le point I 1 du solide S 1 qui coïncide avec I à l’instant considéré, iii) le point I 2 du solide S 2 qui coïncide avec I au même instant.

I y

O x

I

S1

z I1

I1

I2

I2

R a)

S2

Q b) F IG . 16.9.

 c Dunod – Toute reproduction non autorisée est un délit

Un exemple concret de cette vitesse de glissement est fourni par le contact des deux roues en rotation (Fig. 16.9b) : le point géométrique est fixe alors que les points I 1 et I2 , qui appartiennent respectivement aux roues 1 et 2, ont des vitesses a priori non nulles et différentes. Par définition, on appelle vitesse de glissement de S 1 sur S 2, notée v g ou v 1/2 , la vitesse de I1 par rapport à S 2 : v1/2 = vI1 /S2 D’après la composition des mouvements de I1 entre les référentiels R et S2, on a : vI 1/R = v I1 /S2 + ve = v I 1/S2 + vI2/R puisque ve est ici la vitesse du point I2 de S 2 qui coïncide avec I (cf. chapitre 3). Retenons donc : v1/2 = vI 1/S 2 = v I 1/R − v I2 /R Remarque : Notons que la vitesse du glissement ne dépend que des solides en contact ; elle est donc indépendante du référentiel par rapport auquel S 1 et S 2 sont en mouvement.

278

16. Cinématique du solide et des solides en contact

La vitesse de glissement est contenue dans le plan tangent Q commun à S 1 et S 2. Pour le montrer, exprimons de deux manières la vitesse du point géométrique I par rapport à R , en composant les vitesses successivement avec S1 et S 2 . Il vient, d’après la définition de la vitesse d’entraînement : v I/R = v I/S2 + vI2 /R = v I/S1 + vI 1/R

d’où

v 1/2 = vI/S 2 − vI/S 1

Comme vI/S 1 et vI/S2 appartiennent tous deux au plan tangent, il en est de même pour la vitesse de glissement. Il n’y a pas de glissement si v 1/2 = 0. Notons que, dans le cas particulier où S 2 est fixe par rapport à R (vI2 /R = 0), l’absence de glissement se traduit par : v I 1/R = 0. II . 2 . — Exemples a) Disque vertical en contact avec un plan Exprimons la vitesse de glissement v 1/2, d’un disque vertical évoluant sur un axe d’un plan, en fonction des deux paramètres xK , coordonnée du centre K du disque et u , angle de rotation (Fig. 16.10). Désignant par R le référentiel lié au plan, il vient, avec les notations de la figure : v1/2 = v I1/R − vI 2 /R soit, puisque les points I1 et K appartiennent au même solide et vI2 /R = 0 : v1/2 = vI1/R = v K/R + I 1 K × V1/R en désignant par V1/R le vecteur vitesse de rotation de S 1 par rapport à R. y

K

r O

R

O

y

R x

R

I1 θ I 2 xC

K

µ

S1

I1

®



y

Á I2 S2 x



x F IG . 16.11.

F IG . 16.10.

Comme v K/R = ˙xK ex et V1/R = u˙ e z, la vitesse de glissement s’explicite dans la base de R selon : 0 0 x˙K x˙K + r u˙ 0 = 0 + r × 0 ˙ 0 0 0 u R R R R Dans le cas du roulement sans glissement : v1/2 = 0

d’où

x˙K + r u˙ = 0

279

Cinématique du solide et des solides en contact

Par intégration, on obtient DxK = −r Du, ce qui exprime, au signe près, l’égalité des longueurs parcourues par le point géométrique I sur le disque et sur la droite ; le signe moins traduit seulement la diminution de l’angle u lorsque l’abscisse de K augmente. b) Disque en contact ponctuel avec un guide circulaire en mouvement Exprimons la vitesse de glissement v 1/2 du disque en contact avec le guide, en fonction des trois paramètres, a, u et f, représentés sur la figure 16.11 ; u et f fixent la position d’un point du disque S1 et a celle du guide S 2 par rapport au référentiel R . On a v 1/2 = vI 1 /R − vI2 /R où vI1 /R = v K/R + V1 × KI1

et vI 2/R = vO/R + V2 × OI2 = V2 × OI2

puisque, d’une part, I1 et K appartiennent au solide S 1, d’autre part, I 2 et O appartiennent au solide S2 ; V1 et V2 sont les vecteurs vitesse de rotation des solides S1 et S 2 par rapport à R. Avant d’expliciter v 1/2 dans la base tournante (e x  , ey , e z), il convient de noter que la vitesse de rotation du disque S1 par rapport à R a pour expression : ˙ ez V1 = V1/R  + VR /R = (u˙ + f) ˙ ez est le vecteur vitesse de rotation du disque par rapport à R et u˙ e z est celui de la tige puisque f immatérielle OK, liée à R par rapport à R. Quant à V 2 = a˙ e z il représente le vecteur vitesse de rotation du guide par rapport à R. Cela dit, v1/2 s’explicite dans la base de R selon :

R

0 0× u˙

R

0 R−r r 0 + 0 0 × ˙ ˙   0 ( u + f) R R 0



R

0 0 R ˙ ˙ − Ra˙ 0 × 0 = (R − r)u + r( u˙ + f) a˙ 0 R 0 R

Ainsi : ˙ − Ra) v = (R ˙u + rf ˙ ey

 c Dunod – Toute reproduction non autorisée est un délit

˙ = r f. ˙ Si le guide est fixe par rapport à R (a˙ = 0), S’il n’y a pas de glissement, v = 0 d’où R(a˙ − u) ˙ la condition précédente se réduit à Ru˙ = −r f. II . 3 . — Roulement avec pivotement Par rapport au solide S 2, la vitesse d’un point quelconque A 1 de S 1 s’exprime en fonction de celle du point I1, appartenant à S 1, qui coïncide avec I , et de la vitesse de rotation V1/2 de S 1 par rapport à S2 (Fig. 16.12a) : v A1 /S2 = vI1 /S2 + A1I × V1/2 = v 1/2 + A 1I × V1/2 Le vecteur V1/2 peut être considéré comme la somme : i) d’un vecteur V t , situé dans le plan tangent Q commun aux deux solides en I , appelé vitesse angulaire de roulement, ii) et d’un vecteur V n, normal à Q, appelé vitesse angulaire de pivotement. ,

Sur la figure 16.12b, on a représenté les vecteurs roulement et pivotement de la roue avant d’une bicyclette sur le sol.

280

16. Cinématique du solide et des solides en contact

Ωn

Q

Ω1/2

A1

S1 I

Ωt

S2

Pivotement Ω n

Roulement Ωt a)

b) F IG . 16.12.

II . 4 . — Application au différentiel d’un véhicule Le différentiel d’un véhicule est un système mécanique qui permet aux roues d’un même essieu de parcourir des longueurs différentes, lorsque la trajectoire du véhicule n’est pas rectiligne (Fig. 16.13a). Il peut être schématisé par la figure 16.13b, dans laquelle on distingue : i) le carter, auquel est associé le référentiel R = Oxyz lié au véhicule, ii) le boîtier ou coquille représenté par le référentiel R  = Oxy z ; R tourne autour de Oz avec la vitesse angulaire de rotation V0 grâce à un pignon d’attaque qui entraîne le différentiel par la couronne ; iii) les planétaires, disques identiques de rayon r, de vitesses angulaires par rapport à R, V 1 et V2, qui commandent chacune des roues, iv) le satellite, disque métallique de rayon s et d’axe Ox  , qui s’appuie sur les planétaires ; il est entraîné par le boîtier et peut en outre tourner autour de son axe à la vitesse de rotation V s par rapport à R. x Roue 2

x

Différentiel

R (carter)

Ωs Roue 1 I2 M2 N2

Ω2

Roue 2

z Plan´etaires

Ω1

P2 Ω 2 Plan´etaire 2 y y

a)

Satellite

I1 S N M 1 1 Roue 1

O P1 Ω 1 Ω0

Plan´etaire 1

R0 (boîtier) b)

F IG . 16.13.

Traduisons le roulement sans glissement du satellite en contact avec les planétaires. Désignons par N1 et N 2 les points du satellite en contact respectivement avec les points M 1 et M2 des planétaires aux points géométriques I1 et I 2 . Il vient, relativement à R, en appelant P1 et P 2 les centres de planétaires : vM 1 = M1 P1 × V 1 et v M2 = M2P 2 × V 2 D’autre part : v N1 = v S + N 1S × (V s + V0) et vN 2 = vS + N 2S × (V s + V0 ) où (V s + V 0) représente le vecteur vitesse de rotation du satellite par rapport à R. Or, les points O et S étant à une distance constante l’un de l’autre, vS = SO × V 0 .

281

Cinématique du solide et des solides en contact En raison du roulement sans glissement, v M1 = v N 1 et vM 2 = vN2 . Par conséquent, M1P 1 × V1 = SO × V 0 + N 1S × (V s + V0)

et M2 P2 × V2 = SO × V0 + N 2 S × (V s + V 0)

En ajoutant ces deux équations, on trouve, puisque M1 P1 = M2 P2 = SO et N 1 S = −N2S est colinéaire à V 0 : M1P1 × (V1 + V2 ) = 2SO × V0

V1 + V2 = V0 2

d’où

Ainsi, la demi-somme des vitesses angulaires des planétaires est égale à la vitesse angulaire du boîtier. Dans la base de R , les deux équations vectorielles traduisant le roulement sans glissement s’explicitent suivant : 0 0 0 (Vs + V0) −r −r 0 × 0 = 0 × 0 + 0 × 0 0 0 0 V1 V0 −s

0 0 0 (Vs + V 0) −r −r 0 × 0 = 0 × 0 + 0 × 0 0 0 0 V2 V0 s

et

Il en résulte :

s s V 1 = V 0 − (Vs + V0) et V2 = V 0 + (Vs + V0 ) r r Notons que, si Vs = 0, les vitesses angulaires V 1 et V2 sont différentes, ce qui permet aux roues d’un même essieu de parcourir des trajets différents, comme cela est nécessaire dans les virages.

III . — MOUVEMENTS PLANS D’UN SOLIDE III . 1 . — Définition On appelle mouvement plan d’un solide S un mouvement tel que chaque point de S se déplace dans un plan parallèle à un plan Q fixe dans le référentiel considéré R. La section du solide par le plan Q est donc une figure qui évolue, dans R, sans se déformer. L’étude du mouvement de cette figure détermine sans ambiguïté la position de S, d’où son intérêt. La figure 16.14 représente un cylindre en mouvement dans le plan Oxy qui coïncide avec un plan de section droite. y

 c Dunod – Toute reproduction non autorisée est un délit

y R



O O

S

A x x

z

z

x

F IG . 16.14.

III . 2 . — Centre instantané de rotation Soient Q = Oxy le plan fixe et Q  = Ox y le plan invariablement lié à la section de S par Q. La vitesse d’un point A de Q, par rapport à R, s’écrit donc : vA/R = v O/R + V × O A où V, vecteur vitesse de rotation de S par rapport à R, est porté par la normale au plan de la figure. On appelle centre instantané de rotation , brièvement C.I.R, du plan Q , à la date t, le point I de S dont la vitesse, par rapport à R, est nulle : vI/R = 0

d’où

vA/R = vI/R + V × IA = V × IA

282

16. Cinématique du solide et des solides en contact

Ainsi, le mouvement du plan Q , par rapport au plan Q, est tangent à un mouvement de rotation autour de l’axe perpendiculaire en I à Q. III . 3 . — Base et roulante On appelle base et roulante les ensembles de points occupés par le C.I.R, respectivement dans les plans Q et Q . a) Mouvement plan d’un cylindre qui roule sans glisser sur le sol Dans le plan de section transversale, le mouvement se ramène à celui d’un disque en contact ponctuel avec une droite (Fig. 16.15). Comme le point du disque, qui est en contact avec la droite, a une vitesse nulle en raison du roulement sans glissement, le centre instantané de rotation est précisément ce point. La base, lieu géométrique du C.I.R dans Q, est cette droite. La roulante, lieu géométrique du C.I.R dans Q, est le périmètre du disque. y B

y

Roulante Q

O

O

Mur

y

A x

I

Q

K

Base

x

Sol

F IG . 16.15.

O

A

x

F IG . 16.16.

b) Barre contre un mur Considérons une barre, de longueur l , en contact par ses extrémités A et B avec deux plans perpendiculaires Ox et Oy (Fig. 16.16). On a, I étant le C.I.R : vA/R = V × IA et vB/R = V × IB Il en résulte, puisque vA/R et v B/R sont respectivement portés par les axes Ox et Oy, que le C.I.R est l’intersection des demi-droites Ay et Bx. On voit alors aisément que la base est le cercle de centre O et de rayon l, et que la roulante est le cercle de centre K , milieu de la barre, et de rayon l/2.

CONCLUSION Retenons deux résultats pratiques importants. (1) Le champ des vitesses d’un solide S s’exprime par la relation vectorielle suivante, entre les vitesses de deux points quelconques A et B de S par rapport à un référentiel R : vA/R = vB/R + AB × VS/R = v B/R + V S/R × BA VS/R étant le vecteur vitesse de rotation de S par rapport à R.

(2) La vitesse de glissement entre deux solides S 1 et S2 en contact est : v1/2 = v I 1/S2 = vI1 /R − vI2 /R

I 1 et I2 étant les points appartenant à chacun des solides qui coïncident, à l’instant considéré, avec le point géométrique I de contact ; la condition de roulement sans glissement est : v1/2 = 0. Bien qu’approché, le modèle du corps indéformable joue un rôle capital, en raison de ses nombreuses applications techniques en cinématique des solides en contact.

283

Cinématique du solide et des solides en contact

EXERCICES ET PROBLÈMES P16– 1. Mise en rotation d’un plateau Un plateau, de rayon R, est mis en rotation, autour d’un axe Oz, par un disque, de rayon r, qui roule sans glisser sur le pourtour du plateau (Fig. 16.17). L’axe du disque est une tige, passant par le point O1 de Oz, parallèle au plateau, et tournant autour de Oz avec une vitesse angulaire V 0. 1. Établir la relation entre V 0, la vitesse de rotation du plateau V p et celle du disque V. 2. Exprimer la vitesse du point H le plus haut du disque. Cas particulier où le plateau est bloqué. y z

Ω3

Ω0 O1 O

K C3

O Ω

Ω1

C1

Ωp F IG . 16.17.

Ω2 C2

x

F IG . 16.18.

P16– 2. Roulements à billes La figure 16.18 représente trois cercles matériels, C 1 , C2 , C3 , en contact entre eux, qui roulent sans glisser dans leur plan. On désigne par r 1 et r 2 les rayons de C 1 et C 2, et par K le centre de C 3. Établir l’équation qui relie leurs vitesses angulaires V 1, V 2 et V 3 par rapport au référentiel du laboratoire R = Oxyz. P16– 3. Mise en rotation d’un disque à partir d’un axe incliné

 c Dunod – Toute reproduction non autorisée est un délit

Pour faire tourner un disque autour de son axe de révolution Oz, à partir d’un axe incliné Oz  , on utilise un cône qui roule sans glisser sur le disque. Par rapport à un référentiel R = Oxyz, le cône, de demi-angle au sommet g, tourne autour de Oz fixe dans R avec la vitesse angulaire Vc. On désigne par V d la mesure algébrique sur Oz du vecteur vitesse de rotation du disque par rapport à R (Fig. 16.19). 1. Exprimer, en un point I de la génératrice de contact situé à la distance d de O, la condition de roulement sans glissement. 2. Quelle doit être la valeur de g pour que le rapport V d/V c soit égal à 0,5 ? z

z

.

ψ

Ωd z Ωc O

y

O

γ

I x

φ

z

x F IG . 16.19.

.

F IG . 16.20.

y

284

16. Cinématique du solide et des solides en contact

P16– 4. Cône roulant sans glisser sur un plan Un cône, de demi-angle au sommet g , est en contact suivant l’une de ses génératrices avec un plan horizontal Oxy, fixe dans le référentiel du laboratoire R, et roule sans glisser sur Oxy (Fig. 16.20).

1. Exprimer le vecteur vitesse de rotation V du cône par rapport à R , en fonction de la vitesse de ˙ précession c˙ et de la vitesse de rotation propre f. 2. Représenter géométriquement V. En déduire la relation entre V, c˙ et g. P16– 5. Entraînement d’un plateau circulaire Un plateau circulaire P, de rayon R, est mis en rotation autour de son axe, grâce à un système constitué d’une tige T , de demi-longueur l, aux extrémités de laquelle sont articulés deux disques identiques D1 et D2 de rayon r. La tige est perpendiculaire à l’axe de rotation et son centre est situé sur cet axe sous le plateau, à une distance r de P, de telle sorte que D 1 et D2 soient en contact avec P (Fig. 16.21). Le plateau, la tige et les deux disques sont des solides homogènes. On désigne par ˙ et u˙ , les vitesses de rotation de P et de T par rapport au référentiel du laboratoire R = Oxyz et c ˙ 1 et f˙ 2 les vitesses de rotation de D1 et D2 par rapport à T . on appelle f 1. Quelles sont les expressions des vecteurs rotations V 1 et V2 de D 1 et D2 par rapport à R ? Que se passe-t-il si l’on bloque le mouvement de la tige ? ˙ u, ˙ f ˙ 1 et f˙ 2, qui traduisent le roulement sans glissement de 2. Écrire les équations, reliant c, D1 et D2 sur P ? En déduire une relation entre f˙ 1 et f˙ 2 . 3. On bloque le plateau. Que deviennent V 1 et V 2 ? Représenter géométriquement ces vecteurs. z

Couronne

.

ψ I2

.

P

O

φ2 D2 x

y

I1 H

.

θ

y K

T D1

Satellite

O θ

.

φ1

F IG . 16.21.

ϕ

x

F IG . 16.22.

P16– 6. Roulement sans glissement d’un disque-satellite sur un solide-couronne Un disque-satellite (centre K, rayon r), mobile autour de son axe de révolution, roule sans glisser, grâce au porte-satellite OK, sur un solide-couronne, de rayon R, fixe par rapport au référentiel du laboratoire R (Fig. 16.22). Initialement, les angles u et f sont nuls. Établir la relation entre u et f sachant que R/r = 4. P16– 7. Base et roulante dans le mouvement d’un disque sur un axe Un cylindre se déplace le long d’un axe horizontal de telle sorte que son plan médian soit toujours contenu dans un plan vertical. Sa vitesse angulaire est V = −V e z (V > 0) et la vitesse de son centre K est v0 (Fig. 16.23). 1. Trouver la base et la roulante du mouvement sachant que le cylindre roule et glisse sur l’axe des x. 2. Étudier le cas du roulement sans glissement.

285

Cinématique du solide et des solides en contact

D1 y

Ω

y

Ω x

r

I1

D2

K

H

K

Ωs

I2

x

O

F IG . 16.23.

F IG . 16.24.

P16– 8. Mise en rotation de deux disques coaxiaux Deux disques coaxiaux horizontaux, D 1 et D2, sont mis en mouvement par le roulement sans glissement d’une roue verticale, de rayon r, qui tourne autour de son axe et dont le centre décrit un cercle (Fig. 16.24). La vitesse angulaire de la roue autour de son axe est Vs et la vitesse angulaire de la tige HK, de longueur b, qui relie le centre de la roue à l’axe des disques est V. ˙ 1 et f ˙ 2 des deux disques. Trouver, en fonction de r, b, V s et V, les vitesses angulaires f Commenter. P16– 9. Bille roulant sans glisser sur un rail diédrique Une bille sphérique (rayon r) roule sans glisser sur un rail à section droite en forme de dièdre droit (Fig. 16.25). Établir la relation entre la vitesse vK du centre de la bille et sa vitesse de rotation V.

Bille K ω J Rail

I



 c Dunod – Toute reproduction non autorisée est un délit

F IG . 16.25.

17 Éléments cinétiques des systèmes matériels Nous nous proposons de déterminer le centre de masse des systèmes matériels quelconques, déformables ou non, ainsi que la répartition géométrique des masses, afin d’exprimer simplement les concepts cinétiques (quantité de mouvement, moment cinétique, énergie cinétique) qui apparaissent dans l’expression des lois de la dynamique des systèmes, notamment dans le cas de solides.

I . — CENTRE DE MASSE D’UN SYSTÈME MATÉRIEL En mécanique newtonienne, la masse d’un système matériel S d présente la propriété d’additivité : la masse d’un ensemble discret de N corps ponctuels est la somme des masses des différents corps (Fig. 17.1a) : N

M=

mi i=1

Si le système S d peut être assimilé à une distribution continue de masse, on obtient sa masse totale en calculant l’intégrale de volume suivante : M=

r(A) d

dans laquelle r(A) est la masse volumique du système au point A et nant l’ensemble des masses (Fig. 17.1b).

dl

A1 A2 a)

Ai

le domaine volumique conte-

S

d A b)

´

S

c)

d)

F IG . 17.1.

Si la distribution de masse est répartie le long d’un fil, de section transversale constante s de diamètre très faible devant la longueur du fil (Fig. 17.1c), on introduit la masse linéique rl (A) = r(A)s

287

Éléments cinétiques des systèmes matériels et la longueur d l = d

/s d’un élement de fil : M=

r(A) d

rl (A) d l

= C

De même, si la distribution de masse est répartie sur une surface, d’épaisseur e très faible devant les dimensions transversales de la surface (Fig.17.1d), on introduit la masse surfacique rs(A) = r(A)e et l’aire d s = d /e d’un élement de surface : r(A) d

M=

= S

rs(A) d S

Remarque : En mécanique relativiste, la masse conserve la propriété d’être constante, pour un système fermé évidemment, mais perd celle d’additivité. Elle conserve aussi la propriété d’invariance par changement de référentiel galiléen (cf. Relativité et invariance). I . 1 . — Définition du centre de masse On appelle centre de masse d’un système matériel quelconque Sd le barycentre des différents éléments de Sd affectés de leur masse respective. Ainsi, le centre de masse d’un système de N corps ponctuels est le point C défini par : 1 OC = M

N

N

m i OA i

m i CA i = 0

ou

i=1

i=1

où O est une origine quelconque et M la masse totale du système. Si la distribution de masse est continue, les relations précédentes deviennent : OC =

1 M

OA r(A) d

CA r(A) d

ou

=0

Si la distribution de masse est répartie le long d’un fil, on a :

 c Dunod – Toute reproduction non autorisée est un délit

OC =

1 M

OA rl(A) d l

CA rl (A) d l = 0

ou

C

C

De même, si la distribution de masse est répartie sur une surface S : OC =

1 M

S

OA r s(A) d S

ou S

CA rs (A) d S = 0

Remarques : (1) Rappelons que la définition du centre de masse n’exige pas au préalable celle de la gravité ou de la pesanteur (cf. chapitre 13) ; ainsi, dans des expériences newtoniennes sur les collisions, on compare des masses alors que la gravitation n’intervient pas (cf. chapitres 14 et 21). Dans certaines expériences didactiques, on compare des masses en neutralisant le rôle du poids, grâce à une table horizontale à coussin d’air. Dans une cabine spatiale, cette neutralisation du poids s’opère automatiquement par la force d’inertie d’entraînement. Cependant, on mesure le plus souvent les masses en comparant les poids des corps, lesquels sont proportionnels aux masses, en un même lieu (cf. chapitre 7). (2) On allégera parfois l’écriture en n’utilisant que le signe S pour traduire les différentes sommations discrète ou continue.

288

17. Éléments cinétiques des systèmes matériels

I . 2 . — Propriétés du centre de masse a) Associativité Si l’on note Ck les centres de masse de divers systèmes matériels Sk , de masses respectives Mk , deux à deux sans élément commun, le centre de masse de la réunion des Sk est celui des Ck affectés des masses M k : Mk OC k avec M = Mk M OC = k

k

Si un système matériel est une somme de systèmes matériels simples, cette propriété permet de « concentrer » préalablement la masse de ces parties en leurs propres centres de masse puis de déterminer le centre de masse de l’ensemble. Exemples (1) Centre de masse de la molécule de gaz ammoniac NH 3 . Le centre de masse de cette molécule est celui de l’atome d’azote N et du groupement des trois atomes d’hydrogène concentrés en leur centre de masse K (Fig. 17.2). Il en résulte, puisque mN /mH = 14 : CCN CCK CK CN 3 et CC N = C KC N m N CCN + 3m H CC K = 0 d’où =− = 3mH 3m H + mN 17 mN

N H

C C1 C 2 H

K H F IG . 17.2.

D1

D2

F IG . 17.3.

(2) Centre de masse C d’un disque évidé. Le centre de masse C d’un disque évidé, de masse M , est tel que : M 1OC1 = M OC + M2 OC 2 C 1 étant le centre de masse du disque D 1 non évidé, homogène, de masse M 1, et C 2 le centre de masse du disque homogène D 2 , de masse M2, que l’on a enlevé (Fig. 17.3). En choisissant l’origine arbitraire en C 1, on obtient, puisque M = M 1 − M2 : M1 − M 0 = M C1 C + (M1 − M) C 1C2 d’où C 1C = − C1 C 2 M Si le rayon du disque enlevé est la moitié du rayon du disque total, M = 3M1 /4 et C1 C = −C1C 2/3. b) Symétrie matérielle Précisons d’abord qu’un système matériel possède un élément de symétrie matérielle (point, droite, plan) si la masse volumique en tout point de ce système est égale à la masse volumique au point symétrique par rapport à cet élément de symétrie. Ainsi, sur les figures 17.4a, b et c, A et A étant deux points symétriques par rapport à l’élément de symétrie, on a r(A) = r(A ). En remplaçant deux points symétriques par leur centre de masse doté d’une masse double et en composant tous les centres de masse élémentaires, on voit aisément que C se trouve soit au centre de symétrie O, soit sur l’axe de symétrie D, soit dans le plan de symétrie P. Ainsi, le centre de masse d’une tige homogène est son milieu, celui d’une plaque rectangulaire homogène est le point de concours de ses médianes, etc. Retenons que si un système possède un élément de symétrie matérielle, ce dernier contient le centre de masse.

289

Éléments cinétiques des systèmes matériels Axe de symétrie D

A

O

Centre de symétrie

A

A Plan H de symétrie

A

P

H

A

A

a)

b)

c)

F IG . 17.4.

c) Détermination expérimentale du centre de masse à l’aide d’un champ de force uniforme Considérons un système matériel S d dont chacun des éléments A i est soumis à la force m i a0 , a0 étant un champ d’accélération uniforme tel que le champ local de pesanteur ou un champ d’accélération uniforme quelconque. Le torseur des forces qui s’exercent sur Sd est celui associé à un ensemble de vecteurs liés parallèles. C’est donc aussi celui associé à un seul vecteur dont le support passe par le barycentre des points Ai, affectés des masses m i , qui est le centre de masse C. Ce résultat fournit une méthode expérimentale de détermination de C : le champ de pesanteur terrestre étant localement uniforme, on suspend Sd par un fil en différents points ; l’intersection des directions verticales des fils de suspension tracées sur le système Sd donne C. d) Théorèmes de Pappus-Guldin Ces théorèmes, au nombre de deux, sont attribués à Pappus d’Alexandrie en 340 et au mathématicien suisse P. Guldin en 1641 ; ils permettent de déterminer rapidement les centres de masse de courbes et de surfaces matérielles simples. (1) Premier théorème de Pappus-Guldin. Considérons un arc plan AB, de longueur l, situé dans le premier quadrant d’un système d’axes Oxy. Notons rl sa masse linéique (Fig. 17.5a). D’après la définition du centre de masse, on a : r llx C =

AB

xr l d l

et r lly C =

AB

yrl d l

Si rl est uniforme, ces deux équations se simplifient selon :  c Dunod – Toute reproduction non autorisée est un délit

l xC =

x dl AB

et l yC =

y dl AB

L’interprétation de ces deux intégrales est immédiate : elles représentent, au facteur multiplicatif 2p près, les aires Sy et S x des surfaces engendrées par la rotation de l’arc AB autour des axes Oy et Ox respectivement. Ainsi : Sy Sx et xC = yC = 2pl 2pl y

A d

y

R

B O

C

O

x a)

x b)

F IG . 17.5.

290

17. Éléments cinétiques des systèmes matériels

Exemple : Soit à déterminer la position du centre de masse d’un quart de cercle de rayon R (Fig. 17.5b). Comme l = pR/2 et S x = Sy = 2pR 2, xC = y C = 2R/p.

(2) Second théorème de Pappus-Guldin. Considérons une plaque plane D, d’aire S, située dans le premier quadrant d’un système d’axes Oxy. Notons rs sa masse surfacique (Fig. 17.6a). D’après la définition du centre de masse, on a : rsx d x d y

r sSxC =

S

et r sSyC =

r sy d x d y S

Si la plaque est homogène, on peut simplifier par rs : x dxdy

Sx C =

S

et S y C =

Ces intégrales représentent, au facteur 2p près, les volumes plaque autour des axes respectifs Oy et Ox ; on a donc : y

xC =

et

y

yC =

et

2pS

y dx dy S

engendrés par la rotation de la

x

2pS

y

y D

S

y O

x

R

x a)

C

O

x

x

b) F IG . 17.6.

Exemple : Soit à déterminer le centre de masse d’un quart de plaque circulaire de rayon générateur R (Fig. 17.6b). Comme S = pR2 /4 et x = y = 2pR 3 /3, on trouve : xC = y C = 4R/(3p). I . 3 . — Calculs de centres de masse a) Cône plein homogène En raison de la symétrie matérielle, le centre de masse d’un cône homogène (masse M, hauteur h, demi-angle au sommet g) est situé sur l’axe de symétrie matérielle Oz (Fig. 17.7a). Il convient donc de calculer les intégrales suivantes : rd

M=

et Mz C =

zr d

Ces intégrales de volume, a priori triples, peuvent se ramener à des intégrales simples. Pour cela, cherchons l’ensemble des points tels que la quantité à intégrer, zr, soit une constante ; ces points sont situés entre les plans de cotes z et z + d z. Par conséquent, l’élément de volume d vaut : d

=

r

drrdwdz = w

drr r

dwdz = w

r2 2p d z = pr 2 d z 2

avec

r2 = z2 tan 2 g

Il en résulte : h

MzC =

zr d

=

0

2

h

2

zrpr d z = rp tan g

0

z 3 d z = rp

h4 2 tan g 4

291

Éléments cinétiques des systèmes matériels Comme : h

M=

rd

=r

V

0

pr 2 d z = rp tan 2 g

h 0

z2 d z = rp

h3 tan2 g 3

on trouve

3h 4

z

z O

R h

zC =

C2

R µ

dz

C1

° z

O

O

a)

b)

c)

F IG . 17.7.

b) Centre de masse d’une demi-sphère (creuse) Dans ce cas, l’intégrale de surface à calculer se réduit, grâce au même découpage que précédemment, à (Fig. 17.7b) : S

zrs d S

avec

z = R cos u

et

R2 sin u d w d u = R2 sin u d u

dS =

w

Donc : p/2 S

zrs d S =

0

d w = 2pR 2 sin u d u w

3

r s2pR sin u cos u d u = rs 2pR

3

sin 2 u 2

R/2

= rs pR3 0

2

Or M = r s2pR , ce que l’on retrouve en calculant : p/2

 c Dunod – Toute reproduction non autorisée est un délit

S=

dS =

0

2pR 2 sin u d u = 2pR2 {− cos u} 0

p/2

= 2pR2

d’où

zC =

R 2

Remarque : Pour déterminer le centre de masse de l’ensemble formé du cône de masse M 1 surmonté de la demi-sphère adaptée de masse M2 (Fig. 17.7c), il suffit de chercher le centre de masse des centres de masses C1 et C 2 de chaque partie : 1 3h R zC = M1 + M2 h + 4 2 M1 + M2

II . — MOMENTS D’INERTIE II . 1 . — Moment d’inertie par rapport à un axe On appelle moment d’inertie d’un système S d, par rapport à un axe D, la quantité positive : mi di2 =

ID = i

mi Hi A2i i

di = Hi A i étant la distance du point Ai de masse mi à l’axe D (Fig. 17.8a). Pour un système matériel continu, le moment d’inertie a pour expression : ID =

HA2r d

292

17. Éléments cinétiques des systèmes matériels

Il est clair, d’après la définition, que le moment d’inertie d’une réunion de deux systèmes est la somme des moments d’inertie de chacune des parties. Comme I D a les dimensions du produit d’une masse par le carré d’une distance, on l’écrit souvent sous la forme : ID = MR2 M étant la masse du système et R la distance à l’axe d’un point fictif de masse M qui aurait, par rapport à D, le même moment d’inertie que le système ; on appelle cette longueur le rayon de giration. z

Δ

A i(mi ) di

O

Δ

S

H eΔ i y

dl

C x

z

b) A

a) F IG . 17.8.

Exemple : Calculons le moment d’inertie d’une tige homogène, de masse M et de longueur l, par rapport à un axe D perpendiculaire passant par son centre C (Fig. 17.8b). Il vient d’après la définition, en désignant par rl la masse linéique de la tige et par z la coordonnée suivant la tige : ID =

l/2

2

z2 d z = rl × 2 ×

CA rl d l = r l −l/2

l3 Ml2 = 24 12

√ puisque M = rl l. Le rayon de giration de la tige est donc : R = l 3/6. II . 2 . — Opérateur d’inertie et matrice d’inertie Exprimons, dans un système d’axes R = Oxyz, le moment d’inertie d’un système S d par rapport à un axe D de vecteur unitaire eD passant par le point O (Fig. 17.8a) : m iH i A2i =

I OD = i

i

mi  eD × OAi 2

En désignant les composantes de eD et OA i par (a, b, g) et (xi , yi , zi ) respectivement, il vient : IOD = i

m i [(bz i − gyi) 2 + (gxi − azi )2 + (ay i − bxi )2]

soit encore, en développant : IOD = a 2 Ixx + b 2I yy + g2 Izz − 2abIxy − 2bgI yz − 2gaIxz

avec :

m i (y2i + z 2i )

IOx = i

m i(x 2i + z2i )

I Oy =

m i (x2i + y2i )

IOz =

i

i

et Ixy =

m i xi yi i

Iyz =

m i yi zi i

I xz =

m i xi zi i

293

Éléments cinétiques des systèmes matériels

Les quantités IOx , IOy , I Oz sont appelées les moments d’inertie du système Sd par rapport aux axes respectifs Ox, Oy, Oz, et Ixy , Iyz , Ixz les produits d’inertie. L’expression précédente de IOD suggère d’introduire un opérateur [I ] O qui s’explicite dans la base de R suivant une matrice carrée :

R

IOx −Ixy −Ixz

−Ixy I Oy −Iyz

−Ixz −Iyz IOz

Comme eD = aex + bey + ge z, le vecteur [I ] O e D s’explicite selon :

R

IOx −Ixy −Ixz

−Ixy I Oy −Iyz

−Ixz −Iyz IOz

R

a aIOx − bI xy − gIxz b = −aI xy + bIOy − gI yz g R −aIxz − bIyz + gIOz

le moment d’inertie I OD peut être considéré comme le produit scalaire du vecteur unitaire e D et du vecteur [I ]O eD que l’on obtient par l’action de [I ] O appelé opérateur ou tenseur d’inertie, sur e D : I OD = eD · [I] O eD La matrice carrée symétrique suivant laquelle s’explicite l’opérateur [I ]O dans la base orthonormée de R est la matrice d’inertie. II . 3 . — Axes principaux d’inertie La matrice d’inertie étant réelle et symétrique, il est possible de trouver, par changement de base, un système d’axes orthonormés tels que les produits d’inertie soient nuls. Cette base est appelée la base principale d’inertie, ses axes, les axes principaux d’inertie, la matrice, la matrice principale d’inertie et les termes diagonaux I 1, I2 et I3 les moments principaux d’inertie. Dans cette base, [I ] O s’écrit :

R

I1 0 0

0 0 I2 0 0 I3

Notant e1, e2 et e 3 les vecteurs unitaires portés par les axes principaux, on en déduit que : [I] O e1 = I 1e1

[I]O e2 = I 2 e2

[I ] O e 3 = I 3 e 3

 c Dunod – Toute reproduction non autorisée est un délit

On dit que e1, e2 et e 3 sont les vecteurs propres de l’opérateur [I] O et que I1 , I2 et I3 sont les valeurs propres correspondantes. Remarque : Le choix systématique d’une base principale d’inertie permet de ramener le calcul des six coefficients de la matrice à trois calculés une fois pour toutes. Cependant, il implique l’utilisation d’une base qui est en mouvement par rapport au référentiel du laboratoire, d’où le soin qu’il convient d’apporter dans la dérivation des grandeurs vectorielles par rapport au temps. II . 4 . — Détermination des axes principaux d’inertie En l’absence de symétrie, la recherche des axes principaux d’inertie d’un système matériel est délicate. En revanche, si le système possède des symétries matérielles, la détermination des axes principaux est aisée. a) Symétries matérielles et axes principaux (1) Plan de symétrie

294

17. Éléments cinétiques des systèmes matériels

Supposons que le système Sd possède un plan de symétrie matérielle, par exemple Oxy (Fig. 17.9a). Le vecteur [I ]O e z s’explicite selon :

R

IOx −Ixy −I xy IOy −Ixz −Iyz

−Ixz −Iyz I Oz

R

0 −Ixz 0 = −Iyz 1 R I Oz

Or, Ixz = i m i x i zi est nul puisque l’on peut grouper, du fait de la symétrie, deux à deux les éléments qui ont même xi et des valeurs opposées de zi . De même, Iyz = i mi yi z i est nul. D’où : [I]O ez = IOz ez . Tout axe perpendiculaire à un plan de symétrie matérielle est axe principal d’inertie. z

z Axe de symétrie H y

O x

H

O

y

x

Plan de symétrie

a)

b) F IG . 17.9.

(2) Axe de symétrie Supposons que Sd possède un axe de symétrie matérielle, par exemple Oz (Fig. 17.9b). Ici aussi, le vecteur [I ]O ez s’explicite selon : −Ixz −Iyz R I Oz avec Ixz = Iyz = 0. En effet, Ixz = i mi xiz i peut être calculé en groupant deux à deux les éléments qui ont même zi et des valeurs de x i opposées. D’où : [I ]O ez = I Oz ez . Tout axe de symétrie matérielle est axe principal d’inertie. b) Conséquences Nous allons résumer les résultats précédents sous la forme de deux théorèmes très utiles dans la recherche des bases principales d’inertie. (1) Premier théorème Tout trièdre trirectangle, dont deux de ses plans sont plans de symétrie matérielle pour un système, est trièdre principal d’inertie de ce système. En effet, si les plans Oxz et Oyz sont plans de symétrie, les vecteurs e y et e x, respectivement perpendiculaires à ces plans, définissent des axes principaux. D’autre part, Oz est axe de symétrie puisque, d’après les symétries planaires, on peut écrire : m i (xi , yi , zi ) = m i (xi , −y i , zi ) = mi (−x i , −yi , zi )

L’axe Oz est donc axe principal. (2) Deuxième théorème

Tout trièdre trirectangle, dont deux de ses axes sont axes de symétrie matérielle pour un système, est trièdre principal d’inertie de ce système.

295

Éléments cinétiques des systèmes matériels

En effet, si les axes Ox et Oy sont axes de symétrie, les vecteurs e x et e y définissent des axes principaux. D’autre part, Oz est axe de symétrie puisque, d’après les symétries axiales, on peut écrire : mi (x i , yi , zi ) = mi (xi , −yi , −z i ) = mi (−xi , −yi, z i )

L’axe Oz est donc axe principal. c) Associativité

Si le système Sd est la réunion de plusieurs systèmes matériels Sk , le moment d’inertie par rapport à tout axe D est la somme des moments d’inertie des Sk par rapport à ce même axe. Cette propriété qui découle directement de la définition permet de décomposer le système S d en systèmes Sk simples, dont les moments d’inertie sont connus. II . 5 . — Opérateurs principaux d’inertie cylindrique et sphérique L’opérateur principal d’inertie d’un système matériel est cylindrique si deux de ses moments d’inertie principaux, par exemple I1 et I 2, sont égaux. Il est sphérique si les trois moments d’inertie principaux sont égaux. Les matrices d’inertie s’écrivent donc respectivement : I1 0 0 I1 0 0

0 0 I3

I1 0 0

et

0 I1 0

0 0 I1

= I1 [1]

II . 6 . — Théorème d’Huygens-Schteiner La matrice d’inertie dépend de l’origine des axes du trièdre choisi. Étudions l’influence d’une translation des axes en considérant le trièdre initial Oxyz et le nouveau trièdre O x yz  (Fig. 17.10). En désignant par a, b, c les coordonnées de O, il vient : m i (x 2i + y 2i ) =

IOz = i

et

m i [(xi + a)2 + (y i + b) 2] = I O z + M (a 2 + b2 ) + 2a i

i

Ixy =

m i yi i

mi (x i + a)(yi + b)

mi xiy i = i

 c Dunod – Toute reproduction non autorisée est un délit

m ix i + 2b

i

Si O coïncide avec le centre de masse C du système, les relations précédentes se simplifient, car les mi x i , m i y i et termes m i zi s’annulent. Retenons, pour le moment d’inertie ICz et pour le produit d’inertie ICxy associés au centre de masse : IOz = ICz + Md 2

avec d 2 = a2 + b 2

et

IOxy = I Cxy + Mab

La première relation, dans laquelle d est la distance entre les deux axes parallèles passant par les points O et C, est connue sous le nom de théorème de Huygens-Schteiner. Ainsi, le moment d’inertie par rapport à un axe de direction donnée est minimal si cet axe passe par le centre de masse. z

z c

d 

O x

x

O C

a F IG . 17.10.

y b y

296

17. Éléments cinétiques des systèmes matériels

III . — MÉTHODES DE CALCUL DES MOMENTS D’INERTIE III . 1 . — Système ayant la symétrie de révolution C’est le cas du cône plein homogène étudié précédemment (Fig. 17.7a). Les plans Oxz et Oyz sont plans de symétrie matérielle et Oz axe de révolution ; l’opérateur d’inertie [I ]O s’écrit, dans le repère R = Oxyz qui est repère principal :

R

0 0 I1 0 0 I3

I1 0 0

Il est en général plus commode de calculer d’abord le moment d’inertie I3 par rapport à l’axe de révolution ; ensuite, pour faciliter le calcul technique de I1 , il est souvent préférable d’introduire les intermédiaires que sont les moments d’inertie par rapport aux plans Oxz, Oyz et Oxy. En effet : m i y2i

I Oxz = i

et

+

z 2i )

2 i mi (xi

et I 2 =

mi z2i

I Oxy =

i

2 i m i (yi

sont reliés à I1 =

m i x2i

I Oyz =

i

+

z 2i )

par :

I1 + I 2 = I Oxz + IOyz + 2I Oxy D’autre part, IOxz + I Oyz = I3 et IOxz = I Oyz , du fait de la symétrie. Par conséquent : I 1 = I2 = I Oxy +

I3 2

Exemple : Pour déterminer la matrice principale d’inertie d’un cône plein et homogène (Fig. 17.7a), calculons les deux intégrales suivantes : (x 2 + y2 )r d

I3 =

et IOxy =

z 2r d

Les points tels que (x2 + y2 ) ait une valeur déterminée sont situés à l’intérieur d’une couronne, d’épaisseur d z, de rayon intérieur r = (x2 + y2) 1/2 et de rayon extérieur r + d r  . Par conséquent, I3 s’écrit : h r h r dz dz I3 = r2 r2pr d r = 2pr r 3 d r  0

soit : I3 =

0

h

pr 2

Quant à IOxy , il vaut :

Or le volume du cône est

0

=

pr 4 tan a 2

r4 d z =

0

h

I Oxy =

0

z2 rpr2 d z =

pR 2h/3,

d

=

h 0

0

pr 2 d z =

pz 2 tan2 a d z = p tan2 a

Finalement, il vient, en fonction de la masse M = r 3 Mh2 tan2 a 10

pr 5 4 h tan a 10

rpz4 tan2 a d z = rp

0

I3 =

z4 d z =

0

h5 tan 2 a 5

avec R = h tan a, ce que l’on peut retrouver selon : h

=

h

IOxy =

3 2 Mh 5

z3 3

h

= 0

ph3 tan2 a 3

: et

3 3 I 1 = I2 = Mh 2 + Mh2 tan2 a 5 20

297

Éléments cinétiques des systèmes matériels III . 2 . — Système ayant un opérateur d’inertie à symétrie sphérique

Dans ce cas, il est commode, pour déterminer le moment d’inertie I par rapport à tout axe passant par le centre de symétrie O, d’utiliser comme intermédiaire de calcul le moment d’inertie au centre O : m i (x2i + y2i + z2i )

IO = i

Comme I 1 + I2 + I 3 = 2IO et que I1 = I2 = I3 = I, il vient : 2 IO 3

I=

a) Opérateur d’inertie d’une sphère et d’une boule i) Sphère homogène Compte tenu des symétries, le trièdre Oxyz est principal d’inertie (Fig. 17.11). Évaluons I O : IO = S

r 2 rs d S

soit, puisque r est égal au rayon R de la sphère (creuse) : IO = r s R2 S

d S = rsR2 S = MR2

d’où

2 2 I = I O = MR2 3 3

ii) Boule homogène L’analyse est la même que précédemment, mais le calcul de I O est plus laborieux, puisque la masse est répartie sur tout le volume : IO = soit, puisque d

= 4pr2 d r et M = r4pR3 /3 :

IO = 4pr  c Dunod – Toute reproduction non autorisée est un délit

r2r d

R

4

r d r = 4pr

0

r5 5

R

= 4pr 0

R5 3MR2 = 5 5

d’où

I=

2 2 I O = MR 2 3 5

b) Opérateur d’inertie d’une demi-sphère et d’une demi-boule i) Demi-sphère En raison des propriétés d’associativité et d’additivité, le moment d’inertie I (ds) de la demi-sphère (creuse) est la moitié du moment d’inertie I (s) de la sphère (creuse). Par conséquent : I (s) M(s)R 2 = M (ds) R2 = 2 2 puisque les masses sont dans le rapport 1/2. On obtient donc la même expression que pour la sphère entière : 2 I = MR 2 3 I(ds) =

ii) Demi-boule homogène Un raisonnement analogue au précédent donne le même résultat que pour une boule homogène.

298

17. Éléments cinétiques des systèmes matériels z

z O

O

y

x

y

x F IG . 17.11.

F IG . 17.12.

c) Opérateur d’inertie d’un cube plein homogène La matrice principale d’inertie du cube possède la symétrie sphérique puisque les trois axes ont des rôles identiques. Le calcul du moment d’inertie par rapport à l’un quelconque de ses axes principaux est (Fig. 17.12) : x2 d + r y2 d I Oz = (x2 + y2)r d = r Il vient en intégrant, a désignant le côté du cube : 2

x d

a/2

a/2

2

x dx

=

dy

−a/2

−a/2

x3 3

a/2

dz = −a/2

a/2

y −a/2

a/2 −a/2

z

a/2 −a/2

=

a5 12

Comme il en est de même pour la seconde intégrale, on trouve, sachant que M = ra 3 : I Ox = IOy = I Oz =

Ma2 6

III . 3 . — Centre de masse et moments d’inertie de quelques solides Nous avons rassemblé dans le tableau 17.1 les propriétés de masse de quelques solides homogènes qui représentent un grand nombre de solides réels. On remarquera qu’il est possible de déduire de ce tableau d’autres résultats intéressants. Ainsi, la tige s’obtient en faisant R = 0 dans le cylindre plein, la plaque en faisant c = 0 dans le parallélépipède rectangle, etc. D’autre part, les résultats relatifs aux demi-solides s’obtiennent aisément à partir des solides entiers, comme cela a déjà été dit.

IV . — QUANTITÉ DE MOUVEMENT ET MOMENT CINÉTIQUE IV . 1 . — Quantités de mouvement Considérons le mouvement, par rapport à un référentiel R, d’un système quelconque S d , déformable ou non (Fig. 17.13). On peut associer à chaque élément de matière de masse d m, entourant un point A de ce système, de vitesse v, la quantité de mouvement élémentaire : d P = v d m = vr d . En sommant sur tout le volume , on obtient la quantité de mouvement du système : P=

rv d

Un résultat important sur la quantité de mouvement a été établi dans le cas de systèmes de N corps ponctuels (cf. chapitre 13). Pour l’obtenir dans le cas des systèmes continus, on procède de la même façon : P=

r

d OA d dt

=

d dt

rOA d

=

d (MOC) dt

d’où

P = M vC

La quantité de mouvement d’un système matériel est celle de son centre de masse affecté de la masse totale.

299

Éléments cinétiques des systèmes matériels

Corps homogènes de masse M

Centre de masse

Moments et produits d’inertie

l x

R

y O z

Centre

l

I1 = I 2 =

R

Centre

z

O

I3 =

y Cylindre plein z

MR 2 Ml 2 + 2 12

I3 = MR 2

Cylindre creux x

I1 = I 2 =

MR 2 Ml 2 + 4 12

MR2 2

b

O

y

M(b 2 + c2) 12 M(a 2 + c2) I2 = 12 I1 =

c

Centre

x a Parallélépipède rectangle z R O

y

x

Centre

I1 = I2 = I 3 =

2MR 2 3

Centre

I1 = I2 = I 3 =

2MR 2 5

Sphère (creuse) z R  c Dunod – Toute reproduction non autorisée est un délit

O

y

x Boule (pleine) z

R

C h

x

O Cône creux

zC = y

2h 3

I1 = I2 = I3 =

MR2 2

MR 2 Mh2 + 4 2

300

17. Éléments cinétiques des systèmes matériels

R O

x

C

y

zC =

z

R 2

I1 = I2 = I 3 =

2MR 2 3

Demi-sphère (creuse) y

I Ox = I Oy =

R C

xC = y C =

Oz

x

2R p

IOz = MR 2 Ixy =

Quart de cercle matériel y b

4a 3p 4b yC = 3p xC =

C a

Oz

x

Quart de plaque elliptique y R Oz

α C

xC =

x

2R sin a 3 a

MR 2 p

Mb2 Ma2 IOy = 4 4 M I Oz = (a2 + b2 ) 4 Mab I xy = 2p sin(2a) MR2 1− I1 = 4 2a I Ox =

I2 =

sin(2a) MR2 1+ 4 2a

I 3 = I 1 + I2 =

Secteur circulaire

MR2 2

MR2 2

z O x a

I 1 = I2 = M

5R2 a2 + 2 4

I 3 = M a2 +

3R2 2

Centre

R

Tore creux z R C h

x

O

zC =

3h 4

y

Cône plein TAB . 17.1.

I 1 = I2 = I3 =

3Mh2 3MR 2 + 5 20

3MR 2 10

301

Éléments cinétiques des systèmes matériels IV . 2 . — Moment cinétique

On peut définir en un point fixe O de R, le moment cinétique du système S d en sommant les moments cinétiques élémentaires : LO =

OA × rv d

Si O est un autre point fixe de R, on a une relation simple entre les moments cinétiques en O et en O (cf. chapitre 1) : LO =

(OO + OA) × rv d

=

OO  × rv d

+ LO = OO  ×

rv d

+ LO

d’où : L O = LO + OO × P IV . 3 . — Torseur cinétique La relation précédente entre les moments cinétiques étant antisymétrique, on peut associer au système de vecteurs liés élémentaires {rvA d }, le torseur [P] dit torseur cinétique ou des quantités de mouvement. Son champ de moment est donné par le moment cinétique et son vecteur par la quantité de mouvement P du système (cf. chapitre 1). Soulignons que le torseur cinétique, qui est construit à partir de vitesses relatives à un référentiel R, est lui aussi relatif à R.

 c Dunod – Toute reproduction non autorisée est un délit

IV . 4 . — Quantité de mouvement et moment cinétique dans le référentiel du centre de masse Le référentiel du centre de masse R ∗ associé à R est le référentiel en translation par rapport à R et tel que (cf. chapitre 13) : P∗ = 0 Comme P ∗ = Mv∗C , v∗C = 0. Par conséquent, le centre de masse est fixe dans R ∗ . En outre, puisque P∗ = 0, le moment cinétique L ∗ est indépendant du point où on le calcule : L∗O = L∗O + OO × P∗ = L ∗O On pourra donc écrire sans autre précision dans R∗ : L∗ =

CA × rv∗ d

302

17. Éléments cinétiques des systèmes matériels z

z Ai O

R∗

C

x

y y

x

R F IG . 17.13.

IV . 5 . — Rappel du théorème de Kœnig relatif au moment cinétique Ce théorème relie L ∗ et LO relatif à R (cf. chapitre 13). Rappelons son expression : L O = L∗ + OC × P Il en résulte une propriété remarquable de C. En effet, si le point O est confondu avec C, on a : LC = L∗ Donc, le moment cinétique par rapport à R∗ et le moment cinétique en C par rapport à R sont égaux. IV . 6 . — Cas d’un solide a) Solide ayant un point fixe O Soit R = Oxyz le référentiel par rapport auquel on se propose d’exprimer la quantité de mouvement et le moment cinétique d’un solide S ayant un point fixe O (Fig. 17.14a). Dans R, on a, puisque la vitesse d’un point A de S est v = V × OA, V étant le vecteur vitesse de rotation de S par rapport à R : P = M vC = MV × OC et LO =

OA × rv d

=

OA × r(V × OA) d

La formule précédente permet de calculer directement le moment cinétique ; c’est laborieux mais on évite ainsi l’introduction de l’opérateur d’inertie. Pour faire apparaître l’opérateur d’inertie, il suffit d’expliciter l’expression du moment cinétique. Comme les vitesses des différents points de S satisfont au champ des vitesses d’un solide, on obtient : vA/R = V × OA d’où

LO =

VOA2 − OA(OA · V)

rd

z

Ω

z z

C O x

R

y

a

 y

Solide

y

O

S

x

x a)

y a b)

F IG . 17.14.

x

303

Éléments cinétiques des systèmes matériels

Explicitons cette dernière relation dans la base associée à R, dans laquelle (x, y, z) et (V x , V y, Vz) sont les composantes de OA et V respectivement. Il vient, pour la première composante : LO · e x =

[V x (x2 + y2 + z 2) − x(xVx + yV y + zV z)]

rd

soit : LO · ex =

rd

[Vx (y2 + z2) − xyV y − xzVz ] = IOx Vx − Ixy V y − Ixz V z

Comme le calcul les deux autres composantes donnent des résultats analogues, la relation entre le moment cinétique et le vecteur rotation d’un solide S, par rapport à un référentiel R, est la suivante : L O/R = [I]O V [I] O étant l’opérateur d’inertie représenté dans R par la matrice d’inertie :

R

IOx −Iyx −Izx

−Ixy I Oy −Izy

−Ixz −Iyz IOz

Notons que L O et V ne sont pas colinéaires, à moins que V soit porté par un axe principal : LO = [I]O V = [I]O (V e1) = V([I] O e1 ) = V I 1 e1 = I 1 V si l’axe défini par e1 est principal. b) Exemple : Moment cinétique d’une plaque carrée (1) Calcul direct Calculons le moment cinétique d’une plaque carrée homogène tournant autour d’un axe fixe Oz confondu avec l’un de ses côtés (Fig. 17.14b). Comme la vitesse d’un point A de la plaque, de coordonnées x , z, vaut Vx ey , si V = V ez est sa vitesse angulaire, il vient : LO =

OA × (V × OA) rs d x d z =

(x ex + z ez ) × (Vxe y ) rs d x d z

rs étant la masse surfacique. On obtient : a

 c Dunod – Toute reproduction non autorisée est un délit

LO = rsV e z

0

a 0

a

x2 d x d z − rs V e x

a

0

x z d x  d z

0

Effectuons les intégrales : a 0

a 0

2



x dx dz =

x3 3

a 0

a {z} 0

a4 = 3

a

et 0

a 0





x z dx dz =

x2 2

a 0

z2 2

a

= 0

a4 4

En introduisant la masse de la plaque M = rsa2 , on obtient : LO = r sV

a4 a4 e z − ex 3 4

=

Ma2 Ma2 V ez − V ex  3 4

(2) Calcul à l’aide de l’opérateur d’inertie Calculons l’opérateur d’inertie [I] O à l’origine O. Dans R , [I ]O s’explicite par la matrice d’inertie suivante : IOx −Ixy −Ix  z   −I y x IOy −Iy  z −Izy  IOz R −Izx

304

17. Éléments cinétiques des systèmes matériels

dans laquelle les produits d’inertie Iyz et Ix y sont nuls puisque l’axe Oy est perpendiculaire à un plan de symétrie matérielle. Comme V n’a qu’une composante selon ez, le moment cinétique L O/R a pour composantes dans la base de R :

R

IOx  0 −Izx

0 I Oy 0

−I x z 0 I Oz

R

−Ix z v 0 0 = 0 V I Oz v R

Il est donc nécessaire de calculer I Oz et I x z : a

IOz =

0

a 0

x2 rs d x d z =

Ma2 3

et Ix z =

a 0

a 0

xz rs d x d z =

Ma2 4

On retrouve bien le résultat obtenu directement. c) Cas général L’intérêt de la relation entre L O et V dépasse le cas particulier envisagé où S a un point fixe. En effet, notant C le centre de masse du solide en mouvement quelconque par rapport à R, on peut écrire, à l’aide du théorème de Kœnig : LO/R = [I]C V + OC × P Bien que la théorie ne l’exige pas, il est commode, pour calculer l’opérateur d’inertie, d’utiliser une base liée au solide ; dans cette base, les coefficients de [I ] O sont des constantes déterminées une fois pour toutes. On simplifiera encore l’étude en utilisant une base principale d’inertie que l’on aura trouvée à l’aide des éléments de symétrie matérielle du solide. Une fois la base principale adoptée, ce choix doit bien entendu être conservé pour V S/R ; dès lors, le moment cinétique du solide relatif à R sera exprimé dans une base différente de celle de R. On n’oubliera pas alors d’utiliser la relation de Bour (cf. chapitre 3).

CONCLUSION Retenons principalement les points essentiels : (1) Le centre de masse C d’un système quelconque continu est défini par : 1 OC = OA r d M (2) Le moment d’inertie I Oz d’un système par rapport à un axe Oz a pour expression : m i Hi A2i

IOz =

ou

IOz =

r HA 2 d

i

dans le cas d’une distribution continue, le point H étant la projection de A suivant l’axe Oz. En fonction de l’opérateur d’inertie [I ] O, IOz a pour expression : IOz = ez · [I]O e z (3) Les éléments de symétrie jouent un rôle essentiel dans la simplification des calculs et la détermination rapide du centre de masse et des axes principaux d’inertie.

305

Éléments cinétiques des systèmes matériels (4) Le théorème d’Huygens-Schteiner : I OD = I CD + Md 2

d étant la distance qui sépare l’axe OD de l’axe parallèle CD passant par le centre de masse, montre que le moment d’inertie par rapport à un axe de direction déterminée est minimal lorsque cet axe passe par le centre de masse. (5) La quantité de mouvement P et le moment cinétique L O d’un système matériel quelconque ont pour expressions respectives : Sd P = M vC =

et L O =

rv d

r OA × v d

Comme cette dernière expression est difficile à calculer, il est commode d’utiliser le théorème de Kœnig : LO = L ∗ + OC × P

avec

L∗ =

r CA × v∗ d

EXERCICES ET PROBLÈMES P17– 1. Moments d’inertie principaux d’un cerceau homogène 1. Trouver, en fonction de sa masse M et de son rayon R, les moments principaux d’inertie d’un cerceau homogène. 2. En déduire le moment d’inertie par rapport à un axe D tangent au cerceau et contenu dans son plan. P17– 2. Moment d’inertie d’une tige homogène

 c Dunod – Toute reproduction non autorisée est un délit

1. Déterminer le moment d’inertie d’une tige homogène (masse m, longueur l), par rapport à un axe D, passant par l’une de ses extrémités et incliné d’un angle u par rapport à la tige. 2. Quelle est la matrice d’inertie de la tige dans une base principale, en l’une de ses extrémités E ? Retrouver le résultat précédent. P17– 3. Centre d’inertie et moments d’inertie principaux d’un demi-cercle homogène 1. Trouver la position du centre d’inertie d’un demi-cercle homogène, de rayon R (Fig. 17.15). 2. Trouver le moment d’inertie par rapport à un axe perpendiculaire au plan du demi-cercle passant par le sommet S. 3. Quelle est la matrice d’inertie dans une base principale au centre géométrique O du cercle générateur ? y

y

y

S

Oz F IG . 17.15.

x

Oz F IG . 17.16.

x

Oz F IG . 17.17.

x

306

17. Éléments cinétiques des systèmes matériels

P17– 4. Moments d’inertie principaux d’un quart de disque homogène 1. Déterminer la position du centre de masse d’un quart de disque homogène (Fig. 17.16). 2. Quel est le moment d’inertie par rapport à l’axe de symétrie passant par le sommet O ? 3. Quelle est la matrice d’inertie d’un quart de disque matériel homogène, dans une base principale au centre géométrique O du disque générateur de rayon R ? Calculer le produit d’inertie Ixy . P17– 5. Centre d’inertie et moments d’inertie principaux d’un demi-disque homogène 1. Trouver la position du centre d’inertie d’un demi-disque matériel homogène, de rayon R et de masse M (Fig. 17.17). 2. Quelle est la matrice d’inertie dans une base principale au centre géométrique O du disque générateur ? 3. Quel est le moment d’inertie de ce solide par rapport à un diamètre du plan du demi-disque qui fait l’angle u avec l’axe Oy ? P17– 6. Centre d’inertie et moments d’inertie d’une plaque carrée homogène 1. Déterminer la matrice d’inertie d’une plaque carrée homogène, dans la base de Oxyz (Fig. 17.18). 2. En déduire le moment d’inertie par rapport à un axe D, passant par l’un de ses sommets S, contenu dans le plan Oxy et faisant l’angle u avec l’axe Ox. y

a

z

Δ S

θ

O

O

x

y

x F IG . 17.18.

F IG . 17.19.

P17– 7. Centre d’inertie et moments d’inertie d’une demi-boule homogène 1. Trouver la position du centre d’inertie d’une demi-boule homogène (pleine), de rayon R (Fig. 17.19). 2. Quel est son moment d’inertie par rapport à tout diamètre de son plan ? P17– 8. Moments d’inertie principaux d’un double panneau solaire 1. Trouver les moments d’inertie principaux d’un double panneau solaire, de masse M, en son centre O. Les caractéristiques géométriques de ce solide sont données sur la figure 17.20. 2. Le panneau tourne autour d’un axe vertical Oz 0 à la vitesse angulaire V. Son axe Oy fait un angle u avec le plan horizontal Ox0y0 du référentiel terrestre R0 = Ox 0 y0z0 . Donner l’expression du moment cinétique en O, par rapport à R0, dans la base de Oxyz.

307

Éléments cinétiques des systèmes matériels

z

Oz

b

a

l

y

2R O y

x

c F IG . 17.20.

F IG . 17.21.

P17– 9. Moments d’inertie principaux d’une antenne semi-cylindrique concave 1. Quels sont les moments d’inertie principaux, au centre O de l’antenne d’un radar, de masse M, que l’on peut assimiler au demi-cylindre concave de la figure 17.21 ? 2. L’antenne tourne autour de son axe vertical Oz avec la vitesse angulaire V. Donner l’expression du moment cinétique en O, par rapport à un référentiel terrestre R0 = Ox0y0 z0, dans la base de Oxyz. P17– 10. Centre d’inertie et moments d’inertie principaux d’un cône creux 1. Calculer la surface d’un cône creux en fonction de la hauteur h et du demi-angle au sommet g (Fig. 17.22). 2. Déterminer la position du centre de masse. 3. Trouver les moments d’inertie principaux du cône en son sommet. z

R γ

z R h

θ

O a

O F IG . 17.22.

F IG . 17.23.

 c Dunod – Toute reproduction non autorisée est un délit

P17– 11. Moments d’inertie principaux d’un tore homogène 1. Quels sont les moments d’inertie principaux d’un tore creux homogène, de rayon moyen a, de rayon de section droite R, en son centre de masse (Fig. 17.23) ? 2. Même question si le tore est plein. P17– 12. Moment d’inertie d’un anémomètre à coupelles Un anémomètre à coupelles est un appareil que l’on utilise pour mesurer la vitesse du vent. Il est constitué de quatre coupelles ayant la forme de demi-sphères creuses placées comme le montre la figure 17.24. Calculer, en fonction de la masse m de chaque coupelle, de leur rayon R et de la distance b de leurs centres à l’axe de rotation Oz de l’ensemble, le moment d’inertie de l’anémomètre par rapport à Oz.

308

17. Éléments cinétiques des systèmes matériels y 2R b 2b

−b

O

x

Oz b −b

F IG . 17.24.

F IG . 17.25.

P17– 13. Moment d’inertie d’un pendule de torsion Un pendule de torsion est constitué par quatre boules pleines identiques placées aux extrémités de quatre tiges identiques se coupant à angle droit en leur milieu (Fig. 17.25). L’ensemble tourne autour de l’axe perpendiculaire au plan des tiges passant par le centre de masse. Trouver, en fonction de la longueur b de chaque tige, de leur masse m, du rayon R de chaque boule et de leur masse M, le moment d’inertie du pendule par rapport à l’axe de rotation. P17– 14. Moments d’inertie de la molécule d’ammoniac La molécule d’ammoniac NH 3 possède un axe de symétrie d’ordre 3, l’axe Oz défini par l’atome d’azote et le centre du triangle équilatéral formé par les trois atomes d’hydrogène (Fig. 17.26). Dans la base centrale principale, l’opérateur d’inertie s’explicite selon une matrice diagonale dont les éléments s’écrivent en raison de la symétrie : I1 = I  et I2 = I3 = I⊥ 1. Établir l’expression de I 1 , en fonction des masses m N et m H , de la distance a = N − H ≈ 101, 4 pm et de l’angle HNH = h ≈ 107 ◦ . Calculer la valeur correspondante. 2. Même question pour I ⊥ .

101,4 pm

N

x

C H

y

107°

H

O H

F IG . 17.26.

P17– 15. Surface et fil en forme de triangle 1. Une distribution surfacique de masse, homogène est limitée par un triangle quelconque ABC , de côtés a , b et c . À l’aide de considérations géométriques simples, montrer que son centre de masse Gs coïncide avec le point de concours des médianes (Fig. 17.27a). 2. Une distribution linéique de masse, homogène, présente la forme d’un triangle quelconque ABC , de côtés a , b et c (Fig. 17.27b). Elle est constituée d’un fil de cuivre de masse volumique r = 8 900 kg.m−3 et de section s = 25 mm 2 . a) Montrer que son centre de masse G l est le même que celui de trois masses ponctuelles dont on donnera les positions A B C , et les valeurs en fonction de la masse linéique rl , a , b et c . b) Quelle est l’équation vectorielle qui détermine G l ? c) Par multiplication vectorielle, trouver trois équations linéaires reliant les distances de G l aux trois côtés du triangle A B C .

309

Éléments cinétiques des systèmes matériels

d) En déduire que Gl coïncide avec le centre d’un cercle particulier du triangle A  B C  . Trouver ce cercle et exprimer son rayon r en fonction de la surface S  et du périmètre p de ce triangle.

e) Retrouver la relation suivante entre la surface S  , a , b  , c  et p  , établie par Héron d’Alexandrie ( Ier siècle avant J.C.) : 2

S =

p 2

p − a 2

p − b 2

1/2

p − c 2

Calculer la masse du fil triangulaire ainsi que S  et r , dans les trois cas suivants, l’unité de longueur étant le centimètre : a = 10, 5

b = 16, 5

c = 21

a = 12

b = 16

c = 20

a = b = c = 16

3. Pour a = 10, 5 cm , b = 16, 5 cm et c = 21 cm , calculer le moment d’inertie principal du fil triangulaire par rapport à l’axe Gl z normal à son plan et passant par Gl . A

A b

c B

b

c C

a

B

C

a

a)

b) F IG . 17.27.

P17– 16. Quantité de mouvement et moment cinétique d’un ensemble tige et disque

 c Dunod – Toute reproduction non autorisée est un délit

On considère un disque plein homogène, de masse m, de rayon R, de centre C et mobile comme l’indique la figure 17.28a : OA est une tige homogène, de longueur l, de masse m, tournant dans le ˙ plan de la figure autour de l’axe Oz, à la vitesse angulaire u. 1. Le vecteur AC est assujetti à conserver une direction fixe, le disque étant constamment dans le plan Oxy, perpendiculaire à Oz. Exprimer en fonction de l’angle u = (Ox, OA), la quantité de mouvement et le moment cinétique du système en O. 2. Même question si le disque est solidaire de la tige, comme l’indique la figure 17.28b. y

y

A

C

C O

θ

A

O

x a)

θ x b)

F IG . 17.28.

310

17. Éléments cinétiques des systèmes matériels

P17– 17. Quantité de mouvement et moment cinétique de deux tiges homogènes articulées Deux tiges homogènes, de masse m1 et m2 et de longueurs l 1 et l2 , sont articulées en un point A1 (Fig. 17.29). L’ensemble peut osciller dans un plan vertical, autour d’un axe horizontal Oz passant par l’extrémité d’une des tiges. Exprimer, dans la base de R = Oxyz, la quantité de mouvement et le moment cinétique du système en fonction des paramètres angulaires u1 et u 2. z z y O l1 θ1 x

D θ

y

O A1

l2

θ2 F IG . 17.29.

x

V

y

ψ

x

C A

µ0 O

u F IG . 17.30.

F IG . 17.31.

P17– 18. Quantité de mouvement et moment cinétique d’une plaque triangulaire Une plaque triangulaire homogène (masse M), ayant la forme d’un triangle équilatéral de côté a, est mobile autour de l’origine O d’un référentiel R = Oxyz (Fig. 17.30). L’un de ses côtés OA balaie le plan Oxy. On considère le référentiel R  = Oxy  z lié à la plaque tel que Ox coïncide avec la médiane OC et que Oz  soit normal à la plaque. Exprimer, dans la base de R , la quantité de mouvement et le moment cinétique en O de la plaque, par rapport à R. P17– 19. Moment cinétique d’une barre tournant autour d’un axe Une barre homogène (masse m, longueur l, section négligeable) tourne à la vitesse angulaire V constante autour d’un axe D vertical, fixe par rapport à un référentiel R qui passe par son centre O (Fig. 17.31). La barre et l’axe forment à tout instant un angle constant u 0 dans un plan vertical. 1. Calculer directement, en fonction de u 0 et de V, le moment cinétique de L O dans la base liée à la barre. 2. Les deux vecteurs L et V sont-ils colinéaires pour u 0 quelconque ? Pour quelles valeurs de u 0 le sont-ils ?

18 Dynamique des systèmes matériels La dynamique des systèmes matériels s’appuie sur le principe fondamental de la dynamique qui est une généralisation de la loi fondamentale de la mécanique du corps ponctuel (cf. chapitre 4) et des théorèmes relatifs aux mouvements des systèmes de N corps (cf. chapitre 13) aux systèmes déformables quelconques. Il exprime donc la relation, d’une part, entre les éléments cinétiques, la quantité de mouvement et le moment cinétique, et, d’autre part, les forces et les moments qui s’exercent sur eux.

I . — FORCES APPLIQUÉES À UN SYSTÈME

 c Dunod – Toute reproduction non autorisée est un délit

Considérons un système déformable quelconque S d, schématisé par une distribution continue de masse, en mouvement par rapport à un référentiel R. Introduisons le vecteur lié générique (A, f v) représentant la force volumique qui s’exerce sur le point courant A. Un élément de matière, de volume d , autour d’un point A, est soumis à la force f v d exercée par l’ensemble des autres éléments de Sd et par des corps extérieurs à S d (Fig. 18.1). Si le référentiel par rapport auquel on étudie le mouvement du système n’est pas galiléen, f v d contient aussi les forces d’inertie d’entraînement et de Coriolis (cf. chapitre 7). Le système des forces appliquées à S d est donc représenté par l’ensemble des vecteurs liés {(A, fv d )}. z

f vd A O x

R

y

Sd

F IG . 18.1.

I . 1 . — Somme des forces et moment des forces Au système de force représenté par l’ensemble des vecteurs liés {(A, f v d somme S et le moment des forces : S=

fv d

et MO =

OA × f v d

)}, on associe la

312

18. Dynamique des systèmes matériels Il est facile d’établir la relation entre les moments en deux points distincts O et O  . En effet : MO =

d’où :

(OO + OA) × f v d

= OO ×

fv d

+

O A × f v d

M O = MO  + OO × S On voit que, si S = 0, le moment M est indépendant du point où on le calcule ; le système de forces est alors équivalent à un couple. I . 2 . — Torseur-force En raison de la relation entre les moments des forces en deux points distincts, on introduit le torseur-force [F ] dont les éléments de réduction en O sont la somme S et le moment MO. Rappelons que deux systèmes de forces sont torsoriellement équivalents si leurs torseurs associés sont égaux (cf. chapitre 1). Ainsi, le torseur associé aux forces de pesanteur, si le champ de pesanteur est supposé uniforme, est égal à celui associé à une force unique, dont le support passe par le centre de masse confondu ici avec le centre de gravité. De même, le torseur associé à des forces concourantes est égal à celui associé à une force unique dont le support passe par le point de concours. L’exemple représenté sur la figure 18.2 montre que l’équivalence torsorielle a une signification physique précise et limitée. En effet, dans les deux cas, ressort étiré et ressort comprimé, le torseur associé au système des deux forces opposées {F, −F} est nul. L’état physique du ressort est évidemment différent. Comme nous le verrons, ce n’est que relativement au seul principe fondamental de la dynamique que cette équivalence torsorielle de deux systèmes de vecteurs liés a un sens : seuls les mouvements du centre de masse et autour du centre de masse sont les mêmes. A1 F

A2 a)

A1 −F

−F

A2 b)

F

F IG . 18.2.

I . 3 . — Classification des forces a) Forces extérieures et forces intérieures à un système Sd

Comme nous l’avons déjà dit au chapitre 13, la force f v qui agit sur le point matériel A peut être considérée comme la somme de deux contributions : due à tout corps étranger au système déformable Sd ; c’est une force extéi) l’une f v,ex d rieure à Sd ; ii) l’autre f v,in d due à tout autre élément de matière appartenant au système Sd ; cette seconde contribution est une force intérieure à Sd . Aux systèmes de forces {f ex d } et {fin d }, on associe respectivement (S ex , MO,ex ) et (Sin , MO,in ). Ces quantités peuvent être considérées comme les éléments de réduction en O de deux torseurs, celui [Fex ] des forces extérieures et celui [F in] des forces intérieures. b) Forces données et forces inconnues Une seconde classification, indépendante de la précédente, distingue les forces dont on connaît l’expression, c’est le cas des forces fondamentales connues, des forces inconnues qui participent au mouvement du système et que l’on détermine seulement une fois le mouvement connu. Ces forces inconnues, qui sont le plus souvent des forces de contact, augmentent le nombre d’inconnues du problème.

313

Dynamique des systèmes matériels

II . — PRINCIPE FONDAMENTAL DE LA DYNAMIQUE Considérons le mouvement, par rapport à un référentiel galiléen R, d’un système matériel quelconque Sd , fermé, c’est-à-dire un système qui n’échange pas de matière avec le milieu extérieur (Fig. 18.3). Soit [PO] le torseur cinétique de S d par rapport à R au point O fixe de R et [F O,ex] le torseur des forces extérieures en ce même point. II . 1 . — Énoncé du principe fondamental Par rapport à tout référentiel galiléen R et en un point fixe O de R, le mouvement d’un système matériel Sd fermé satisfait à l’équation torsorielle suivante : d[PO ] = [FO,ex ] dt dans laquelle [FO,ex ] représente le torseur des forces exercées par les corps environnants. Remarque : Précisons bien, le torseur cinétique [PO] est relatif au référentiel galiléen R et sa dérivée est relative à la base de R. z

O x

Sd,2

Sd,1 y

Sd

R F IG . 18.3.

II . 2 . — Opposition des actions réciproques

 c Dunod – Toute reproduction non autorisée est un délit

Dans l’exposé de Newton de la dynamique, la troisième loi, appelée aussi principe des actions réciproques entre deux corps ponctuels ou principe de l’action et de la réaction, permet d’étendre la dynamique du corpuscule à celle des systèmes matériels quelconques (cf. chapitre 4 et 13). Dans la formulation torsorielle présente, l’axiomatique contenue dans le principe fondamental initial est plus forte ; aussi l’opposition entre les actions réciproques apparaît-elle comme un théorème. a) Théorème des actions réciproques Si un système matériel S d,1 exerce sur un autre système S d,2 le torseur force [F 1→2 ], Sd,2 exerce sur Sd,1 le torseur force opposé [F2→1 ] tel que : [F1→2 ] + [F 2→1 ] = [0] Pour établir le théorème de l’opposition des actions réciproques, subdivisons un système S d en deux sous-systèmes S d,1 et S d,2 et écrivons pour S d, S d,1 et S d,2, le principe fondamental de la dynamique (Fig. 18.3). Il vient, au point O, respectivement : d[P] d[P1] d[P 2 ] = [Fex ] = [Fex ] 1 et = [F ex]2 dt dt dt Si l’on distingue dans [Fex ]1 et [F ex]2 les forces d’interaction entre Sd,1 et S d,2 , on obtient : [Fex ]1 = [Fex→1 ] + [F 2→1]

et [Fex ]2 = [Fex→2 ] + [F 1→2 ],

avec

[Fex ] = [F ex→1] + [Fex→2 ]

314

18. Dynamique des systèmes matériels

D’autre part, en raison du caractère additif de [P] , on a : d[P] = [Fex ] + [F1→2 ] + [F2→1 ] dt Il en résulte, par comparaison avec l’équation torsorielle relative à S d, que [F 1→2 ] + [F2→1] = [0], d’où : F 1→2 + F 2→1 = 0 et M O,1→2 + MO,2→1 = 0 [P] = [P]1 + [P]2

d’où

Évidemment, si les systèmes S d,1 et Sd,2 se réduisent à des points matériels, on retrouve la troisième loi de Newton : F 1→2 + F2→1 = 0 Remarque : Entre deux particules chargées en mouvement, les forces magnétiques semblent ne pas satisfaire à l’opposition des actions réciproques. En réalité, cette opposition est rétablie si l’on considère les forces de Lorentz et non pas les seules contributions magnétiques à ces forces. Ce rétablissement prouve que les champs électrique et magnétique créés par les charges en mouvement sont fondamentalement imbriqués (cf. Relativité et invariance). b) Exemples Les exemples relatifs à l’opposition des actions réciproques entre deux corps ont déjà été considérés et sont nombreux. Rappelons qu’entre le Soleil et la Terre, assimilées à leurs centres, les forces de gravitation sont opposées. De même, entre deux particules chargées électriquement, les forces électrostatiques sont opposées (Fig. 18.4). Enfin, concernant les forces de contact, si un guide matériel G m exerce sur un corps ponctuel A une force de réaction R, A exerce sur Gm la force −R.

A1 F2→1

O

O

A2 F1→2

Rotor −G

Petit rotor

F IG . 18.4.

F IG . 18.5.

L’étude des actions de contact entre solides fournira de nombreux exemples d’opposition des moments de forces (cf. chapitres 19 et 25). Un exemple spectaculaire est fourni par le couple G qu’exerce le moteur lié à la carlingue d’un hélicoptère sur la grande hélice. En raison de l’opposition des actions réciproques, la grande hélice exerce un couple opposé −G, lequel tend à faire tourner la carlingue dans le sens opposé. On maintient l’hélicoptère sur sa ligne de vol à l’aide d’une petite hélice placée à l’arrière (Fig. 18.5).

III . — THÉORÈMES GÉNÉRAUX DE LA DYNAMIQUE III . 1 . — Théorèmes de la quantité de mouvement et du moment cinétique Si on explicite vectoriellement le principe fondamental de la dynamique des systèmes matériels, on obtient les théorèmes de la quantité de mouvement et du moment cinétique : dP dt

= S ex R

et

d LO dt

= M O,ex R

315

Dynamique des systèmes matériels

Comme P = MvC , le théorème de la quantité de mouvement est aussi appelé théorème du centre de masse : Le mouvement du centre de masse d’un système matériel est celui d’un corps ponctuel de masse égale à la masse du système et soumis à la somme des forces Sex . Remarques : (1) Les auteurs anglo-saxons appellent P et L respectivement moment linéaire et moment angulaire. Les conservations des grandeurs vectorielles P et L sont alors des conservations de moments. On trouve la justification de ces appellations en mécanique analytique où ces quantités sont les moments conjugués associés aux variables de position (cf. chapitre 24). (2) Si le référentiel n’est pas galiléen, il faut ajouter les forces d’inertie d’entraînement et de Coriolis aux forces créées par la présence des autres corps (cf. chapitre 7). Si le point où l’on exprime le torseur cinétique et le torseur des forces extérieures n’est pas fixe dans le référentiel galiléen considéré, la forme du théorème du moment cinétique diffère par l’addition d’un terme complémentaire. En effet, si l’on introduit ce point mobile O  , il vient : d LO = M O,ex soit dt

d (LO  + OO × P) = M O ,ex + OO × Sex dt

en introduisant O . On en déduit : d L O dP + vO × P + OO  × = MO ,ex + OO × Sex dt dt ce qui donne, en simplifiant : d LO  + vO × P = MO ,ex dt

 c Dunod – Toute reproduction non autorisée est un délit

Dans le cas particulier important où le point mobile est le centre de masse C, vC ×P = 0. Le théorème du moment cinétique prend alors la forme simple suivante : d LC = MC,ex dt

Remarques : (1) Il n’est pas inutile de souligner que l’application des théorèmes généraux (quantité de mouvement et moment cinétique), en un point, fournit les mêmes informations que celle en tout autre point. Le choix du point est donc strictement d’ordre technique : par exemple, on appliquera souvent le théorème du moment cinétique au point où s’exerce une réaction afin d’éliminer les inconnues supplémentaires. (2) Une application délicate du théorème du moment cinétique en un point mobile concerne le mouvement d’un solide S1 en contact avec un autre solide S 2 . Le point en lequel il convient dans ce cas d’écrire le théorème, afin d’éliminer la réaction de contact, est le point géométrique de contact I dont la vitesse est généralement non nulle. Les points I1 et I 2, qui ne coïncident avec I qu’exceptionnellement, n’ont pas cette propriété (cf. Exercices).

316

18. Dynamique des systèmes matériels

III . 2 . — Écriture en termes de bilan Les théorèmes de la quantité de mouvement et du moment cinétique peuvent s’écrire sous forme de bilans de grandeurs entre deux dates infiniment proches t et t + d t (cf. chapitre 13). Dans le cas de la quantité de mouvement, on a : d P = Sex d t Or, a priori : d P = dP(r) + dP (c) or dP(r) représente le terme reçu par l’échange avec le milieu extérieur et dP (c) le terme de création. On en déduit : dP(r) = Sex d t

et dP (c) = 0

Comme le terme de création est toujours nul, la quantité de mouvement d’un système quelconque Sd est une grandeur conservative. De même, le théorème du moment cinétique peut s’écrire : d LO = MO,ex d t Comme, a priori : d LO = dL(Or) + dL (Oc) (r)

(c)

dLO étant le terme reçu et dLO

le terme créé, on en conclut : dL(Or) = M O,ex d t

et dL (Oc) = 0

Le moment cinétique d’un système quelconque Sd est aussi une grandeur conservative. III . 3 . — Théorèmes généraux de la somme dynamique et du moment dynamique a) Somme dynamique On appelle somme dynamique d’un système matériel la quantité suivante analogue à la somme cinétique, mais construite à partir de l’accélération. Pour un système continu, on a : D=

ra d

d’où

D=

Ainsi : D=

d dt

rv d

=

d d P= (MvC ) dt dt

dP = Ma C dt

b) Moment dynamique Le moment dynamique d’un système matériel quelconque est la quantité suivante analogue au moment cinétique, mais construite à partir des accélérations : NO =

OA × ra d

317

Dynamique des systèmes matériels soit, si O est un point fixe de R : NO =

d dt

OA × rv d



v × rv d

Comme la seconde intégrale est nulle, la relation entre moment dynamique et moment cinétique est très simple dans ce cas : d LO NO = dt En un point O , mobile par rapport à R , on aurait : N O =

d dt

O A × rv d



(v − v O ) × rv d

soit

N O =

d LO + dt

vO × rv d

Ainsi, en un point mobile, la relation entre moment dynamique et moment cinétique fait apparaître un terme supplémentaire : d L O NO  = + vO × P dt c) Théorèmes généraux D’après ce qui précède, les théorèmes généraux de la dynamique peuvent s’écrire aussi, à l’aide de la somme et du moment dynamique : D = M aC = Sex

et N O = M O,ex

Cette dernière relation, contrairement à celle exprimée à l’aide du théorème du moment cinétique, est indépendante de la mobilité éventuelle du point O. En effet, en introduisant un point mobile O  , il vient : N O + OO × D = MO ,ex + OO × Sex soit N O = M O,ex puisque D = Sex. Ainsi :

 c Dunod – Toute reproduction non autorisée est un délit

En un point quelconque, mobile ou non, le moment dynamique d’un système matériel est égal au moment des forces extérieures. C’est sans doute pour cette raison, fixité ou non du point considéré, que certains auteurs utilisent cette dernière formulation des théorèmes généraux. Pour un physicien, cette écriture présente l’inconvénient majeur de sous-estimer l’importance considérable des éléments cinétiques, à la lumière des lois de conservation, telles qu’elles se manifestent dans les problèmes de collisions, en mécanique des fluides, en relativité (cf. Relativité et invariance) et en quantique (cf. Quantique). Rappelons que Newton, dès l’énoncé de sa deuxième loi, avait déjà compris toute l’importance du concept de quantité de mouvement en privilégiant ce dernier à celui, même pas technique, de quantité d’accélération. d) Autre énoncé du principe fondamental On peut pousser encore plus loin l’usage du formalisme torsoriel en considérant le torseur dynamique [D] dont les éléments de réduction sont respectivement D et NO . Le principe fondamental de la dynamique s’énonce alors par l’égalité du torseur dynamique [D] et du torseur des forces extérieures [F ]ex : [D] = [F ex ] égalité valable en n’importe quel point, fixe ou non par rapport au référentiel galiléen considéré.

318

18. Dynamique des systèmes matériels

III . 4 . — Cas d’un seul solide L’étude du mouvement d’un seul solide S relève évidemment des théorèmes généraux précédents. Cependant, dans ce cas particulier, le moment cinétique du solide au centre de masse C est relié à son vecteur vitesse de rotation V par une relation simple de la forme : LC = [I]C V On sait que le mouvement d’un solide est caractérisé a priori par six degrés de liberté ; par conséquent l’application des théorèmes généraux, qui fournit six équations scalaires, permet de déterminer ce mouvement si les forces qui s’exercent sur lui sont connues, c’est-à-dire si le solide n’est pas soumis à des liaisons. Dans le cas où les réactions aux liaisons ne sont que partiellement connues, il est indispensable de comparer le nombre de degrés de liberté et le nombre d’équations. Si le solide a un point fixe, le nombre de degrés de liberté se réduit à trois ; il est alors préférable d’appliquer le théorème du moment cinétique en ce point. Sinon, il vaut mieux appliquer ce théorème au centre de masse C. On a alors : dP = Ma C = Sex dt

et

d LC = MC,ex dt

avec LC = [I] C V

Enfin, on exploite efficacement ces équations en adoptant des axes commodes et en cherchant des constantes de mouvement : si toutes les forces sont parallèles, la projection de la quantité de mouvement du solide suivant cet axe est constante ; si le solide tourne autour d’un axe, on applique le théorème du moment cinétique en projection sur cet axe. Nous envisagerons ultérieurement plusieurs exemples concrets de mouvements d’un solide (cf. chapitres 25 et 26).

IV . — APPLICATIONS DES THÉORÈMES GÉNÉRAUX Les applications des théorèmes généraux de la dynamique aux systèmes matériels sont très nombreuses, car elles concernent tous les systèmes, déformables ou non, vivants ou non. Nous allons d’abord expliciter, sur un système déformable simple constitué de deux solides, la démarche d’analyse la plus appropriée pour minimiser les risques d’erreurs. Nous donnons ensuite quelques exemples d’application aux systèmes vivants, contrairement à l’ancien point de vue vitaliste, selon lequel ces systèmes ne satisferaient pas aux lois de la physique. IV . 1 . — Système articulé Considérons, par rapport au référentiel terrestre, le mouvement d’un système déformable S d formé de deux solides (1) et (2) articulés, une tige T et un disque D (Fig. 18.6). y g D

θ

T O

T

ψ F IG . 18.6.

D

x

319

Dynamique des systèmes matériels a) Définition du système

Définir un système, c’est délimiter la frontière qui le sépare de l’extérieur. On dit parfois isoler, mais ce terme ayant un sens précis en physique (influence nulle du milieu extérieur), il vaut mieux l’éviter. Dans l’étude du mouvement de ce système, on pourra appliquer les théorèmes généraux soit à l’ensemble du système soit au disque seul. b) Nature du référentiel Généralement, le référentiel considéré est le référentiel terrestre R. Comme nous l’avons vu au chapitre 7, ce référentiel réalise, avec une excellente approximation, un référentiel galiléen, pourvu que l’on introduise la notion de poids et que la force de Coriolis terrestre soit négligeable, ce qui est généralement le cas. Si le référentiel d’analyse est en mouvement accéléré par rapport à R, il faut ajouter, dans le bilan des forces, les forces d’inertie d’entraînement et de Coriolis. c) Nombre de degrés de liberté C’est le nombre de paramètres indépendants permettant de déterminer la position du système. Dans l’exemple considéré, la position de la tige est définie par un paramètre angulaire c puisque ce solide a un point fixe et que son mouvement de rotation est contenu dans un plan. La position du disque est, elle, définie par l’angle u, puisque le disque tourne dans son plan et que son centre est un point de la tige. Au total, le système possède donc deux degrés de liberté. d) Bilan des forces i) Forces extérieures Le poids de la tige −mg e y appliqué en son milieu T, le poids du disque −Mg e y appliqué en son centre D, le torseur des actions en O de vecteur RO, lequel est normal au plan Oxy si la liaison est parfaite (cf. chapitre 20). ii) Forces intérieures Le torseur des actions de contact en D qu’exerce la tige sur le disque, de vecteur R D et de moment GD, normal à ez si la liaison est parfaite, et le torseur des actions disque sur tige [−R D , −GD ].

 c Dunod – Toute reproduction non autorisée est un délit

e) Application des théorèmes généraux Les théorèmes généraux fournissent deux équations différentielles comportant uniquement les inconnues c et u. Dans le cas considéré où les liaisons sont supposées parfaites (cf. chapitre 20), GOz = 0 et GDz = 0. On obtient alors directement les deux équations différentielles en appliquant le théorème du moment cinétique en projection, respectivement : i) à l’ensemble S d, au point fixe O : d LO = OT × mg + OD × Mg + G O dt ii) au disque D, en son centre de masse D, (2)

d LD = GD dt Il est ainsi judicieux, pour obtenir directement le mouvement, d’appliquer le théorème du moment cinétique en un point où une réaction s’exerce afin d’éliminer cette inconnue. Le théorème de la quantité de mouvement sert alors à déterminer la réaction, une fois le mouvement connu.

320

18. Dynamique des systèmes matériels

f) Comparaison du nombre d’équations et du nombre d’inconnues Cette étape permet de faire le bilan du nombre E d’équations fournies par les théorèmes généraux, a priori six équations scalaires par solide, et du nombre I d’inconnues égal au nombre de paramètres nécessaires pour fixer la position du système (degrés de liberté) augmenté du nombre d’inconnues supplémentaires introduites par les forces de contact. Une fois assuré que E = I, on passe à l’étape de projection dans une base commode, ce denier choix étant d’ordre uniquement technique. g) Projection des équations vectorielles En fonction des paramètres c et u, les moments cinétiques de S d en O et de D en D s’expriment comme suit, en utilisant la propriété d’additivité et le théorème de Kœnig : (1)

(2)

L O = LO + L O

avec

(1)

LO =

ml 2 ˙ c ez 3

(2)

(2)

et L O = Ml2c˙ ez + LD

(2)

En outre, LD et OT × mg + OD × Mg s’explicitent respectivement selon : MR2 ˙ u ez 2

et R

(ml/2 + Ml) cos c (ml/2 + Ml) sin c 0

×

R

0 −g 0

On obtient alors les deux équations différentielles suivantes : 2 ml2 ¨ l ¨ + MR ¨ c + Ml2 c u = − m + Ml g cos c 3 2 2

et

MR 2 ¨ u=0 2

h) Résolution des équations différentielles La deuxième équation donne évidemment : u˙ = Cte = u˙ 0 . Quant à la première, elle s’écrit aussi, en posant ε = c + p/2, ml2 l + Ml2 ε¨ + m + Ml g sin ε = 0 3 2 On reconnaît, si ε est petit (c ≈ −p/2) , l’équation caractéristique d’un oscillateur harmonique de pulsation v0 et de période T 0 = 2p/v 0 (cf. chapitre 10) : v 20 =

(m/2 + M) g (m/3 + M) l

i) Analyse critique des solutions Cette dernière phase a au moins le mérite de rappeler que la mécanique est une discipline expérimentale. Il convient donc de tester la valeur du modèle représentant le système réel étudié. En outre, elle permet de s’assurer qu’aucune erreur ne s’est glissée : on a pu « oublier » une force, se tromper de signe dans une projection, etc. Ce type d’erreur peut être évité en comparant les résultats obtenus à une analyse intuitive rapide : par exemple une erreur de signe dans l’équation différentielle du mouvement d’un système qui « doit osciller » peut être corrigée aisément. Une manière de détecter efficacement la plupart des erreurs faites par des débutants consiste à contrôler systématiquement l’homogénéité des formules physiques : toute comparaison n’a de sens que si elle concerne deux grandeurs de même dimension. Enfin, les ordres de grandeurs constituent aussi un moyen puissant de contrôle, d’où l’intérêt des applications numériques concrètes.

321

Dynamique des systèmes matériels IV . 2 . — Systèmes vivants

Nous allons analyser le mouvement de quelques exemples de systèmes constitués d’êtres vivants, en commençant par une transposition de la célèbre expérience de Galilée sur la chute d’un corps du haut du mât d’un voilier. a) Chute d’un chat du haut du mât d’un voilier Répétons la célèbre expérience de la chute d’un boulet du haut du mât d’un voilier R , en mouvement de translation rectiligne uniforme par rapport au référentiel terrestre R g , à la vitesse constante v e . On sait que le boulet tombe à la verticale du mât. Mais qu’en serait-il si on remplaçait cet objet inanimé par un chat ? S’introduirait-il dans l’expérience « un élément de contingence qui ne rendrait plus nécessaire qu’il ne vienne s’écraser au pied du mât » ? Le résultat obtenu est évidemment celui que prévoit l’analyse newtonienne, à partir de la simple application des théorèmes généraux de la dynamique au chat. i) D’après le théorème de la quantité de mouvement, on a, par rapport à R galiléen, en négligeant l’influence de l’air : dP = Mc aC = Mc g dt

d’où

aC = g

vC = g t

et rC =

1 2 gt 2

puisque la vitesse initiale est nulle et l’origine prise à la position initiale. Le centre de masse du chat a donc même trajectoire que celle du boulet. Si la hauteur est h = 4 m , la durée de chute est : tc =

2×4 9, 81

1/2

≈ 0, 9 s

ii) L’application du théorème du moment cinétique au centre de masse mobile C du chat conduit à un moment cinétique constant, puisque le moment du poids en C est nul :

 c Dunod – Toute reproduction non autorisée est un délit

d LC =0 dt

entraîne

L C = Cte

Rien n’empêche évidemment le chat de tourner autour de C , en se déformant, de telle sorte que son moment cinétique en ce point garde sa valeur initiale nulle. Cette capacité à se retourner doit être attribuée aux capacités propres de l’animal et donc aux forces intérieures au chat, qui n’interviennent pas dans la conservation du moment cinétique (cf. chapitre 20). b) Poids d’un aquarium ouvert lorsque le poisson qu’il contient se met en mouvement Un aquarium de masse M a = 5 kg , reposant sur le plateau de gauche d’une balance, est en équilibre avec une tare, de même masse, placée sur le plateau de droite. Il contient un poisson initialement immobile de masse m = 100 g incluse dans Ma. Lorsque le poisson se met en mouvement dans l’aquarium, la quantité de mouvement du système constitué par l’aquarium varie puisque le centre de masse du système {aquarium-poisson} se déplace. Notons que le système n’est pas isolé : certes le poids de l’aquarium demeure constant, mais pas la réaction qu’exerce le plateau sur lui. L’équilibre de la balance est celui des plateaux, précisément des forces de réaction R a et Rt qu’exercent respectivement l’aquarium sur le plateau de gauche et la tare sur le plateau de droite.

322

18. Dynamique des systèmes matériels

En appliquant le théorème de la quantité de mouvement à l’aquarium et à la tare (masse M t ), on obtient respectivement : d Pa d Pt = Ra + M a g et = Rt + Mt g dt dt Il en résulte, puisque Ma = Mt : R a = Rt +

d Pa d Pt − dt dt

R a = Rt +

soit

d P x,a d Px,t − dt dt

en projection selon l’axe Ox vertical descendant. Initialement, il y a équilibre des plateaux car les quantités de mouvement sont toutes deux nulles : Ra = Rt . Si le poisson acquiert un mouvement selon la verticale, l’équilibre est rompu selon : Ra = R t +

d P x,a dt

Si le poisson, initialement au fond de l’aquarium, remonte brutalement, l’avantage est au plateau de la tare qui s’enfonce ; avec un vecteur accélération vertical de 5 m.s −2 , on trouve : Ra < Rt

puisque

d Px,a ≈ −0, 1 × 5 = −0, 5 N dt

à comparer à Ra et Rt qui valent tous deux à l’équilibre 5 × 9, 8 = 49 N . Cette différence est aisément détectable avec une balance sensible. c) Peintre sur une nacelle avec poulie Pour se hisser le long d’un mur, un peintre, de masse m p = 70 kg , utilise une nacelle, de masse mn = 20 kg , et une corde enroulée sur une poulie. L’une des extrémités est attachée à la nacelle et l’autre à un contrepoids K , de masse MK (Fig. 18.7). Par contact au point H de la corde qui porte K , le peintre exerce la force de tension F p→c dirigée selon la verticale. La corde est supposée inextensible et de masse négligeable. Appliquons le théorème de la quantité de mouvement au système nacelle-peintre, par rapport au référentiel terrestre. Il vient, en désignant par C le centre de masse de ce système, par T(N ) la tension du fil à l’extrémité supérieure N de la nacelle et par Fc→p la force exercée par la corde sur le peintre : (mn + m p)a C = (mn + m p)g + T(N) + Fc→p

soit

z = −(mn + mp )g + T (N ) − Fp→c,z (mn + mp)¨

en tenant compte de l’opposition des forces réciproques et en projetant selon la verticale ascendante. Pour le contrepoids et le brin de corde, on obtient de façon analogue, puisque la corde est de masse négligeable : M K aC  = MK g + T(H) + Fp→c

soit

MK ¨z = MK g − T (H ) + Fp→c,z

car, la corde étant inextensible, les accélérations de C et de C sont identiques. Il vient, en sommant les deux équations projetées membre à membre, les tensions T (N ) et T (H) étant égales (corde inextensible et poulie de masse négligeable) : (mn + mp + MK ) ¨ z = (M K − m n − mp)g

soit

¨ z=

M K − mn − mp g ≈ 0, 52 m.s−2 M K + mn + mp

323

Dynamique des systèmes matériels

L’accélération est nulle si M K = m n + m p . Dans ces conditions, l’application du théorème de la quantité de mouvement à l’ensemble {peintre-nacelle-contrepoids} donne : dP = (M K + mn + m p) g + R = 0 dt

d’où

R = −2(mn + m p) g

On suppose réalisée la condition précédente, le contrepoids étant au même niveau que la nacelle, de telle sorte que le peintre peut se voir dans un miroir plan accroché au contrepoids. Le peintre grimpe le long du brin de corde NI qui tient la nacelle, afin d’atteindre la poulie. Écrivons à nouveau le théorème de la quantité de mouvement aux deux systèmes, le peintre avec la nacelle d’une part et le contrepoids d’autre part. On a, respectivement, en désignant par Q le point où le peintre se trouve sur la corde : d Pp = (mn + mp )g + T(Q) dt

et

d PK = M Kg + T(K ) dt

Comme MK = mn + mn et T(K) = T(Q) , les accélérations de part et d’autre sont égales. Les vitesses aussi puisque leurs valeurs initiales le sont. Il en résulte que le contrepoids se déplace vers la poulie avec la même vitesse que le peintre, ce qui permet à ce dernier de s’observer au cours de l’ascension. J

T(H)

I

T(N) N

H

Mur

g K

Nacelle

Contrepoids  c Dunod – Toute reproduction non autorisée est un délit

F IG . 18.7. — Peintre sur une nacelle avec poulie

V . — LOIS DE CONSERVATION V . 1 . — Variation de la quantité de mouvement et du moment cinétique Nous avons vu que la quantité de mouvement et le moment cinétique d’un système matériel quelconque étaient des grandeurs conservatives, c’est-à-dire que, dans le bilan de ces grandeurs, les termes de création de quantité de mouvement et de moment cinétique sont toujours nuls ; quant aux forces et aux moments, ils traduisent l’échange avec le milieu extérieur : DP = P(r) =

Fex d t

(r)

et DL O = LO =

MO,ex d t

324

18. Dynamique des systèmes matériels

V . 2 . — Conservation de la quantité de mouvement et du moment cinétique Pour un système S d isolé, c’est-à-dire seul dans l’espace ou soumis à des actions négligeables, on a Sex = 0 et MO,ex = 0. Il en résulte que P et LO se conservent : DP = P(r) = 0

(r)

et DL O = L O = 0

d’où

P = Cte

et

LO = Cte

Il en est de même si le système est soumis à des forces et des moments qui se neutralisent : S ex = 0 et MO,ex = 0 ; on dit qu’il est pseudo-isolé. Ce cas est très fréquent. V . 3 . — Exemples de conservation a) Boule sur un plan horizontal Considérons une boule se déplaçant sans frottement sur un plan horizontal ; comme la boule reste en contact avec le plan, elle est soumise à deux forces verticales (pesanteur et réaction normale) qui s’annulent. Sa quantité de mouvement est donc constante. b) Danseuse en rotation et personne sur un tabouret d’inertie Le système déformable que constitue une danseuse tournant autour d’un axe vertical est soumis au poids et aux actions de contact exercées par le sol (Fig. 18.8). Or le poids étant vertical, son moment par rapport à l’axe vertical est nul. En outre, le moment des actions de contact suivant l’axe est aussi nul car la liaison avec le sol peut être supposée parfaite (cf. chapitre 20). Le moment de toutes les actions extérieures par rapport à l’axe de rotation est donc nul. Il en résulte que le moment cinétique par rapport à cet axe est constant. Ainsi, en ramenant ses bras le long du corps, ce qui diminue son moment d’inertie, la danseuse augmente sa vitesse de rotation. Ωi

Ωf

Ωi

Ωf

g B2

F IG . 18.8.

B1

F IG . 18.9.

Une variante pédagogique de cette expérience consiste à illustrer la conservation de la projection verticale du moment cinétique du système constitué d’une personne portant des masses identiques à bout de bras et assise sur un tabouret (Fig. 18.9) ; ce dernier tourne parfaitement autour d’un axe vertical, ce qui implique, comme nous le verrons plus loin (cf. chapitre 20), que la projection du moment des actions de contact axe-tabouret sous ce dernier soit nulle. Lorsque la personne, initialement les bras tendus, ramène ses bras le long du corps, la vitesse de rotation finale Vf du système, assimilé à un solide, est plus élevée que sa vitesse de rotation initiale Vi . On a, en effet, en raison de la conservation de la composante du moment cinétique suivant l’axe de rotation : d’où Vf > V i Ii Vi = I f V f puisque les moments d’inertie satisfont à l’inégalité If < Ii .

325

Dynamique des systèmes matériels

Ordre de grandeur : une danseuse peut tripler sa vitesse de rotation initiale, qui est de l’ordre d’un tour par seconde, en ramenant ses bras le long du corps. En effet, si on la représente par un cylindre de masse m = 55 kg et de rayon de gyration 15 cm , lorsque ses bras, de longueur 50 cm et de masse 3,5 kg, sont verticaux, on a : 2 × 3, 5 × (2 × 0, 5) 2 ml 2 = 1, 2 + = 3, 5 kg . m 2 3 3 Soulignons que seules les actions extérieures (poids et action de contact axe-tabouret) interviennent, d’où l’inutilité d’analyser le rôle précis des forces intérieures, par exemple l’action du corps sur les bras. En outre, le seul référentiel à considérer est celui du laboratoire, pratiquement galiléen, dans lequel évidemment aucune force d’inertie n’est à considérer. Raisonner dans le référentiel non galiléen lié au tabouret est inutile, inefficace et source d’erreurs. I f ∼ 55 × 0, 152 = 1, 2 kg . m 2

et Ii ∼ If +

Remarque : Les cuisiniers affirment pouvoir distinguer un œuf dur d’un œuf cru en les faisant tourner sur la table de travail autour d’un axe vertical. Le premier garde pratiquement sa vitesse angulaire car son moment d’inertie ne change pas ; le second s’immobilise rapidement car, en tournant, les masses intérieures s’éloignent de l’axe de rotation et augmentent le moment d’inertie. c) Mise en mouvement d’un tabouret d’inertie à l’aide d’une roue en rotation Une personne, assise sur un tabouret d’inertie au repos et tenant à bout de bras l’axe horizontal d’une roue en rotation, peut provoquer la mise en rotation du tabouret en inclinant l’axe de la roue par rapport à l’horizontale (Fig. 18.10). En effet, on a, d’après la conservation du moment cinétique, suivant l’axe vertical Oz, du système constitué de la personne sur le tabouret (t, p) et de la roue (r) : ,p LOz = 0 = LtOz + LrOz

d’où

t,p

LOz = −L rOz

Ainsi, les moments cinétiques suivant l’axe vertical de la personne sur le tabouret et de la roue sont opposés. Il en résulte que le tabouret et la roue tournent en sens inverses. L (t,p)

LOz

(r)

 c Dunod – Toute reproduction non autorisée est un délit

LOz

T

w

L Lune

a)

b) F IG . 18.10.

Terre F IG . 18.11.

d) Conservation du moment cinétique de la Lune Du fait de son mouvement de révolution autour de la Terre et de sa vitesse de rotation propre V L autour de son axe de rotation, la Lune, assimilée à une sphère de masse ML et de rayon R L , possède un moment cinétique total LT (Fig. 18.11). Ce dernier a pour expression, en désignant par r = TL la distance Terre-Lune et w˙ la vitesse angulaire du mouvement orbital : 2 L T = TL × M L vL + L∗ avec TL × ML vL = ML r2w˙ e z et L∗ = IV ≈ ML R 2LV L 5 On sait que le moment cinétique orbital se conserve puisque la force de gravitation exercée par la Terre est centrale (cf. chapitre 12). Quant au moment cinétique propre, il se conserve aussi puisque le moment

326

18. Dynamique des systèmes matériels

des actions gravitationnelles au centre L est nul. Comme la vitesse angulaire w˙ de révolution est égale à la vitesse de rotation propre VL , la Lune présente toujours la même face à la Terre.

CONCLUSION Il convient de retenir les points fondamentaux : (1) Les théorèmes généraux de la dynamique des systèmes matériels fermés quelconques s’écrivent, en un point fixe O : dP d LO = maC = Sex et = MO,ex dt dt Ces dérivées s’entendent relativement au référentiel R par rapport auquel on étudie le mouvement. Si R n’est pas galiléen, il faut ajouter les forces d’inertie d’entraînement et de Coriolis. (2) Dans le cas d’un solide S, le théorème du moment cinétique ne diffère de ce qui précède que par l’équation qui relie le moment cinétique au centre de masse C et le vecteur vitesse angulaire de rotation de S par rapport à R : LC = [I]C V

Pour la technique du calcul, précisons que la quantité de mouvement et le moment cinétique sont souvent exprimés dans une base liée au solide qui ne coïncide pas nécessairement avec la base du référentiel par rapport auquel on applique les théorèmes généraux. Il faut alors utiliser la composition des dérivations de Bour. (3) Par une démarche rationnelle d’analyse on peut aboutir de façon rapide et sûre aux équations différentielles du mouvement d’un système déformable. Une fois ces équations résolues, il convient de contrôler ces résultats, sinon par l’expérience, au moins par notre intuition newtonienne. Il n’est pas inutile d’ajouter que les théorèmes généraux concernent aussi les systèmes vivants. (4) Enfin, des exemples d’illustration de cas où la quantité de mouvement et le moment cinétique se conservent rappellent qu’il convient d’exploiter les équations vectorielles avant de les expliciter en trois équations dans une base quelconque.

EXERCICES ET PROBLÈMES P18– 1. Conservation de la quantité de mouvement Un homme, de masse m 1, se trouve sur un radeau de masse m 2 qui repose à la surface d’un lac. La résistance de l’eau est négligée. Trouver la relation entre le déplacement r1 de l’homme sur le radeau et le déplacement Dr2 du radeau par rapport à la rive. Commenter. P18– 2. Conservation du moment cinétique Un homme, assimilé à un point matériel de masse m, se déplace sur le bord d’un disque circulaire, avec une vitesse vh par rapport au disque. Ce dernier, de masse M et de rayon R, peut tourner sans frottement autour de son axe de révolution vertical. Initialement l’ensemble est au repos. 1. Donner les expressions des moments cinétiques du disque et de l’homme. 2. Établir la relation entre la vitesse de déplacement de l’homme sur le disque et la vitesse de rotation du disque. En déduire l’angle dont on a tourné le plateau, en fonction du rapport des masses, lorsque l’homme a parcouru un demi-tour.

327

Dynamique des systèmes matériels

P18– 3. Cerceau lesté roulant sans glisser sur une droite Un cerceau homogène C (centre K, masse M, rayon R) lesté par un point matériel A (masse m) situé sur son périmètre, roule sans glisser sur une droite fixe, horizontale, d’un référentiel terrestre R = Oxyz. Son plan reste dans le plan Oxy, Oy étant la verticale ascendante et Ox un axe horizontal (Fig. 18.12). 1. Trouver l’équation différentielle du mouvement en u = (KI, KA), à l’aide du théorème du moment cinétique appliqué au point de contact I. 2. Quelle est la période des petits mouvements ?

y g r O

g C1

K θ

m1

A(m)

A

I

x

F IG . 18.12.

r1 F IG . 18.13.

P18– 4. Mouvement du centre de masse d’une poulie mobile Calculer l’accélération a 1, suivant la verticale ascendante, du centre C1 du disque dans le système représenté par la figure 18.13, sachant qu’il y a roulement sans glissement des poulies sur le fil. La grande poulie est un cercle matériel, de masse m1 et de rayon r1 ; la masse de A est m et celle de la petite poulie négligeable. Discuter la valeur de a 1 suivant le rapport h = m/m1.

 c Dunod – Toute reproduction non autorisée est un délit

P18– 5. Rupture d’une cheminée d’usine Une tige homogène T , de masse m et de longueur l, tourne dans un plan vertical Oxy autour d’un axe horizontal Oz passant par l’une de ses extrémités O, fixe dans un référentiel terrestre R = Oxyz. On désigne par u l’angle que fait la tige avec la verticale descendante Ox et par g la norme du champ de pesanteur terrestre. 1. a) Appliquer les théorèmes généraux de la mécanique à la tige : le théorème de la quantité de mouvement, en projection dans la base cylindrique {e r, eu , ez}, et le théorème du moment cinétique en O. On admet que le torseur des actions de contact en O a pour éléments de réduction R et GO→T = G O→T ez : b) En déduire une intégrale première du mouvement, sachant que u˙ = 0 lorsque u = p. c) Montrer que la réaction R qui s’exerce sur la tige a pour expression : 3mg mg R = −a mg cos u − er+ sin u eu 2 4 a étant un facteur positif que l’on déterminera. 2. La tige OT en léger mouvement autour de sa position u = p représente schématiquement une cheminée d’usine fixée au sol en O (Fig. 18.14). On se propose d’évaluer la répartition de la tension qui s’exerce en un point quelconque K de la tige situé à la distance r de O. Pour cela, on considère

328

18. Dynamique des systèmes matériels

la portion de tige OK formant le solide S1 et on désigne par [R2→1 , G2→1,K ] le torseur des actions qu’exerce sur S1 le reste KT de la tige. (1) a) Montrer que le moment cinétique de S 1, au point K , a pour expression : L K = −g(mr3/l) u˙ e z, g étant une constante positive que l’on déterminera. b) Appliquer le théorème du moment cinétique à S 1 au point mobile K. En déduire que G 2→1,K se met sous la forme : G2→1,K = lmgl sin u u(1 − u)2 ez , u étant le rapport r/l et l un facteur positif que l’on déterminera. c) Représenter avec soin le graphe f (u) = u(1 − u) 2 dans le domaine 0  u  1. Montrer que la valeur de G2→1,K passe par un maximum pour une valeur de u que l’on calculera. En déduire l’endroit de la cheminée où se produit préférentiellement une éventuelle rupture. y

T K

B

I g

r Oz

C

θ y O

x F IG . 18.14.

α

A x

F IG . 18.15.

P18– 6. Mouvement d’une barre en contact sans frottement avec deux plans perpendiculaires Une barre AB homogène, de section négligeable (masse m, longueur l), est posée sans vitesse initiale dans le plan vertical Oxy : les extrémités A et B sont en contact, sans frottement, avec les axes Ox et Oy respectivement (Fig. 18.15). On repère la position de la barre par l’angle a = (Ox, OC). À l’instant initial a = a0 . 1. Quelle est la trajectoire de C ? Appliquer le théorème du centre de masse et en déduire deux équations du mouvement. 2. Trouver l’équation du mouvement en a, à l’aide du théorème du moment cinétique au point mobile I, intersection des directions des réactions de contact en A et B. 3. En déduire une intégrale première du mouvement, c’est-à-dire une équation reliant a˙ et a. 4. Écrire la condition qui réalise la rupture du contact avec le plan vertical. Exprimer, en fonction de a0 , la valeur a1 de a à l’instant où le contact en B cesse. P18– 7. Mouvement d’une chaîne Une chaîne métallique AB, de longueur constante L = 1 m et de masse M, est constituée d’un ensemble de maillons assimilés à des points matériels reliés les uns aux autres. On la pose sur une table horizontale, près d’un bord O, de telle sorte qu’une partie de la chaîne pende verticalement (Fig. 18.16). 1. Sachant que le frottement solide de la table sur la chaîne est négligeable, établir l’équation différentielle à laquelle satisfait le mouvement de l’extrémité basse B de la chaîne. 2. Quelle est la nature du mouvement ? Initialement x = 1 cm ; trouver la durée au bout de laquelle la longueur x = OB est égale à L.

329

Dynamique des systèmes matériels A

O g

Table B

x

F IG . 18.16.

P18– 8. Mécanique du football Un ballon de football peut être considéré approximativement comme une sphère (creuse), de rayon r = 11 cm et de masse M = 0, 420 kg . Cette sphère, en matériau polymère, d’épaisseur négligeable devant r , renferme de l’air sous une pression de 2 bar et une température de 300 K . À titre d’information, les dimensions des terrains utilisés en France, lors de la Coupe du Monde 1998, étaient 105 m × 68 m . 1. a) La masse de l’air est négligeable devant celle de l’enveloppe du ballon assimilé à une sphère (creuse). Trouver, en fonction de M et r , le moment d’inertie I du ballon par rapport à un axe passant par son centre de masse C . Calculer I en précisant son unité SI. b) Avec vos connaissances de thermodynamique, justifier l’approximation précédente par un calcul simple du nombre de moles d’air contenues dans le ballon. On prendra comme valeur de la masse molaire de l’air à 300 K : Mm = 29 g . On rappelle que 1 bar = 105 Pa et que la constante universelle des gaz parfaits vaut R ≈ 8, 314 J.mol−1.K−1 . 2. Un joueur communique au ballon, selon l’axe horizontal Cx , une vitesse v C , de valeur vC = 108 km.h −1 , ainsi qu’une rotation angulaire stationnaire V , orientée selon l’axe horizontal Cy , perpendiculaire à vC (Fig. 18.17a). a) Exprimer les vitesses des points supérieur et inférieur du ballon, si la vitesse angulaire vaut, en tours par seconde, V = 5 tr.s−1 . Calculer ces vitesses en précisant l’unité SI. b) Quelle devrait-être la valeur de V , en tour par seconde, pour que la vitesse du point inférieur du ballon soit nulle ?

 c Dunod – Toute reproduction non autorisée est un délit

c) Lorsque le ballon reste en contact avec la pelouse du terrain, la valeur précédente correspond à une condition cinématique bien connue ; laquelle ? Les valeurs précédentes des vitesses sont-elles réalistes dans une telle phase de jeu ? 3. On s’intéresse au cas où le ballon est tiré de l’un des quatre coins du terrain vers le gardien adverse : c’est le coup de pied de coin ou corner. À l’instant initial, C se trouve au point O , situé à 11 cm du sol. Sa vitesse initiale vi est contenue dans le plan vertical Oyz qui fait un angle u avec le plan vertical contenant les deux poteaux de la cage (Fig. 18.17b) ; dans ce plan, le vecteur vi est incliné d’un angle ai par rapport au plan horizontal du terrain. En outre, la vitesse de rotation angulaire communiquée par le tireur est nulle. On suppose qu’il n’y a pas de vent et que l’air ambiant n’exerce aucune force ou moment sur le ballon. a) Le référentiel R , lié au terrain, est supposé galiléen. Justifier, en quelques lignes les raisons pour lesquelles un tel référentiel terrestre peut être considéré comme galiléen. b) Écrire, sous forme vectorielle, pour le ballon, le théorème de la quantité de mouvement et le théorème du moment cinétique au centre de masse C , par rapport à R . Dans ce dernier cas, le théorème du moment cinétique peut être appliqué au point mobile C sans terme complémentaire. Justifier. c) Quelles sont les équations différentielles du mouvement auxquelles satisfait le mouvement de C dans ce plan ? Il s’agit d’un mouvement bien connu ; sans intégrer les équations différentielles, pouvez-

330

18. Dynamique des systèmes matériels

vous dire lequel et donner la nature de la trajectoire ? Montrer que la trajectoire de C est contenue dans un plan que l’on précisera. Le tireur de corner peut-il envoyer le ballon directement dans la cage adverse ? d) Calculer la somme des moments des forces extérieures qui s’exercent sur le ballon. Ce dernier acquiert-il une vitesse angulaire ? Si oui, calculer sa valeur en tours par seconde, si non, justifier. 4. Le tireur communique au ballon non seulement la vitesse initiale v i précédente contenue dans le plan vertical OYz , mais aussi une vitesse angulaire V = Vz ez autour d’un axe vertical. L’air ambiant exerce alors sur le ballon une force dite de Magnus, appliquée en C et d’expression FM = 2ra b V × v C où ra = 1, 3 , en unité SI, est la masse volumique de l’air et b le volume du ballon. a) Quelle est la dimension physique de la quantité m = 2r a

b /M

? Calculer sa valeur.

b) Comment s’écrivent vectoriellement le théorème du centre de masse et le théorème du moment cinétique en C ? Que peut-on dire du vecteur vitesse de rotation angulaire du ballon ? c) Un tireur adroit envoie directement le ballon dans la cage adverse, en communiquant à V z un signe convenable. Préciser lequel en le justifiant. d) En décomposant la vitesse de C en deux contributions vectorielles, l’une verticale vC,v et l’autre vC,h dans un plan horizontal, selon vC = vC,v + v C,h , montrer que vC,h satisfait à l’équation vectorielle suivante : d v C,h = VM × vC,h dt VM étant un vecteur dont on donnera la direction, le sens et l’expression de la norme en fonction de m et V . Commenter. z g

S

Terrain de football x

Cy

vC

y

Premier poteau

u

Oz

I a)

b) F IG . 18.17.

Second poteau

19 Lois de Coulomb sur le frottement solide

Dans la pratique, il arrive très souvent que l’on ait à étudier le mouvement d’un solide en contact avec un autre solide. On doit alors tenir compte, en plus des forces à distance telles que la pesanteur, de nouvelles forces dites forces de contact (Fig. 19.1a). C’était le cas lorsqu’on avait étudié le mouvement sans frottement d’un point matériel gêné astreint, par des liaisons bilatérale ou unilatérale, à demeurer en contact avec un guide matériel (cf. chapitre 9). C’est le cas aussi si on abandonne un solide S 1 sur un plan incliné S 2 (Fig. 19.1b) ; on constate que le solide reste au repos pourvu que l’angle d’inclinaison soit suffisamment faible ; si l’on augmente ce dernier, S 1 aborde un mouvement de glissement.

 c Dunod – Toute reproduction non autorisée est un délit

Ces forces de contact jouent généralement un rôle essentiel dans la vie courante ; par exemple, la marche à pied n’est possible que grâce à ces forces.

Bien qu’elles réduisent le nombre de paramètres dont dépend le mouvement d’un solide, l’étude de ce mouvement est toujours plus difficile qu’en leur absence. En effet, le calcul de ces forces est très complexe, car il dépend de la nature exacte de l’interaction entre des ensembles de particules, de la position de ces particules au voisinage des surfaces et par conséquent de la structure fines des surfaces. Ces forces ne sont donc pas fondamentalement simples, comme les forces de gravitation ou les forces électromagnétiques : les quelques informations que l’on a sur elles sont valables dans des domaines limités et on peut dire que leur complexité augmente avec la précision désirée. Aussi est-il nécessaire de proposer d’abord un modèle macroscopique simplifié de ces actions.

g

g

Forces de contact a

a)

b) F IG . 19.1.

332

19. Lois de Coulomb sur le frottement solide

I . — ACTIONS DE CONTACT I . 1 . — Modèle macroscopique On schématise les actions de contact qu’exerce, sur un solide S 1 , un autre solide S 2 en considérant que, sur la surface de S1, en contact avec S 2, agit un ensemble de forces. On désigne par R la somme de ces forces de contact et par GI la somme de leurs moments (Fig. 19.2) ; R et G I sont les éléments de réduction du torseur des actions de contact [R I ] au point I. Introduisant le plan tangent commun Q et sa normale n, orientée vers le solide S 1 qui subit l’action, R et G I s’écrivent aussi : R = R t + Rn

et GI = G I,t + GI,n

Par définition, R n est la réaction normale, Rt la force de frottement ou force de résistance au glissement ou force d’adhérence, G I,n le moment de résistance au pivotement, GI,t le moment de résistance au roulement au point I. Si le contact est quasi ponctuel, GI est négligeable, car les forces de contact passent toutes près de I. Le plus souvent, on le suppose nul ; cependant, il est indispensable d’en tenir compte dans certains cas, notamment lorsqu’il n’y a pas de frottement ou pas de glissement. Rn G z O x

R

S1 Gn

Gt y

Q

Rt

I S2

R F IG . 19.2.

I . 2 . — Origine microscopique des forces de contact L’origine microscopique des forces de contact est de nature électromagnétique entre particules chargées ; en comprimant un solide sur un autre, on réduit les dimensions des zones de confinement des particules chargées qui constituent les solides. On montre alors que ces particules s’agitent davantage en cognant plus souvent les parois de ces zones (cf. Quantique). Les forces moyennes qui résultent à l’échelle macroscopique de la variation de quantité de mouvement, en un point I de la surface de contact, sont alors bien décrites par les vecteurs R et GI .

II . — LOIS SUR LE FROTTEMENT SOLIDE Les lois phénoménologiques du frottement de glissement entre solides ont été introduites dès le siècle par Léonardo di Vinci puis étudiées par l’ingénieur français G. Amontons en 1699 et par C. Coulomb en 1779. XVe

333

Lois de Coulomb sur le frottement solide II . 1 . — Réaction normale Rn a) Son sens

La réaction normale exercée sur S 1, par le solide de S2 , est dirigée vers l’intérieur de S 1. Ce résultat se conçoit aisément puisque la réaction s’oppose généralement à la pénétration d’un solide dans l’autre : la réaction normale est donc une force répulsive. De même, la réaction normale qu’exerce S1 sur S2 est répulsive, puisque −R n est dirigée de S1 vers S2. Du point de vue microscopique, on peut avancer que, lors de la pénétration de S 1 dans S2, les atomes de S2 sont comprimés et donc exercent ensemble une force répulsive vers S 1.

Remarque : Si les solides sont de même nature et ont une grande surface de contact plane (deux glaces ou deux glaçons par exemple), on observe au contraire une réaction attractive, car les atomes appartenant aux deux solides peuvent se mêler intimement entre eux. Dans la suite, nous exclurons ce cas singulier de solides possédant de grandes surfaces lisses de contact. b) Sa norme La réaction normale a une valeur arbitraire qui dépend des conditions du mouvement ou de l’équilibre et des autres actions extérieures qui s’exercent sur S 1. Elle ne pourra donc être déterminée qu’une fois le mouvement connu. II . 2 . — Réaction tangentielle Rt Les lois auxquelles satisfait la réaction tangentielle sont différentes suivant la valeur nulle ou non de la vitesse de glissement vg , que l’on notera parfois explicitement v1/2, lorsqu’il s’agira du glissement du solide S1 sur le solide S 2. a) Cas où vg = 0

 c Dunod – Toute reproduction non autorisée est un délit

Exerçons sur le solide S 1, en contact sans glissement avec le solide S2, une force de traction F située dans le plan Q tangent en I aux surfaces limitant S1 et S 2 (Fig. 19.1b). L’expérience montre que la vitesse de glissement reste nulle tant que F n’atteint pas une valeur maximale Rt,m  telle que : Rt,m = m sRn 

m s étant un facteur de proportionnalité appelé facteur d’adhérence statique ou de frottement statique. Expérimentalement, on constate les deux propriétés suivantes exprimées sous forme de lois. 1e loi : Le facteur de frottement m s est indépendant de l’aire de la surface de contact. 2e loi : Le facteur de frottement m s est indépendant de Rn .

La courbe expérimentale représentée sur la figure 19.3a, relative au contact entre deux feuilles de papier, traduit bien cette loi. On a donc, dans le cas du non-glissement, l’inégalité suivante : Rt   m sR n Géométriquement, la réaction R est située à l’intérieur d’un cône de révolution, d’axe la normale en I au plan Q, de sommet I et de demi-angle fs = arctan m s ; ce cône est appelé cône de frottement (Fig. 19.3b).

334

19. Lois de Coulomb sur le frottement solide Rt 14 R

10 6

S1

Ás

¹s = 0;38 Q

I

2 10

20

30

S2

Rn

a)

b) F IG . 19.3.

b) Cas où vg = 0

1e loi : La force de frottement R t qu’exerce S 2 sur S1 a même support que la vitesse de glissement vg : R t × vg = 0. 2e loi : La force de frottement R t qu’exerce S2 sur S 1 a un sens opposé à celui de la vitesse de glissement v g : Rt · vg < 0. 3e loi : Pour une vitesse de glissement fixée, la norme de la force de frottement est proportionnelle à la norme de la réaction normale. On a, en introduisant le facteur d’adhérence dynamique ou de frottement dynamique m : Rt  = mRn On définit aussi l’angle d’adhérence ou de frottement dynamique f par : tan f = m. 4e loi : Le facteur de frottement dynamique est indépendant de la vitesse de glissement, et inférieur au facteur de frottement statique : m < ms. Le tableau 19.1 donne les valeurs de m pour quelques matériaux. Contact

m

acier – acier

0,2

bois – bois

0,3

garniture frein – acier

0,45

caoutchouc – bitume

0,6

TAB . 19.1.

Remarques : (1) L’étude du contact caoutchouc-bitume est décisive dans le choix des pneumatiques de véhicules. Une analyse détaillée montre que la dépendance m(v g), dans ce cas, n’est pas simple et qu’elle varie notablement avec la charge du véhicule et la pression de gonflage (Fig. 19.4). (2) On peut généraliser les lois de Coulomb au frottement de roulement et de pivotement. Il suffit de remplacer v 1/2 et R t , respectivement par V 1/2,t et GI,t pour le frottement de roulement, et par V 1/2,n et GI,n pour le frottement de pivotement ; les facteurs de proportionnalité sont différents de m .

335

Lois de Coulomb sur le frottement solide

¹

Contact pneumatique-bitume

¹s

m1 0

Patin S1 Piste S2

v 1/2 F IG . 19.4.

Ressort E

Fd

F IG . 19.5.

II . 3 . — Mesure du facteur de frottement Pour déterminer expérimentalement le facteur de frottement statique entre deux surfaces planes de solides S1 et S2 en contact, on exerce sur S1 une force F d, à l’aide d’un dynamomètre à ressort (Fig. 19.5). À la naissance du glissement de S 1 sur S 2, la valeur F d de cette force de traction est égale à msRn . On a donc, puisque Rn ≈ m 1 g : ms =

Fd m1 g

L’expérience faite avec deux plaques métalliques, dont l’une est recouverte d’une couche d’un matériau qui augmente l’adhérence, a donné le résultat suivant : Fd = 0, 5 dN pour m1 = 0, 1 kg, d’où ms ≈ 0, 51. Remarque : Une méthode de détermination du facteur de frottement m consiste à mesurer la période des oscillations horizontales d’une plaque qui repose sur des galets (méthode de Timochenko) ; cette plaque est soumise à deux causes de mouvement antagonistes : l’une due au sens de rotation des galets, l’autre à l’action du poids d’une masselotte (cf. Exercices).

 c Dunod – Toute reproduction non autorisée est un délit

II . 4 . — Description plus précise du contact Une analyse plus fine montre que le contact de deux surfaces est en réalité constitué de microcontacts dont l’aire effective A e est très inférieure à l’aire apparente des surfaces. Ces micro-contacts se font par les aspérités qui parsèment les surfaces et se déforment sous l’action des forces normales et tangentielles. Les composantes normale et tangentielle de la réaction R se mettent alors sous la forme : Rn = pA e

et Rt,m = sA e

p étant un coefficient de pression associé à la réaction normale et s un coefficient de cisaillement, tous deux caractéristiques du matériau. Il en résulte que : ms =

s Rt,m  = p Rn

On explique le faible domaine de variation de ms , lorsqu’on change de matériau, par une modification dans le même rapport des coefficients s et p.

336

19. Lois de Coulomb sur le frottement solide

III . — APPLICATIONS III . 1 . — Statique d’un solide sur un plan incliné Considérons un solide S en contact avec un plan incliné d’un angle a par rapport à l’horizontale (Fig. 19.6a). D’après le théorème du centre de masse, l’immobilité de S sur le plan implique que la somme des forces extérieures soit nulle. En effet, on a : maC = mg + R avec

aC = 0 R

y

Rn R x ®

Rn

O

Filets g

Vis

® Rt

Rt a)

b) F IG . 19.6.

Explicitant suivant les axes Ox et Oy , il vient : 0 = mg sin a + Rt

et 0 = −mg cos a + R n

Rt et Rn étant les composantes tangentielle et normale de la réaction R . Par conséquent : R n = mg cos a > 0 et Rt = −mg sin a < 0 Comme il n’y a pas de glissement : |Rt |  ms |Rn|

soit

tan a  m s

et a  f s

L’immobilité implique donc l’inégalité suivante : a  fs f s étant le demi-angle du cône de frottement. Ce résultat, selon lequel le solide S ne glisse pas sur le plan incliné pourvu que l’angle du plan soit inférieur à l’angle de frottement f s , est connu sous le nom d’effet arc-boutement. Il permet d’expliquer qu’une vis ne se desserre pas spontanément si la condition précédente est respectée. Sur la figure 19.6b, on a représenté schématiquement une vis enfoncée verticalement dans un bâti ; ce dernier exerce sur la vis et donc sur ses filets, inclinés d’un angle a par rapport au plan horizontal perpendiculaire à l’axe de la vis, des forces verticales qui s’opposent à la pénétration de la vis. Ainsi, l’un des filets est soumis à une force verticale ascendante R , de composantes R n selon la normale au filet et Rt selon sa tangente. Comme dans le cas précédent, la vis restera immobile si la condition a < f s est réalisée. Il en résulte, fs étant plus grand pour une vis à bois que pour une vis à métal, que les angles choisis pour a sont différents dans les deux cas ; par exemple : a(bois) ∼ 20 ◦

et a(métal) ∼ 8 ◦

337

Lois de Coulomb sur le frottement solide

Remarques : (1) La condition d’arc-boutement ne fait pas intervenir la masse du solide, ce que confirme approximativement l’expérience. En effet, si l’on considère plusieurs solides, de même nature, mais de masses différentes, en contact avec un plan incliné d’angle variable, on constate qu’ils se mettent à glisser, de façon hésitante, à partir d’un certain angle (Fig. 19.1b) ; ils s’arrêtent puis accélèrent, s’arrêtent à nouveau, etc. Ce comportement s’explique par les qualités différentes des surfaces en contact et par toutes sortes de matériaux étrangers tels que poussières, graisses, oxydes, etc. (2) Le cas singulier où a = p/2 correspond évidemment à l’impossibilité du contact : Rn = 0 . III . 2 . — Roue porteuse Afin de comprendre l’intérêt de la roue porteuse, comparons les forces qui sont nécessaires pour déplacer, à vitesse constante, deux masses identiques, l’une ayant la forme d’un cube et l’autre celle d’un cylindre (Fig. 19.7). y

y

R

g

C O

R

g

F1 x

C O

I

F2 x

b)

a) F IG . 19.7.

a) Calcul de F1 pour un cube Avec les notations habituelles, le théorème du centre de masse appliqué au cube (masse m ), qui est soumis à son poids, à la réaction du plan horizontal et à F1 , s’écrit, puisque la vitesse est constante : ma = mg + R + F1 = 0 En explicitant dans la base de Oxy , on obtient :  c Dunod – Toute reproduction non autorisée est un délit

0 = Rt + F1

et 0 = −mg + Rn

Il en résulte, puisqu’il y a glissement, que |Rt | = m|R n| . Comme R t < 0 , il vient Rt = −mRn et par conséquent : F1 = mmg . b) Calcul de F2 pour un cylindre Si le cylindre (masse m , rayon r ) roule sans glisser, les théorèmes généraux donnent, la vitesse du centre de masse C étant constante, ainsi que le moment cinétique en C : ma = mg + R + F2 = 0

et

d LC = GC = GI + CI × R = 0 dt

G I étant le moment des actions de contact en I . En explicitant, on obtient, puisque ¨ u , d’après x = −r ¨ la condition de roulement sans glissement (x˙ + Ru˙ = 0) : 0 = R t + F2

0 = −mg + R n

I¨ u = −I

¨ x = 0 = G Iz + rRt r

338

19. Lois de Coulomb sur le frottement solide

Par conséquent, F2 = −Rt = G Iz/r . Comme le contact est quasi ponctuel, G Iz  mmgr ; on en déduit que : F2  mmg

et

F2  F1

On explique ainsi ce que l’expérience courante montre quotidiennement : il est plus facile de faire rouler un objet que de le faire glisser, d’où l’intérêt des véhicules à roues. III . 3 . — Marche à pied L’analyse des forces qui s’exercent sur une personne marchant sur un sol horizontal montre que la marche n’est possible que grâce aux forces de frottement ! En effet, le théorème du centre de masse appliqué à la personne, de masse M , donne (Fig. 19.8) : MaC = Mg + R soit

Ma C = R t et 0 = M g + R n

selon le plan horizontal et la verticale respectivement. Ainsi, le mouvement n’est possible que grâce aux forces de frottement. Il convient cependant de ne pas attribuer à ces forces un rôle autre que celui d’un intermédiaire passif : c’est en prenant appui sur le sol grâce aux frottements que l’individu peut se déplacer ; si les frottements sont insuffisants l’appui et donc la marche sont impossibles. g g

y r

x

R

C I

®

Oz F IG . 19.8.

F IG . 19.9.

III . 4 . — Roue motrice La plupart des véhicules à roues se déplacent grâce à un moteur dont la fonction est d’imposer une rotation à des roues motrices qui roulent sans glisser sur le sol. Les actions qui s’exercent sur la roue, de centre de masse C , sont le couple-moteur M C = −Mm ez avec Mm > 0 (le signe moins est seulement dû à l’orientation de l’axe des z ), le poids mg et la réaction R exercée par le sol. Étudions dans quelles conditions, une telle roue peut rouler sans glisser, à vitesse constante, sur un plan incliné dans le sens de la montée (Fig. 19.9). Pour cela, écrivons dans le référentiel R = Oxyz les théorèmes généraux de la mécanique. Il vient : d LC = GI + CI × R + M C = 0 dt puisque la vitesse de C est constante, ainsi que la vitesse angulaire en raison du roulement sans glissement. En explicitant dans la base de R , on obtient : maC = mg + R = 0

0 = −mg sin a + Rt

et

0 = −mg cos a + Rn

et 0 = GIz + rR t − M m

Pour que la condition de non-glissement soit satisfaite, il faut que : |Rt|  ms|R n| puisque Mm  |GIz| .

soit

|M m − G Iz| M ≈ m  ms mg cos a r r

339

Lois de Coulomb sur le frottement solide Par conséquent, la montée n’est possible que si :

i) L’adhérence m s est suffisamment grande. La bonne valeur de ms , dans le contact caoutchoucbitume ( ms ≈ 0, 6 ), explique la relative facilité avec laquelle un camion avec remorque franchit une rampe inclinée. De ce point de vue, le contact acier-acier dans le transport par voie ferrée ( ms ≈ 0, 2 ) est un handicap pour obtenir une grande force de traction, alors qu’il est énergétiquement avantageux. ii) La masse m des roues motrices est suffisamment grande ; c’est ce qui est réalisé dans les trains de montagnes qui sont équipés de deux locomotives, non pour aller plus vite mais pour augmenter la masse des roues motrices. iii) Le couple moteur n’est pas trop grand, sinon il y a glissement et les roues patinent. Remarque : L’analyse des équations du mouvement montre que la force de frottement Rt est, dans le cas d’une roue motrice, dirigée dans le sens du mouvement : R t = mg sin a . Bien que ce soit elle qui, en dernier lieu, permette la montée, il convient là aussi de ne pas lui attribuer un rôle autre que celui d’un intermédiaire passif : le rôle actif est évidemment joué par le couple-moteur M C . III . 5 . — Disque vertical en mouvement sur un axe horizontal Soit un disque D (masse m , rayon r ) posé verticalement sur l’axe horizontal Ox d’un référentiel terrestre R = Oxyz , avec initialement une vitesse angulaire égale à u˙0 = −V 0 e z (V 0 > 0) et une vitesse de son centre de masse C égale à v0 e x (Fig. 19.10). y D

g C

r

N µ I

O

Ω0

x

F IG . 19.10.

 c Dunod – Toute reproduction non autorisée est un délit

Les forces extérieures étant le poids mg = −mg e y et la réaction du sol R = Rt ex + Rn ey , les théorèmes généraux s’écrivent, pour le mouvement de D dans le plan vertical Oxy : ma C = mg + R

et

d LC = CI × R dt

et

0 0 = u mr2 ¨ 2

ce qui donne, en explicitant dans la base de R : ¨x m 0 = 0

Rt 0 −mg + R n 0 0

0 −r × 0

Rt Rn 0

On obtient donc les équations suivantes : ¨x =

Rt m

2R u= t r¨ m

Rn = mg

Les deux premières équations contiennent les trois inconnues x, u, R t . Par conséquent, la prise en compte des lois de Coulomb sur le frottement est indispensable.

340

19. Lois de Coulomb sur le frottement solide

Cependant, il convient, au préalable, d’étudier le comportement de la vitesse de glissement du disque sur l’axe, vg = v I1 /R , I1 étant le point de D qui coïncide avec le point de contact géométrique I . Comme I1 et C appartiennent à D , on a vI1 = v C + IC × V, ce qui s’explicite selon :

R

˙x 0 0 + r 0 0

×

0 0 = ˙u

˙x + ru˙ 0 0

En combinant les équations du mouvement, on obtient : d vg 3Rt 3Rt = (¨x + r ¨ u) ex = ex = dt m m On voit, en multipliant les deux membres de l’égalité par v g , que : d dt

v 2g 2

=

3Rt ·vg< 0 m

puisque R t et vg sont opposés. Ainsi, la norme de la vitesse de glissement v g ne peut que diminuer. Il en résulte que le mouvement se termine par une phase sans glissement.

CONCLUSION Énumérons les résultats essentiels. (1) Les forces de contact jouent un rôle déterminant dans la vie courante, du fait de leur présence dans la plupart des problèmes mécaniques concrets (vis, roue porteuse, roue motrice, etc.). (2) Elles diminuent le nombre de degrés de liberté mais augmentent le nombre d’inconnues du problème. En effet, ces forces phénoménologiques traduisent un effet global d’actions microscopiques complexes, d’où une ignorance partielle de leurs expressions. (3) Malgré leur caractère très approché, les forces de contact rendent compte de façon satisfaisante du mouvement des solides. Deux cas doivent être considérés suivant que la vitesse de glissement vg d’un solide sur l’autre est nulle ou non : Si v g = 0 , on a l’inégalité Rt   ms Rn  . Si v g =  0 , on a l’égalité Rt  = mR n . (4) Enfin, dans la méthode de résolution d’un problème où ces forces interviennent, il convient de distinguer les différentes phases de mouvement, d’abord le glissement puis le non-glissement, et de traduire algébriquement la relation entre les normes des composantes tangentielle et normale des réactions. Les aspects énergétiques des forces de contact seront étudiés ultérieurement (cf. chapitre 20).

341

Lois de Coulomb sur le frottement solide

EXERCICES ET PROBLÈMES P19– 1. Mise en mouvement d’un bloc sur un plan incliné Étudier le mouvement du système représenté sur la figure 19.11. La masse de la poulie est négligeable, le fil est inextensible et le facteur de frottement de la masse 2 sur le plan incliné est m. Application numérique : h = m1/m2 = 0, 5 ; a = 20◦ et m = 0, 15. Barre

g

m2 x

g

Planche

α

F

m1

F IG . 19.11.

F IG . 19.12.

P19– 2. Contact de deux solides avec frottement Une barre, de masse m 1, est placée sur une planche de masse m2 ; l’ensemble repose sans frottement sur un plan horizontal (Fig. 19.12). Le facteur de frottement entre la barre et la planche est m. On exerce sur la planche une force horizontale F dont l’intensité croît linéairement avec le temps : F(t) = at, a étant une constante. 1. Écrire les équations différentielles du mouvement de la barre et de la planche. 2. Déterminer l’instant t0 à partir duquel la planche glisse sous la barre. 3. Quelles sont les accélérations de la barre et de la planche dans les phases de non-glissement et de glissement ? P19– 3. Cylindre sur un tapis roulant incliné

 c Dunod – Toute reproduction non autorisée est un délit

Un cylindre, de masse m, de rayon r, est posé, sans vitesse, sur un tapis roulant. Ce tapis, incliné d’un angle a sur l’horizontale, se déplace à la vitesse constante v0 suivant l’axe Ox d’un référentiel terrestre R = Oxyz (Fig. 19.13). On note g l’intensité du champ de pesanteur terrestre et m le facteur de frottement. 1. Exprimer la vitesse de glissement v g en fonction de v 0 et des paramètres suivants : x C (position du centre de masse C suivant l’axe x) et u (angle de rotation du cylindre). 2. Écrire les équations différentielles du mouvement. 3. Trouver, en fonction de g, a et m, la dérivée par rapport au temps de la vitesse de glissement. 4. Étudier les différentes phases du mouvement. z g

y

C

v0 θ

g x

I α O

O

C

y I

x

F IG . 19.13.

F IG . 19.14.

P19– 4. Sphère sur un plan horizontal Une sphère homogène (masse m, rayon r) se déplace en contact avec le plan horizontal Oxy d’un référentiel terrestre R = Oxyz. 1. Écrire, sous forme vectorielle, le théorème du centre de masse C et le théorème du moment cinétique appliqué en C (Fig. 19.14).

342

19. Lois de Coulomb sur le frottement solide

2. Établir la relation vectorielle entre la dérivée par rapport au temps de la vitesse de glissement v g et la force de réaction tangentielle R t. En déduire que le mouvement final est toujours un mouvement sans glissement. P19– 5. Expérience de Timochenko : oscillations d’une barre reposant sur deux galets Une barre de section carrée, de masse M et de centre de gravité C, repose en O 1 et en O 2 sur deux galets G1 et G 2 de centres fixes. Elle est reliée par un fil à une masselotte, de masse m, qu’on assimilera à un point A (Fig. 19.15). Le galet G 1 est mis en rotation par un moteur, dans le sens indiqué sur la figure, avec une vitesse angulaire constante et assez grande pour que le frottement en O1 , de facteur m constant, se fasse toujours dans le même sens. Le galet G2 et la poulie P, sur laquelle s’enroule le fil, sont eux, en revanche, mis en rotation par la barre ; ils tournent parfaitement autour de leurs axes et leurs masses peuvent être négligées. Le contact en O2 entre la barre et le galet G 2 se fait sans frottement. On suppose que le fil n’a aucune masse, aucune raideur et aucun allongement. On notera, comme sur la figure, l la longueur O1O 2, 2h le côté de la section de la barre, F t la tension du fil, N1 et T1 les composantes de la réaction R 1 en O1 et N2 la composante normale de la réaction R2 en O2. 1. Appliquer le théorème de la quantité de mouvement successivement à la masselotte seule et à la barre seule. Appliquer le théorème du moment cinétique à la barre seule. 2. En déduire, en fonction de x, N 1 et T1 . 3. Quelle est l’équation du mouvement ? Montrer que ce mouvement est oscillant. En déduire la période des oscillations et l’abscisse du point autour duquel se font ces oscillations. Comment peut-on utiliser ce dispositif pour mesurer le facteur de frottement m ? N2 g

z

N1

2h

C O2 G2

O1 T 1 x l

G1

P A

Ft

Ω0

g

x

S x

O

F IG . 19.15.

F IG . 19.16.

P19– 6. Cylindre sur un plan horizontal Un cylindre de révolution S, homogène, de masse M, de rayon R, est posé sur un plan horizontal Oxy, le contact se faisant le long d’une génératrice (Fig. 19.16). À l’instant t = 0, son axe de révolution est parallèle à Oy et son mouvement se réduit à une rotation de vitesse angulaire V 0 positive autour de cet axe. Le facteur de frottement du cylindre sur le plan est m. À quel instant t1 , la vitesse de glissement de S sur le plan s’annule-t-elle ? Quelle est la distance parcourue entre t = 0 et t = t 1 ? Quel est le mouvement de S après t1 ? P19– 7. Influence du freinage sur le mouvement propre d’un véhicule Un véhicule se déplace en ligne droite suivant un axe horizontal Ox de telle manière que son plan de symétrie coïncide avec un plan fixe que l’on prend comme plan de figure (Fig. 19.17). Toutes les forces sont contenues dans ce plan. On suppose pour simplifier que les centres des roues sont à la même hauteur h, au-dessus du sol, que le centre de masse C. 1. Quelle est la relation entre la vitesse du centre de masse x˙ C et la vitesse de rotation des roues ˙ du fait du roulement sans glissement ? u,

343

Lois de Coulomb sur le frottement solide

2. Appliquer le théorème du centre de masse et le théorème du moment cinétique dans le référentiel du centre de masse. 3. En déduire les expressions des composantes normales N 1 et N 2 des réactions qu’exercent le sol au niveau des roues. 4. Montrer que, si x¨C > 0, N 1 > N2 et si ¨ xC < 0, N 1 < N2. Commenter.

N2 N1

F IG . 19.17.

P19– 8. Différentes phases du mouvement d’un disque

 c Dunod – Toute reproduction non autorisée est un délit

Un disque D (masse m, rayon r) est posé verticalement sur l’axe horizontal Ox d’un référentiel terrestre R = Oxyz, avec une vitesse angulaire égale à ˙u0 = −V0 ez (V0 > 0) et une vitesse de son centre de masse C égale à v0 ex (Fig. 19.10). Étudier le mouvement dans les trois cas suivants : v0 > rV0, v 0 = rV0 et v0 < rV0 . On représentera, dans chacun des cas, la vitesse de glissement et la force de frottement en fonction du temps.

20 Énergétique des systèmes matériels

Le travail de l’ensemble des forces qui s’exercent sur un système matériel s’obtient en sommant les travaux de toutes les forces appliquées aux différents points de ce système, forces extérieures et forces intérieures au système. Nous nous proposons de généraliser au cas de systèmes matériels quelconques, constitués de corps ponctuels et de solides, la relation existant entre l’énergie cinétique d’un corpuscule et le travail des forces appliquées. Nous serons alors conduits à établir, pour ces systèmes, les expressions de l’énergie cinétique, de l’énergie potentielle et de l’énergie mécanique. Nous analyserons enfin le cas important où, en raison de la nature parfaite des liaisons, l’énergie mécanique du système se conserve. Établissons d’abord l’expression du travail des forces qui s’exercent sur un système matériel.

I . — TRAVAIL DES FORCES QUI S’EXERCENT SUR UN SYSTÈME Considérons, dans un référentiel R, un système de corps ponctuels, a priori déformable, S d , dont l’élément générique Ai a une vitesse vi (Fig. 20.1).

z

F2→1

Sd O x

R

Fi

z0

A1 A2 Ai Aj

F1→2

y O

y

R0 x

F IG . 20.1.

345

Énergétique des systèmes matériels I . 1 . — Travail des forces extérieures et travail des forces intérieures

On sait que la puissance des forces {A i , F i} qui s’exercent sur S d s’écrit, relativement à R (cf. chapitre 13) : Fi · v i = F ex→i · vi + Fj→i · v i = Pex + P in P= i

i

i,j

où Fex→i désigne la force qu’un élément extérieur à Sd exerce sur Ai et Fj→i la force qu’exerce, sur ce même point Ai , un autre point A j de Sd. Le premier terme représente la puissance des forces extérieures à Sd , le second, la puissance des forces intérieures. La somme discrète une distribution continue.

doit être remplacée par une intégrale si le système peut être représenté par

On obtient le travail élémentaire en multipliant P par la durée élémentaire d t : où

dW = dW ex + dWin

dWex = i

Fex→i · d OAi

et

dW in = i,j

F j→i · d OAi

L’intégration entre les positions initiale et finale des différents points du système donne le travail des forces extérieures à Sd et celui des forces intérieures à S d : W ex = i

et Win =

Fex→i · d OAi

i,j

Fj→i · d OAi

En général, le calcul de ces intégrales est facilité par les propriétés particulières des forces. I . 2 . — Propriété remarquable du travail des forces intérieures Nous nous proposons d’établir une propriété remarquable du travail des forces intérieures directement reliée à l’opposition des actions intérieures réciproques. Exprimons, dans deux référentiels R et R  en mouvement quelconque l’un par rapport à l’autre, la puissance des forces intérieures à un système Sd (Fig. 20.1) :

 c Dunod – Toute reproduction non autorisée est un délit

Pin =

i,j

Fj→i · v i

et P in =

i,j

Fj→i · vi

d’où

Pin − Pin =

i,j

Fj→i · (vi − v i )

En tenant compte de la composition des vitesses entre R et R , vi = v i + vO + V × O  A i, si V est le vecteur vitesse de rotation de R  par rapport à R il vient : Pin − Pin =

i,j

Fj→i · (vO + V × O A i ) =

i,j

Fj→i · vO +

i,j

O A i × F j→i · V

Comme la somme et le moment des forces intérieures sont nuls : Sin =

Fj→i = 0

et MO,in =

i,j

i,j

O A i × Fj→i = 0

il en résulte que : P in = Pin

et

dWin = dWin

Ainsi, la puissance et le travail des forces intérieures d’un système sont indépendants du référentiel par rapport auquel on les calcule.

346

20. Énergétique des systèmes matériels

Ce résultat incite à calculer la puissance ou le travail des forces intérieures dans le référentiel où ils sont les plus simples à étudier. C’est là une propriété remarquable que confère l’opposition des actions réciproques. Soulignons que P in et dWin sont en général non nuls.

Dans de nombreux cas, le travail des forces intérieures d’un système matériel déformable S d peut se mettre sous la forme de la différentielle d’une fonction : dW in = − d Ep,in (r 1, r2 , . . . , rn) Ep,in étant l’énergie potentielle associée aux forces intérieures. C’est le cas de l’énergie gravitationnelle ou de l’énergie électrostatique entre plusieurs charges électriques (cf. Électromagnétisme). I . 3 . — Travail des forces extérieures. Énergies potentielles associées Pour calculer le travail élémentaire des forces extérieures qui s’exercent sur chacun des éléments du système, on somme sur tous ces éléments. Fréquemment, le travail élémentaire peut se mettre sous la forme de la différentielle d’une fonction : dW ex = − d Ep,ex , Ep,ex étant l’énergie potentielle extérieure associée. a) Travail des forces de pesanteur. Énergie potentielle de pesanteur Le travail élémentaire des forces de pesanteur qui s’exercent sur un système matériel a pour expression, si le champ de pesanteur g est uniforme, ce que nous supposerons (Fig. 20.2a) : dW ex = i

On a donc :

m ig · d OAi = g ·

i

mi d OAi = Mg · d OC = d(Mg · OC) E pg = −Mg · OC + Cte

dWex = − d E pg avec

Si z C désigne la cote du centre de masse C suivant l’axe Oz vertical ascendant, E pg s’écrit : E pg = Mgz C + Cte Le travail du poids entre les positions 1 et 2 du centre de masse C est alors : W ex = −DEpg = −{Mgz C }21 = Mg(zC,1 − zC,2 ) y

z zC

O x

ae

C Sd

g

R a)

y

y

O

O z

R

z

R b)

F IG . 20.2.

V Hi

C

Ai

Sd

Sd

x

y

O x

R

y x c)

R

347

Énergétique des systèmes matériels b) Travail des forces d’inertie d’entraînement de translation. Énergie potentielle associée

Désignons par R  = O x y z  un référentiel en translation accélérée par rapport à un référentiel galiléen R. Dans R, le travail élémentaire de la force d’inertie d’entraînement de translation a pour expression, puisque aO ne dépend pas du point Ai de S d (Fig. 20.2b) : dW ex = − D’où :

i

mi aO · d OAi = −aO ·

dW ex = − d Epe

avec

i

m i d O A i = −MaO · d OC

E pe = MaO · OC + Cte

Cette expression est analogue à celle de l’énergie potentielle de pesanteur : le champ de pesanteur uniforme g est remplacé par le champ d’accélération uniforme −aO . c) Travail des forces d’inertie d’entraînement de rotation uniforme. Énergie potentielle centrifuge Intéressons-nous au mouvement d’un système matériel S d par rapport à un référentiel R en rotation uniforme autour d’un axe fixe du référentiel du laboratoire R, supposé galiléen. Chaque élément du système, de masse mi , est soumis à une force centrifuge de la forme mi V 2Hi Ai , V étant la vitesse angulaire de rotation uniforme de R  par rapport à R et Hi la projection de A i sur l’axe de rotation (Fig. 20.2c). Calculons le travail élémentaire des forces centrifuges qui s’exercent sur le système : dWex = i

m i V2H iA i · d OA i = V 2

i

mi Hi Ai · d Hi Ai =

V2 2

mi d(H i Ai )2 = d i

V2 IOz 2

puisque HiA i ·d OHi = 0 et IOz est le moment d’inertie du système par rapport à l’axe Oz. Finalement, dW = − d Epc

avec

Epc = −

V2 IOz + Cte 2

II . — TRAVAIL DES FORCES QUI S’EXERCENT SUR UN SOLIDE

 c Dunod – Toute reproduction non autorisée est un délit

II . 1 . — Travail des forces intérieures dans un solide Associons au solide S considéré en mouvement par rapport au référentiel R un repère R  (Fig. 20.3). Dans R, tous les points du solide sont fixes ; par conséquent, la puissance des forces dans ce référentiel est nulle : P in = 0. Comme cette puissance est indépendante du référentiel, en raison de l’opposition des actions réciproques, la puissance et donc le travail des forces intérieures à un solide sont nuls : P in = 0 et W in = 0 z z S

y

O y

O x

x R

R

S1 I1 Q

I I2 S2

F IG . 20.3.

F IG . 20.4.

348

20. Énergétique des systèmes matériels

II . 2 . — Travail élémentaire des forces extérieures Évaluons le travail élémentaire des forces extérieures qui s’exercent sur un solide S. Si A est un point de S, il vient, compte tenu du champ des vitesses d’un solide, de vecteur vitesse angulaire V par rapport au référentiel d’étude R : Fex→i · vi d t =

dWex = i

ce qui s’écrit : dW ex = i

Fex→i · v A d t +

i

Fex→i · (vA + Ai A × V) d t

(AAi × Fex→i ) V d t

i

soit

dW ex = (Sex · v A + MA,ex · V) d t

en désignant par Sex et M A,ex la somme et le moment des forces extérieures. En introduisant le torseur des forces extérieures [F ex ]A et le torseur cinématique [v] du solide S, le travail des forces extérieures qui s’exercent sur un solide se met sous la forme du produit scalaire des deux torseurs précédents : dWex = [Fex ][v] d t II . 3 . — Travail des forces extérieures de contact Appliquons le résultat général précédent au cas où la force extérieure qui s’exerce sur un solide S 1 est due à un autre solide S2 fixe dans R, en contact ponctuel en I avec S 1. Notant v I1 la vitesse du point I1 de S1 qui coïncide avec I, R la somme des actions de contact qu’exerce S 2 sur S1 et G I le moment en I de ces actions, il vient (Fig. 20.4) : dW ex = R · vI1 d t + G I · V1 d t V1/R désignant la vitesse de rotation de S 1 par rapport à R. Comme le contact est ponctuel, le moment GI est nul. Il en résulte que dWex se réduit à : dWex = R · vI 1 d t

Ce travail est nul dans deux cas : 1er cas : R est normal à vI1 qui est contenu dans le plan tangent Q : il n’y a pas de frottement.

Exemple : Une masselotte A glisse sans frottement sur une tige T qui tourne uniformément dans un plan horizontal autour d’un axe vertical Oz avec la vitesse de rotation V (Fig. 20.5). Relativement au référentiel R  , dans lequel la tige est fixe, le travail dWex = R · vI 1/R  d t est nul car vI1 /R est normal à R. En revanche, dans R, dW ex = R · vI1 d t n’est pas nul, car vI 1 /R n’est pas normal à R. 2e cas : v I1 = 0 et donc il n’y a pas de glissement, puisque v I1 = vI2 = 0, v I2 étant la vitesse du point I2 de S2 qui coïncide avec I 1. z z g

V

y

O x

R

T

A

R R

x F IG . 20.5.

y

vA

I2

I1

S2 F IG . 20.6.

S1

349

Énergétique des systèmes matériels

Exemple : Une sphère roule sans glisser sur un plan incliné. Bien que la réaction ne soit pas normale au plan, la puissance des forces de contact est nulle, car la vitesse v I1 du point de la sphère en contact avec le plan est nulle (Fig. 20.6).

III . — TRAVAIL TOTAL DES ACTIONS DE CONTACT ENTRE SOLIDES Considérons deux solides S 1 et S 2 en mouvement par rapport à un référentiel R et assujettis à rester en contact pseudo-ponctuel : le moment des actions de contact n’est pas négligeable. Notons I l’un des points géométriques de contact et désignons par I1 et I2 les points respectifs de S 1 et S 2 qui coïncident avec I à l’instant considéré (Fig. 20.4). III . 1 . — Expression de la puissance totale des actions de contact Si R 2→1 et G 2→1 sont la somme et le moment des actions de contact qu’exerce S 2 sur S1 , le travail élémentaire de ces forces de contact s’écrit : dW2→1 = (R2→1 · vI1 + G 2→1 · V 1 ) d t

D’après l’opposition des actions réciproques, le travail élémentaire des forces de contact qu’exerce S1 sur S2 en I est égal à : dW1→2 = −(R 2→1 · v I2 + G 2→1 · V2) d t. Donc, le travail élémentaire total dWt a pour expression : dWt = dW2→1 + dW1→2 = [R2→1 · (vI1 − vI2 ) + G 2→1 · (V1 − V2 )] d t

Finalement, en introduisant la vitesse de glissement de S 1 par rapport à S 2 en I et la vitesse angulaire relative : v1/2 = v I1 − vI2 et V1/2 = V1 − V2 on obtient l’expression suivante de la puissance totale des actions de contact : Pt = R 2→1 · v1/2,I + G2→1,I · V 1/2

 c Dunod – Toute reproduction non autorisée est un délit

Si R 2→1·v1/2,I = 0, on peut négliger légitimement le terme G 2→1,I ·V1/2 . Cela revient à assimiler le contact à un contact ponctuel. Dans ce dernier cas, comme v 1/2,I est porté par le plan tangent en I et que la composante tangentielle R2→1,t de R 2→1 est opposée à v1/2,I , la puissance totale des actions de contact se réduit à : Pt = R 2→1,t · v 1/2 En raison des lois de Coulomb sur le frottement (cf. chapitre 19), cette puissance est négative ou nulle. Elle est nulle dans les deux cas suivants : s’il n’y a pas de glissement (v 1/2 = 0) et en l’absence de frottement (R2→1,t = 0). Rappelons que cette puissance totale est indépendante du référentiel par rapport auquel on la calcule, en raison de l’opposition des actions réciproques, d’où l’intérêt de la calculer dans le référentiel le plus commode. III . 2 . — Exemples a) Masselotte sur une tige en rotation Dans le cas de la masselotte glissant sur une tige en rotation (Fig. 20.5), la puissance totale des actions de contact a pour expression : Pt = R 2→1 · v1/2 = Rx x˙ 

R 2→1 étant la force qu’exerce la tige T sur la masselotte A et v 1/2 = ˙xe x , la vitesse de glissement de A sur T .

350

20. Énergétique des systèmes matériels

b) Système articulé Considérons le système articulé constitué par une tige T et un disque D articulés au centre D de D (Fig. 20.7). L’autre extrémité O de T est fixe dans le référentiel R = Oxyz. Les actions de contact interviennent en O où l’un des solides, le bâti, est fixe dans R et en D où les deux solides T et D sont articulés. (1) Si R (1) et GO désignent la somme et le moment des actions de contact { axe Oz → T }, la puissance totale des actions de contact en O s’écrit : (1) (1) Pt = R(1) · vO/R + GO · VT = GOz u˙

car vO/R = 0

et V T = u˙ ez

(2)

Désignons par R(2) et GD la somme et le moment des actions de contact {T → D} en D. Calculons la puissance totale des actions de contact dans le référentiel R = Ox y z lié à la tige T , où le calcul est le plus simple car vD = 0 : (2) (2) ˙ ˙ Pt = GD · V D/R = GD z (f − u)

puisque

˙ − u) ˙ ez VD/T = ( f

y y

O

y T θ

R

x

D

x

R D

R

C

φ

D θ x

F IG . 20.7.

F IG . 20.8.

III . 3 . — Applications aux liaisons. Liaisons parfaites a) Définition d’une liaison On appelle liaison toute relation imposée aux degrés de liberté, c’est-à-dire aux paramètres indépendants qui définissent la position d’un système, et cela indépendamment des lois de la dynamique. Les liaisons sont réalisées grâce à des forces de contact dont le torseur n’est que partiellement connu (cf. chapitre 19). Il en résulte que la réduction du nombre de degrés de liberté ne se traduit pas par une diminution du nombre d’inconnues : au contraire, ce nombre dépasse généralement le nombre d’équations, auquel cas le problème ne peut être résolu que par l’adjonction d’hypothèses supplémentaires. b) Liaisons holonomes Les liaisons les plus importantes sont les liaisons holonomes, c’est-à-dire celles qui se traduisent par une relation entre les différents degrés de liberté, à l’exclusion de leurs dérivées par rapport au temps. Exemples : i) La liaison qui impose à un point A d’évoluer suivant une trajectoire rectiligne Ox est une liaison holonome : y = 0 z = 0. ii) De même, le contact permanent d’un disque de rayon R avec un plan (Fig. 20.8) est une liaison holonome que l’on traduit par y = R, y étant la coordonnée du centre de masse C suivant la verticale ascendante Oy.

351

Énergétique des systèmes matériels

Remarques : (1) Cette dernière liaison holonome est qualifiée de bilatérale pour la distinguer de la liaison unilatérale caractérisée par l’inégalité y  R (cf. chapitre 9). (2) Une liaison, qui fait intervenir des dérivées et qui par intégration donne une relation supplémentaire entre les degrés de liberté, est qualifiée de semi-holonome. Ainsi, dans le cas du roulement sans glissement du disque précédent sur l’axe Ox, la relation entre les variables x et u est : ˙x = −Ru˙ ; en intégrant et en supposant que x = 0 si u = 0, on obtient la relation x = −Ru. c) Liaison parfaite Une liaison est parfaite si la puissance totale des actions de contact est nulle. Comme cette puissance est indépendante du référentiel considéré, le caractère parfait d’une liaison est une propriété intrinsèque. Exemples : i) Dans le cas de la masselotte glissant sur une tige en rotation uniforme (Fig. 20.5), la liaison est parfaite si : Pt = R 2→1 · v1/2 = Rx x˙  = 0 d’où R x = 0 puisque x˙  = 0 ii) Dans le cas de la sphère roulant sans glisser sur un plan incliné (Fig. 20.6), la liaison est parfaite : Pt = R 2→1 · v1/2 = 0 car v 1/2 = 0 iii) Liaison pivot Un solide S en rotation autour d’un axe Oz d’un référentiel R réalise une liaison pivot (Fig. 20.9). Notant [V, v A] le torseur des vitesses de S par rapport à R, en un point A de Oz, et [R, G A] le torseur des actions de contact au même point, de l’axe sur S, la puissance totale des actions de contact s’écrit : Pt = POz→S = R · vA + GA · V = GA · V

 c Dunod – Toute reproduction non autorisée est un délit

puisque la vitesse des points de l’axe Oz est nulle. Cette liaison est donc parfaite si GA est perpendiculaire à V. La liaison pivot est parfaite si le moment des actions de contact par rapport à l’axe de rotation est nul. Nous verrons ultérieurement comment l’on réalise techniquement une telle liaison (cf. chapitre 25). Sur la figure 20.9b, on a représenté la schématisation normalisée d’une liaison pivot. S z

A O a)

b) F IG . 20.9.

iv) Liaison pivot-glissant Une variation de la liaison pivot est la liaison pivot-glissant. Dans ce cas, S tourne et glisse à la fois, le long de l’axe (Fig. 20.10a). Comme vA = 0, la puissance totale des actions de contact est : Pt = R · vA + GA · V

Elle n’est nulle, quel que soit l’état cinématique de S, que si R · v A = 0 et GA · V = 0. La liaison pivot-glissant est parfaite si, à la fois, GA et R sont perpendiculaires à l’axe de rotation. Sur la figure 20.10b, on a représenté la schématisation normalisée d’une liaison pivot-glissant.

352

20. Énergétique des systèmes matériels S z

A O

b)

a) F IG . 20.10.

IV . — THÉORÈMES DE L’ÉNERGIE IV . 1 . — Théorème de l’énergie cinétique En appliquant le théorème de l’énergie cinétique à chacun des points A d’un système matériel continu déformable Sd , en mouvement par rapport à un référentiel R (cf. chapitre 5), et en sommant sur tous ces points, on obtient le théorème de l’énergie cinétique (cf. chapitre 12) : d Ek = Pex + P in dt Le théorème de l’énergie cinétique s’écrit aussi sous forme différentielle ou intégrale : d E k = dWex + dWin

et

DE k = Wex + Win

Rappelons que l’énergie cinétique de S d a pour expression, d’après le théorème de Kœnig (cf. chapitre 13) : 1 Ek = MvC2 + Ek∗ 2 ∗ où Ek est l’énergie cinétique de S d dans le référentiel du centre de masse. Remarques : (1) Soulignons que, contrairement aux théorèmes de la quantité de mouvement et du moment cinétique, le théorème de l’énergie cinétique fait intervenir les forces intérieures. Cependant, comme la puissance des forces intérieures est celle d’un torseur nul, elle est indépendante du référentiel considéré. (2) Si le référentiel n’est pas galiléen, il faut ajouter la contribution des forces d’inertie d’entraînement, celle de la force de Coriolis étant nulle. IV . 2 . — Théorème de l’énergie mécanique Si on distingue les forces qui dérivent d’une énergie potentielle des autres, la puissance de ces forces extérieures ou intérieures, dites conservatives, s’écrit, respectivement : −d Ep,ex / d t et −d Ep,in / d t. nc Notant P nc ex et P in les puissances des autres forces non conservatives extérieures et intérieures, le théorème de l’énergie cinétique devient :

d (Ek + E p,ex + Ep,in ) = Pexnc + P nc in dt On obtient alors le théorème de l’énergie mécanique : d Em nc = Pex + P innc avec Em = Ek + E p,ex + Ep,in dt

353

Énergétique des systèmes matériels On retient souvent ce théorème sous ses formes différentielle et intégrale : d Em = dWexnc + dW nc et DE m = Wexnc + Winnc in IV . 3 . — Cas d’un seul solide

Dans le cas particulier où le système se réduit à un seul solide S, la puissance des forces intérieures est nulle, Pin = 0, et l’énergie cinétique dans le référentiel du centre du masse R∗ s’exprime simplement en fonction du vecteur vitesse angulaire V de S par rapport au référentiel d’analyse R. a) Expression de l’énergie cinétique d’un solide Deux cas se présentent suivant que le solide S a un point fixe ou non dans le référentiel R. i) S a un point fixe O dans R. Il vient : Ek =

1 2

rv2 d

=

1 2

rv · (V × OA) d

=

1 V· 2

OA × rv d

d’où : Ek =

1 LO · V 2

Si le solide tourne autour d’un axe, avec la vitesse angulaire V = V eu , l’énergie cinétique prend une forme plus simple. En effet : 1 1 E k = L O · V = V 2(eu · [I]O eu ) 2 2

d’où

Ek =

1 IOu V 2 2

I Ou = eu · [I]O eu étant le moment d’inertie de S par rapport à l’axe Ou (cf. chapitre 17).

ii) S n’a pas de point fixe dans R. Il est naturel d’introduire le référentiel du centre de masse R ∗. Dans ce référentiel, l’énergie cinétique s’écrit :

 c Dunod – Toute reproduction non autorisée est un délit

E k∗ =

1 2

v ∗2r d

=

1 2

v∗ · (V × CA) r d

=

1 V· 2

CA × v∗ r d

1 = L ∗C · V 2

puisque v∗ = V × CA. Comme L ∗C = LC , il en résulte, d’après le théorème de Kœnig : Ek =

1 1 1 MvC2 + LC · V = (P · v C + LC · V) 2 2 2

Si S tourne autour d’un axe, avec la vitesse angulaire V = V eu , on a comme précédemment : Ek =

1 2 1 MvC + ICu V2 2 2

où I Cu = e u · [I]C eu est le moment d’inertie de S par rapport à l’axe Cu (cf. chapitre 17). Remarque : La quantité P · vC + L C · V apparaît comme le produit scalaire du torseur des quantités de mouvement [P, L C] par le torseur cinématique du solide [V, v C].

354

20. Énergétique des systèmes matériels

b) Théorème de l’énergie mécanique pour un seul solide La puissance des forces intérieures à un seul solide S étant nulle, l’application du théorème de l’énergie mécanique à S donne : d Em = P nc ex dt

avec

Em = Ek + E p,ex

soit aussi, sous forme différentielle ou intégrale : d Em = dWnc ex

et DEm = Wexnc

Remarque : Si le système déformable Sd est constitué de plusieurs solides, la puissance des forces intérieures au système se réduit à celle des actions de contact entre les solides qui le composent. On obtient alors, pour le système : d Ek = P ex + dt

i=j

P in(Si S j )

d’où l’intérêt de rappeler le résultat suivant : la puissance totale des actions intérieures de contact entre plusieurs solides est nulle lorsqu’il n’y a pas de frottement ou de glissement. c) Exemple Considérons un cerceau (masse m, rayon R) roulant et glissant sur un axe incliné d’un angle a par rapport à un plan horizontal (Fig. 20.11). Le théorème de l’énergie mécanique donne, avec les notations habituelles : d Em = R · v I1 dt

avec

Em = Ek + E p =

1 2 1 ˙2 mv + I C u − mg · OC + Cte 2 C 2

et

vI 1 = v C + IC × u˙ ez

En explicitant, on trouve : Em=

1 2 1 m ˙xC + mR2 ˙u2 − (−mgx C sin a − mgR) + Cte 2 2

et

˙ R · v I1 = R t (˙x C + Ru)

La résolution d’un tel problème, qui possède deux degrés de liberté (xC , u) et des inconnues supplémentaires telles que Rt , exige que l’on applique aussi les théorèmes du centre de masse et du moment cinétique. Il convient alors de connaître les conditions initiales et d’étudier la variation de la vitesse de glissement v g du cerceau sur l’axe (cf. Exercices).

y

C

g x O

® F IG . 20.11.

µ I

355

Énergétique des systèmes matériels

V . — CONSERVATION DE L’ÉNERGIE MÉCANIQUE V . 1 . — Caractère non conservatif de l’énergie mécanique Nous avons vu que l’énergie mécanique d’un système matériel n’était pas une grandeur conservative : dans le bilan de cette grandeur, le travail des forces extérieures traduisait l’échange avec le milieu extérieur (cf. chapitre 13) et le travail des forces intérieures est un terme de production d’énergie mécanique. Entre deux instants voisins t et t + d t, on a : d Em = dEm(r) + dEm(c)

avec

dE (mr) = dW (exnc)

et dE (mc) = dW in(nc)

ce qui donne, en intégrant : (nc) DEm = Em(r) + Em(c) = Wex(nc) + W (in)

C’est précisément la recherche d’une grandeur énergétique conservative qui a conduit à postuler le premier principe de la thermodynamique (cf. Thermodynamique). Bien que non conservative, l’énergie mécanique E m peut se conserver occasionnellement. C’est le cas si la puissance P(nc) des forces qui ne dérivent pas d’une énergie potentielle est nulle : P (nc) = 0 entraîne

d Em =0 dt

d’où

Em = Cte

Comme l’équation différentielle obtenue par application de la conservation de Em est du premier ordre et non du deuxième, on l’appelle aussi l’intégrale première de l’énergie mécanique. Remarque : La conservation occasionnelle de l’énergie mécanique est à l’origine du qualificatif conservatives pour les forces qui dérivent d’une énergie potentielle.

 c Dunod – Toute reproduction non autorisée est un délit

V . 2 . — Cas d’un système isolé ou pseudo-isolé Considérons le cas où les forces extérieures n’ont aucune contribution à la fois dans l’énergie po(nc) tentielle extérieure Ep,ex et dans la puissance des forces extérieures non conservatives P ex , soit parce que le système est isolé (les forces extérieures sont inexistantes), soit parce que le système est pseudoisolé (la contribution globale des forces extérieures est nulle). Le théorème de l’énergie mécanique se réduit alors à : d Em = Pin(nc) avec Em = Ek + E p,in dt Ainsi, l’énergie mécanique d’un système isolé ou pseudo-isolé ne se conserve pas à moins que la puissance des forces intérieures qui ne dérivent pas d’une énergie potentielle soit nulle. Il est instructif d’appliquer ce résultat aux cas déjà considérés d’une personne sur un tabouret d’inertie et d’un œuf cru (cf. chapitre 18). Ces deux systèmes sont déformables et pseudo-isolés, car la puissance des actions extérieures de contact est pratiquement nulle. L’augmentation de la vitesse de rotation du tabouret, lorsque la personne ramène ses bras sur l’axe de rotation, est due à la puissance positive des forces intérieures ; en effet, on a, l’énergie potentielle de pesanteur ne variant ainsi que la projection sur l’axe du moment cinétique Lz = I i Vi = I f Vf (cf. chapitre 18) : DEm = DEk = DEk = W (innc)

avec

DEk =

If V2f Ii V 2i = I i Vi (Vf − Vi ) > 0 − 2 2

Quant à la diminution de la vitesse de rotation de l’œuf cru, elle est due à la puissance négative des forces intérieures de viscosité (cf. chapitre 31).

356

20. Énergétique des systèmes matériels

V . 3 . — Exemples a) Système articulé tige-roue Considérons le système articulé tige-roue représenté sur la figure 20.12. Si la liaison pivot en O 1 est parfaite et si la roue roule sans glisser sur le plan horizontal Oxy, la puissance des forces extérieures de contact est nulle. Le système est donc pseudo-isolé. Son énergie mécanique se conserve à condition que la puissance totale des actions de contact au niveau de l’articulation en C soit nulle, c’est-à-dire que la liaison en C soit parfaite. Notons que, dans ce mouvement horizontal, l’énergie potentielle de pesanteur reste constante. z O1 Tige

g

C Roue y

O x

F IG . 20.12.

b) Exemple simple d’une bifurcation Le régulateur à boules présente un intérêt à la fois pratique, pédagogique et scientifique. On connaît son rôle majeur dans la maîtrise du fonctionnement des machines à vapeur au XIX e siècle. On peut le schématiser par un système constitué de tiges articulées, de masse négligeable, de deux masselottes identiques A et B, de même masse m et d’une troisième masselotte D de masse 2m (Fig. 20.13). L’ensemble déformable, caractérisé par l’angle u variable que font les tiges avec l’axe de rotation, peut tourner symétriquement et de façon parfaite autour d’un axe vertical ascendant Oz. Un moteur, exerçant un moment d’action G m ez, lui impose une vitesse angulaire uniforme V. O

z θ

B (m)

l l A (m)

D(2 m)

Poulie

g

Manivelle

F IG . 20.13.

Dans le référentiel tournant non galiléen, la puissance des forces non conservatives qu’exerce le moteur est nulle puisque P (nc) = Gm · V avec V  = 0. On a donc, dans ce référentiel non galiléen, conservation de l’énergie mécanique : Em = Cte. Dans R  , l’énergie cinétique du système a pour expression : 1 1 1 1 ˙ 2 + 1 (2m)(2lu˙ sin u) 2 = 4ml2 u˙ 2 (1 + sin2 u) E k/R = mv2A + mv2B + (2m)v2D = 2 × m(2lu) 2 2 2 2 2 puisque OA = OB = 2l et OD = 2l cos u. L’énergie potentielle a deux contributions, l’énergie potentielle de pesanteur E pg et l’énergie potentielle d’inertie centrifuge Epc : R ,

Epg = mgzA + mgz B + 2mzD + Cte = −8mgl cos u + Cte = 8mgl(1 − cos u)

357

Énergétique des systèmes matériels car z A = z B = zD = −2l cos u , et E pc = −IOz

V2 V2 + Cte = − 2 × m(2l sin u) 2 + Cte = −4mV 2l2 sin2 u 2 2

si l’origine de l’énergie potentielle totale est prise pour u = 0 et si le moment d’inertie de la masse en D est nul. Ainsi : V2 Ep = 8mgl 1 − cos u − sin 2 u 2g/l Il en résulte que l’énergie mécanique totale E m dans R  a pour expression, en posant v 0 = (g/l)1/2 et u = V/v0 : Em = Ek + E p = 4ml2u˙ 2 (1 + sin2 u) + 8mgl 1 − cos u −

u2 sin 2 u 2

= Cte

Étudions, à l’aide de Ep (u), les différentes positions d’équilibre stable de ce système dans le référentiel tournant R  ; la fonction E p(u) que l’on obtient en ajoutant les deux contributions E pg et E pc est paire (Fig. 20.14a). Les positions d’équilibre sont définies par l’équation : d E p = 8mgl sin u 1 − u 2 cos u = 0 du d’où les solutions u 1 = 0, u 2 = p et u3 tel que cos u 3 = 1/u 2. Ces positions d’équilibre sont stables si la dérivée seconde est positive : K=

d2 E p d u2

e

= 8mgl cos u(1 − u2 cos u) + u2 sin 2 u

e

On a donc : (K ) 0 = 8mgl(1 − u 2)

(K)p = −8mgl(1 + u 2)

et (K ) u3 = 8mglu2 (1 − 1/u4 )

Il existe ainsi une position d’équilibre instable ( u 2 = p ) et deux positions d’équilibre stable :

 c Dunod – Toute reproduction non autorisée est un délit

i) l’une u 1 = 0 pour V < v0, ii) l’autre u 3 = ±arcos(1/u 2) pour V > v0. Le graphe donnant E p en fonction de u, pour V > v0 conforte cette l’analyse. On voit que, si V  v0 , u3 ≈ p/2. E p Énergies

¼ 2

2ml  Em

Epg

Ep

0 ¼

µ1

0

 Epc

a)

µe

B !0

Ω

µ −

¼ 2 b)

F IG . 20.14.

358

20. Énergétique des systèmes matériels

Si l’on représente graphiquement la position d’équilibre stable en fonction de V, on obtient le graphe de la figure 20.14b : tant que V < v 0, la position d’équilibre est ue = 0 ; dès que V > v0 , la position d’équilibre prend une valeur non nulle qui varie avec V jusqu’à la valeur p/2 lorsque V devient très grand par rapport à v0 . Au point B de la courbe, défini par V = v0, se produit un changement qualitatif de la position d’équilibre sous l’action du paramètre extérieur V ; on dit que le système subit en ce point une bifurcation. Ordre de grandeur : Si l = 5 cm, v0 = (9, 81/0, 05) 1/2 ≈ 14 rad . s −1 soit 2, 2 tours par seconde. Une telle vitesse critique peut être aisément mise en évidence. Comme on l’a déjà vu pour le mouvement d’un point matériel, la conservation de l’énergie mécanique permet, dans le cas d’un mouvement unidimensionnel, de procéder à une étude qualitative qui est précieuse si l’équation différentielle n’est pas simple à résoudre. L’énergie cinétique étant non négative, les seuls mouvements possibles sont ceux pour lesquels l’énergie mécanique est supérieure à l’énergie potentielle. Remarque : On pourrait être tenté d’établir ce résultat à partir de l’énergie dans le référentiel terrestre R. Cependant, dans ce référentiel, l’énergie mécanique n’est pas une constante puisque sa variation est égale au travail du moment Gm exercé par le moteur, lequel n’est pas nul. Pour parvenir à l’équation du mouvement, il est nécessaire d’exprimer au préalable ce moment à l’aide du théorème du moment cinétique en projection suivant l’axe de rotation.

CONCLUSION Retenons les points essentiels de l’énergétique des systèmes. (1) Le théorème de l’énergie cinétique, contrairement aux théorèmes du centre de masse et du moment cinétique, contient aussi la contribution des forces intérieures. (2) Ce théorème fait intervenir des grandeurs mécaniques scalaires ; il est donc commode d’emploi, mais en contrepartie il ne fournit qu’une seule équation, ce qui le destine préférentiellement aux mouvements unidimensionnels. (3) La manière la plus efficace de l’utiliser consiste à séparer les forces qui dérivent d’une énergie potentielle des autres et à l’écrire sous la forme : d Em = P ex(nc) + P in(nc) dt (nc)

soit aussi

DEm = Wex(nc) + W in(nc)

(nc)

où Em est l’énergie mécanique, Pex et P in les puissances des forces extérieures et intérieures qui (nc) (nc) ne dépendent pas d’une énergie potentielle ; Wex et Win sont les travaux correspondants. (4) Dans le cas important d’un seul solide, W in = 0 et l’énergie cinétique s’exprime simplement en fonction du vecteur vitesse de rotation. (nc)

(nc)

(5) Si W (nc) = Wex + Win = 0, ce qui est fréquent, ce théorème fournit alors une intégrale première qui traduit la conservation de l’énergie mécanique : E m = E k +Ep = Cte. L’intérêt que présente a priori cette intégrale première de l’énergie, en tant qu’équation différentielle du premier ordre, est limité par son caractère quadratique difficilement exploitable. En revanche, son intérêt physique est de tout premier plan. Écrit sous la forme d’une grandeur scalaire qui se conserve, le théorème de l’énergie mécanique permet non seulement une résolution rapide de problèmes à une dimension, mais aussi et surtout une introduction de l’un des concepts les plus fondamentaux de la physique, l’énergie.

359

Énergétique des systèmes matériels

EXERCICES ET PROBLÈMES P20– 1. Oscillations d’un volant roulant sans glisser sur un guide circulaire Un volant repose par deux tourillons sur un guide circulaire (Fig. 20.15). Ce volant, parfaitement équilibré, a son centre de masse situé sur son axe de révolution. On utilise les angles u et f pour caractériser la position du volant dans le plan vertical Oxy du référentiel terrestre R = Oxyz. On désigne par I le moment d’inertie du volant par rapport à son axe de révolution, R le rayon du guide et r le rayon des tourillons. 1. Établir la condition de roulement sans glissement du volant sur son guide. 2. Trouver l’équation différentielle du mouvement en u. 3. En déduire la période des petites oscillations. 4. Retrouver l’équation différentielle, établie en 2 . , à l’aide du théorème du moment cinétique appliquée au point de contact. O g

R

y

θ C

r

O

Δ

g

l µ

f x

V C

x F IG . 20.15.

F IG . 20.16.

P20– 2. Pendule composé formé d’une tige et d’un disque articulés Un système articulé est constitué d’une tige homogène, de masse m, de longueur l, et d’un disque homogène, de masse M et de rayon r (Fig. 20.16). Ce système, suspendu à un axe horizontal D, est abandonné avec un angle u 0 et une vitesse de rotation du disque V 0. Les liaisons pivot en O et au centre C du disque sont parfaites.

 c Dunod – Toute reproduction non autorisée est un délit

1. Trouver deux constantes du mouvement. 2. Quelle est la période des petites oscillations du pendule pesant ainsi formé ? Comparer cette période à celle du pendule constitué par la tige et le disque rigidement liés. P20– 3. Conservation de l’énergie d’un cerceau lesté Retrouver, par la conservation de l’énergie, l’équation différentielle du mouvement du cerceau lesté représenté sur la figure 18.12. Discuter la nature des différents mouvements. P20– 4. Système articulé de deux barres Deux barres homogènes, identiques (masse m, longueur l) sont articulées l’une sur l’autre sans frottement et reposent sur un plan horizontal parfaitement lisse. On les maintient dans un plan vertical dans la position indiquée sur la figure 20.17 et on les abandonne. Les extrémités B et C glissent sans frottement et les barres tombent en restant dans le plan vertical initial. 1. Quelle est l’expression de l’énergie cinétique du système en fonction de l’angle u = (Ox, OC 1) ?

360

20. Énergétique des systèmes matériels 2. Établir l’équation différentielle en u.

3. Calculer, en fonction de la hauteur h, la vitesse de l’articulation A au moment où elle arrive en contact avec le plan horizontal. y

z A

g

C1

h B

O

θ

g C

Ω ∆ O

y

θ C x

F IG . 20.17.

F IG . 20.18.

P20– 5. Mouvement d’une plaque homogène de forme carrée Le référentiel terrestre R est considéré comme galiléen. Soit R  = Ox y z un trièdre orthonormé, O étant un point fixe de R et Oz la verticale ascendante ; le champ de pesanteur uniforme est noté : g = −g ez . Une plaque homogène plane ayant la forme d’un carré C, de côté 2a, de masse m, de centre de masse C, est mobile autour d’un axe matériel D confondu avec Ox de telle sorte que C soit astreint à se déplacer dans le plan Oy z, un de ses sommets restant fixé en O. La position de C à tout instant est repérée par l’angle u = (g, OC) (Fig. 20.18). L’axe D est mis en rotation autour de la verticale par un moteur qui lui impose un vecteur rotation constant V = V ez. On se propose d’étudier le mouvement de C par rapport au référentiel R  . La liaison est parfaite en O. ˙ l’énergie cinétique Ek de C dans R . 1. Exprimer, en fonction de u, 2. Donner, en fonction de u, l’expression de l’énergie potentielle de pesanteur E pg de C.  3. Quelle est l’énergie potentielle centrifuge E pc en fonction de V et u ?

4. Calculer l’angle u e qui correspond à une position d’équilibre stable de C par rapport à R  . Montrer que u e n’est différent de 0 que si V est supérieur à une valeur v c que l’on déterminera en fonction de g et a. Quelle valeur faut-il donner à V pour avoir ue = 60◦ ? P20– 6. Pendule elliptique Le pendule elliptique est un pendule pesant, dont l’axe de l’articulation O 1z est lié à un petit véhicule de masse négligeable (Fig. 20.19). Ce dernier peut se déplacer sans frottement suivant l’axe horizontal Ox d’un référentiel terrestre R = Oxyz dans lequel Oy est la verticale ascendante. Toutes les liaisons entre solides, par l’intermédiaire du véhicule et de roulements à billes au niveau de l’articulation, sont parfaites. Initialement, le pendule est écarté de la verticale et abandonné sans vitesse initiale. 1. Montrer que le centre de masse C du système, situé à une distance l de O 1, évolue le long d’une droite fixe. 2. Établir l’équation différentielle du mouvement en u = (−Oy 1 , O1 C) en appliquant le théorème de l’énergie dans R. On désigne par I le moment d’inertie du pendule par rapport à l’axe O 1z. 3. Trouver la période des petits mouvements d’oscillation.

4. Quelle est la trajectoire d’un point de l’axe du pendule ?

361

Énergétique des systèmes matériels y

y1 g

Oz

x

O 1z

z . θ

g

x1

Roue . φ

Disque θ O

l

C

I A

F IG . 20.19.

F IG . 20.20.

P20– 7. Mise en rotation d’un disque par la chute d’une masse Un disque est mis en rotation par la chute verticale d’une masselotte A (masse m), grâce à un système intermédiaire constitué d’une roue (rayon b) solidaire d’une tige (section circulaire de rayon r) matérialisant son axe (Fig. 20.20). En tombant, la masse tire sur un fil initialement enroulé sur la tige ; la rotation de la tige entraîne celle de la roue qui roule sans glisser sur le disque en un point situé à une distance l de l’axe de rotation Oz. On désigne par I1 et I2 les moments d’inertie du disque et du dispositif intermédiaire par rapport à leurs axes respectifs. 1. Exprimer les conditions de roulement sans glissement de la roue sur le disque et du fil sur la tige. En déduire que le système n’a qu’un degré de liberté. 2. Sachant que toutes les liaisons sont parfaites, établir l’équation différentielle à laquelle satisfait l’angle u qui caractérise la rotation du disque. En déduire la nature du mouvement. P20– 8. Étude de la chute d’une barre par l’énergie Retrouver, à l’aide de l’énergie, l’équation différentielle du mouvement de la barre représentée sur la figure 18.15.

 c Dunod – Toute reproduction non autorisée est un délit

P20– 9. Cylindre sur un véhicule accéléré Un cylindre plein, homogène (masse m, rayon r) est placé sur la plateforme d’un véhicule (Fig. 20.21). Le cylindre étant immobile,le véhicule part du repos et prend une accélération a 0 constante selon l’axe horizontal Ox. Il roule sans glisser sur la longueur l de la plateforme avant d’atteindre un obstacle. En appliquant le théorème de l’énergie dans le référentiel du véhicule, trouver la distance L parcourue par ce dernier, lorsque le cylindre a atteint l’obstacle. z g

Moteur z



θ O

x

z1

B

y C x

F IG . 20.21.

F IG . 20.22.

362

20. Énergétique des systèmes matériels

P20– 10. Mouvement d’un secteur circulaire dans un référentiel tournant Un secteur circulaire plein homogène, de masse M, de rayon R et de demi-angle a, est relié verticalement à une tige horizontale OB à l’aide d’une articulation pivot au centre B du disque générateur (Fig. 20.22). La tige, de longueur b et de masse négligeable, tourne uniformément, avec une vitesse angulaire V autour de l’axe vertical Oz d’un référentiel terrestre R = Oxyz. Les liaisons en O et B sont parfaites. On désigne par c la distance de B au centre de masse C. 1. Trouver la position du centre de masse du secteur circulaire. 2. Calculer les moments principaux d’inertie I 3 , I1 et I2 respectivement par rapport aux axes Bz 1 , axe de symétrie, Bx1, perpendiculaire au plan du secteur et By 1 qui forme, avec les deux premiers axes, un trièdre orthonormé direct Bx1y1 z1. En déduire l’expression de l’énergie cinétique du système, dans le référentiel tournant R  = Ox y  z lié à la tige, en fonction de l’angle u que l’axe de symétrie du secteur fait avec la verticale ascendante. 3. Quelle est, en fonction de u, l’énergie potentielle de pesanteur ? Montrer qu’il est nécessaire d’introduire une autre énergie potentielle d’expression : V2 Ml 2 + I3 + (I 2 − I3 ) sin 2 u 2 4. En déduire l’équation différentielle du mouvement. Étudier les petits mouvements dans le cas d’un disque, d’un demi-disque et d’un quart de disque. Ep,c = −

P20– 11. Bille sphérique creuse dans un guide diédrique en rotation Une bille sphérique creuse (masse m, rayon r) roule sans glisser sur un rail circulaire à section droite en forme de dièdre droit (Fig. 20.23a). La trajectoire de son centre de masse par rapport au rail est un cercle de rayon R. Le rail tourne uniformément autour d’un axe vertical avec la vitesse angulaire Vr par rapport au référentiel terrestre R = Oxyz supposé galiléen, Oz étant la verticale ascendante (Fig. 20.23b). 1. Montrer que, dans le référentiel R  = Oxyz lié au rail, la vitesse√v C du centre de masse de la bille est reliée à sa vitesse de rotation V par l’équation : vC = rV 2/2. En déduire l’énergie cinétique de la bille dans R . 2. Établir les expressions de l’énergie potentielle de pesanteur et de l’énergie potentielle centrifuge dans le référentiel tournant R . En déduire l’énergie mécanique E m dans R et l’équation du mouvement en u. 3. Discuter les positions d’équilibre possibles de la bille en fonction de V. 4. Établir les expressions de l’énergie cinétique de la bille par rapport à R, de l’énergie potentielle de pesanteur. En déduire l’énergie mécanique Em. Cette énergie mécanique ne se conserve pas. Pourquoi ? z

V

Bille C R

ω J Rail

I

q

a)

b) F IG . 20.23.

g

363

Énergétique des systèmes matériels P20– 12. Machine d’Atwood dansante (MAD)

Le système mécanique représenté sur la figure 20.24a est une variante complexe de la machine d’Atwood historique, conçue par le physicien anglais G. Atwood en 1784 (Fig. 20.24b). Les masses de A et B , qu’on assimile à des corps ponctuels, sont respectivement m et M . Les poulies identiques ont pour rayon R et pour moment d’inertie par rapport à leur axe de révolution I p . La liaison verrou entre les poulies et leurs axes respectifs de rotation est parfaite. En outre, la distance D qui sépare les axes des poulies est constante. Le fil reliant les deux masses est inextensible et il ne glisse pas dans la gorge des poulies. On désigne par z la coordonnée fixant la position de B , selon la verticale descendante Oz , à partir du plan horizontal passant par O , et par u l’angle que fait la direction du pendule IA avec la verticale descendante (Fig. 20.24a). L’angle de rotation de la poulie est w . I w

Ox

y

y

O

D Poulie 1

r

g

g

u zA A(M) z

B(M) a)

A(M) z

B(M) b)

F IG . 20.24.

1. Machine d’Atwood historique à deux poulies Les mouvements des masselottes A et B sont rectilignes, selon la verticale (Fig. 20.24b).

 c Dunod – Toute reproduction non autorisée est un délit

a) Établir, en fonction de z et w , la condition de roulement sans glissement de la poulie sur le fil. Exprimer, en fonction de z et de la coordonnée zA de A , l’inextensibilité du fil. En déduire le nombre de degrés de liberté du système. b) Établir, en fonction de z ou ˙z , les expressions de l’énergie potentielle et de l’énergie cinétique du système. c) Que signifie l’expression « La liaison entre les poulies et leurs axes respectifs de rotation est parfaite » ? Que peut-on dire de la puissance des forces de frottement de contact qu’exercent les poulies sur le fil ? En déduire, à partir du théorème de l’énergie, l’équation du mouvement et la nature de ce dernier. d) Quel est l’intérêt pédagogique de cette machine ? e) Calculer l’accélération du mouvement pour m = 120 g , M = 360 g , I p = 7 × 10−6 kg.m2 et R = 2, 5 cm . 2. Machine d’Atwood Dansante MAD ou SAM (Swinging Atwood Machine) Comme précédemment, la masselotte B a un mouvement rectiligne vertical, mais A peut aussi osciller (Fig. 20.24a). a) Justifier l’existence de deux poulies et non d’une seule comme dans l’expérience historique d’Atwood. b) Donner, en fonction de z et w , la condition de roulement sans glissement de la poulie sur le fil. Exprimer l’intextensibilité du fil en fonction de z , u et r = IA . En déduire le nombre de degrés de

364

20. Énergétique des systèmes matériels

liberté du système. Pourquoi l’application du théorème de l’énergie mécanique ne suffit-elle pas pour analyser le mouvement ? c) Établir, en fonction de r , u , r˙ et u˙ , les expressions de l’énergie potentielle et de l’énergie cinétique du système. d) En déduire l’équation suivante, issue de l’application du théorème de l’énergie : 1 ˙ 2 + 1 mr2 u˙2 + mg(R sin u − r cos u) + Mg(r − Ru) = Cte M t (˙r − Ru) 2 2 dans laquelle Mt est un coefficient que l’on exprimera en fonction de M , m , I p et R . e) Quel est l’intérêt d’appliquer le théorème du moment cinétique à la masselotte A , au point mobile I de contact du fil avec la poulie 1 ? Déduire de ce théorème la seconde équation du mouvement : u + 2 r˙ ˙u − Rr˙ u˙ = −g sin u r¨ f) Retrouver l’équation du mouvement de la machine d’Atwood historique.

P20– 13. Pendule pesant inversé sur un chariot en mouvement Un pendule O A est constitué d’une tige de masse négligeable et de longueur l . Son extrémité A porte une masselotte de masse m et son extrémité O est articulée sur un véhicule (masse M ) en mouvement rectiligne horizontal sans frottement, selon l’axe Ox du référentiel du laboratoire R . Ce dernier est soumis à la force de propulsion horizontale Fp = F p ex . On désigne par Oz la verticale ascendante et par u l’angle que fait la direction du pendule avec la verticale (Fig. 20.25). En outre, R est la force qu’exerce le véhicule sur le pendule, et R la force verticale qu’exerce le rail sur le véhicule. La liaison étant supposée parfaite, le moment des actions de contact en O  est normal à l’axe de rotation O y . 1. Appliquer le théorème du centre de masse C au véhicule en mouvement dans le plan vertical Ozx . En déduire deux équations auxquelles satisfont la coordonnée x de C et les composantes des réactions R et R , respectivement X , Z et X  , Z  . 2. Établir les deux équations différentielles issues du théorème du centre de masse appliqué au pendule. 3. Trouver l’équation différentielle issue de l’application du théorème du moment cinétique appliqué au point mobile O de R . Que devient-elle dans le cas des petis mouvements ? 4. En déduire ¨ x et ¨ u en fonction de x u et F p .

5. On pose q1 = x , p 1 = M x˙ , q2 = lu , p2 = Mlu˙ . Exprimer la matrice de transformation donnant la matrice colonne dérivée par rapport au temps de la matrice colonne formée par q 1 , p1 , q2 , p 2. Calculer cette matrice pour l = 0, 4 m , m = 0, 1 kg et M = 0, 3 kg . z u

g

A

O0y Fp

C O

x F IG . 20.25.

21 Mécanique des chocs L’étude des chocs entre solides, ou plus largement entre systèmes quelconques, relève des théorèmes généraux de la mécanique, tout comme celle des collisions de particules que nous avons vue au chapitre 14. Cependant, en raison des caractéristiques des chocs, les théorèmes généraux prennent une forme simplifiée, techniquement très commode d’emploi et fondamentalement liée aux bilans de quantité de mouvement, de moment cinétique et d’énergie cinétique.

I . — DYNAMIQUE DES CHOCS

 c Dunod – Toute reproduction non autorisée est un délit

I . 1 . — Définition des chocs On appelle choc entre deux solides le contact de leurs surfaces, à un instant t, alors que ces surfaces n’étaient pas en contact dans les instants voisins antérieur et parfois postérieur à cette date. L’expérience courante montre que la durée d’un choc est toujours très faible devant les durées caractéristiques qui interviennent dans l’analyse. Il en résulte que le choc est localisé dans l’espace : par exemple, si on lance une boule de billard sur une seconde boule immobile, le choc ne commence que lorsque les deux boules entrent en contact et cesse dès la fin du contact ; en dehors de cette période, les mouvements des boules sont des mouvements classiques de solides soumis à des forces bien répertoriées : poids, réaction du tapis, résistance de l’air. En outre, les variations de vitesse sont fortes car les solides exercent entre eux des forces de réaction importantes lorsqu’on tente de les comprimer : aux vitesses de pénétration avant le choc succèdent des vitesses d’expulsion après le choc. I . 2 . — Bilans de quantité de mouvement et de moment cinétique Rappelons les équations-bilans de la quantité de mouvement et du moment cinétique d’un système matériel (cf. chapitres 13 et 18) : DP = P(r) =

Fex d t

et DL O = L(Or) =

MO,ex d t

Ces bilans de grandeurs conservatives, appliqués aux chocs, caractérisés par une très faible durée, font apparaître naturellement les termes d’échange particuliers appelés respectivement percussion et moment de percussion : P

(r)

t2

= t1

F ex d t

et

(r) LO

t2

= t1

MO,ex d t

366

21. Mécanique des chocs

Notons que, si la force et le moment sont des grandeurs finies, c’est-à-dire si les normes de F et M O sont limitées par des valeurs majorantes, les termes d’échange correspondants sont négligeables. En effet, remplaçant par exemple la norme de Fex par une valeur majorante Fm , on obtient : t2



t1

F ex d t < Fm

t2 t1

d t = Fm t ≈ 0

puisque t est très faible. C’est le cas de la plupart des forces, telles que le poids, à l’exclusion des forces de réaction entre solides. Retenons donc l’expression des théorèmes relatifs aux chocs : t2

DP = P(r) = t1

Fex d t

et DL O = L(Or) =

t2 t1

M O,ex d t

Dans le cas où il n’y a pas d’échange avec le milieu extérieur (système isolé ou pseudo-isolé), la quantité de mouvement et le moment cinétique se conservent : P = Cte

et

LO = Cte

soit, en indiçant par i les grandeurs initiales et par f les grandeurs finales : P i = Pf

et

LO,i = LO,f

Notons que la condition d’isolement du système est ici peu restrictive, puisque seules les percussions associées aux forces de réaction ne sont pas négligeables. De même, la condition d’application du théorème du moment cinétique en un point fixe n’est plus impérative car les déplacements sont négligeables au cours d’un choc : le point où l’on calcule le moment cinétique pourra donc être quelconque. I . 3 . — Exemples a) Désintégration d’un système isolé Considérons un système constitué de deux cylindres aplatis, de masses respectives m 1 et m2 , reliés par un ressort ; ce dernier est comprimé grâce à un fil (Fig. 21.1). Cet ensemble évolue sans frottement sur une table à coussin d’air horizontale. La réaction de contact étant à chaque instant opposée au poids, le système est pseudo-isolé. Si l’on brûle le fil, le mouvement ultérieur de l’ensemble satisfait à l’équation de conservation suivante puisque l’échange est nul : p = p 1 + p2

soit

(m 1 + m2)v = m 1v1 + m2v2

b) Pendule balistique Une tige homogène, de longueur l et de masse M, peut tourner parfaitement autour d’un axe fixe horizontal qui passe par l’une de ses extrémités O (Fig. 21.2). Un corps ponctuel A (masse m), de vitesse v, vient s’y fixer à une distance b de O, alors que la tige était au repos en position verticale. Écrivons pour l’ensemble le bilan de moment cinétique en O, en projection suivant l’axe de rotation. Il vient, le moment des actions de contact étant perpendiculaire à l’axe de rotation : DLOz = 0 soit 0 + mv b = (I + mb2 )Vf où I = Ml 2/3 est le moment d’inertie de la tige par rapport à l’axe Oz et V f la vitesse de rotation de la tige dans l’état final, après le choc. On en déduit la vitesse v en déterminant V f , laquelle est la vitesse angulaire initiale dans le mouvement pendulaire ultérieur.

367

Mécanique des chocs O g

p1

p

θm

b C

Fil

v

p2 F IG . 21.1.

A

F IG . 21.2.

En effet, le mouvement ultérieur du système satisfait à la conservation de l’énergie mécanique. Si u m est l’angle maximal dont s’écarte la tige de la verticale, on a : Em = E k + Ep = Cte

d’où

l 1 (I + mb2 )V2f + −Mg − mgb 2 2

l = 0 + −Mg − mgb cos um 2

Il en résulte que : v=

I + mb 2 Vf mb

avec

Vf =

2g(Ml/2 + mb)(1 − cos u m) I + mb 2

1/2

Notons que la quantité de mouvement de l’ensemble ne se conserve pas, puisque l’échange n’est pas nul. L’application du théorème de la quantité de mouvement permet précisément d’obtenir cet échange : l l P (r) = M + mb Vf ex − (mv + 0) = M + mb Vf − mv ex 2 2

II . — ÉNERGÉTIQUE DES CHOCS

 c Dunod – Toute reproduction non autorisée est un délit

II . 1 . — Théorème de l’énergie cinétique Appliquons le théorème de l’énergie cinétique au système qui subit un choc, entre les instants voisins t1 et t 2 = t 1 + t. Il vient : DEk = W ex + W in Dans la liste de ces travaux, le travail des forces finies est négligeable puisque les déplacements le sont au cours d’un choc. Notons que W ex est nul si le système n’est soumis qu’à des forces extérieures finies. En ce qui concerne le travail des forces de contact, il est généralement négatif car, même si les forces de frottement sont nulles, le système subit une déformation locale et donc consomme une partie de l’énergie cinétique. Ainsi, la diminution d’énergie cinétique d’un système, au cours d’un choc, n’est pas uniquement liée aux forces de frottement ; une partie sert à le déformer. Dans le cas où le système est isolé ou pseudo-isolé, on a : DEk = W in  0

368

21. Mécanique des chocs

II . 2 . — Choc élastique et choc inélastique a) Définitions On dit qu’un choc entre deux systèmes, formant un ensemble isolé ou pseudo-isolé, est élastique si l’énergie cinétique totale du système est conservée : Ek,i = E k,f

soit

DE k = Ek,f − E k,i = 0

Dans tous les autres cas, le choc est inélastique : l’énergie cinétique perdue permet de modifier la nature des systèmes en les déformant, en les brisant en morceaux ou en augmentant leur température. On dit, en thermodynamique, que l’énergie interne ou que l’état interne des systèmes en interaction a changé (cf. Thermodynamique). Cette définition des chocs élastiques est compatible avec celle que l’on donne généralement dans les collisions entre particules, notamment en relativité (cf. chapitre 14 et Relativité et invariance). b) Facteur de restitution en énergie cinétique Il est naturel de caractériser un choc inélastique par le facteur de restitution en énergie, c’est-à-dire le rapport entre l’énergie cinétique finale E k,f du système et son énergie cinétique initiale E k,i : εe =

E k,f E k,i

avec

0  εe  1

La limite supérieure ε e = 1 correspond à un choc élastique (Ek,f = Ek,i), alors que la limite inférieure εe = 0 traduit la disparition totale de l’énergie cinétique disponible ; un tel choc est parfaitement inélastique. Les balles « superélastiques » des enfants ont un facteur ε e proche de l’unité. On peut mesurer ce facteur en comparant la hauteur hf atteinte par une telle balle que l’on l’abandonne d’une hauteur h i et qui subit un choc au contact du sol (Fig. 21.3a). L’énergie cinétique de la balle, avant le choc, est mvi2/2 = mgh i, en raison du théorème de l’énergie mécanique appliqué à la balle dans son mouvement antérieur. Son énergie cinétique, après le choc, est mv2f /2 = mgh f , d’après le théorème de l’énergie mécanique appliqué à la balle dans son mouvement ultérieur. Il en résulte que : εe =

v2f v2i

=

hf hi

Par exemple, en laissant tomber une balle « superélastique » d’une hauteur h i = 150 cm, on a trouvé hf = 118 cm. On en déduit : ε e = 0, 78 ≈ 0, 8. v1 − v 2

v 1 − v 2 S1

h h

S2

a)

b) F IG . 21.3.

369

Mécanique des chocs c) Facteur de restitution en composante normale de la vitesse relative

On utilise encore le facteur de restitution en composante normale de la vitesse relative, introduit par Newton (Fig. 21.3b). C’est le rapport εn des composantes normales (au plan tangent des deux systèmes en contact) de la différence des vitesses des points qui entrent en contact après et avant le choc : εn = −

(vf ,1 − v f ,2) · e n (vi,1 − v i,2) · e n

en étant le vecteur unitaire normal aux surfaces en contact. Dans l’exemple précédent facteurs de restitution ε e et ε n vf ,1 εn = − vi,1

où la vitesse du sol est nulle, on a la relation suivante entre les deux : hf 1/2 v · en d’où εn = f ,1 = = ε1e /2 = 0, 88 hi vf ,1 · en

III . — APPLICATIONS III . 1 . — Choc élastique de deux boules de billard Écrivons les théorèmes de la mécanique des chocs pour un système de deux boules identiques, telles que des boules de billard évoluant dans un plan horizontal (Fig. 21.4a). Si l’on néglige l’influence des forces extérieures de frottement et la déformation des solides, on a :

 c Dunod – Toute reproduction non autorisée est un délit

DP = 0

et DEk = 0

ce qui donne, en supposant la deuxième boule initialement immobile et en primant les grandeurs après le choc : p21 p 2 p 2 = 1 + 2 p 1 = p1 + p2 et 2m 2m 2m La simplification par la masse commune m donne deux équations qui montrent que les directions des boules, après le choc, sont orthogonales (Fig. 21.4b). En outre, le contact se faisant sans frottement, les échanges de quantité de mouvement par le contact sont des vecteurs qui passent par le centre de masse des boules ; le moment cinétique de chaque boule se conserve donc et par conséquent la vitesse angulaire aussi : V 1 = V1 et V 2 = V 2 p1 p1

p1

a)

b)

p2

F IG . 21.4.

III . 2 . — Choc inélastique de deux solides de masses différentes Un solide, de masse m 1, tombe, avec une vitesse de translation v 1 , sur un second solide initialement immobile, de masse m2 (Fig. 21.5). Supposons que les deux masses aient même vitesse v f après la collision et que l’échange de quantité de mouvement entre elles soit dirigé suivant la droite qui joint les deux centres de masse. Cette dernière hypothèse facilite l’étude de la rotation de chaque so-

370

21. Mécanique des chocs

lide puisque, l’échange de moment cinétique entre les solides étant nul, ces derniers gardent leur mouvement de rotation après le choc. La conservation de la quantité de mouvement de l’ensemble donne : m1v1 = (m 1 + m 2)vf d’où le facteur de restitution en énergie : (m1 + m2)v 2f /2 E k,f v m1 = = f = 2 m 1 v 1/2 m 1 + m2 E k,i v1 Deux cas méritent d’être étudiés : εe =

i) m 2  m1 . Le facteur ε e ≈ 1 ; il n’y a pratiquement pas d’énergie cinétique perdue. C’est le cas lorsqu’on souhaite que toute l’énergie cinétique soit utilisée pour mettre en mouvement la cible : ainsi, pour enfoncer un clou (masse m2) avec un marteau (masse m 1 ) on utilise un marteau lourd. ii) m 2  m 1 . Le facteur εe ≈ 0. La grande variation d’énergie cinétique est utilisée pour modifier l’état de la cible. Ce cas modélise le fonctionnement du marteau léger qu’utilisent les cantonniers pour briser des cailloux.

C1

θ1 C

C2

I F IG . 21.5.

F IG . 21.6.

III . 3 . — Choc élastique et sans frottement d’une boule sur un plan fixe Une boule (masse m 1, rayon r) heurte un plan fixe avec une vitesse v1 de son centre de masse et une vitesse angulaire V1 (Fig. 21.6) ; v1 fait l’angle u 1 avec la normale au plan. D’après les théorèmes généraux appliqués à la boule, on a, en primant les grandeurs cinématiques après le choc : 2m 1r2 (V1 − V1 ) = CI × P (r) DP = m 1(v1 − v1 ) = P (r) et DL C = 5 P(r) étant la quantité de mouvement reçue par la boule au point de contact I. Supposons le choc élastique ; alors P (r) est normal au plan fixe et la boule garde son énergie cinétique. Les équations précédentes donnent : 2m 1R2   v1,x − v 1,x = P(xr) v1,y − v1,y = 0 (V 1 − V1 ) = 0 et 5 puisque CI et P(r) sont colinéaires. En appelant u 1 l’angle que fait v1 avec la normale, la deuxième équation donne : v1 sin u 1 = −v1 sin u1 . Comme le choc est élastique et la vitesse de rotation inchangée V1 = V 1, la norme de la vitesse du centre de masse est elle aussi inchangée : v 1 = v1. On en déduit la relation suivante, bien connue en optique géométrique (loi de la réflexion) : u1 = −u1

371

Mécanique des chocs III . 4 . — Quantité de mouvement reçue sur l’axe a) Expression

Dans le problème concret du pendule balistique (Fig. 21.2), distinguons, dans la quantité de mouvement reçue P(r) , la contribution P(cr) associée aux actions de contact qu’exerce l’axe de rotation de celle P(Ar) due au projectile A. L’application des théorèmes généraux de la mécanique des chocs à la tige seule donne : (r) (r) (r) DP = P A + P (cr) et DL O = LO,A + LO,c (r)

(r)

où LO,A = OA × PA est le moment cinétique reçu en O, de la part de A. Comme DP = M v C − 0 = MV × OC et DLO = I OV − 0, il vient : P(cr) = DP − P (Ar) = MV × OC − P(Ar)

et L(Or,)c = DLO − L (Or,)A = IOV − OA × P (Ar)

b) Annulation des échanges sur l’axe On évite l’usure et donc la détérioration de la liaison en O en annulant les échanges au niveau de l’axe de rotation : (r) (r) 0 = MV × OC − PA et 0 = IOV − OA × P A d’où : IO V = OA × (M V × OC) = M[V(OC · OA) − OC(V · OA)] = MV(OC · OA)

puisque V est perpendiculaire à OA. Il en résulte, en identifiant, que le moment d’inertie doit être égal à : IO = M (OC · OA) = Mbd

 c Dunod – Toute reproduction non autorisée est un délit

d étant la distance du centre de masse à l’axe de rotation et b = OA la distance de l’axe de rotation à la direction de la quantité de mouvement re¸ue en A. La condition précédente s’écrit aussi : IC I C = IO − Md2 = Mbd − Md 2 = Md(b − d) d’où b = d + Md Dans le cas considéré d’un pendule balistique constitué d’une tige homogène oscillant autour d’un axe horizontal, on a d = l/2 et IC = Ml2/12 = Md 2/3 ; on annule donc la quantité de mouvement reçue sur l’axe en heurtant la tige à la distance b = 2l/3 de l’axe de rotation. Remarques : (1) Nous verrons au chapitre 25 que la longueur b = I O/(Md) représente la longueur du pendule simple synchrone du pendule pesant constitué par la tige. (2) Le mouvement de rotation d’une raquette de tennis autour du poignet d’un joueur peut être grossièrement assimilé au précédent : le joueur ressent les effets de l’opposé de la quantité de mouvement reçue sur l’axe au moment où la raquette heurte la balle. Ces effets sont négligeables si le joueur saisit le manche de la raquette à une distance b du point où se produit le choc sur la balle. Une même analyse pourrait être transposée à la bonne utilisation d’un marteau pour enfoncer un clou.

CONCLUSION Rappelons les points les plus importants : (1) Les théorèmes généraux de la mécanique des chocs ont pour expressions : t+t

DP = P (r) = t (r)

Fex d t

et DL O = L(Or) =

t+t t

M O,ex d t

où P (r) et LO désignent les échanges de quantité de mouvement et de moment cinétique par l’intermédiaire des forces et des moments extérieurs.

372

21. Mécanique des chocs

(2) En raison de la très faible durée des chocs, ces échanges sont négligeables si les forces sont finies : DP = 0 et DLO = 0 d’où la conservation de la quantité de mouvement et du moment cinétique : Pi = Pf et L O,i = LO,f (3) On distingue le choc élastique du choc inélastique à l’aide de l’énergie cinétique. Si l’énergie cinétique du système est conservée, le choc est élastique ; sinon il est inélastique. Dans le cas extrême où la perte d’énergie cinétique du système est maximale, le choc est parfaitement inélastique. On introduit généralement le facteur de restitution en énergie cinétique εe, rapport de l’énergie cinétique finale sur l’énergie initiale ; ce facteur vaut 1 pour une collision élastique et 0 pour une collision parfaitement inélastique.

EXERCICES ET PROBLÈMES P21– 1. Pendule balistique Un solide peut tourner sans frottement autour d’un axe horizontal Oz fixe dans un référentiel terrestre. On désigne par M sa masse, C son centre d’inertie et I son moment d’inertie par rapport à l’axe Oz (Fig. 21.7). À une date t = 0, un projectile de dimensions négligeables, de masse m, de vitesse v horizontale et orthogonale à Oz, heurte le solide au repos, en un point K où il reste collé. 1. Déterminer la vitesse angulaire V de l’ensemble, immédiatement après le choc, en fonction de m, v, I , h = OH et a = (OK , OC). 2. Exprimer les composantes, sur les axes Ox et Oy, de la quantité de mouvement du système, immédiatement après le choc, en fonction de M , l = OC, V, m, h et a. 3. Pour quelles valeurs de h et a, la réaction qui s’exerce sur l’axe est-elle nulle ? Déterminer alors l’énergie cinétique du système, immédiatement après le choc, en fonction de m, v, h et I . En déduire l’expression de la vitesse v du projectile en fonction de g, h, I, m et um l’élongation angulaire maximale après le choc. y g

Oz

v K

αC H

x OC = l OH = h F IG . 21.7.

H h

g C I

F IG . 21.8.

P21– 2. Coup horizontal sur une boule de billard Avec une queue de billard, on fournit, à une boule de billard (rayon r, masse m), une quantité de mouvement horizontale, au cours d’un choc localisé en un point de la boule situé à la distance h du plan du billard (Fig. 21.8). 1. Établir la relation entre la vitesse angulaire V  acquise par la bille et la vitesse v de son centre de masse. 2. En exprimant la vitesse de glissement après le choc, en fonction de v C, r et h, étudier le mouvement ultérieur de la bille dans le cas où h > 7r/5. Mêmes questions pour h = 7r/5 et h < 7r/5.

373

Mécanique des chocs P21– 3. Balle superélastique

Une balle sphérique superélastique (masse m, rayon r) heurte le sol horizontal Oxy avec une vitesse v et une vitesse angulaire V perpendiculaire au plan vertical Oxy contenant v (Fig. 21.9). Le contact a lieu avec frottement. y 1. Écrire les théorèmes de la mécanique des chocs appliqués à la balle, relatifs à la quantité de mouvement, au moment cinétique en son centre Ω de masse et à l’énergie cinétique. On supposera que la quantité de mouvev ment fournie à la balle est contenue dans le plan vertical Ozx. Le système d’équations obtenu peut-il être résolu sans l’adjonction d’équations suppléx mentaires ? F IG . 21.9. 2. Le choc est inélastique du fait des frottements, mais la variation d’énergie cinétique de la balle peut être négligée ainsi que la variation de la valeur absolue de la composante de la vitesse v. Que deviennent les équations précédentes ? En déduire les relations donnant la composante tangentielle de v et la rotation angulaire V après le choc, en fonction de ces mêmes quantités avant le choc. 3. Étudier le cas où V = 0 et la composante v x de la vitesse suivant l’axe horizontal Ox est positive. P21– 4. Collision inélastique parfaite de deux disques en rotation Un disque D 1 ayant un mouvement de rotation, sans frottement, de vitesse uniforme V1 , autour d’un axe vertical Oz, est mis en contact avec un disque D2 , tournant autour du même axe, sans frottement, à la vitesse uniforme V2 (Fig. 21.10). Grâce à des frottements entre D1 et D2 , les deux disques forment, au bout d’une courte durée, un solide unique D qui tourne autour de Oz à la vitesse V. 1. Exprimer V en fonction de V1 , V2 et des moments d’inertie respectifs I1 et I 2 . 2. Calculer la variation d’énergie cinétique. Commenter. O z

A V2

g

 c Dunod – Toute reproduction non autorisée est un délit

D2 D1

l

h

V1 F IG . 21.10.

F IG . 21.11.

P21– 5. Chute protégée par une corde Un alpiniste (masse 75 kg , harnachement compris), assimilé à un corpuscule A , est attaché à une corde de longueur l = 100 m , dont l’autre extrémité est fixée en O . À la suite d’un faux-pas, il fait une chute mais est finalement retenu par la corde (Fig. 21.10), laquelle limite la hauteur de chute à h = 50 m . 1. Exprimer la percussion reçue par l’alpiniste, en fonction de m , h et g . Calculer sa valeur. 2. En réalité, en raison de son élasticité, la corde s’allonge d’une longueur totale z m , sous l’action de la charge.

374

21. Mécanique des chocs

a) En admettant que l’énergie potentielle de pesanteur fournie par la chute s’est totalement transformée en énergie élastique de la forme Kz 2m/2 , K étant la raideur équivalente de la corde élastique, établir l’expression de zm en fonction de m , K , h et g accélération due à la pesanteur. b) En déduire la force maximale F m subie par l’alpiniste. Comparer sa valeur à son poids. Calculer la raideur K pour r = Fm /(mg) = 15 . 3. Le module d’Young E de la corde est relié à sa raideur K , sa section s et sa longueur l par : 1 s = E l

∂l ∂F

1 s = S lK

S étant l’entropie (cf. Thermodynamique). a) Exprimer F m en introduisant E , s et le facteur de chute fc = h/l . b) Calculer le module d’Young d’une telle corde de diamètre 1 cm . c) Trouver une estimation de la durée du choc, c’est-à-dire de la durée que la corde met pour s’allonger de h à h + zm . Calculer alors la valeur moyenne de la force qui s’exerce pendant le choc. P21– 6. Pendule balistique industriel En vue de déformer une pièce métallique, on utilise l’impact sur cette pièce d’un chariot de masse M c = 333 kg , articulée à l’extrémité inférieure d’un double portique formé de deux bras, de longueur l = 3, 5 m et de masse unitaire M p = 250 kg (Fig. 21.12). On désigne par Ox l’axe vertical descendant et par Oz l’axe de rotation. 1. a) Exprimer l’énergie potentielle de pesanteur E p de l’ensemble chariot et double portique, en fonction de l’angle d’inclinaison a par rapport à la verticale descendante. b) Quelle est, en fonction de la vitesse angulaire a˙ , l’énergie cinétique E k de l’ensemble, sachant que le mouvement du chariot se réduit à une translation. 2. a) Établir l’équation reliant l’angle initial a i à la vitesse finale v f du chariot à l’instant de l’impact pour lequel af = 0 . En déduire ai , si vf = 20 km.h −1 . b) Quelle aurait été la valeur de a i si toute la masse du pendule avait été concentrée au centre de masse du chariot ? Oz

y

g

a Pièce à déformer x F IG . 21.12.

22 Mécanique des systèmes ouverts. Théorèmes d’Euler Jusqu’à présent, nous n’avons considéré que des systèmes fermés, c’est-à-dire des ensembles matériels identifiés. C’est ainsi que dans l’évaluation des taux de variation de quantité de mouvement, du moment cinétique et de l’énergie, qui apparaissent dans l’écriture des théorèmes généraux de la mécanique, on considérait, au cours du mouvement, les mêmes éléments de matière. Les systèmes ouverts sont des ensembles matériels formant le contenu matériel d’une surface fermée à travers laquelle ils peuvent échanger, avec le milieu extérieur, non seulement de l’énergie, mais aussi de la matière. Cette surface est souvent appelée surface de contrôle Sc ou surface de référence. Nous supposerons pour simplifier l’étude que Sc est indéformable et fixe dans le référentiel considéré. Nous nous proposons d’établir la formulation particulière que prennent les théorèmes de la mécanique dans le cas des systèmes ouverts. Cependant, avant toute étude générale, il est instructif d’analyser quelques exemples de systèmes ouverts.

 c Dunod – Toute reproduction non autorisée est un délit

I . — EXEMPLES DE SYSTÈMES OUVERTS Les exemples de systèmes matériels qui échangent de la masse avec le milieu extérieur sont nombreux puisque tous les êtres vivants sont des systèmes ouverts. Les fusées, les comètes, qui évoluent autour du Soleil, et les gouttes d’eau, qui traversent l’atmosphère chargée de vapeur d’eau, sont aussi des exemples importants de systèmes ouverts. I . 1 . — Tabouret d’inertie avec abandon de masse Reprenons l’expérience classique illustrant la conservation de la projection verticale du moment cinétique d’un système constitué d’une personne assise sur un tabouret, lequel peut tourner parfaitement autour d’un axe vertical (cf. chapitres 18 et 25). Si la personne, initialement les bras tendus avec des masses identiques aux extrémités, ramène ses bras le long du corps, la vitesse de rotation finale Vf est plus élevée que sa vitesse de rotation initiale V i. Proposons une légère variante de cette expérience : les bras restant tendus, la personne en rotation abandonne (sans vitesse par rapport à elle) des masses identiques (Fig. 22.1). Le théorème du moment cinétique appliqué au système fermé que constitue l’ensemble {personne et masses}, donne, en projection suivant l’axe Oz : (f ) d LOz = Mex,Oz = 0 dt

376

22. Mécanique des systèmes ouverts. Théorèmes d’Euler

puisque la liaison est parfaite et que la projection du moment du poids en O suivant l’axe de rotation (f ) est nulle. Il en résulte que L Oz = Cte. y B2

Vi

B1

g Vf B 1

B2

b

O

b

B1 µ

x

B2 F IG . 22.1.

Appliquons cette conservation entre l’instant t où les masselottes sont encore tenues aux extrémités B1 et B2 et l’instant t + d t, où, bien qu’abandonnées, elles sont dans les voisinages respectifs de B1 et B2. En désignant par b = OB1 = OB 2 la longueur des bras, v 1 et v2 les vitesses des masselottes après abandon (sans vitesse), et IOz le moment d’inertie de la personne par rapport à l’axe de rotation, il vient : (I Oz + 2mb2)V i = I OzV f + (OB 1 × mv1 + OB 2 × mv 2)z Comme OB2 = −OB1 et v1 = −v 2 = bVf eu, on obtient : (IOz +2mb 2)V i = I OzVf +2(OB 1×mv 1)

soit

(I Oz+2mb 2)V i = (I Oz Vf +2mb 2)V f

et

Vf = Vi

C’est bien ce que l’on constate expérimentalement : l’abandon des masses ne modifie pas la vitesse angulaire de rotation du système. I . 2 . — Fusée Une fusée est un système matériel ouvert qui se déplace grâce à l’éjection de masse, en général des gaz brûlés (Fig. 22.2). Évaluons, par rapport au référentiel terrestre R, supposé galiléen, la variation de quantité de mouvement du système fermé caractérisé par la masse M(t) à l’instant t. À l’instant t + d t, la masse de la fusée est devenue M(t + d t) = M(t) + d M, plus faible que M (t), car la fusée a éjecté une masse (− d M > 0) de gaz. On a donc :

i) à l’instant t, P (f ) (t) = M (t)v(t) ; le signe f , qui est un indice et non un exposant, rappelle que le système auquel on applique les théorèmes généraux est fermé, ii) à l’instant t + d t, P (f )(t + d t) = M(t + d t)v(t + d t) + (− d M )[v(t + d t) + u], u étant la vitesse d’éjection des gaz par rapport à la fusée. On vérifie bien qu’à l’instant t + d t, on applique les théorèmes généraux au même système dont une partie a pour masse M(t + d t) < M(t) et l’autre − d M > 0. Il vient, en faisant la différence des quantités de mouvement :

d P(f ) = (M + d M )(v + d v) + (− d M)(v + d v + u) − Mv = M d v − d Mu Si Sex désigne la somme des forces extérieures, généralement le poids Mg et la résistance de l’air R, le théorème de la quantité de mouvement d P (f )/ d t = Sex donne : M

dv dM − u = Mg + R soit dt dt

M

dv + qmu = M g + R où dt

qm = −

dM dt

377

Mécanique des systèmes ouverts. Théorèmes d’Euler

est le débit-masse de gaz éjectés, positif car d M est négatif. Très souvent, on met le résultat précédent sous la forme : dv = M(t)g + R − q mu M(t) dt Ainsi, tout se passe comme si l’on pouvait considérer la fusée comme un système fermé soumis à la somme des forces habituelles et à une « force supplémentaire » −qmu. Cette dernière est appelée force propulsive car c’est elle qui détermine généralement le sens du mouvement. Remarque : Insistons bien sur l’expression tout se passe comme si ; donner à cette force supplémentaire une autre interprétation que celle provenant d’un bilan de quantité de mouvement pourrait conduire à des résultats erronés. M(t + dt) M(t) z R

O

Comète g S

(− dM ) > 0

y

T

x F IG . 22.2.

F IG . 22.3.

I . 3 . — Comète perdant de la masse Une comète est constituée en partie de glace. Aussi, au cours de son mouvement autour du Soleil, et sous l’action du rayonnement solaire perd-elle de la masse par sublimation (Fig. 22.3). Évaluons, pour le système fermé caractérisé par la masse M (t) à l’instant t, la variation de quantité de mouvement par rapport au référentiel galiléen de Copernic. On a : i) à l’instant t, P (f )(t) = M (t)v(t),

 c Dunod – Toute reproduction non autorisée est un délit

ii) à l’instant t + d t : P (f ) (t + d t) = M(t + d t)v(t + d t) + (− d M )v(t + d t). Comme pour la fusée, le système auquel on applique les théorèmes généraux est constitué des mêmes éléments. À l’instant t + d t, une partie de la glace (− d M > 0) a été sublimée.

Explicitons la différence des quantités de mouvement précédentes. Comme v(t + d t) = v + d v, il vient : d P(f ) = (M + d M )(v + d v) + (− d M)(v + d v) − Mv = M(v + d v) − M v = M d v En utilisant le théorème de la quantité de mouvement, d P(f ) / d t = S ex, S ex désignant la somme des forces qui s’exercent sur la comète, principalement la gravitation, il vient : M(t)

dv = Sex dt

Remarque : Ce résultat n’est pas surprenant : c’est celui de la fusée avec une vitesse d’éjection u nulle.

378

22. Mécanique des systèmes ouverts. Théorèmes d’Euler

I . 4 . — Goutte d’eau évoluant dans une atmosphère chargée de vapeur d’eau Lorsqu’une goutte d’eau évolue dans une atmosphère de vapeur saturante d’eau, sa masse peut augmenter du fait de la condensation. Évaluons, par rapport au référentiel terrestre, la variation de quantité de mouvement du système fermé, de masse M(t + d t), constitué de la goutte d’eau, de masse M(t), et de la masse d’eau d M > 0 qui se condense à t + d t. On a : i) à l’instant t, P(f ) (t) = M (t)v(t) + d M × 0 = Mv ,

ii) à l’instant t + d t, P (f )(t + d t) = M (t + d t)v(t + d t) = (M + d M )(v + d v). Là aussi, on vérifie que le système considéré est bien fermé, puisque aux instants t et à t + d t ce sont les mêmes éléments de masse qui sont pris en compte. Effectuons la différence de ces quantités de mouvement : d P(f ) = M d v + v d M + d M × d v

soit

d P(f ) = M d v + v d M

puisque le terme d M × d v est négligeable. Si Sex désigne la somme des forces extérieures qui s’exercent sur la goutte entre les instants t et t + d t, le théorème de la quantité de mouvement, d P(f )/ d t = S ex , donne : dv dM +v = Sex M dt dt

II . — CARACTÈRE CONSERVATIF DE LA MASSE Nous avons caractérisé un système ouvert par le contenu matériel variable d’une surface fermée fixe Sc par rapport au référentiel d’étude R (Fig. 22.4a). À travers S c s’effectuent des échanges de matière avec l’extérieur. La masse d’un système matériel est une grandeur conservative, c’est-à-dire que le terme de création de masse dans un bilan de masse est nul (cf. chapitre 13) : entre deux dates voisines t et t + d t, on a : d M = dM(r) + dM(c)

avec

dM (c) = 0

dM r étant la masse reçue à travers la surface S c. Ainsi, la variation de la masse du contenu matériel de la surface Sc ne peut être attribuée qu’à l’échange de masse à travers la surface.

Considérons le cas particulier important où les échanges sont localisés en deux points de S c . C’est la situation fréquente où, pendant la durée élémentaire d t, Sc reçoit la masse dme par l’orifice d’entrée Ae et perd la masse dm s par l’orifice de sortie A s (Fig. 22.4b). On a : d M = dM(r) = dme − dms

soit aussi

dme dms dM = qm,e − qm,s − = dt dt dt

en introduisant le débit-masse qm = dm/ d t à l’entrée et à la sortie. L’équation de conservation de la masse s’écrit donc : dM = (qm )es dt En régime stationnaire d M/ d t = 0 et par conséquent : (qm) es = 0 soit : qm,e = qm,s = q m

379

Mécanique des systèmes ouverts. Théorèmes d’Euler

n d

Ae v e

dS

ne

ΔSe

a)

ns

As ΔSs

vs Sc

b) F IG . 22.4.

Remarques : (1) Dans le cas de la fusée et de la comète, la variation d M de la masse se réduit à −dm s puisque dme = 0. Pour la goutte d’eau d M = dm e. (2) Notons l’analogie entre la conservation de la masse et la conservation de la charge électrique (cf. Électromagnétisme). (3) En relativité, l’équation-bilan de masse n’existe pas de façon autonome, car la masse n’est qu’une forme de l’énergie (cf. Relativité et invariance).

III . — THÉORÈME DE LA QUANTITÉ DE MOUVEMENT III . 1 . — Énoncé D’après ce qui précède, l’équation-bilan de la masse s’écrit, dans le cas où les échanges de matière sont localisés en deux points, l’entrée A e et la sortie A s (Fig. 22.4b) : d M = dme − dm s ce qui donne

M(t) + dm e = M(t + d t) + dms

Appliquons le théorème de la quantité de mouvement, entre les instants t et t + d t, au système fermé de masse M(t) + dme = M(t + d t) + dms . Il vient : d P(f ) = Sex d t

avec

P(f ) (t) = P(t) + dme ve

et P(f ) (t + d t) = P(t + d t) + dm svs

 c Dunod – Toute reproduction non autorisée est un délit

P(t) étant la quantité de mouvement du contenu matériel de S c à l’instant t. Il en résulte, en effectuant la différence : d P + dmsv s − dme ve = Sex d t soit d P = Sex d t + (dmv) es Cette équation traduit le bilan de quantité de mouvement relatif à un système ouvert, entre deux instants infiniment voisins ; le premier terme de droite est un échange par l’intermédiaire des forces et le second un échange par transfert de matière. Comme la quantité de mouvement est une grandeur conservative, il n’y a pas de terme de création. En divisant le bilan précédent par d t, on obtient : dP = S ex + (qm v)es dt Notons que ce bilan est relatif au référentiel R par rapport auquel la surface de contrôle S c est fixe et indéformable. Si R n’est pas galiléen, il faut ajouter, aux forces dues à la présence d’autres corps, les forces d’inertie d’entraînement et de Coriolis. III . 2 . — Théorème d’Euler relatif à la quantité de mouvement En régime stationnaire (d P/ d t = 0), le théorème de la quantité de mouvement d’un système ouvert se réduit au théorème d’Euler, du nom du mathématicien suisse L. Euler. Comme q m,e = q m,s = qm , il s’écrit : qm(vs − ve) = S ex

380

22. Mécanique des systèmes ouverts. Théorèmes d’Euler

III . 3 . — Exemples d’application du théorème de la quantité de mouvement a) Équation du mouvement de la fusée Le fonctionnement de la fusée relève de l’application directe du théorème d’Euler par rapport au référentiel non galiléen R lié à l’armature de la fusée. Dans ce cas, v e = 0, v s = u et S ex est la somme des forces suivantes : le poids M(t) g, la résultante aérodynamique R et la force d’inertie d’entraînement −M (t) a, a désignant l’accélération d’un point quelconque de la fusée, du fait de la translation, par rapport à un référentiel galiléen R. Il en résulte que : qmu = M(t)g + R − M (t) a On retrouve bien l’équation déjà établie, dans laquelle u est la vitesse d’éjection des gaz par rapport à la fusée : M(t) a = M(t)g + R − qmu Ordre de grandeur : La force de poussée au décollage du lanceur Ariane 5, dont la masse totale est 730 t, est obtenue grâce au moteur Vulcain, qui développe une poussée de 0, 885 MN, (1 MN = 106 N) et de deux propulseurs à poudre de poussée 2 × 5, 3 = 10, 6 MN La résultante R est proportionnelle au carré de la vitesse (cf. chapitre 31) : R = C x r aSv 2/2 ; dans cette expression, Cx est un facteur sans dimension qui dépend de la forme de la fusée, ra est la masse volumique de l’air et S l’aire de la surface de la section de la fusée dans un plan perpendiculaire à la vitesse. Exprimons le gain de vitesse que procure la force de poussée en supposant la résultante aérodynamique nulle. Cette hypothèse apparemment simpliste donne de bons résultats, car, en début d’ascension, alors que la fusée se trouve dans des zones atmosphériques denses, sa vitesse est faible et, en fin d’ascension, alors que la vitesse est grande, l’air est raréfié. En divisant les deux membres de l’équation précédente par la masse M et en intégrant entre l’instant initial pris comme origine et l’instant t au bout duquel la totalité du comburant et du carburant a été consommée, on obtient : t

v − v0 = −

0

g dt −

M(t)

u M0

dM M

soit, en négligeant les variations du champ de pesanteur avec l’altitude : v − v 0 = −gt + u ln

M0 M(t)

Exemple : Si M0 = 158 t, t = 590 s M(t) = 9, 7 t et u = 4 200 m . s −1, on trouve : v − v0 < 6, 01 km . s−1.

Ainsi, il est impossible, avec une fusée à un seul étage, de communiquer à une masse, située dans le voisinage de la Terre, une vitesse égale à la vitesse d’évasion de 11, 2 km . s −1 (cf. chapitre 12).

Pour obtenir cette vitesse, on utilise des fusées gigognes constituées généralement de plusieurs étages qui se séparent du reste de la fusée lorsqu’ils ont consommé leur réserve de comburant et carburant. L’accroissement global de vitesse est alors la somme des accroissements de vitesse de chaque étage. Sur la figure 22.5, on a explicité les caractéristiques du lanceur européen Ariane 5.

381

Mécanique des systèmes ouverts. Théorèmes d’Euler

Coiffe Charge utile Case à équipements Étage à propergol stockable (EPS)

54 m

Réservoir d’oxygène liquide Étage d’accélération à poudre (EAP) Réservoir d’hydrogène liquide Étage principal cryogénique (EPC)

Moteur vulcain Masse au décollage 770 t F IG . 22.5.

Remarques : (1) On caractérise souvent la performance des propergols (comburants et carburants) par le rapport i = u/g, homogène à une durée, appelée impulsion spécifique. Cette dernière vaut respectivement 273 s, 431 s et 324 s, pour les différents étages. (2) Afin de développer les lancements de satellites à bas coût, l’Europe compte se doter d’un nouveau lanceur Ariane 6, dont les deux premiers étages seront constitués de propulseurs à propergol solide. (3) Comme cela a déjà été dit, l’équation du mouvement de la comète est analogue à celle de la fusée ; dans ce cas, u = 0 et la somme des forces se réduit à l’attraction gravitationnelle.

 c Dunod – Toute reproduction non autorisée est un délit

b) Équation du mouvement de la goutte d’eau Le théorème de la quantité de mouvement précédemment établi permet de retrouver l’équation du mouvement de la goutte d’eau. Il suffit de noter que les vitesses de transfert de masse sont nulles. Il vient alors : dP dv dM = S ex soit M +v = Sex dt dt dt c) Propulsion d’un avion à turboréacteur Les moteurs d’un avion à turboréacteur, se déplaçant à la vitesse v reçoivent de l’air qu’ils rejettent, avec une vitesse u par rapport à l’avion (Fig. 22.6). Comme la masse de combustible brûlé est très faible devant celle de l’air transféré, nous la négligeons. Dans le référentiel non galiléen lié à l’avion, le théorème d’Euler donne : qm(vs − ve) = M(t)g + R − M(t)a

avec ve tel que 0 = v e + v et vs = u. Il vient donc :

M(t)a = M(t)g + R − qm(u + v)

382

22. Mécanique des systèmes ouverts. Théorèmes d’Euler

g θ

R u F IG . 22.6.

La quantité −qm(u + v) est la force propulsive. En projetant cette équation suivant la direction du déplacement qui fait l’angle u avec l’horizontale, on trouve : M(t)a = −M(t)g sin u + Rt + qm (u − v ) Exemple : lorsque M(t) = 4 t, u = 600 m . s −1, q m = 90 kg . s−1 et u = 30 ◦, on obtient les valeurs suivantes des forces, lorsque la vitesse v est constante (a = 0) et vaut v = 720 km . h−1 soit v = 200 m . s−1 : M(t)g sin u = 19, 62 kN et Rt = −16, 38 kN qm (u − v) = 36 kN d) Bande transporteuse de minerai On transporte fréquemment du minerai concassé à l’aide d’une bande transporteuse qui se déplace à vitesse constante v. Considérons une surface de contrôle Sc qui entoure une longueur de bande (Fig. 22.7). On distingue deux entrées, E1 par laquelle arrive le minerai avec un débit-masse qm , E2 (b) par laquelle entre la bande avec un débit qm , et une sortie S par laquelle sortent la bande et le minerai avec les mêmes débits respectifs. En l’absence de frottement, si on ne déverse pas de minerai, aucune force horizontale F h n’est nécessaire pour maintenir cette vitesse. En revanche, lorsqu’on déverse du minerai, une telle force devient nécessaire ; son expression s’obtient directement à l’aide du théorème d’Euler. Si v e désigne la vitesse du minerai à l’entrée, on a, en régime stationnaire : q (mb) (v − v) + qm(v − v e) = S ex

soit

qm (v − ve ) = Sex

Sex désignant la somme des forces qui s’exercent sur le système. On a donc, suivant un axe horizontal : q mv = F h. Minerai

E1 E2

S Minerai

v Bande transporteuse

Sc F IG . 22.7.

IV . — THÉORÈME DU MOMENT CINÉTIQUE IV . 1 . — Énoncé Appliquons le théorème du moment cinétique, entre les instants infiniment voisins t et t + d t, au système fermé de masse M(t) + dm e = M(t + d t) + dm s, (Fig. 22.4b). Il vient : (f )

d LO = M O,ex dt

avec : (f )

(f )

LO (t) = L O (t) + OAe × dme v e et LO (t + d t) = L O(t + d t) + OA s × dms vs

383

Mécanique des systèmes ouverts. Théorèmes d’Euler

L O(t) étant le moment cinétique en un point fixe O du contenu matériel de la surface S c à l’instant t. Il en résulte, en effectuant la différence : d L(Of ) = d L O + OAs × dms vs − OAe × dme ve = M O,ex d t

d’où

d LO = M O,ex d t + (OA × dmv) es

Cette équation exprime le bilan du moment cinétique entre les deux instants t et t + d t : le premier terme de droite représente l’échange par l’intermédiaire des moments de force et le second l’échange associée au transfert de matière. Comme le moment cinétique est une grandeur conservative, il n’y a pas de terme de création. En divisant par d t, on obtient : d LO = MO,ex + (qmOA × v) es dt Remarque : Comme pour la quantité de mouvement, ce bilan est relatif au référentiel R par rapport auquel la surface de contrôle S c est fixe. Si R n’est pas galiléen, il faut ajouter, aux forces dues à la présence d’autres corps, les forces d’inertie d’entraînement et de Coriolis. IV . 2 . — Théorème d’Euler relatif au moment cinétique

En régime stationnaire (d L O / d t = 0), le théorème du moment cinétique d’un système ouvert se réduit à, puisque q m = q m,e = q m,s : q m(OAs × v s − OAe × ve ) = MO,ex IV . 3 . — Tourniquet hydraulique et tourniquet à air

 c Dunod – Toute reproduction non autorisée est un délit

Dans le tourniquet hydraulique destiné à l’arrosage des jardins, l’eau pénètre en E situé sur l’axe vertical de rotation et est expulsée par les ajutages A et B situés aux extrémités de deux bras horizontaux symétriques (Fig. 22.8a). Dans le tourniquet de laboratoire à air, qui est en verre, on remplace simplement l’eau par l’air. Le débit-masse de fluide sortant par chaque ajutage est rsu , r étant la masse volumique du fluide, s la section des ajutages et u la vitesse d’éjection du fluide par rapport aux bras, en A et B respectivement. y

z

−u

B

u

O

A

O

b E a)



Sc −u

µ

B

u

A b

Ω0

x O

b)

¿

t c)

F IG . 22.8.

Adoptons comme surface de contrôle S c la surface cylindrique, fixe dans R , définie par l’axe Oz du tourniquet et par la surface de base balayée par les bras (Fig. 22.8b). Appelant b la longueur des bras et u l’angle de rotation, on a, pour les vitesses par rapport à Sc des particules de fluide qui émergent des ajutages : v(A) = u + b u˙ eu et v(B) = −u − bu˙ eu = −v(A)

384

22. Mécanique des systèmes ouverts. Théorèmes d’Euler

a) Équation différentielle du mouvement Le mouvement satisfait à l’équation suivante : d LO + OA × rsuv(A) + OB × rsuv(B) − OE × 2rsuv(E) = M O,ex dt Comme la contribution de l’eau à l’entrée est nulle ( OE et v(E) sont parallèles) et que la somme M Oz,ex des moments par rapport à cet axe des forces de pesanteur et des actions de contact sur la liaison pivot parfaite, est nulle, on obtient, en projection suivant l’axe de rotation Oz : d LOz + 2rsub(bu˙ − u cos b) = 0 dt Dans cette équation, b est l’angle que fait la vitesse u avec la normale au bras du tourniquet. Ce résultat traduit essentiellement la conservation du moment cinétique selon l’axe de rotation. En introduisant le moment d’inertie I du tourniquet rempli de fluide, on obtient l’équation différentielle linéaire suivante : u˙ ¨ u+ =a u + 2rsub(bu˙ − u cos b) = 0 soit I¨ t sous sa forme canonique dans laquelle : I 2rsu2b cos b et a = 2rsub2 I La résolution de cette équation différentielle linéaire est caractéristique de la mise en rotation d’une machine tournante (cf. chapitre 25). La solution s’écrit : ˙u = Cte × exp − t + at t Comme at = V 0 = u cos b/b , il vient, si à l’instant initial ˙u = 0 : t u˙ = V0 1 − exp − t ˙ La courbe u(t) (Fig. 22.8c) montre qu’en régime stationnaire V0 est pratiquement atteint au bout de quelques valeurs de t . t=

Remarques : (1) Soulignons que le terme proportionnel à u˙ n’est pas dû à un quelconque frottement visqueux, mais au caractère ouvert du système. Un terme supplémentaire de frottement visqueux, de la forme −a u˙ , aurait un effet analogue ! La nouvelle durée caractéristique serait t = t + I/a . (2) On peut retrouver l’équation caractéristique du tourniquet en adoptant une surface de contrôle entourant les bras en rotation. Le référentiel d’étude n’étant pas galiléen, il faut prendre en compte les forces d’inertie d’entraînement et de Coriolis (cf. Exercices). b) Régime stationnaire En régime stationnaire, l’équation différentielle précédente se réduit à : 2rsub(bu˙ − u cos b) = 0 d’où

u˙ = V0 =

u cos b b

Notons que la vitesse angulaire V 0 est maximale pour b = 0 et nulle pour b = p/2 . C’est bien ce que montre l’expérience faite avec un tourniquet hydraulique de jardin dont les ajutages peuvent être différemment orientés.

385

Mécanique des systèmes ouverts. Théorèmes d’Euler c) Ordres de grandeur

Dans le cas d’un tourniquet d’arrosage de jardin pour lequel b = 0, 5 m , I = 0, 25 kg.m 2 , b = 0 , u = 10 m.s−1 et s = 3 cm2 , on trouve : u I ≈ 0, 17 s V0 = = 20 rad.s−1 soit 3, 2 tr.s −1 et t = 2rsub2 b Avec un tourniquet à air, le résultat est analogue mais plus spectaculaire, en raison de la faible valeur de b et de la liaison verre-verre pratiquement parfaite ; il suffit de souffler légèrement pour observer une rotation rapide de la tête du tourniquet. En effet, avec b = 7 cm et u = 5 m.s −1 , on obtient, pour b=0 : 5 u ≈ 71, 4 rad.s−1 soit 11, 3 tr.s−1 V0 = = 0, 07 b Remarque : Le fonctionnement du tourniquet à eau ou air aspiré, et non expiré, a préoccupé de grands physiciens de Princeton, notamment R. Feynman et A. Wheeler.

V . — THÉORÈME DE L’ÉNERGIE CINÉTIQUE V . 1 . — Énoncé Appliquons le théorème de l’énergie cinétique, entre les instants infiniment voisins t et t + d t, au système fermé de masse M(t) + dm e = M(t + d t) + dm s. Il vient : (f )

 c Dunod – Toute reproduction non autorisée est un délit

d Ek

= dWex + dWin

dWex et dW in étant les travaux élémentaires des forces extérieures et intérieures qui s’exercent sur le système fermé. Or : 1 1 (f ) (f ) Ek (t) = Ek (t) + dmev 2e et Ek (t + d t) = Ek(t + d t) + dms vs2 2 2 Ek(t) étant l’énergie cinétique du contenu matériel de la surface S c à l’instant t. Il en résulte, en faisant la différence : 1 1 (f ) d Ek = d Ek + dms v2s − dme ve2 = dW ex + dW in 2 2 d’où : e 1 d Ek = dWex + dmv2 + dWin 2 s Cette équation traduit le bilan d’énergie cinétique du système ouvert Sc : les deux premiers termes représentent l’échange d’énergie cinétique, l’un par l’intermédiaire du travail des forces extérieures, l’autre par le transfert de matière ; le troisième est le terme de création. En divisant par d t, on trouve : d Ek = Pex + P in + dt

qmv 2 2

e s

en introduisant les puissances Pex et Pin des forces extérieures et des forces intérieures. Comme pour les deux premiers théorèmes, ce bilan est relatif au référentiel R par rapport auquel la surface de contrôle Sc est fixe et indéformable. Remarque : Si R n’est pas galiléen, il faut ajouter à la puissance des forces dues à la présence d’autres corps celle de la force d’inertie d’entraînement, la force de Coriolis ne travaillant pas.

386

22. Mécanique des systèmes ouverts. Théorèmes d’Euler

V . 2 . — Théorème d’Euler relatif à l’énergie cinétique En régime stationnaire (d E k/ d t = 0), le théorème de l’énergie cinétique relatif à un système ouvert se réduit à, puisque qm,e = q m,s = qm : qm

v 2s v2 − e 2 2

= Pex + Pin

V . 3 . — Exemples a) Analyse énergétique de la bande transporteuse Calculons la puissance de la force horizontale F h = qmv à exercer, en régime stationnaire, sur la bande transporteuse de minerai (Fig. 22.7), sachant que le matériau tombe sur la bande avec une vitesse ve négligeable : Fh · v = Fh v = q mv 2

Cette puissance représente le double du terme d’énergie cinétique. Il en résulte, puisque les autres forces extérieures ne travaillent pas : qbm 2 q q m v2 (v − v2 ) + m (v − 0)2 = F hv + Pin soit = q m v 2 + Pin 2 2 2 La puissance des forces intérieures vaut donc : P in = −qmv 2/2. Elle est négative car elle est due aux forces intérieures de frottement qui s’exercent sur la surface de contact entre le minerai et la bande, ce qui permet l’entraînement du minerai. b) Analyse énergétique du tourniquet hydraulique Dans le tourniquet hydraulique en régime stationnaire, une masse d’eau dm e pénètre avec une vitesse ve par rapport au sol et émerge aux deux extrémités avec une vitesse nulle. Il en résulte une perte d’énergie cinétique que l’on doit attribuer à l’augmentation de l’énergie potentielle de pesanteur dme gh, h étant la hauteur du tourniquet, à la différence de pression entre l’entrée et la sortie et au travail des forces de viscosité.

CONCLUSION Retenons la formulation suivante des théorèmes généraux de la mécanique des systèmes ouverts échangeant de la matière en deux points, l’entrée Ae et la sortie As : e

q m v2 dP d LO d Ek = Pex + P in + = Sex + (qmv)es = M O,ex + (q mOA × v)es dt dt dt 2 s P, LO et E k étant les éléments cinétiques du contenu matériel d’une surface de contrôle S c , indéformable et fixe par rapport au référentiel R considéré. Cette formulation s’inspire des bilans de grandeurs : les termes (qmv)es , (qmOA × v)es et (q m v2 /2)es représentent les échanges dus aux transferts de masse du système ouvert étudié ; seule l’énergie cinétique est une grandeur non conservative en raison de la puissance non nulle des forces intérieures. L’importance pratique de l’étude est considérable comme le montrent les exemples traités (fusée, turboréacteur, etc.). Dans le cas fréquent des régimes stationnaires, les résultats précédents sont connus sous le nom de théorèmes d’Euler et s’écrivent : s v2 s s = Pex + P in et qm qm(v)e = Sex qm(OA × v)e = MO,ex 2 e

387

Mécanique des systèmes ouverts. Théorèmes d’Euler

EXERCICES ET PROBLÈMES P22– 1. Phase balistique du lancement d’une fusée Une fusée, en mouvement vertical, a les caractéristiques suivantes : – masse des structures et de l’équipement : m e = M – masse au départ du mélange propulsif : m p = M – la vitesse (relative à la fusée) d’éjection des gaz brûlés est constante : u = 2gT (g ≈ 9, 80 m . s −2) – débit-masse des gaz brûlés : q m = 2M /T .

On néglige la résistance de l’air et la variation du champ de pesanteur g avec l’altitude. 1. Exprimer, en fonction de T, l’accélération à tout instant, la vitesse v 1 et l’altitude z 1 de la fusée au moment où la combustion du carburant se termine. 2. Sachant que T = 200 s, calculer v 1, z1 et l’altitude maximale zm atteinte par la fusée. P22– 2. Système articulé en rotation Une tige cylindrique de section négligeable, de masse M, porte deux petites boules, de masse m, qui peuvent coulisser sans frottement. Cette tige tourne sans frottement avec la vitesse angulaire V0 autour d’un axe vertical Oz, qui lui est perpendiculaire et qui passe par son centre de masse (Fig. 22.9) ; les deux boules sont maintenues à la distance d de l’axe de rotation à l’aide de deux fils. On coupe les fils et on abandonne le système à lui-même. 1. Trouver, en appliquant le théorème du moment cinétique, la vitesse de rotation angulaire V 1 de la tige lorsque les boules atteignent les extrémités de la tige situées à la distance l de l’axe. En déduire, à l’aide de l’énergie, la vitesse de glissement v des boules sur la tige. 2. Calculer la vitesse angulaire V 2 lorsque les deux boules quittent les extrémités de la tige.

z

 c Dunod – Toute reproduction non autorisée est un délit

z

F

g O

y O

x F IG . 22.9.

F IG . 22.10.

P22– 3. Chaîne soulevée à vitesse constante Une chaîne homogène, de masse linéique r l , repose en tas sur le plan horizontal d’un référentiel terrestre (Fig. 22.10). On soulève la chaîne, à vitesse constante v, en exerçant une force F sur son extrémité supérieure. Exprimer F en fonction de v et de la cote z du centre de masse de la partie rectiligne de la chaîne.

388

22. Mécanique des systèmes ouverts. Théorèmes d’Euler

P22– 4. Déplacement d’un véhicule par perte de matière Un véhicule, rempli d’eau, est en mouvement sur un plan horizontal, grâce à l’échappement d’eau par un orifice de section s situé à l’arrière (Fig. 22.11). Exprimer, en fonction du débit-masse d’eau q m et de la vitesse u horizontale d’éjection par rapport au chariot, la force horizontale exercée par le sol qui permet au chariot d’avancer à vitesse constante ; on appliquera le théorème d’Euler à la surface de contrôle pSc , en pointillés sur la figure, fixe par rapport au camion. Application numérique : s = 10 cm2 et u = 10 m . s −1 . S

g

g

h u F IG . 22.11.

F IG . 22.12.

P22– 5. Avion à décollage vertical Chacun des deux moteurs d’un avion à décollage vertical, de masse 7 tonnes, éjecte vers le bas 60 kg d’air et 1 kg de carburant par seconde, à la vitesse de 600 m . s−1 (Fig. 22.12). Calculer la valeur de l’accélération verticale au décollage, lorsque la vitesse de l’avion est nulle. P22– 6. Tourniquet en référentiel non galiléen Retrouver l’équation du tourniquet en appliquant le théorème du moment cinétique dans un référentiel non galiléen lié aux bras du tourniquet. Quelle force doit-on exercer au milieu de l’un des bras pour le maintenir immobile ?

23 Statique La statique est l’étude des conditions pour lesquelles les corps sont immobiles, relativement à un référentiel R, galiléen ou non, lié à l’observateur. Ces conditions concernent notamment la répartition des forces qui s’exercent sur ces corps au repos. Cette étude conduit donc à l’analyse des solutions qui réalisent la constance des degrés de liberté {qi } du système considéré. Nous allons d’abord établir, dans le cas de systèmes de corps ponctuels et de solides, les conséquences qu’entraîne l’immobilité sur les forces. Nous étendrons ensuite ces résultats à des systèmes déformables tels que les fils. La statique des fluides sera envisagée ultérieurement (cf. chapitre 28).

I . — STATIQUE DES CORPS PONCTUELS ET DES SOLIDES Les équations de la statique qui expriment l’immobilité d’un système sont celles de la dynamique qi } des degrés de liberté. Si le dans lesquelles on a annulé les dérivées premières {˙q i} et secondes { ¨ système est constitué de N corps ponctuels et s solides, les conditions de l’immobilité du système sont celles de l’immobilité de chacun des N corpuscules et de chacun des s solides.

 c Dunod – Toute reproduction non autorisée est un délit

I . 1 . — Statique d’un corps ponctuel Considérons un corpuscule A soumis à plusieurs forces (force à distance, force de contact et éventuellement forces d’inertie). Comme la position de A est une constante (r = Cte, v = 0), d’après la loi fondamentale de la dynamique d p/ d t = F, on doit avoir : F=0 F contient, en outre, la force d’inertie d’entraînement, la force Si le référentiel R n’est pas galiléen, d’inertie de Coriolis étant nulle du fait de l’immobilité. Ainsi, pour qu’un corpuscule, initialement immobile par rapport à un référentiel R, reste au repos, F des forces qui lui sont appliquées soit nulle. Évidemment, si le corps il faut et il suffit que la somme n’est pas initialement immobile, cette condition sur la somme des forces n’implique pas l’immobilité ultérieure puisqu’alors a = 0 et donc v = Cte. Il convient donc de distinguer l’immobilité (par rapport à un référentiel) et l’équilibre mécanique F = 0. que l’on définit par la seule condition

390

23. Statique

Exemple : Considérons une masselotte immobile sur une tige en mouvement sous l’action de son poids mg = −mg ez, de la réaction R sans frottement qu’exerce la tige, de la tension T = −K (r −l 0 ) e r due au ressort et de la force d’inertie d’entraînement Fie (Fig. 23.1) : Fie = −m ae = −m V × (V × OA) = m V 2HA H étant la projection de A suivant l’axe de rotation. Cherchons la position de repos de A sur la tige, dans le cas où cette dernière tourne uniformément autour d’un axe vertical, en faisant un angle constant avec cet axe : w˙ = V = Cte et u = u0 = Cte Écrivons que la somme de ces quatre forces est nulle : m g + R − K (r − l 0 ) er + m V2 HA = 0 ce qui donne, en projetant dans la base sphérique {eu , ew, e r } et en notant que V = V ez et OA = r e r : −mg

R

− sin u 0 0 0 V 2 sin u 0 cos u0 Ru 0 0 0 Rw + mr 0 + + = 2 2 cos u 0 R −K (r − l 0) R 0 R V sin u0 R 0

La dernière équation fournit la relation entre V, u0 , r pour que A soit immobile sur la tige. Les deux autres équations permettent d’exprimer les composantes de la réaction. On retrouve bien que la réaction est contenue dans le plan vertical Ouz, comme les autres forces : Rw = 0. H Ω

z

y r

θ0

I2

A

g

eϕ O ϕ

x

R2 K

C

g

y

P θ0

u O



Ás Rt

® I1

F IG . 23.1.

x

F IG . 23.2.

I . 2 . — Statique d’un solide a) Condition nécessaire et suffisante Considérons un solide S soumis à un système de plusieurs forces dans un référentiel R. Le nombre de ses degrés de liberté est six, par exemple les trois coordonnées du centre de masse et les trois angles d’Euler (cf. chapitre 16). D’après les théorèmes généraux de la mécanique, S initialement au repos le reste si le torseur des forces extérieures est nul, précisément si la somme et le moment des forces extérieures sont tous deux nuls : S ex = 0

et MI,ex = 0

I étant un point quelconque de R car, la somme des forces extérieures étant nulle, la somme des moments de ces forces a même valeur en tout point (cf. chapitre 1).

391

Statique b) Rôle des actions de contact

Dans le torseur des actions extérieures, il convient de distinguer les actions de contact, en parties inconnues, des actions connues. Notant [F, M] le torseur des forces connues (somme et moment) et [R, G] le torseur des forces de contact (réaction et moment des actions de contact), la condition précédente s’écrit, en un point quelconque I : F+R = 0

et

M I + GI = 0

Comme un même solide peut être au repos sous l’action d’un autre torseur de forces connues, la relation précédente doit être interprétée comme une « réaction » adaptée du torseur des actions de contact (R, GI) au torseur des actions connues. c) Traduction analytique On explicite généralement les équations vectorielles précédentes dans la base la plus commode. Dans cette dernière on obtient alors les six équations suivantes : F x + Rx = 0 F y + Ry = 0 F z + Rz = 0

M x + Gx = 0 M y + Gy = 0 M z + Gz = 0

et

Il existe des cas particuliers importants qu’il est utile d’envisager. i) Si toutes les forces appliquées sont parallèles ou concourantes, le torseur est celui d’une force unique (cf. chapitre 1). L’annulation de ce torseur ne fournit que trois équations. ii) Si toutes les forces appliquées sont situées dans un même plan, par exemple Oxy, il est judicieux d’utiliser un système d’axes lié à ce plan ; les équations de l’équilibre se réduisent alors aux trois équations suivantes : F x + Rx = 0

Fy + R y = 0

et

Mz + Gz = 0

iii) Si le torseur des forces appliquées au solide est celui associé à trois forces, son équivalence à un torseur nul impose aux trois forces d’être coplanaires et concourantes (cf. chapitre 1). d) Exemple : équilibre d’une échelle

 c Dunod – Toute reproduction non autorisée est un délit

Pour simplifier l’analyse, représentons l’échelle par une tige I 1I 2 , de longueur L, en contact ponctuel avec le sol horizontal Ox et le mur vertical Oy (Fig. 23.2). On suppose que le contact en I 2 est sans frottement. Les équations vectorielles de la statique s’écrivent, avec des notations explicites : R1 + R 2 + mg = 0

et I1C × mg + I 1I2 × R 2 = 0

En désignant par l la distance I1C, qui sépare I1 du centre de masse C, et a l’angle que fait OC avec l’axe Ox, les équations précédentes, qui traduisent l’équilibre, se projettent dans la base (ex , e y ) selon : X 1 + X2 = 0

Y 1 + mg = 0

et

mgl cos a − LX 2 sin a = 0

On en déduit : Y1 = −mg

X1 = −X2 = −

mgl L tan a

et

|X1| l  tan fs = L tan a |Y 1|

f s étant l’angle de frottement statique échelle-sol. La condition d’immobilité est donc : l 1 L 2 Ces résultats peuvent être retrouvés géométriquement. Les trois forces qui s’exercent sur la barre étant concourantes, la réaction R1 passe par le point K intersection de R 2 et du poids mg. tan a tan f s 

392

23. Statique

I . 3 . — Statique d’un ensemble de plusieurs solides La statique d’un ensemble de plusieurs solides conduit à un système d’équations algébriques. Ce système possède plusieurs inconnues qui sont, d’une part ses degrés de liberté, d’autre part les actions arbitraires telles que les forces et les moments de contact. Si le nombre d’équations est égal au nombre d’inconnues, la résolution du système d’équations linéaires obtenues est possible ; l’ensemble est dit isostatique. Si le nombre d’équations est supérieur au nombre d’équations, la résolution est impossible ; on dit que l’ensemble est hyperstatique. Dans ce cas, on ne progresse qu’en introduisant des équations supplémentaires qui traduisent la déformation des solides réels. En général, on s’arrange pour réaliser l’isostaticité afin de pouvoir évaluer et donc maîtriser les efforts qui s’exercent aux articulations, sans équations supplémentaires. Un exemple typique de système articulé est celui représenté sur la figure 23.3a. Trois tiges AD, DB et CF, de masses négligeables et situées dans un même plan vertical, forment un système à cinq articulations A, B, C, D, E. L’ensemble est soumis à la force verticale extérieure F1 appliquée à l’extrémité F de la tige CF. Cette dernière est articulée au point E : on a EC = a/2 et on pose EF = c. R4

y D 4 F F1

2b

E 5

30◦

D

R5 C 3

1 A

2a

F

C

E

4a B 2

R1

F1 x

a)

R3

A

R2

B

b) F IG . 23.3.

On désigne par R 1 la force qui s’exerce en A sur AD, R2 la force qui s’exerce en B sur DB, R3 la force exercée par FC en C sur DB, R4 la force exercée par DB en D sur AD et R 5 la force exercée par DB en E sur FC. Chacune de ces cinq articulations introduit deux inconnues de contact, d’où dix inconnues au total. Or l’immobilité de chaque tige dans le plan se traduit par trois équations scalaires (deux issues de la somme des forces et une provenant de la somme des moments) ; on a ainsi en tout dix équations. Le calcul des réactions est donc impossible. On peut rendre le système isostatique en imposant à l’une des forces une ligne d’action déterminée ; par exemple, supposons que la réaction R3 soit normale à DB en C. L’immobilité de FC donne les équations : F1 + R5 − R 3 = 0

et EF × F 1 + EC × (−R3 ) = 0

En explicitant, avec des notations cohérentes (Xi est la composante suivant x de la force R i), on obtient : √ 3 R3 a X5 − R3 =0 − F1 + Y5 − =0 cF 1 − R3 = 0 2 2 2 d’où √ 2c c 3 c R3 = F1 X5 = F1 Y 5 = F1 1 + a a a

393

Statique De même, l’immobilité de la tige AD est traduite par les équations vectorielles : R1 + R4 − R 5 = 0 et AD × R 4 + AE × (−R5 ) = 0 Enfin l’immobilité de la tige DB donne : R2 − R4 + R 3 = 0

et BD × (−R 4) + BC × R3 = 0

Il en résulte, en explicitant ces équations vectorielles : X1 + X 4 − X5 = 0 Y 1 + Y4 − Y5 = 0 2bX 4 − bX 5 = 0 et √ R3 3 X2 − X4 + R3 =0 Y 2 − Y4 + =0 2aY4 + 2bX4 − 2aR3 = 0 2 2 On en déduit : √ √ X5 c 3 c 3 c c X4 = F1 = X 1 X2 = − F1 Y4 = −Y2 = F 1 et Y 1 = 1 + F1 = 2 2a 2a 2a 2a Sur la figure 23.3b, on peut vérifier que les forces F1, R 5 et −R3 sont concourantes, ainsi que les forces R1, R4 et −R 5 . Quant aux forces R2, −R 4 et R 3, elles sont parallèles.

II . — STATIQUE DES FILS Les fils sont des courbes matérielles, déformables, de sections très inférieures à leurs longueurs, que l’on utilise souvent pour relier les solides entre eux. Les conditions de la statique de ces systèmes sont celles de chacun de leurs éléments. Il convient donc d’exprimer les forces qui s’exercent sur chaque élément et d’annuler leur somme et leur moment.

 c Dunod – Toute reproduction non autorisée est un délit

II . 1 . — Analyse des forces appliquées à un élément de fil Considérons l’élément d s = MM du fil AB. Il est soumis aux forces de pesanteur rl d s g et aux forces exercées par les parties situées à droite et à gauche de l’élément. Si on néglige leur moment, c’està-dire la rigidité du fil, on peut caractériser ces dernières forces par le vecteur tension du fil T(M , t), tangent au fil, qu’exerce la partie droite du fil sur la partie gauche (Fig. 23.4). La résultante des forces de tension qui s’exercent sur l’élément d s s’écrit : ∂T ds T(s + d s, t) − T(s, t) ≈ ∂s y g A g

B

A +

M ( s) ds M ( s + ds) F IG . 23.4.

B

T α

a O F IG . 23.5.

x

394

23. Statique

II . 2 . — Équation fondamentale de la dynamique des fils Exprimons la loi fondamentale de la dynamique pour l’élément d s, de masse r l d s. On a : rl d s a = rl d s g +

∂T ds ∂s

a=g+

d’où

1 ∂T rl ∂s

II . 3 . — Équation de la statique Le fil étant immobile, on a : a = 0 et ∂ T/∂s = d T/ d s, puisque la variable temps n’intervient pas. Par conséquent : 1 dT =0 rl d s

g+

En introduisant les vecteurs unitaires et , tangent au fil, et e n orienté dans le sens de la concavité, l’équation vectorielle précédente s’écrit, si R désigne le rayon de courbure : g+

1 rl

dT T et + e n = 0 ds R

a) Statique d’un fil pesant homogène Cherchons la forme que prend un fil homogène immobile sous l’action de son seul poids. Avec les notations de la figure 23.5, la relation vectorielle traduisant le repos fournit, suivant les axes horizontal Ox et vertical Oy, les deux équations scalaires suivantes : d (T cos a) = 0 et ds

1 d (T sin a) = 0 rld s

−g+

Il vient, en intégrant : T cos a = C1

T sin a = r l g s + C2

et

tan a =

rl gs + C 2 C1

C 1 et C2 étant deux constantes. On en déduit : s=

C1 tan a − C 2 r lg

et

ds =

C1 d a rl g cos2 a

En outre d x = d s cos a et d y = d s sin a. Par conséquent : x=

cos a d s =

da cos a

C1 r lg

et y =

sin a d s =

C1 r lg

Posant a = C1 /(rl g), on obtient après intégration : x = a ln tan

p a + 4 2

+ x0

et

y=

a + y0 cos a

On élimine a en remarquant que, dans l’équation précédente donnant x, on a : exp

x − x0 a

= tan

1 + tan(a/2) p a + = 4 2 1 − tan(a/2)

d a sin a cos2 a

395

Statique d’où : exp

x − x0 x − x0 + exp − a a

=

1 + tan(a/2) 1 − tan(a/2) 1 + tan 2 a/2 2 + = =2 2 1 − tan(a/2) 1 + tan(a/2) 1 − tan a/2 cos a

On trouve finalement : cosh

x − x0 a

=

y − y0 1 = cos a a

d’où

y − y 0 = a cosh

x − x0 a

C’est l’équation d’une chaînette. Sur la figure 23.5, les constantes x 0 et y 0 ont été prises égales à 0. b) Application à l’enroulement d’un fil sur un solide Considérons un fil, de masse négligeable (r l ≈ 0) en contact avec un solide fixe. Chaque élément est soumis à une force supplémentaire de réaction R l d s (Fig. 23.6).

A

B

en D

C

®e

M ® et R l ds F IG . 23.6.

La condition de l’immobilité d’un élément de fil, en contact avec le solide au point M, s’écrit : 0=

dT d(T e t ) de dT + Rl = + Rl = et + T t + Rl ds ds ds ds

Or d et / d s = en d a/ d s avec a = (AC, et ). Par conséquent, on obtient, en projetant :

 c Dunod – Toute reproduction non autorisée est un délit

dT + R l,t = 0 ds

et

T

da + Rl,n = 0 ds

Comme T d a/ d s > 0, Rl,n < 0 ; la réaction Rl d s qu’exerce le solide sur le fil est bien dirigée vers l’extérieur du solide. Remarque : Si le contact s’effectue sans frottement ou s’il n’y a pas de contact, R l,t = 0 ; la norme de T se conserve alors le long du fil. Étudions la variation de la tension du fil en présence de frottement. Dans l’hypothèse d’un glissement dans le sens ACDB (Rl,t < 0), les équations précédentes donnent : dT = −Rl,t ds

et

T

da = −R l,n ds

Puisque la relation |Rl,t | = m|Rl,n | s’écrit algébriquement : −Rl,t = −mRl,n , il en résulte : dT da = mT ds ds

soit

dT = m da T

396

23. Statique

En intégrant entre 0 et l’angle d’enroulement ae , on trouve : T D = TC exp(mae ) Exemple : Pour une corde enroulée autour d’une bitte d’amarrage en fonte (m = 0, 3) , on a, avec trois tours (ae = 6p), TD ≈ 286 × T C (Fig. 23.6). La force TD à appliquer sur la corde pour la faire glisser est donc bien plus grande que la force TC . Ce résultat est bien connu et largement utilisé dans la pratique portuaire.

III . — MÉTHODE DES TRAVAUX VIRTUELS Cette méthode des travaux virtuels, qui est à l’origine de la formulation lagrangienne de la mécanique (cf. chapitre 24), s’appuie sur la notion de déplacement virtuel. Elle présente l’avantage de ne pas exiger le calcul de toutes les réactions entre les différentes parties des systèmes considérés et donc d’être plus efficace. III . 1 . — Déplacement virtuel élémentaire On appelle déplacement virtuel élémentaire tout déplacement élémentaire arbitraire, à une date fixée, compatible avec les liaisons qui définissent le système considéré. On notera ce déplacement d v r pour le distinguer du déplacement réel élémentaire d r. Remarque : Bien que la notation dr pour le déplacement virtuel soit répandu, nous l’avons évitée, car elle pourrait laisser penser que ce déplacement n’est pas une différentielle au même titre que d r ; en outre le travail élémentaire réel et le travail élémentaire virtuel seraient notés de la même façon : dW, d’où une source de confusion possible. Exemples : i) Sur une tige, le déplacement virtuel d’une masselotte est d v x alors que son déplacement élémentaire réel est d x = v x d t. On note ainsi que d v x est arbitraire et donc indépendant de la vitesse, contrairement à d x (Fig. 23.7a). d vr

y A

dr

y

drr dv r

O

A dx

A

dv x x O a)

θ b)

x

O

θ c)

x

F IG . 23.7.

ii) Dans le plan Oxy, le déplacement virtuel du point A est (cf. Fig. 23.7b) : d v r = dv r er + r dv u eu avec dv r et dvu arbitraires, alors que le déplacement élémentaire réel est : d r = d r e r + r d u eu . Contrairement au déplacement élémentaire réel, pour lequel d r = ˙r d t et d u = u˙ d t, le déplacement élémentaire virtuel n’est pas, a priori, tangent à la trajectoire. iii) Si un point A coulisse sans frottement sur une tige en mouvement (Fig. 23.7c), la liaison dépend du temps. Par conséquent le déplacement virtuel de A, compatible avec cette liaison à l’instant t fixé, ne comporte pas de contribution angulaire, d’où : dv r = d v r er.

397

Statique III . 2 . — Travail virtuel élémentaire

À une date quelconque t, on appelle travail virtuel élémentaire d’une force F, appliquée à un élément d’un système, le travail élémentaire de F dans un déplacement virtuel élémentaire à cette date : dv W = F · dv r Remarque : Attirons l’attention sur le travail non nul de la force de Coriolis dans un déplacement virtuel élémentaire ; en effet, d v r n’est pas a priori tangent à la trajectoire dans le référentiel non galiléen considéré. III . 3 . — Théorème des travaux virtuels relatif à la statique a) Énoncé Un système matériel quelconque S d, déformable ou non, est immobile, si chacun des points A (t) qui le constituent l’est sous l’action de toutes les forces F A qui agissent sur lui. On a donc, dv r étant quelconque : (t) (t) (t) FA = 0 d’où FA · dv rA = 0 et FA · dv r A = 0 A

(t)

en sommant sur tous les points du système. Distinguons dans F A les forces données F (d) des forces de liaison F(l) et limitons-nous au cas particulier important où les liaisons sont parfaites. Rappelons qu’il en est ainsi dans deux cas importants, en l’absence de frottement et en l’absence de glissement même s’il y a frottement. On a donc :

A

F(Al) · d vr A = 0 d’où

A

F (Ad) · d v r = 0

Ainsi, pour qu’un système matériel S d , à liaisons parfaites, initialement immobile, demeure au repos, il faut et il suffit que la somme des travaux virtuels élémentaires des forces données soit nulle.

 c Dunod – Toute reproduction non autorisée est un délit

Remarque : L’attrait de ce théorème est directement lié à la simplicité des relations géométriques entre les paramètres au cours de déplacements virtuels élémentaires. b) Exemples (1) Équilibre bielle-manivelle Proposons-nous de déterminer le couple G z à appliquer suivant l’axe Oz d’une manivelle OM lorsque l’extrémité de la bielle MB est soumise à une force F = −F ex (Fig. 23.8). Évaluons les travaux de ces actions au cours d’un déplacement virtuel de B compatible avec la liaison en ce point : dv W(F) = F · d vr = −F d v x

et dv W (G) = Gz dvu

À partir de la relation l2 = r 2 + x2 − 2xr cos u dans le triangle OMB, on obtient, en différentiant : dx −xr sin u = du x − r cos u

Par conséquent, si la puissance des forces de frottement est nulle, on a : d vW = −F d vx + Gz dv u = F

xr sin u x sin u + Gz dv u = 0 d’où Gz = −Fr x − r cos u x − r cos u

398

23. Statique

Si la bielle est longue devant la manivelle (x  r), alors Gz ≈ −Fr sin u. y Manivelle

M

r

A

Bielle

l θ

l B

x

Oz

F1

C2

l F2

α

O

F

C1

B

x

F IG . 23.8.

F IG . 23.9.

(2) Levier à pression On appelle levier à pression le système de deux tiges articulées identiques dont l’une a une extrémité immobile O et l’autre une extrémité B qui peut se déplacer suivant un axe (Fig. 23.9). On exerce en A une force verticale F1 = −F1 ey dirigée vers le bas. Afin de calculer la force F 2 = −F 2 ex que doit exercer le bâti en B pour que le système soit immobile, considérons un déplacement angulaire virtuel élémentaire dv a. Comme : yA = l sin a

y C1 =

l sin a 2

yC 2 =

l sin a 2

et xB = 2l cos a

il vient : d yA = l cos a d a

d yC 1 =

l cos a d a 2

d y C2 =

l cos a d a 2

et

d x B = −2l sin a d a

On en déduit, puisque F1 · dv rA = −F1 l cos a d va et F 2 · dv rB = 2F 2l sin a dv a : (−F 1l cos a − mgl cos a + 2F 2l sin a) d v a = 0 d’où F 2 =

F1 + mg 2 tan a

On voit aisément l’intérêt du système : pour F1 donné et a faible, F 2 peut être très grand, ce qui permet de comprimer des objets en B. III . 4 . — Équilibre stable d’un système matériel à liaisons parfaites a) Condition d’équilibre Considérons un système matériel S d à liaisons parfaites, caractérisé par une énergie potentielle E p(q) fonction de la seule coordonnée q. D’après le théorème des travaux virtuels, on a, à l’équilibre : A

F · d vr = − d v Ep(q) = 0

Comme cette différentielle s’écrit aussi : d v Ep = (d Ep / d q) dv q, il en résulte que : d Ep =0 dq La nature de l’équilibre est déterminée par l’allure de la fonction Ep(q) au voisinage de la valeur q 0 de q à l’équilibre (Fig. 23.10).

399

Statique

Ωt

Oz g Ep

Ep

l m

l

l O

q1q 0 q2

O

q

a)

q 1 q0 q2

y

z

θ

K

y

m l

2m

q

V

b) F IG . 23.10.

x x

F IG . 23.11.

b) Exemple Le système plan schématisé sur la figure 23.11 est constitué de trois masselottes reliées par des tiges de même longueur l et de masses négligeables. La raideur du ressort est K et sa longueur à vide l0. L’ensemble tourne uniformément à la vitesse V autour de l’axe vertical descendant Ox. Exprimons l’énergie potentielle totale, somme de l’énergie potentielle de pesanteur, de l’énergie potentielle élastique et de l’énergie potentielle centrifuge (cf. chapitre 20) : Ep = −6mgl cos u +

1 V 2ml2 sin2 u 2 K (2l cos u − l0) + 2 × − 2 2

+ Cte

En dérivant par rapport à u, on obtient : d Ep = 2l sin u[3mg + Kl0 − l(2K + mV 2 ) cos u] du

 c Dunod – Toute reproduction non autorisée est un délit

Par conséquent, le système possède deux positions d’équilibre, u1 = 0 et u2 tel que : cos u 2 =

3mg + Kl 0 l(2K + mV2)

Cette seconde position suppose la condition suivante : 3mg + Kl0 < l(2K + mV2)

soit

V  Vc

avec

Vc =

3mg + K (l 0 − 2l) ml

1/2

En calculant la dérivée seconde pour les deux positions d’équilibre, on trouve : d2 Ep d u2

1

= 2l[3mg + K (l0 − 2l) − mlV2 ]

et

d2 E p d u2

= 2l 2(2K + mV2 ) sin2 u 2 > 0 2

La première position est stable pour V < V c, alors que la seconde l’est toujours pourvu qu’elle existe, c’est-à-dire que V < Vc .

400

23. Statique

CONCLUSION Retenons les conditions nécessaires de la statique : (1) pour un corps ponctuel, F = 0, (2) pour un solide, S ex = 0 et Mex = 0, (3) pour un ensemble de s solides, s équations vectorielles doubles telles que les précédentes, (4) pour un système continu déformable, tel qu’un fil simple, une équation vectorielle pour chaque élément. Signalons la méthode des travaux virtuels très adaptée si on ne souhaite pas calculer les actions de liaison. Dans ce contexte, le cas des systèmes matériels soumis à des liaisons parfaites et à des forces qui dérivent d’une énergie potentielle peut être analysé simplement lorsque cette dernière ne dépend que d’un seul paramètre q : il suffit de chercher les solutions qui réalisent la condition d E p/ d q = 0. Suivant le signe positif ou négatif de la dérivée seconde, d2 E p / d q2, l’équilibre est respectivement stable ou instable.

EXERCICES ET PROBLÈMES P23– 1. Échelle contre un mur et sur un sol lisse Une échelle AB, de masse m et de longueur l, est appuyée contre un mur de hauteur h (Fig. 23.12). Les contacts en A et D étant dépourvus de frottement, on maintient l’échelle inclinée d’un angle a par rapport à la verticale, grâce à un fil horizontal OA. Calculer les réactions des supports en A et D ainsi que la tension du fil. B

F

T

A

D h

α

g

C

A θ

l O

A

F IG . 23.12.

O

C h

g

H F IG . 23.13.

30 ◦

D B Mg

E F IG . 23.14.

P23– 2. Tableau accroché à un mur lisse On désire suspendre un tableau, de hauteur h, sur un mur lisse, de telle sorte que le clou d’attache F sur le mur soit au même niveau que le point le plus haut du tableau (Fig. 23.13). 1. Montrer que le point d’attache A à l’arrière du tableau doit être situé à une distance déterminée du point le plus bas du tableau. 2. Évaluer, en fonction de la longueur l du fil et de h, l’angle que fait le tableau avec la verticale. En déduire que l doit être compris entre deux valeurs que l’on exprimera en fonction de h. P23– 3. Efforts aux liaisons d’une console constituée de deux tiges On désire étudier les efforts aux liaisons dans la console représentée sur la figure 23.14. La barre AB, de longueur l, est horizontale et la barre DE, de longueur 4l/3, fait un angle de 30 ◦ avec la verticale. La barre horizontale supporte une charge uniformément répartie de valeur P. On néglige le poids des barres. Calculer les efforts en A, D et E. Vérifier les résultats obtenus à l’aide d’une construction géométrique.

401

Statique P23– 4. Console mobile

Une console mobile ABC, ayant la forme d’un triangle rectangle isocèle (AB = AC = h), est installée sur un tuyau de diamètre 2r, comme le montre la figure 23.15. Le facteur de frottement statique entre la console et le tuyau est m s. On néglige le poids de la console devant la charge Mg placée sur AC. Calculer la distance minimale x à l’axe du tuyau pour laquelle la charge mg peut être supportée sans qu’il y ait glissement de la console le long du tuyau. x A

Mg

A

O C

h

g

x

g



h H B

A

B

B

F IG . 23.15.

F IG . 23.16.

P23– 5. Homme sur une échelle double Une échelle double se compose de deux échelles simples AO et BO de même longueur l, de même poids Mg, articulées sans frottement au sommet commun O (Fig. 23.16). Soit 2a, l’angle au sommet des deux échelles et m le facteur de frottement avec le sol. Un homme H de poids mg monte sur l’échelle AO à une distance x du sommet O. Montrer que si l’angle a augmente, c’est l’échelle BO qui glissera la première. Discuter l’influence de x dans le cas particulier où m = M. P23– 6. Stabilité de l’équilibre d’un disque Un disque D, pouvant tourner sans frottement autour d’un axe horizontal, est soumis à l’action d’un ressort et à celle d’une masselotte A (masse m), suspendue à l’extrémité d’une barre solidaire passant par son axe (Fig. 23.17). Un fil inextensible relie une extrémité du ressort et le point de la barre situé sur le pourtour du disque ; le fil ne glisse pas sur la poulie. Lorsque le ressort est au repos, la barre est verticale et l’angle de rotation u est nul. Étudier la stabilité de l’équilibre du disque.

 c Dunod – Toute reproduction non autorisée est un délit

O l A

y

θ

g

C1

g θ1

r K

C2 θ2

F

x F IG . 23.17.

F IG . 23.18.

P23– 7. Équilibre d’un pendule double pesant (méthode des travaux virtuels) Un pendule pesant double, constitué de deux tiges identiques, de masse m, de longueur l, est écarté de sa position d’équilibre verticale, grâce à une force horizontale F appliquée à l’extrémité de la tige inférieure (Fig. 23.18). Les liaisons aux deux articulations sont parfaites. Trouver les valeurs des angles u1 et u 2 que font, avec la verticale descendante, ces deux tiges à l’équilibre.

402

23. Statique

P23– 8. Équilibre de divers solides géométriques On suspend quatre solides homogènes, un demi-cercle matériel, un demi-disque matériel, une demi-sphère matérielle et une demi-boule matérielle, de masse m et de rayon R . 1. On rappelle que les centres de masse sont situés sur l’axe de symétrie de ces solides à une distance a du centre K du cercle, du disque, de la sphère ou de la boule qui vaut respectivement : a=

2R p

4R 3p

a=

a=

R 2

et a =

3R 8

Dans les quatre cas, calculer la valeur de l’angle a que fait la tangente au solide avec le plan horizontal à l’équilibre. Commenter l’influence de R . 2. On fait osciller chaque solide autour de leur point de suspension. Trouver la période des petites oscillations. Commenter l’influence de R . P23– 9. Équilibre d’un solide en contact avec un plan horizontal Un solide de révolution, constitué d’une base demi-sphérique, de rayon R , qui peut rouler sans glisser sur un plan horizontal, est en équilibre sous l’action de son poids et des forces de contact. 1. Montrer que l’équilibre est stable si la coordonnée verticale z C du centre de masse C du solide est inférieure à R . 2. On considère le solide homogène formé d’une demi-sphère de contact, creuse, surmontée d’un cylindre creux de rayon R et de hauteur h . À quelle condition l’équilibre est-il stable ? P23– 10. Équilibre d’une masselotte et point de Fermat d’un triangle Une masselotte A (masse m ) est maintenue en équilibre par trois fils aux extrémités desquels on a fixé trois masses ponctuelles m1 , m 2 et m 3 . Comme le montre la figure 23.19, ces trois fils passent par trois orifices ponctuels A1 , A 2 , A3 que l’on a aménagés sur une table horizontale. A3

A1

A

O A2

m3 m1

g

x m2 F IG . 23.19.

1. Établir la condition d’équilibre. Trouver la position d’équilibre dans les deux cas suivants. a) L’une des masses ( m 3 ) est très inférieure aux deux autres. b) L’une des masses ( m 3 ) est très supérieure aux deux autres.

Statique

403

2. Si les trois masses sont égales, la position d’équilibre est appelée point de Fermat ou point de Torricelli. a) Comment, d’après ce qui précède, peut-on définir un tel point ? b) Montrer que ce point n’existe que si le triangle A 1A2 A3 n’a pas d’angle supérieur à 2p/3 . 3. a) Quelle est l’énergie potentielle d’un tel système, en fonction des cotes des trois masses, comptées le long d’un axe vertical descendant dont l’origine est prise dans le plan de la table ? b) Que peut-on dire de la somme des distances AA i à l’équilibre ? En déduire une autre définition du point de Fermat.

 c Dunod – Toute reproduction non autorisée est un délit

4. Montrer que le point de Fermat d’un triangle peut être obtenu par l’intersection de trois cercles circonscrits aux triangles équilatéraux A1A 3B2 , A1A2 B 3 et A 2 A3B1 , adossés aux trois côtés du triangle A1A 2A 3 .

24 Lagrangien et hamiltonien Le mouvement d’une particule et plus généralement d’un système matériel, peut être étudié à partir d’un formalisme dit analytique développé au XIXe siècle par le mathématicien italo-français A. Lagrange et le physicien écossais W. Hamilton. Ce formalisme s’appuie principalement sur deux fonctions scalaires ayant la dimension physique d’une énergie : le lagrangien L et l’hamiltonien H. Son intérêt principal est, d’une part qu’il se prête mieux à l’extension de la mécanique aux autres domaines de la physique (optique, physique quantique, etc.), et d’autre part qu’il permet de trouver plus facilement les grandeurs physiques qui se conservent dans le problème considéré.

I . — ÉQUATIONS DE LAGRANGE ET LAGRANGIEN I . 1 . — Espace de configuration Considérons un système matériel dont la position est déterminée par n variables indépendantes, ou degrés de liberté, q1 , q2, . . . , q i , . . . , qn , notés {qi }. L’espace de configuration est l’espace à n dimensions dans lequel la position du système est représentée par un point de coordonnées {q i }. Au cours du temps, ce point décrit une trajectoire dans cet espace. Pour un corps ponctuel évoluant dans l’espace physique habituel, l’espace de configuration s’identifie à cet espace (Fig. 24.1a). En revanche, il n’en est pas de même pour un pendule simple, dont la position est caractérisée par l’angle u qu’il fait avec la verticale descendante (Fig. 24.1b), ou pour un ensemble de trois particules évoluant sur un même axe Ox (Fig. 24.1c). z

z A g

O x

O l

y θ

a)

A

b)

A1

A2

A3

x

c)

F IG . 24.1.

Pour un système constitué de N particules, en mouvement quelconque les unes par rapport aux autres, le nombre de degrés de liberté est n = 3N . L’espace de configuration a donc dans ce cas 3N dimensions.

405

Lagrangien et hamiltonien

Rappelons qu’un point de l’espace de configuration détermine la position du système et non son état mécanique, lequel exige, en outre, la connaissance de l’ensemble des vitesses ou mieux des quantités de mouvement. I . 2 . — Équations de Lagrange Considérons un système matériel en mouvement par rapport à un référentiel galiléen R , sous l’action de forces extérieures et intérieures. Si le système dépend de n degrés de liberté, Lagrange a montré que le mouvement satisfaisait aux n équations éponymes : d dt

∂E k ∂ ˙qi



∂Ek = Qi ∂q i

où Ek({q i , ˙qi}, t) est l’énergie cinétique et Q i le coefficient de dq i , déplacement virtuel (à temps fixé) de qi , dans l’expression du travail élémentaire des forces (cf. chapitre 23). I . 3 . — Lagrangien Si les coefficients Q i peuvent se mettre sous la forme suivante : Qi = −

d dt

∂Fp ∂ q˙i

+

∂Fp ∂qi

F p étant une fonction de {qi , ˙qi} et t , alors les équations de Lagrange deviennent : d dt

∂L ∂ q˙ i



∂L = 0 où L({qi , q˙ i }, t) = Ek ({q i, q˙ i}, t) − F p({q i , q˙ i}, t) ∂q i

est le lagrangien L du système. Pour les systèmes dits naturels, définis par des contraintes ou liaisons indépendantes du temps et des forces qui dérivent d’une énergie potentielle E p(qi ) ne dépendant que des degrés de liberté, Fp se réduit à cette dernière énergie, d’où le lagrangien :

 c Dunod – Toute reproduction non autorisée est un délit

L({q i, q˙ i }) = Ek ({q i , q˙ i }) − Ep ({qi }) Remarque : Cette définition n’est valable qu’en mécanique newtonienne et pour des particules chargées qui ne sont pas en interaction magnétique. En effet, le lagrangien associé au mouvement d’une particule chargée, plongée dans un champ électromagnétique, comporte un terme supplémentaire : L = Ek − E p + qv · A q étant la charge électrique, v la vitesse et A le potentiel vecteur (cf. Électromagnétisme). Exemples (1) Lagrangien d’un oscillateur harmonique unidimensionnel On a (cf. chapitre 10) : 1 1 1 1 Ek = mx˙ 2 Ep = Kx2 d’où L(x˙, x) = mx˙2 − Kx 2 2 2 2 2 x étant l’allongement du ressort de raideur K (Fig. 24.2a).

406

24. Lagrangien et hamiltonien z

A

O

x

O

a)

b) F IG . 24.2.

(2) Lagrangien d’un corps ponctuel en chute libre Dans ce cas simple (cf. chapitre 4) 1 1 Ek = mz˙2 Ep = mgz d’où L( ˙z, z) = m˙z2 − mgz 2 2 z étant la coordonnée suivant la verticale ascendante (Fig. 24.2b). (3) Lagrangien d’un pendule simple On sait que (cf. chapitre 9) : 1 E k = ml 2u˙ 2 2

Ep = −mgl cos u

d’où

L(u˙ , u) =

1 2 2 ml u˙ + mgl cos u 2

u désignant l’angle que fait le pendule avec la verticale descendante (Fig. 24.1b). I . 4 . — Moments conjugués Le moment conjugué p i , associé à la variable qi, est la quantité suivante : pi =

∂L ∂ q˙ i

Pour un corps ponctuel en chute libre (Fig. 24.2b), le moment conjugué est : p z = mz˙ ; c’est donc sa quantité de mouvement, d’où le nom de moment linéaire donné parfois à ce concept. Rappelons que le mot moment vient du mot latin momentum qui signifie mouvement. Pour un pendule simple (Fig. 24.1b), le moment conjugué est : pu = ml2 u˙ ; c’est donc son moment cinétique, d’où le nom de moment angulaire donné parfois à ce concept. I . 5 . — Équations de Lagrange de systèmes soumis à des forces conservatives Les équations de Lagrange d’un système, soumis uniquement à des forces qui dérivent d’une énergie potentielle, sont les n équations différentielles suivantes du mouvement d’un système à n paramètres de configuration : d pi ∂L = dt ∂q i On pourrait démontrer ces équations à partir de la loi fondamentale de la dynamique appliquée à chacun des corpuscules constituant le système, ce qui présente un intérêt limité. La démonstration s’appuie sur les notions de déplacement virtuel et de travail virtuel d’une force (cf. chapitre 23). Dans cette présentation, nous admettons les équations de Lagrange comme nous avons admis la loi fondamentale de la dynamique pour dvelopper la mécanique sous sa forme vectorielle.

407

Lagrangien et hamiltonien Exemples (1) Oscillateur harmonique unidimensionnel Comme L = m ˙x2 /2 − Kx2 /2, il vient p x = m ˙x, d’où (Fig. 24.2a) : d (m˙x) = −Kx dt

K m

x + v20x = 0 avec v 20 = et ¨

(2) Pendule simple ˙ on a (Fig. 24.1b) : Là, puisque L = ml 2u˙ 2/2 + mgl cos u, pu = ml 2u, d (ml 2u˙ ) = −mgl sin u dt

et

¨ u + v20 sin u = 0 avec

v20 =

g l

(3) Pendule pesant Dans ce cas, le lagrangien s’écrit : L = I u˙ 2/2 + mgl cos u, où I désigne le moment d’inertie par rapport à l’axe de rotation, l la distance du centre de masse C à l’axe Oz et −mgl cos u l’nergie potentielle (Fig. 24.3a). Il en résulte que pu = I u˙ s’identifie au moment cinétique, d’où : d ˙ (I u) = −mgl sin u dt

soit

¨ u + v20 sin u = 0 avec

v20 =

mgl I

K1

A2

K2

Oz l g

C

A1

K12

θ x1 x

a)

x2 b)

F IG . 24.3.

 c Dunod – Toute reproduction non autorisée est un délit

(4) Oscillateurs élastiques en interaction L’énergie cinétique des deux pendules élastique horizontaux a pour expression (Fig. 24.3b) : 1 1 Ek = m1 ˙x12 + m2x˙ 22 2 2 x1 et x 2 étant les coordonnées des masselottes par rapport à leurs positions de repos. Quant à l’énergie potentielle d’origine élastique, elle s’écrit : Ep(x1, x2) =

1 1 1 K1x12 + K2 x22 + K 12 (x2 − x1 ) 2 2 2 2

d’où le lagrangien : 1 1 1 1 1 m1 ˙x21 + m 2x˙22 − K1x21 − K2 x22 − K 12(x2 − x1)2 2 2 2 2 2 Il en résulte les deux moments conjugués et les deux équations de Lagrange correspondantes : ∂L ∂L ∂L ∂L p1 = = m1 ˙x1 p2 = = m2 x˙ 2 et p˙ 1 = p˙ 2 = ∂x1 ∂x2 ∂ x˙1 ∂x˙ 2 soit, en explicitant les deux dernières équations : L=

m1¨x1 = −K1 x1 + K12 (x2 − x1)

et m2¨x2 = −K 2 x2 + K12(x 1 − x2)

Le mouvement de ces oscillateurs élastiques couplés sera analysé en détail ultérieurement (cf. chapitre 27).

408

24. Lagrangien et hamiltonien

I . 6 . — Principe d’Hamilton Les n équations de Lagrange auxquelles satisfait un système matériel conservatif à n degrés de liberté peuvent être formulées de façon variationnelle, comme l’est le principe de Fermat en optique géométrique (cf. Optique). Cette formulation est connue en mécanique sous le nom de principe d’Hamilton. Énoncé Entre deux instants t 1 et t 2, le mouvement d’un système matériel est celui qui réalise une valeur stationnaire de l’action S : dS = 0 avec S =

t2

t1

L(qi, q˙ i, t) d t

Le mot stationnaire a la même signification que dans le principe de Fermat : lorsqu’on calcule l’intégrale S le long de plusieurs chemins voisins pour lesquels les instants t1 et t 2 sont fixés, la trajectoire effectivement suivi par le systme est celle qui réalise soit un minimum de S soit un maximum. Comme l’action S est minimale pour toute portion suffisamment petite de la trajectoire, ce principe est aussi connu sous le nom de principe de moindre action ou principe de Maupertuis. L’avantage de cette présentation, qui est équivalente aux précédentes vectorielle ou lagangienne, est précisément d’établir des liens étroits entre les différentes disciplines de la physique, ici entre la mécanique newtonienne et l’optique géométrique (cf. Optique).

II . — HAMILTONIEN ET ÉQUATIONS CANONIQUES II . 1 . — Espace des phases Dans la théorie des équations de Lagrange, les variables étaient les n degrés de liberté ou coordonnées du point représentatif du système matériel dans l’espace de configuration. Les n équations différentielles du deuxième ordre relatives à ces variables formaient alors les équations du mouvement. Dans la théorie d’Hamilton, on s’intéresse plutôt à l’état mécanique du système, lequel dépend de 2n variables indépendantes : les n coordonnées {q i} de l’espace de configuration et les n moments conjugués { pi = ∂L/∂ q˙ i }. On appelle espace des phases l’espace à 2n dimensions dans lequel un point figuratif représente l’état mécanique du système. Ce nombre de variables coïncide évidemment avec le nombre des conditions initiales qui caractérisent l’état initial du système : généralement n paramètres de position et n paramètres de quantité de mouvement. Exemples : (1) Oscillateur harmonique Là, q 1 = x et p1 = m˙x (Fig. 24.2a). Ces variables q1 et p 1 = mx˙ sont reliées par l’équation de conservation de l’énergie mécanique : p2 Kx2 p2x Kx2 Em = x + + = Cte soit =1 2m 2 2mE m 2Em Dans l’espace des phases, la trajectoire décrite par le point représentatif de l’état mécanique est donc une ellipse dont les demi-axes (2mEm )1/2 et (2Em/K) 1/2 , qui dépendent de l’énergie, représentent les valeurs maximales xm et pm de l’élongation et de la quantité de mouvement. Il est commode de récrire l’équation précédente en faisant apparaître ces quantités px pm

2

+

x xm

2

=1

avec p m = (2mE m) 1/2

et xm =

2Em K

1/2

409

Lagrangien et hamiltonien

px

pz

E m > Em

pm Em 0

h

xm

0

x

z

b)

a) F IG . 24.4.

(2) Chute libre Les variables q 1 = z et p 1 = m ˙z sont reliées par l’équation de conservation de l’énergie mécanique (Fig. 24.4b) : p2 E m = z + mgz = Cte = mgh 2m si initialement pz = 0 et z = h. Ainsi, dans l’espace des phases, la courbe décrite par le point représentant l’état mécanique du système est la portion de parabole pour laquelle p z < 0. Notons que la branche positive de la parabole ( pz > 0) correspond à la trajectoire décrite réversiblement, c’est-à-dire en changeant le signe du paramètre d’évolution : le corps ponctuel remonte l’axe vertical à partir de sa position la plus basse avec une vitesse initiale non nulle jusqu’à la hauteur h où sa vitesse est nulle (Fig. 24.4b). II . 2 . — Hamiltonien a) Définition

 c Dunod – Toute reproduction non autorisée est un délit

Pour un système matériel en mouvement par rapport à un référentiel galiléen R, sous l’action de forces extérieures et intérieures qui dérivent d’une énergie potentielle E p({q i}, t), l’hamiltonien est la fonction suivante, dépendant des paramètres de position {qi } , des moments conjugués {p i } et éventuellement du temps : H=

i

pi q˙ i − L

Cette transformation est connue sous le nom de transformation de Legendre. Elle permet de passer du lagrangien, fonction associée aux variables ({qi }, {q˙ i }, t), à l’hamiltonien, fonction d’état des variables ({qi }, {pi}, t). En effet : ∂L ∂L ∂L dL = d qi + d q˙ i + dt ˙ ∂q q ∂t ∂ i i i et dH = d

i

pi q˙i − d L =

i

(pi d ˙qi + q˙ i d pi ) −

i

∂L ∂L ∂L d qi + d q˙ i − dt ∂ q˙ i ∂qi ∂t

soit, puisque p i = ∂L/∂ ˙qi et ˙pi = ∂L/∂qi : dH =

i

(˙qi d p i − p˙ i d qi ) −

∂L dt ∂t

410

24. Lagrangien et hamiltonien

b) Équations canoniques d’Hamilton Évaluons, de deux façons, la différentielle de la fonction d’Hamilton. En considérant H comme une fonction des 2n variables ({qi , pi }) et du temps t, on a : ∂H ∂H ∂H dH = d qi + d pi + dt ∂qi ∂pi ∂t i Si l’on identifie H à l’expression établie précédemment : dH = on en déduit :

i

∂H d qi = ∂pi dt

( q˙ i d pi − p˙ i d q i ) − ∂H d pi =− ∂qi dt

∂L dt ∂t

et

∂H ∂L =− ∂t ∂t

Les 2n premières équations du premier ordre en q i et pi sont les équations canoniques du mouvement. On voit, en outre, que si le lagrangien dépend explicitement du temps, il en est de même pour l’hamiltonien. c) Cas des systèmes naturels Un système est dit naturel si l’énergie potentielle et les contraintes qui lui sont imposées ne dépendent pas explicitement du temps. Il en résulte que les coordonnées d’un point A quelconque du système ne sont fonctions que des paramètres de configuration {q i} , lesquels sont indépendants les uns des autres : OA({qi }). On en déduit l’expression de la vitesse de A : d OA ∂OA d q i ∂OA vA = q˙ i = = dt dt ∂q i ∂qi i i et la forme quadratique suivante de l’énergie cinétique : ∂OA 1 m Av 2A = Ek = · ∂q 2 i i A A j en posant : ∂OA aij = · ∂q i A

∂OA ∂qj

q˙ i ˙qj =

aij q˙i q˙ j i

j

∂OA ∂qj

L’énergie cinétique ne dépend donc pas explicitement du temps. Comme en outre E p = Ep({qi }), il vient : ∂L ∂E k = = ˙ i q˙ j = 2E k 2a ijq˙ j et pi = pi q˙i = 2 aij q ∂ q˙i ∂ ˙qi j i i j d’où :

H=

i

pi q˙ i − L = Ek + E p = E

L’hamiltonien d’un système naturel s’identifie à son énergie. Exemples 1) Pour un oscillateur harmonique élastique à une dimension : 1 1 L = m ˙x2 − Kx 2 d’où px = m x˙ 2 2 Il en résulte l’équation bien connue du mouvement :

et H = pxx˙ − L =

∂H d px =− = −Kx dt ∂x

p2x 1 + Kx2 2m 2

411

Lagrangien et hamiltonien

2) Pour une particule se déplaçant dans un plan sous l’action d’une force centrale qui dérive d’une énergie potentielle Ep(r). On a, avec les notations habituelles : 1 L = m(r˙2 + r2u˙ 2) − E p(r) 2 On en déduit l’hamiltonien :

d’où

1 H= 2m

pr =

∂L = m ˙r ∂ ˙r

p2u r2

+ E p (r)

p2r +

et pu =

∂L = mr2 u˙ ∂u˙

et les équations du mouvement : pr pu p 2u ∂E p p p u˙ = ˙ ˙ = − + = 0 r u m mr2 ∂r mr3 On retrouve la conservation du moment cinétique (pu = Cte) et l’équation du mouvement radial. r˙ =

II . 3 . — Conservation de l’énergie et des moments conjugués a) Conservation de l’énergie Pour un système matériel naturel, en mouvement par rapport à un référentiel galiléen, l’hamiltonien H ne dépend pas explicitement du temps. On en déduit, en tenant compte des équations canoniques : ∂H d qi ∂H d p i ∂H ∂L dH = + + =− = 0 d’où H = Cte dt ∂qi d t ∂pi d t ∂t ∂t i L’invariance par rapport au temps de l’hamiltonien implique la conservation de l’hamiltonien et donc de l’énergie si le système est naturel. On dit que, pour de tels systèmes, le temps est homogène b) Conservation des moments conjugués

 c Dunod – Toute reproduction non autorisée est un délit

Si l’une des variables q i de l’espace de configuration n’apparaît pas explicitement dans l’hamiltonien, le lagrangien ou l’énergie potentielle, le moment conjugué p i associé est une constante du mouvement. En effet, il vient, d’après les équations de Lagrange : p i = Cte Ainsi, à une variable cachée de l’espace de configuration correspond une constante du mouvement. Dans l’exemple de la particule en mouvement sous l’action d’une force centrale, l’énergie potentielle Ep (r) ne dépend pas de la variable angulaire u ; le moment conjugué p u, qui s’identifie au moment cinétique de la particule, est donc une constante. Plus généralement, si l’hamiltonien est invariant par rapport aux trois coordonnées cartésiennes x, y, z du centre de masse C, c’est-à-dire par translation, les moments conjugués p x, p y , pz sont des constantes. L’invariance de l’hamiltonien par translation implique la conservation de la quantité de mouvement du système. On dit que l’espace est homogène. De façon analogue, si l’hamiltonien est invariant par rapport aux trois coordonnées angulaires dans le mouvement autour du centre de masse, c’est-à-dire par rotation, les trois moments conjugués L1 , L2 , L 3 sont des constantes. L’invariance de l’hamiltonien par rotation implique la conservation du moment cinétique du système. On dit que l’espace est isotrope.

412

24. Lagrangien et hamiltonien

III . — LAGRANGIEN ET HAMILTONIEN D’UNE PARTICULE CHARGÉE III . 1 . — Lagrangien d’une particule chargée Dans le cas important en physique d’une particule chargée plongée dans un champ électromagnétique (E, B), montrons que l’expression suivante du lagrangien : L = E k − qV + qv · A dans laquelle q est la charge et (V, A) désigne le potentiel électromagnétique, est compatible avec la loi fondamentale de la mécanique et avec l’expression de la force de Lorentz (cf. Électromagnétisme) : dp = q(E + v × B) dt Suivant la coordonnée x, le moment conjugué Px s’écrit : ∂L = m˙x + qAx = px + qA x = P x ∂˙x Ainsi le moment conjugué P d’une particule chargée dans un champ électromagnétique ne s’identifie pas à sa quantité de mouvement p = mv : P = p + qA L’équation de Lagrange associée à x donne : d Px ∂L = dt ∂x

d px d Ax ∂V ∂A x ∂Ay ∂Az +q = −q + q vx + vy + vz dt dt ∂x ∂x ∂x ∂x

soit

Comme A est une fonction de (x, y, z, t), on a aussi : ∂Ax ∂Ax d x ∂Ax d y ∂Ax d z ∂Ax ∂Ax ∂Ax ∂Ax d Ax = + + + = + vx + vy + vz ∂t ∂x d t ∂y d t ∂z d t ∂t ∂x ∂y ∂z dt Il en résulte que : d px ∂V ∂Ax ∂Ay ∂Ax = −q −q + qvy − dt ∂x ∂t ∂x ∂y

+ qvx

∂Az ∂Ax − ∂x ∂z

La relation entre le champ et le potentiel électromagnétiques étant (cf. Électromagnétisme) : E = − grad V −

∂A ∂t

et

B = rot A

les composantes du champ électromagnétique sont les suivantes : −

∂V ∂A x − ∂x ∂t



∂V ∂A y − ∂y ∂t



∂V ∂A z − ∂z ∂t

∂Az ∂A y − ∂y ∂z et

∂Ax ∂A z − ∂z ∂x ∂Ay ∂A x − ∂x ∂y

413

Lagrangien et hamiltonien

La loi fondamentale de la dynamique d p/ d t = q(E + v × B) donne alors, comme première composante : ∂V ∂Ax ∂Ay ∂Ax ∂Ax ∂Az d px = −q −q + qvy − qv z − − dt ∂x ∂t ∂x ∂y ∂z ∂x On retrouve bien le même résultat pour d p x/ d t que précédemment. Retenons donc l’expression suivante du lagrangien d’une particule chargée, en mécanique de Newton : L=

1 mv 2 − qV + qv · A 2

III . 2 . — Hamiltonien d’une particule chargée Procédons à la transformation de Legendre suivante : H = v · P − L. Il vient : H = v · (p + qA) − E k + qV − qv · A = v · p − Ek + qV = E k + qV

puisque

v · p = 2Ek

Ainsi, l’hamiltonien d’une particule chargée s’identifie à son énergie totale : H=

(P − qA)2 p2 + qV = + qV 2m 2m

Remarque : En relativité restreinte, le lagrangien et l’hamiltonien ont pour expressions respectives (cf. Relativité et invariance) : L=

mc2 − qV + qV · A et H = gmc2 + qV g

IV . — DÉTERMINISME, IMPRÉDICTIBILITÉ ET CHAOS

 c Dunod – Toute reproduction non autorisée est un délit

IV . 1 . — Systèmes intégrables On dit qu’un système dissipatif ou non est intégrable si le nombre de degrés de liberté est égal au nombre de constantes du mouvement. Par exemple, la chute des corps dans le vide est intégrable en raison de la conservation de l’énergie (cf. chapitre 4), le mouvement parfait d’un solide autour d’un axe aussi car l’énergie se conserve (cf. chapitre 25). Le mouvement de Kepler dans le problème à deux corps, à deux degrés de liberté dans le référentiel du centre de masse, est intégrable, lui aussi, car l’énergie mécanique se conserve ainsi que le moment cinétique selon l’axe perpendiculaire au plan du mouvement (cf. chapitre 13). Quant au mouvement de la toupie symétrique, il est intégrable, car l’énergie mécanique se conserve, le moment cinétique selon l’axe vertical Oz et le moment cinétique selon l’axe Oz  de la toupie (cf. chapitre 26). Dans certains cas, comme la Machine d’Atwood Dansante (MAD), (cf. chapitre 20), le système n’est a priori pas intégrable, car le système a deux degrés de liberté et une seule constante d’intégration ; cependant, si les poulies sont de rayons et de masses nuls et le rapport des deux masses égal à trois, le système devient intégrable, ce qui correspond à l’émergence d’une symétrie cachée non encore identifiée.

414

24. Lagrangien et hamiltonien

IV . 2 . — Déterminisme a) Définition On dit qu’un système mécanique est déterministe si la connaissance, à un instant, des forces qui agissent sur lui et de son état, permet de déterminer son état à tout autre instant. L’état donné est le plus souvent l’état initial, caractérisé par les valeurs initiales des degrés de liberté et des moments conjugués, mais ce n’est pas nécessaire ; la connaissance de l’état final permettrait de restituer les états antérieurs. Aussi peut-on dire que le déterminisme implique que tout soit connu : le passé, le présent et le futur. Le déterminisme rigoureux, qui donnerait à la science un caractère totalement prédictif, fut remarqué pour la première fois par P. Laplace, en 1814, dans les termes suivants : « Nous devons envisager l’état présent de l’Univers comme l’effet de son état antérieur, et comme la cause de celui qui va suivre. Une intelligence qui, pour un instant donné, connaîtrait toutes les forces dont la nature est animée et la situation respective des êtres qui la composent, si d’ailleurs elle était assez vaste pour soumettre ces données à l’analyse, embrasserait dans la même formule les mouvements des plus grands corps de l’Univers et ceux du plus léger atome : rien ne serait incertain pour elle, et l’avenir, comme le passé, serait présent à ses yeux. » Il n’est pas inutile de souligner les hypothèses restrictives (en italique), nécessaires au déterminisme, rappelées par l’auteur lui-même. b) Exemple Considérons un pendule simple évoluant, sans frottement, dans le champ de pesanteur (Fig. 24.1b). L’hamiltonien, qui s’identifie dans ce cas à l’énergie mécanique, s’écrit : H=

p2u − mgl cos u = E m d’où 2ml2

p u = [2ml2 (Em + mgl cos u)]1/2

On sait que deux cas peuvent se produire (cf. chapitre 9) : i) −mgl < E m < mgl Le pendule oscille entre les valeurs opposées, u 1 et −u1 , telles que u1 = arccos(−Em /mgl), ce qui traduit l’annulation de pu. ii) E m  mgl Le moment conjugué p u est une fonction périodique de u, lequel ne s’annule jamais. Le mouvement du pendule est révolutif. Dans l’espace des phases, la trajectoire du point représentatif du système est celle représentée sur la figure 24.5. On y distingue aisément deux zones : l’une pour laquelle le mouvement est oscillatoire, l’autre pour laquelle le mouvement est révolutif. On voit que les conditions initiales permettent de déterminer sans ambiguïté le type de mouvement du pendule dans l’espace des phases : mouvement oscillatoire ou mouvement révolutif. IV . 3 . — Imprédictibilité et chaos a) Sensibilité aux conditions initiales Pour la plupart des systèmes dynamiques intéressants en physique, le nombre de constantes du mouvement est inférieur au nombre de degrés de liberté. Cela se traduit, dans l’espace des phases, par une complexité des courbes obtenues ; on constate une imbrication des zones correspondant aux différents types de mouvement.

415

Lagrangien et hamiltonien . ml 2θ

p

Em > mgl

Δp

Em = mgl -π

O

π

θ

Δq 0

q

Em < mgl F IG . 24.5.

F IG . 24.6.

Par exemple, considérons le système constitué par une pièce de monnaie lancée en l’air et terminant sa chute sur une table. Cette pièce est assimilable à un solide ayant la forme d’un cylindre plat sur les faces duquel sont inscrits des signes distincts, pile et face. Son mouvement a six degrés de liberté et quatre constantes du mouvement : l’énergie mécanique, somme de l’énergie cinétique et de l’énergie potentielle de pesanteur, et les trois composantes du moment cinétique au centre de masse (cf. chapitre 26) ; dans l’espace des phases, les points représentatifs des deux types de mouvement pile ou face, au moment où la pièce atteint la table, ne se trouvent pas dans des zones bien séparées ; au contraire, ils semblent disposés au hasard. On comprend alors qu’une légère imprécision sur la donnée des conditions initiales ait une grande influence sur le résultat obtenu au point de rendre l’analyse imprédictive : bien que le système soit déterministe, des conditions initiales très proches peuvent donner des résultats qualitativement très différents. Sur la figure 24.6, on a représenté à l’aide d’une croix et d’un point, dans l’espace des phases, deux types de mouvements possibles de ce système dynamique. Pour des conditions initiales connues même avec une excellente précision, il est impossible de prévoir la nature du mouvement, car les points représentant les deux types de mouvements possibles se trouvent à l’intérieur du domaine d’incertitude Dq Dp. Il apparaît donc dans ce type de mouvement une imprédictibilité expérimentale qui nécessite, comme le disait déjà H. Poincaré en 1892, des méthodes nouvelles d’analyse de nature qualitative.

 c Dunod – Toute reproduction non autorisée est un délit

De tels systèmes, déterministes mais imprédictifs car sensibles aux conditions initiales, sont dits chaotiques. b) Sources de chaos De façon générale, les sources de chaos sont attribuées aux termes non linéaires dans les équations qui régissent le mouvement. Ces non-linéarités sont, par exemple, le frottement visqueux proportionnel au carré de la vitesse, les forces associées au développement à l’ordre trois de l’énergie potentielle autour d’une position d’équilibre, les forces d’inertie d’entraînement et de Coriolis, l’accélération convective d’un fluide et les forces de gravitation en 1/r2, lesquelles sont à l’origine de l’instabilité des systèmes en Astrophysique. Dans l’espace des phases, ces non-linéarités provoquent des modifications des courbes d’évolution des systèmes linéaires correspondants (cf. chapitre 10) ; ces courbes présentent des points de convergence nouveaux appelés attracteurs étranges. Depuis quelques années, l’étude du comportement chaotique des systèmes dynamiques complexes est en plein essor, au point d’être à l’origine d’un renouveau de la mécanique. Ce développement, initié par Poincaré au début du XXe siècle, doit être attribué en partie aux grandes possibilités offertes par la simulation numérique sur ordinateur. Enrichi par l’analyse thermodynamique du rôle de la création

416

24. Lagrangien et hamiltonien

d’entropie dans de tels systèmes, il peut déboucher sur une nouvelle interprétation des comportements de systèmes aussi complexes que les êtres vivants (cf. Thermodynamique). Remarque : En électronique, les composants actifs, tels que l’amplificateur opérationnel avec sa source auxiliaire d’énergie, introduisent aussi des termes non linéaires dans les équations des circuits. De même en optique, les lasers sont des composants non linéaires qui peuvent par irradiation de matériaux provoquer des comportements chaotiques (cf. Optique).

CONCLUSION Les équations du mouvement des systèmes naturels, à n degrés de liberté {q i }, dont les forces dérivent d’une énergie potentielle, peuvent être obtenues par des méthodes analytiques. (1) La première est la méthode des n équations de Lagrange où les variables sont les n degrés de liberté {qi }. Si : L = E k − Ep est le lagrangien, fonction d’état du système des variables {qi }, { q˙i } et du temps t, on a : ∂L d pi = dt ∂qi



pi =

∂L ∂ q˙ i

est le moment conjugué de la variable qi . (2) La deuxième est la méthode des 2n équations canoniques de Hamilton où les variables sont le temps t, les n degrés de liberté {qi } et les n moments conjugués {p i }. Si : H=

i

pi q˙ i − L

est l’hamiltonien, fonction d’état des variables {qi }, {pi } et du temps t, on a : ∂H d qi = dt ∂pi

et

∂H d pi =− dt ∂q i

Dans les systèmes naturels, les plus importants en physique, l’hamiltonien du système s’identifie à son énergie. La relation entre les lois de conservation et les propriétés d’invariance du système doit être soulignée. i) L’énergie se conserve si le temps n’apparaît pas explicitement dans l’expression de l’hamiltonien : le temps est homogène. ii) Le moment conjugué associé à une variable spatiale se conserve si cette dernière n’apparaît pas explicitement dans l’expression de l’hamiltonien ; s’il en est ainsi pour les trois coordonnées cartésiennes, l’espace est uniforme. iii) Le moment conjugué associé à une variable angulaire se conserve si cette dernière n’apparaît pas explicitement dans l’expression de l’hamiltonien ; s’il en est ainsi pour les trois coordonnées angulaires de l’espace physique, l’espace est dit isotrope. Enfin, soulignons la différence entre le déterminisme des lois et l’imprédictibilité du comportement des systèmes complexes. Une grande sensibilité aux conditions initiales, du fait de la présence de fortes non-linéarités dans les équations du mouvement, rend ces systèmes imprédictibles voire chaotiques.

417

Lagrangien et hamiltonien

EXERCICES ET PROBLÈMES P24– 1. Pendule elliptique constitué de deux masselottes Un solide est constitué d’une tige, homogène, de longueur l, de masse m, aux extrémités de laquelle sont fixées deux masselottes A1 et A2 , de masses respectives m 1 et m2 (Fig. 24.7). L’extrémité A1 glisse sans frottement le long de l’axe horizontal Ox d’un référentiel terrestre R = Oxyz et la tige évolue dans le plan vertical Oxy. 1. Exprimer, en fonction de x = OA 1 et u = (−Oy, A1 A2 ), l’énergie cinétique, l’énergie potentielle, le lagrangien et les moments conjugués px et pu . 2. Trouver les équations différentielles du mouvement. 3. En déduire la nature des petits mouvements, sachant qu’à l’instant initial, x = 0, u = u 0, ˙x = 0 ˙ et u = 0.

y O g

x

g

O

A1

O

x

A1 x

g

A3

l

l θ

θ

A2

F IG . 24.7.

x

O1

F IG . 24.8.

A

2m

m K

A2 m F IG . 24.9.

P24– 2. Pendule simple excité par déplacement horizontal L’extrémité O 1 d’un pendule simple, de longueur l et de masse m, a un mouvement oscillatoire horizontal de la forme : x1(t) = dm cos(vt) (Fig. 24.8).

 c Dunod – Toute reproduction non autorisée est un délit

1. Établir l’équation différentielle du mouvement du pendule. On posera v 20 = g/l. 2. Étudier le cas des petits mouvements en u. P24– 3. Oscillateur sur une poulie On considère le système de la figure 24.9 constitué de trois masselottes A 1 , A 2 et A3 , de masses respectives m, m, 2m, reliées par un ressort de raideur K et par un fil inextensible. Les paramètres de configuration sont la position x de A 1, suivant la verticale descendante, et l’allongement y du ressort. La poulie, de rayon r, a une masse négligeable. 1. Donner les expressions des moments conjugués. 2. Trouver l’hamiltonien du système. 3. En déduire les équations différentielles et la nature du mouvement, sachant qu’initialement x = 0 , y = 2mg/K , x˙ = 0 et ˙y = 0 . On posera v 21 = 4K/(3m).

418

24. Lagrangien et hamiltonien

P24– 4. Petites oscillations d’un cerceau lesté 1. Retrouver, à l’aide des équations de Lagrange, l’équation différentielle des petits mouvements du cerceau lesté représenté sur la figure 18.12. 2. Même question en utilisant les équations canoniques. P24– 5. Équations canoniques du problème de Kepler Un corpuscule, de masse m, est soumis à la force dérivant de l’énergie potentielle E p = K /r.

1. Trouver l’expression de l’hamiltonien en fonction des variables (r, u) et des moments conjugués (pr, p u ). 2. En déduire les équations du mouvement. P24– 6. Pendule double À l’extrémité A d’un pendule simple OA (masse m, longueur l), on accroche un deuxième pendule simple AB identique au premier (Fig. 24.10). On désigne par u1 et u2 les angles que font avec la verticale descendante respectivement OA et AB. 1. Établir les équations différentielles du mouvement. 2. Que deviennent ces équations dans le cas des petits angles ?

O l g

θ1

A1

A2

l θ2

x1

x2

A

g

A2 x2

A3 x3

B a)

F IG . 24.10.

A4 x x 1 4 b)

F IG . 24.11.

P24– 7. Machines d’Atwood simple et double 1. Une machine d’Atwood simple est constituée de deux masselottes A 1 et A2 , de masses m1 et m2, reliées par un fil inextensible (Fig. 24.11a). Établir l’équation différentielle et la nature du mouvement, sachant que la masse de la poulie est négligeable. 2. On remplace la masselotte A 1 par une machine d’Atwood comme le montre la figure 24.11b, les masses des poulies étant négligeables. Trouver les équations différentielles du mouvement. P24– 8. Modélisation de la molécule de dioxyde de carbone On représente les trois atomes de la molécule symétrique de dioxyde de carbone par trois points matériels alignés, de masses respectives m1, m2 et m 1. Ces points se déplacent sous l’action des forces interatomiques que l’on peut représenter par deux ressorts identiques de raideur K. On désigne par x1, x2, x3 les coordonnées de position par rapport aux positions d’équilibre, dans le référentiel du centre de masse R∗ .

419

Lagrangien et hamiltonien 1. Établir les équations différentielles du mouvement à l’aide du lagrangien du système.

2. Quelle équation relie les coordonnées x1 , x2 , x3 , sachant que les atomes de la molécule peuvent être simultanément au repos ? 3. En déduire les deux équations différentielles du mouvement. P24– 9. Système de poulies On réalise le système de poulies identiques et de masselottes représenté sur la figure 24.12. Les centres des poulies P1 et P2 sont fixes, alors que le centre de la poulie P3 est mobile. Les poulies, toutes de rayon r, ont une masse négligeable et les masses suspendues sont m 1, m 2 et m 3 . En outre le fil est inextensible. 1. Exprimer le lagrangien du système en fonction des coordonnées x 1 et x2 des masselottes A1 et A2 comptées à partir du bâti reliant les centres des poulies 1 et 2. 2. En déduire la valeur des accélérations de A 1 , A 2 et A 3. S B P3

P1

g

z z g θ

P2

A2

O

A1

Ω

y

Moteur

A3 F IG . 24.12.

F IG . 24.13.

 c Dunod – Toute reproduction non autorisée est un délit

P24– 10. Lagrangien et énergie d’un système articulé en rotation uniforme Un système déformable est constitué de quatre tiges identiques (masse m, longueur l) formant un losange articulé (Fig. 24.13). Il est astreint à tourner autour d’un axe vertical ascendant Oz grâce à un moteur qui lui impose une vitesse angulaire de rotation constante V . L’extrémité supérieure B du losange est reliée à un ressort de raideur K = mg/l et de longueur à vide l 0 = l/2 ; g est l’intensité du champ de pesanteur. L’autre extrémité du ressort est fixée au point F de l’axe Oz à la distance L = 9l/2 de O. 1. Montrer que l’énergie cinétique du système dans le référentiel tournant R  = Ox y z a pour expression : Ek = Ju˙ 2 (1 + 3 sin2 u)/2, u étant l’angle que font les tiges avec l’axe vertical et J une quantité que l’on exprimera en fonction de m et l. 2. L’énergie potentielle du système dans R  se met sous la forme : E p = 4mgl 1 − cos u −

sin 2 u 2

1+

V2 V 20

Calculer V2 en fonction de g et l ; en déduire le lagrangien et l’énergie du système dans R .

420

24. Lagrangien et hamiltonien

P24– 11. Bille creuse dans un guide diédrique Retrouver, à l’aide des équations de Lagrange, l’équation différentielle du mouvement d’une bille creuse dans un guide diédrique en rotation uniforme (Fig. 20.23). P24– 12. Invariant d’un pendule simple Un pendule simple, constitué d’une masselotte A (masse m = 150 g ), accrochée à l’extrémité d’un fil (longueur OA = l = 25 cm ), oscille dans un plan vertical autour de sa position d’équilibre, en s’écartant peu de cette dernière. 1. Exprimer, en fonction de l’angle w , que fait la direction du pendule avec la verticale descendante Ox et de sa dérivée w˙ , l’énergie cinétique E k et l’énergie potentielle de pesanteur E p , l’origine de cette dernière étant prise à la position d’équilibre. 2. Quelle est la constante du mouvement du pendule en l’absence de frottement ? Calculer la pulsation v0 , la fréquence f 0 et la période T0 du mouvement oscillatoire. 3. a) Quelle est l’expression du moment cinétique de A en O ? b) Montrer que la trajectoire du pendule dans l’espace des phases (w, L z ) est une ellipse dont on exprimera les axes a et b en fonction de l’énergie mécanique Em du pendule et des caractéristiques de ce dernier. c) Exprimer l’aire de l’ellipse en fonction de E m et T0 . 4. On tire sur le fil en faisant varier lentement sa longueur (Fig. 24.14), de sorte que d l/ d t  l/T 0 .

a) Exprimer la variation élémentaire d’énergie lorsque l varie de d l . En déduire qu’en moyenne, on a : d Em d T0 + =0 Em T0

b) L’énergie et la période ne se conservent pas si on tire lentement sur le fil, mais une autre grandeur se conserve. Laquelle ? Comparer cet invariant à l’aire de l’ellipse. Quelle est la dimension physique de cet invariant ? c) Quel est le rapport entre les amplitudes du mouvement oscillant lorsque la longueur du fil passe progressivement de li = 25 cm à l f = 12, 5 cm ? d) Cet invariant est qualifié d’adiabatique, ce qui signifie, en thermodynamique, sans échange thermique avec le milieu extérieur. Commenter.

O g

l w

A

F IG . 24.14. — Invariant d’un pendule simple

25 Mouvement d’un solide autour d’un axe fixe Le mouvement d’un solide autour d’un axe fixe est l’un des mouvements les plus importants dans la pratique, car on l’observe partout : dans les machines outils des ateliers, dans les véhicules, dans les appareils de mesures, etc. Le pendule pesant en est aussi un exemple qui est intéressant à la fois sur les plans pratique et pédagogique. Cependant, la rotation est à l’origine d’un inconvénient majeur : des réactions très intenses sur les paliers de fixation peuvent prendre naissance et produire des contraintes mécaniques qui provoquent l’usure rapide de ces paliers. C’est ce que montre l’expérience représentée sur la figure 25.1a, dans laquelle on fait tourner une tige AB autour d’un axe qui passe par son centre K et qui fait un angle u différent de 0 ◦ ou 90◦ avec elle. i) En fixant à l’une des extrémités A de la tige une masselotte, on constate que la rotation de la tige perturbe de façon sensible les paliers de fixation. ii) Cette perturbation est affaiblie si on fixe une masselotte identique à l’autre extrémité B, symétrique de A par rapport à K, de telle sorte que le centre de masse C soit situé sur l’axe de rotation en K.  c Dunod – Toute reproduction non autorisée est un délit

iii) Elle s’atténue encore davantage si la tige est perpendiculaire à l’axe de rotation. Comme l’usure rapide des paliers P 1 et P2 doit être évitée, en raison des graves dangers que peut occasionner leur rupture, pour des vitesses angulaires importantes, il est nécessaire de dégager de l’étude du mouvement d’un solide autour d’un axe les conditions dites d’équilibrage qui évitent de tels inconvénients. Voyons d’abord comment on réalise la liaison pivot qui permet le mouvement de rotation d’un solide autour d’un axe fixe. B K P1

µ Ω P2

A

x

Solide Tige

Ω

O

y

a) F IG . 25.1.

R

S b)

Δ z

422

25. Mouvement d’un solide autour d’un axe fixe

I . — LA LIAISON PIVOT I . 1 . — Définition Soit un solide quelconque S pouvant tourner autour d’un axe maintenu en ses extrémités par des paliers de fixation. L’axe est confondu avec l’axe Oz d’un référentiel terrestre R = Oxyz. Cette liaison qui impose à S un seul degré de liberté angulaire est une liaison pivot (Fig. 25.1b). Elle est parfaite si la puissance des forces de contact est nulle, soit en notant R et G O les éléments, au point O, du torseur des actions de contact qu’exerce l’axe de rotation Oz sur S : Pt = P = G O · V = G Oz V = 0 d’où G Oz = 0

La liaison est donc parfaite si GO est normal à V.

I . 2 . — Réalisation technologiqued’une liaison pivot parfaite Il existe plusieurs manières de réaliser une liaison pivot parfaite. Les schémas de la figure 25.2 représentent quatre exemples d’une telle réalisation : (a) par contact quasi ponctuel, utilisé notamment dans l’horlogerie, (b) par contact à couteau, comme dans une balance, (c) par roulements à billes dans les machines tournantes, (d) par coussin d’air sans axe matériel, si la rotation est très rapide.

air a)

b)

c)

d)

F IG . 25.2.

II . — MOUVEMENT D’UN SOLIDE AUTOUR D’UN AXE FIXE II . 1 . — Analyse générale Considérons un solide quelconque S, de masse m, en rotation autour de l’axe Oz du référentiel terrestre R supposé galiléen (Fig. 25.3). Les liaisons permettant ce mouvement sont supposées isodynamiques, c’est-à-dire que toute liaison superflue est exclue. Notons [F, MO ] la somme et le moment des forces extérieures autres que les forces de contact et [R, GO ] la somme et le moment des forces de contact au point fixe O . x x θ y

R θ y

R

C l

O

H

F IG . 25.3.

S

z

423

Mouvement d’un solide autour d’un axe fixe

Le solide a un degré de liberté, mais les actions de contact introduisent des inconnues supplémentaires, qui sont au nombre de cinq, si la liaison pivot est parfaite : trois pour R et deux pour G O , puisque GOz = 0. Les six équations scalaires issues des théorèmes de la quantité de mouvement et du moment cinétique sont donc nécessaires. Cependant, dans ce cas, les actions de contact a priori inconnues ont une puissance nulle, ce qui confère au théorème de l’énergie cinétique une grande efficacité : comme il n’y a qu’un seul degré de liberté, ce théorème fournit d’emblée l’équation du mouvement. En effet : d Ek d Ek = (F + R) · vO + (MO + GO) · V se réduit à = M Oz · V = MOzu˙ dt dt L’énergie cinétique étant Ek = IOz u˙ 2/2, où I Oz est le moment d’inertie du solide S par rapport à l’axe de rotation Oz , on trouve : d Ek uu˙ = M Oz u˙ = IOz ¨ dt

soit

Iu¨ = M Oz

en écartant la solution sans intérêt u˙ = 0. On retrouve aisément cette dernière équation à l’aide du théorème du moment cinétique : d LO = MO + G O dt

donne

d(LO · e z) = M O · ez dt

la liaison étant parfaite et donc GO · ez nul. Il en résulte : d L Oz = M Oz dt

d’où

IOzu¨ = M Oz

II . 2 . — Pendule pesant a) Définition

 c Dunod – Toute reproduction non autorisée est un délit

Un pendule pesant est un solide en mouvement autour d’un axe fixe horizontal sous l’action de son poids. Évidemment, l’action du poids n’est efficace que si l’axe de rotation ne passe pas par le centre de masse du pendule (Fig. 25.4a). b) Conservation de l’énergie mécanique L’application du théorème de l’énergie, pour ce système conservatif, donne une équation analogue qui se prête bien à une discussion qualitative du mouvement. L’énergie cinétique a pour expression : ˙2 I u Ek = Oz 2 Quant à l’énergie potentielle de pesanteur, elle s’écrit, si l désigne la distance du centre de masse C à l’axe de rotation : Ep = −mg · OC + Cte soit E p = −mgl cos u en explicitant et en adoptant comme origine de Ep sa valeur pour u = p/2. Comme la puissance des forces non conservatives est nulle, l’énergie mécanique est constante : Em =

I Oz u˙ 2 − mgl cos u = Cte 2

424

25. Mouvement d’un solide autour d’un axe fixe mgl Ep (µ) Em O z

l

g

µ



- µ0

C -

¼ 2

a)

0

Ek

- mgl b)

µ0

¼

µ

¼ 2

F IG . 25.4.

c) Nature du mouvement Discutons, à l’aide du graphe E p (u), les différents mouvements possibles suivant la valeur de l’énergie Em (Fig. 25.4b) et par conséquent suivant les conditions initiales : I u˙ 2 Em = Oz 0 − mgl cos u 0 2 i) Si E m  mgl, le mouvement est révolutif.

ii) Si −mgl < E m < mgl, le mouvement du pendule est oscillatoire entre deux valeurs symétriques de l’angle u. Dans le cas où Em est voisin de 0, u reste petit et l’équation différentielle du deuxième ordre se réduit à : mgl 1/2 u + v20 u = 0 en posant v 0 = u ≈ −mgl u soit ¨ I Oz ¨ IOz Le mouvement est donc sinusoïdal de la forme : u = um cos(v0 t + f) = um cos 2p

t +f T0

avec

T0 =

2p I Oz = 2p mgl v0

1/2

Les constantes um et f sont évidemment déterminées par les conditions initiales. Ordre de grandeur : Pour une tige homogène, de masse m et de longueur L = 20 cm, qui oscille autour de l’une de ses extrémités, l = L/2 et le moment d’inertie est I = mL 2/3. Par conséquent, la période vaut : 1/2 mL2 /3 2L 1/2 T0 = 2p = 2p = 0, 73 s mgL/2 3g Notons que la masse m n’intervient dans cette expression, du fait de l’égalité de la masse grave et de la masse inerte (cf. chapitre 7). La mesure de la durée de dix oscillations confirme une telle valeur avec une bonne précision. d) Pendule de Kater Le pendule de Kater est un pendule pesant réversible que le britannique H. Kater mit au point en 1817, afin de déterminer la valeur du champ de pesanteur g , avec une meilleure précision que celle que permettait le pendule pesant, en raison de la difficulté à connaître avec précision la distance l = OC dans ce dernier (Fig 25.5). Le moment d’inertie I par rapport à l’axe de rotation Oz s’exprime simplement en fonction du moment d’inertie ICz par rapport à l’axe parallèle Cz : IOz = ICz + ml2 = m(R2 + l2)

425

Mouvement d’un solide autour d’un axe fixe R étant le rayon du gyration (cf. chapitre 17). Il en résulte une nouvelle expression de T 0 : R 2/l + l g

T0 = 2p

1/2

On voit que le pendule simple, de longueur ls = R2 /l + l , est synchrone de ce pendule pesant. En outre, si l’on fait osciller le pendule autour d’un axe O  z , parallèle à Oz et passant par le point , situé sur la droite OC à l’opposé de O , à la distance l  de ce point, on trouve une période de même expression : O

T 0

= 2p

R2 /l + l g

1/2

d’où

T 0 = T0

pour

l =

r2 l

On dit que les axes Oz et O z sont réciproques. La période s’écrit alors simplement en fonction de la distance OO = L = l + R 2/l : T0 = 2p(L/g) 1/2 . Si le pendule bat la seconde, c’est-à-dire si T 0 = 2 s , on a la relation simple suivante g = p2L qui permet de déterminer g à partir de la mesure précise de L . Ce pendule n’est plus utilisé depuis 1930, car les techniques actuelles, surtout celles s’appuyant sur des données satellitaires, sont bien plus précises. . L = Iµ

Oz l g

u

Em > mgl Em = mgl

Cz l0 O0 z

x F IG . 25.5.



0

¼

θ

Em < mgl

F IG . 25.6.

 c Dunod – Toute reproduction non autorisée est un délit

e) Analyse dans l’espace des phases L’état mécanique du pendule pesant étant caractérisé par la donnée à la fois de u et de sa dérivée ˙ plus précisément du moment cinétique L = I u, ˙ il est préférable d’étudier les différents mouvements u, dans l’espace des phase s (u, L). L’énergie mécanique Em s’écrit en fonction de u et L : L2 − mgl cos u d’où 2I On retrouve les deux types de mouvement : Em =

L = [2I(Em + mgl cos u)]1/2

i) E m  mgl. Le moment conjugué L est une fonction périodique de u, lequel ne s’annule jamais. Le mouvement du pendule est révolutif. ii) −mgl < Em < mgl. Le pendule oscille entre deux valeurs opposées de l’angle, u 1 et −u1 , telles que u1 = arccos(−E m/mgl). Dans l’espace des phases, la trajectoire du point représentatif du système est celle représentée sur la figure 25.6. On y distingue aisément deux zones : l’une pour laquelle le mouvement est oscillatoire, l’autre pour laquelle le mouvement est révolutif. On voit que les conditions initiales permettent de déterminer sans ambiguïté le type de mouvement du pendule dans l’espace des phases : mouvement oscillatoire ou mouvement révolutif.

426

25. Mouvement d’un solide autour d’un axe fixe

f) Influence de l’amplitude sur la période Si l’amplitude d’un pendule simple n’est pas assez faible pour justifier l’approximation sin u = u, l’équation différentielle du mouvement comporte des termes non linéaires en u puisqu’elle s’écrit : ¨ u + v20 sin u = 0 En remplaçant sin u par un développement à l’ordre trois en u, on obtient l’équation différentielle suivante : u3 v2 ¨ = 0 soit ¨ u + v20 u − u + v 20u − 0 u3 = 0 3! 6 Une analyse, analogue à celle déja effectuée dans le cas du pendule simple (cf. chapitre 10), permet d’obtenir une expression approchée de l’influence de l’amplitude des oscillations sur la période : v = v0

u2 1− m 8

1/2

d’où

u2 1− m 8

T = T0

−1/2

≈ T0 1 +

u2m 16

II . 3 . — Pendule de torsion a) Définition Un pendule de torsion est un solide en mouvement autour d’un axe, qui passe par son centre de masse, sous l’action d’un couple de rappel, proportionnel à l’angle de rotation, exercé par un fil (Fig. 25.7). z g

y

θ

x

y

x

S F IG . 25.7.

b) Équation du mouvement Supposons que le solide soit soumis en outre à un couple de frottement visqueux, c’est-à-dire de la forme −a ˙u. L’équation I¨ u = M Oz donne : I¨ u = −Cu − a u˙

soit

u˙ ¨ u + + v20u = 0 te

en introduisant la durée de relaxation te en énergie et la pulsation propre v0 : 1 a C = et v20 = te I I Cette équation est caractéristique d’un mouvement sinusoïdal amorti par frottement visqueux (cf. chapitre 10). c) Nature du mouvement Dans le cas fréquent où t e est suffisamment grand (Q = v0 te  1/2), le mouvement admet comme équation horaire (cf. chapitre 10) : t 1 1/2 u = A exp − cos(vat + f) où v a = v0 1 − 2te 4Q2

427

Mouvement d’un solide autour d’un axe fixe

est la pulsation de ce mouvement sinusoïdal amorti. Les constantes A et f sont déterminées par les conditions initiales. On vérifie bien que, pour te infini, le mouvement est harmonique de période : T0 =

2p = 2p v0

I C

1/2

Une application intéressante du pendule de torsion est la détermination expérimentale du moment d’inertie d’un satellite artificiel par rapport à son axe de révolution, avant son lancement. Par exemple, si T0 = 12 s et C = 3, 5 × 10 3 N . m . rad−1 , on trouve : T2 I = 02 C = 12, 7 × 10 3 kg . m 2 4p

III . — MACHINES TOURNANTES Une machine tournante est un solide de révolution S constitué généralement par le rotor d’un moteur, l’arbre de transmission et le rotor d’une machine utilisatrice. Les stators du moteur et de l’outil exercent respectivement le moment moteur Gm = Gm ez et le moment résistant Gr = G r ez : lors d’une mise en route, Gm > 0 et G r < 0 (Fig. 25.8a). Les frottements, supposés visqueux, exercent un couple ˙ z, u˙ étant la vitesse angulaire de la machine. de frottement : Gf = −a ue III . 1 . — Mise en rotation ˙ Par Le moment des forces extérieures par rapport à l’axe de rotation est : G Oz = Gm + G r − a u. conséquent, I étant le moment d’inertie par rapport à l’axe de rotation, il vient : u = G m + G r − au˙ I¨

a G + Gr u + u˙ = m soit ¨ I I

Comme a/I et (Gm + G r )/a sont respectivement homogènes à l’inverse d’une durée et à une vitesse angulaire, on pose :

 c Dunod – Toute reproduction non autorisée est un délit

1 a = t I

V0 =

Gm + Gr a

d’où l’équation canonique

˙u V0 ¨ u+ = t t

Avec les conditions initiales u0 = 0 et u˙ 0 = 0, on trouve pour V = u˙ et u : V = V 0 1 − exp −

t t

et u = V 0t − V0 t 1 − exp − Machine

x Moteur

t t

Ω Ω0

Rotor O y

z Stator

Arbre de transmission a)

0

Stator

¿

t b)

F IG . 25.8.

428

25. Mouvement d’un solide autour d’un axe fixe

III . 2 . — Discussion a) Vitesse de régime La vitesse de régime est la vitesse de rotation acquise au bout d’une durée suffisamment longue. Alors V ≈ V0 comme le montre le graphe V(t) sur la figure 25.8b. Notons que : i) cette vitesse est atteinte d’autant plus vite que t est petit (frottements forts et inertie faible), ii) elle est d’autant plus grande que les frottements sont faibles (grâce à des lubrifiants) et que le moment moteur résultant (G m + Gr ) est grand. En particulier, si la machine fonctionne à vide (Gr = 0), V0 est plus élevée qu’en charge. b) Travail en régime stationnaire La machine tournante ayant atteint sa vitesse de régime, son énergie cinétique reste constante ; le travail moteur est alors transformé en énergie interne ou en chaleur par l’intermédiaire des frottements (cf. Thermodynamique). c) Volant d’inertie Soit une machine tournant en régime stationnaire (V ≈ V 0 ). Calculons la variation relative de la vitesse de rotation DV/V0 qu’entraîne l’action d’un couple supplémentaire indésirable Gs entre les instants voisins t et t + Dt. En appliquant le théorème de l’énergie cinétique, on obtient : D

1 2 IV = (P m + P r + Ps )Dt 2

où Pm , P r et Ps représentent respectivement les puissances des couples moteur résistant et supplémentaire. Il en résulte la variation relative de la vitesse de rotation : (P + P r + Ps)Dt DV = m V0 IV 20 Ainsi, en faisant tourner les machines à haut régime (V0 grand) et en leur donnant une grande inertie de rotation (I grand), on évite les brusques variations de leur vitesse angulaire. C’est pour satisfaire à cette seconde condition qu’on ajoute souvent un volant d’inertie sur l’axe de rotation des machines.

IV . — ÉQUILIBRAGE DES MACHINES TOURNANTES L’équilibrage d’une machine tournante consiste à rendre les actions qu’exerce la machine sur les paliers de fixation de l’axe de rotation indépendantes du mouvement. IV . 1 . — Intérêt de l’équilibrage Les équations issues de l’application des théorèmes généraux montrent que le torseur [−R, −G O ] des actions de contact qu’exerce le rotor d’une machine tournante sur les paliers dépend non seulement du torseur des forces données [F, MO] mais aussi du mouvement : dP d LO et − GO = MO − dt dt Ces actions ne seront totalement maîtrisées que si elles sont indépendantes du mouvement, c’est-à-dire si les termes liés au mouvement ont une contribution nulle : d P/ d t = 0 et d L O/ d t = 0. Il en résulte que : −R = F −

429

Mouvement d’un solide autour d’un axe fixe i) Le centre de masse doit être situé sur l’axe de rotation ; c’est l’équilibrage statique.

ii) Le moment cinétique doit être porté par l’axe de rotation, autrement dit l’axe de rotation doit être axe principal d’inertie ; c’est l’équilibrage dynamique. Lorsque la machine est équilibrée, le torseur des actions qu’elle exerce sur les paliers de fixation est égal au torseur des actions connues qui s’exercent sur elle : −R = F et −G O = MO . IV . 2 . — Réalisation de l’équilibrage On réalise l’équilibrage d’une machine en imposant les deux conditions précédentes. Par exemple, on équilibre la rotation de la tige dans l’expérience initiale de la figure 25.1a en fixant la tige au centre de masse et en imposant qu’elle fasse un angle droit avec l’axe. Dans la pratique, on ne distingue pas ces deux conditions ; on procède globalement à l’aide d’une machine équilibreuse, dont le principe repose sur la détection des vibrations qui apparaissent avec une machine non équilibrée. IV . 3 . — Calcul des efforts sur les paliers de fixation Le calcul des efforts exercés par la machine tournante sur les paliers de fixation exige l’explicitation des théorèmes de la quantité de mouvement et du moment cinétique. Pour exprimer ces équations vectorielles, il est commode d’utiliser la base du repère R  = Ox yz lié au solide S et tel que le centre de masse C soit contenu dans le plan Ox z (25.3). D’après le théorème de la quantité de mouvement, il vient, relativement à R : m aC = F + R

La vitesse vC et l’accélération a C de C s’écrivent respectivement, dans la base de R : 0 −lu˙ 2 et u l u˙ l¨   0 0 R R

 c Dunod – Toute reproduction non autorisée est un délit

En projetant la relation vectorielle précédente dans cette même base, on obtient les trois équations suivantes : 0 = Fz + Rz −ml u˙2 = Fx + Rx  ml ¨ u = F y + Ry Quant au théorème du moment cinétique appliqué en O, il donne, relativement à R : d LO dt

= M O + GO R

L’opérateur d’inertie du solide en O, [I ]O , le vecteur vitesse angulaire de S par rapport à R, V, et le moment cinétique en O, LO, de S par rapport à R, s’explicitent dans la base de R respectivement selon : 0 I Ox −I xy −Ix z −Ix  zu˙ 0 et −Iyx  IOy −Iy z −Iy  zu˙ ˙ −Izx  −Izy IOz I Oz u˙ R R R u Pour dériver LO par rapport au temps, relativement à R, il suffit de composer les dérivations : dL dt

= R

dL dt

R

+V×L

430

25. Mouvement d’un solide autour d’un axe fixe

On obtient, en explicitant :

R

u −Ix z ¨ u + −Iy z ¨ ¨ IOz u R

0 GOx  −Ix  zu˙ MOx 0 × GOy  MOy + −Iy  zu˙ = ˙u ˙ IOz u R R  MOz R  GOz

soit, les trois nouvelles équations suivantes : −I x z¨ u + Iy zu˙ 2 = MOx  + G Ox −I yz ¨ u − Ix zu˙ 2 = MOy  + G Oy −IOz ¨ u = MOz + GOz On en déduit les expressions des composantes, dans la base de R , de −R et de −M O : −Rx = Fx  + mlu˙ 2, et

¨ −R y = Fy  − mlu,

−G Ox = Mx  + Ix z¨ u − I y zu˙ 2,

−R z = Fz

−GOy = My  + Iy z ¨ u + Ix z ˙u2 ,

−GOz = 0

On retrouve les conditions de l’équilibrage en faisant l = 0, I xz = 0 et Iy  z = 0, ce qui revient à imposer le centre de masse sur l’axe et à rendre principal cet axe. En général, lorsqu’il sort de la fabrication, le rotor d’une machine tournante ne satisfait pas à ces conditions. Supposons que la distance de C à l’axe de rotation soit seulement de 0, 1 mm. Comparée au poids de S, la contribution ml u˙ 2 à −R d’une rotation uniforme vaut mg pour un rotor tournant à 3 000 tours . min−1 et atteint 100 mg pour un rotor tournant à 30 000 tours . min −1 . Cet exemple montre toute l’importance d’un usinage soigné assurant un centre d’inertie C très voisin de l’axe de rotation. On corrige les imperfections éventuelles du solide tournant en enlevant ou en ajoutant de la matière. IV . 4 . — Exemples a) Équilibrage d’un vilebrequin Le vilebrequin est la pièce importante d’un moteur d’automobile qui permet de transformer le mouvement de translation des pistons en un mouvement de rotation transmis aux roues (Fig. 25.9). Comme les différentes bielles viennent s’articuler sur lui, cette pièce ne présente pas de symétrie matérielle, alors que sa vitesse de rotation est importante, environ 5 000 tours . min−1 . L’équilibrage statique et dynamique du vilebrequin est donc une opération indispensable que l’on réalise le plus souvent par retrait de masses ponctuelles. Piston

x Bielle

Tourillon Arbre

C z1 O z 2

Maneton F IG . 25.9.

F IG . 25.10.

z

431

Mouvement d’un solide autour d’un axe fixe b) Équilibrage d’une roue d’automobile à l’aide de deux masselottes

Notons m1 et m2 les masses des masselottes A1 et A2 ajoutées respectivement aux points de coordonnées x1, y1 , z1 et x 2 , y2, z2 dans le repère Ox yz lié à la roue ; soit en outre I 0xz et I0y z les produits d’inertie de la roue seule (Fig. 25.10). Les deux conditions d’équilibrage, C sur l’axe Oz et Oz axe principal d’inertie donnent : (1) ml + m x + m x = 0 Ix0z Iy0z

+ +

1 1 2 2  m1 y1 + m 2 y2 m1x1z1 + m2x 2z2 m1y1z1 + m2y 2z2

= 0

(2)

= 0

(3)

= 0

(4)

Les équations (2) et (4) combinées montrent que les deux masselottes doivent être situées dans deux plans différents, normaux à Oz. En effet, puisqu’à la fois I0y z = 0 et m1y 1 = 0 : I 0y z + m1y 1(z 1 − z2 ) = 0

donne

z2 = z 1

 c Dunod – Toute reproduction non autorisée est un délit

CONCLUSION Retenons les points essentiels. (1) Le mouvement d’un solide autour d’un axe est réalisé par une liaison parfaite caractérisée par un moment nul des actions de contact par rapport à l’axe de rotation. (2) Si la liaison est parfaite, le mouvement est obtenu aisément en appliquant le théorème de l’énergie. Sinon, il faut appliquer les théorèmes de la quantité de mouvement et du moment cinétique, lesquels sont indispensables dès que l’on veut connaître les actions de contact. (3) L’analyse du rôle de ces actions permet de dégager une notion technologique fondamentale : Tout système en rotation nécessite une symétrie matérielle de révolution autour de son axe de rotation. Dans le cas contraire, les fortes réactions qui prennent naissance risquent de détériorer rapidement les paliers de fixation de l’axe de la machine. Cependant, si la symétrie matérielle de révolution autour de l’axe de rotation ne peut être réalisée, le double équilibrage statique et dynamique permet de rétablir les avantages mécaniques de cette symétrie ; c’est ce que l’on fait dans la pratique avec le vilebrequin et les roues des automobiles. Un exemple simple et important de machine tournante est le pendule pesant : rappelons que, si son énergie mécanique est proche de sa valeur à l’équilibre, son mouvement est harmonique avec la période T 0 = 2p[I/(mgl)] 1/2 .

EXERCICES ET PROBLÈMES P25– 1. Mise en rotation d’une plaque à l’aide d’un projectile Une plaque carrée, de côté a, de masse M, peut tourner parfaitement autour d’un axe horizontal coïncidant avec l’un de ses côtés (Fig. 25.11). Elle est mise en mouvement, à partir de sa position d’équilibre, par un projectile de masse m et de vitesse v qui la heurte en son centre. 1. Trouver l’équation différentielle du mouvement de rotation de la plaque. 2. Quelle doit être la vitesse minimale du projectile pour que la plaque fasse un demi-tour si m = 10 g, M = 1 kg, a = 1 m ?

432

25. Mouvement d’un solide autour d’un axe fixe z

α

z1 g

b

O

O

C

g

y θ a

θ

x

F IG . 25.11.

F IG . 25.12.

P25– 2. Rotation autour d’un axe incliné par rapport à la verticale L’axe Oz autour duquel peut tourner parfaitement une porte rectangulaire plane, homogène, de côtés a et b, fait l’angle a constant avec la verticale du lieu Oz 1 représentée sur la figure 25.12 dans le plan vertical Ozx. Étudier, à l’aide du théorème du moment cinétique, le mouvement de cette porte par rapport au référentiel terrestre Oxyz dont le plan Ozx est vertical. Retrouver les cas classiques a = 0 et a = p/2. P25– 3. Réactions sur l’axe d’un disque en rotation Un disque de masse 10 kg tourne uniformément autour d’un axe, à la vitesse angulaire de 9 000 tours . min −1. L’axe matériel est maintenu vertical par deux points de fixation, l’un O 1 situé à 20 cm du plan du disque, l’autre O2 à 50 cm. Le centre de masse C du disque est excentré de 0, 04 mm de l’axe (Fig. 25.13). Calculer les forces horizontales qui s’exercent sur O1 et O 2 . z

g

g C

A2

y θ

O1 x O2 F IG . 25.13.

S

x A1

F IG . 25.14.

P25– 4. Mesure du moment d’inertie d’un satellite artificiel Afin de déterminer le moment d’inertie d’un satellite artificiel S, par rapport à son axe de révolution Oz, on le suspend par un fil métallique et on le fait osciller dans un plan horizontal autour de sa position d’équilibre (Fig. 25.14). Le fil exerce un couple de torsion de rappel de la forme −Cu, C étant une constante positive et u l’angle de rotation du solide autour de l’axe vertical, compté à partir de l’équilibre. 1. Établir l’équation différentielle de mouvement et en déduire la période T 0 des oscillations. 2. On ajoute deux masselottes identiques A 1 et A2, de masse m, situées symétriquement à la distance d de l’axe vertical. Montrer que la mesure des périodes T0 et T, lorsqu’il y a les surcharges, permet de déterminer le moment d’inertie I0 du solide par rapport à Oz.

433

Mouvement d’un solide autour d’un axe fixe P25– 5. Horloge de campagne

Le balancier d’une horloge est constitué d’une tige et d’un disque fixé à son extrémité inférieure (Fig. 25.15). La tige est assimilable à un segment matériel homogène , de longueur l = 1 m, de masse mt = 0, 300 kg. Le disque homogène a une masse m d = 0, 250 kg et un rayon R = 15 cm. L’ensemble forme un solide de masse m. L’axe du disque est parallèle à l’axe de rotation horizontal OY du balancier, perpendiculaire au plan de la figure. La liaison avec l’axe OY est une liaison pivot parfaite. Le balancier constitue un pendule qui commande l’avancement des aiguilles de l’horloge. À l’instant choisi comme origine, le pendule est immobile et son axe de symétrie D fait avec la verticale descendante OZ un angle u 0. On désigne par g ≈ 9, 80 m . s−2 l’intensité du champ de pesanteur terrestre. 1. a) Montrer, par des considérations qualitatives sur l’énergie, que le mouvement est périodique si les frottements peuvent être négligés. b) En notant d la distance qui sépare le centre de masse C du balancier de l’axe OY et I son moment d’inertie par rapport à cet axe, établir l’équation différentielle du mouvement, si les frottements sont négligeables. c) L’angle u 0 étant très petit, trouver u(t) ainsi que l’expression littérale de la période T 0 des oscillations. 2. a) Calculer en cm la distance d. b) Quel est le moment d’inertie I t de la tige ainsi que celui I d du disque par rapport à OY ? En déduire le moment d’inertie total I. Calculer I et T0 en unités SI. OY

y

X (l,mt )

g

b

g

θ

B α

(R,md ) Z F IG . 25.15.

O

A

D θ x

F IG . 25.16.

 c Dunod – Toute reproduction non autorisée est un délit

P25– 6. Sismographe de La Coste Une tige, de masse négligeable et de longueur l, portant un point matériel A, de masse m, oscille sans frottement autour de l’axe Oz horizontal d’un référentiel terrestre R = Oxyz ; Oy est la verticale ascendante. Un ressort, d’extrémités B et D, exerce sur la tige une force de rappel K × DB proportionnelle à sa longueur DB. Sur la figure 25.16 sont précisées les notations : OA = l, OB = b, OD = d, (Ox, OB) = a = Cte et (Ox, OA) = u. 1. Établir l’équation différentielle en u. 2. À quelle condition sur m, g, l, d, b, K et a, l’angle définissant la position de repos est-il nul ? 3. Trouver l’expression de la période des petites oscillations. On donne l = 5 cm. Quelle doit être la valeur de a pour que la période T0 soit de 20 s ? P25– 7. Pendule pesant constitué d’une demi-boule Un pendule pesant est constitué d’une demi-boule homogène, de rayon r, fixée à l’extrémité d’une tige de longueur égale à r et de masse négligeable (Fig. 25.17). Il oscille, de façon parfaite, dans un plan vertical, autour d’un axe horizontal. Déterminer la période des petites oscillations. Application numérique : r = 5 cm.

434

25. Mouvement d’un solide autour d’un axe fixe y g

g

O µ

K C

K θ

C

x

I

F IG . 25.17.

F IG . 25.18.

P25– 8. Oscillations d’un demi-cylindre plein en contact avec un plan Une demi-cylindre plein homogène, de rayon r, en contact avec un plan horizontal, oscille dans un plan vertical, autour de sa position d’équilibre, en roulant sans glisser sur le plan (Fig. 25.18). Déterminer la période des petites oscillations. Application numérique : r = 5 cm. P25– 9. Mise en rotation d’un disque autour d’un axe vertical On considère le système représenté sur la figure 25.19. Le disque plein homogène D (masse M, rayon R) est mis en mouvement autour de son axe fixe Oz par deux petits disques D1 et D2 , identiques (masse négligeable, rayon r), qui roulent sans glisser sur D ; les centres de ces disques sont reliés par une tige horizontale T (masse m, longueur 2l < 2R). On désigne par c l’angle de rotation de D, u l’angle de rotation de T , f1 l’angle de rotation du disque D1 autour de son axe et f2 l’angle de rotation du disque D2 autour du même axe. Toutes les autres liaisons sont parfaites. 1. Écrire les deux conditions de roulement sans glissement. En déduire que le système dépend de deux degrés de liberté.

2. Quelles sont les deux équations différentielles du mouvement ? En déduire la nature du mouvement. ˙ en fonction du temps 3. Un moteur exerce sur la tige un moment égal à : G m − a u˙ ez. Trouver c ˙ = 0. On exclura le cas singulier c ˙ = u. ˙ dans le cas où initialement c z

D2

D

D1

O µ

x F IG . 25.19.

y

26 Gyroscope. Mouvement d’un solide autour d’un point. Effets microscopiques Le mouvement le plus général d’un solide peut être décomposé en deux : le mouvement de son centre de masse C et son mouvement autour de son centre de masse C , précisément par rapport au référentiel du centre de masse R∗ dans lequel C est fixe. Par conséquent, l’étude du mouvement d’un solide autour d’un point a comme champ d’application, outre la dynamique du solide fixé en un point, celle du solide libre, d’où son intérêt. Une fois la réalisation technique d’une liaison sphérique parfaite précisée, nous étudierons le mouvement le plus spectaculaire, celui du gyroscope dont l’analyse est sur le plan théorique la plus simple et dont les applications sont les plus importantes. Nous analyserons ensuite le mouvement d’une toupie dans l’approximation gyroscopique, avec son application en magnétisme.

 c Dunod – Toute reproduction non autorisée est un délit

Nous terminerons par les mouvement dits de Poinsot et de Lagrange et Poisson, leur étude s’avérant laborieuse, et les applications mineures. Voyons d’abord comment on réalise une liaison sphérique permettant à un solide de prendre toutes les positions autour d’un point qui peut être son centre de masse.

I . — LA LIAISON SPHÉRIQUE Le solide S considéré a un point fixe dans le référentiel du laboratoire ou dans le référentiel du centre de masse. Il a donc trois degrés de liberté de rotation qu’on pourra prendre égaux aux angles d’Euler (cf. chapitre 16) : l’angle de précession c, l’angle de nutation u et l’angle de rotation propre f (Fig. 26.1a). I . 1 . — Réalisation technologique Techniquement, on réalise une telle liaison à l’aide d’une suspension à la Cardan, du nom du mathématicien italien G. Cardano ; cette suspension permet de combiner trois liaisons pivots indépendantes, grâce à deux anneaux, l’un extérieur Aex et l’autre intérieur Ain (Fig. 26.1a). On obtient ainsi tous les mouvements du solide S autour de O. On peut aussi réaliser cette liaison à l’aide d’une rotule, c’est-à-dire une sphère liée au solide S, qui évolue dans une coquille sphérique dont le rayon est voisin de celui de la rotule (Fig. 26.1b).

436

26. Gyroscope. Mouvement d’un solide autour d’un point. Effets microscopiques z z

θ

S

Δ

y

x Ain

φ

O y

x

R

ψ

O

u A ex

a)

b) F IG . 26.1.

I . 2 . — Liaison sphérique parfaite Si la liaison est parfaite, le moment en O (ou en C ) des actions de contact est nul. En effet, puisque O (ou C dans R∗ ) est fixe dans R, on a, en désignant par [R, GO ] le torseur des actions de contact : P c = P t = R · vO + GO · V = G O · V = 0

quel que soit

V si

GO = 0

II . — GYROSCOPE II . 1 . — Définition Le gyroscope est un solide de révolution tournant autour de son axe, à très grande vitesse, et suspendu par son centre de masse C , de façon parfaite. Cet instrument a été inventé par Foucault en 1850 et appelé ainsi parce qu’il permet de visualiser (scope) le mouvement de rotation (gyro) de la Terre sans avoir à observer les étoiles. La grande vitesse de rotation propre est obtenue grâce à un moteur électrique autonome qui est souvent incorporé dans le gyroscope ; en général, la vitesse de rotation du « gyro » est comprise entre 8 000 et 20 000 tours par minute. En outre, une suspension à la Cardan permet au gyro de prendre librement toutes les positions autour de son centre de masse. II . 2 . — Propriété essentielle La propriété essentielle du gyro découle de l’application du théorème du moment cinétique en C dans un référentiel galiléen R. La liaison étant parfaite et le moment des forces données (poids en C, etc.) nul, on a : d LC =0 dt R Ainsi, par rapport à un référentiel galiléen R, le moment cinétique LC est une constante vectorielle ; comme, en outre, du fait de la symétrie, LC et son vecteur vitesse angulaire de rotation V sont colinéaires, l’axe du gyro pointe constamment un même point de R (Fig. 26.2) : LC = Cte

d’où

V = Cte

437

Gyroscope. Mouvement d’un solide autour d’un point. Effets microscopiques

Dans le cas d’expériences précises où le référentiel galiléen considéré est le référentiel de Copernic, l’axe du gyro reste orienté vers un même point du ciel, indépendamment du mouvement de son support lié à la Terre (Fig. 26.2a). Cette propriété du gyro s’observe aussi dans le référentiel terrestre : monté sur un tabouret en rotation, le gyro maintient son axe fixe par rapport à ce référentiel (Fig. 26.2b). C’est précisément cette propriété qui permet d’expliquer l’étonnante stabilité d’une assiette de cirque en rotation rapide autour de l’extrémité d’une baguette. ΩT

Ω Gyroscope

Ω

N

Ω

Gyroscope ΩT

O ¸

T

Tabouret

Terre S a)

b) F IG . 26.2.

II . 3 . — Comportement paradoxal d’un gyroscope À l’aide d’une baguette, exerçons en un point A de l’axe D du gyro, distinct de C, une force occasionnelle Foc verticale (Fig. 26.1a) : on constate que l’axe se déplace selon une direction horizontale. Si cette force est horizontale, alors l’axe D se déplace verticalement. Comparé au mouvement qu’acquiert une simple tige, suspendue par son centre de masse et soumise aux mêmes forces, le comportement de l’axe D est paradoxal. Pourtant la simple analyse, effectuée à partir du théorème du moment cinétique appliqué au centre de masse, donne :

 c Dunod – Toute reproduction non autorisée est un délit

d LC dt

R

= CA × F oc

Si Foc est verticale, la variation de moment cinétique, et donc celle de l’axe D , sont portées par un vecteur horizontal. En revanche, si F oc est horizontal, la variation de moment cinétique présente une contribution verticale. II . 4 . — Applications du gyroscope a) Mise en évidence de la rotation de la Terre Par rapport à l’axe D du gyro qui est fixe dans le référentiel de Copernic R 0 lié aux étoiles, la Terre a un mouvement de rotation propre caractérisé par le vecteur VT de R par rapport à R0 , égal en norme à 7, 3 × 10−5 rad . s −1. Par conséquent, la rotation apparente de D par rapport à R est : VD/R = VR0/R = −VT Ainsi, dans le référentiel terrestre, on peut observer l’axe du gyroscope tourner, autour de l’axe sud-nord de rotation de la Terre, avec une vitesse angulaire égale à l’opposé de la vitesse de rotation propre de la Terre, d’où la mise en évidence et la mesure de cette vitesse.

438

26. Gyroscope. Mouvement d’un solide autour d’un point. Effets microscopiques

b) Gyrocompas de Foucault En 1852, Foucault fit remarquer, à l’Académie des Sciences, que la rotation d’un corps à la surface de la Terre pouvait suffire à indiquer le plan du méridien et la latitude du lieu. L’instrument correspondant, appelé gyrocompas, est précieux pour l’orientation, notamment à bord de véhicules dont les masses métalliques excluent l’usage d’une boussole magnétique (sous-marins, avions, fusées, etc.). Considérons un gyroscope de Foucault suspendu à la cardan en son centre de masse, au moyen des deux anneaux : Ain et Aex (Fig. 26.1a). On sait que, par rapport au référentiel du laboratoire, l’axe D du gyro tourne autour de l’axe sud-nord défini par le vecteur rotation de la Terre V T . En réalité, en raison des frottements, D s’aligne suivant VT. i) Détermination du méridien On impose à l’axe D du gyro d’être horizontal en maintenant A in horizontal (Fig. 26.3) ; l’axe s’immobilise alors selon l’axe horizontal Dm , contenu dans le plan du méridien local formé par V T et la verticale du lieu. Étoile polaire ΩT

Δ

N Δm

ΩT Plan m´eridien O

T

¸

Horizontale locale

Terre S F IG . 26.3.

ii) Détermination de la latitude On impose ensuite à l’axe D du gyro de se mouvoir dans un plan vertical ; on y parvient en rendant Aex solidaire du bâti. L’axe D s’oriente alors suivant V T , c’est-à-dire suivant la direction de l’étoile polaire (Fig. 26.3). On en déduit la latitude du lieu qui est donnée par l’angle (Dm , VT ) . c) Couple gyroscopique On appelle couple gyroscopique le moment qu’exerce un gyroscope sur son support lorsqu’on contraint son axe à changer d’orientation. L’application du théorème du moment cinétique, en C, par rapport au référentiel terrestre R supposé galiléen, donne : dLC = G C, ex→g dt R G C, ex→g étant le moment des actions exercées par le support sur le gyroscope, c’est-à-dire, d’après l’opposition des actions réciproques, l’opposé du couple gyroscopique GC, g→ex . Comme, en outre, le moment cinétique LC est un vecteur porté par l’axe D du gyro, il vient, en introduisant V D/R : d LC dt

= R

d LC dt

D

+ VD/R × LC = V D/R × LC = −GC, g→ex

puisque

d LC dt

=0 D

Gyroscope. Mouvement d’un solide autour d’un point. Effets microscopiques

439

Ainsi, un gyro, soumis à un changement d’orientation, exerce sur le milieu extérieur un couple gyroscopique d’expression : GC, g→ex = LC × VD/R C’est ce que l’on constate lorsqu’on impose manuellement une rotation à l’axe d’une roue en rotation autour de son axe (Fig. 26.4a) ; par exemple, en faisant tourner, autour d’un axe vertical, l’axe de révolution horizontal d’un ventilateur électrique, on ressent au niveau des poignets les effets de ce couple gyroscopique. Si ce couple s’exerce sur un individu assis sur un tabouret d’inertie, initialement au repos, il provoque une rotation du tabouret, de telle sorte que le moment cinétique total, suivant l’axe de rotation, soit nul (Fig. 26.4b).

Couple gyroscopique V LC

LC

a)

b) F IG . 26.4.

d) Stabilisation de trajectoire

 c Dunod – Toute reproduction non autorisée est un délit

Embarqué dans un véhicule accéléré (navire, avion, cabine spatiale, etc.), un gyro permet de maintenir ce véhicule sur la trajectoire prévue. En effet, si la trajectoire de ce dernier vient à changer accidentellement, l’axe du gyro, fixe par rapport aux étoiles, change d’orientation par rapport au véhicule. Ce changement est alors utilisé pour déclencher un mécanisme chargé de redresser sa trajectoire. Remarque : Pour stabiliser la trajectoire d’un véhicule, on utilise actuellement des gyromètres laser ou gyrolasers qui permettent de mesurer la vitesse angulaire du véhicule et par conséquent de corriger cette trajectoire. L’avantage des gyrolasers vient de leur mode de fonctionnement non mécanique, mais essentiellement optique (cf. Optique et Quantique) : on provoque un phénomène d’interférence entre les trajets optiques des faisceaux lumineux parcourus dans des sens opposés ; comme le décalage interférentiel dépend de la vitesse de rotation angulaire du gyrolaser, on accède à la valeur de cette vitesse. e) Gyroscope anti-roulis Le couple gyroscopique exercé par un lourd gyro, sur le navire dans lequel il est embarqué, permet d’enrayer le début d’un roulis. En effet, dès que le navire est soumis à un couple de rotation autour d’un axe longitudinal, un mécanisme impose à l’axe D du gyro, initialement vertical, une vitesse de rotation VD/R autour d’un axe horizontal ; le couple gyroscopique L C ×VD/R qui en résulte permet provoquer une rétroaction permettant de redresser le navire. Ordre de grandeur : un gyroscope de navire est constitué d’un disque de diamètre 2R = 3 m et de masse m = 50 tonnes. Sa vitesse de rotation propre est V = 15 tr . s−1 et celle de précession VD/R = 1, 2 rad . s−1. Le couple gyroscopique vaut donc : G C = LC VD/R =

mR2 5 × 104 × 1, 5 2 V V D/R = × 2p × 15 × 1, 2 = 6, 375 × 10 6 N . m 2 2

440

26. Gyroscope. Mouvement d’un solide autour d’un point. Effets microscopiques

III . — APPROXIMATION GYROSCOPIQUE On dit que le mouvement, autour d’un point fixe, d’un solide S, ayant la symétrie de révolution, satisfait à l’approximation gyroscopique si sa vitesse de rotation autour de son axe de révolution est très grande devant toutes les autres vitesses de rotation, ce que l’on traduit par : V ≈ V ez  Cette approximation joue un rôle capital en mécanique, car de nombreux systèmes et instruments en rotation possèdent un tel mouvement, notamment la toupie des enfants. III . 1 . — Équation du mouvement Le théorème du moment cinétique appliqué au solide S, par exemple une toupie (Fig. 26.5), au point de fixation O, donne vectoriellement : d LO = OC × mg dt

d LO mgl ≈ LO × LO dt

ce qui s’écrit

puisque le moment cinétique est pratiquement porté par le vecteur OC de longueur l. III . 2 . — Nature du mouvement De l’équation vectorielle approchée précédente, on déduit aisément deux propriétés remarquables. i) En multipliant les deux membres de l’équation par L O , on voit que : LO ·

d LO mgl ≈ LO · LO × dt LO

= 0 soit

d L2O d L 2O = ≈ 0 et dt dt

L O ≈ Cte

ii) Si on multiplie les deux membres par ez , on trouve : ez ·

d LO mgl = 0 soit ≈ ez · L O × dt LO

d d LOz ≈ 0 et (LO · e z) = dt dt

LOz ≈ Cte

Ainsi, au cours du mouvement, la norme L O de LO et sa projection L Oz du moment cinétique suivant un axe vertical sont approximativement constants. Il en résulte que le vecteur L O décrit un cône de sommet O, d’axe Oz défini par le champ de pesanteur et de demi-angle au sommet u 0 que fait L O avec l’axe vertical Oz (Fig. 26.5). Retenons donc : L’axe de rotation propre Oz précessionne autour de la direction du champ de pesanteur. Ce mouvement de précession à angle constant suppose évidemment que l’on ait négligé les forces de frottement. Dans le cas contraire, la vitesse de rotation propre ainsi que la vitesse de précession s’affaiblissent et l’angle de nutation augmente jusqu’à la valeur u = p à l’équilibre qui réalise l’orientation de l’axe de la toupie selon le champ de pesanteur. L’équation vectorielle précédente, caractéristique du mouvement, s’écrit aussi : d LO dt

R

≈ Vp × LO

avec

Vp = −

ml mgl mgl g= ez ≈ ez LO LO I 3V

ce qui montre que LO , de norme constante, tourne par rapport à R avec une vitesse angulaire Vp portée par la direction du champ extérieur (cf. chapitre 3). Cette vitesse V p est la vitesse angulaire de précession.

Gyroscope. Mouvement d’un solide autour d’un point. Effets microscopiques

441

z z A g

A L

H θ0

C

y O

x F IG . 26.5.

On peut retrouver cette vitesse de précession en procédant d’une autre façon, plus progressive : si l’on désigne par A et A les extrémités du vecteur L O, aux instants t et t + d t (Fig. 26.5), on a : d LO dt

= R

AA dc = L O sin u0 eu dt dt

et OC × mg = mgl sin u 0 eu

˙ = mgl/LO . puisque ez × ez = − sin u e u . On en déduit, d’après le théorème du moment cinétique : c Retenons donc l’expression suivante de la vitesse de précession en fonction du moment d’inertie I3 par rapport à l’axe de révolution et de la vitesse de rotation propre V : Vp =

mgl gl ez = 2 ez I3 V RV

R étant le rayon de giration : I 3 = mR2 . Ordre de grandeur : Pour une toupie conique, de hauteur h = 5 cm et de rayon de base R b = 3 cm, qui tourne à la vitesse de 30 tr . s−1, on trouve, puisque l = 3h/4 et I 3 = 3mR2b/10 : Vp =

5gh 5 × 9, 80 × 0, 05 mgl = 2 ≈ ≈ 7, 2 rad . s−1 2Rb V 2 × 9 × 10 −4 × 2p × 30 IV

La durée d’une précession complète est donc T p = 2p/V p = 0, 87 s

 c Dunod – Toute reproduction non autorisée est un délit

III . 3 . — Stabilité gyroscopique Une grande rotation autour de l’axe de révolution confère au solide en mouvement une grande stabilité puisque, comme nous l’avons vu, l’effet d’une force verticale, comme le poids, est de déplacer le centre de masse, non pas dans un plan vertical, mais dans un plan horizontal. On peut illustrer cette stabilité à l’aide d’un cerceau (masse m et rayon r) qui roule sans glisser sur un plan horizontal. L’expérience courante nous apprend que ce système se maintient verticalement si la rotation est suffisante et qu’une légère perturbation par rapport au plan vertical est facilement neutralisée. On l’interprète à l’aide du théorème du moment cinétique, appliqué au point géométrique de contact I mobile, par lequel passe constamment la réaction qu’exerce le sol : d LI + vI × mvC = IC × mg avec L I = LC + IC × mv C ≈ 2mr 2V dt Comme vI et vC sont pratiquement colinéaires dans l’approximation gyroscopique, le terme complémentaire vI × mv C est négligeable. Quant à l’influence du poids lorsque le cerceau s’écarte d’un plan vertical, elle se traduit par une variation horizontale d LI du moment cinétique et par conséquent par une précession de l’axe du cerceau autour de la verticale (Fig. 26.6). Ainsi le cerceau garde son inclinaison par rapport à la verticale sous l’effet du poids ou de toute autre force verticale.

442

26. Gyroscope. Mouvement d’un solide autour d’un point. Effets microscopiques

Remarque : C’est par un raisonnement analogue que l’on serait tenté d’expliquer la bonne stabilité d’une bicyclette en mouvement dans une position verticale, si la rotation propre des roues est suffisante. Une analyse minutieuse montre que c’est, en réalité, la force centrifuge dans le référentiel non galiléen lié au vélo qui permet d’expliquer cette stabilité. z g C

LC

O x

y LI dLI = IC ×mg

I F IG . 26.6.

III . 4 . — Interprétation de la précession des équinoxes On sait que le Soleil se trouve apparemment dans le plan de l’écliptique, c’est-à-dire dans le plan du mouvement de révolution du centre de la Terre autour du Soleil (cf. chapitre 13). Il semble donc percer, deux fois dans l’année, le plan équatorial terrestre, une première fois à l’équinoxe de printemps où il émerge du plan équatorial, et une seconde fois à l’équinoxe d’automne où il passe en-dessous de ce plan (Fig. 26.7a). La précession des équinoxes est la très lente variation de la position apparente du Soleil, dans le ciel, à chaque printemps, ce que l’on attribue à la sphéricité imparfaite de la Terre ; on sait, en effet, depuis Newton, que la Terre est renflée à l’Équateur. Le moment des actions gravitationnelles dues aux autres astres, en T, n’est donc pas nul. On montre qu’une expression suffisament précise de ce moment est la suivante : 3 MS ML et I 3 > I 1 M T = A sin(2u) e u avec A = − G(I3 − I1) 3 + 3 4 rS rL G étant la constante de gravitation (cette expression est la moitié de celle établie de façon simpliste en exercice au chapitre 6). La présence de la différence des moments d’inertie, I3 − I 1 , rappelle que ces actions sont liées au défaut de symétrie de la Terre. Les contributions solaire et lunaire apparaissent par le rapport MK/r 3, M K étant la masse de l’astre exerçant la force et r la distance au centre de la Terre, comme dans les marées. L’influence de la Lune est donc 2,17 fois plus forte que celle du Soleil (cf. chapitre 7). z

Plan équatorial

N

23 ◦ 26

θ0 y

A

w

T Soleil

H

Plan de l'écliptique

z0

E Terre

A

x0

a)

b) F IG . 26.7.

Terre S

Ligne des nœuds

P

y0

ψ x u φ

443

Gyroscope. Mouvement d’un solide autour d’un point. Effets microscopiques

Dans le référentiel géocentrique R g , le mouvement de la Terre peut donc être considéré comme un mouvement de Lagrange et Poisson dans l’approximation gyroscopique : en effet, la vitesse de rotation propre V = f˙ ≈ 7, 3 × 10 −5 rad . s−1 est très grande devant la vitesse de précession Vp ; en outre l’angle de nutation u que fait l’axe de révolution avec l’axe perpendiculaire au plan de l’écliptique est pratiquement constant et vaut u 0 ≈ 23, 5◦. ˙ = mgl/LO si le Par analogie avec le cas de la toupie dans l’approximation gyroscopique, où c moment en O est MO = OC × mg = mgl sin u e u, on trouve la vitesse de précession : A sin(2u)/ sin u 3 I3 − I1 c˙ = =− G ˙ 2 I3 I3f

M S ML + 3 r 3S rL

cos u ˙ f

˙ en rappelant que T2a /rS3 = 4p2/(GMS ) (3 e loi de Kepler) et que On simplifie le calcul de c ˙ = 2p/Tj , Ta et T j étant respectivement les périodes annuelle et journalière : f ˙ = − 3 G I 3 − I1 MS × 3, 17 × cos u = −3p × 3, 17 × c 2 2p/Tj I3 r 3S

I 3 − I1 I3

Tj T 2a

cos u

On en déduit l’angle de précession des équinoxes après une année : ˙ a = −3p × 3, 17 × I3 − I1 cT I3

Tj cos u Ta

Comme Ta = 365, 25 T j , (I3 − I1 )/I 3 = 1/306 et cos u0 = 0, 917, on trouve la valeur suivante : ˙ a ≈ −2, 45 × 10−4 rad cT

et

T=

2p 2p T a ≈ 259 T a ≈ 259 siècles = ˙ 245 × 10 −6 |c|

Remarquons que, depuis environ 2 000 ans, le déplacement a été de 27, 8 ◦ ; c’est pratiquement 360◦ /12 = 30 ◦, c’est-à-dire la distance angulaire qui sépare deux constellations successives de l’ensemble des douze constellations que semble parcourir le Soleil, de façon uniforme, au cours de sa révolution annuelle apparente. Cet ensemble de constellations est le zodiaque (bestiaire en grec) : Bélier, Taureau, Gémeaux, Cancer, Lion, Vierge, Balance, Scorpion, Sagittaire, Capricorne, Verseau, Poissons.

 c Dunod – Toute reproduction non autorisée est un délit

Il y a 2 000 ans, le Soleil se trouvait dans la constellation du Bélier le jour de l’équinoxe de printemps (20-21 mars), alors qu’à cette même date de l’année, il était, en 2007, dans celle des Poissons (une constellation en arrière en raison du signe moins). Ainsi, il existe un décalage continuel entre la constellation dans laquelle se trouvait effectivement le Soleil le jour de notre naissance et le signe zodiacal que les adeptes de l’astrologie attribuent à chacun d’entre nous, en affirmant prévoir notre avenir, évidemment dans un contexte plus psychologique et commercial que scientifique !

IV . — L’APPROXIMATION GYROSCOPIQUE EN MAGNÉTISME L’interaction d’un dipôle magnétique rigide et d’un champ magnétique fournit un exemple important de mouvement dans l’approximation gyroscopique. IV . 1 . — Interaction d’un moment magnétique avec un champ magnétique Considérons un petit barreau magnétique caractérisé par un moment magnétique m porté par sa direction principale (Fig. 26.8a). On montre qu’une telle grandeur vectorielle traduit l’interaction du dipôle avec un champ magnétique appliqué B (cf. Électromagnétisme) : un tel champ exerce en effet sur le dipôle un moment de force, en son centre de masse, d’expression : m × B.

444

26. Gyroscope. Mouvement d’un solide autour d’un point. Effets microscopiques

L’application du théorème du moment cinétique au centre de masse du dipôle, par rapport au référentiel du laboratoire R, donne l’équation vectorielle suivante : d LC dt

R

= m×B

Or, l’expérience et l’analyse montrent qu’il existe une relation de proportionnalité entre le moment magnétique d’un tel dipôle et son moment cinétique : m = gL C La quantité g est appelée le coefficient gyromagnétique. B

B

μ

C

O

μ v

r A

a)

−e

b) F IG . 26.8.

On peut montrer qu’il en est bien ainsi dans la représentation simpliste que Bohr fait de l’atome d’hydrogène : l’électron (charge −e ), en mouvement circulaire autour du proton, est assimilé à une petite spire conductrice parcourue par un courant (Fig. 26.8b). Le moment magnétique de cette spire élémentaire a pour expression (cf. Électromagnétisme) : dq 2 −e 2 −evr pr n = pr n = n dt 2 T où S est l’aire du disque défini par la trajectoire circulaire, n la normale orientée de ce disque, v la vitesse du mouvement orbital et T = 2pr/v la période de révolution du mouvement de l’électron. Comme le moment cinétique L O au centre O du noyau est : LO = OA × me v = me vr ez on a bien : m = iS n =

m = g(e) LO

avec

g(e) = −

e 2m e

On voit que g a les dimensions d’une charge électrique massique. IV . 2 . — Précession d’un moment magnétique Le théorème du moment cinétique donne donc, pour un dipôle magnétique m soumis à un champ magnétique B : d LC = gL C × B = −gB × L C dt R soit : d LC = VL × LC avec V L = −gB dt R Dans le cas d’un électron orbitant autour d’un noyau, cette vitesse angulaire, dite de Larmor, a pour expression : e B VL = −g (e) B = 2m e

445

Gyroscope. Mouvement d’un solide autour d’un point. Effets microscopiques

Ainsi, comme pour la toupie dans l’approximation gyroscopique, le vecteur moment cinétique a une norme constante et une projection selon la direction du champ magnétique constante. Par conséquent, le moment magnétique précessionne autour de B avec une vitesse angulaire proportionnelle à B. Ordre de grandeur : Calculons V L pour l’atome de Bohr dans un champ magnétique, de l’ordre de 1 T : 1, 6 × 10−19 eB ≈ 0, 88 × 1011 rad . s−1 VL ≈ = −30 2m e 2 × 0, 91 × 10 Il est instructif de comparer cette valeur à la vitesse angulaire de l’électron sur son orbite. Dans l’état fondamental de l’atome d’hydrogène on a (cf. Quantique) : aB ≈ 53 pm

v≈

c ≈ 2 × 106 m . s −1 d’où 137

v ≈ 4 × 1016 rad . s −1 aB

et

VL 1 v/a B

On en déduit que la perturbation apportée par le champ magnétique au mouvement orbital de l’électron est très faible. Ce résultat est connu sous le nom de théorème de Larmor (cf. Électromagnétisme). IV . 3 . — Relaxation Sous l’action de forces antagonistes, dues par exemple à l’agitation thermique ou à la simple présence de dipôles magnétiques identiques dans le voisinage, on observe un phénomène de relaxation : la précession ralentit progressivement jusqu’à devenir nulle lorsque le moment du dipôle magnétique est orienté selon la direction et le sens du champ B. On retrouve ici une situation analogue à celle de la toupie qui, du fait d’une relaxation dû aux frottements, finit par s’orienter suivant le champ de pesanteur. IV . 4 . — Résonance magnétique nucléaire

 c Dunod – Toute reproduction non autorisée est un délit

La résonance magnétique nucléaire (RMN) est une technique inventée séparément par les physiciens américains E. Purcell et F. Bloch en 1946. On soumet les moments magnétiques nucléaires intrinsèques, dit de spin S , de la matière condensée (liquide ou solide) à un fort champ magnétique uniforme, B0 = B0 ez , de l’ordre de quelques teslas (cf. Quantique). Pour réaliser des champs aussi intenses, on utilise un électro-aimant à supraconducteurs que l’on plonge dans de l’hélium liquide (Fig. 26.9a). D’après ce qui précède, ce champ magnétique produit une précession des moments magnétiques de spin, avec une vitesse angulaire qui vaut VL = gsB 0, si gs est le coefficient gyromagnétique de spin du noyau, c’est-à-dire le rapport entre son moment magnétique intrinsèque ms et son spin. En raison de la relaxation, les moments magnétiques s’alignent selon B0 , soit en position parallèle, soit en position antiparallèle. L’énergie d’interaction magnétique −ms · B 0 (cf. Électromagnétisme) prend donc les deux valeurs suivantes : E− = −m s,zB0

et E+ = ms,z B0

où ms,z est la projection, suivant l’axe Oz défini par B0 , du moment magnétique de m s . On dit que B0 lève la dégénérescence des niveaux d’énergie ; l’écartement énergétique vaut alors : DE = ms,z B0 − (−m s,z B0) = 2m s,zB 0 Pour un noyau à un seul proton, m s,z vaut (cf. Quantique) : m s,z = mN

avec

mN =

e  ≈ 5, 05 × 10−27 J.T−1 2mp

446

26. Gyroscope. Mouvement d’un solide autour d’un point. Effets microscopiques

On soumet, en outre, l’échantillon à un champ magnétique supplémentaire b = b m ex , produit par une bobine. Ce champ, beaucoup plus faible que B0 , est orthogonal à ce dernier et tourne à la vitesse angulaire V ez autour de la direction de B 0 (Fig. 26.9b). Vide Échantillon

Vide z

Azote liquide

Hélium liquide

Hélium liquide

Azote liquide

B0

B0 mN pour b  0 y y O mN pour b  0 Vt (résonnance) b x x

Bobines supraconductrices

Enceinte

Générateur HF Bobines supraconductrices a)

b) F IG . 26.9.

Appliquons le théorème du moment cinétique, au dipôle magnétique de moment m N , en son centre de masse, par rapport au référentiel du laboratoire R. On a, puisque R est galiléen : dS dt

R

= m N × (B 0 + b)

d’où

dS dt

R

= −g (s N) (B0 + b) × S

puisque mN = g(s N) S , g (s N) étant le coefficient gyromagnétique de spin du noyau. Dans le référentiel R  , qui tourne autour de Oz , à la vitesse angulaire V = V e z par rapport à R , les champs B0 et b sont stationnaires. Il vient donc, d’après la formule de Bour (cf. chapitre 3) : dS dt

ce qui donne : dS dt

R

= −g(s N)

R

+ V × S = −g (s N) (B0 + b) × S

B0 + b +

V (N ) gs

×S

soit

dS dt

R

= V× S

en introduisant : V0 = −g(s N) B0

V b = −g(s N) b

et V = V0 + V b − V ez

Ainsi, le moment magnétique mN précessionne autour de l’axe Oz , avec le vecteur vitesse angulaire V . Si la condition V = V0 , dite de résonance, est satisfaite, on a : V = Vb = −g (s N)b Ainsi, à la résonance, le moment magnétique mN précessionne autour de b, avec la vitesse angulaire (N ) Vb = −g s b, puis finit par s’orienter selon b en raison de la relaxation. Cette technique permet d’accéder, par l’intermédiaire de la pulsation V du champ tournant, aux facteurs gyromagnétiques des noyaux, précisément des protons pour l’hydrogène. Comme ces facteurs varient avec l’environnement spatial de ces protons, on en tire des informations précieuses sur cet

Gyroscope. Mouvement d’un solide autour d’un point. Effets microscopiques

447

environnement. Si s(r) caractérise cet environnement des protons dans l’échantillon considéré, la condition de résonance s’explicite selon : V(r) = V0 [1 − s(r)] avec V 0 = g(s p) B0 Le coefficient gyroscopique de spin du proton s’écrit (cf. Quantique) : mN e e avec g(s p) ≈ 5, 585 et g(s p) = g(s p) = 2m p 2mp 

Ordre de grandeur : Comme g(s p) ≈ 26, 87 × 10 7 rad.s−1 .T−1 V V = 26, 87 × 10 7 × B0 et n = = 42,6 × 106 × B 0 2p pour la fréquence. On en déduit la fréquence correspondante en MHz : n ≈ 300 MHz pour B 0 = 7 T . L’utilisation d’un champ magnétique B0 très intense permet d’obtenir une très grande sensibilité. Le domaine électromagnétique concerné est celui de la radio ( l = c/n = 10 m).

V . — MOUVEMENT DE POINSOT V . 1 . — Définition On appelle mouvement de Poinsot (du nom du mathématicien français L. Poinsot) le mouvement d’un solide autour d’un point fixe O tel qu’en ce point le moment des actions extérieures soit nul. Ce mouvement serait, par exemple, celui d’un ballon de rugby soumis à la seule action de la pesanteur, dans le référentiel du centre de masse. En effet, dans R ∗ d’origine C, le moment du poids en ce point est nul. V . 2 . — Propriété fondamentale Comme M O = 0, L O = Cte. D’autre part, la puissance des actions en O, MO · V, étant nulle, E k = Cte. Dans un mouvement de Poinsot, le moment cinétique et l’énergie cinétique se conservent.

 c Dunod – Toute reproduction non autorisée est un délit

V . 3 . — Équations d’Euler d’un solide L’étude du mouvement d’un solide S autour d’un point nécessite évidemment trois équations dans lesquelles n’apparaissent pas les inconnues que sont les composantes de R. Aussi est-il judicieux d’appliquer le théorème du moment cinétique au point fixe O, où le moment des actions de contact est nul. Si le référentiel d’analyse est le référentiel terrestre R, le moment des forces extérieures se réduit à celui de forces connues M O, car R est appproximativement galiléen. Désignons par L O le moment cinétique de S par rapport à R au point de fixation O pris comme origine de R. Il vient : d LO = MO dt R Rappelons que si le référentiel d’analyse est R ∗ , le théorème du moment cinétique en C s’écrit aussi simplement, sans intervention des forces d’inertie : d L∗C = MC dt Associons au solide S le repère principal d’inertie R  = Ox y z . Si V est le vecteur vitesse de rotation de S par rapport à R, on a la relation suivante entre l’opérateur d’inertie [I ] O, V et L O : LO = [I]O V

448

26. Gyroscope. Mouvement d’un solide autour d’un point. Effets microscopiques

Dans la base de R , [I ]O , V et LO s’écrivent respectivement :

R

I1 0 0

0 I2 0

0 0 I3

V1 V2 R V 3

et R

I1V 1 I2V 2 I3V 3

En tenant compte de la formule de Bour, on obtient : d LO dt

d LO dt

= R

R

+ V × L O = MO

ce qui se projette selon :

˙1 I1 V V1 I1 V1 M1 ˙ V2 × I2 V2 = M2 I2 V2 + ˙     V V I 3 3 3 R I3 V3 R R R M3 On en déduit les trois équations suivantes du mouvement du solide autour d’un point : I1V˙ 1 + (I3 − I2)V 2 V3 = M 1 I2V˙ 2 + (I1 − I3)V 3 V1 = M 2 I3V˙ 3 + (I2 − I1)V 1 V2 = M 3 Ces équations du mouvement, exprimées en fonction des composantes du vecteur vitesse de rotation du solide, ont été établies par Euler, d’où leur nom équations d’Euler. Précisons que, dans ces équations, M O et V apparaissent par leurs composantes dans la base de R  lié au solide. Aussi ces trois équations sont-elles généralement difficiles à exploiter. Nous allons voir cependant qu’elles sont très instructives. V . 4 . — Vitesses de rotation stationnaires Les équations d’Euler permettent d’établir le résultat suivant concernant la conservation de la vitesse angulaire de rotation d’un solide au cours du mouvement : Toute rotation d’un solide autour d’un axe principal d’inertie est stationnaire. Montrons-le à partir des équations d’Euler, qui s’écrivent dans ce cas : ˙ 1 + (I3 − I2 )V 2V 3 = 0 I1 V ˙ 2 + (I1 − I3 )V 3V 1 = 0 I2 V ˙ 3 + (I2 − I1 )V 1V 2 = 0 I3 V Notons d’abord que, si V est fixe par rapport à R, il l’est aussi par rapport à S . En effet, d’après la composition des dérivations de Bour, on a : dV dt

= R

dV dt

S

+V×V =

dV dt

S

Plusieurs cas doivent être considérés. a) La matrice principale est quelconque (I1 = I2 = I3 )

Les trois solutions suivantes, pour lesquelles deux des composantes de v sont nulles, satisfont aux équations d’Euler : i) V1 = Cte V3 = V2 = 0 ii) V 2 = Cte V1 = V3 = 0 iii) V 3 = Cte V1 = V 2 = 0. Le vecteur V est donc dirigé suivant l’un des axes principaux Ox  , Oy, Oz .

449

Gyroscope. Mouvement d’un solide autour d’un point. Effets microscopiques b) La matrice principale est cylindrique (I1 = I2 = I3)

Dans ce cas de symétrie cylindrique, les deux solutions suivantes satisfont aux équations d’Euler : i) V3 = Cte V1 = V2 = 0 ii) V 1 = Cte V2 = Cte V3 = 0. La rotation s’effectue donc à vitesse angulaire constante, soit suivant l’axe Oz  , soit suivant un axe perpendiculaire à Oz, ce qui confirme la proposition énoncée. Exemple : La rotation d’un ballon de rugby autour de son axe de révolution ou autour d’un axe perpendiculaire est stationnaire (Fig. 26.10a). En revanche, une rotation, portée par un axe quelconque, varie au cours du temps. c) La matrice principale est sphérique (I1 = I 2 = I 3) Dans ce cas de symétrie sphérique, où tout axe est principal d’inertie, toute vitesse de rotation est solution stationnaire des équations d’Euler. Exemple : C’est le cas d’un ballon de football (Fig. 26.10b) ; en raison de la symétrie sphérique, toute rotation initiale est stationnaire. V

z

z

V

V

C

C

g

g y

O

y

O

x

x a)

b) F IG . 26.10.

V . 5 . — Mouvement de Poinsot d’un solide ayant une symétrie matérielle de révolution Le moment cinétique en O étant une constante vectorielle, choisissons, pour rendre les calculs plus commodes, l’axe Oz de R suivant LO (Fig. 26.11).  c Dunod – Toute reproduction non autorisée est un délit

z z

LO θ

g

φ O x

ψ

y w y

φ  u x

F IG . 26.11.

a) Caractéristiques du mouvement Exprimons, dans la base intermédiaire du repère de Resal, (e u , ew , ez ), le moment cinétique LO/R. Cette base est base principale d’inertie du fait de la symétrie matérielle de révolution : l’opérateur d’inertie [I ]O a donc même expression dans R et dans Re = Ouwz.

450

26. Gyroscope. Mouvement d’un solide autour d’un point. Effets microscopiques

Quant à V et L O , ils s’écrivent, en fonction des angles d’Euler : u˙ I 1u˙ ˙c sin u et I 1c˙ sin u ˙ ˙ cos u + f˙ ˙ cos u + f) Re c Re I 3 ( c Comme LO/R s’écrit aussi LO/R = L sin u e w + L cos u ez  , L étant sa norme, les trois équations différentielles du mouvement de Poinsot sont les suivantes : ˙ cos u + f˙ ) = L cos u I1 c˙ sin u = L sin u et I 3( c I1u˙ = 0 Par conséquent, pourvu que 0 < u < p, on a : u = Cte

˙ = L = Cte c I1

˙ = f

1 1 L − c˙ cos u = L cos u − I3 I 3 I1

= Cte

Dans un mouvement de Poinsot d’un solide ayant la symétrie de révolution, l’angle de nutation ainsi que les vitesses angulaires de précession et de rotation propre sont stationnaires. b) Conséquences i) La norme de V est constante : ˙ 2 sin 2 u + ( c ˙ cos u + w) ˙ 2 = Cte u˙ 2 + c ii) Sa composante V z suivant Oz est une constante. Donc, le vecteur V décrit, dans R , un ˙ définie à partir de l’angle (Ox , Ou) = −f. Ce cône, relatif au cône avec la vitesse de rotation −f référentiel R lié au solide, est appelé le cône du solide. Son angle au sommet a est relié à l’angle de nutation u par la relation : I3 c˙ sin u L sin u/I 1 Vw = = = tan u tan a = ˙ ˙ I1 L cos u/I 3 Vz  c cos u + f iii) La composante V z suivant Oz est aussi une constante ; V décrit donc dans R un cône avec la vitesse c˙ définie à partir de l’angle c = (Ox, Ou). Comme ce cône est lié à R, on l’appelle cône de base. Les figures 26.12a et 26.12b représentent le cône du solide et le cône de base dans les cas où a > u et a < u respectivement. Ces deux cônes sont tangents suivant l’axe portant V. Ce dernier étant un axe instantané de rotation, le cône du solide roule sans glisser sur le cône de base, d’où le nom de cône roulant qu’on lui donne parfois. z µ z Coˆ ne du solide

µ

Coˆ ne de base V

® O Ã

x

z

y w

Coˆ ne du solide

z V

y w

®

y

O Ã

Á x

Coˆ ne de base

x

u

y Á x u

a)

b) F IG . 26.12.

451

Gyroscope. Mouvement d’un solide autour d’un point. Effets microscopiques c) Exemple : mouvement de Poinsot de la Terre

La Terre peut être assimilée à un solide de révolution autour de l’axe des pôles Oz  , tel que I 3 > I1 puisqu’elle est renflée à l’équateur. En première approximation, le moment des actions gravitationnelles qui s’exercent sur elle, en son centre T, est nul. Le vecteur vitesse angulaire devrait donc, selon la ˙ Exprimons V z en fonction théorie précédente, tourner autour de Oz  avec la vitesse angulaire − f. de u : L cos u L cos u Vz = c˙ cos u + f˙ = + f˙ = d’où : I1 I3 ˙ = L cos u −f

1 1 − I1 I3

=

L cos u I3

I 3 − I1 I1

= Vz

I 3 − I1 I1



V z 305

car

1 I 3 − I1 ≈ 305 I1

Comme la période de rotation autour de Oz  est de 1 jour, celle de cette précession, dite « libre » par opposition à celle due à l’action gravitationnelle non nulle du Soleil, est de 305 jours. On a observé un phénomène analogue de précession de V autour de Oz  avec une période de 400 jours et une amplitude angulaire d’environ 15 secondes d’arc. L’écart de période avec les prévisions de la théorie précédente a été expliqué par Newcomb, à la fin du siècle dernier, à partir des déformations de la Terre sous l’effet de sa rotation.

VI . — MOUVEMENT DE LAGRANGE ET POISSON VI . 1 . — Définition On appelle mouvement de Lagrange et Poisson le mouvement d’un solide homogène ayant la symétrie matérielle de révolution, mobile autour d’un point fixe O et soumis à des forces dont le moment en O est dirigé suivant la ligne des nœuds Ou (Fig. 26.13). Une grande variété de solides physiques (toupie, instruments gyroscopiques de navigation, mouvement de la Terre dans le référentiel du centre de masse, etc.) ont un tel mouvement. z z  c Dunod – Toute reproduction non autorisée est un délit

g

θ

y

C

φ

y O ψ x

w

u

φ x

F IG . 26.13.

VI . 2 . — Équations du mouvement d’une toupie symétrique dans le champ de pesanteur Le moment du poids, M O = OC × mg, est colinéaire à la ligne des nœuds Ou. La toupie en mouvement autour du point fixe O possède trois degrés de liberté que l’on prend égaux aux angles d’Euler. Nous devons par conséquent trouver trois équations scalaires dans lesquelles figurent comme seules inconnues c, u, f. C’est ce que fournit le théorème du moment cinétique appliqué en O par

452

26. Gyroscope. Mouvement d’un solide autour d’un point. Effets microscopiques

rapport à R. Comme la liaison sphérique en O est supposée parfaite, le moment en ce point des actions de contact est nul. Donc : d LO/R = OC × mg dt R Il est commode d’expliciter cette relation dans la base de Resal : Commençons par exprimer LO/R : u˙ I 1u˙ I1 0 0 0 I1 0 c˙ sin u I 1 c˙ sin u = ˙ ˙ ˙ cos u + f) 0 I 3 Re c˙ cos u + f Re 0 Re I3 (c Le moment cinétique L O étant exprimé dans la base de R e , la dérivation par rapport au temps, relative à R, s’obtient en tenant compte de la composition des dérivations de Bour : d LO dt

= R

d LO dt

Re

+ Ve × LO

˙ ez est la vitesse angulaire de rotation de R e par rapport à R. Donc, (d L O / d t) où Ve = u˙ e u + c R s’explicite selon : u I1 ¨ u˙ I1u˙ ¨ sin u + c˙ u˙ cos u) I1 (c + c˙ sin u × I1c˙ sin u d ˙ ˙ ˙ ˙ cos u + f) ˙ (c cos u + f) Re I3 R e c cos u Re I3 ( c dt En désignant la longueur OC par l, l’explicitation du moment du poids OC × mg donne :

Re

0 0 0 × mg − sin u l Re − cos u

d’où

˙ z sin u = mgl sin u I1(¨ u − c˙ 2 sin u cos u) + I 3cv ¨ sin u + 2 c˙ u˙ cos u) − I3u( ˙ c˙ cos u + f) ˙ =0 I1(c I3

d ˙ =0 ˙ cos u + f) (c dt

Cette dernière équation exprime la conservation de la composante L z du moment cinétique suivant l’axe de révolution Oz  : ˙ = Cte Lz = I 3 v z = I3(c˙ cos u + f) La deuxième équation s’intègre aisément, une fois multipliée par sin u : d(c˙ sin 2 u) I1 − I 3vz u˙ sin u = 0 dt

donne

d (I1c˙ sin 2 u + L z cos u) = 0 dt

Cette équation était prévisible puisque le moment du poids est perpendiculaire à Oz ; elle traduit la conservation de la composante du moment cinétique suivant l’axe vertical Oz : ˙ sin2 u + Lz cos u = Cte L z = I1 c Une troisième intégrale première peut être obtenue à partir des deux premières équations ; c’est l’intégrale première de l’énergie mécanique que l’on sait être une conséquence des théorèmes généraux pour un solide, si les seules forces qui travaillent dérivent d’une énergie potentielle. Écrivons le théorème de l’énergie mécanique dans R : E m = Ek + Ep = Cte avec : Ek =

1 1 ˙ 2 sin 2 u + I3 v2z ) LO · V = (I 1u˙ 2 + I1 c 2 2

et E p = mgz C = mgl cos u

Gyroscope. Mouvement d’un solide autour d’un point. Effets microscopiques

453

On en déduit :

1 (I 1 u˙ 2 + I1 c˙ 2 sin 2u + I 3 v2z ) + mgl cos u = E m 2 Compte tenu de ce qui précède, cette dernière équation se met sous la forme : L2z 1 (Lz − Lz cos u) 2 + + mgl cos u = E m I 1u˙2 + 2 I3 I 1 sin2 u soit :

I 1u˙ 2 (Lz − Lz cos u) 2 1 L 2z + Ep,ef (u) = Em où E p,ef (u) = mgl cos u + + 2 2 I3 2I1 sin 2 u est l’énergie potentielle effective, l’énergie potentielle de pesanteur étant prise dans le plan horizontal ˙ et f˙ Oxy . Une fois établies l’équation du mouvement et l’évolution de u, il est possible de trouver c à l’aide des relations : Lz − Lz cos u ˙ = L z − c ˙ cos u et c˙ = f I3 I1 sin 2 u Posé ainsi, ce problème se prête bien à une discussion de mouvement. VI . 3 . — Discussion Traçons le graphe de E p,ef (u) : Ep,ef prend des valeurs infinies pour u = 0 et u = p et passe par un minimum E0 pour 0 < u < p. D’après l’équation I 1u˙ 2/2 = Em − Ep,ef (u), E m doit être supérieur ou égal à Ep,ef (u) (Fig. 26.14). Ep,ef

Em

E0  c Dunod – Toute reproduction non autorisée est un délit

0 µ1

µ0

µ2 p

µ

F IG . 26.14.

a) Em > E 0

L’angle u évolue entre deux valeurs extrêmes u 1 et u2 . Trois cas doivent être considérés suivant le signe de la vitesse de précession c˙ : ˙ garde le même signe (Fig. 26.15a) ; le point C décrit une sphère de rayon l et précessionne i) c toujours dans le même sens tout en oscillant entre les parallèles u 1 et u2. ˙ change de signe et s’annule pour une valeur de u comprise entre u 1 et u2 (Fig. 26.15b) ; ii) c le point C oscille donc entre deux parallèles mais, la vitesse de précession changeant de signe, il décrit des festons bouclés. ˙ change de signe et s’annule en même temps que u˙ (Fig. 26.15c) ; à l’instant où cette condiiii) c tion est réalisée, le point C a une vitesse nulle ; la trajectoire présente alors des points de rebroussement qui, selon l’équation de conservation de l’énergie, doivent être situés sur le parallèle correspondant à l’énergie potentielle maximale.

454

26. Gyroscope. Mouvement d’un solide autour d’un point. Effets microscopiques θ2

θ2

θ1

θ2

θ1

θ1

O

O

O

a)

b)

c)

F IG . 26.15.

b) Em = E 0

˙ sont alors des Dans ce cas, u = u 0 et u˙ = 0. Le mouvement est stationnaire en u ; c˙ et f constantes : la toupie précessionne et tourne uniformément sur elle-même. Ce mouvement, à nutation constante (u = u 0 ) doit satisfaire à l’équation suivante issue de la première des trois équations différentielles, dans laquelle on impose u˙ = 0 : ˙ ˙ 20 cos u 0 − Lz  c0 + mgl = 0 c I1 I1

˙ 0 sont : Les deux solutions en c ˙0 = c

Lz  1 ± 2 cos u 0 I 1

L2z  4mgl cos u 0 − I1 I 21

1/2

=

Lz 4mgl I1 cos u 0 1± 1− 2I 1 cos u 0 L 2z

1/2

Notons que, pour cos u0 < 0, soit p/2  u 0  p, ces solutions existent toujours. En revanche, pour cos u0 > 0, soit 0  u 0  p/2, elles n’existent que si : 4mgl I1 cos u 0 (4mgl I1 cos u0)1/2    1 soit v z L2z I3 Ainsi, pour une valeur déterminée de la rotation propre, communiquée initialement à la toupie, le mouvement à nutation constante, avec le centre de masse au-dessus du point de fixation, admet deux vitesses de précession uniforme, l’une lente et l’autre rapide, pourvu que la vitesse de rotation propre initiale soit suffisante. Remarque : On montre que la condition précédente est aussi la condition de stabilité de la précession lente. C’est la raison pour laquelle on n’observe pas, dans le cas d’une telle toupie, de mouvement de précession rapide. Si u ≈ 0, la condition précédente se réduit à : v z  (4mgl I 1) 1/2/I 3. On dit que la toupie est « dormante » (Fig. 26.16a). z V

g C

K C l O

O a)

b) F IG . 26.16.

F IG . 26.17.

Ordre de grandeur : Pour une toupie demi-boule, de rayon R = 2 cm, et dont la queue a une masse négligeable (Fig. 26.17b), on a (cf. chapitre 17) : 3R 5R 2mR 2 = l=R− I3 = 8 8 5

455

Gyroscope. Mouvement d’un solide autour d’un point. Effets microscopiques On obtient I1, au point de contact O avec le sol, en appliquant le théorème d’Huygens : I1 = I C,1 + mR2

5 8

2

= IK,1 − mR2

3 8

2

+ mR2

5 8

2

=

13mR2 20

puisque IK,1 = 2mR 2/5 . On trouve donc : (4mgl I1) 1/2 = 70, 6 rad . s−1 I3

d’où

Vz   11, 2 tr . s−1

Remarque : Cette rotation rapide de la toupie autour de l’axe vertical, avec le centre de masse C en position basse, est stable. alors que celle, avec C en position haute est instable. En modifiant légèrement la géométrie de la toupie, on peut s’arranger pour obtenir l’inverse. La toupie initialement lancée, en rotation dormante avec C en position basse, se redresse alors pour se placer en rotation dormante, avec C en position haute (Fig. 26.17). On a ainsi réalisé un « tippe-top ».

VI . 4 . — Influence d’une force de frottement. Relaxation Nous avons supposé jusqu’à maintenant que la liaison en O était parfaite, c’est-à-dire que les frottements avaient une influence négligeable. Cependant l’expérience montre qu’il est indispensable de les prendre en compte si l’on veut expliquer le ralentissement de la vitesse de rotation propre f˙ et l’augmentation de l’angle de nutation u. Cette dernière cesse lorsque la toupie atteint sa position d’équilibre stable, u = p, qui correspond à l’orientation du moment cinétique selon la direction et le sens du champ de pesanteur terrestre g.

CONCLUSION

 c Dunod – Toute reproduction non autorisée est un délit

Retenons les points essentiels relatifs au mouvement d’un solide autoour d’un point fixe. (1) On réalise une liaison sphérique parfaite généralement à l’aide d’une suspension à la cardan. Le moment des actions de contact au point de suspension est alors nul. (2) La propriété essentielle d’un gyroscope, c’est-à-dire d’un solide ayant une symétrie de révolution autour d’un axe et doté d’une grande vitesse angulaire autour de cet axe, est la fixité de son axe de rotation par rapport à un référentiel galiléen, d’où ses nombreuses applications dans la navigation maritime, aérienne ou spatiale. Cette grande vitesse de rotation donne au gyroscope un comportement surprenant si on exerce sur son axe une force horizontale ou une force verticale, mais évidemment conforme aux lois de la mécanique. (3) Dans l’approximation gyroscopique, le mouvement autour d’un point O d’un solide, de grande vitesse angulaire autour de son axe de révolution, satisfait à l’équation vectorielle suivante : d LO = Vp × LO dt

456

26. Gyroscope. Mouvement d’un solide autour d’un point. Effets microscopiques

Vp étant le vecteur vitesse angulaire de précession. Dans le cas d’une toupie, la vitesse angulaire de précession autour de la verticale a pour expression : ml g Vp = − LO On interprète ainsi la précession des équinoxes et le comportement des corps en rotation rapide. (4) Les dipôles magnétiques rigides, de moment m, en interaction avec un champ magnétique B, satisfont à une équation du mouvement analogue à la précédente : d LO = V p × L O avec Vp = −gB dt g étant le coefficient gyromagnétique reliant le moment magnétique m au moment cinétique L O . (5) À l’échelle microscopique, les électrons et les nucléons dotés d’un moment magnétique, évoluent dans un champ magnétique selon une équation analogue à celle obtenue en mécanique dans l’approximation gyroscopique. La technique de la résonance magnétique nucléaire, qui en est issue, est largement utilisée pour caractériser les matériaux en chimie et en biologie, ce qui montre l’intérêt et l’actualité du sujet. (6) Le mouvement de Poinsot est celui de tout solide en mouvement autour d’un point O soumis à un moment nul en ce point : le moment cinétique et l’énergie cinétique sont des constantes. (7) Le mouvement d’un solide de révolution, autour d’un point, telle une toupie en rotation, soumise à son poids et à la réaction, est un mouvement de Lagrange et Poisson caractérisé par un moment des forces dirigé suivant la ligne des nœuds ; l’analyse de ce mouvement est longue et son intérêt limité, sauf dans le cas de l’approximation gyroscopique.

EXERCICES ET PROBLÈMES P26– 1. Mouvement d’un ballon de rugby Un ballon de rugby est assimilé à un solide, de centre d’inertie C et d’axe de révolution Cz  (Fig. 26.10a). Le ballon est lancé dans le champ de pesanteur supposé uniforme. On veut étudier le mouvement du vecteur V par rapport au référentiel R = Cx y z  lié au ballon, en négligeant la résistance de l’air. 1. À l’instant initial, le vecteur vitesse de rotation du ballon est V 0 = V 0 e z . Montrer que le vecteur V reste constant au cours du mouvement. 2. Donner les composantes (V 1, V 2, V 3) de V dans une base principale du ballon, en fonction du temps, lorsque V fait initialement un angle a avec e z . P26– 2. Rotation stationnaire d’une station orbitale en mouvement de Poinsot Une station orbitale, de masse M, peut être assimilée à un tore creux de rayon moyen R (Fig. 26.18). Elle est placée sur une orbite terrestre avec une rotation propre V 0 par rapport au référentiel de Copernic R0 . Les actions gravitationnelles qui s’exercent sur elle sont équivalentes à une force unique M G(C) appliquée au centre de masse C. 1. Sachant que l’angle u que fait initialement V 0 avec l’axe de révolution Cz est nul, montrer que la vitesse de rotation est stationnaire.

Gyroscope. Mouvement d’un solide autour d’un point. Effets microscopiques

457

2. Appliquer la loi fondamentale de la mécanique à un passager, de masse m, par rapport au référentiel R lié à la station, en supposant que le passager est soumis en outre à une force occasionnelle F oc . Pour quelle vitesse angulaire V 0 le passager ressent-il un champ moyen égal au champ de pesanteur terrestre ? Application numérique : R = 20 m. z V0 C R

F IG . 26.18.

P26– 3. Instabilité de la rotation d’un solide dans le mouvement de Poinsot 1. Montrer que, dans le mouvement de Poinsot d’un solide, la vitesse de rotation V, de composantes (V1 = V0, V2 = 0, V 3 = 0) dans la base principale du solide, est stationnaire. 2. Établir les équations différentielles auxquelles satisfont, du fait de perturbations, les écarts aux valeurs précédentes : dV2 et dV 3 , du deuxième ordre par rapport à V0 . 3. En déduire que la vitesse de rotation stationnaire autour du premier axe est stable, si I 1 est le plus petit des moments d’inertie ou le plus grand. P26– 4. Toupie dormante Une toupie est constituée d’un cylindre homogène, de hauteur h et de diamètre 2R. À l’une de ses extrémités, on a accolé un petit cône, de dimension et de masse négligeable, afin que le contact de son sommet O avec un plan horizontal fixe Oxy soit ponctuel et parfait. 1. Établir, en fonction de g, h et du rapport h/R, l’expression de la vitesse angulaire minimale qu’il faut donner au cylindre pour qu’il se comporte en toupie dormante.

 c Dunod – Toute reproduction non autorisée est un délit

2. Calculer la valeur, en tours par seconde, de la vitesse angulaire minimale dans le cas d’une telle toupie pour laquelle h = 0, 5 cm et R = 1, 5 cm. On donne à cette toupie que l’on incline d’un certain angle une vitesse double de la précédente. Quelle est sa vitesse de précession en tours par seconde ? 3. Quelle serait la valeur, en tours par seconde, de la vitesse angulaire minimale dans le cas d’un long cylindre tel que h = 13 cm et R = 0, 5 cm ? Commenter. P26– 5. Mouvement gyroscopique d’une demi-boule Un solide de révolution est constitué d’une demi-boule homogène, de masse m et de rayon R = 8 cm, surmontée par une tige HO, de longueur h = 5 cm et de masse négligeable, matérialisant son axe à partir du centre H de la boule entière (Fig. 26.19). On la fait tourner de façon parfaite, autour de O, en lui communiquant une grande rotation propre autour de son axe de révolution et en l’inclinant d’un angle u0 par rapport à la verticale. 1. Quelle est l’expression de la vitesse angulaire de précession du solide ? 2. Calculer cette vitesse angulaire sachant que la vitesse communiquée est de 100 tours par seconde. En déduire la durée de précession de l’axe de révolution.

458

26. Gyroscope. Mouvement d’un solide autour d’un point. Effets microscopiques z g

C H

u h O

y

x F IG . 26.19.

P26– 6. Cylindre creux roulant sans glisser sur un plan horizontal On met en contact, avec un plan horizontal parfaitement lisse, un cylindre creux homogène (masse m, rayon r, longueur l) le long de l’une de ses génératrices (Fig. 26.20). Initialement, la vitesse de ˙ 0 = 0 et la vitesse de rotation autour son centre de masse C est v0 , la vitesse de rotation propre est f ˙ d’un axe vertical est c0 = 0. 1. Étudier le mouvement de C. 2. Trouver les équations différentielles et la nature des mouvements de rotation en c et f. On désignera par I le point de la génératrice en lequel s’exerce la résultante R des forces de contact. ˙0f ˙ 0 satisfait à une condition que l’on 3. Montrer que le contact n’est réalisé que si le produit c exprimera. z

z

z

w

θ z

g

g

x

O

y

C

φ y

C x

x

z F IG . 26.20.

x

I φ

u

F IG . 26.21.

P26– 7. Stabilité d’une roue roulant sans glisser sur un plan horizontal Une roue (masse m et rayon R), roule sans glisser sur un plan horizontal (Fig. 26.21). 1. Établir, en appliquant le théorème du moment cinétique, au point de contact I, les trois équations différentielles du mouvement en fonction des angles d’Euler c, u, f. 2. À quelle relation doivent satisfaire les valeurs initiales u 0, c˙0 et f˙ 0 pour qu’un mouvement stationnaire en u soit possible ? ˙ est très grand devant les autres vitesses angulaires, montrer que u 0 = p/2 3. Dans le cas où | f| ˙ 0 | est supérieure à une valeur que l’on déterminera. ˙ 0 = 0 correspond à un mouvement stable si | f et c Applications au cerceau, au disque et à une roue dont la masse est surtout répartie sur l’axe. ˙ est très grand 4. Même question pour le mouvement stationnaire u 0 = p/2 et f˙ 0 = 0, si |c| ˙ devant les autres vitesses. Donner la condition sur c0 .

Gyroscope. Mouvement d’un solide autour d’un point. Effets microscopiques

459

P26– 8. Couple cabreur ou piqueur d’un avion Un avion initialement en mouvement rectiligne horizontal amorce un virage. Montrer qu’il est soumis à un couple cabreur ou piqueur suivant le signe du produit f˙ c˙ de la vitesse angulaire de rotation propre par la vitesse angulaire de précession. P26– 9. Équations de Lagrange et équations canoniques du mouvement d’une toupie Retrouver les trois équations du mouvement d’une toupie symétrique autour d’un point fixe O : 1. À l’aide des équations de Lagrange.

 c Dunod – Toute reproduction non autorisée est un délit

2. À l’aide des équations canoniques de Hamilton.

27 Oscillateurs couplés. Cas de N oscillateurs identiques Nous savons qu’un système stable, à un degré de liberté, faiblement écarté de sa position d’équilibre, oscille sinusoïdalement autour de cette position, en l’absence de frottement. Nous proposons ici d’étudier le comportement de deux oscillateurs harmoniques couplés en analysant l’influence et l’intérêt du couplage. Nous étendons ensuite l’étude aux systèmes constitués de N oscillateurs identiques, ce qui présente un intérêt considérable car, d’une part un tel système est une représentation efficace des solides cristallins unidimensionnels, et d’autre part il prépare à l’analyse de la propagation des ondes dans un milieu continu.

I . — SYSTÈME DE DEUX OSCILLATEURS COUPLÉS I . 1 . — Deux oscillateurs élastiques reliés par un ressort Considérons le système mécanique de la figure 27.1, constitué de deux masselottes ponctuelles A 1 , A2 et de trois ressorts. Nous supposons, pour simplifier, que ce système évolue sans frottement solide ou visqueux suivant l’axe horizontal d’un référentiel terrestre R = Oxyz, à l’aide par exemple d’une tige sur laquelle coulissent parfaitement les deux masselottes. a) Équations différentielles en l’absence de ressort intermédiaire Si l’on supprime le ressort intermédiaire, l’ensemble se comporte comme deux oscillateurs indépendants. Les équations différentielles, auxquelles satisfont ces oscillateurs, sont alors, avec des notations habituelles, m 1 et m 2 pour les masses, K 1 et K 2 pour les raideurs des ressorts : m1 ¨x1 = −K1 x 1

et

m 2¨ x2 = −K2 x2

où x1 et x2 sont les coordonnées suivant l’axe Ox de A 1 et A 2 comptées à partir de leurs positions de repos. Leurs pulsations respectives sont v1 = (K1/m 1)1/2 et v 2 = (K2/m 2) 1/2 (cf. chapitre 10). b) Équations différentielles en présence d’un ressort intermédiaire Le rôle du ressort intermédiaire, de raideur K 12 , est de réaliser précisément une interaction ou un couplage entre les deux oscillateurs. L’influence de ce ressort se traduit sur A 1 par la force supplémentaire −K12 (x 1 − x2 ) e x et sur A2 par la force analogue −K12 (x 2 − x1 ) ex, toutes deux

461

Oscillateurs couplés. Cas de N oscillateurs identiques

g

y R

A 1(m 1)

O K1

A2 (m2 )

K 12

x1

K2

x2

x

F IG . 27.1.

proportionnelles à la différence des allongements. Des considérations physiques simples, telles que le blocage de A2 , son déplacement réel vers la droite et donc la compression du ressort 2, permettent d’écrire correctement cette force. Les équations différentielles du mouvement sont : m1 ¨ x1 = −K1 x1 − K12 (x1 − x2)

et m2 ¨ x 2 = −K2 x2 − K12 (x 2 − x 1 )

ce qui s’écrit aussi : m1 ¨ x1 + (K1 + K12 ) x1 − K 12 x2 = 0 et

m2 ¨ x 2 + (K 2 + K12 ) x 2 − K 12 x1 = 0

I . 2 . — Analyse énergétique L’analyse énergétique s’obtient aisément en ajoutant les deux équations précédentes du mouvement, une fois la première multipliée par x˙ 1 et la seconde par x˙2 , ce qui permet de faire apparaître l’énergie cinétique de l’ensemble. On trouve, en effet : d 1 1 1 1 m1 x˙21 + m 2 x˙ 22 + (K 1x21 + K2 x22 ) + K12(x1 − x 2) 2 = 0 dt 2 2 2 2 Il vient, en explicitant les diverses contributions énergétiques cinétiques et potentielles : d (1) (2) Ek + Ek + E p(1) + E p(2) + Ep(in) = 0 soit dt

 c Dunod – Toute reproduction non autorisée est un délit

avec :

1 1 (i) E m(i) = E k + Ep(i) = m 1 x˙ 2i + K1x 2i 2 2

d dt

Em(1) + E (m2) + Ep(in) = 0

et Ep(in) =

1 K12 (x 1 − x2 ) 2 2

Il en résulte l’équation suivante de conservation de l’énergie : Em =

i

Em(i) + Ep(in) = Cte

Ainsi, en raison du couplage, l’énergie mécanique du système n’est pas égale à la somme des énergies (1) (2) E m et Em des deux oscillateurs : un terme énergétique supplémentaire E (pin) apparaît en raison du couplage. L’énergie totale Em se répartit entre les deux oscillateurs de façon généralement inégale et variable au cours du temps. Évidemment, si les deux oscillateurs sont découplés (K 12 = 0), l’énergie potentielle d’interaction est nulle ; l’énergie mécanique de l’ensemble se réduit alors à la somme des énergies de chaque oscillateur, lesquelles sont des constantes : (in) Ep

Em = Em,1 + Em,2

avec Em,1 = Cte

et

Em,2 = Cte

462

27. Oscillateurs couplés. Cas de N oscillateurs identiques

a) Pendules pesants couplés Un système équivalent au précédent est celui constitué de deux pendules pesants de masses m 1 et m2 mais de même longueur l . Les tiges sont reliées par un ressort de raideur K c (Fig. 27.2). En l’absence de couplage, les deux pendules oscillent dans le voisinage de leurs positions d’équilibre verticales, avec la pulsation propre v0 = (g/l)1/2 , g étant la norme du champ de pesanteur terrestre. Le rôle du ressort intermédiaire est évidemment de réaliser le couplage entre les deux oscillateurs. Ses extrémités sont reliées aux tiges, à une distance b des axes horizontaux de rotation, aux points B1 et B2 . Le théorème de l’énergie mécanique appliquée à la première masselotte, s’écrit, dans l’approximation des petits mouvements : d(Ek + Epg ) = −K c(bu1 − bu2)bu˙ 1 dt

avec

1 2 2 m1l u˙ 1 − m1gl cos u 1 2

Ek + E pg =

ce qui donne, en dérivant et en simplifiant : u1 + m1¨

m 1g + Kc b2 (u1 − u 2) = 0 l

De même, on obtient, pour la masselotte A 2 : u2 + m2¨

m 2g + Kc b2 (u2 − u 1) = 0 l

En introduisant les degrés de liberté linéiques x1 = lu 1 et x2 = lu2 , ces deux équations deviennent : m1 ¨ x1 = −

m1 g Kc b2 x1 − 2 (x1 − x 2) l l

et

x2 = − m¨

m2 g Kc b 2 x2 − 2 (x2 − x1) l l

On obtient alors les mêmes équations canoniques que celles obtenues précédemment : m 1 x¨1 + (K1 + K12) x 1 − K12 x2 = 0

et

m 2 x¨2 + (K 2 + K 12 ) x2 − K12 x 1 = 0

en posant K1 = m 1g/l, K 2 = m 2 g/l et K12 = K cb2/l 2. L’intérêt expérimental de ce montage sur le précédent est la possibilité qu’il offre de modifier aisément le coefficient de couplage K12 en faisant varier la distance d’attache b . O1 y g

O2 y l B1

K12

u1

l B2 u2

A1

A2 F IG . 27.2.

I . 3 . — Modes propres de vibration des oscillateurs couplés On appelle modes propres de vibration d’un ensemble d’oscillateurs couplés les mouvements harmoniques avec la même pulsation propre V de tous les oscillateurs.

463

Oscillateurs couplés. Cas de N oscillateurs identiques

Ainsi, pour le système précédent des deux oscillateurs couplés, les modes propres sont les solutions harmoniques particulières des deux équations du mouvement : x1 (t) = x 1,m cos(V t − f1)

et x 2(t) = x2,m cos(V t − f2)

Afin d’éviter une confusion entre les indices portant sur le nombre d’oscillateurs et ceux permettant de distinguer les différents modes de vibration, qui sont en même nombre, nous adoptons dans la suite la nouvelle notation suivante dans laquelle l’indice supérieur entre parenthèses donne l’oscillateur et l’indice inférieur le mode propre de vibration. On écrira ainsi, en notation complexe, dans laquelle q = 1 ou 2 : x (q)(t) = Re{x (q)(t)}

x(q) (t) = x(mq) exp(jV t)

avec

et x (mq) = x (mq) exp(jf (q))

a) Recherche des pulsations propres En injectant ce type de solution dans les équations différentielles précédentes, on obtient le système linéaire suivant de deux équations algébriques : (−m1 V2 + K1 + K 12) x(m1) − K 12 x(m2) = 0

−K12 x(m1) + (−m2 V2 + K 2 + K12 ) x(m2) = 0 (1)

(2)

Les solutions ne présentent d’intérêt que si les amplitudes complexes xm et xm ne sont pas nulles ; aussi le déterminant de leurs coefficients doit-il être nul, ce qui fournit l’équation caractéristique suivante : 2 (−m 1V 2 + K 1 + K 12 )(−m 2V 2 + K2 + K12 ) − K12 = 0 soit V4 + bV2 + c = 0

qui est l’équation canonique du deuxième degré en V2 , dans laquelle : b=−

K 1 + K12 K 2 + K12 + m1 m2

et c = (K1 + K 12)(K2 + K12 ) −

K212 K1 K2 + K1 K12 + K2K 12 = m1 m2 m1 m 2

 c Dunod – Toute reproduction non autorisée est un délit

Cette équation admet généralement deux racines réelles et positives V2a et V 2b et donc deux pulsations propres du système des deux oscillateurs couplés. Remarque : Évidemment, en imposant K12 = 0 (pas de ressort intermédiaire) dans l’équation aux valeurs propres précédente, on obtiendrait : V4 − V 2

K1 K2 + m1 m2

+

K1 K2 =0 m1 m2

de solution évidente Va = (K 1/m1) 1/2 = v1 et Vb = (K2 /m2 ) 1/2 = v 2 . Les deux solutions de l’équation précédente du deuxième degré en V 2 s’écrivent : V2 = −

4c b 1± 1− 2 2 b

1/2

soit

V2a = V 20(1 − x)

et V 2b = V20 (1 + x)

V20 étant la moyenne arithmétique de ces solutions et x , appelé facteur de couplage, leur demi-écart relatif. Explicitement : V20 =

V 2a + V2b 1 b =− = 2 2 2

K 1 + K 12 K 2 + K12 + m1 m2

et x =

V2b − V2a 4c = 1− 2 2 2V0 b

1/2

464

27. Oscillateurs couplés. Cas de N oscillateurs identiques

Sur la figure 27.3, on a représenté la variation des carrés des pulsations en fonction du couplage, pour une pulsation V0 fixée. Les carrés des deux pulsations propres, égaux à V20 pour un couplage nul, s’écartent linéairement et symétriquement lorsque le couplage augmente. On dit que le couplage lève une dégénérescence des carrés des pulsations propres. V2

V 2b

V20

V2a x F IG . 27.3. — Variation des carrés des pulsations propres en fonction du couplage

I . 4 . — Nature des solutions La solution générale du système différentiel est une combinaison linéaire de solutions complexes de la forme exp(jVat), exp(−jV a t), exp(jVb t) et exp(−jVb t). Par conséquent, elle peut s’écrire : 1) 1) cos(V at + fa ) + x (m,b cos(Vb t + f b) x(1) = x(m,a

2) 2) et x (2) = x(m,a cos(Vat + f a ) + x(m,b cos(Vb t + fb)

l’indice supérieur (1) ou (2) étant relatif à la masselotte considérée et l’indice inférieur a ou b au mode propre. Les amplitudes ne sont pas indépendantes, puisque leur rapport change, avec la pulsation propre, selon le système algébrique précédent : 2) x(m,a (1)

xm,a

−m1 V2a + K1 + K 12 = ca = K 12

(2)

et

x m,b (1)

x m,b

= cb =

−m2 V2b + K 2 + K12 K 12

Finalement, x(1) et x (2) ont pour expressions respectives : et

(1) 1) x(1) = x (m,a cos(V at + fa ) + xm,b cos(V b t + fb) (2) 2) x(2) = x (m,a cos(V at + fa ) + xm,b cos(V b t + fb)

Les conditions initiales sur les positions et les vitesses, au nombre de quatre, permettent de déterminer (1) (2) les constantes am,a , xm,b , fa et fb . I . 5 . — Détermination des pulsations propres de vibration Pour trouver les modes normaux de vibration d’un système, il faut donc, successivement, injecter (1) (2) dans les équations différentielles du mouvement les solutions de la forme x m exp(jVt) et xm exp(jVt), (1) (2) afin d’obtenir un système d’équations algébriques en xm et x m , puis annuler le déterminant de la matrice formée par les coefficients de x(m1) et x (m2) . Donnons quelques exemples. a) Deux masses reliées par un ressort Dans le cas de deux masses différentes reliées par un ressort (Fig. 27.4a), les équations différentielles s’obtiennent en faisant dans les équations initiales K 1 = K2 = 0 et K 12 = K : m1¨ x(1) = −K (x(1) − x (2) )

et

x (2) = −K (x(2) − x (1)) m 2¨

465

Oscillateurs couplés. Cas de N oscillateurs identiques Les équations algébriques obtenues en injectant les solutions harmoniques donnent : (−m1V 2 + K ) x(m1) − K x(m2) = 0

−K x (m1) + (−m2V 2 + K ) x(m2) = 0 m1

m2

K

m

m

K

a)

b) F IG . 27.4.

L’équation caractéristique du deuxième degré s’en déduit aisément : V 2 V2 − K

1 1 + m1 m2

=0

Les deux pulsations propres sont donc : 1 1 + V a = 0 et Vb = K m1 m2

1/2

La pulsation Va correspond au mouvement en phase de l’ensemble : elle est nulle, car, le système n’étant soumis à aucune force extérieure horizontale, un tel mouvement est exclu. Elle correspond à une accélération nulle puisque cette dernière apparaît sous la forme de −V2x (1) ou −V 2 x(2) . Comme, pour cette pulsation, on a : ¨x = 0 d’où ˙x = v et x = vt + b la solution générale du système s’écrit :

 c Dunod – Toute reproduction non autorisée est un délit

1) cos(Vbt + f b ) x(1) = v1 t + b 1 + x(m,b

1) et x(2) = ca(v1 t + b 1) + c b x (m,b cos(Vbt + f b)

La pulsation Vb correspond à des mouvements des masses en opposition de phase. On peut la déterminer à partir du nœud de vibration qui est situé en un point du ressort tel que les pulsations associées aux deux portions de ressort soient égales : K K  = m1 m2 K et K  désignant les raideurs correspondantes. Comme ces raideurs sont dans le rapport m 1/m 2, les longueurs correspondantes, l et l , doivent être dans le rapport m2/m 1. On a donc l = m 2 l/(m1 + m2) et l  = m1 l/(m1 + m2), d’où : K =

m1 K m1 + m 2

et K  =

m2 K m1 + m 2

Si m1 = m 2 = m, V b = (2K /m)1/2 (Fig. 27.4b). Le point immobile est le milieu du ressort. Le mouvement harmonique de pulsation Vb est alors celui d’une masse m/2 accrochée à un ressort de raideur K. Ce résultat était prévisible puisque, dans le référentiel du centre de masse, ce problème à deux corps se ramène à celui de la masse réduite m = m/2.

466

27. Oscillateurs couplés. Cas de N oscillateurs identiques

Cet exemple peut illustrer les vibrations des molécules diatomiques, formées d’atomes identiques, comme celles du dioxygène O 2 . À partir de la fréquence d’oscillation, f 0 = 0, 4655 × 10 14 Hz, que l’on obtient par spectroscopie (cf. Quantique), on trouve pour la constante de couplage, puisque m = MO/N A, NA étant le nombre d’Avogadro et MO = 16 g la masse d’une mole d’atomes de dioxygène : 16 × 10 −3 f02 MO f02 2 2 K = 4p × = 4p × ≈ 1 137 N.m −1 2NA NA

C

O1

O2

u1

u2 F IG . 27.5.

b) Volants couplés par une tige de torsion Deux volants, de moments d’inertie respectifs I 1 et I2, sont réunis sur un arbre dont la constante de torsion est C (Fig. 27.5). Pour déterminer les pulsations propres, établissons les équations différentielles des mouvements de rotation autour de leur axe commun. Il vient, en appliquant le théorème du moment cinétique en projection sur l’axe de rotation, les liaisons étant parfaites : u1 = −C(u 1 − u 2) I 1¨

et I2 ¨ u2 = −C(u2 − u 1)

I 1 et I 2 étant les moments d’inertie. La recherche des solutions en exp(jVt) fournit les deux équations algébriques : (−I1V 2 + C)u (m1) − Cu (m2) = 0 et − Cu (m1) + (−I2 V2 + C)u(m2) = 0 On obtient l’équation caractéristique suivante : (−I 1V2 + C)(−I2 V2 + C) − C 2 = 0 soit V2[I 1I 2V 2 − C(I1 + I2 )] = 0 d’où les pulsations propres : V a = 0 et

Vb =

C(I 1 + I 2 ) I1 I2

1/2

La valeur nulle était prévisible, puisque, le système étant soumis à un moment de force total nul selon l’axe de rotation, les deux volants ne peuvent osciller en phase. I . 6 . — Étude d’un système symétrique Le cas d’un système symétrique est très important car le couplage des deux oscillateurs est alors maximal. a) Pulsations propres L’équation caractéristique des deux oscillateurs élastiques couplés devient, pour m 1 = m 2 = m et K1 = K 2 = K : K + K 12 K K + 2K12 + =0 V 4 − 2V 2 m m m

467

Oscillateurs couplés. Cas de N oscillateurs identiques d’où les racines évidentes : V 2a =

K m

et V2b =

K + 2K 12 m

On en déduit les expressions suivantes de la moyenne des carrés des pulsations propres et du facteur de couplage : V 2a + V2b V 2b − V2a K + K12 K12 2 et x = V0 = = = 2 2 m K + K 12 2V0 b) Élongations On obtient alors les élongations en déterminant préalablement les facteurs C a et C b . Il vient, d’après ce qui précède : −V2b m + K + K12 −V2a m + K + K12 Ca = = 1 et Cb = = −1 K12 K12 d’où : x(1) = xm,a cos(V at + fa ) + x m,b cos(Vb t + f b)

et x (2) = xm,a cos(Vat + f a ) − xm,b cos(Vb t + fb)

Supposons qu’initialement x (1) = 2a, x(2) = 0, x˙ (1) = 0, x˙ (2) = 0. On obtient : 2a = x m,a cos f a + xm,b cos fb 0 = −xm,a Va sin fa − x m,bVb sin f b

0 = x m,a cos fa − xm,b cos fb 0 = −xm,a V a sin f a + x m,bVb sin fb

ce qui donne fa = 0, fb = 0, x m,a = a, x m,b = a. La solution précise est donc, dans ce cas : x (1) = a cos(Va t) + a cos(Vb t)

x(2) = a cos(V at) − a cos(Vb t)

et

Ainsi, les mouvements des masselottes ne sont pas harmoniques : ce sont des combinaisons simples de deux mouvements harmoniques (Fig. 27.6).

 c Dunod – Toute reproduction non autorisée est un délit

Notons cependant que leur demi-somme (x (1) + x (2))/2 et leur demi-différence (x(1) − x (2) )/2 évoluent, elles, harmoniquement avec les pulsations Va et V b , appelées pulsations propres : x(1) + x(2) = a cos(V at) 2

x (1)

et

x (1) − x(2) = a cos(Vb t) 2

x(2)

t

F IG . 27.6.

t

468

27. Oscillateurs couplés. Cas de N oscillateurs identiques

c) Excitation des deux modes propres Des conditions initiales particulières permettent de ne sélectionner que l’une des solutions harmoniques. Montrons-le sur l’exemple simple précédent. i) Réalisation du mode 1 Écartons les masselottes de la même distance a et dans le même sens, et abandonnons-les. Il vient, d’après ce qui précède : a = x m,a cos f a + xm,b cos fb

a = xm,a cos f a − xm,b cos fb

0 = −Va xm,a sin fa − Vb x m,b sin f b

0 = −Va x m,a sin f a + Vb xm,b sin fb

On en déduit que fa = 0, fb = 0, xm,a = a, xm,b = 0 ; finalement : x(1) = a cos(Vat)

x (2) = a cos(V at)

et

Ainsi, l’ensemble vibre, de façon harmonique, à la pulsation Va . On dit que ces conditions initiales excitent la première pulsation propre V a. Ce mode étant symétrique, nous le désignerons dans la suite par Vs . ii) Réalisation du mode 2 Écartons les masselottes de la même distance a mais en opposition, et abandonnons-les. Il vient : a = xm,a cos fa + x m,b cos fb

−a = x m,a cos f a − xm,b cos fb

0 = −Va xm,a sin fa − V bx m,b sin fb

0 = −V axm,a sin f a + Vbxm,b sin f b

On en déduit que fa = 0, fb = 0, am,a = 0, am,b = a et : x(1) = a cos(Vb t)

x(2) = −a cos(Vb t)

et

Ainsi excité par ces conditions initiales, l’ensemble vibre de façon harmonique à la pulsation V b . Ce mode étant antisymétrique, nous le désignerons dans la suite par V a . Notons que, si le système vibre dans le mode 1, on pourrait, sans perturber le mouvement, remplacer le ressort assurant l’interaction par une tige rigide. De la même façon, si le système vibre dans le mode 2, le point milieu est fixe ; on pourrait sans perturbation fixer le point milieu. Chacune des masselottes est soumise à l’action d’un ressort de raideur K et à celle d’un « demi-ressort » de raideur 2K12 . Remarque : Ce dernier résultat, suivant lequel on multiplie par 2 la raideur d’un ressort en le coupant en deux, peut être établi de la façon suivante : on considère deux ressorts, de raideurs K1 et K2 , reliés entre eux l’un à la suite de l’autre, et on fait subir à l’extrémité du second une certaine tension ; cette tension s’exerce aussi à l’extrémité du premier alors que les allongements x(1) et x (2) s’ajoutent : T = K 1x(1) = K2 x(2)

d’où

x = x (1) + x(2) =

T T =T + K 1 K2

On en déduit que la raideur équivalente K eq de l’ensemble est telle que : 1 1 1 = + Keq K1 K2

soit

K eq =

K1 K 2 K1 + K2

Comme dans ce cas K1 = K2 = K1/2 , alors K1/2 = 2K eq.

1 1 + K1 K2

469

Oscillateurs couplés. Cas de N oscillateurs identiques

II . — COORDONNÉES NORMALES Lorsqu’un système est constitué de deux ou plus généralement N oscillateurs harmoniques couplés, on facilite l’étude de l’ensemble en introduisant un ensemble particulier de coordonnées pour lequel le système se comporte comme deux ou N oscillateurs harmoniques indépendants. Précisément, les coordonnées normales d’un système de N oscillateurs couplés sont les N coordonnées {Qi } telles que l’énergie mécanique du système se mette sous une forme canonique caractéristique de l’énergie d’un ensemble de N oscillateurs harmoniques indépendants, dont les pulsations sont les pulsations normales. On a donc, pour de tels systèmes, les équations génériques suivantes : ˙ 2i Q 1 ¨i + V2i Q i = 0 ou d Q + V 2i Q2i = 0 dt 2 2 ˙ i . Il en résulte que l’énergie mécanique en multipliant les deux membres de la première équation par Q d’un tel système a pour expression : N

Em =

i=1

1 2 1 2 2 Q˙ + Vi Qi 2 i 2

Ainsi, pour un système à deux degrés de liberté, les coordonnées normales Q 1 et Q2 sont telles que : 1 1 1 1 Em = Q˙ 21 + V 21Q21 + Q˙ 22 + V22 Q22 2 2 2 2 Remarque : Ainsi définies, les coordonnées {Qi } sont obtenues à un facteur multiplicatif près. II . 1 . — Système symétrique

 c Dunod – Toute reproduction non autorisée est un délit

Pour le système symétrique étudié précédemment, la solution était évidente ; en effet, comme x (1) et x(2) valent respectivement : et x(2) = a cos(V st) − a cos(Va t) x(1) = a cos(Vs t) + a cos(V at) la demi-somme et la demi-différence sont deux nouvelles coordonnées qui oscillent harmoniquement : x(1) + x (2) x(1) − x(2) q1 = = a cos(V st) et q2 = = a cos(V at) 2 2 Pour trouver de façon systématique les coordonnées normales, exprimons l’énergie en fonction de q1 et q2 . Il vient, en remplaçant, dans l’expression des énergies cinétique et potentielle Ek et E p , x(1) et x(2) respectivement par q1 + q2 et q 1 − q2 : 1 1 Ek = m( ˙x(1)) 2 + m(˙x(2) )2 = m( ˙q21 + q˙ 22) 2 2 et 1 1 1 Ep = K (x (1))2 + K (x(2))2 + K12 (x(1) − x (2)) 2 = Kq21 + Kq22 + 2K12 q22 2 2 2 Il en résulte, en introduisant les carrés des deux pulsations propres d’expressions respectives V 21 = K /m et V22 = (K + 2K 12)/m : 2

Em =

2

m q˙ 2i +

mV2i q2i

ce qui s’écrit

i=1

Em =

i=1

1 ˙2 1 2 2 Q + V i Qi 2 i 2

en fonction des coordonnées normales Q1 et Q2 , lesquelles sont reliées à q1 et q2 par : Q 1 = (2m) 1/2 q1 =

m 2

1/2

(x(1) + x (2))

et Q2 = (2m)1/2 q2 =

m 2

1/2

(x(1) − x(2) )

470

27. Oscillateurs couplés. Cas de N oscillateurs identiques

Réciproquement : x(1) =

1/2

1 2m

et x (2) =

(Q1 + Q 2)

1 2m

1/2

(Q1 − Q2 )

II . 2 . — Système quelconque Pour un système quelconque, qui ne présente donc aucune symétrie particulière, une écriture matricielle des équations du mouvement s’impose techniquement lorsqu’on cherche à déterminer les coordonnées normales. a) Écriture matricielle des équations du mouvement Les équations différentielles du mouvement du système non symétrique à deux degrés de liberté peuvent se mettre sous une forme matricielle, plus condensée : [¨ X] = −[M][X ]

avec

[X ] =

x(1) x(2)

et [M] =

(K 1 + K12 )/m 1 −K12 /m2

−K12 /m 1 (K2 + K12 )/m2

La recherche de solutions en exp(jVt) , telle que : [X] = [X m] exp(jVt)

avec

x (m1) (2) xm

[X m ] =

donne, en introduisant la matrice identité [I ] : ([M] − V 2[I])[X m ] = [0]

ce qui nécessite

det ([M] − V2 ) = 0

puisque [X ]m = [0] . On obtient donc, sous une forme concise, l’équation caractéristique donnant les deux pulsations propres. On en déduit alors la relation entre x(m1) et x (m2) , pour chacune des pulsations propres, à l’aide de l’équation : (K1 + K12 )/m 1 − V2 −K12 /m2

−K12 /m1 (K2 + K12 )/m2 − V2

(1)

0 xm (2) = 0 xm

b) Traitement matriciel du cas symétrique L’application de l’analyse matricielle précédente au système symétrique donne : det ([M] − V 2[I]) = 0 avec

(K + K12 )/m − V2 −K 12/m

−K 12 /m (K + K12 )/m − V2

On trouve alors l’équation du deuxième degré V4 +b V2 + c = 0 , et donc les deux carrés des pulsations propres : V2 = V20(1 ∓ x)

avec

V20 = −

b K + K12 = 2 m

et x = 1 −

4c b2

1/2

=

K12 K + K12

Par exemple, pour K = K0 , K 12 = 2K0 , m = m0 et K 0 /m0 = 100 rad2 .s−2 , on trouve, dans cette même unité SI, puisque le facteur de couplage x vaut 2/3 : V20 = 3

K0 = 300 m0

V21 = 300 ×

1 = 100 et 3

V22 = 300 ×

5 = 500 3

471

Oscillateurs couplés. Cas de N oscillateurs identiques Il en résulte : V1 = 10 rad.s−1

V1 ≈ 22, 4 rad.s−1

2p ≈ 0, 63 s V1

et T1 =

2p ≈ 0, 28 s V2

T2 =

Cherchons les vecteurs propres, précisément les vecteurs tels que : [M][u] = V2 [u] [u] étant la matrice colonne formée par les deux composantes du vecteur propre u dans la base initiale. Il vient, en explicitant : (K + K 12 )/m −K 12 /m

(1)

−K12 /m (K + K12)/m

(1)

u1 2 u1 (2) = V1 u1 u(12)

d’où la relation suivante entre les deux composantes de u1 : u(12)

K + K12 (1) K12 (2) K u1 − u1 = −V 21 u(11) = u (11) d’où m m m

(1) u1

K 12/m =1 K 12/m

=

De la même manière, on trouve pour les composantes de u2 : (2)

K + 2K 12 (1) K 12 (1) K + K 12 (2) (1) − u2 + u2 = V22 u2 = u2 m m m

u2

d’où

u(21)

=

K12 /m = −1 −K12 /m

Dans ce cas symétrique, la relation établie précédemment entre les coordonnées normales {Q 1, Q2 } et les coordonnées initiales {x(1) , x(2) } s’écrit matriciellement :

 c Dunod – Toute reproduction non autorisée est un délit

[X] = [P][Q]

avec

[Q] =

Q1 Q2

1 2m

et [P] =

1/2

1 1

1 −1

[X] étant la matrice colonne des coordonnées initiales, [Q] celle des coordonnées normales Q1 et Q2 , et [P] la matrice de passage. Notons que cette dernière s’obtient, au coefficient multiplicatif (2m) −1/2 près, en juxtaposant les matrices colonnes des deux vecteurs propres. On retrouve, en explicitant : (1)

x

=

1 2m

1/2

(Q1 + Q 2)

et x

(2)

1 = 2m

1/2

(Q1 − Q2 )

On passe évidemment des coordonnées normales [X ] aux coordonnées initiales [Q] en inversant l’équation précédente : −1

[Q] = [P] [X]



−1

[P]

2m = −2

1 2m

1/2

−1 −1

m −1 = 1 2

−1/2

En explicitant, on retrouve : Q1 =

m 2

1/2

(x(1) + x (2))

et Q2 =

m 2

1/2

(x(1) − x (2) )

1 1

1 −1

472

27. Oscillateurs couplés. Cas de N oscillateurs identiques

c) Traitement matriciel d’un système non symétrique Pour un système non symétrique, les carrés des pulsations propres sont solutions de l’équation du deuxième degré : V 4 + b V 2 + c = 0 avec b = − On trouve :

b V21 = − (1 − x) 2

K 1 + K 12 K2 + K12 + m1 m2

et c =

b V22 = − (1 + x) 2

x = 1−

avec

K1K 2 + K 1K 12 + K 2K 12 m 1 m2 4c b2

1/2

Dans le cas concret suivant, pour lequel : K1 =

K12 K2 = K0 = 2 3

m1 =

m2 = m0 2

et

11 2

K0 m0

K0 = 100 rad2.s−2 m0

en unité SI. On trouve : b=−

11 K 0 = 550 et 2 m0

c=

2

= 55 000

d’où la valeur du facteur de couplage x ≈ 0, 52 . On en déduit, en unités SI : V20 = −

b = 275 2

V 21 ≈ 131, 38 et

V 22 ≈ 418, 62

Les pulsations propres valent donc : V1 ≈ 11, 46 rad.s−1 et V 2 ≈ 20, 46 rad.s −1 ce qui correspond aux périodes T 1 ≈ 0, 55 s et T2 ≈ 0, 31 s . Le calcul de la matrice [M] donne, en unités SI : [M] =

300 −200 (K1 + K12 )/m 1 −K12 /m1 = −100 250 −K12 /m2 (K 2 + K 12)/m 2

Notons que la trace de la matrice M , ou somme de ces éléments diagonaux, est égale à la somme des valeurs des carrés des pulsations propres. On détermine les vecteurs propres selon : [M] =

300 −200 −100 250

(1) u(1) 2 u (2) = V 1 u u (2)

On obtient alors la relation suivante entre les deux composantes de u 1 : u (12) (1) u1

=

300 − 131, 38 168, 62 = 200 200

et

u(22) (1) u2

=

100 100 = 250 − 418, 62 −168, 62

Matricielllement, on a la relation suivante donnant [X] à partir de [Q] , [P] étant la matrice carrée formée à partir des deux vecteurs propres : [X] = [P][Q]

avec

[P] ≈

2 1, 68

−1, 68 1

473

Oscillateurs couplés. Cas de N oscillateurs identiques en divisant tous les éléments de cette matrice par 102 et en arrondissant. Il en résulte : [Q] = [P]−1[X]

avec

[P] −1 =

1 1 4, 82 −1, 68

1, 68 2

ce qui s’explicite selon : Q1 ≈

1 (x (1) + 1, 68 x(2) ) 4, 82

et Q2 ≈

1 (−1, 68 x(1) + 2 x (2)) 4, 82

On obtient aisément les équations donnant les coordonnées x(1) et x (2) à partir de [X] = [P][Q] : x(1) ≈ 2 Q 1 − 1, 68 Q2

et x(2) ≈ 1, 68 Q 1 + Q 2

Notons que l’on a, puisque [X] = [P][Q] et [ ¨ X] = −[M ][X] : ¨ [X¨] = −[M ][P][Q] = [P][ Q]

d’où

[Q¨] = −[P]−1 [M][P][Q]

soit : ¨ ] = −[D][Q] [Q

avec

[D] = [P]−1 [M][P]

Dans le cas concret considéré, on vérifie bien que la matrice [D] est diagonale : 1 1 4, 82 −1, 68

1, 68 2

300 −200 −100 250

2 −1, 68 131 ≈ 1, 68 1 0

0 418

III . — MODES DE COUPLAGE DE DEUX OSCILLATEURS III . 1 . — Divers types de couplage a) Couplage élastique

 c Dunod – Toute reproduction non autorisée est un délit

Dans l’exemple précédent, le couplage était assuré par un ressort intermédiaire. On dit que ces deux oscillateurs sont associés par un couplage élastique. b) Couplage par inertie Dans le couplage par inertie, les deux oscillateurs interagissent par l’intermédiaire d’une masse ; c’est le cas du pendule double pesant (Fig. 27.7). Les équations différentielles de ce système sont les suivantes (cf. Exercices) : ¨ u(2) g g ¨ ¨ u(1) + u(2) + ¨ u(1) + u(2) = 0 + u(1) = 0 et l l 2 c) Couplage par frottement Sur la figure 27.8, on a représenté un système de deux oscillateurs élastiques couplés par un amortisseur. Les équations différentielles du système sont les suivantes (cf. Exercices) : x(1) 1 + ¨

K a (1) (˙x − x˙ (2)) + x (1) = 0 m m

et

x(2) + ¨

K a (2) (˙x − x˙ (1)) + x(2) = 0 m m

474

27. Oscillateurs couplés. Cas de N oscillateurs identiques y

O l µ (1)

g

A 1(m 1) l µ

(2)

A1 (m1 )

A2 (m2) K

x

x(1)

A 2 (m2 ) K

x(2) F IG . 27.8.

F IG . 27.7.

III . 2 . — Analogie électrique On sait que le comportement électrique d’un circuit RLC est analogue au comportement mécanique d’un pendule élastique (cf. chapitres 10 et 11). Par conséquent, on peut prévoir, pour des systèmes électriques couplés, des équations analogues à celles des systèmes mécaniques (cf. Électronique). Dans le premier circuit (Fig. 27.9a) le couplage est du type capacitif : le système est équivalent aux deux oscillateurs élastiques reliés par un ressort. Dans le second (Fig. 27.9b), il est du type inductif, M étant un coefficient d’inductance algébrique : le système est un équivalent électrique du pendule double. L

C i1

R

A C i2 C12 B

i1 C

L R

R

M i C 2 L

a)

L

R

b) F IG . 27.9.

IV . — MOUVEMENT FORCÉ D’UN SYSTÈME DE DEUX OSCILLATEURS Exerçons sur un système de deux oscillateurs harmoniques couplés, tel que celui étudié en I, la force supplémentaire sinusoïdale suivante appliquée en A1 (Fig. 27.10a) : F1(t) = F m cos(vt) ex. A1

F 1(t)

A2

F1 (t) A 2

A1 x

K1

m1

K12

m2

x

K2

K1

m1

a)

K 12

m2

b) F IG . 27.10.

IV . 1 . — Équations différentielles Les équations différentielles s’obtiennent aisément ; il suffit d’ajouter aux équations de base initiales un second membre à la première équation : m1¨ x(1) = −K 1x (1) − K12 (x(1) − x(2) ) + Fm cos(vt)

et

m 2¨ x(2) = −K 2x (2) − K12 (x(2) − x(1) )

et

¨x(2) = −

soit, en divisant par les masses : x(1) = − ¨

K1 K12 + m1 m1

x (1) +

K12 (2) Fm x + cos(vt) m1 m1

K 2 K12 + m2 m2

x(2) +

K12 (1) x m2

475

Oscillateurs couplés. Cas de N oscillateurs identiques

En réalité, il faudrait ajouter à ces équations un terme d’amortissement qui impose une solution forcée sinusoïdale de pulsation v, comme pour les systèmes à un degré de liberté (cf. chapitre 11). L’absence de terme d’amortissement se traduit par une amplitude des oscillations infiniment grande. IV . 2 . — Amplitudes complexes Comme pour les systèmes forcés à un degré de liberté, cherchons des solutions complexes de la forme : et x(2) = x (m2) exp(jvt) x(1) = x(m1) exp(jvt) x(m1) et x(m2) désignant les amplitudes des élongations. Ce système d’équations différentielles se transforme en un système d’équations algébriques : K1 K12 K Fm K K2 K12 et − 12 x(m1) + −v 2 + x(m1) − 12 x(m2) = x(m2) = 0 −v 2 + + + m1 m1 m1 m1 m2 m2 m2 D’où : −v2 + K 2/m2 + K12/m2 Fm/m 1 x(m1) = (−v2 + K1 /m1 + K12 /m 1) (−v 2 + K 2/m 2 + K12 /m2 ) − K 212 /(m 1m 2) et

x(m2) =

(K12 /m2 ) Fm /m1 (−v2 + K1 /m1 + K12 /m 1) (−v 2 + K 2/m 2 + K12 /m2 ) − K 212 /(m 1m 2) (1)

(2)

Dans ce cas où on a négligé les frottements, les amplitudes x m et xm sont réelles et deviennent infinies lorsque la pulsation d’excitation v est égale à l’une des deux pulsations propres V a et Vb . C’est le phénomène connu de résonance (cf. chapitre 11). IV . 3 . — Absorbeur de vibration

 c Dunod – Toute reproduction non autorisée est un délit

Dans le système précédent, supprimons le troisième ressort de raideur K 2 (Fig. 27.10b). Comme (1) m1 = m2 = m, et K 2 = 0, l’amplitude complexe x m se réduit à : −v2 + K12/m 2 Fm /m1 xm(1) = −v2 + K1 /m1 + K12 /m1 −v2 + K12 /m2 − K 212 /(m1 m 2) On en déduit que : x(m1) = 0

pour

v=

K12 m2

1/2

Ainsi, il est possible d’étouffer les vibrations d’un oscillateur, excité par une force sinusoïdale de pulsation v, en lui adjoignant un second oscillateur dont la pulsation propre coïncide avec la pulsation excitatrice. Ce résultat est souvent utilisé en pratique.

V . — COUPLAGE DE N OSCILLATEURS IDENTIQUES V . 1 . — Équations différentielles du mouvement Considérons un système formant une chaîne horizontale de N masselottes identiques {A n}, dont les positions d’équilibre sont régulièrement espacées de la distance d (Fig. 27.11) ; ces masselottes de masse m interagissent par des ressorts identiques de raideur K. Supposons en outre que les extrémités A0 et AN+1 de la chaîne soient fixées. Ce système présente de l’intérêt car il schématise bien l’arrangement et le comportement des atomes suivant une direction dans un solide.

476

27. Oscillateurs couplés. Cas de N oscillateurs identiques

Désignons désormais par x (n) = nd, la position de la n e masselotte au repos, d étant la distance qui sépare deux masselottes consécutives. On appelle alors C (n) le déplacement longitudinal de la masselotte An, par rapport à sa position de repos. Ω 2!0

A0

A1 K

An−1 K

An K

x(n1)

An+1 K

AN K

AN+1 kd

K

0

−¼

x(n) x(n+1) F IG . 27.11.

¼

F IG . 27.12.

Appliquons la loi fondamentale de la dynamique à cette masselotte. Il vient, en projection suivant la direction de la chaîne : ¨ (n) = −K (C(n) − C(n−1) ) − K (C(n) − C(n+1)) mC

¨ (n) + 2v 2C (n) − v 2(C(n−1) + C (n+1)) = 0 C 0 0

soit

en divisant par m et en introduisant v20 = K /m. Une manière de déterminer les modes propres de vibration consisterait à chercher, comme pour les systèmes à deux degrés de liberté, des solutions de la forme exp(jVt) et à résoudre le système des N équations algébriques qui en résulte. Cependant la résolution de ce système d’équations différentielles n’est pas aisée, car la méthode s’avère vite difficile à mettre en œuvre dès que le nombre d’oscillateurs dépasse quelques unités. Aussi est-il judicieux de traiter d’abord le cas d’un nombre infini de masselottes très proches les unes des autres, ce qui permet de retrouver une équation différentielle caractéristique dont la solution est bien connue. V . 2 . — Extension au cas continu : N infini L’équation différentielle à laquelle satisfait C (n) est donc : ¨ (n) − v20 (C(n+1) − C(n) ) + v 20(C(n) − C(n−1) ) = 0 C

Le système étant continu, nous pouvons remplacer C(n) (t) par la fonction C(x, t). Puisque la distance d est très faible, on a : C(n+1) − C(n) ∂ C(x, t) ≈ d ∂x

C (n) − C(n−1) ∂ C(x, t) ≈ d ∂x

et n

n−1

¨ (n) par ∂ 2 C(x, t)/∂t2, on obtient l’approximation suivante : Par conséquent, en remplaçant C ∂2C(x, t) − v 20 d ∂t 2 Comme :

1 d

∂ C(x, t) ∂x

∂ C(x, t) ∂x

n



n



∂ C(x, t) ∂x

∂ C(x, t) ∂x

n−1



=0 n−1

∂2 C(x, t) ∂x2

il vient finalement : 2 ∂ 2 C(x, t) 2 2 ∂ C(x, t) = 0 soit d − v 0 ∂t2 ∂x2

1 ∂2C(x, t) ∂2C(x, t) = 2 ∂x2 ∂t 2 v

477

Oscillateurs couplés. Cas de N oscillateurs identiques

en posant v = v0 d, puisque v0 d a les dimensions d’une vitesse. Cette équation différentielle est caractéristique de la propagation d’une onde le long de la chaîne. On sait que la solution a pour expression (cf. Optique) : x x + C− t + C(x, t) = C+ t − v v C+ et C− étant deux fonctions quelconques des variables (t − x/v) et (t + x/v). Si la fonction est sinusoïdale, la solution est dite harmonique ou monochromatique. On a donc, en notation complexe, des fonctions de la forme : C(x, t) = C exp jV t ∓

x v

= C exp(jVt) exp(∓jkx)

où k = V/v, appelé nombre d’onde, a la dimension de l’inverse d’une longueur. V . 3 . — Modes propres de vibration d’un ensemble de N oscillateurs identiques Le résultat précédent suggère de chercher des solutions harmoniques de l’équation différentielle : ¨ (n) + 2v 2C(n) − v2 (C (n−1) + C(n+1)) = 0 C 0

0

qui soit de la forme suivante :

C(n) = C exp(jknd) exp(jVt) Il vient, en injectant ce type de solution dans l’équation précédente : −V2 + 2v20 − v20 [exp(−jkd) + exp(jkd)] = −V2 + 2v20[1 − cos(kd)] = −V 2 + 4v20 sin2

kd 2

=0

On en déduit la relation suivante entre la pulsation propre V et la grandeur k introduite : V = 2v0 sin

kd 2

La courbe V(k) est la courbe de dispersion (Fig. 27.12) ; ainsi, à une valeur V de la pulsation propre correspondent deux valeurs k et −k. La solution générale C (n) se met donc sous la forme : (n)

 c Dunod – Toute reproduction non autorisée est un délit

C

= A exp(jVt) exp(jknd) + B exp(jVt) exp(−jknd)

Comme les extrémités sont fixées : C0 = 0 et C(N+1) = 0, quel que soit t, il vient : C(0) (0) = (A + B) = 0

et

ce qui implique : A+B = 0

et

C(N+1) (0) = A exp[jk(N + 1) d] + B exp[−jk(N + 1) d] = 0

A exp[jk(N + 1) d] + B exp[−jk(N + 1) d] = 2jA sin[k(N + 1) d] = 0

Les valeurs de k, et par conséquent celles de V, qui conviennent sont donc, k étant un nombre entier : kk = k

p (N + 1) d

et

Vk = 2v 0 sin k

p 2(N + 1)

Finalement, comme B = −A, C(n) a pour expression générale, en posant a m = 2jA : (n)

C

N

am sin

= k=1

nkp N+1

exp(jVk t)

478

27. Oscillateurs couplés. Cas de N oscillateurs identiques

V . 4 . — Application à deux oscillateurs identiques En imposant N = 2 dans les expressions précédentes, on trouve : V a = 2v 0 sin

p 6

et Vb = 2v 0 sin

= v0

√ p = v0 3 3

ce qui correspond bien aux valeurs précédemment trouvées lorsque K1 = K2 = K12 = K. Les solutions sont alors : 2 nkp (n) am sin = exp(jV k t) C 3 k=1

On voit que, dans le mode a : C(a1)

p = a m sin 3

et C(a2)

exp(jVa t) = am



3 exp(jV at) 2

= a m sin

2p 3

√ 3 exp(jV a t) = am exp(jVa t) 2

= am sin

2p 3

√ 3 exp(jV b t) = am exp(jVb t) 2

alors que, dans le mode b : (1) Cb

et (2) Cb

= a m sin

4p 3

exp(jV b t) = −a m



3 exp(jV b t) 2

√ En passant aux notations réelles et en introduisant a = a m 3/2, on retrouve bien les solutions harmoniques de pulsations propres Va et Vb : C (a1) = C(a2) = a cos(Va t)

et C(b1) = −C(b2) = a cos(V b t)

CONCLUSION Rappelons les points suivants importants. (1) Les mouvements de deux oscillateurs en interaction, autour de leurs positions d’équilibre, ne sont pas harmoniques mais des combinaisons linéaires de deux mouvements harmoniques. (2) On détermine les deux pulsations propres V a et V a du système en annulant le déterminant des coefficients qui apparaissent dans le système d’équations algébriques issu de la recherche de solutions harmoniques. Elles sont solutions d’une équation du deuxième degré en V2 de la forme : V 4 + b V2 + c = 0

ce qui donne

V2a = V 20 (1 − x)

et V2b = V20 (1 + x)

où V20 = −b/2 est la moyenne des carrés des pulsations propres et x = (1 − 4c/b 2 )1/2 le facteur de couplage. (3) Pour un système symétrique, V a correspond à un mode de vibration propre symétrique, V b correspond à un mode de vibration propre antisymétrique. (4) La nature des mouvements suggère de rechercher de nouvelles coordonnées qui évoluent suivant des lois harmoniques indépendantes, appelées modes propres de vibration.

479

Oscillateurs couplés. Cas de N oscillateurs identiques

(5) Le nombre de modes propres est égal au nombre de degrés de liberté du système ; c’est ce que confirme l’analyse de l’ensemble de N oscillateurs identiques formant une chaîne périodique. (6) Si les oscillateurs couplés sont soumis à un régime forcé sinusoïdal, on observe, comme pour les oscillateurs à un degré de liberté, un phénomène de résonance pour des valeurs de la pulsation excitatrice égales aux pulsations propres du système. (7) Le système de N oscillateurs identiques en interaction joue un rôle essentiel en physique, car il constitue une représentation efficace d’un solide unidimensionnel. On détermine les modes propres de vibration en cherchant des solutions de la forme : C(n) = C exp(jknd) exp(jVt) n étant l’oscillateur de rang n et d la distance qui sépare deux oscillateurs successifs.

EXERCICES ET PROBLÈMES P27– 1. Système de poulie et ressorts On considère le système représenté sur la figure 27.13. La poulie a une masse m 1 , un rayon R et un moment d’inertie I par rapport à son axe de révolution. Le premier ressort a une raideur K1 et le second une raideur K2. 1. Le second pendule est enlevé. a) Écrire la condition d’équilibre de la poulie et calculer les tensions des fils en I et J. b) Exprimer que la poulie roule sans glisser sur le fil. En déduire la relation entre la vitesse du centre de la poulie et la vitesse de l’extrémité J du ressort. c) À l’aide des théorèmes généraux, montrer que le centre de masse de la poulie oscille et calculer la période des oscillations. 2. On suspend le second pendule de masse m 2.

 c Dunod – Toute reproduction non autorisée est un délit

a) Trouver les équations différentielles auxquelles satisfont les coordonnées x 1 et x2 des points et A 2 respectivement. A1 b) Quelles sont les pulsations propres du système, dans le cas où m 2 = m1 + I /R2 et K1/K 2 = 3/8 ? On donnera les résultats en fonction de v20 = K 2/m 2.

K1

g

O1

J

O2

b

K

A1

I

K2

l

l K

θ1 A2

F IG . 27.13.

(1) F IG . 27.14.

g

g

θ2 (2) F IG . 27.15.

480

27. Oscillateurs couplés. Cas de N oscillateurs identiques

P27– 2. Système de deux barres couplées par un ressort Deux barres identiques, de masse m, de longueur l , forment deux pendules pesants reliés en leur milieu par un ressort de raideur K (Fig. 27.14). Ces barres tournent sans frottement autour d’axes horizontaux. Initialement les angles d’inclinaison par rapport à la verticale sont u1 = 0 et u2 = u0 et les vitesses sont nulles. On pose v20 = g/l et v 2c = K /m. 1. Établir les équations différentielles des petits mouvements. 2 . Quelle est la nature du mouvement ? P27– 3. Pendule de Wilberforce Un pendule est constitué d’un ressort auquel est accroché un solide ayant la forme d’un cylindre rigidement lié à une tige horizontale (Fig. 27.15). L’ensemble, muni de deux masselottes destinées à faire varier le moment d’inertie par rapport à l’axe du mouvement, forme un solide homogène de masse m ayant la symétrie matérielle par rapport à l’axe vertical. Lorsqu’on écarte le pendule de sa position d’équilibre, l’oscillation de haut en bas est accompagnée d’une rotation latérale. On appelle K la raideur du ressort, C sa constante de torsion et R le rayon de giration. On se propose d’expliquer les oscillations observées en introduisant, dans l’énergie potentielle élastique, un terme de couplage d’expression : E p,c = l x u, l étant une constante de couplage, x la variable de position comptée le long de la verticale descendante à partir de la position de repos, le ressort étant chargé, et u la variable angulaire comptée à partir du repos. 1. Exprimer, en fonction de x et u, l’énergie potentielle élastique totale du système et en déduire la force et le couple qui s’exercent sur lui. 2. Établir les équations différentielles du mouvement. 3. On impose, à l’aide des deux masselottes, la condition C/(mR 2) = K /m = v 20. Trouver les vibrations propres de cet oscillateur à deux degrés de liberté. P27– 4. Vibrations forcées d’une molécule ionique Une molécule ionique A −B+ est soumise à l’action d’un champ électrique uniforme, parallèle à l’axe Ox de la molécule et évoluant sinusoïdalement, avec une pulsation v. Les masses des ions sont respectivement m1 et m 2. La composante algébrique du champ suivant Ox, orienté de A − vers B+ , s’écrit : Ex = Em cos(vt). 1. Écrire les équations vérifiées par les composantes algébriques x 1 et x 2 suivant Ox, des déplacements des ions A− et B+ , par rapport à leur position de repos, dans le référentiel galiléen du centre de masse. 2. Montrer qu’il existe une solution particulière correspondant à des oscillations sinusoïdales des ions A− et B+ , de même pulsation v que le champ. En déduire que les ions vibrent en opposition de phase et calculer le rapport des amplitudes des vibrations. P27– 5. Pendule double 1. Établir les équations différentielles du pendule double constitué de deux pendules simples identiques. 2 . Trouver les modes normaux de vibration d’un tel pendule dans l’approximation des petits angles.

481

Oscillateurs couplés. Cas de N oscillateurs identiques P27– 6. Modes normaux de vibration longitudinale de la molécule CO 2

On étudie les mouvements de vibration de la molécule triatomique de dioxyde de carbone CO 2 : on désigne par m1 la masse d’un atome d’oxygène (masse molaire M1 = 16 g) et par m2 la masse d’un atome de carbone (masse molaire M2 = 12 g). 1. Établir, en utilisant la loi fondamentale de la dynamique pour chacune des particules, les équations différentielles du mouvement des atomes dans le référentiel du centre de masse R ∗ de la molécule ; on désigne par x1, x2 et x 3 les déplacements dans R ∗ , à partir de leurs positions de repos, des atomes O, C, O respectivement. 2. Trouver les modes normaux de vibration. 3. Pour chacun de ces modes de vibration, déterminer la relation entre les amplitudes complexes de ces mouvements harmoniques. Calculer le rapport des pulsations propres. 4. Quelles seraient, en Hz, les fréquences des modes propres de vibration, si la raideur K avait la même valeur que celle de la liaison dans la molécule CO de monoxyde de carbone, c’est-à-dire K = 1, 9 × 103 N . m −1 ? P27– 7. Pendule élastique et pendule simple couplés Un pendule élastique (masse m 1 , raideur K , longueur à vide l0 ) oscille horizontalement en entraînant un pendule simple (masse m 2, longueur l) (Fig. 27.16). La liaison pivot entre les deux pendules en A1 est parfaite. 1. Exprimer, en fonction de l’allongement x du ressort et de l’angle u que fait le pendule simple avec la verticale, la quantité de mouvement, l’énergie cinétique et l’énergie potentielle du système. 2. Établir les deux équations différentielles du mouvement. 3. Retrouver les équations précédentes à l’aide des équations de Lagrange. 4. On se place dans l’approximation des petits mouvements et dans le cas où : m 1 = 3m2 = 3m et K = 4mg/l = 4mv20 . a) Quelles sont les équations linéarisées du mouvement ?

 c Dunod – Toute reproduction non autorisée est un délit

b) En déduire les pulsations propres du système. c) Établir les expressions de x(t) et u(t) si le système est abandonné sans vitesse avec : x = 0 et u = u0 . d) Quelles sont les coordonnées normales du système ?

K1 K

A1 θ

x l

m1

g K2

A2 F IG . 27.16.

g

α

m2 F IG . 27.17.

482

27. Oscillateurs couplés. Cas de N oscillateurs identiques

P27– 8. Étouffeur de vibration Dans le montage mécanique de la figure 27.17, les solides S 1 et S2 , de masses m1 et m 2, ont un mouvement de translation verticale. Le solide S1, suspendu à un bâti fixe par un ressort et un amortisseur de caractéristiques (K1, a), est soumis à une force supplémentaire de mesure algébrique F(t) = Fm cos(vt) selon la verticale descendante. Montrer que l’association de l’oscillateur 2, de masse réglable, et de l’oscillateur principal permet d’obtenir un régime stationnaire où la vitesse de S1 est négligeable. P27– 9. Chaîne de trois oscillateurs mécaniques identiques On considère trois oscillateurs identiques constitués par trois masselottes de même m et quatre ressorts de même raideur K (Fig. 27.18). Déterminer les pulsations propres du système en cherchant des solutions de la forme : xn = A exp(jknd) exp(jVt) d étant la distance qui sépare les trois oscillateurs à l’équilibre et n pouvant prendre les valeurs 0, 1, 2, 3, 4.

K

A1 m

K

A2

K

m F IG . 27.18.

A3 m

K

28 Introduction à la mécanique des fluides. Statique des fluides Dans cette introduction à la mécanique des fluides, nous nous proposons d’appliquer les théorèmes généraux de la statique des systèmes matériels (cf. chapitre 23) à une catégorie particulière de systèmes déformables, les fluides. Avant toute étude de la statique des fluides, il apparaît indispensable de préciser la définition de tels systèmes et de présenter les grandeurs qui les caractérisent à l’échelon macroscopique.

I . — DÉFINITIONS ET GRANDEURS CARACTÉRISTIQUES I . 1 . — Définition d’un fluide Un fluide est un milieu matériel continu, déformable, qui peut s’écouler. Afin de préciser cette définition intuitivement claire, examinons plus précisément les propriétés de continuité, de déformation et d’écoulement d’un milieu.

 c Dunod – Toute reproduction non autorisée est un délit

a) Milieu continu Un milieu est continu si le nombre de particules physiques contenues dans un volume élémentaire, à l’échelon macroscopique, est suffisamment grand pour que l’on puisse négliger toute fluctuation de ce nombre. Par exemple, dans les conditions usuelles de température et de pression, un cube d’air de 1 mm de côté contient environ 3 × 10 7 molécules.

En revanche, l’élément de volume autour d’un élément ponctuel P du fluide est cinématiquement caractérisé par un seul vecteur vitesse vP et un seul vecteur accélération a P par rapport au référentiel du laboratoire R (Fig. 28.1) ; on l’appelle particule macroscopique ou plus brièvement particule de fluide qu’il faut impérativement distinguer des particules physiques de dimensions évidemment bien plus faibles. z P

vP

A d O x

R F IG . 28.1.

y

484

28. Introduction à la mécanique des fluides. Statique des fluides

Une analyse microscopique montre que le milieu est continu si toute distance caractéristique du fluide est très grande devant le libre parcours moyen  des particules, c’est-à-dire la distance qui sépare en moyenne deux collisions (cf. Thermodynamique). Ainsi, dans le cas d’un gaz à la température ordinaire et à la pression atmosphérique,  de l’ordre de 100 nm est très faible devant toute dimension D du récipient qui le contient. L’échelle d’analyse considérée ici est mésoscopique, c’est-à-dire intermédiaire entre les niveaux macroscopique et microscopique. b) Milieu déformable Un milieu est déformable si on peut considérer que les différents éléments matériels qui le constituent sont à des distances mutuelles variables ; c’est ce qui le distingue du solide (indéformable). c) Écoulement Alors que dans un solide les forces d’interaction maintiennent à des distances pratiquement constantes les différentes particules macroscopiques qui le constituent, dans un fluide ces particules sont très mobiles les unes par rapport aux autres : elles se déplacent facilement sous l’action de très faibles efforts. Cette faible interaction confère au fluide la propriété de s’écouler et d’épouser la forme du récipient qui le contient. Il existe principalement deux états fluides de la matière : l’état liquide et l’état gazeux. Les liquides n’ont pas de forme propre mais ont un volume propre, alors que les gaz n’ont ni forme propre ni volume propre ; ils tendent à occuper tout l’espace offert. On caractérise les fluides par leur compressibilité, c’est-à-dire leur capacité à changer de volume lorsqu’on exerce sur eux des forces. Les liquides ont une compressibilité négligeable, aussi les considère-t-on généralement comme incompressibles : leur masse volumique r est pratiquement constante. Par exemple, l’eau a un coefficient de compressibilité isotherme qui vaut à la température ordinaire : 1 ∂ (kT )l = − ∼ 5 × 10 −10 Pa−1 ∂p T Quant aux gaz, ils sont aisément compressibles. Par exemple, pour un gaz parfait à la pression atmosphérique, kT vaut : 1 ∂ 1 (kT) g = − = ≈ 10−5 Pa −1  (k T )l ∂p T p Cependant, très souvent les gaz ne subissent que des forces assez faibles pour que l’on puisse négliger les variations de masse volumique ; ils se comportent alors comme des fluides dont l’écoulement est incompressible. I . 2 . — Description d’un fluide Le plus souvent, on s’intéresse aux propriétés de l’ensemble du fluide que l’on caractérise globalement par des fonctions du temps et des coordonnées d’un point A fixe par rapport au référentiel du laboratoire R . Par exemple, autour de chaque point A de R, occupé par le fluide, on définit la masse volumique par le rapport de la masse dm d’un élément de matière par le volume d qu’il occupe : dm r= d Généralement r dépend des coordonnées spatiales (x, y, z) de A et du temps t. On écrira donc : r(A, t)

ou

r(x, y, z, t)

Introduction à la mécanique des fluides. Statique des fluides

485

De même, on caractérise la répartition des vitesses par le champ des vitesses v(A, t) ou v(x, y, z, t) , ainsi que la répartition de la température dans le fluide par le champ des températures T (A, t) ou T (x, y, z, t) . Cette description, dite description eulérienne car proposée par Euler en 1775, est très commode car les forces auxquelles est soumis un élément de fluide s’expriment généralement sous la forme d’un champ de forces. En outre, elle est très adaptée à une étude expérimentale que l’on met en œuvre en plaçant des capteurs en des endroits déterminés, fixes par rapport au référentiel du laboratoire R.

Dès qu’elle fut publiée en 1755, cette contribution d’Euler fut saluée par Lagrange, à l’origine d’une autre description, dans laquelle les coordonnées x , y , z d’un élément de fluide sont considérées comme des fonctions du temps t et de ses coordonnées initiales x 0 , y 0 et z0 . Cette description lagrangienne est moins commode que la précédente. Évidemment, ces deux descriptions ne diffèrent que par leur commodité ; il n’est pas inutile de préciser que l’accélération d’un élément de fluide a certes deux expressions différentes, mais une même valeur. Remarque : On a souvent utilisé le champ des vitesses d’un solide comme intermédiaire de calcul pour connaître, en fonction des paramètres, l’expression de la vitesse d’un point déterminé du solide, par exemple celle de son centre de masse. Mais on n’a jamais exploité la dépendance spatiotemporelle de ce champ, comme on le fait systématiquement en mécanique des fluides.

II . — PRESSION II . 1 . — Tenseur des contraintes Considérons une surface fermée S délimitant un volume V de fluide ou plus généralement d’un milieu continu, et séparons ce volume en deux parties 1 et 2 (Fig. 28.2) par une surface S12. Cette surface est traversée par des particules dans les deux sens, avec des quantités de mouvement différentes. Il en résulte un taux de variation de quantité de mouvement et par conséquent une force d’interaction entre les deux parties à travers cette surface de séparation. La force élémentaire qu’exerce la partie 1 sur la partie 2, à travers l’élément de surface d S de S12 , est proportionnelle à la quantité de mouvement associée aux particules qui traversent d S et donc à cet élément de surface ; aussi pouvons-nous écrire :

 c Dunod – Toute reproduction non autorisée est un délit

d F1→2 = fs,1→2 d S La force surfacique f s,1→2 est appelée le vecteur-contrainte au centre de l’élément de surface. Elle n’est pas en général dirigée suivant la normale n1→2 orientée de 1 vers 2. Cependant, on a, dans l’approximation linéaire : f s,1→2 = −[s]n1→2 où [s] , ainsi défini à partir de f s,1→2 avec un signe moins, est un opérateur linéaire qui permet de passer du vecteur n1→2 au vecteur-contrainte fs,1→2 ; on l’appelle le tenseur des contraintes. f s,1→2 n1→2

S12 dS 1

2 F IG . 28.2.

S

486

28. Introduction à la mécanique des fluides. Statique des fluides

Le vecteur-contrainte peut être considéré comme la somme de deux vecteurs-contraintes, l’un colinéaire à n1→2 , l’autre perpendiculaire à n 1→2 . On peut donc mettre fs,1→2 sous la forme suivante : f s,1→2 = −[s n] n1→2 − [st ] n 1→2 dans laquelle −[sn ] n1→2 est la contrainte normale et −[s t ] n1→2 la contrainte tangentielle.

Si cette dernière, qui traduit l’existence de forces de frottement visqueux entre les parties 1 et 2, peut être négligée, on dit que le fluide est non visqueux ou qu’il ne présente pas de viscosité (cf. chapitre 31) ; ainsi, dans les conditions usuelles, l’eau liquide est pratiquement un fluide non visqueux contrairement à l’huile. Remarque : Si la viscosité du fluide est négligeable, ainsi que les transferts thermiques en son sein, on qualifie généralement le fluide de parfait. Cette perfection ne doit pas être confondue avec celle d’un gaz parfait, défini lui, comme un fluide suffisamment dilué pour que les interactions entre les atomes ou les molécules qui le constituent soient négligeables. Comme un gaz parfait présente une viscosité non négligeable (cf. Thermodynamique), il n’est pas parfait au sens donné en mécanique des fluides. C’est la raison pour laquelle nous avons évité l’usage de l’expression ambiguë de fluide parfait. II . 2 . — Pression dans un fluide au repos Pour un fluide au repos dans R , la contrainte tangentielle −[s t ] n 1→2 est nulle et la contrainte normale −[sn ] n1→2 se met sous la forme : −[sn ] n1→2 = p n1→2

Si la quantité scalaire p est positive, ce qui est généralement le cas, on l’appelle la pression du fluide au point A considéré. La force élémentaire normale qu’exerce 1 sur 2 à travers la surface élémentaire d S a alors pour expression : d F1→2,n = p n1→2 d S Ainsi, la force normale qu’exerce localement la partie 1 du milieu continu sur la partie 2 traduit une répulsion. On montre que cette répulsion est indépendante de l’orientation de l’élément de surface, ce que confirme l’expérience. Si la surface S12 sépare, dans le milieu continu, une partie 1 solide et une partie 2 fluide, l’analyse est la même : la force normale qu’exerce la paroi solide sur le fluide s’écrit : d F1→2,n = p n1→2 d S = −p n ex d S

nex étant la normale orientée vers l’extérieur du fluide, c’est-à-dire de 2 vers 1. L’unité SI de pression est le pascal : 1 Pa = 1 N.m −2. C’est une unité petite puisque la pression atmosphérique, au niveau de la mer, vaut : pa = 1 atm ≈ 1, 013 × 10 5 Pa . Aussi utilise-t-on souvent le bar ou l’hectopascal (cf. Thermodynamique) : 1 bar = 105 Pa = 0, 1 MPa = 100 kPa = 1 000 hPa

et

1 hPa = 10 2 Pa = 10−3 bar = 1 mbar

Remarques : (1) Dans certains cas rares, on observe non pas une répulsion du milieu mais une attraction ; la pression est alors négative ! On attribue cette attraction à des effets de surface ; aussi cette force attractive est-elle appelée force de tension superficielle (cf. Thermodynamique).

487

Introduction à la mécanique des fluides. Statique des fluides

(2) Notons qu’un bar est approximativement la pression qu’exerce sur une surface de 1 cm 2 la force de pesanteur correspondant à une masse de 1 kg ; en effet 9,80/10 −4 ≈ 1 bar. II . 3 . — Interprétation microscopique de la pression dans un gaz L’interprétation microscopique de la pression dans un gaz permet de préciser la définition macroscopique précédente. La pression est la valeur statistique moyenne de la force qu’exerce, par unité de surface, une partie du gaz sur l’autre, en raison de l’échange de quantité de mouvement à travers cette surface. Pour l’exprimer, on s’appuie sur la théorie cinétique des gaz élaborée par Maxwell en 1872. On retrouve alors l’équation d’état du gaz parfait (cf. Thermodynamique). II . 4 . — Distribution volumique associée aux forces de pression Considérons une portion de fluide à l’intérieur d’une surface fermée S (Fig. 28.3a). On sait que, dans le cas d’un fluide au repos, l’extérieur de cette portion (fluide ou solide) exerce sur cette dernière des forces de pression caractérisées par la distribution surfacique : fs,ex→S = p nex→S = −p n

où n = nS→ex = −nex→S est la normale sortante de la surface fermée S au point considéré. z S

dS d

O x

z

[σ]n n

z + dz z

M

dS

O

y x

a)

y b)

F IG . 28.3.

Cette distribution surfacique de force de pression peut être remplacée par une distribution volumique f v = − grad p. En effet, on montre (cf. annexe 5) que :

 c Dunod – Toute reproduction non autorisée est un délit

Fs = −

S

pn dS =

fv d

avec

f v = − grad p

Ainsi, on peut remplacer une distribution surfacique de forces de pression par une distribution volumique dont la force volumique est fv = − grad p. Remarques : (1) On peut retrouver cette équivalence en analysant la somme des forces qui s’exercent sur un élément de volume cylindrique de hauteur d z et de section droite d S dont le rayon de base est très petit devant d z (Fig. 28.3b). Puisque les forces de pression sur la surface latérale se neutralisent en raison de la symétrie, les forces de pression agissent seulement sur les bases du cylindre. Il en résulte : ∂p ∂p F s,z = p(x, y, z) d S − p(x, y, z + d z) d S = − dz dS = − d ∂z ∂z En généralisant à trois dimensions, on retrouve bien : f v = − grad p. (2) Les forces volumiques de pression statique dépendent d’une énergie potentielle volumique puisque l’expression : f v = − grad p est caractéristique d’une force qui dérive d’une énergie potentielle. Ainsi, dans le cas d’un fluide réel au repos, la pression peut être considérée comme une énergie potentielle volumique.

488

28. Introduction à la mécanique des fluides. Statique des fluides

III . — ÉQUATION D’ÉTAT D’UN FLUIDE On appelle équation d’état d’un fluide l’équation reliant entre eux la pression p, le volume la température T de ce fluide (cf. Thermodynamique).

et

III . 1 . — Gaz parfait Dans le cas d’un gaz parfait, c’est-à-dire un gaz suffisamment dilué pour que l’on puisse négliger les interactions à distance entre les différentes particules qui le constituent, l’équation d’état est : p

= nRT

n étant le nombre de moles et R = 8, 314 J . mol −1 . K −1 la constante universelle des gaz parfaits, produit du nombre de Avogadro NA par la constante fondamentale de Boltzmann k B . Comme n est aussi le rapport de la masse m de gaz sur sa masse molaire M, l’équation d’état se met sous les formes suivantes : m R p = RT = mrT ou p = rrT en posant r = M M et en introduisant la masse volumique r = m/ . Dans le cas du diazote N 2 (M = 28 g . mol−1 ), la constante r vaut : r = 8, 314/0, 028 = 297, 14 J.T−1kg −1 . III . 2 . — Gaz réels Pour les gaz réels, l’équation d’état est moins simple que la précédente. Celle donnée par J.D. Van der Waals permet dans beaucoup de cas de décrire convenablement de tels gaz : a p + n 2 2 ( − nb) = nRT On voit que, pour n2a/ 2  p et nb  , on retrouve l’équation du gaz parfait. Le terme an2 / 2 , qui traduit la prise en compte de l’interaction purement attractive entre les molécules du gaz, est équivalent à une pression supplémentaire, d’où son nom de pression moléculaire. Quant au terme b, appelé le covolume molaire, il prend en compte le volume occupé par les molécules du fait de leurs dimensions non nulles : − nb est alors le volume réellement accessible aux molécules (cf. Thermodynamique). III . 3 . — Liquides Pour un liquide, l’équation d’état reliant le volume, la pression et la température, peut se mettre sous la forme : =

0 [1

+ a(T − T 0) − k T (p − p 0)]

ce qui s’écrit aussi r[1 + a(T − T 0) − k T (p − p0 )] = r0

où 0 est le volume du liquide à T = T 0 = 273 K , p = p 0 = 1 bar , r0 la masse volumique correspondante ; a est le coefficient de dilatation isobare et k T est le coefficient de compressibilité isotherme : 1 ∂ 1 ∂ et k T = a = ∂T p ∂p T Par exemple pour l’eau liquide, a (T0, p 0) = 0, 5 × 10 −3 K−1 et kT = 0, 5 × 10−9 Pa−1 . La faiblesse de ce dernier coefficient montre que l’on peut souvent négliger la compressibilité d’un écoulement hydraulique.

489

Introduction à la mécanique des fluides. Statique des fluides

IV . — STATIQUE DES FLUIDES IV . 1 . — Équation de la statique des fluides Comme c’est le cas pour tout système mécanique, on obtient l’équation de la statique d’un fluide, en exprimant l’immobilité de chacun des éléments qui le constituent dans l’équation du mouvement par rapport au référentiel R considéré (cf. chapitre 23). Si R est le référentiel terrestre, cette équation du mouvement s’écrit, pour un élément de volume d , de masse volumique r : rd

a = rd

g − grad p d

− 2r d

VT × v

soit

a=g−

grad p − 2VT × v r

r g d étant le poids, qui contient la force d’inertie d’entraînement, −2r d V T ×v la force d’inertie la force de pression. En explicitant les composantes de cette de Coriolis (cf. chapitre 7) et − grad p d équation vectorielle dans une base cartésienne, on obtient les trois équations du mouvement. a) Statique dans le référentiel du laboratoire En statique, le référentiel terrestre peut être considéré comme galiléen avec une excellente approximation puisque, la vitesse étant nulle, la force d’inertie de Coriolis terrestre l’est aussi, et que le champ de pesanteur contient la force d’inertie d’entraînement (cf. chapitre 7). Les forces qui s’exercent sur le et les forces de pression. Comme la somme de ces fluide sont donc : la pesanteur de la forme r g d forces doit être nulle, il vient : rg d

− grad p d

=0

soit

grad p = r g

Les vecteurs grad p et g étant colinéaires, les surfaces isobares et les surfaces équipotentielles de pesanteur sont confondues (Fig. 28.4a). Ainsi, la surface libre d’un liquide en équilibre définit localement un plan horizontal (cf. chapitre 7). Remarque : Dans ce qui précède, on a négligé les forces de tension superficielle qui apparaissent aux interfaces, ce qui est souvent légitime. Cependant, c’est grâce à elles qu’on explique l’ascension des liquides dans les tubes capillaires et la forme sphérique des gouttes (cf. Thermodynamique).

 c Dunod – Toute reproduction non autorisée est un délit

y

y

y R

R g

O

x

O

z R -a 0

x

a

g

V2 r ga

a0

g

x

ga a)

®

V b)

c)

F IG . 28.4.

b) Statique dans un référentiel non galiléen Si le référentiel considéré n’est pas galiléen, il faut tenir compte, en outre, de la force d’inertie d’entraînement, celle de Coriolis étant nulle du fait du repos dans ce référentiel. On remplace alors le champ de pesanteur g par le champ de pesanteur apparent ga = g − a e (cf. chapitre 7). Il vient donc : grad p = r(g − a e) = rga

490

28. Introduction à la mécanique des fluides. Statique des fluides

Exemples : i) Le référentiel accéléré a un mouvement de translation Dans un référentiel R  en translation rectiligne dont l’accélération par rapport au référentiel du laboratoire R est a0 , la surface libre d’un liquide en équilibre est normale au champ de pesanteur apparent et donc inclinée par rapport à l’horizontale de l’angle a tel que tan a = a0 /g (Fig. 28.4b). La mesure de a permet donc à un observateur de R de déterminer l’accélération a0 de R . On détermine l’équation des surfaces isobares en intégrant les équations différentielles partielles issues des projections de l’équation vectorielle précédente : ∂p = −ra0 ∂x

Il vient :

∂p = −rg ∂y

et

∂p =0 ∂z

∂p(x , y ) d f (y  ) = −rg = d y ∂y

p(x , y ) = −ra0 x + f (y)

et

f (y ) = −rgy  + Cte

p(x y ) = −ra 0x − rgy + Cte

Il en résulte : et

On détermine la constante en écrivant que la pression en un point de la surface du liquide est la pression atmosphérique pa. L’équation des isobares est donc : y = −

ra0  x + Cte g

ii) Le référentiel accéléré a un mouvement de rotation uniforme Dans un référentiel tournant, la surface libre d’un liquide au repos, qui est normale à g a , prend la forme d’un paraboloïde de révolution. En effet, projetons la loi fondamentale de la statique des fluides dans le référentiel tournant. Il vient (Fig. 28.4c) : ∂p ∂p = −rg = rV 2 r et ∂r ∂z car la pression ne dépend pas de la variable angulaire w en raison de la symétrie de révolution. On en déduit : rV2 r2 ∂ p(r, z) d f (z) + f (z) et p(r, z) = = −rg = ∂z 2 dz Il en résulte : rV2 r2 − rgz + Cte p(r, z) = − 2 On détermine la constante en exprimant par exemple que la pression au point le plus bas de la surface du fluide en rotation, de cote z 0, est la pression atmosphérique p a : p(0, z0 ) = −rgz0 + Cte = pa

d’où

p(r, z) =

rV2r 2 − rg(z − z0) + pa 2

L’équation de la surface du liquide est donc donnée par l’équation suivante : z = z0 +

V 2r2 2g

On a réalisé des miroirs paraboliques pour télescopes en figeant du verre fondu en rotation (cf. Optique). L’intérêt est rendu manifeste en envoyant un faisceau lumineux parallèle suffisament large sur la surface du liquide en rotation. Le faisceau réfléchi converge en un point de l’axe de rotation ; on peut faire varier la hauteur du foyer en changeant la vitesse de rotation.

Introduction à la mécanique des fluides. Statique des fluides

491

Remarque : Il convient de noter que, dans cet exemple, la mise en mouvement du fluide n’est rendue possible que par les forces de viscosité qui, en s’exerçant au niveau des parois du récipient, transmettent au fluide le mouvement de rotation du flacon (cf. chapitre 31). IV . 2 . — Variation de la pression avec l’altitude dans un fluide incompressible a) Expression de la différence de pression Dans un fluide dont le coefficient de compressibilité est négligeable, comme c’est en général le cas pour les liquides, la relation vectorielle traduisant le repos par rapport au référentiel du laboratoire s’écrit : grad p = r g avec r indépendant de p et de la position du point dans l’espace. Explicitons cette équation suivant un axe vertical ascendant et intégrons. Il vient : dp = −rg dz

donne

p = −rgz + Cte

Il en résulte la relation suivante entre les pressions en deux points A et B : pA − pB = −rg(zA − z B) Ce résultat est connu sous le nom historique de principe de Pascal, du nom de B. Pascal : Si le coefficient de compressibilité d’un fluide est négligeable, la différence de pression, entre deux points A et B de ce fluide au repos, est égale au poids d’une colonne de ce fluide, de section unité, dont la hauteur est égale à la différence d’altitude de A et B. Si pA varie de Dp, pB varie aussi de Dp. On dit que les liquides transmettent intégralement les variations de pression. Remarque : Ce comportement diffère de celui des solides qui, eux, transmettent les forces et non les pressions. Pour s’en convaincre, il suffit d’écrire la condition d’équilibre d’un solide sous l’action de deux forces extérieures F1→S et F2→S ; on a :

 c Dunod – Toute reproduction non autorisée est un délit

F1→S + F2→S = 0

d’où

F 1→S = −F2→S = FS→2

b) Applications Les applications du résultat précédent sont nombreuses. (1) Pression dans un lac ou dans un océan À une profondeur h dans un lac ou dans un océan (Fig. 28.5), la pression vaut, puisque p a ≈ 100 kPa et r = 1 000 kg . m −3 : p = pa + rgh

soit

p ≈ (100 + 9, 81 × h) kPa

Ainsi, pour h = 10 m, la pression est pratiquement le double de la pression atmosphérique. (2) Baromètre à liquide Le baromètre à mercure est un appareil très simple qui permet de mesurer la pression atmosphérique (Fig. 28.6). Il est constitué d’un tube d’environ 80 cm de long, que l’on a renversé dans une cuve à mercure une fois rempli à ras bord. On constate que le mercure descend dans le tube jusqu’à une hau-

492

28. Introduction à la mécanique des fluides. Statique des fluides z g

g O

pm  0

pa h = 10 m pa

pB = pa

≈ 2pa F IG . 28.5.

F IG . 28.6.

teur qui donne la pression atmosphérique. En effet, la pression au point B du mercure situé dans le plan horizontal est aussi la pression atmosphérique p a ; cette pression est reliée à la pression p m de la vapeur de mercure au sommet du tube, qui est négligeable, par l’équation : rm ghm = pB − p m ≈ p B, hm étant la hauteur de mercure en mètre et r m = 13 568 kg . m−3 la masse volumique du mercure. L’expérience montre que hm ≈ 0, 76 m. Par conséquent : pB = p a = rm ghm ≈ 133, 1 × 10 3 × 0, 76 = 1, 013 × 105 Pa Le mercure est le liquide qui, en raison de sa forte masse volumique, donne, à température ambiante, la hauteur la plus faible. Dans le cas de l’eau (masse volumique r e = 998, 2 kg . m−3 ), on obtiendrait une hauteur he bien plus grande : rmgh m = re ghe

d’où

he = hm

13 568 rm = 0, 76 × = 10, 33 m 998, 2 re

(3) Mesure des pressions Les appareils généralement utilisés pour mesurer les pressions sont appelés des manomètres. Le plus simple est le tube piézométrique vertical en U dont l’une des branches est reliée au point où l’on veut faire la mesure ; la pression au-dessus de la pression atmosphérique est donnée par rgh, h étant la différence de hauteur de liquide dans le tube (Fig. 28.7a). On améliore la sensibilité du manomètre en utilisant un liquide de faible masse volumique et en inclinant l’une des branches du tube (Fig. 28.7b). Fluide

Fluide pa

g

g

h

pa h

a)

b) F IG . 28.7.

F IG . 28.8.

(4) Expérience de Torricelli L’expérience du physicien italien du XVIIe siècle E. Torricelli consiste à montrer que la hauteur de mercure dans différents tubes ne dépend pas de la forme du tube, conformément aux résultats de l’analyse précédente (Fig. 28.8). (5) Expérience du tonneau de Pascal Cette expérience spectaculaire, faite par Pascal en 1646, consiste à provoquer l’explosion d’un tonneau en bois, rempli d’eau, en ajoutant une petite quantité de liquide au sommet d’une longue colonne d’eau verticale de faible section ( s ≈ 1 cm2 (Fig. 28.9). Comme la hauteur d’eau provoque une augmentation notable de la pression du liquide dans le tonneau, le bois ne résiste pas aux forces exercées par le liquide, si la hauteur d’eau atteint 10 m . Avec un autre récipient, tel un cylindrique métallique fermé par un couvercle, on constate qu’une hauteur de 8 m fait jaillir l’eau à la jointure du couvercle.

493

Introduction à la mécanique des fluides. Statique des fluides pa

g

pa

g

h p a + ρgh

ρ1

h1

ρ2

h2 pf

F IG . 28.9.

F IG . 28.10.

(6) Ensemble de deux liquides de masses volumiques différentes Dans ce cas, les deux liquides se superposent, le plus dense en bas, la surface de séparation étant horizontale (Fig. 28.10). La pression en un point du fond est : pf = p a + r1 gh 1 + r2gh 2 pa étant la pression atmosphérique, h1 la hauteur du liquide le moins dense et h2 la hauteur du liquide le plus dense. IV . 3 . — Variation de la pression avec l’altitude dans un fluide Dans un fluide dont le coefficient de compressibilité n’est pas négligeable, la masse volumique r dépend de la pression et de la température. C’est le cas généralement des gaz.

 c Dunod – Toute reproduction non autorisée est un délit

a) Expression de la pression dans un gaz parfait Comme l’équation d’état d’un gaz parfait s’écrit p = rrT (cf. Thermodynamique), l’équation de la statique donne en projection suivant la verticale ascendante : dp dp pg g soit dz = −rg = − =− rT p rT dz Dans l’hypothèse où la température est uniforme, on obtient, en intégrant, la formule dite du nivellement barométrique : p g z rT ln p = p(0) exp − avec z = = − z soit z rT g Cte p(0) désignant la pression à z = 0 et z une hauteur caractéristique qui dépend de la température et du fluide. Remarque : En introduisant la masse m1 d’une particule, l’argument de l’exponentielle se met sous la forme : −m1 gz/(kBT ) puisque r = R/M = R/(NA m 1) = k B/m1 . Écrite ainsi, cette formule rappelle la distribution de Boltzmann (cf. Thermodynamique). b) Pression d’un gaz uniforme dans un flacon Calculons la variation relative de pression, pour une variation d’altitude de 20 cm, dans le cas d’un gaz diatomique, tel que le dioxygène, dans les conditions standard ( T = 298 K et pa = 1 bar) : dp dz =− z p

avec

z=

8, 314 × 298 RT ≈ 15, 8 × 10 3 m ≈ 0, 016 × 9, 81 Mg

494

28. Introduction à la mécanique des fluides. Statique des fluides

d’où : Dp Dz 0, 2 ≈− ≈− ≈ −1, 26 × 10−5 p 15, 8 × 10 3 z Ainsi, Dp/p  1. Aussi considère-t-on que la pression d’un gaz dans un flacon est uniforme. c) Application à la variation de la pression de l’air avec l’altitude Sur la figure 28.11a, on a représenté le graphe p(z) pour l’air à la température constante T = 298 K : 8, 314 × 298 RT ≈ 8 427 m z= = 0, 029 × 9, 81 Mg En réalité, la formule précédente traduit bien la variation de pression avec l’altitude dans la stratosphère, c’est-à-dire entre 11 et 20 km d’altitude, où la température est uniforme (T ≈ 217, 55 K soit −55, 6 ◦ C). Dans la troposphère (0 < z < 11 km), elle convient moins, car la température varie suivant une loi affine (Fig. 28.11b) : T = T 0 − Bz

avec

T 0 = 288, 16 K et

B = 6, 5 K . km−1

On admet généralement que la pression est pratiquement nulle à 30 km d’altitude. Notons que la formule approchée pa = ragH, qui suppose l’air incompressible, donne un ordre de grandeur de H : H∼

10 5 pa = ≈ 10 km 1, 3 × 9, 81 r ag

z (km)

z (km)

30 20

30 T = cte

10 ξ 0

20 10

p(0)

p

0

a)

210

290

T (K )

b) Variation de la température F IG . 28.11.

IV . 4 . — Forces de pression exercées par un liquide sur une paroi Le calcul des forces de pression exercées par un liquide sur une paroi joue un rôle capital dans la réalisation des retenues d’eau et des barrages. On le mène aisément en rappelant que la pression varie avec l’altitude z selon la relation simple : p(z) = −rgz + p a si l’origine de la coordonnée est prise en un point où la pression est la pression atmosphérique pa . Comme les forces de pression atmosphérique s’exercent sur tout le système délimité par une surface fermée, le torseur associé est nul. Aussi, peuvent-elles être supprimées d’emblée dans le calcul.

495

Introduction à la mécanique des fluides. Statique des fluides a) Centre de poussée

Le centre de poussée des forces de pression exercées par un liquide sur une paroi est le point K en lequel le moment des forces de pression est nul : MK = 0

MO + KO × F = 0

d’où

et MO = OK × F

O étant un point fixe. Si toutes les forces de pression sont concourantes en un point O, le centre de poussée K est situé en ce point. b) Paroi cylindrique d’axe horizontal Une paroi ayant la forme d’un quart de cylindre, de rayon R, d’axe horizontal et de longueur l, retient une masse d’eau (Fig. 28.12). Toutes les forces élémentaires de pression sont normales à la paroi et donc passent par des points situés sur l’axe. L’ensemble est donc torsoriellement équivalent à une force unique passant par le centre O de l’axe du cylindre générateur (cf. chapitre 1). Le centre de poussée est alors en O. Comme la pression s’écrit p(z) = pa − rgz, la composante horizontale de la somme des forces a pour expression, en omettant la contribution de la pression atmosphérique : 0

(−rgz) d Sv = −rgl

Fh =

z d z = rgl −R

R2 2

d Sv étant l’élément de surface vertical.

Quant à la composante verticale de la force, elle s’écrit, si d S h est l’élément de surface horizontal : R

pR2 4 0 puisque l’intégrale représente l’aire d’un quart de cercle. Les signes obtenus s’interprètent aisément : la force horizontale est bien selon l’axe Ox et la force verticale est bien descendante. Il en résulte que la force de poussée passe par le point O et fait l’angle a = arctan(p/2) = 57, 5◦ avec l’horizontale. Pour l = 20 m et R = 3 m, ces forces valent respectivement, en norme : Fv =

(−rgz) d Sh = −rgl

Fh = 0, 88 MN et z  c Dunod – Toute reproduction non autorisée est un délit

O g R

y 57,5° dS h dSv

dFh

dFv

z d y = −rgl ×

F v = 1, 4 MN z Ox

K

F IG . 28.12.

g

y dF

h

F IG . 28.13.

c) Paroi plane rectangulaire verticale Considérons une paroi plane rectangulaire verticale Ozx, O étant un point de l’interface airliquide et Oz un axe vertical ascendant (Fig. 28.13). Si l’on omet la force due à la pression atmosphérique qui agit en tout point du système, la force élémentaire qu’exerce un liquide sur un élément de paroi verticale homogène a pour expression, z étant négatif : d F = p n d S = −rgz d S ey

496

28. Introduction à la mécanique des fluides. Statique des fluides

En intégrant sur toute la surface et en introduisant le centre de masse C de cette paroi de hauteur h, on obtient : h F = −rg z d S ey = −rgS zC ey soit F = rgS e y 2 S puisque z c = −h/2. En outre, le moment en O de ces actions s’écrit : MO =

OA × d F = e x

soit : MO = rg e x S

S

rgz 2 d S − e z

(y2 + z 2) d S − rg ez

S

S

rgxz d S

xz d S

On reconnaît, à la masse surfacique M/S près, le moment d’inertie Ixx par rapport à l’axe des x et le produit d’inertie Ixz de la paroi. Par conséquent, MO s’écrit : S MO = rg (IOx e x − Ixz ez) M Le torseur associé à l’ensemble des forces de pression parallèles qui s’exercent sur la paroi est celui d’une force unique F qui passe par le centre de poussée K de la paroi, tel que (cf. chapitre 1) : MO = OK × F = (xK ex + z K ez) × F y ey = F y(−zK ex + x K ez )

avec

F y = −rgS zC

En identifiant, on en déduit les coordonnées de K : I Ox Ixz zK = et x K = Mz C MzC 2 Dans le cas considéré, on a, puisque IOx = Mh /3 et Ixz = 0 : zK = −2h/3 et x K = 0. Notons que le centre de poussée ne coïncide pas avec le centre de la paroi.

V . — THÉORÈME D’ARCHIMÈDE ET CORPS FLOTTANTS V . 1 . — Théorème d’Archimède Considérons un corps immergé dans un fluide au repos. Il occupe un volume du fluide des forces de pression (Fig. 28.14a). g

g

Cf

et subit de la part

Eau O

a)

b) F IG . 28.14.

Ce système de forces élémentaires admet une résultante qui est opposée au poids du fluide de remplacement. En effet, en l’absence de corps immergé, ce fluide de remplacement serait au repos sous l’action des forces de pression exercées par le fluide environnant et des forces de pesanteur. On a donc, en désignant par [Fp] le torseur des forces de pression et par [Fg ] celui des forces de pesanteur : [Fp] + [F g] = [0] Comme le torseur [Fg] est celui associé à la force de pesanteur du fluide de remplacement, il en résulte le théorème suivant : Tout corps immergé dans un fluide au repos est soumis de la part du fluide à une poussée verticale, opposée à g, égale au poids du volume du fluide de remplacement et appliquée au centre de masse C f de ce fluide.

497

Introduction à la mécanique des fluides. Statique des fluides

Ce résultat est historiquement connu sous le nom de principe d’Archimède, homme de science grec du IIIe siècle avant J. C. Le point C f est appelé le centre de carène : il coïncide avec le centre de masse C du corps si ce dernier est homogène et totalement immergé. Ordres de grandeur : Comparons, à son poids, la force de poussée qui s’exerce sur une pierre, de volume = 1 L et de masse volumique rp = 1 800 kg . m−3, entièrement immergée dans l’eau : Mg = rpg

= 17, 66 N alors que

F p = reg

= 9, 792 N

Le rapport des deux forces est donc celui des masses volumiques. Dans le cas d’un objet plongé dans l’air, ce rapport est faible ; par exemple, pour un homme, on doit comparer sa masse volumique r h ≈ 1 000 kg . m−3 (proche de celle de l’eau) à la masse volumique de l’air r a ≈ 1, 3 kg . m −3 .

Si on abandonne un corps immergé dans un fluide, dont la masse volumique r f est inférieure à sa masse volumique r , il est soumis à une force résultante verticale descendante ; il tombe donc au fond du récipient. Si les masses volumiques sont égales, il reste immobile en n’importe quel point du fluide. Enfin si la masse volumique du fluide est supérieure à sa masse volumique, il est soumis à une force verticale ascendante ; une partie du corps n’est plus immergée : le corps flotte. Dans un sous-marin, on réalise les trois cas précédents en pompant de l’eau, ce qui fait varier sa masse volumique. Dans le cas d’un ballon gonflé d’hydrogène, par exemple, on peut admettre qu’il est soumis à la poussée d’Archimède, s’il évolue à très faible vitesse dans l’atmosphère, comme c’est généralement le cas. La force ascensionnelle a alors pour expression : F a = Mb g − M a g

 c Dunod – Toute reproduction non autorisée est un délit

Ma étant la masse de l’air de remplacement et M b < Ma la masse du ballon. Ainsi, un ballon dont la masse totale Mb , équipage et matériel compris, est 500 kg peut s’élever grâce à un volume d’air de remplacement de l’ordre de 500 m3 . Remarques : (1) Pour être valable, le théorème d’Archimède doit être appliqué strictement dans les conditions de sa démonstration, laquelle suppose le repos du ou des fluides avant que l’on introduise le corps à immerger. Considérons, par exemple, une sphère matérielle pouvant tourner parfaitement autour d’un axe horizontal fixé sur la paroi latérale d’un récipient (Fig. 28.14b) ; une moitié de la sphère est plongée dans l’eau et l’autre dans l’air. Ce cas est exclu par le théorème d’Archimède, puisque les deux fluides ne pourraient coexister en équilibre en l’absence de la sphère. Une analyse incorrecte qui aboutirait à un paradoxe consisterait à dire que les deux demi-sphères sont soumises à des forces de poussée différentes, appliquées aux centres de masses de chaque demi-sphère, et à en conclure que la sphère devrait tourner autour de son axe. En réalité, dans ce cas, les forces de pression sont toutes normales à la sphère et passent donc par l’axe de rotation. Comme le moment de toutes ces forces par rapport à l’axe est nul, la sphère reste immobile. C’est évidemment ce que confirme l’expérience. (2) Le théorème d’Archimède peut être appliqué si l’objet immergé, totalement ou partiellement, est en mouvement, à faible vitesse, encore faut-il que l’on puisse remplacer l’objet par le fluide environnant. Si ce n’est pas le cas, comme avec un avion, le problème doit être totalement reconsidéré, les forces qu’exerce l’air sur l’avion étant nettement plus importantes que le poids de l’air de remplacement (cf. chapitre 31).

498

28. Introduction à la mécanique des fluides. Statique des fluides

V . 2 . — Corps flottants Les corps flottent sur un liquide si la masse volumique du liquide de remplacement est supérieure à celle du corps ; seule alors une partie de ce dernier, la carène, est immergée (Fig. 28.15). Le corps est soumis à deux forces : son poids Mg et la poussée d’Archimède −M r g qui est l’opposée du poids des fluides de remplacement. Si l est le volume de carène et g le volume de la partie émergente, la condition d’équilibre s’écrit : Mg − M rg = 0

avec

Mr = rl

l

+ rg

g

≈ rl

l

la masse volumique rg des gaz étant négligeable devant celle des liquides. Dans le cas de fluides tels que l’air et l’eau, le rapport des masses volumiques est de 0, 0013. h l

h

F IG . 28.15.

Exemple : Une barge flottant sur l’eau, que l’on peut schématiser par une boîte rectangulaire ouverte, de dimensions L = 10 m, l = 4 m, h = 3 m et de masse M = 20 t , s’enfonce dans l’eau d’une hauteur h  telle que : Mg − r e Llh g = 0

d’où

Mg = re Llhg

et h  =

M = 0, 5 m re Ll

CONCLUSION Rappelons les points essentiels : (1) Un fluide est un milieu continu qui peut se déformer et s’écouler. (2) La description la plus adaptée pour l’étude d’un fluide est celle d’Euler où les variables (x, y, z, t) sont les coordonnées (x, y, z) du point A du référentiel, où se trouve la particule P de fluide, et t l’instant considéré. (3) La pression est une grandeur scalaire directement reliée à la force surfacique normale qu’exerce localement une partie du milieu continu sur une autre : d F 1→2,n = p n 1→2 d S. (4) L’équation fondamentale de la statique des fluides s’écrit : grad p = r(g − a e ) dans un référentiel quelconque, ae étant l’accélération d’entraînement par rapport au référentiel du laboratoire. (5) Les liquides transmettent intégralement les pressions, car on peut négliger leurs coefficients de compressibilité. (6) Enfin, le théorème d’Archimède exprime simplement la force de poussée ascendante qu’exerce un fluide sur un corps immergé : c’est le poids du fluide de remplacement.

EXERCICES ET PROBLÈMES P28– 1. Accéléromètre à liquide Montrer qu’il est possible de connaître l’accélération a 0 d’un véhicule à partir de la différence h des niveaux d’un liquide dans les branches verticales d’un tube en U qu’il transporte. La distance des branches du tube est d. Application numérique : h/d = 0, 577.

499

Introduction à la mécanique des fluides. Statique des fluides

P28– 2. Eau dans une cuve rectangulaire Une cuve rectangulaire, remplie d’eau jusqu’à une hauteur h 0 = 15 cm, est soumise à une accéleration constante a = a ex, le long d’un axe horizontal, par rapport à un référentiel terrestre. Dans le référentiel R lié à la cuve, l’eau est immobile. La base de la cuve est un carré de côté b = 15 cm.

1. Trouver l’expression de la pression en tout point du fluide, si l’eau occupe toute la base de la cuve. En déduire l’équation de la surface de l’eau, sachant que l’eau monte sur l’une faces verticales de la cuve jusqu’à une hauteur h. 2. On constate que, pour une valeur de l’accélération, l’une des faces verticales de la cuve n’est plus en contact avec le liquide. Calculer a. Quelle est alors la hauteur de l’eau sur l’autre face ? P28– 3. Liquide en rotation uniforme Un flacon cylindrique ouvert, de hauteur h = 40 cm et de diamètre 2R = 10 cm, contient de l’eau jusqu’à une hauteur he = 10 cm. On fait tourner le flacon autour de son axe avec la vitesse angulaire V. 1. Quelle est l’équation caractérisant la surface libre du liquide ? Pour quelle valeur de V le centre du fond du récipient est-il découvert ? 2. Trouver la valeur maximale de V que l’on peut atteindre sans que l’eau déborde du flacon. 3. Calculer la pression au fond du flacon si ce dernier tourne à la vitesse de 5 tr . s −1. P28– 4. Baromètre à deux liquides Un baromètre est constitué de deux tubes accolés, de même axe et de sections différentes, S1 = 1 cm 2 pour le tube inférieur, S2 = 0, 1 cm2 pour le tube supérieur plus fin (Fig. 28.16). L’ensemble, contenant du mercure (densité 13, 6) et de la glycérine (densité 1, 26), plonge dans une cuve à mercure, de section S 3 = 5 cm2 . z2

Glycérine

 c Dunod – Toute reproduction non autorisée est un délit

h2 z1 z3

1 r0 +Dp

r0

Eau

2 Aniline

h1 Mercure F IG . 28.16.

F IG . 28.17.

1. Sachant que la hauteur de mercure au-dessus de la cuve est h 1 = 74 cm, calculer la hauteur h2 de glycérine, si la pression extérieure est p 0 = 1 bar. 2. La pression p 0 varie de 1%. Trouver, en mm, la variation Dh 2 de la hauteur de glycérine. Comparer cette hauteur à celle que l’on aurait obtenue avec un baromètre à mercure ordinaire. Commenter. P28– 5. Manomètre à deux liquides Un manomètre est constitué de deux récipients, de sections différentes, S 1 = 10 cm 2 et S2 = 15 cm 2, reliés par un tube horizontal, de section s = 0, 2 cm2 (Fig. 28.17). L’ensemble contient de l’eau (masse volumique 998 kg . m−3 ) et de l’aniline (masse volumique 1 024 kg . m −3), non miscibles. Initialement la pression au-dessus des deux liquides est la pression atmosphérique p 0.

500

28. Introduction à la mécanique des fluides. Statique des fluides 1. Comparer les hauteurs d’eau et d’aniline au-dessus du ménisque.

2. On provoque une surpression Dp au-dessus du récipient 1. Quel est le déplacement Dz m du ménisque eau-aniline ? En déduire la sensibilité Dzm /Dp du manomètre. Commenter. P28– 6. Tube en U en rotation uniforme Un tube en U est constitué de deux branches verticales identiques, de diamètre négligeable, distantes de l = 15 cm (Fig. 28.18). Il contient de l’huile et de l’eau. À l’équilibre, on mesure une hauteur d’huile h = 12, 5 cm, alors que la hauteur d’eau au dessus de la surface de séparation huile-eau est e = 10 cm. 1. Quelle est la densité de l’huile ? 2. On fait tourner le tube en U, avec une vitesse angulaire uniforme V, autour d’un axe vertical passant par un point de la base du tube situé entre les deux branches. Trouver la nouvelle hauteur d’eau e au-dessus de l’interface I huile-eau, sachant que le tube effectue un tour par seconde. Application aux deux cas suivants : a) l’axe de rotation est la branche contenant l’huile, b) l’axe de rotation est la verticale passant par le milieu de la base du tube.

Huile

h V

e Eau

F IG . 28.18.

V h

F IG . 28.19.

P28– 7. Tube coudé en rotation uniforme On place au-dessus d’un liquide, en équilibre avec l’air ambiant, à une distance d = 10 cm, la partie transversale d’un tube coudé, de longueur l = 20 cm et de section négligeable (Fig. 28.19). En faisant tourner le tube à une vitesse angulaire uniforme V, on constate que le liquide monte dans le tube d’une hauteur h. La température de l’air, considéré comme un gaz parfait, de masse molaire M = 29 g . mol−1, est 273 K. 1. Pourquoi le liquide monte-t-il dans le tube vertical ? 2. Trouver, en fonction de V, la variation de pression de l’air au point de la portion horizontale du tube situé à la distance r de l’axe de rotation. Ce résultat était-il prévisible ? 3. Calculer la hauteur de montée h de l’eau dans le cas où la vitesse de rotation est de 10 tours par seconde. On prendra r eg = 9 792 N . m −3 et Pa = 1 bar. P28– 8. Variation de pression dans la troposphère Établir l’expression de la variation de la pression en fonction de l’altitude z, dans la troposphère, sachant que la température suit la loi affine suivante : T = T0 − Bz avec T0 = 288, 16 K et B = 6, 50 K . km−1 . On posera n = g/(rB), g étant la norme du champ de pesanteur terrestre et r la constante des gaz parfaits relative à l’unité de masse d’air.

501

Introduction à la mécanique des fluides. Statique des fluides

P28– 9. Statique d’un aréomètre Un aréomètre est constitué par une tige graduée, de section s, fixée à la partie supérieure d’un flotteur, de volume . On l’immerge dans un liquide de masse volumique r. Il s’enfonce jusqu’à une hauteur x, comptée à partir de la hauteur d’immersion xe dans l’eau, puis demeure immobile (Fig. 28.20). Établir la relation qui lie la hauteur x à la densité d = r/re du liquide, r e étant la masse volumique de l’eau. g

x xe

g

R

g a

F IG . 28.20.

θ h

F IG . 28.21.

h

F IG . 28.22.

P28– 10. Statique d’un cube de bois partiellement immergé Un cube de bois, de côté a = 1 m, de masse volumique r b = 500 kg . m−3 , flotte sur l’eau (masse volumique r = 1 000 kg . m−3 ), ses arêtes étant verticales ou horizontales (Fig. 28.21). Quelle est la profondeur h dont il s’enfonce lorsqu’il est immobile ? Trouver la période des oscillations autour de la position de repos. P28– 11. Statique d’un tronc d’arbre flottant sur l’eau Un tronc d’arbre flotte sur l’eau. On peut l’assimiler à un cylindre plein homogène de rayon R = 1 m. Il s’enfonce dans l’eau d’une hauteur h = 0, 87 cm puis se maintient au repos (Fig. 28.22). Trouver sa masse volumique ra.

 c Dunod – Toute reproduction non autorisée est un délit

P28– 12. Statique d’une bille entre deux liquides Une bille de masse volumique rb = 850 kg . m−3 est immergée dans un récipient contenant de l’eau (re = 1 000 kg . m −3) et de l’huile (rh = 750 kg . m−3 ) non miscibles. Trouver la position de repos de la bille ; on calculera la fraction x du volume qui est immergé dans l’eau. P28– 13. Statique d’une sphère flottant sur un liquide Une sphère pleine homogène de rayon R et de masse volumique r s peut flotter dans un liquide de masse volumique re . On désigne par X la partie du diamètre vertical immergé et on pose a = r s /re . 1. Pour quelles valeurs de a y a-t-il immersion totale et demi-immersion ? 2. Montrer que, si la sphère est immobile, X satisfait à l’équation b − X = c/X 2. Trouver b et c en fonction de R et a. 3. La sphère est immobile pour X tel que 0 < X < 2R. On l’enfonce légèrement et on l’abandonne. Quel est son mouvement ?

502

28. Introduction à la mécanique des fluides. Statique des fluides

P28– 14. Extraction d’un liquide d’un récipient cylindrique Un récipient cylindrique, de diamètre D = 0, 8 m et de hauteur h = 1, 2 m, contient de l’eau sur une hauteur l0 = 1 m. La pression de l’air au-dessus de l’eau, que l’on considère comme un gaz parfait, est la pression atmosphérique p a = 1 bar. Ce récipient est muni d’une ouverture sur sa face supérieure et d’un robinet, en bas, par lequel peut s’écouler l’eau. On ferme l’ouverture à l’aide d’un bouchon et on ouvre le robinet. Quel est le volume de liquide que l’on peut extraire du récipient ? P28– 15. Barrage à profil parabolique Un barrage, de largeur a = 15 m, a un profil parabolique d’équation : z y 2 −1 = h0 b où h0 = 12 m et b = 4 m (Fig. 28.23). Calculer la somme des forces exercées par l’eau : on déterminera les composantes verticale et horizontale. g

z O

y

b Eau g h

Béton

h0

H

L F IG . 28.23.

F IG . 28.24.

P28– 16. Barrage en béton à profil triangulaire Un barrage en béton, dont la section droite a une forme triangulaire (hauteur H = 20 m, longueur de l’assise L), retient de l’eau sur une largeur l = 100 m et sur une profondeur h = 15 m (Fig. 28.24). Le facteur de frottement statique entre le barrage et le sol est m s = 0, 1. Quelle doit être la valeur minimale de L pour que le barrage soit en équilibre ? P28– 17. Verre plein ou vide sur un flotteur Sur un flotteur en liège, au repos sur la surface libre de l’eau d’un cristallisoir, on a déposé un verre rempli d’eau jusqu’à une hauteur h (Fig. 28.25). La hauteur de l’eau dans le cristallisoir est z 1. On vide l’eau du verre dans le cristallisoir ; la nouvelle hauteur de l’eau dans le cristallisoir est z 2. 1. Écrire la relation exprimant la conservation de la masse. On notera h 1 et h2 les hauteurs immergées du flotteur dans les deux cas. 2. Traduire, dans les deux cas, la condition de repos du système. On désignera par M la masse de tout ce qui flotte en dehors de l’eau du verre. 3. Comparer z 1 et z 2.

503

Introduction à la mécanique des fluides. Statique des fluides z z a z1

h

g

g h

s S

0

R

F IG . 28.25.

F IG . 28.26.

P28– 18. Eau dans un entonnoir retourné Un entonnoir conique transparent, de masse M, est constitué d’un cône de hauteur h = 10 cm dont la base est un cercle de diamètre 2R = 10 cm (Fig. 28.26). Il repose par sa base sur un plan horizontal. On le remplit d’eau par le tube mince sans masse qui le surmonte. Quelle doit être la masse de l’entonnoir pour que le niveau de l’eau versée atteigne la hauteur zm = 8 cm avant que l’eau ne s’échappe entre le bord de l’entonnoir et le plan ? P28– 19. Eau dans une cloche hémisphérique Une cloche hémisphérique, de rayon R et de masse M = 2 kg, est posée sur une table lisse, la base plane et circulaire en bas (Fig. 28.27). On verse de l’eau par une ouverture supérieure jusqu’à une hauteur h. Quelle doit être la hauteur maximale hm au-delà de laquelle la cloche se soulève ? Commenter. z g A h

R

Eau Ω

 c Dunod – Toute reproduction non autorisée est un délit

F IG . 28.27.

F IG . 28.28.

P28– 20. Effet paradoxal de la force centrifuge Un flacon cylindrique ouvert, de 30 cm de diamètre, est rempli d’eau. Il comporte en son sein une balle de ping-pong très légère (rb = 500 kg . m−3 ) qui est maintenu à une hauteur de 50 cm par un fil (Fig. 28.28). On fait tourner le flacon autour de son axe avec la vitesse angulaire V. 1. Comment s’oriente, par rapport à la verticale du lieu, le fil à l’équilibre, dans le référentiel tournant ? Justifier. 2. À la place du flacon rempli d’eau, on met, sur le plateau en rotation, en dehors de l’axe, un petit pot de fleur contenant une tige souple longiligne. Comment s’incline la tige par rapport à l’axe vertical ?

29 Cinématique des fluides Nous nous proposons dans ce chapitre d’étudier les principaux types de mouvement de fluide, notamment en vue de préparer l’analyse dynamique du mouvement d’un fluide. Nous nous intéresserons en particulier aux écoulements stationnaires, irrotationnels, plans et incompressibles. Avant tout, il convient d’exprimer, dans le cadre de la description cinématique la plus adaptée, le champ des vitesses dans un fluide.

I . — CHAMP DES VITESSES DANS UN FLUIDE Rappelons que la description d’Euler, qui consiste à privilégier le champ des grandeurs caractéristiques d’un fluide, est généralement la plus commode pour étudier le mouvement d’un fluide dans son ensemble (cf. chapitre 28). En effet, on s’intéresse à la vitesse prise par une particule P lorsqu’elle se trouve en un point A fixe du référentiel d’analyse R (Fig. 29.1a). L’ensemble des vecteurs vitesses des particules en tout point de R constitue le champ des vitesses du fluide. Il s’écrit, v(r, t) en désignant par A le point fixe de R où se trouve la particule à l’instant t et par r le vecteur OA . z

z P

O x

P ( t + dt) P ( t)

v(A,t) A

A

O

y R

x

a)

A y

R b)

F IG . 29.1.

I . 1 . — Champ local a) Relation entre les composantes du champ des vitesses en deux points voisins Le champ local des vitesses dans un fluide est la relation entre les vitesses de deux particules de fluide P et P situées, au même instant t, en des points voisins A et A fixes dans le référentiel d’analyse R (Fig. 29.1b). En introduisant les vecteurs positions r et r + e de ces points, avec e = AA  , on écrit : v(A, t) = v(r, t) et v(A , t) = v(r + e, t)

505

Cinématique des fluides

Désignons par vx , vy , vz les composantes de v(A, t) dans la base de R et par v x , v y , vz les composantes de v(A , t) dans cette même base. À l’aide d’un développement de Taylor, calculons, au premier ordre près, la différence des premières composantes des vitesses des deux particules : vx − vx = e x

∂vx ∂vx ∂vx + ey + ez ∂x ∂y ∂z

ce qui s’écrit symboliquement vx − vx = (e · ∇)vx

ex , ey, e z étant les composantes de e et ∇ l’opérateur vectoriel nabla de composantes ∂/∂x, ∂/∂y, ∂/∂x. De même, on a, pour les autres composantes : v y − vy = (e · ∇)vy

et v z − v z = (e · ∇)v z

b) Expression du champ local des vitesses Afin de comparer le champ local des vitesses dans un fluide au champ des vitesses dans un solide, il est judicieux d’écrire vx − v x = (e · ∇)vx sous la forme suivante : 1 2

∂vx ∂vz − ∂z ∂x

ez −

1 2

∂vy ∂v ∂v 1 ∂v y ∂vx 1 − x e y + x ex + + ey + 2 ∂x 2 ∂x ∂y ∂x ∂y

∂v x ∂v + z ∂z ∂x

ez

Cette expression suggère d’introduire : i) le vecteur vorticité V , ou vecteur tourbillon, défini selon : V=

1 rot v 2

Les composantes de V dans la base de R sont alors : Vx =

1 2

∂v z ∂vy − ∂y ∂z

Vy =

1 2

∂vx ∂v z − ∂z ∂x

Vz =

1 2

∂vy ∂v x − ∂x ∂y

ii) d’autre part les quantités suivantes : a1=

∂vx ∂x

a2 =

∂v y ∂y

a3 =

∂vz ∂z

et

 c Dunod – Toute reproduction non autorisée est un délit

g1 =

1 2

∂v y ∂vz + ∂z ∂y

g2 =

1 2

La différence v x − vx s’écrit alors :

vx − v x = V y ez − Vzey + a 1ex + g 3e y + g2 e z

∂vz ∂vx + ∂x ∂z

d’où

g3 =

1 2

∂vx ∂vy + ∂y ∂x

vx = vx + Vyez − Vz e y + a 1ex + g3e y + g2 ez

On obtient par permutation circulaire les équations analogues pour les deux autres composantes : v y = vy + Vz e x − Vxe z + a2e y + g1e z + g3 ex

et v z = v z + Vx e y − Vy ex + a3 ez + g 2ex + g1e y

Apparaît naturellement le vecteur d dilatation-déformation du fluide, de composantes : d x = a 1e x + g3e y + g2 ez

dy = g 3e x + a 2ey + g1e z

dz = g2e x + g1ey + a 3ez

Il en résulte que le champ local des vitesses dans un fluide s’écrit : v(A, t) = v(A, t) + V × AA  + d

Le champ local des vitesses d’un fluide est donc la somme du champ de vitesse caractéristique d’un solide, c’est-à-dire une translation et une rotation, et d’un champ de dilatation-déformation. Ce dernier

506

29. Cinématique des fluides

dérive d’une certaine fonction de déformation f : d = grad f

soit

dx =

∂f ∂e x

dy =

∂f ∂ey

dz =

∂f ∂ez

avec :

1 (a1 e 2x + a2 e2y + a3 e 2z ) + g1 ey ez + g 2 exez + g3 ex e y 2 Le champ local des vitesses s’écrit, en introduisant la fonction déformation : f=

v(A , t) = v(A, t) + V × AA + grad f Notons que le champ vectoriel dilatation-déformation d et le champ e = AA  n’étant pas colinéaires, la relation linéaire entre d et e est de nature tensorielle : d = [D] e On peut alors mettre [D] , appelé tenseur dilatation-déformation sous la forme d’une somme [D] = [A] + [G] d’un tenseur dilatation [A] et d’un tenseur déformation [G] . Dans la base de R , ces tenseurs s’explicitent respectivement selon une matrice diagonale et une matrice à termes diagonaux nuls : a1 0 0 0 g3 g 2 g3 0 g 1 0 a2 0 [A] → [G] → 0 0 a 0 R R g2 g 1 3 Ainsi, le champ des vitesses dans un fluide a pour expression : v(A , t) = v(A, t) + V × AA + [D]AA  Remarque : L’expression précédente du champ des vitesses dans un fluide a suggéré à certains auteurs l’introduction d’un nouvel opérateur différentiel, le tenseur-gradient d’un vecteur, t-grad, tel que : d v = t-grad d r = [T] d r à la manière de d f = grad f · d r. I . 2 . — Lignes de courant. Tube de courant Les lignes de courant sont les lignes qui, à un instant fixé, sont tangentes, en chaque point, au vecteur vitesse. Elles sont donc définies par l’équation : d r = l v(r, t), l étant un coefficient de proportionnalité. En coordonnées cartésiennes, elles satisfont aux équations : dx dy dz = = v x (r, t) v y (r, t) vz (r, t) Il ne faut pas confondre une ligne de courant et la trajectoire d’une particule. Rappelons que la trajectoire est l’ensemble des positions prises par une particule déterminée P au cours du temps. La figure 29.2a représente la trajectoire d’une particule P passant par les points voisins A et A  aux instants t et t + d t , avec les vitesses respectives : vP(t) = v(A, t)

et v P(t + d t) = v(A, t + d t)

On visualise cette trajectoire en photographiant, par des instantanés successifs réguliers, la particule P que l’on a préalablement identifiée, à l’aide d’une couleur par exemple.

507

Cinématique des fluides

Les lignes de courant donnent, elles, à un instant fixé, la direction de la vitesse des particules en tout point occupé par le fluide ; elles représentent donc la topographie du champ des vitesses à cet instant. On visualise les lignes de courant en photographiant, en instantané (faible durée de pose), des particules métalliques, de masse volumique voisine de celle du fluide, par exemple de la poudre d’aluminium. Chaque particule donne sur la photographie un petit trait (de longueur proportionnelle à la faible durée de pose) orienté selon la vitesse au point considéré. On obtient ainsi à cet instant la topographie du champ des vitesses. Sur la figure 29.2b, on a représenté la ligne de courant tangente aux vecteurs vitesses des particules P et P qui se trouvent, au même instant, respectivement aux points A et A  . On a : v P(t) = v(A, t)

et vP (t) = v(A , t)

Notons que deux lignes de courant ne peuvent se couper en un même point, car la vitesse devrait avoir en ce point deux directions différentes, ce qui est impossible. vP (t + dt)

v(A ,t)

v(A, t)

vP (t)

A

A A

A Trajectoire de P

Ligne de courant

a)

b) F IG . 29.2.

Tube de courant F IG . 29.3.

Si le champ de vitesse est stationnaire, c’est-à-dire s’il ne dépend pas explicitement du temps, les lignes de courant et les trajectoires coïncident, puisque : vP (t + d t) = v(A , t + d t) = v(A , t)

 c Dunod – Toute reproduction non autorisée est un délit

L’ensemble des lignes de courant qui s’appuient sur un contour fermé forme un tube de courant (Fig. 29.3). Remarque : Les lignes de courant de matière rappellent les lignes de courant de charge dans les conducteurs électriques (cf. Électromagnétisme). I . 3 . — Influence des obstacles solides sur les lignes de courant Dans la plupart des écoulements, le fluide évolue autour d’obstacles fixes ou mobiles par rapport au référentiel d’analyse. En raison de forces de frottement entre le fluide et l’obstacle, on admet que les particules de fluide, qui sont en contact avec les points de ces obstacles, ont même vitesse que ces derniers : la différence des vitesses des particules fluide et solide en contact est donc nulle. Des obstacles solides fixes, placés normalement à la direction des lignes de courants, produisent des points d’arrêt, c’est-à-dire des points où la vitesse du fluide est nulle (Fig. 29.4).

508

29. Cinématique des fluides

Obstacle A Point d'arrêt

F IG . 29.4.

II . — ACCÉLÉRATION D’UNE PARTICULE DE FLUIDE II . 1 . — Expression de l’accélération d’une particule de fluide En description lagrangienne, la particule de fluide a sa position caractérisée par le vecteur r, fonction du temps et de sa position initiale r0 , comme c’est le cas habituellement en mécanique. Son accélération s’obtient aisément en dérivant r deux fois par rapport au temps. En description eulérienne, l’expression de l’accélération s’obtient à partir de la fonction vectorielle v(r, t). La dérivée partielle ∂ v/∂t, qui a un sens mathématique précis (les autres variables sont maintenues fixes), désigne concrètement le taux de variation de v lorsqu’on se place au point A fixe par rapport au référentiel R d’analyse. La différentielle totale d v , qui s’écrit : dv =

∂v ∂v ∂v ∂v dt + dx + dy + dz ∂t ∂x ∂y ∂z

donne la variation élémentaire de v lorsque les quatre variables x, y, z, t varient de façon infinitésimale. On en déduit la dérivée (totale) : dv ∂v ∂v = + dt ∂t ∂x

dx dt

+

∂v ∂y

dy ∂v + dt ∂z

dz dt

On obtient ainsi le taux de variation de v , entre les deux instants voisins t et t + d t et pour deux points infiniment voisins A et A de R : AA  = d x e x + d y ey + d z e z.

Si A et A  représentent les positions successives d’une même particule de fluide P, aux instants successifs t et t + d t, les quantités d x/ d t, d y/ d t et d z/ d t sont les composantes de la vitesse de cette particule. La dérivée totale précédente est alors le taux de variation de v lorsqu’on suit la particule, c’est-à-dire son accélération ; aussi appelle-t-on cette dérivée totale la dérivée particulaire que l’on note : Dv ∂v ∂v ∂v ∂v = + vx + vy + vz a= Dt ∂t ∂x ∂y ∂t Ainsi, l’accélération comporte deux termes : le premier représente le taux de variation au cours du temps de la vitesse en un point fixe de l’espace, alors que le second, dit contribution advective, représente le taux de variation dans l’espace de la vitesse à un instant fixé. Ainsi : a=

∂v + (v · ∇)v ∂t

avec

v · ∇ = vx

∂ ∂ ∂ + vy + vz ∂x ∂y ∂z

Cette dernière expression peut être mise sous une autre forme, en utilisant la relation vectorielle : (v · ∇)v =

1 grad v 2 + rot v × v 2

509

Cinématique des fluides On a, en effet, en explicitant la première composante cartésienne du membre de droite : 1 ∂ 2 (v + v 2y + v2z ) + vz 2 ∂x x

∂v x ∂vz − ∂z ∂x

− vy

∂v y ∂vx − ∂x ∂y

= vx

∂v x ∂v x ∂vx + vy + vz ∂x ∂y ∂z

ce qui est précisément la première composante du membre de gauche. En procédant de même pour les deux autres composantes, on trouve l’égalité recherchée. Retenons donc les deux expressions suivantes de l’accélération : a=

∂v + (v · ∇)v ∂t

et

a=

1 ∂v + grad v 2 + rot v × v 2 ∂t

Remarque : Les expressions de l’accélération sont évidemment identiques dans les deux descriptions lagrangienne et eulérienne. On peut le vérifier sur l’exemple simple d’une bille abandonnée en chute libre selon une trajectoire rectiligne descendante Ox . On sait que, le long de l’axe Ox , en description lagrangienne, on a : x = gt 2 /2, v = gt et a = g On trouve évidemment v et a en dérivant une fois, puis deux fois x(t) . En description eulérienne, on aurait, en fonction de x et t : v(x, t) = gt =

2x 2x t= 2 t t

d’où

a=

−2x 2x 2 ∂v ∂ × =g +v v = 2 + ∂t ∂x t t t

II . 2 . — Dérivée particulaire d’une grandeur quelconque Considérons une fonction scalaire quelconque qui dépend des variables d’Euler, par exemple la masse volumique : r(r, t). Sa dérivée particulaire s’écrit, en procédant comme précédemment : Dr ∂r ∂r ∂r ∂r = + vx + vy + vz ∂t ∂x ∂y ∂t Dt

soit

Dr ∂r = + (v · ∇)r ∂t Dt

 c Dunod – Toute reproduction non autorisée est un délit

Remarques : 1) Le résultat précédent se généralise à tout autre grandeur vectorielle, la force par exemple, ou scalaire comme la température. 2) On emploie parfois le qualificatif convective au lieu d’advective, ces deux mots ayant des racines latines en rapport avec le verbe venir. On utilise le plus souvent le mot advection pour exprimer un transport de matière et on réserve le mot convection si ce transport de matière est accompagné d’une variation de température. II . 3 . — Régimes stationnaires Les régimes stationnaires sont définis par la valeur constamment nulle de la dérivée partielle par rapport au temps de tout champ : ∂v =0 ∂t

∂r =0 ∂t

···

En un point donné, les champs n’évoluent pas. La dérivée particulaire se réduit alors à sa contribution advective : pour un champ des vitesses, cette contribution s’écrit (v · ∇)v ; Elle joue un rôle essentiel en régime stationnaire lorsqu’un fluide s’écoule à travers un étranglement puisque a se réduit alors à (v · ∇)v (Fig. 29.5).

510

29. Cinématique des fluides

S1

S2 F IG . 29.5.

III . — BILAN DE MASSE. DÉBIT-MASSE III . 1 . — Caractère conservatif de la masse Rappelons que la masse d’un système, en mécanique newtonienne, est une grandeur conservative, c’est-à-dire que, dans le bilan de masse d’un système, entre les instants voisins t et t + d t, le terme de création ou de production de masse est nul (cf. chapitre 22) : d M = dM(r) + dM(c)

avec

dM (c) = 0

Autrement dit, la variation de masse d’un système ne peut être attribuée qu’à la masse échangée, précisément reçue, à travers la surface qui le délimite. Remarque : En relativité, un bilan analogue est inclus naturellement dans celui de l’énergie. III . 2 . — Vecteur courant volumique de masse. Débit-masse, débit-volume On appelle vecteur courant volumique de masse le vecteur défini par : Jm = r v où r est la masse volumique et v la vitesse de la particule P au point A de R où elle se trouve. Il s’agit là aussi, comme pour v , d’un champ de vecteur ; on précisera parfois Jm (A, t) ou J m(r, t). Le débit-masse q m est la masse de la quantité de matière qui traverse toute surface pendant l’unité de temps ; on l’exprime donc en kg . s−1. Considérons un élément de surface d S , de normale n (Fig. 29.6). Pendant la durée élémentaire d t, la quantité de matière qui traverse la surface élémentaire d S est contenue dans le cylindre de base d S et de hauteur v · n d t = vn d t. Par conséquent, le débit-masse élémentaire dqm est : dqm =

d 2m r vn d t d S = = rv n d S = J m · n d S dt dt

Il en résulte que, pour une surface quelconque S : qm = S

Jm · n d S =

S

rv · n d S

Ainsi, le débit-masse à travers une surface est le flux du courant volumique de masse à travers cette surface. Remarque : Soulignons à nouveau l’analogie avec l’électrocinétique : l’intensité du courant électrique, qui est un débit de charge à travers une surface, est le flux du courant volumique de charge à travers cette surface.

511

Cinématique des fluides

On s’intéresse parfois au débit-volume qui représente le volume de matière qui traverse une surface pendant l’unité de temps. D’après ce qui précède, il a pour expression : qv = S

v ·n dS

Pour un écoulement incompressible, la relation entre qm et qv est simple : qm = rqv . v dt

n Jm

dS

n

n

dS

S

Jm

d

S

S F IG . 29.6.

n

F IG . 29.7.

F IG . 29.8.

III . 3 . — Bilan de masse a) Écriture locale du bilan de masse Considérons le système ouvert, de volume , délimité par une surface S , fixe dans R (Fig. 29.7). La masse étant une grandeur conservative, sa variation, pendant la durée d t est due à la masse qui pénètre dans le système à travers S pendant cette même durée (cf. chapitre 22) : d M = dM(r)

avec

dM = d

rd

et dM (r) = − d t

S

Jm · n d S

n étant la normale sortante. Il en résulte, en utilisant la formule d’Ostrogradsky (cf. annexe 4) et en permutant les opérateurs d / d t et intégration : ∂r + div Jm d = 0 ∂t Comme est quelconque, on en déduit localement la relation suivante, appelée souvent l’équation de continuité relative à la masse :

 c Dunod – Toute reproduction non autorisée est un délit

∂r = − div Jm ∂t

ou

∂r = − div(rv) ∂t

Remarques : (1) La permutation des opérateurs d / d t et intégration ne pose pas de problème car S est fixe dans R ; en outre, l’opérateur d / d t a dû être transformé en ∂/∂t car l’intégrale en volume ne dépend que de t, alors que r dépend de t mais aussi de x, y et z. (2) L’ensemble des résultats obtenus dans ce paragraphe rappelle des résultats analogues relatifs à la charge électrique : conservation de la charge, débit-charge appelé intensité de courant, équation de continuité correspondante (cf. Électromagnétisme). b) Régime stationnaire En régime stationnaire, l’équation de conservation de la masse se réduit aux équations intégrale et locale suivantes : Jm · n d S = 0 ou div Jm = 0 S

Ainsi, le flux de Jm à travers une surface fermée est nul. Si l’écoulement est incompressible ou si le fluide est un liquide de compressibilité négligeable, la relation précédente se réduit à : S

v · n d S = 0 ou

div v = 0

512

29. Cinématique des fluides

Il en résulte que le flux de ce vecteur à travers deux surfaces ouvertes quelconques, s’appuyant sur un même contour, est le même (Fig. 29.8) ; pour établir ce résultat, il suffit de noter que le flux à travers la surface fermée, constituée des deux surfaces ouvertes, est nul et de noter que le vecteur unitaire de l’une des surfaces est l’opposé de celui de la surface fermée aux mêmes points. Le débit-masse ne dépend donc pas de la surface ouverte qui s’appuie sur un contour déterminé. Exemple : Considérons un fluide s’écoulant dans une canalisation de section variable, en régime stationnaire (Fig. 29.5). Supposons que Jm soit uniforme sur des sections transversales éloignées de l’étranglement. On a, d’après ce qui précède, avec des notations évidentes : S

Jm · n d S = −J m,1 S 1 + Jm,2 S 2 = 0

puisque le flux de Jm à travers la surface latérale est nul. Finalement, on trouve : r1 v 1S 1 = r2 v 2S2. Si r est constant, la relation précédente se réduit à : v 1S 1 = v 2S2 . Ainsi, dans l’étranglement précédent, le rapport v 2/v 1 des vitesses est égal au rapport inverse S1 /S2 des surfaces des sections droites. Ce résultat peut être mis en évidence à l’aide d’une seringue de gros diamètre surmonté d’un tuyau de plus faible diamètre. Une fois la seringue remplie d’eau colorée, on peut vérifier que le rapport des déplacements du ménisque dans la seringue et dans le tube sont dans le rapport inverse des sections. c) Autre écriture de l’équation locale du bilan de masse L’équation locale de conservation peut s’écrire autrement en fonction de la dérivée totale de la masse volumique. En effet (cf. annexe 4) : ∂r = − div(rv) = −∇ · (rv) = −r∇ · v − (v · ∇)r ∂t donnent, en éliminant ∂r/∂t : Dr = −r∇ · v Dt

ou

et

D r ∂r = + (v · ∇)r Dt ∂t

1 Dr = − div v r Dt

IV . — DIFFÉRENTS TYPES D’ÉCOULEMENTS A priori, les écoulements dépendent de trois variables dans l’espace physique habituel. Cependant, de nombreux écoulements réels importants ne dépendent que de deux variables, en raison d’une symétrie plane ou cylindrique. IV . 1 . — Écoulements bidimensionnels a) Écoulements plans Un écoulement est plan lorsque le vecteur vitesse v(r, t) est constamment parallèle à un plan fixe et ne varie pas en tout point d’un axe perpendiculaire à ce plan. Notant Q = Oxy ce plan et Oz l’axe normal à Oxy, on a donc (Fig. 29.9a) : v(r, t) = v(x, y, t)

et v(x, y, t) · ez = 0

Ce type d’écoulement est très intéressant car il correspond à un modèle utile d’écoulement réel.

513

Cinématique des fluides b) Écoulements à symétrie de révolution

Un écoulement présente une symétrie de révolution si on peut décrire le champ de vitesse v, dans un demi-plan méridien, à l’aide des seules coordonnées polaires cylindriques (r, z) avec r = (x 2 + y2 )1/2 (Fig. 29.9b) ; la contribution orthoradiale vw de la vitesse ne dépend pas de la coordonnée w.

z z vz

v(x, y, t) y

r w

Q

x a)

vw vr

b) F IG . 29.9.

IV . 2 . — Écoulement irrotationnel. Fonction potentiel des vitesses Dans le domaine occupé par un fluide, un écoulement est irrotationnel si le vecteur tourbillon V du champ de vitesse est nul : V=0

ou

rot v = 0

Cette condition entraîne une simplification notable de l’expression de l’accélération :

 c Dunod – Toute reproduction non autorisée est un délit

a=

∂v 1 + grad v 2 ∂t 2

Les écoulements peuvent être considérés comme irrotationnels dans de nombreux cas. Les plus simples sont les écoulements selon un champ de vitesse uniforme (Fig. 29.12) ou selon un champ de vitesse radial (Fig. 29.13), comme nous le verrons un peu plus loin. Il existe d’autres exemples importants d’écoulements irrotationnels : l’écoulement d’air autour d’une aile d’avion, la propagation des ondes sonores dans l’air ou dans l’eau et la propagation des ondes à la surface de l’eau. On trouve l’expression intégrale caractéristique d’un mouvement irrotationnel en utilisant l’égalité vectorielle de Stokes-Ampère ; cette dernière traduit l’égalité entre la circulation d’un vecteur le long d’une courbe fermée C et le flux de son rotationnel à travers une surface ouverte S qui s’appuie sur C (cf. annexe 5) : C

v·dr=

S

rot v · n d S

d’où C

v·dr= 0

Ainsi, dans un écoulement irrotationnel, la circulation du vecteur vitesse le long d’une courbe fermée est nulle. Comme le rotationnel d’un gradient est toujours nul, le champ des vitesses peut se mettre sous la forme : v = − grad F où F(r, t) est le potentiel des vitesses, par analogie avec l’énergie potentielle E p dont dérive une force conservative F = − grad Ep. Les surfaces F = Cte, à un instant donné, sont équipotentielles et le vecteur v leur est normal, puisque sur de telles surfaces on a : grad F · d r = −v · d r = 0.

514

29. Cinématique des fluides En coordonnées cartésiennes, v s’écrit : vx= −

∂F ∂x

∂F ∂y

vz = −

∂F vw = − r∂w

vz = −

vy = −

∂F ∂z

et en coordonnées polaires cylindriques (Fig. 29.9b) : ∂F vr = − ∂r

∂F ∂z

Remarque : Certains auteurs définissent F par la relation v = grad F sans le signe moins. Cela ne modifie en rien l’analyse, mais le nom de potentiel des vitesses est alors peu adapté si l’on souhaite souligner l’analogie entre le champ des vitesses v = − grad F en cinématique des fluides et le champ des courants volumiques J = gE = −g grad V dans les conducteurs ohmiques, en électrocinétique stationnaire (cf. Électromagnétisme). On a bien, de façon analogue, div v = 0 et div J = 0. IV . 3 . — Écoulement incompressible. Fonction courant de vitesse Un écoulement est incompressible si la masse volumique r d’un élément de fluide est constante au cours de son mouvement, c’est-à-dire si : Dr =0 Dt Il concerne principalement les liquides, mais aussi les gaz dans la mesure où les variations de pression ne sont pas trop importantes. Cette condition est peu restrictive puisque, dans la plupart des cas pratiques importants, elle est vérifiée. Il en résulte, puisque D r/ D t = −r div v : div v = 0

ou

v = rot A

la divergence du rotationnel d’un champ de vecteurs A(r, t) étant toujours nulle (cf. annexe 4). Par conséquent, l’égalité vectorielle de Stokes-Ampère donne, si S est une surface quelconque qui s’appuie sur un contour fermé C : C

A·dr=

S

rot A · n d S =

S

v ·n dS

Si le contour plan est contenu dans le plan de l’écoulement, le flux de v à travers la surface plane qu’il délimite est nul (Fig. 29.10). Il en résulte que l’intégrale curviligne est nulle. Comme le contour considéré a une forme plane quelconque, A est normal au plan de l’écoulement. Ce vecteur peut donc se mettre sous la forme A = −C ez, C étant une fonction des coordonnées dans le plan appelée fonction courant des vitesses. L’équation v = rot(−Cez) donne alors, en coordonnées cartésiennes : vx = −

∂C ∂y

et

vy =

∂C ∂x

515

Cinématique des fluides Ψ = Cte

n

– = Cte A

C

v

F IG . 29.10.

F IG . 29.11.

Les lignes définies par C = Cte sont les lignes de courant, tangentes au vecteur vitesse et perpendiculaires aux lignes équipotentielles ; en effet, pour tout déplacement d r sur une ligne de courant, on a, puisque d C = 0 : dC =

∂C ∂C dx + d y = vy d x − vx d y = 0 soit ∂x ∂y

dx dy = vx vy

Par conséquent, d r et v sont colinéaires et normaux aux lignes équipotentielles (Fig. 29.11). En coordonnées polaires cylindriques, on a (cf. annexe 3) : vr =

1 ∂(−C) 1 ∂C =− r ∂u r ∂u

vu = −

∂(−C) ∂C = ∂r ∂r

D’autre part, l’équation d’une ligne de courant, dans ce système de coordonnées, s’écrit : dr rdu = vr vu Dans ce contexte, il est utile de rappeler ici l’expression de la divergence d’un vecteur en coordonnées polaires cylindriques (r, u, z) (cf. annexe 4) :

 c Dunod – Toute reproduction non autorisée est un délit

div v =

∂vz 1 ∂ (rv r) 1 ∂vu + + r ∂r r ∂u ∂z

IV . 4 . — Écoulements laminaires et turbulents Un écoulement est laminaire si sa vitesse est régulière dans le temps et dans l’espace. Dans le cas contraire, il est turbulent, c’est-à-dire de vitesse chaotique dans l’espace et dans le temps. Ces deux types d’écoulement, auxquels on associe souvent les noms de Poiseuille et Venturi respectivement, s’observent aisément lorsqu’on verse le contenu d’une bouteille d’eau ou lorsqu’on ouvre un robinet : si la vitesse de l’écoulement est faible le régime est laminaire, sinon il est turbulent. Dans le cas de l’huile, on constate que l’écoulement est laminaire pour une vitesse plus grande, ce que l’on explique par la grande viscosité de l’huile. Il existe un critère numérique permettant de distinguer ces deux types d’écoulement ; c’est le nombre de Reynolds (cf. chapitre 31). IV . 5 . — Écoulements irrotationnels, incompressibles, plans et stationnaires Les écoulements irrotationnels, incompressibles, plans et stationnaires constituent très souvent une bonne approximation d’écoulements réels de liquides ou de gaz.

516

29. Cinématique des fluides

a) Équations différentielles des fonctions potentiel et courant des vitesses Éliminons la vitesse entre les équations v = − grad F et div v = 0. Il vient : − div grad F = 0 d’où

F = 0

Cette dernière équation s’explicite, en coordonnées cartésiennes dans le plan, selon : ∂ 2F ∂2 F =0 + ∂x2 ∂y 2 Cette équation, connue sous le nom d’équation de Laplace, rappelle celle à laquelle satisfait le potentiel gravitationnel en l’absence de masses (cf. chapitre 6) ou le potentiel électrique en l’absence de charges (cf. Électromagnétisme). De même, si on élimine v entre les deux équations v = rot(−C e z) et rot v = 0, on trouve : rot rot(C ez) = grad div(C e z) − C ez = 0 avec

div(C ez) =

∂C =0 ∂z

On obtient ainsi une équation analogue pour la fonction C : C = 0

ce qui s’explicite selon

∂ 2C ∂2 C + =0 ∂x2 ∂y2

en coordonnées cartésiennes dans le plan. b) Superposition de deux champs de vitesses Si F 1 et F2 sont des solutions de l’équation de Laplace, toute combinaison linéaire de F1 et F2 est aussi solution de cette équation. En effet, comme c’est une équation linéaire, on a : D(a1F 1 + a2F 2) = a 1DF 1 + a 2DF 2 = 0 Il en est de même pour la fonction courant. Il en résulte que toute combinaison linéaire de solutions simples de ces équations peut représenter l’écoulement d’un fluide. Les fonctions F et C satisfont à des relations analogues ; aussi introduit-on souvent la fonction complexe qui les regroupe toutes les deux : W (z) = F(x, y) + jC(x, y)



z = x + jy

On l’appelle le potentiel complexe des vitesses. On montre en effet que les relations précédemment établies : ∂F ∂C ∂F ∂C et vy = − vx = − =− = ∂x ∂y ∂y ∂x permettent de définir la fonction W (z) de la variable complexe z. c) Conditions aux limites Pour résoudre l’équation de Laplace à laquelle satisfont les deux fonctions F et C, on s’appuie largement sur des considérations de symétrie, comme on le fait en électrostatique ou en magnétostatique (cf. Électromagnétisme). En outre, on tient compte des conditions aux limites imposées à ces fonctions. Ces conditions sont de deux types : i) les solutions cherchées en présence d’obstacles doivent redonner les solutions théoriques en leur absence lorsque ces obstacles sont éloignés du point considéré, ii) les particules de fluide en contact avec l’obstacle solide ont même vitesse orthogonale v n que les particules de l’obstacle, ce qui traduit l’impénétrabilité de l’obstacle.

517

Cinématique des fluides

V . — EXEMPLES DE CHAMPS DE VITESSE IRROTATIONNELS Nous décrivons quelques champs de vitesse irrotationnels importants à l’aide des fonctions potentiel des vitesses F et courant des vitesses C : le champ de vitesse uniforme, le champ de vitesse radial et le champ de vitesse dipolaire. V . 1 . — Champ de vitesse uniforme Choisissons un système d’axes Oxy tel que v x = v0 et vy = 0 (Fig. 29.12). Comme v0 = −∂ F/∂x, il vient, en intégrant : F = −v 0 x + f (y) Or −∂F/∂y = 0 entraîne que ∂ f /∂y = 0, d’où f (y) = Cte et : F = −v 0 x + Cte Ainsi, les lignes de potentiel des vitesses sont des droites parallèles à l’axe Oy. Quant aux lignes de courant, on les détermine de façon analogue : v0 = −

∂C ∂y

entraîne

avec

C = −v 0y + g(x)

∂C =0 ∂x

Par conséquent : d g/ d x = 0 et g(x) = Cte. Finalement : C = −v 0 y + Cte Ainsi, les lignes de courant sont des droites parallèles à l’axe Ox. Les constantes étant arbitraires, on peut les choisir nulles. Retenons donc : C = −v 0 y

F = −v 0 x

et

W (z) = −v 0 z

y

y

Ψ  c Dunod – Toute reproduction non autorisée est un délit

Ψ O

v0

θ

– x

x

O –

F IG . 29.12.

F IG . 29.13.

V . 2 . — Champ de vitesse radial. Source et puits Choisissons l’origine du système d’axes au point O où les directions des vitesses sont concourantes (Fig. 29.13). On dit que O est une source ou un puits suivant que les vitesses divergent à partir de O ou convergent vers O. Il est alors naturel d’utiliser les coordonnées polaires (r, u) au lieu de (x, y). Comme v est radial, les lignes de courant sont les droites passant par O. L’équation de ces lignes est : C = C1u, C1 étant une constante.

518

29. Cinématique des fluides

Les lignes équipotentielles sont des cercles de centre O. Pour déterminer leur équation, il suffit d’écrire que le flux de v à travers un cylindre, d’axe Oz et de longueur l, ne dépend pas du rayon de base du cylindre : C2 er 2pr l v r = Cte d’où v = vr er = r C 2 étant une constante. Comme vu = −(1/r)∂F/∂u = 0, F ne dépend que de r. On en déduit alors : dF dr ∂F C2 vr = =− =− d’où d F = −C2 et F(r) = −C 2 ln r dr r ∂r r en intégrant. Les constantes C 1 et C2 qui apparaissent dans C et F sont directement reliées ; en effet, l’égalité : C2 −C 1 1 ∂C vr = − = donne r ∂u r r Par conséquent, C2 = −C 1 = K . Relions la constante K au débit-volume à travers un cylindre de rayon r et de hauteur l égale à l’unité : qv =

S

v ·n dS =

K er · n l r d u = K2p r

d’où

K=

qv 2p

Finalement, les fonctions C et F caractéristiques d’un champ radial sont : C=−

qv u = −K u 2p

F=−

qv ln r = −K ln r 2p

et

W = F + jC = −K ln r − jKu = −K ln z

puisque z = r exp(ju). Si q v est positif, vr = K /r > 0, l’écoulement est divergent : O est une source. S’il est négatif, vr < 0, l’écoulement est convergent : O est un puits. V . 3 . — Champ de vitesse dipolaire Considérons deux champs de vitesses radiales de même norme, l’un convergent d’origine un puits P, l’autre divergent d’origine une source S, P et S étant deux points distants de a ; on dit que P et S forment un doublet (Fig. 29.14). Si la distance a est très faible devant la distance d’observation r, le doublet devient un dipôle. A αp

αs

θp θs

x

P O S a F IG . 29.14.

D’après ce qui précède, le potentiel des vitesses en un point A de coordonnées polaires (r, u) est la somme des potentiels des vitesses : Fs = −K ln rs

et Fp = K ln r p



rs = SA

r p = PA

et K > 0.

519

Cinématique des fluides On a donc, puisque rp − rs ≈ a cos u  r : F = Fs + F p = −K ln

rs rp

= −K ln 1 −

rp − r s rp

soit

F≈

Ka cos u r

Quant à la fonction courant, elle s’écrit, avec les notations de la figure 29.14 : C = Cs + Cp = −K (us − up ) ≈ −K(u + as − u + ap ) = −K(as + ap) ≈ −K

a sin u r

puisque r(as + ap ) ≈ a sin u. En introduisant le moment dipolaire : m = m ex

avec m = Ka

et ex le vecteur unitaire orienté de P vers S, les fonctions courant et potentiel d’un dipôle s’écrivent respectivement : C=−

m sin u r

F=

m cos u r

et

et

vu = −

W (z) =

m exp(−ju) r

On en déduit les composantes de la vitesse : vr = −

∂F m cos u = ∂r r2

1 ∂F m sin u = r ∂u r2

Remarques : (1) Notons la différence entre le potentiel des vitesses de ce dipôle, qui est en 1/r, et le potentiel du dipôle en électrostatique qui est en 1/r 2 (cf. Électromagnétisme) ; cette différence est due à la symétrie cylindrique de ce problème ; rappelons que les potentiels produits par un puits ou une source sont logarithmiques et non en 1/r. (2) Le potentiel des vitesses d’un dipôle présente une singularité à l’origine O. Cependant la circulation de v le long d’un contour circulaire, de centre O et de rayon R, doit être nulle puisque le champ est irrotationnel. Vérifions-le : G=  c Dunod – Toute reproduction non autorisée est un délit

C

v·dr=

2p

v uR d u = C

0

m sin u m R du = 2 R R

2p

sin u d u = 0 0

VI . — ÉCOULEMENT D’UN FLUIDE AUTOUR D’UN OBSTACLE L’écoulement incompressible d’un fluide autour d’un obstacle cylindrique est un modèle important d’analyse. En effet, très souvent on peut ramener des cas concrets, tels que l’écoulement d’un fluide autour d’une aile d’avion, à un tel modèle par diverses transformations mathématiques. Les conditions auxquelles le champ des vitesses recherché doit satisfaire sont : i) Le champ des vitesses dérive d’un potentiel de vitesse F et d’une fonction courant C qui vérifient tous deux l’équation de Laplace. ii) Au-delà de la couche limite qui recouvre l’obstacle cylindrique, la vitesse doit être tangente à l’obstacle et donc la composante normale de v nulle ; le cercle C limitant la section droite du cylindre est par conséquent une ligne de courant. iii) Loin de l’obstacle le champ de vitesse doit être uniforme. Une première étape consiste à superposer un champ uniforme et un champ dipolaire.

520

29. Cinématique des fluides

VI . 1 . — Superposition d’un champ uniforme et d’un champ dipolaire Superposons les deux contributions d’un champ uniforme et d’un champ dipolaire. Il vient, en coordonnées polaires (r, u) : m sin u C(r, u) = −v0 r sin u − r puisque x = r cos u et y = r sin u et que l’origine des coordonnées est prise en O, centre du cercle C de l’obstacle (Fig. 29.15). On en déduit les composantes de la vitesse : vr = −

m 1 ∂C = cos u v0 + 2 r ∂u r

vu =

∂C m = sin u −v0 + 2 ∂r r

Comme vr (R) = 0, le moment dipolaire a la valeur m = −v0 R2 , R étant le rayon de l’obstacle. Il en résulte : C(r, u) = −v0 r sin u 1 −

R2 r2

vr = v0 cos u 1 −

R2 r2

et vu = −v0 sin u 1 +

R2 r2

On voit que pour r = R, c’est-à-dire en tout point du cercle C, C a une valeur constante. Ce cercle est donc une ligne de courant conformément à ce que nous attendions. Notons que la vitesse aux points A et A  du cercle, situés sur le diamètre parallèle à la vitesse v 0 , est nulle ; en effet, en ces points d’arrêt, r = R et u = p ou 0. Cette superposition représente approximativement ce que l’on observe, du seul point de vue cinématique, lorsqu’on perturbe l’écoulement uniforme d’un fluide par un cylindre dont l’axe est perpendiculaire au plan de l’écoulement. Notons que l’écoulement ainsi schématisé présente une symétrie cinématique par rapport au plan perpendiculaire à la direction moyenne du fluide passant par le centre O du cylindre. Selon ce modèle, le fluide en mouvement sur l’obstacle n’exercerait aucune force sur ce dernier. Ce résultat paradoxal est connu sous le nom de paradoxe de d’Alembert. On lève ce paradoxe en tenant compte de la viscosité du fluide (cf. chapitre 31). y – v0

C A

O

F IG . 29.15.

v θ A

µ O

x

F IG . 29.16.

VI . 2 . — Champ de vitesse d’un vortex On appelle vortex ou tourbillon-plan un écoulement plan caractérisé par une fonction courant C de la forme : C = K ln r K étant une constante (Fig. 29.16). Les lignes de courant sont donc des cercles concentriques centrés en O. Notons que, par rapport à l’écoulement radial, la géométrie de ces lignes est inversée. On a donc pour la vitesse : 1 ∂C ∂C K K vr = − = d’où v = e u =0 vu = r ∂u ∂r r r

521

Cinématique des fluides

Pour trouver l’expression du vecteur tourbillon, utilisons l’équation vectorielle de Stokes-Ampère sur un cercle de rayon r entourant l’origine O :

C

v·dr=

S

rot v · n d S

donne

2pr v u =

V 2 pr 2

d’où

V=

vu K = 2 r r

On exprime souvent la fonction courant C à l’aide de la circulation G de v le long d’un contour circulaire de centre O et de rayon quelconque R : G= C

2p

v·dr=

0

K R d u = 2pK R

On en déduit l’expression suivante de C : G ln r 2p

C=

Remarque : La circulation G a une valeur indépendante du rayon R, qui est différente de 0, en raison de la singularité à l’origine O où v est infini. VI . 3 . — Addition d’un vortex au centre Dans l’écoulement incompressible d’un fluide autour d’un obstacle cylindrique, la fonction courant C s’obtient en ajoutant, à la superposition d’un champ uniforme et d’un champ dipolaire, la contribution d’un vortex de constante K. On obtient, en introduisant par commodité la constante K ln(1/R) : C = −v0 r sin u 1 − On en déduit : vr = −

R2 1 ∂C = v0 cos u 1 − 2 r ∂u r

R2 r2

vu =

+ K ln

r R

∂C R2 = −v 0 sin u 1 + 2 ∂r r

+

K r

 c Dunod – Toute reproduction non autorisée est un délit

Déterminons la position des points d’arrêt. Les coordonnées de ces points satisfont aux équations suivantes qui traduisent l’annulation des composantes de la vitesse : cos u 1 −

R2 r2

= 0 et

− v 0 sin u 1 +

R2 r2

+

K =0 r

On distingue deux cas : cos u = 0 et cos u = 0. a) cos u = 0 En remplaçant sin u par ±1 dans la dernière équation, on trouve : ∓v0 1 +

R2 r2

+

K =0 r

soit

r2 ∓

K r + R2 = 0 v0

Cette équation du deuxième degré donne la solution physiquement acceptable suivante : 4R2 v20 K 1± 1− r= K2 2v0 puisque D = K2/v 02 − 4R 2  0 et r > R.

1/2

522

29. Cinématique des fluides

– i) Si v 0 < K /2R, l’équation précédente, écrite sous la forme (2v 0 r/K − 1)2 = 1 − 4R 2v02/K 2 , donne : R2 rv0 rv 0 R2 v 20 r2 1 + soit 1 + =1 = K2 r2 K K r2 On en déduit que : sin u =

K =1 rv 0 (1 + R2 /r2)

d’où

u=

p 2

Le point représentatif est le point A sur la figure 29.17a. – ii) Si v 0 = K /2R, alors r = K/(2v0 ) = R et : sin u =

K K p = = 1 d’où u = v 0r (1 + R2 /r 2) 2v0 R 2

Le point représentatif est le point A sur la figure 29.17b. y

y

y

A A Oz

A1

Oz

x

a)

A2 Oz

x

b)

x

c)

F IG . 29.17.

b) cos u = 0

On a, pour cos = 0, d’après les deux équations qui traduisent l’annulation de la vitesse : r=R

et

sin u =

K 2Rv0

ce qui n’a de sens que pour v 0  K /(2R). Les points représentatifs sont les points A 1 et A2 sur la figure 29.17c. Nous analyserons ultérieurement l’aspect dynamique de l’écoulement d’un fluide autour d’un obstacle cylindrique, notamment à propos de l’effet Magnus (cf. chapitre 30).

CONCLUSION Rappelons les résultats essentiels. (1) Le champ local des vitesses d’un fluide est celui d’un solide auquel il faut ajouter le vecteur dilatation-déformation d : v(A ) = v(A) + V × AA + d

avec

V=

1 rot v et d = [D]AA  2

[D] étant le tenseur dilatation-déformation. (2) Les lignes de courant sont, à chaque instant, parallèles en chaque point au vecteur vitesse.

523

Cinématique des fluides (3) L’accélération a d’une particule de fluide est la dérivée particulaire de la vitesse v : a=

∂v 1 ∂v + (v · ∇)v = + grad v2 + rot v × v 2 ∂t ∂t

(4) L’équation de continuité relative à la conservation de la masse s’écrit : ∂r = − div Jm ∂t

avec

Jm = r v

ou

Dr = −r div v Dt

(5) Les caractéristiques des différents types d’écoulement de fluide sont : i) écoulement stationnaire : ∂ v/∂t = 0, ii) écoulement plan : v(r, t) = v(x, y, t) et v(x, y, t) · e z = 0, iii) écoulement irrotationnel : rot v = 0 soit v = − grad F, F étant le potentiel des vitesses,

iv) écoulement incompressible : div v = 0 soit v = rot(−C e z ) pour un écoulement plan, C étant la fonction courant des vitesses, v) écoulement laminaire : vitesse régulière dans le temps et dans l’espace. (6) Les modèles simples d’écoulements stationnaires, incompressibles, irrotationnels et plans de fluides décrivent efficacement les écoulements de fluides dans des situations concrètes.

EXERCICES ET PROBLÈMES P29– 1. Lignes de courant dans un fluide En régime stationnaire, le champ de vitesse d’un fluide, en coordonnées cartésiennes, est donné par : vx = 2x, v y = 2y, vz = −z. Trouver l’équation de la ligne de courant qui passe par le point A de coordonnées (2, 1, 1).

 c Dunod – Toute reproduction non autorisée est un délit

P29– 2. Profil gaussien de vitesse d’un fluide dans une conduite Un fluide s’écoule dans une conduite cylindrique avec une distribution radiale des vitesses de la forme : v = v0 exp(−2r 2 /w2), r étant la coordonnée radiale et w une constante égale à la moitié du rayon r0 de la conduite. Calculer le rapport des vitesses v(r 0 )/v 0 et le débit-volume q v en fonction de v0 et r 0. En déduire la vitesse moyenne vm du fluide. P29– 3. Profil parabolique de vitesse d’un liquide dans une conduite Un liquide, de masse volumique r, s’écoule dans une conduite cylindrique avec une distribution radiale des vitesses de la forme : v = v 0(1 − r 2 /r20 ), r étant la coordonnée radiale et r0 le rayon de la conduite. 1. Trouver le débit-masse q m en fonction de r, v 0 et r 0. Quelle est la vitesse moyenne v m du liquide ? 2. Calculer le débit d’énergie cinétique. Le comparer à celui obtenu en supposant que la vitesse est uniforme et égale à la vitesse moyenne.

524

29. Cinématique des fluides

P29– 4. Accélération d’une particule de fluide dans un champ de vitesse radial Le champ de vitesse dans un fluide est un champ radial de source O. Il a pour expression, en un point A, en m . s−1 : 1 v = vr e r avec v r = 2 r r étant la distance OA. Trouver la valeur de l’accélération d’une particule P de fluide lorsqu’elle se trouve à 1 m de O. P29– 5. Variation de masse volumique d’un fluide Le champ de vitesse d’un fluide a pour expression, en coordonnées polaires et en cm . s −1 : 3 e r + 4 cos u eu r Calculer div v au point A, de coordonnées r = 200 m et u = p/6. En déduire le taux de variation de sa masse volumique en ce point où sa masse volumique vaut 100 kg . m−3 . v=

P29– 6. Écoulement incompressible d’air dans une conduite On considère un écoulement incompressible d’air dans une conduite cylindrique de 20 cm de diamètre, à une pression de 3 bar et une vitesse de 3 m . s−1 . Sachant que cet air, à la température de 293 K, peut être considéré comme parfait, calculer le débit-masse. P29– 7. Vitesses d’un liquide dans une conduite de section variable Dans une conduite de 15 cm de diamètre circule du pétrole avec un débit-volume de 100 L . s −1 . Le fluide passe ensuite dans une conduite de diamètre 7, 5 cm. Calculer les vitesses dans les deux conduites. P29– 8. Vecteur rotation d’un solide et vorticité Établir la relation entre le vecteur rotation V s d’un cylindre de révolution tournant autour de son axe de révolution et sa vorticité V . P29– 9. Champ de vitesse d’un fluide Dans un fluide, le champ de vitesse en coordonnées cartésiennes a pour expression : v = (y2 + z 2) e x + (x2 + z 2) e y + (x2 + y2) e z En déduire que ce champ est celui d’un fluide en écoulement incompressible. Quelle est l’expression du vecteur tourbillon ? P29– 10. Champ de vitesse dans une tornade On modélise le champ de vitesse, dans un plan horizontal, d’une tornade d’axe vertical selon : i) dans le cœur de la tornade, défini par r < r c, la vitesse orthoradiale est v c = Vt r, ii) à l’extérieur du cœur, r > r c : ve = K /r. Calculer K et le vecteur tourbillon V.

Cinématique des fluides

525

P29– 11. Superposition de deux champs de vitesse irrotationnels On superpose deux champs de vitesse irrotationnels, uniformes, bidimensionnels, de potentiels respectifs : F 1 = 30 x m 2 . s −1 et F 2 = 40 y m 2 . s−1 Déterminer le champ de vitesse résultant et la nature des lignes de champ. P29– 12. Superposition d’un écoulement uniforme et d’un écoulement radial divergent 1. Trouver le potentiel de vitesse et la fonction de courant qui caractérisent la superposition d’un écoulement uniforme de vitesse v 0 = v0 e x et d’un écoulement radial divergent. En déduire la vitesse en coordonnées polaires (r, u). 2. Quelle est la ligne de courant qui coïncide avec l’axe Ox pour r infini ? Trouver la position des points d’arrêt. P29– 13. Écoulement dans un dièdre droit Étudier cinématiquement l’écoulement stationnaire, irrotationnel, incompressible, plan d’un fluide pour lequel le potentiel complexe est : W (z) = K z2 /2. On rappelle que z = x + jy et W = F + jC, F étant le potentiel des vitesses et C la fonction de courant.

 c Dunod – Toute reproduction non autorisée est un délit

Trouver le champ des vitesses en coordonnées cartésiennes.

30 Équation d’Euler et relation de Bernoulli Dans ce chapitre, nous établissons d’abord l’équation d’Euler à laquelle satisfait le champ des vitesses d’un fluide non visqueux, c’est-à-dire d’un fluide dont les parties, de part et d’autre de la surface qui les sépare, n’exercent l’une sur l’autre que des forces normales à cette surface. Nous en déduisons la relation énergétique de Bernoulli, souvent plus facile à exploiter, notamment pour interpréter diverses expériences courantes dont certaines sont spectaculaires ; par exemple, si on envoie de l’air comprimé dans un entonnoir contenant une balle de ping-pong, on constate que la balle n’est pas expulsée mais aspirée au point de se maintenir immobile, dans le champ de pesanteur, une fois l’entonnoir retourné (Fig. 30.1). Air comprimé S 12 dS

g p < p0 Entonnoir

p0

(1)

F IG . 30.1.

f s,1→2 n1→2 (2)

S

F IG . 30.2.

I . — ÉQUATION D’EULER L’équation d’Euler est l’équation que fournit, dans la description d’Euler, la loi fondamentale de la dynamique de Newton lorsqu’on l’applique à un élément de fluide non visqueux. Aussi convient-il, avant tout, de préciser les caractéristiques d’un tel fluide. I . 1 . — Fluide non visqueux Un fluide est non visqueux si la force qu’exerce une partie (1) de fluide sur une partie (2), en contact avec la première par la surface élémentaire d S, se réduit à une force normale à d S (Fig. 28.2). Précisément : d F 1→2 = p d S n1→2 Dans ces conditions, sa viscosité, qui traduit un phénomène de transport de quantité de mouvement (cf. Thermodynamique), est nulle, ainsi que tout autre phénomène de transport diffusif. Une telle schémati-

527

Équation d’Euler et relation de Bernoulli

sation est certes utile dans une première analyse, mais son intérêt est limité, car certains phénomènes ne peuvent être interprétés qu’en introduisant la viscosité (cf. chapitre 31) : par exemple, un récipient rempli d’un liquide entraîne toujours ce dernier si on le fait tourner (Fig. 28.4c)). En outre, l’absence de tout transfert diffusif, par exemple thermique entre deux régions de températures différentes, combinée à celle de viscosité, exclut toute cause d’irréversibilité (cf. Thermodynamique). On pourra alors admettre que tout fluide (gaz ou liquide) suit une loi isentropique. Dans le cas d’un gaz parfait, au sens de la thermodynamique, une telle évolution satisfait à la loi de Laplace, laquelle relie la pression p et le volume V selon l’équation : pV g = Cte g étant le rapport des capacités thermiques à volume constant et à pression constante. Remarque : Précisons à nouveau que nous avons évité le qualificatif parfait, souvent employé à tort pour un fluide non visqueux, gaz ou liquide, afin d’éviter toute confusion avec le gaz parfait, au sens de la thermodynamique, dont la viscosité n’est pas nulle. I . 2 . — Équation d’Euler Appliquons la loi fondamentale de la mécanique à un élément de fluide non visqueux, de masse r d , dans un référentiel terrestre R approximativement galiléen. En l’absence de viscosité, cet élément est soumis à la force de pesanteur r g d et aux forces de pression − grad p d . Par conséquent : r d a = − grad p d + rg d En remplaçant l’accélération a par l’expression eulérienne suivante (cf. chapitre 29) : a=

∂v + (v · ∇)v ∂t

on obtient, après simplification, l’équation vectorielle d’Euler :

 c Dunod – Toute reproduction non autorisée est un délit

r

∂v + r(v · ∇)v = − grad p + rg ∂t

Cette équation vectorielle d’explicite en trois équations scalaires, alors que les inconnues sont au nombre de cinq : vx, v y, v z, p et r. Les deux équations qui manquent sont fournies par l’équation d’état du fluide, qui fait généralement intervenir la température, et par l’équation traduisant le caractère conservatif de la masse (cf. chapitre 29) : ∂r = − div J m = − div(rv) ∂t Cependant l’analyse complète est impossible car l’équation d’Euler n’est pas linéaire : en effet, le premier membre fait apparaître la vitesse au carré. Aussi préfère-t-on le plus souvent exploiter la relation énergétique de Bernoulli qui en découle aisément. I . 3 . — Méthodes d’analyse de la mécanique des fluides La résolution de l’équation d’Euler étant rendue impossible par la présence du terme non linéaire (v · ∇)v, il est judicieux d’évaluer qualitativement, avant tout traitement, les différents termes et de négliger ceux qui n’ont pas de contribution majeure. Il en résulte que, en mécanique des fluides, probablement plus qu’ailleurs, l’analyse dimensionnelle et la similitude jouent des rôles essentiels.

528

30. Équation d’Euler et relation de Bernoulli

a) Analyse dimensionnelle L’analyse dimensionnelle consiste à déterminer les dimensions physiques des grandeurs qui interviennent dans les équations du mouvement et à les comparer à des quantités de même dimension physique : on établit ainsi des critères portant sur des nombres sans dimension qui permettent de situer l’importance relative des différents termes qui figurent dans l’équation d’Euler. Ainsi plusieurs nombres sans dimension ont été introduits et définis en fonction de caractéristiques du fluide (masse volumique r par exemple) et des grandeurs particulières qui apparaissent naturellement (pression p, vitesse v, longueur L, champ de pesanteur g) : i) Si l’on compare grad p et ra, on a, dimensionnellement : || grad p|| r||a|| Aussi définit-on le nombre d’Euler par : dim

=

Eu =

p/L p = −2 r v2 rLT

p rv2

ii) Si l’on compare ra et rg, il vient, dimensionnellement : r||a|| rLT −2 L2 T−2 v2 = = = rg r||g|| gL gL On définit alors le nombre de Froude Fr selon : dim

Fr =

v2 gL

b) Similitude Pour des raisons de coût et de difficulté dans la mise en œuvre, l’analyse expérimentale est souvent conduite sur des modèles réduits réels représentant à petite échelle des situations concrètes. Cette similitude est de nature dynamique comme on l’a vu avec l’introduction des nombres précédents mais aussi de nature géométrique et cinématique. i) Similitude géométrique Si entre le modèle réduit (m) et le prototype (p), les longueurs sont dans le rapport l, les surfaces doivent l’être dans le rapport l2 et les volumes dans le rapport l 3 : Lm =l Lp

d’où

Sm = Sp

Lm Lp

2

=l

2

m p

=

Lm Lp

3

= l3

ii) Similitude cinématique À partir des équations aux dimensions des grandeurs cinématiques et cinétiques, on peut trouver les rapports des vitesses et celui des accélérations entre les mouvements analogues dans le modèle et dans le prototype. En fonction du rapport l des longueurs et du rapport t des durées, on a : Lm T p vm l = = vp t Lp Tm

et

am Lm = ap Lp

Tp Tm

2

=

l t2

I . 4 . — Influence de la force de Coriolis terrestre Nous savons que le référentiel terrestre est un bon référentiel galiléen pourvu que l’on substitue la pesanteur à la gravitation et que les vitesses des particules soient suffisamment faibles (cf. chapitre 7).

529

Équation d’Euler et relation de Bernoulli

Si cette dernière condition n’est pas réalisée, il importe de prendre en compte la force de Coriolis dans le bilan des forces qui s’exercent sur un élément de matière. Il en résulte que l’équation d’Euler s’écrit rigoureusement : ∂v + r(v · ∇)v = − grad p + r g − 2rVT × v r ∂t Le terme de Coriolis terrestre −2rV × v est en général négligeable devant le terme advectif r(v · ∇)v. Cependant, dans le cas des masses d’air atmosphérique, où la vitesse est typiquement v ∼ 20 m . s−1 et l’échelle synoptique des distances R ∼ 106 m, il est instructif de comparer ces deux termes : 2VT v ∼ 2 × 7, 3 × 10−5 × 20 ≈ 3 × 10−3 m . s−2 alors que

(v . ∇)v ∼

v2 ≈ 4 × 10 −4 m . s−2 R

Si on néglige le terme advectif devant celui de Coriolis, l’équation précédente donne, en régime stationnaire et dans un plan horizontal : 0 = − grad p − 2rVv × v

soit

2rv × V v = grad p

Vv étant la contribution verticale de V T . Cette équation caractérise un écoulement géostrophique, c’est-à-dire qui tourne à l’échelle terrestre. On en déduit le sens de l’enroulement en spirale des masses d’air autour d’une dépression ; on retrouve bien le sens des aiguilles d’une montre de la spirale dans la photographie de la figure 7.11b, prise dans l’hémisphère nord. Pour un anticyclone, c’est l’inverse. Si on change d’hémisphère, c’est l’inverse. Remarque : Pour une échelle des distances beaucoup plus petite (R ∼ 1 m), le terme de Coriolis est négligeable : c’est ainsi que sa mise en évidence en vidangeant l’eau d’une baignoire, exigerait le repos initial du liquide, avec une grande précision : v 2 /R  2VTv

soit

v  2VT R ∼ 0, 15 mm . s−1

 c Dunod – Toute reproduction non autorisée est un délit

I . 5 . — Application de l’équation d’Euler : ondes superficielles de gravité L’équation d’Euler permet d’expliquer le mouvement d’oscillation de la surface libre horizontale d’un liquide en écoulement incompressible tel que l’eau, dans un récipient de hauteur H, si on perturbe sa surface, par exemple en jetant une pierre. Pour simplifier l’analyse, négligeons les termes non linéaires de l’équation d’Euler et considérons le mouvement dans un plan vertical Oxy d’un référentiel terrestre, Ox étant un axe horizontal et Oy la verticale ascendante (Fig. 30.3). y

g

H x F IG . 30.3.

En projetant suivant ces deux axes, on obtient les équations scalaires suivantes : 1 ∂p ∂vx =− r ∂x ∂t

1 ∂p ∂vy −g =− r ∂y ∂t

530

30. Équation d’Euler et relation de Bernoulli

À ces deux équations comportant les trois inconnues vx , v y , p , il faut ajouter deux autres équations, d’abord le bilan de la masse de ce fluide incompressible : ∂v ∂vy div v = x + =0 ∂x ∂y ensuite l’équation définissant la surface du liquide en pression à l’altitude : p = −rg(H + h) + pa

h étant la hauteur du liquide comptée à partir de la hauteur moyenne H , pa la pression atmosphérique à l’altitude nulle. La première de ces quatre équations devient : ∂h ∂vx ∂2v x ∂2 h ce qui donne = −g = −g 2 ∂t ∂x ∂t ∂x ∂x en dérivant par rapport à x. Or, on peut admettre que les déplacements verticaux h sont faibles par rapport à H ( h  H ), et ne dépendent pas de y. Il en résulte, en tenant compte de div v = 0 : ∂h ≈− ∂t soit, en dérivant par rapport au temps : vy =

H

0

∂vx ∂v x d y = −H ∂x ∂x

∂ 2h ∂ 2v x = −H ∂t 2 ∂x ∂t Finalement, nous obtenons l’équation caractéristique d’un phénomène ondulatoire : 1 ∂2h ∂ 2h = ∂x 2 v2 ∂t 2

avec v = (gH) 1/2

La solution d’une telle équation est de la forme (cf. Optique ou Électromagnétisme) : x x h(x, t) = h + t − + h− t + v v Ces ondes à la surface du liquide sont appelées ondes superficielles de gravité. Remarquons que, s’agissant d’un phénomène de surface, la vitesse de propagation est indépendante de la masse volumique. Exemple : Dans un canal à section rectangulaire, de profondeur H = 3 m, la vitesse de propagation vaut : v = (9, 81 × 3)1/2 = 5, 4 m . s −1. L’expérience permet de vérifier cette formule, tant que h est petit devant H comme nous l’avons supposé. si ce n’est pas le cas, l’analyse précédente ne convient plus en raison de l’influence non négligeable des phénomènes non linéaires ; on peut observer alors la propagation de solitons (cf. chapitre 32).

II . — RELATION DE BERNOULLI La relation de Bernoulli, du nom du mathématicien et physicien suisse D. Bernoulli qui l’établit en 1738, est l’équation énergétique associée à l’équation d’Euler ; elle est plus commode à utiliser dans la plupart des problèmes concrets qui se posent. II . 1 . — Équation de l’énergie Récrivons l’équation d’Euler en utilisant l’égalité suivante : (v · ∇)v =

1 grad v 2 + rot v × v 2

531

Équation d’Euler et relation de Bernoulli Il vient : r

v2 ∂v + r grad 2 ∂t

+ r rot v × v = − grad p + rg

Multiplions les deux membres de cette équation d’Euler par le déplacement élémentaire d r = v d t. Il vient : v2 ∂v · d r + r grad · d r = − grad p · d r + rg · d r r 2 ∂t que l’écoulement soit irrotationnel ou non, puisque (rot v × v) · v d t = 0. On en déduit, en divisant par r : dp v2 ∂v =− · dr + d + d(g · r) 2 r ∂t soit : ∂v · dr + d ∂t

v2 −g·r 2

=−

dp r

Notons que v 2/2 est l’énergie cinétique massique, alors que −g · r représente l’énergie potentielle massique de pesanteur, à une constante additive près. Si z désigne la coordonnée de l’élément de fluide selon la verticale ascendante, rappelons que −g · r = gz . II . 2 . — Écoulement stationnaire incompressible Pour un écoulement stationnaire (∂v/∂t = 0), l’équation précédente devient : d

dp v2 =0 −g·r + r 2

Si l’écoulement est incompressible ( r = Cte) , l’intégration de l’équation stationnaire précédente est immédiate :

 c Dunod – Toute reproduction non autorisée est un délit

v2 p + g z + = Cte 2 r C’est ce résultat, relatif à un fluide, en écoulement stationnaire, parfait, incompressible, soumis aux forces de pression et aux forces de pesanteur, qui est connu habituellement sous le nom de relation de Bernoulli. Soulignons que ce résultat n’est valable que le long de la trajectoire d’une particule de fluide et donc, le régime étant stationnaire, le long d’une ligne de courant. Il ne s’étend à tout le fluide que dans le cas d’un mouvement irrotationnel (rot v = 0), car alors : (rot v × v) · d r = 0 même si

d r = v d t

L’interprétation énergétique de la relation de Bernoulli est aisée : la somme des trois termes énergétiques, l’énergie cinétique massique, l’énergie potentielle massique de pesanteur et l’énergie potentielle massique associée aux forces de pression p/r, est constante. Comme les forces de frottement ont été négligées, on retrouve la conservation de l’énergie mécanique massique, somme de ces trois énergies par unité de masse. On écrit quelquefois la relation de Bernoulli en termes d’énergies volumiques : rv2 + p + rgz = Cte 2

532

30. Équation d’Euler et relation de Bernoulli

L’énergie cinétique volumique rv 2/2, homogène à une pression, est parfois appelée la pression dynamique et la somme p+rv2 /2 la pression totale ; évidemment, rgz est l’énergie potentielle de pesanteur par unité de volume. L’écriture en termes de hauteur est, elle aussi, intéressante ; en divisant l’équation par rg, on a : p v2 = Cte +z+ rg 2g où z est la hauteur réelle, v2/(2g) la hauteur « cinétique » et p/(rg) la hauteur « piézométrique ». Remarque : Dans le cas des gaz en écoulement incompressible, pour lesquels r est faible, on néglige généralement z devant les autres termes. II . 3 . — Écoulement barotrope d’un gaz On dit que l’écoulement d’un gaz est barotrope si la masse volumique n’est fonction que de la pression : r(p). Si la quantité dp/r(p) peut se mettre sous la forme de la différentielle d’une certaine fonction f (p) , la relation de Bernoulli donne, en régime stationnaire : v2 + g z + f (p) = Cte 2 Précisons à nouveau que ce résultat n’est valable que le long de la trajectoire d’une particule de fluide et donc le long d’une ligne de courant puisque le régime est stationnaire. Il ne s’étend à tout le fluide que si le mouvement est irrotationnel. Exemples i) Gaz parfait en évolution isotherme. Pour calculer f (p), utilisons l’équation d’état du gaz parfait p = nRT/V = mrT/V = rrT. Il vient : dp dp = rT = d(rT ln p) r p

d’où

f = rT ln p + Cte

ii) Gaz parfait en évolution adiabatique réversible. Dans ce cas, on sait que, p g = Cte, g étant le rapport des capacités thermiques à pression et à volume constants (cf. Thermodynamique). On a donc p/rg = K , K étant une constante, et : dp K 1/g (1−1/g) = K1/g p−1/g d p = d p 1 − 1/g r

d’où

f=

K 1/g (1−1/g) + Cte p 1 − 1/g

II . 4 . — Influence d’un travail supplémentaire extérieur Des forces extérieures occasionnelles, qui s’exercent sur un fluide non visqueux, en écoulement stationnaire et incompressible, fournissent un travail dont il faut tenir compte. La relation de Bernoulli se généralise alors selon : p v2 + gz + 2 r

B

= w AB A

A et B étant des points du référentiel R appartenant à une même ligne de courant et w AB le travail supplémentaire reçu par l’unité de masse de fluide entre A et B, en dehors des forces de pression. Notons que ce travail peut être positif ou négatif suivant la nature de ces forces. Par exemple, si le fluide fournit du travail au milieu extérieur en faisant tourner un moteur, w AB < 0. Dans le cas d’une pompe hydraulique, c’est le contraire, wAB > 0.

533

Équation d’Euler et relation de Bernoulli

Exemple : Une pompe hydraulique, installée sur une conduite, sert à aspirer l’eau au repos d’un bac et à la refouler à une hauteur de 8 m avec une vitesse de 6 m . s−1 (Fig. 30.4). Le débit-volume de la pompe est qv = 20 L . s−1. Comme la pression aux points A et B est la pression atmosphérique, on a, par unité de masse de fluide : w AB

1 2 p = v + gz + 2 r

B

= A

1 2 v + g(z B − z A) ≈ 98 J . kg −1 2 B

On en déduit la puissance de la pompe : P=

dm w AB = q mw AB = r q vw AB ≈ 2 kW dt Pompe

g

B 8m

A Bac F IG . 30.4.

Remarque : Lorsqu’un tel système échange avec le milieu extérieur de l’énergie à la fois par travail et par chaleur, l’expression précédente, valable pour un écoulement stationnaire et incompressible, se généralise en thermodynamique selon : v2 p + gz + + u 2 r

B

= w AB + qAB A

où u est l’énergie interne massique et w AB le travail reçu par unité de masse et q AB la chaleur reçue par unité de masse (cf. Thermodynamique). Cette généralisation n’est qu’une manifestation du premier principe de la thermodynamique. On voit alors apparaître naturellement l’enthalpie massique h = u + p/r.

 c Dunod – Toute reproduction non autorisée est un délit

III . — APPLICATIONS DE LA RELATION DE BERNOULLI III . 1 . — Effet Venturi On appelle effet Venturi (du nom du physicien italien du XIX e siècle G. Venturi) la diminution de la pression observée lorsque l’écoulement incompressible d’un fluide subit un étranglement. Un exemple bien connu est celui de la trompe à eau (Fig. 30.5a). La relation de Bernoulli et l’équation-bilan de la masse s’écrivent, respectivement, pour les points A et B : pA +

rv2A rv2 + rgzA = pB + B + rgz B 2 2

et v AsA = v BsB

Par conséquent : pA − pB =

r 2 v 2 A

s 2A − 1 − rg(zA − zB ) s 2B

Comme zA − z B ≈ 0 et sA/sB > 1, p B est inférieure à p A.

534

30. Équation d’Euler et relation de Bernoulli A

z

Capteurs de pression

zA B zB Trompe a` eau

A Conduite

a)

B Tube de Venturi b)

F IG . 30.5.

Par exemple, si s A = 1 cm2 , sB = 0, 20 cm 2, v A = 2 m . s −1 et zA − z B = 2 cm, on en déduit que : pA − pB ≈ 48 kPa. La formule de l’effet Venturi peut s’écrire aussi :

v 2A = 2

pA + rgz A − pB − rgzB r s2A /s2B − 1

d’où

qm = rsAv A = (2r) 1/2 s A

pA + rgzA − p B − rgz B s2A/s 2B − 1

1/2

Cet effet permet donc de mesurer le débit-masse de fluide connaissant la différence de pression (p A −pB) et la différence de hauteur (zA − zB ). C’est ce que l’on réalise dans un tube de Venturi que l’on insère généralement en position horizontale (zA = zB) à la place d’un tronçon de conduite (Fig. 30.5b). III . 2 . — Tube de Pitot Le tube de Pitot est une invention de l’ingénieur français H. Pitot qui date de 1772. C’est un double tube très fin que l’on place parallèlement aux lignes de courant d’un fluide en écoulement stationnaire incompressible (Fig. 30.6). Le tube intérieur, percé d’un petit trou A à l’avant, est complètement indépendant du tube extérieur qui lui est percé de plusieurs petits trous (généralement cinq) répartis suivant un cercle annulaire en B. Il permet de mesurer la vitesse v du fluide à partir de la différence de pression entre l’orifice en A à l’extrémité du tube, où la vitesse est négligeable (point d’arrêt), et l’orifice en B placé latéralement où la vitesse du fluide n’est pas modifiée, car le tube suffisamment fin ne perturbe pas l’écoulement. D’après la relation de Bernoulli, appliquée à une même ligne de courant pratiquement horizontale passant par A et B, on a, r désignant la masse volumique du fluide : 0 + p A + rgzA =

rv 2 + pB + rgzB 2

2 v = (pA − pB ) r

d’où

1/2

La différence (pA − pB) est généralement mesurée à l’aide d’un manomètre différentiel à eau : re v = 2 gh r

1/2

re = 1 000 kg . m−3 étant la masse volumique de l’eau et h la dénivellation. On améliore la sensibilité de ce manomètre en l’inclinant d’un angle a par rapport à l’horizontale, ce qui revient à remplacer g par g sin a . Cet appareil est souvent utilisé avec des gaz, par exemple, pour mesurer la vitesse des avions ou pour mesurer la vitesse de l’air issu d’une turbine ; dans une telle mesure, une dénivellation de 5 cm dans un manomètre à eau correspond à la vitesse : v=

2 × 9, 80 × 1 000 × 0, 05 1, 3

1/2

= 27, 5 m . s −1

535

Équation d’Euler et relation de Bernoulli

A v

S

pA B

A

pB

h A hS h B F IG . 30.6.

F IG . 30.7.

III . 3 . — Siphon Un siphon est un tube, en U renversé, que l’on utilise pour vidanger un récipient rempli d’un liquide (Fig. 30.7). Appliquons la relation de Bernoulli à la ligne de courant ASB entrant dans le siphon. Il vient, en désignant par r la masse volumique du liquide et en négligeant la faible vitesse du liquide au point A de la surface libre : rv2 rv2 0 + p A + rgzA = S + pS + rgz S = B + pB + rgzB 2 2 où la pression pB à l’extrémité inférieure B du siphon est égale à la pression atmosphérique p a . Comme le liquide est incompressible et la section du tube uniforme, la conservation du débit-masse en S et B donne : rsvS = rsvB

d’où

vS = vB

et p S − p a = −rg(zS − z B) = −rghS

hS étant la hauteur de S comptée à partir de B. Quant à la vitesse d’écoulement v B, elle vaut, puisque p A = pa : v B = (2ghA ) 1/2 hA étant la hauteur de A comptée à partir de B. Ordre de grandeur : Dans le cas d’un siphon rempli d’eau, avec h A = 20 cm et h S = 25 cm, on trouve : pS − pa = −rgh S = −2, 45 kPa et vB = 2 m . s−1 III . 4 . — Écoulement d’un liquide par un orifice. Théorème de Torricelli

 c Dunod – Toute reproduction non autorisée est un délit

Ce théorème, établi en 1 664 par Torricelli, donne la vitesse d’écoulement d’un liquide par un orifice, sous l’action de la pesanteur. Par exemple, calculons la vitesse de l’eau à l’extrémité B d’une conduite alimentée par un barrage (Fig. 30.8a), sachant que la surface libre du liquide est suffisamment grande, comparée à la section de la conduite, pour que l’on puisse considérer que le mouvement est stationnaire. La relation de Bernoulli entre les points A et B en lesquels p A = p B = p0 (pression atmosphérique) donne : rvA2 rv2 + rgz A = B + rgzB soit v2B − v 2A = 2g(zA − z B) 2 2 Comme vA sA = vB sB en raison de la conservation du débit-volume q v , il vient, en fonction de la hauteur h = zA − z B : v2B 1 −

s2B s2A

= 2gh

d’où

vB =

2gh 1 − s2B /s2A

1/2

≈ (2gh)1/2

si

s B  sA

Pour h = 100 m (barrage en montagne), vB ≈ 44, 7 m.s−1 . Ainsi, la vitesse de la particule de fluide à la sortie est celle qu’elle aurait acquise après une chute libre de hauteur h. Notons que cette vitesse est

536

30. Équation d’Euler et relation de Bernoulli

indépendante de la masse, ce qui n’est guère surprenant en raison de l’égalité des masses grave et inerte (cf. chapitre 7).

A

h2 Eau

B2

g

g

h1

Barrage

h

B1

B a)

b) F IG . 30.8.

On peut tester ces prévisions à l’aide d’un récipient dont la paroi verticale est percée de trous à des hauteurs différentes sur une même verticale (Fig. 30.8b). Les jets émergents sont paraboliques et la distance, qui sépare dans un plan horizontal les points d’impact des jets, varie sensiblement comme h 1/2 : v B1 = v B2

h1 h2

1/2

On vérifie aussi que les jets de deux liquides, de masses volumiques différentes, sont géométriquement identiques. Les écarts que l’on constate entre prévisions et expériences doivent être attribués aux hypothèses simplificatrices : influences de l’air et de la viscosité négligées, régime d’écoulement supposé stationnaire. En outre, la section de l’écoulement n’est pas égale à celle de l’orifice ; elle est plus faible dans un rapport compris entre 0, 6 et 1 suivant que la forme de l’orifice est évasée ou non. Une manière de déterminer cette section consiste à photographier le jet. III . 5 . — Pulvérisateur Un pulvérisateur est constitué d’un flacon dans lequel plonge un tube fin vertical V atteignant le liquide à pulvériser. À l’extrémité supérieure, on crée mécaniquement, à l’aide d’une poire en caoutchouc par exemple, un courant d’air horizontal dont la vitesse est maximale à l’extrémité effilée E d’un tube horizontal H (Fig. 30.9). Il en résulte, d’après la relation de Bernoulli, écrite le long d’une ligne horizontale, une pression plus faible en E (dépression) et donc une aspiration du liquide dans V. En sortant de V, le liquide est pulvérisé en gouttelettes et dispersé par le jet d’air. III . 6 . — Équilibre stable d’une balle légère au-dessus d’un jet d’air – Effet Coanda˘ Une balle légère, par exemple de ping-pong, peut être maintenue immobile grâce à l’action d’un jet d’air vertical ascendant (Fig. 30.10a). La pression au-dessus de la balle est la pression atmosphérique p a alors que, juste en dessous de la balle, la vitesse étant nulle, la pression est plus élevée (p a + rv2 /2), ce qui permet de compenser le poids de la balle. Le plus surprenant est la stabilité de cet équilibre. Pour le montrer, supposons un léger déplacement latéral de la balle, vers la gauche par exemple (Fig. 30.10b). L’explication est fournie par le champ des vitesses dans un plan normal à la direction moyenne Oz du jet. Comme ce dernier est turbulent, la

537

Équation d’Euler et relation de Bernoulli pa Air

pa

E

H

p < pa

g

V

g

a) F IG . 30.9.

b) F IG . 30.10.

distribution des vitesses a un profil gaussien, ce qui est caractéristique d’une superposition statistique d’un phénomène aléatoire comme la turbulence (cf. chapitre 31) ; c’est bien ce que confirme l’étude expérimentale, avec un tube de Pitot par exemple ; le champ des vitesses a pour expression, en fonction de la coordonnée radiale r et de z, dont l’origine est la position de la source du jet : v(r, z) =

r2 Cte exp − z w(z) 2

ez

Si la balle se déplace légèrement vers la gauche, le fluide acquiert latéralement une vitesse plus faible sur le bord gauche de la balle que sur le bord droit, ce qui provoque, d’après l’équation de Bernoulli, une surpression sur ce dernier qui ramène la balle sur l’axe du jet. C’est l’effet Coanda˘ du nom de l’ingénieur roumain H. Coand˘a qui l’a découvert en 1930. Notons que la position stable de la balle sur l’axe vertical du jet varie elle-aussi de façon aléatoire en raison de la turbulence. Remarque : La forme du jet rappelle la structure de l’onde électromagnétique gaussienne issue d’un laser, laquelle est aussi attribuée à une superposition statistique de phénomènes aléatoires.

 c Dunod – Toute reproduction non autorisée est un délit

III . 7 . — Lévitation d’une balle légère et attraction paradoxale Interprétons la lévitation de la balle de ping-pong dans l’entonnoir (Fig.30.1). L’écoulement étant supposé incompressible, le débit-volume q v est constant ; par conséquent, la section augmentant au fur et à mesure que le fluide avance, la vitesse de ce dernier diminue ; la pression augmente donc jusqu’à la pression extérieure qui est la pression atmosphérique pa . Il en résulte que la pression dans l’entonnoir est inférieure à la pression atmosphérique : la balle est aspirée. Une autre expérience consiste à souffler, à l’aide d’un tube, un courant d’air entre deux balles de ping-pong suspendues et légèrement espacées ; curieusement les balles se rapprochent. L’équation de Bernoulli permet d’expliquer ce rapprochement : l’augmentation de la vitesse de l’air entre les balles provoque une diminution de la pression dans cette région, d’où l’attraction observée. III . 8 . — Effet Magnus On appelle effet Magnus (du nom du physicien allemand du milieu du XIXe siècle G. Magnus) l’effet dû à la portance qu’exerce un fluide sur un corps solide en rotation. a) Analyse préalable Un cylindre en rotation, autour de son axe de révolution fixe, avec une vitesse angulaire V = V e z est placé dans un fluide en écoulement incompressible, de vitesse v0 perpendiculaire à l’axe du cylindre (Fig. 30.11a). Comme les particules de fluide en contact avec le cylindre sont animées d’une vitesse

538

30. Équation d’Euler et relation de Bernoulli

orthoradiale, le potentiel des vitesses et la fonction courant qui décrivent le mieux cet écoulement sont ceux associés à la superposition d’un écoulement uniforme, d’un dipôle et d’un vortex (cf. chapitre 29). y C

v(G)

G

Ω = Ωez

M θ

v0

FM

y

D

v

x

A v(D)

N

Oz

a)

x b)

F IG . 30.11.

Sur l’axe Oy perpendiculaire à l’écoulement, le fluide est entraîné différemment par le cylindre : la vitesse du fluide est plus grande lorsque le sens de la vitesse d’entraînement est celui de la vitesse d’écoulement. Il en résulte, en appliquant la relation de Bernoulli, une différence de pression et donc une portance dirigée suivant l’axe Oy, soit perpendiculaire à la direction de l’écoulement et au vecteur rotation du cylindre. Précisément, la portance N est orientée comme −ey car la vitesse est plus grande et donc la pression plus faible en D qu’en G . L’analyse de cet écoulement faite au chapitre 29 permet d’établir l’expression de la portance. Comparons, à l’aide de la relation de Bernoulli, la pression en un point courant M du cercle horizontal C, de rayon R, et la pression au point d’arrêt A. On a, en désignant par v 0 et p0 vitesse et pression en A (cf. chapitre 29) : v 02 p 0 v2 p + = + 2 2 r r

d’où

v2 v 2 2K p p0 p0 v02 K2 = + 0 − = + (1 − 4 sin2 u) − + v0 sin u 2 2 2 2 2R r r r R

puisque, le long d’une ligne de courant (r = R) , v = v u = −2v0 sin u + K /R. Il vient : 2K p p0 v 20 = + (1 − 4 sin 2 u) − r r R 2

K − v 0 sin u 4R

La contribution des deux premiers termes du deuxième membre à la force qui s’exerce sur le cylindre est nulle, car elle correspond à un écoulement symétrique dû à la superposition d’un champ uniforme et d’un champ dipolaire (cf. chapitre 29). Reste le dernier terme qui donne, pour une hauteur l du cylindre, une certaine contribution à la force. Suivant la direction de l’écoulement, cette force s’écrit : F x = F · ex =

S

−p e x · n d S =

2p 0

2K r R

K − v0 sin u Rl cos u d u = 0 4R

Suivant une direction perpendiculaire à l’écoulement, on trouve la portance : Fy = F · e y =

S

−p ey · n d S =

2p 0

2K K r − v0 sin u Rl sin u d u = −r2pKv0l = −2r v 0 V 4R R

= pR 2l et la vitesse angulaire du cylindre qui est reliée à la en introduisant le volume du cylindre 2 constante K par V = K /R (cf. chapitre 29). On en déduit l’expression vectorielle de la portance : N = 2r

v0 × V

539

Équation d’Euler et relation de Bernoulli

Remarques : (1) Cette analyse de l’influence d’un obstacle cylindrique en rotation, placé normalement dans un écoulement initialement uniforme, pourrait paraître sans intérêt pratique majeur en raison de la forme particulière de l’obstacle. Il n’en est rien, car l’étude de l’influence d’un obstacle, tel qu’une aile d’avion sur un écoulement uniforme, peut se ramener à celle qui vient d’être faite. (2) La superposition d’un vortex pour interpréter l’effet Magnus implique une viscosité non nulle du fluide. b) Observation de l’effet Magnus Dans la vie courante, on observe l’effet Magnus sur des objets tels que des solides à symétrie sphérique ou cylindrique (balle de tennis ou mieux de ping-pong, ballons de football et de rugby) se déplaçant, dans l’air au repos, avec une certaine vitesse de rotation. En l’absence de fluide, la balle aurait une trajectoire plane contenue dans le plan P formé par la vitesse initiale du centre de masse et la pesanteur. Si elle se déplace dans le fluide avec un vecteur vitesse de rotation V = V ez et une vitesse de son centre de masse v = v ex (v > 0), le problème se ramène au modèle précédent en faisant v0 = −v (Fig. 30.11b). La trajectoire de la balle quitte alors le plan vertical P contenant v et V, en raison de la portance due à l’effet Magnus, qui s’écrit : FM = 2r v0 × V = 2r

V×v

Exemples : Au football, lors d’un tir de corner, un joueur adroit peut communiquer au ballon une vitesse v et une vitesse angulaire V convenablement orientée, de telle sorte que le ballon pénètre directement dans la cage, grâce à l’air (cf. Exercice, chapitre 18). En tennis ou en ping-pong, le lift, c’est-à-dire la rotation de la balle produite par une action montante de la raquette, induit une portance verticale descendante : l’effet Magnus abaisse la trajectoire ; en revanche, le coupé obtenu par une action descendante de la raquette induit une portance verticale ascendante.

 c Dunod – Toute reproduction non autorisée est un délit

c) Application de l’effet Magnus La portance par effet Magnus, sur un cylindre en rotation dans un écoulement d’air, a été utilisée pour la propulsion éolienne de navires : on remplace les voiles par des cylindres d’axes verticaux de quelques mètres de hauteur qui tournent autour de ces axes avec une vitesse angulaire de 3 ou 4 tours par seconde. La puissance fournie par le vent est bien plus grande que celle qui est nécessaire pour faire tourner les cylindres.

IV . — ÉQUATIONS-BILANS DANS LES SYSTÈMES OUVERTS FLUIDES La plupart des systèmes ouverts qui jouent un rôle important dans la pratique utilisent des fluides. Aussi convient-il d’abord d’établir les équations-bilans locales de la mécanique des systèmes ouverts fluides. Nous rappellerons d’abord les résultats établis au chapitre 22 dans le cas des systèmes ouverts quelconques définis comme le contenu matériel d’une surface fermée S c , appelée surface de contrôle. IV . 1 . — Bilan de quantité de mouvement a) Écriture globale Le bilan global de quantité de mouvement pour un système ouvert s’écrit (cf. chapitre 22) : d P = Sex d t + (qmv)es d t

avec

P=

rv d

540

30. Équation d’Euler et relation de Bernoulli

Dans cette équation, S ex d t représente l’échange de quantité de mouvement avec l’extérieur par l’intermédiaire de la somme des forces extérieures qui s’exercent sur le système ; le second terme représente l’échange advectif à travers Sc dû au transfert de matière. La quantité de mouvement étant une grandeur conservative, il n’y a pas de terme de création. Évidemment, si R n’est pas galiléen, il faut ajouter à Sex les forces d’inertie d’entraînement et de Coriolis. En régime stationnaire ( d P = 0 ), on obtient le théorème d’Euler relatif à la quantité de mouvement : qm(vs − ve) = S ex b) Écriture locale L’écriture locale du bilan de la quantité de mouvement s’obtient en combinant l’équation d’Euler et l’équation-bilan de la masse : ∂r ∂v + r(v · grad)v = − grad p + f v et = − div(rv) r ∂t ∂t f v étant une force volumique généralisant la force volumique de pesanteur rg. Projetons la première équation suivant l’axe Ox du référentiel R et multiplions la seconde par la composante vx de la vitesse. Il vient : r

∂p ∂v x ∂r + fv,x et vx = −vx div(rv) = −r(v · grad)v x − ∂t ∂x ∂t

En effectuant la somme de ces équations, on trouve : ∂ (rv x) ∂p = −r(v · grad)vx − v x div(rv) − + fv,x ∂t ∂x avec : div(rvvx) = v x div(rv) + r(v · grad)v x

En étendant ce dernier résultat aux deux autres composantes, on obtient les trois équations qui expriment le bilan local de la quantité de mouvement d’un système ouvert fluide : ∂p ∂ (rvx ) + f v,x = − div(rv vx ) − ∂t ∂x ∂p ∂ (rvy ) = − div(rv vy ) − + fv,y ∂t ∂y ∂p ∂ (rvz ) + fv,z = − div(rv v z ) − ∂t ∂z Les premiers membres représentent les composantes du taux de variation de la quantité de mouvement volumique. Quant aux seconds membres, ils sont constitués de trois termes d’échange : l’échange advectif lié au transfert de matière, l’échange dû aux forces de pression par l’intermédiaire de la surface qui délimite le système, l’échange attribué aux forces d’interaction à distance avec l’extérieur. En régime stationnaire, les premiers membres sont évidemment nuls. c) Réaction de jet Sur un chariot C, glissant sans frottement sur un rail, on a fixé un récipient présentant sur sa partie latérale inférieure, un orifice A , d’aire s , par lequel l’eau qu’il contient peut s’écouler (Fig. 30.12). On dépose l’ensemble sur le fond horizontal d’un bassin B rempli d’eau.

541

Équation d’Euler et relation de Bernoulli

Bassin

Chariot

S

g

Sc

Rail

A

Eau s

F IG . 30.12.

Initialement, l’eau est au même niveau dans B et dans C . Lorsqu’on ouvre l’orifice, il ne se passe rien puisque l’eau reste en équilibre, la pression étant la même en tout point de n’importe quel plan horizontal. i) Niveau de l’eau plus haut dans le chariot Pour obtenir une expulsion d’eau, il suffit d’ajouter de l’eau dans le chariot et ainsi provoquer une surpression de l’eau du chariot au point A par rapport à la pression en un point voisin dans le bassin. Appliquons, par rapport au référentiel Rc lié au chariot, en mouvement de translation accélérée par rapport à R , le long de l’axe horizontal Ox , le théorème d’Euler au contenu matériel de la surface fermée Sc comprenant la masse du chariot et celle du fluide transporté. Il vient, la masse de l’ensemble étant Mc + M t : qmu s − qmue = (Mc + Mt )g − p e s ex − paS ez + Fe/c + R − (Mc + Mt )a où F e/c est la force qu’exerce l’eau à l’extérieur du chariot sur la paroi de ce dernier et R la réaction du rail sur le châssis. Il vient, en projetant le long de Ox, puisque us,x = u et ue,x = 0 : qmu = −pe s + Fe/c,x + Rx − (M c + Mt )a x

d’où

(Mc + M t )ax = −pe s + F e/c,x + Rx − rsu2

Ainsi, tout se passe comme si le système {chariot-fluide transporté} était soumis à une force supplémentaire, opposée au sens de la vitesse d’expulsion du fluide, qu’on appelle souvent réaction de jet :

 c Dunod – Toute reproduction non autorisée est un délit

R j = −rsu2e x Si, à l’instant initial, les surfaces d’eau sont au même niveau à l’intérieur et à l’extérieur du chariot, de l’eau du chariot n’est pas expulsée, d’où une réaction de jet nulle ; il en est de même de la somme des forces de pression. Le système garde donc son état de repos dans le fluide. En ajoutant de l’eau, la réaction de jet provoque un mouvement opposé au sens d’écoulement de l’eau à travers l’orifice. En l’absence de frottement ( Rx = 0 ) et d’accélération initiale, la force occasionnelle nécessaire pour maintenir immobile le chariot est l’opposée de la réaction de jet : Foc = −Rj = rsu2 ex Expérimentalement, on fixe le chariot sur une plateforme pouvant glisser le long d’un rail, avec un facteur de frottement faible, aux extrémités duquel on a placé deux butées (Fig. 30.12). ii) Niveau de l’eau plus bas dans le chariot En retirant de l’eau du chariot, le niveau de l’eau y est plus bas que dans le bassin. On analyse le mouvement comme précédemment en appliquant le théorème d’Euler au contenu matériel de la surface fermée Sc , ce qui donne, en projetant sur l’axe horizontal : qmux = −pe s + F e/c,x + R x − (M c + Mt)ax

Cependant, dans ce cas, la pénétration se produit à l’entrée avec une vitesse négligeable de l’eau, d’où une réaction de jet nulle.

542

30. Équation d’Euler et relation de Bernoulli

Cette dissymétrie entre les cas i) et ii) est connue sous le nom de paradoxe de Bergeron, ingénieur français du XIX e siècle. IV . 2 . — Bilan d’énergie cinétique a) Écriture globale Le théorème de l’énergie cinétique, relatif aux systèmes ouverts, se présente sous une forme analogue au théorème de la quantité de mouvement, mais avec une contribution des forces intérieures (cf. chapitre 22) : d Ek = P ex + dt

1 qm v 2 2

e s

soit aussi

+ P in

d Ek = dWex +

1 qm v2 2

e s

d t + dW in

Distinguons, dans les travaux des forces extérieures, les contributions des forces de pression, des forces dérivant d’une énergie potentielle et des forces occasionnelles. En outre, séparons les termes relatifs au système ouvert, délimité par la surface de contrôle Sc , de ceux associés aux masses dme et dms qui entre et sort respectivement. Les travaux des forces qui s’exercent sur ces masses s’écrivent, en introduisant l’énergie potentielle massique ep,ex des forces extérieures et le travail des forces de s s pression (cf. Thermodynamique) : − dm e p,ex + dm p/r . e e On a donc :

s p s (nc) − d Ep,ex − dm ep,ex + dWoc e r e De même, dans les travaux des forces intérieures, distinguons les forces qui dérivent d’une énergie potentielle intérieure des forces intérieures non conservatives. On a :

dW ex = dWp − dm

dWin = − d E p,in − dm ep,in

s e

+ dWin(nc)

Il en résulte, en injectant ces travaux dans le bilan d’énergie cinétique : d(Ek + Ep,ex + E p,in) = d t q m

p v2 + ep,ex + ep,in + 2 r

e s

(nc) + dWp + dWoc + dW(innc)

En régime stationnaire, le premier terme est nul et q m,e = q m,s , d’où : qm d t

p v2 + e p,ex + e p,in + 2 r

s e

= dW p + dWoc + dWin(nc)

On retrouve la relation de Bernoulli dans ses conditions de validité qui sont, rappelons-les : i) pas de travail des forces intérieures conservatives (e p,in = Cte) , car l’écoulement du fluide est incompressible, (nc)

ii) pas de travail de forces occasionnelles (dW oc = 0) , car il n’y a ni travail fourni, ni travail des forces de frottement avec le milieu extérieur, ni travail des forces intérieures non conservatives, iii) pas de travail des forces de pression sur la surface de contrôle, en dehors de celui qui s’exerce sur les masses en transit (dWp = 0). Comme e p,ex = gz, il vient : p v2 + gz + 2 r

s

=0 e

soit

p v2 + gz + = Cte 2 r

La généralisation de la relation de Bernoulli constitue précisément le premier principe de la thermodynamique. On doit ajouter à l’équation précédente l’énergie interne massique u, qui prend en compte

543

Équation d’Euler et relation de Bernoulli

notamment l’énergie potentielle des forces intérieures ainsi que l’agitation des particules, c’est-à-dire la température (cf. Thermodynamique). Notons qu’une grandeur énergétique s’introduit naturellement, l’enthalpie massique : h = u + p/r. b) Écriture locale Comme rv 2/2 représente l’énergie cinétique volumique, calculons l’expression de ∂ (rv 2/2)/∂t : ∂ ∂t

rv2 2

=

v2 ∂r ∂ +r ∂t 2 ∂t

v2 2

=−

v2 ∂v puisque div(rv) + rv · ∂t 2

∂r = − div(rv) ∂t

Il est aisé de faire apparaître le terme advectif associé à l’énergie cinétique, puisque : div r Il vient :

∂ ∂t

rv2 2

v2 v 2

=

v2 div(rv) + rv · grad 2

= − div r

v2 v 2

+ rv ·

v2 2

v2 ∂v + rv · grad 2 ∂t

Les deux derniers termes peuvent être regroupés sous la forme : rv ·

∂v + grad ∂t

v2 2

soit

rv · (a − rot v × v)

d’après l’expression de l’accélération. En simplifiant, on trouve : rv ·

v2 ∂v + grad 2 ∂t

= rv · a

Il en résulte, si on introduit la somme des forces volumiques fv = r a, l’écriture suivante du bilan local d’énergie cinétique :

 c Dunod – Toute reproduction non autorisée est un délit

∂ ∂t

rv 2 2

rv2 v = − div 2

+ fv · v

L’interprétation est aisée : la variation de l’énergie cinétique volumique est égale à la puissance des forces volumiques augmentée du terme d’advection − div (rv 2/2)v . Dans les forces volumiques, sont prises en compte les forces de pression (− grad p), les forces de pesanteur (r g), auxquelles on ajoute d’autres forces telles que les forces d’inertie d’entraînement, si le référentiel n’est pas galiléen, les forces de viscosité et toute force occasionnelle. En régime stationnaire, le premier membre est nul. IV . 3 . — Applications Dans la plupart des cas, la pression atmosphérique uniforme p a s’exerce sur toute la surface de contrôle Sc (Fig. 30.13) ; montrons que sa contribution est nulle : F= Sc

pa(−n) d S =

− grad pa d

=0

puisque

grad p a = 0

Il en résulte que les seules pressions à considérer sont les pressions au-dessus de la pression atmosphérique, c’est-à-dire les pressions fournies par les manomètres.

544

30. Équation d’Euler et relation de Bernoulli n Sc F IG . 30.13.

a) Force horizontale exercée par un fluide sur un coude dans une conduite horizontale Appliquons le théorème d’Euler au système ouvert limité par la surface de contrôle S c = A1B 1B 2A2 (Fig. 30.14a). Il vient : qm (v2 − v1 ) = Mg + Fp1 + F p2 + R

où Mg est le poids du fluide, Fp 1 , Fp 2 les forces de pression manométriques, qui s’exercent sur les faces d’entrée et de sortie, et R la force exercée par le coude sur le fluide. En projetant dans un plan horizontal, on obtient : qm(v2 − v 1) = F p1 + F p2 + Rh L’opposé de Rh , c’est-à-dire la force horizontale exercée par le fluide sur le coude, est donc : −Rh = Fp1 + F p2 − q m(v2 − v1)

B1

B2

v2 µ

v1 A1

v2 − v 1

v2

A2

µ v1

Sc

a)

b) F IG . 30.14.

Comme v 1 = v2 = v , il vient, d’après la relation de Bernoulli, p1 = p 2 = p. On en déduit : Fp1 = Fp2 = ps, si s désigne l’aire de la section droite de la conduite, et on peut écrire, e étant le vecteur unitaire porté par le vecteur v2 − v1 orienté vers l’intérieur du coude (Fig. 30.14b) : v2 − v 1 = 2v sin

u 2

e

et Fp1 + F p2 = ps

v1 v 2 u ps − = − (v2 − v1) = −2ps sin 2 v1 v 2 v

On trouve donc : −Rh = −2(q m v + ps) sin

u 2

e

e

On voit que le fluide exerce sur le coude une force −R h orientée vers l’extérieur du coude.

Exemple : si la pression manométrique exercée par le fluide, par exemple de l’eau, est 3 bar , le diamètre de la conduite 10 cm , la vitesse v de l’eau 10 m.s−1 , q m = 320 kg.s −1 et u = p/3 , on trouve : ps = 3 × 10 5 × p(0, 05) 2 ≈ 2 356 N

q mv = 3 200 N d’où

Rh ≈ 5 556 N

545

Équation d’Euler et relation de Bernoulli b) Action d’un jet d’eau sur une plaque plane verticale

Considérons un jet d’eau horizontal, de section s, tombant avec une vitesse v sur une plaque plane verticale maintenue immobile (Fig. 30.15). Le jet se sépare en deux jets tangents à la plaque. Exprimons la force qu’exerce le jet sur la plaque, en régime stationnaire, si la plaque fait un angle u avec la direction du jet incident. Pour cela, appliquons le théorème d’Euler au fluide contenu dans la surface de contrôle S c qui entoure le jet incident ainsi que ses parties émergentes. Il vient, selon un plan horizontal, en désignant par q m = rsv le débit-masse à l’entrée, par qm,1 , qm,2 les débits-masses aux sorties et par R la force qu’exerce le jet sur la plaque : qm,1 vs,1 + qm,2 vs,2 − q m v = −R

L’écoulement du fluide étant incompressible, le débit-volume se conserve : qv = qv,1 + qv,2 soit

sv = s 1vs,1 + s 2vs,2

si s1 et s2 désignent les sections à l’émergence du jet de la surface de contrôle. En raison de l’absence de force de frottement, la relation de Bernoulli, écrite le long d’une ligne de courant horizontale, tangente à la vitesse v du jet avant l’impact, puis tangente à la plaque après, avec une vitesse vs, donne : pa +

rv 2 rv2 = pa + s 2 2

d’où

A1 s2

Sc µ

s

Lignes de courant

y Ae

As

R

A2 s1

x F IG . 30.15.

 c Dunod – Toute reproduction non autorisée est un délit

vs = v

Sc F IG . 30.16.

Projetons l’équation d’Euler selon les directions x et y, respectivement normale et tangente à la plaque. On obtient : −q mv sin u = −R et r q v,1v − r qv,2v − r q vv cos u = 0 d’où : R = rsv2 sin u et q v,1 − q v,2 = qv cos u Il en résulte, en tenant compte de la conservation du débit-volume : qv qv qv,2 = (1 − cos u) qv,1 = (1 + cos u) 2 2 On vérifie bien que cette force est maximale si la plaque est normale au jet (u = p/2).

Ordre de grandeur : Pour s = 12, 5 cm 2 , v = 35 m.s−1 et u = p/2 , la force qu’exerce la plaque sur le jet vaut R = rsv 2 ≈ 1 531 N , soit l’équivalent du poids d’une plaque de masse égale à 156 kg . c) Propulsion d’un avion à hélice Le rôle de l’hélice d’un avion est d’aspirer de l’air et de l’éjecter avec une vitesse v s par rapport à l’avion R . Si l’avion se déplace à vitesse constante horizontale v, par rapport au référentiel terrestre R, l’air, au repos dans R, a une vitesse par rapport à l’avion égale à ve = −v . La surface de contrôle Sc, immobile dans R , est une surface cylindrique dont les surfaces de base A e et A s sont normales

546

30. Équation d’Euler et relation de Bernoulli

à v (Fig. 30.16). La force horizontale Fh qu’exerce l’avion sur le fluide contenu dans S c est telle que : qm(vs + v) = F h

qm étant le débit-masse d’air aspiré par l’hélice. Les forces de pression atmosphérique n’ont aucune contribution puisque la pression atmosphérique s’exerce uniformément sur toute la surface. On en déduit la force Fp = −Fh qu’exerce le fluide sur l’avion, c’est-à-dire la force propulsive : Fp = −qm(v s + v)

CONCLUSION Rappelons les principaux résultats. (1) L’équation d’Euler donne la relation entre le champ des vitesses et le champ des pressions dans un fluide non visqueux : v2 ∂v + grad ∂t 2

1 + rot v × v = − grad p + g r

(2) La relation de Bernoulli :

p v2 − g · r + = Cte 2 r est la traduction énergétique de l’équation d’Euler, si l’écoulement du fluide non visqueux est stationnaire et incompressible. Pour un écoulement rotationnel, la constante de l’équation n’est relative qu’à une ligne de courant ; s’il est irrotationnel, elle concerne tout le fluide. (3) Les applications de la relation de Bernoulli ou de sa généralisation, lorsque le fluide fournit ou reçoit un travail supplémentaire, sont nombreuses (tube de Pitot, tube de Venturi, pompe, etc.). (4) L’effet Magnus permet d’expliquer l’influence de l’air sur des objets en rotation, précisément les mouvements spectaculaires des balles de tennis ou des ballons de foot-ball. (5) Enfin, la dynamique des fluides non visqueux jouent un rôle majeur dans le fonctionnement des systèmes ouverts.

EXERCICES ET PROBLÈMES P30– 1. Ondes de gravité On produit à une extrémité d’un bassin, de profondeur H = 25 cm et de longueur L = 40 m, une onde de surface, de période T = 4 s , se propageant le long de cette grande dimension. Calculer sa longueur d’onde. Quelle est la durée nécessaire à l’onde pour atteindre l’autre extrémité et revenir à l’endroit où la perturbation a été créée ? P30– 2. Oscillations d’un liquide dans un tube en U 1. Quelle force faut-il exercer sur la surface du liquide contenu dans l’une des branches d’un tube en U pour que la dénivellation entre les deux branches soit h ? Application numérique : la masse volumique est r = 1 000 kg . m −3, la section du tube s = 4 cm2 et h = 5 cm. 2. On abandonne le système à lui-même à partir de cette dénivellation. Montrer que la surface libre du liquide oscille avec une période que l’on exprimera en fonction de la longueur l du tube rempli de liquide. Application numérique pour l = 20 cm.

547

Équation d’Euler et relation de Bernoulli

P30– 3. Jet d’eau de Genève Le débit-masse à la base du jet d’eau de Genève est q m = 500 kg . s −1 . Le diamètre de la section du tuyau, à la base, est d = 11 cm et le rendement énergétique vaut r = 0, 75 en raison du frottement de l’air. Trouver la hauteur du jet et calculer la puissance mécanique nécessaire pour l’alimenter. P30– 4. Durée de vidange d’un réservoir Un réservoir parallélépipédique, de section S, rempli d’eau jusqu’à une hauteur H, est vidangé par un orifice, de section s, situé au fond. En supposant le régime stationnaire et l’écoulement incompressible, trouver la durée pour le vidanger à moitié de son volume. Application numérique : S/s = 12 et H = 25 cm. P30– 5. Puissance fournie par une turbine hydraulique avec diffuseur Une conduite, à section droite circulaire, amène l’eau d’un barrage dans une turbine ; à la sortie, l’eau est évacuée dans un canal. On désigne par hb le niveau de la surface libre de l’eau du barrage, compté à partir de celui de la surface libre de l’eau dans le canal ; de même, h s est le niveau de l’eau à la sortie S de la turbine à partir de la même origine. 1. Trouver, en fonction de la vitesse v s de l’eau, en sortie S, h b et h s, la puissance fournie par la turbine par unité de volume d’eau. Application : hb = 70 m, hs = 5 m, débit-volume q v = 6 m3 . s−1 , diamètre de la conduite à la sortie de la turbine 1 m. 2. On adapte à la sortie de la turbine un diffuseur, de forme suffisamment évasée, qui rejette l’eau dans le canal avec une vitesse négligeable sur son axe en D situé à une profondeur de 2 m (Fig. 30.17). Quelle est la puissance fournie par la turbine dans ce cas ? En déduire l’intérêt du diffuseur. z hs

E Turbine S

O Canal D Diffuseur

 c Dunod – Toute reproduction non autorisée est un délit

F IG . 30.17.

P30– 6. Puissance fournie par une pompe de distribution d’essence Une pompe de distribution d’essence aspire le carburant contenu dans une cuve enterrée et le refoule, à débit-volume constant qv = 2 L . s−1 , sous la forme d’un jet de diamètre D = 3 cm. Audessus du liquide dans la cuve, la pression est la pression atmosphérique. La masse volumique de l’essence est 750 kg . m−3 et la différence de hauteur, entre le jet et le niveau de l’essence dans la cuve, est h = 3, 2 m. On prend en compte la qualité de l’installation, du fait de la perte de charge et les performances de la pompe, en introduisant le rendement r = 0, 7. Quelle est la puissance de la pompe ? P30– 7. Écoulement d’eau dans un canal de section variable De l’eau circule dans un canal de section rectangulaire ; sa vitesse est v 1 = 1, 4 m . s −1 lorsque sa largeur est l1 = 1, 1 m et sa profondeur h 1 = 0, 9 m. Quelle doit être la largeur du canal l 2 à un endroit où sa profondeur est h2 = 0, 7 m ? En déduire la vitesse v2.

548

30. Équation d’Euler et relation de Bernoulli

P30– 8. Lance à eau Une lance à eau se termine par un embout, de section 10 cm 2, adapté sur une conduite souple de section 50 cm2. En régime stationnaire, le débit-volume de la lance est de 50 L . s −1 . Quelle force horizontale doit-on exercer pour maintenir la lance immobile ? P30– 9. Déplacement d’un chariot à l’aide d’un jet Un chariot de masse M c est mis en mouvement horizontal par un jet d’eau stationnaire comme le montre la figure 30.18. Le jet d’eau, de vitesse horizontale Vex par rapport au référentiel terrestre, suit dans le chariot une trajectoire circulaire de rayon R. Il quitte le chariot avec une vitesse relative égale en norme v à la vitesse relative à l’entrée, mais inclinée de l’angle u par rapport à l’horizontale. Exprimer l’accélération a du chariot en fonction de sa vitesse v, u, V , Mc, r et de la section s du jet ; on appliquera le théorème d’Euler à la surface S c entourant l’ensemble fluide et chariot et on négligera l’influence de la force de frottement exercée par le sol. 60°

θ g

Sc

Sc x

F IG . 30.18.

As

Vex Ae

v x

F IG . 30.19.

P30– 10. Jet d’eau sur une aube Un jet d’eau stationnaire, de section s = 6 cm 2, est envoyé horizontalement sur une aube hydraulique à la vitesse de V = 20 m . s −1 (Fig. 30.19). Il émerge horizontalement avec une vitesse par rapport à l’aube qui fait un angle u = 60◦ avec la direction incidente. L’aube a une vitesse de translation v = v e x avec v = 10 m . s −1. Calculer la force F que l’on doit exercer sur elle pour que la vitesse v demeure constante. P30– 11. Réaction de jet Un liquide s’échappe par un orifice, percé sur la surface latérale d’un réservoir, avec une vitesse horizontale u = 10 m . s−1 et un débit-masse qm = 1 kg . s−1 . Quelle force faut-il exercer sur le réservoir pour le maintenir en équilibre sur un plan horizontal ? P30– 12. Force nécessaire pour maintenir immobile une plaque soumise à l’action d’un jet d’eau Une plaque plane, qui peut se déplacer sans frottement sur un plan horizontal, est soumise à l’action un jet d’eau horizontal de débit-volume qv = 30 L . s−1 et de section s = 0, 01 m 2. Quelle force faut-il exercer pour maintenir cette plaque immobile ? P30– 13. Conduite en forme de coude de section variable Une conduite d’eau horizontale, en forme de coude de section variable, change la direction d’écoulement du fluide d’un angle de 60 ◦. Sa section est de 200 cm2 à l’entrée et de 100 cm2 à la sortie. La vitesse du fluide à l’entrée est de 10 m . s −1 alors que la pression manométrique est de 5 bar. Calculer la force horizontale qu’exerce le fluide sur la conduite en régime stationnaire. P30– 14. Équilibre d’une plaque soumise à l’action d’un jet d’eau On maintient en équilibre, grâce à un jet d’eau horizontal et stationnaire, une plaque, carrée, homogène (masse m, côté b), qui peut tourner autour d’un axe horizontal. Ce jet, de section s et de vitesse v, est dirigé sur la plaque (Fig. 30.20). La distance du point d’impact I à l’axe est d.

549

Équation d’Euler et relation de Bernoulli

En utilisant le théorème d’Euler, relatif au moment cinétique, au système ouvert délimité par une surface de contrôle entourant la plaque, trouver l’angle a que fait la plaque avec la verticale à l’équilibre. Application numérique : b = 0, 9 m, m = 250 kg, s = 10 cm 2, v = 30 m . s −1, d = 0, 7 m. v1 g

O

α

D

d v

α I F IG . 30.20.

v2 F IG . 30.21.

P30– 15. Action d’un jet sur un auget d’une turbine Pelton Une turbine Pelton fonctionne sous une chute d’eau de 1 200 m. L’auget a la forme symétrique représentée sur la figure 30.21 : la direction du jet incident, de diamètre D = 20 cm coïncide avec l’axe de symétrie et les jets émergents font un angle a = 120◦ avec cet axe. On désigne par u = 30 m . s −1 la vitesse de translation de l’auget. Trouver l’expression de la force qu’exerce le jet sur l’auget. P30– 16. Anémomètre à laser Pour déterminer la vitesse d’écoulement v d’un fluide, on utilise des particules légères, entraînées par le fluide et capables de diffuser la lumière qu’elles reçoivent occasionnellement. Cette lumière est celle produite par l’interférence de deux ondes monochromatiques, de longueur d’onde l, dont les vecteurs d’ondes font entre eux un angle a.

 c Dunod – Toute reproduction non autorisée est un délit

Sachant que l’interfrange est i = l/(2 sin a/2), quelle est la fréquence f D du signal lumineux diffusé par une particule qui se déplace perpendiculairement à la direction des franges ? En déduire v dans le cas où l = 632, 8 nm, a = 10 ◦ et fD = 33 kHz.

31 Fluides visqueux Jusqu’à présent, nous avons négligé la contribution de la viscosité dans les forces qu’exercent, l’une sur l’autre, les parties d’un fluide par l’intermédiaire de la surface qui les sépare. Cette hypothèse s’avère satisfaisante pour tous les fluides au repos et pour certains d’entre eux faiblement visqueux, comme l’eau. En revanche, pour d’autres, comme l’huile, la prise en compte de la viscosité est indispensable pour décrire leur comportement. C’est même le cas pour des fluides peu visqueux dans certaines circonstances, par exemple l’eau dans un récipient en rotation uniforme autour d’un axe vertical (Fig. 28.4c) ; la forme parabolique de la surface de l’eau ne peut être comprise que si la paroi du récipient peut entraîner le liquide en contact, et par conséquent si le liquide présente une certaine viscosité, même faible. Un autre exemple qualitatif est fourni par le montage de la figure 31.1 : un disque D horizontal est suspendu en son centre par un fil de torsion ; un second disque Dm , identique au premier, pouvant être mis en rotation par un moteur électrique, est placé sous le premier à une distance de quelques millimètres. On constate, en faisant tourner Dm que D est entraîné, ce que l’on interprète par la viscosité de l’air. Fil de torsion D g

Dm

Eau

Moteur électrique

g

a)

Glycérine

b)

F IG . 31.1.

F IG . 31.2.

Enfin un dernier exemple introductif, pouvant déboucher sur une mesure, consiste à constater la différence de durée de chute d’une petite bille d’acier dans deux récipients, l’un rempli d’eau l’autre de glycérine (Fig. 31.2). Si l’on tient compte du poids mg et de la poussée d’Archimède, d’expression −(mrf /r)g, on ne peut pas rendre compte de la différence des durées de chute, alors que les rapports des masses volumiques du fluide (rf ) et de l’acier (r) sont du même ordre de grandeur : rf r

= e

1 000 ≈ 0, 13 et 7 800

respectivement dans l’eau et dans la glycérine.

rf r

= g

1 260 ≈ 0, 16 7 800

551

Fluides visqueux

Nous nous proposons dans ce chapitre de définir de façon simple la viscosité et d’étudier son influence en dynamique des fluides, à la fois sur les plans théorique et pratique.

I . — VISCOSITÉ I . 1 . — Définition Le vecteur-contrainte des forces qu’exerce une partie 1 d’un fluide sur une seconde partie 2, à travers la surface élémentaire de contact d S, est la somme de deux contributions, l’une normale à d S, et l’autre tangentielle : d F1→2 = f s,1→2 d S = p n1→2 d S − [s t ] n 1→2 d S La contrainte tangentielle traduit l’existence de forces de frottement qu’exerce 1 sur 2. Comme il s’agit d’une force de frottement, le sens de la force élémentaire tangentielle −[st ]n 1→2 d S qu’exerce 1 sur 2 est opposé à celui de la vitesse de glissement v2/1 de 2 sur 1 au point considéré (Fig. 31.3). Cette force tangentielle est souvent proportionnelle au taux de variation dans l’espace de cette vitesse de glissement ; la viscosité h est ce coefficient de proportionnalité : d F1→2 = −[st ] n1→2 d S = −h

d v 2/1 dS dx

x désignant la variable suivant la normale n 1→2 . Ce résultat a été établi expérimentalement pour la plupart des fluides dans un état proche de l’état d’équilibre. On qualifie alors le fluide de newtonien. Si la force de viscosité n’est pas proportionnelle au taux de variation de la vitesse de glissement n’est pas satisfaite, le fluide est dit non newtonien. x x n 1→2

 c Dunod – Toute reproduction non autorisée est un délit

−[σt ]n1→2

2

v2/1

dS F IG . 31.3.

Pm z

1

vm

v(x)



Q

L

Q Pf

O

dx

z F IG . 31.4.

L’étude expérimentale de la force de viscosité pourrait être réalisée comme le montre la figure 31.4. On considère un fluide entre deux plaques parallèles, l’une Pf fixe et l’autre Pm mobile, placée à la distance L de la première et se déplaçant parallèlement à elle-même à la vitesse v m . L’expérience montre que le fluide est en partie entraîné par la plaque mobile, en raison des forces de frottement qui apparaissent du fait de la différence des vitesses entre les particules de fluide proches de P m et les particules solides de P m qui, elles, ont la vitesse vm . Le fluide visqueux adhère aux plaques, contrairement au fluide non visqueux qui glisserait sur elles : au contact, les vitesses des particules du fluide et des particules des plaques sont égales. On constate que la force F m, par unité de surface S de P m , qui est nécessaire pour déplacer Pm à vitesse vm constante, est de la forme : Fm vm =h L S

552

31. Fluides visqueux

h étant la viscosité. Cette dernière s’exprime en pascal-seconde ( Pa . s) dans le système international. En effet, d’après la relation précédente, on a : dim h =

F L = Pa × T × S L × T−1

Le pascal-seconde est aussi appelé le poiseuille (du nom du physicien français J. L. Poiseuille). Ordre de grandeur : Sur le tableau 31.1, on a rassemblé les valeurs de h de quelques fluides. La viscosité de l’eau à 293 K ( 20 ◦ C) est 1, 1 mPa . s, alors que celle d’une huile moteur telle que SAE 30 est 0, 3 Pa . s, soit 300 fois plus. Notons que, pour un fluide au repos, la vitesse de glissement étant nulle, la contrainte tangentielle est nulle. Corps à 293 K

Air

Acide étanoïque

Eau

Huile d’olive

Huile moteur SAE

Huile de ricin

Glycérine

h (Pa . s) 1, 82 × 10−5 1, 006 × 10−3 1, 2 × 10−3 0, 1 0, 3 1, 2 1, 49 n (m 2 . s −1) 1, 51 × 10−5 1, 008 × 10−6 1, 14 × 10 −6 0, 11 × 10 −3 0, 4 × 10−3 1, 24 × 10 −3 1, 2 × 10 −3 TAB . 31.1.

I . 2 . — Transport de la quantité de mouvement L’expérience précédente peut être interprétée comme un transport, suivant l’axe de déplacement Ox, de la quantité de mouvement selon Oz dans le fluide : les particules proches de la plaque mobile ont une quantité de mouvement v(x, t) = v z(x, t) e z qui diffuse dans le fluide en respectant la contrainte d’une vitesse nulle lorsqu’elles sont en contact avec la plaque fixe P f . a) Force volumique de viscosité Considérons la tranche de fluide comprise entre x et x + d x et de surface S (Fig. 31.4). Cette tranche est soumise à l’action des forces de viscosité exercées par le fluide. Ces forces ont deux contributions, l’une due au fluide qui se trouve au-dessous du plan Q, de coordonnée x, et l’autre due au fluide placé au-dessus du plan Q de coordonnée x + d x. La force résultante s’écrit donc : −hS

∂vz ∂x

(x) + hS

∂vz (x + d x) ∂x

Il vient, en effectuant et en introduisant le volume élémentaire d hS

∂ 2v z dx ∂x2

soit

h

∂ 2 vz d ∂x2

= S dx :

Comme v(x) = vz(x) ez, la force volumique de viscosité a pour expression générale hDv , l’opérateur D désignant le laplacien (cf. annexe 4). b) Équation de diffusion Appliquons la loi fondamentale de la dynamique à la tranche de fluide comprise entre x et x + d x et de surface S . Comme l’accélération qu’elle acquiert en raison des forces de viscosité est ∂v z/∂t , il vient : r

∂vz ∂ 2vz =h 2 ∂t ∂x

553

Fluides visqueux

On reconnaît là une équation caractéristique de diffusion de la vitesse vz (cf. Thermodynamique). Le coefficient n = h/r qui apparaît naturellement est appelé la viscosité cinématique : n s’exprime en m2 .s−1 dans le système international. Sur le tableau 31.1, on a donné aussi quelques valeurs de viscosité cinématique. Notons qu’une durée caractéristique du phénomène de diffusion peut être obtenue en effectuant le rapport T = L2/n , L étant une longueur caractéristique. Ainsi, on montre que la diffusion dans un récipient rempli d’eau de diamètre L = 10 cm , est lente : 10−2 L2 = 9 000 s soit T = 2, 5 h = 1, 1 × 10−6 n ce que l’on peut mettre en évidence en saupoudrant au préalable la surface de l’eau avec du café moulu. La différence constatée entre la durée observée et la durée précédente doit être attribuée à l’advection qui accompagne la diffusion. T=

c) Interprétation microscopique Une interprétation microscopique simple de la viscosité, analogue à celle qui est donnée en diffusion de particules physiques et en diffusion thermique, consiste à exprimer le transport de quantité de mouvement par les molécules (cf. Thermodynamique). On trouve, en fonction du coefficient de diffusion de particules D : h = D m nv =

 nv m vm = r v m puisque 3 3

D=

vm 3

m étant la masse d’une particule, nv le nombre de particules par unité de volume,  leur libre parcours moyen, vm leur vitesse moyenne et r la masse volumique du milieu. (1) Cas des gaz Bien que sommaire, cette théorie fournit les paramètres dont dépend la viscosité des gaz. En effet, d’après la théorie cinétique des gaz parfaits (cf. Thermodynamique), on a, k B étant la constante de Boltzmann et T la température : 1 p0 √ n v = kBT 4pR2 2 Par conséquent, la viscosité d’un gaz parfait varie selon :  c Dunod – Toute reproduction non autorisée est un délit

nv =

h = Cte ×

vm =

3kBT pm

1/2

(mT )1/2 R2

Cette simple analyse permet de retrouver l’ordre de grandeur de la viscosité de l’air. En effet, sa masse molaire étant M = 0, 029 kg . mol−1 et le rayon d’une molécule R = 0, 1 nm, on a, à la température T = 300 K et sous la pression p0 = 1 bar : m = 48 × 10−27 kg

n v = 24 × 1024 m−3

v m = 470 m.s −1

 ≈ 0, 2 mm

d’où h ≈ 3, 6 × 10 −5 Pa.s, à comparer à la valeur expérimentale égale à la moitié de la valeur prédite par ce simple modèle. (2) Cas des liquides La théorie précédente ne convient pas du tout, car les interactions entre particules, négligeables dans les gaz suffisamment dilués, jouent un rôle majeur dans les liquides. L’effet de la température est d’affaiblir ces interactions qui sont de type van der Waals (cf. Électromagnétisme et Thermodynamique), d’où l’observation d’une forte diminution de la viscosité des liquides lorsque la température augmente,

554

31. Fluides visqueux

précisément selon une loi de la forme : h=

h0 1 + a(T − T0) + b(T − T0 ) 2

h0 étant la viscosité à la température T0 = 273, 15 K .

II . — DYNAMIQUE DES FLUIDES VISQUEUX II . 1 . — Équation de Navier-Stokes La prise en compte des forces de viscosité dans l’étude dynamique du mouvement d’un fluide donne l’équation de Navier-Stokes, des noms de l’ingénieur français H. Navier, inspiré par la théorie moléculaire, et du physicien irlandais G. Stokes, adepte lui d’un modèle de mécanique des milieux continus. En réalité, le pas décisif pour introduire la viscosité fut franchi par A. Barré de Saint-Venant, avant Stokes, dès 1843. Dans le cas général, l’analyse est inextricable. Si le fluide est newtonien, son écoulement incompressible et sa viscosité constante, la force de viscosité se simplifie et s’écrit hDv . La prise en compte de cette force de viscosité dans l’équation d’Euler permet d’obtenir l’équation suivante : r

∂v + r(v · grad)v = − grad p + rg + hDv ∂t

En divisant les deux membres de cette équation par r , il vient : ∂v 1 + (v · grad)v = − grad p + g + nDv r ∂t Application à la mise en rotation d’un fluide Reprenons l’expérience de la mise en rotation uniforme d’un récipient contenant un fluide visqueux tel que la glycérine (Fig. 28.4c). Si la vitesse de rotation est suffisamment faible, l’équation précédente se réduit à : ∂v 1 = − grad p + g + nDv r ∂t En prenant le rotationnel de cette équation, on obtient une équation de diffusion à laquelle satisfait le vecteur vorticité V = rot v/2 : ∂V = nDV ∂t Cette équation traduit la diffusion dans le fluide de la vorticité du récipient solide, laquelle est égale à sa vitesse de rotation (cf. chapitre 29). Expérimentalement, on constate une mise en rotation progressive du fluide. Cette dernière est d’autant plus difficile que la viscosité cinématique est plus faible. Dans un fluide non visqueux, la diffusion de la vorticité serait impossible. On obtient un ordre de grandeur de la durée caractéristique du phénomène de diffusion en notant que n a les dimensions d’une longueur au carré divisée par une durée : n=

L2 T

d’où

T=

L2 rL2 = n h

555

Fluides visqueux Par exemple, pour de la glycérine, dans un récipient de diamètre 5 cm : T=

(5 × 10−2) 2 ≈2s 1, 2 × 10 −3

Réciproquement, en communiquant à l’aide d’un simple agitateur une vorticité à de la glycérine, contenue dans un récipient au repos, on constate que la vorticité diffuse vers le récipient au point de mettre ce dernier en rotation. II . 2 . — Aspect énergétique. Perte de charge Comme pour l’établissement de la relation de Bernoulli, dans le cas d’écoulements parfaits des fluides non visqueux, récrivons l’équation d’Euler en utilisant l’égalité : 1 (v · ∇)v = grad v 2 + rot v × v 2 Il vient : v2 ∂v r + r grad + r rot v × v = − grad p + rg + hDv ∂t 2 Multiplions les deux membres de cette équation d’Euler par le déplacement élémentaire d r = v d t. On obtient : r

∂v · d r + r grad ∂t

v2 2

· d r = − grad p · d r + rg · d r + hDv · d r

Pour un écoulement stationnaire (∂v/∂t = 0) et incompressible (r = Cte), l’équation précédente s’écrit :

 c Dunod – Toute reproduction non autorisée est un délit

d

p v2 −g·r+ 2 r

= nDv · v d t

d e m = nDv · v d t

soit

avec

em =

p v2 + gz + 2 r

si Oz désigne la verticale ascendante. L’interprétation énergétique de l’équation est immédiate : la variation de l’énergie mécanique massique em , somme de l’énergie cinétique massique, de l’énergie potentielle massique de pesanteur et de l’énergie potentielle massique associée aux forces de pression p/r, est égale au travail des forces de viscosité. L’énergie mécanique massique em d’un système fluide, qui ne reçoit pas de travail extérieur, ne se conserve pas mais diminue en raison des forces de viscosité. Ce résultat attendu peut être mis en évidence à l’aide du montage simple représenté sur la figure 31.5a. De l’eau s’écoule dans une canalisation horizontale selon l’axe Oz ; des tubes verticaux permettent de visualiser la perte de pression au fur et à mesure que l’on s’éloigne de la source d’alimentation en eau. Montrons, par le calcul, que l’énergie mécanique du fluide diminue effectivement. Pour cela, nous admettons que le champ des vitesses a l’allure parabolique représentée sur la figure 31.5b, ce qui est raisonnable. Il vient pour le travail massique fourni : L

dw = d t

nDv · v d

En intégrant par parties, on trouve : dw = d t pr2 n

∂v z vz ∂r

= dt

n 0

L 0

− dtn

L 0

∂2 vz v z pr2 d r ∂r 2 ∂vz ∂r

2

pr2 d r

556

31. Fluides visqueux Robinet

pa

g

pa

h1

h4

D O

z

O

z

a)

b) F IG . 31.5.

soit, puisque les vitesses sont nulles sur les parois de la canalisation et ∂v z/∂r = 0 : L

dw = − d t n

0

∂vz ∂r

2

pr 2 d r < 0

On introduit souvent la charge d’une canalisation, c’est-à-dire l’énergie mécanique massique du fluide exprimée sous forme d’une hauteur hc : hc =

em p v 2/2 + g z + p/r v2 =z+ = + 2g rg g g

Dans le montage considéré (Fig. 31.5a), la canalisation est horizontale et la vitesse du fluide constante, puisque le fluide est incompressible et la section droite uniforme. La perte d’énergie mécanique mécanique massique du fluide, ou perte de charge, se manifeste donc par une diminution de la pression le long de la canalisation lorsqu’on s’éloigne de la source : Dhc = Dp/(rg). II . 3 . — Nombre de Reynolds L’équation de Navier-Stokes est techniquement très difficile à résoudre, voire impossible. Aussi, est-il judicieux, préalablement à toute analyse, de comparer la signification des différents termes de cette équation : ∂v r + r(v · grad)v = − grad p + rg + hDv ∂t a) Définition Le nombre de Reynolds permet de comparer le terme d’advection, r(v·grad)v, ou transport global de matière, et le terme de viscosité hDv, de même dimension, à l’origine de la diffusion du champ de vitesse. Dimensionnellement, le rapport de ces deux termes est, si v et L désignent respectivement une vitesse et une longueur caractéristiques du problème considéré : rv2 /L rLv Lv = = d’où h n hDv

Re =

rLv Lv = h n

Le nombre de Reynolds a été introduit au XIX e siècle par l’ingénieur anglais O. Reynolds.

557

Fluides visqueux

Ordres de grandeur : Pour une bille, de diamètre 1 cm et de vitesse 10 m.s −1 , qui tombe verticalement dans l’air, Re vaut : 1, 29 × 0, 01 × 10 rLv ≈ 717 ≈ 1, 8 × 10−5 h Pour un nageur, qui se déplace lentement dans l’eau, à une vitesse de 0, 1 m.s−1 : 103 × 1 × 0, 1 rLv = 105 ≈ 10 −3 h Pour un petit poisson ( L = 1 cm ), de vitesse 1 cm.s−1 , Re ≈ 102 . Enfin, pour des spermatozoïdes humains, on trouve, puisque L ≈ 1 mm , v = 30 mm.s−1 et r ≈ 10 3 kg.m −3 : Re ≈ 3 × 10−5 . Dans ce dernier cas, où Re  1 , la force de viscosité Fv peut être négligée devant le terme d’accélération ma . b) Régime laminaire et régime turbulent Le montage simple de la figure 31.5a permet de montrer qu’il existe deux régimes d’écoulement d’un fluide, ici l’eau, dans une conduite horizontale de section uniforme. i) Régime laminaire Pour Re faible, ce que l’on obtient avec des vitesses d’écoulement faibles, des systèmes de petite taille ou des fluides de forte viscosité, le régime est laminaire. Le terme quadratique d’advection dans l’accélération étant négligeable devant le terme de viscosité, l’équation de Navier-Stokes est linéaire. La diffusion du champ de vitesse est le phénomène prédominant. ii) Régime turbulent En revanche, si Re est grand, c’est l’inverse. La diffusion est négligeable devant l’advection, laquelle se manifeste par la présence de nombreux tourbillons. Le régime est turbulent, c’est-à-dire chaotique et donc imprévisible.

 c Dunod – Toute reproduction non autorisée est un délit

Remarque : La turbulence est encore de nos jours un phénomène qui n’est pas totalement maîtrisé, même si, dans certaines circonstances particulières, on peut la prendre en compte en introduisant des paramètres phénoménologiques introduits de façon ad hoc. La difficulté vient de l’équation de Navier-Stokes qui relève de la mécanique des milieux continus, alors que la turbulence met en jeu, elle, de multiples échelles de longueur, notamment l’échelle microscopique ignorée dans cette équation. iii) Entre les deux régimes extrêmes précédents, coexistent diffusion et advection ; on voit naître quelques tourbillons. c) Écoulements similaires On dit que deux écoulements dans des conduites géométriquement semblables sont similaires s’ils sont caractérisés par un même nombre de Reynolds. Par exemple, un écoulement d’eau dans une conduite cylindrique de 1 m de diamètre, avec une vitesse de 0, 1 m . s−1 , est similaire à un écoulement dans une conduite de 0, 1 m de diamètre avec une vitesse de 1 m . s−1 . De même, l’analyse aérodynamique du mouvement d’une sphère, de diamètre 5 m, se déplaçant dans l’air, à la vitesse de 10 m.s−1 , est similaire de celle du mouvement d’une sphère, de diamètre 1 m, à la vitesse de 50 m.s−1 . Ce résultat est essentiel pour l’expérimentation à faible coût sur maquettes, dans les domaines de la construction aéronautique ou maritime.

558

31. Fluides visqueux

II . 4 . — Écoulement dans une conduite étroite a) Régime laminaire. Formule de Poiseuille Considérons une conduite cylindrique, horizontale, d’axe Oz, de diamètre D = 2R, dans laquelle s’écoule, de façon stationnaire, laminaire et incompressible, un fluide visqueux. Montrons que, dans de telles conditions, comme on l’a supposé précédemment, le profil des vitesses dans une section droite est parabolique (Fig. 31.5b). Pour cela, écrivons l’équation de Navier-Stokes en régime stationnaire, en l’absence de champ de pesanteur : r(v · grad)v = − grad p + hDv Comme l’écoulement du fluide est stationnaire et laminaire, le champ des vitesses a pour expression, compte tenu de la symétrie cylindrique : v(r, z) = v(r, z) e z r étant la coordonnée radiale. En outre, l’écoulement étant incompressible, on a (cf. chapitre 29) : Dr = −r div v = 0 d’où Dt

div v = 0 et

∂vz =0 ∂z

Il en résulte que la vitesse ne dépend pas de z. Elle ne dépend pas non plus de la variable angulaire u, en raison de la symétrie cylindrique. Finalement, la vitesse ne dépend que de la distance r du point à l’axe : vz (r). Puisque (v · grad)v = vz d v z/ d z = 0, l’équation de Navier-Stokes se réduit à : − grad p + h Dv = 0 En projetant cette équation selon les trois axes de la base cylindrique (e r, eu , ez), on obtient : ∂p =0 ∂r

∂p =0 ∂u



∂p + hDvz (r) = 0 ∂z

avec (cf. annexe 5) :

1 ∂ ∂vz r r ∂r ∂r La dernière des trois équations précédentes donne : Dv z(r) =

d p(z) 1 ∂ ∂v z r =h dz r ∂r ∂r

= Cte = K

puisque les deux membres de l’égalité sont des fonctions de variables indépendantes. Il en résulte, d’une part : p − p2 K= 1 L p1 et p2 étant des pressions dans les plans de front distants de L, d’autre part : d d vz r dr dr

=

Kr h

d’où

r d vz Kr2 + Cte = dr 2h

Cette constante est nulle car, pour r = 0, d vz/ d r a une valeur finie. En simplifiant par r et en intégrant une seconde fois, on trouve : vz(r) =

Kr2 + Cte 4h

559

Fluides visqueux On détermine Cte par la contrainte vz (R) = 0 : Cte = −KR2 /(4h)

d’où

vz (r) =

p 1 − p2 2 (R − r 2) 4hL

Le débit-masse dans la conduite s’obtient alors selon : qm =

rvz (r) d S =

Jm · n d S =

R

r 0

p1 − p 2 2 (p − p2) (R − r2)2pr d r = rp 1 4hL 2hL

R 0

(R2 − r2)r d r

En intégrant, on trouve : p 1 − p2 qm = rp 2hL

R 2r 2 r4 − 2 4

R

soit 0

qm =

p D4 p1 − p2 r 128 h L

Ce résultat, relatif au débit-masse dans une conduite, est connue sous le nom de formule de Poiseuille. Notons que qm est proportionnel à la perte de pression linéique (p 1 −p2 )/L et à la puissance quatrième du diamètre D de la conduite ; en outre, il est inversement proportionnel à la viscosité cinématique n. En outre, la perte de pression linéique est proportionnelle à la perte d’énergie linéique et donc à la perte de charge linéique : p 1 − p2 h − h2 = rg 1 L L La formule de Poiseuille est parfois exprimée à l’aide du débit-volume qv , qui vaut, le liquide étant incompressible, qm/r : p D 4 p1 − p2 qv = L 128 h On peut vérifier expérimentalement la loi de Poiseuille à l’aide d’un montage analogue à celui de la figure 31.5a. C’est ce qui a été fait, avec une excellente précision (inférieure à 0, 001), pour des liquides tels que l’eau, le benzène et la glycérine s’écoulant dans des tubes cylindriques de 0, 2 mm de diamètre. Il est instructif d’exprimer la perte de pression linéique en fonction du nombre de Reynolds, lequel peut être pris égal à : Re = vmD/n où vm est la vitesse moyenne du fluide sur une section de la canalisation. Une telle vitesse est directement reliée au débit-volume puisque :

 c Dunod – Toute reproduction non autorisée est un délit

qv =

1 dt

vz (r) d S d t =

Par conséquent : vm = On peut alors écrire :

qv 4q v = S pD2

1 S

vz (r) d S

et vm =

et Re =

vmD r 4q v = n h pD

v z (r) d S

rv2 p1 − p2 = m f (Re) 2D L f (Re) étant un facteur de perte de charge fonction de Re. On trouve son expression dans le cas considéré, à partir des deux écritures de la perte de pression linéique : p1 − p2 rvm2 rv2m 128hq v 32hvm 64 × = = = = f (Re) 2D 2D L pD 4 D2 rDv m/h

d’où

f (Re) =

64 Re

Ainsi, en régime laminaire, la courbe lg f en fonction de lg Re est une droite de pente −1, ce que l’on constate expérimentalement (Fig. 31.6).

560

31. Fluides visqueux

Ordre de grandeur : Pour de l’eau, qui s’écoule dans un tube de diamètre D = 5 mm, avec un débit de 7, 55 mL . s−1 , on trouve : vm =

vm D 4qv 4 × 7, 55 × 10 −6 5 × 10 −3 × 0, 38 −1 = = = 0 , 38 m . s et Re = = 1 727 pD 2 p × 25 × 10−6 1, 1 × 10−6 n lg f

lg(2×10−2 )

Régime turbulent (pente −1/4)

Régime laminaire (pente −1)

lg(4×103 )

lg Re

F IG . 31.6.

b) Régime turbulent En augmentant progressivement q v, on fait naître d’abord les premiers tourbillons, puis un régime turbulent s’établit dès que Re atteint une valeur de l’ordre de 3 000. L’analyse expérimentale précédente peut alors être prolongée dans le cas des fortes valeurs du nombre de Reynolds. On détermine le facteur f de perte de pression relative, pour chaque valeur de Re obtenue, en changeant qv . Sur la figure 31.6 on distingue clairement les deux régimes. L’expérience montre que la pente de la courbe en régime turbulent n’est pas −1 mais −1/4. II . 5 . — Écoulement laminaire de Couette La méthode de Couette, imaginée par l’ingénieur français M. Couette, consiste à mesurer le couple que subit un cylindre C d’axe vertical, du fait des forces de viscosité exercées par un liquide ; ce dernier est mis en mouvement en faisant tourner un cylindre extérieur Ce (Fig. 31.7). z

R C

h

A

Ce

O Ve

Re

F IG . 31.7.

Le cylindre C, de rayon R et de hauteur h, est maintenu fixe, grâce à un couple mécanique antagoniste (−Cue ), alors que le cylindre extérieur C e , de rayon Re , tourne avec une vitesse angulaire Ve constante. La vitesse des particules de fluide passe donc de 0 à V e Re lorsque la coordonnée radiale r varie entre R et Re.

561

Fluides visqueux

Compte tenu du mouvement orthoradial imposé au fluide par C e et de la symétrie cylindrique, la vitesse du fluide n’a qu’une composante orthoradiale et cette dernière ne dépend que de r. La force que subit l’élément de surface latérale hR d u du cylindre C a pour expression : d F = hRh

dv dr

R

d u eu

d’où la somme des moments des forces de viscosité selon l’axe de rotation : OA × d F = 2phhR 2

G Oz = e z ·

dv dr

R

Si les rayons Re et R sont voisins, on peut admettre avec une bonne approximation que le gradient de vitesse est uniforme et vaut : dv dr

= R

VeR e Re − R

d’où

GOz = 2phhR2

Ve R e Re − R

En exprimant l’équilibre du cylindre C sous l’action des forces de viscosité et du couple de torsion, on obtient : Ve Re Cue (Re − R) 2phhR 2 − Cu e = 0 soit h = 2phR eR2 Ve Re− R De la mesure de ue, on déduit la viscosité. Donnons les autres de grandeur d’un tel viscosimètre vendu dans le commerce : D ex = 2Re = 27, 6 mm

D in = 2R = 25, 15 mm

C = 67, 37 × 10

−6

N.m

h = 92, 39 mm

V e = 2p rad.s −1

Il en résulte, pour l’eau, un angle de torsion égal à : ue =

2phhRe R2 V e ≈ 0, 096 rad ≈ 5, 5◦ C(R e − R)

 c Dunod – Toute reproduction non autorisée est un délit

III . — FORCES EXERCÉES PAR LES FLUIDES EN MOUVEMENT L’expérience courante montre qu’un corps solide en mouvement dans un fluide réel subit de la part de ce fluide des forces dont la somme comporte deux contributions : l’une, parallèle au mouvement et opposée à la vitesse relative du fluide par rapport au solide, est la traînée T ; l’autre, perpendiculaire à cette vitesse, est la portance N. III . 1 . — Facteur Cx La traînée peut se mettre sous la forme générale : rf v2 S 2 dans laquelle rf est la masse volumique du fluide, v la vitesse de translation de l’objet considéré, S la surface maximale de sa section dans un plan perpendiculaire à la direction de la vitesse et C x un facteur sans dimension qui ne dépend que du nombre de Reynolds Re = rf vD/h. Sur la figure 31.8, on a représenté approximativement la variation de lg Cx en fonction de lg Re. On distingue deux régions : pour Re suffisamment faible, lg Cx décroît linéairement avec lg Re ; pour Re compris entre 100 et 200 000, Cx est pratiquement constant. T = C x(Re)

562

31. Fluides visqueux lg Cx

0

lg Re

lg 100 F IG . 31.8.

III . 2 . — Faible nombre de Reynolds a) Formule de Stokes Si Re  100, le facteur C x varie sensiblement suivant la loi : ln C x = − ln Re + Cte

d’où

Cx Re = Cte

Par conséquent, la traînée s’écrit :

Cte rf v 2 S S = Cte × hv 2 2D Re Pour une sphère de rayon r, on a évidemment S = pr 2 et D = 2r ; on montre, en outre, que la constante numérique est égale dans ce cas à 24. Il en résulte l’expression suivante de la traînée : T = Cx

rf v 2 S= 2

T = 6phr v On trouve ainsi l’expression de la force de frottement visqueux linéaire dont le coefficient a vaut 6phr, h étant la viscosité du fluide dans lequel évolue la sphère. Il est instructif d’exprimer cette force T en fonction du faible nombre de Reynolds Re = rv r/h : T = 6phr v = 6pRe

h2 r

On voit d’une part que la quantité h2 /r est homogène à une force, et surtout que des corps, de même Re , sont soumis à une même traînée, quelles que soient leurs autres caractéristiques. Remarque : Pour des nombres de Reynolds encore plus faibles, on peut négliger le terme d’accélération d’un corps devant la force de viscosité, ce qui donne la relation suivante, en désignant par Fs une force supplémentaire, par exemple le poids de ce corps : dv Fs = −av + Fs ≈ 0 d’où v ≈ a dt Dans ce cas, le vecteur vitesse v est proportionnel à la force F s . C’est la raison pour laquelle, on dit parfois que la mécanique d’Aristote, généralement incorrecte, trouve un domaine de validité, si les corps considérés à l’échelle macroscopique sont de masse négligeable. m

b) Chute d’une bille sphérique dans de la glycérine Reprenons avec soin l’étude de la chute d’une petite bille d’acier dans le récipient contenant de la glycérine (Fig. 31.2). Avec de telles billes, de masse volumique r = 7 800 kg.m−3 et de diamètre D = 2r = 2 mm, on a mesuré une vitesse limite égale à vl = 13, 3 mm.s−1 dans de la glycérine (masse volumique rf = 1 260 kg.m −3 ). Calculons le nombre de Reynolds : Re =

1 260 × 13, 3 × 10−3 × 2 × 10 −3 r f vD ≈ 0, 02 = 1, 49 h

563

Fluides visqueux

Nous pouvons donc appliquer la loi fondamentale de la dynamique à cette bille A, de masse m, sachant que les forces qui s’exercent sur elle sont : la pesanteur mg = r g, étant le volume de la bille, la poussée d’Archimède −mg, avec m  = r f et la force de frottement visqueux sous sa forme linéaire de Stokes Ff = −a v, avec a = 6phr. Il vient : r

dv = dt

r 1−

rf r

g−av

d’où

dv v + = ga dt t

en posant ga = 1 −

rf r

g

et t =

r a

La solution de cette équation différentielle linéaire est bien connue (cf. annexe 4) : v = Cte × exp −

t + g at t

soit

v = ga t − g at exp −

t t

puisque, initialement, la particule a été abandonnée sans vitesse. Ainsi, dans la glycérine, pour laquelle t est faible car a est grand, la bille atteint rapidement la vitesse limite vl = ga t = mga/a. Notons que la vitesse ne dépend plus des instants antérieurs : la vitesse est proportionnelle à la force ! Pour une fine gouttelette d’eau, de rayon r = 1 mm, en chute dans l’air, le calcul de la vitesse limite donne : vl =

2 r2 2 10 −12 (r − rf )g = × × (1 000 − 1, 3) × 9, 80 ≈ 0, 12 mm.s−1 9 h 9 1, 8 × 10 −5

c) Applications (1) Viscosimètre à chute L’expérience précédente fournit une méthode de détermination de la viscosité à partir de la vitesse limite, une fois établie la relation de Stokes entre a et h : a = 6phr, r étant le rayon de la bille. Dans ces conditions : h=

(4pr3/3)r(1 − r f /r)g 2 r2 a mga = = = (r − r f )g 6pr 6prvl 6prv l 9 vl

Dans les conditions de l’expérience précédente, on trouve, puisque v l = 13, 2 mm.s−1 :

 c Dunod – Toute reproduction non autorisée est un délit

h=

2 r2 2 10 −6 × (7, 8 − 1, 26) × 103 × 9, 81 ≈ 1, 2 Pa.s (r − r f )g = × 9 vl 9 13, 2 × 10 −3

C’est ainsi que fonctionne le viscosimètre à chute. (2) Séparation de grains en fonction de leur taille Les grains sphériques d’une poudre peuvent être séparés et classés en fonction de leur rayon, puisque leur vitesse limite, au cours d’une chute dans un fluide visqueux, est proportionnelle à r 2 : vl =

2 r2 (r − rf )g 9 h

C’est ainsi que l’on classe les grains d’émeri. On peut rendre le procédé encore plus efficace en remplaçant le champ de pesanteur par un champ apparent plus intense, ce que l’on obtient aisément dans un récipient en rotation uniforme (cf. chapitre 28). Comme ce champ présente une contribution radiale positive, les grains tombent en s’éloignant de l’axe de rotation. Remarque : Les grains, de masse volumique plus faible que celle du liquide, se rassemblent sur l’axe, puisqu’ils s’élèvent en suivant la direction du champ apparent précédent (cf. chapitre 28).

564

31. Fluides visqueux

(3) Expérience de Millikan Un exemple important est fourni par l’expérience réalisée en 1911 par le physicien américain R. Millikan afin de déterminer la charge de l’électron. Il étudia la chute verticale d’une gouttelette d’huile soumise à son poids, à une force de Stokes et à la force qu’exerce un champ électrique uniforme lorsque la goutte acquiert une charge électrique (cf. Électromagnétisme et chapitre 4). À l’aide d’une analyse du mouvement analogue à celle utilisée dans le viscosimètre à chute, il prouva que la charge électrique acquise par les gouttelettes était, au signe près, un multiple entier de la charge élémentaire e = 1, 6 × 10−19 C , ce qui lui valut le prix Nobel en 1923. Contentons-nous ici de calculer le nombre de Reynolds de la gouttelle d’huile, dans le cas précis de l’expérience : rayon r ≈ 2, 76 mm , vitesse v ≈ 0, 858 mm.s−1 , viscosité de l’air h ≈ 1, 824 × 10 −5 P.s : Re =

rvr 1, 29 × 0, 858 × 10−3 × 2, 76 × 10−6 ≈ ≈ 1, 67 × 10 −2 −5 1, 824 × 10 h

III . 3 . — Fort nombre de Reynolds a) Formule quadratique Si Re est suffisamment grand, C x est pratiquement constant ; la traînée est alors proportionnelle au carré de la vitesse : rf v 2 S T = Cx 2 Le facteur C x dépend de la turbulence du régime imposé par l’objet immergé. Le cas d’objet en mouvement dans l’air, ou immergé dans un courant d’air, est évidemment capital. L’appareil représenté sur la figure 31.9a permet de mesurer le facteur Cx d’objets de formes différentes.

Cx = 0,4

Soufflerie a)

Cx = 0,45

Cx = 0,04

b) F IG . 31.9.

À l’aide d’une balance, on compense par le poids de masses marquées la traînée qu’exerce de l’air en mouvement, issu d’une soufflerie, sur un objet de forme géométrique déterminée. Le facteur Cx a la valeur la plus faible lorsque l’objet a une forme aérodynamique (Fig. 31.9b). b) Goutte de pluie en chute dans l’air Il existe dans ce cas aussi une vitesse limite. En effet, la loi fondamentale de la dynamique s’écrit, pour une bille sphérique soumise à son poids, à la poussée d’Archimède et à la force de frottement T : r

dv = dt

r 1−

rf r

g − b v2

v v

d’où

d v bv 2 v + = ga dt r v

en posant ga = 1 −

On en déduit la vitesse limite suivante en faisant a = d v/ d t = 0 : C x rf v 2l S = ga 2 r

d’où

vl =

2ga r Cx rf S

1/2

=

8gr(r/rf − 1) 3Cx

1/2

rf r

g

565

Fluides visqueux

Calculons le nombre de Reynolds pour une goutte de pluie dans l’air, assez petite pour que l’on puisse la supposer sphérique. Comme son diamètre est de l’ordre de 1 mm et sa vitesse de 5 m.s −1 , on a : r f vL 1, 3 × 5 × 1 × 10−3 = = 3 610 1, 8 × 10 −6 h ce qui justifie la forme quadratique adoptée. La valeur de la vitesse prévue par l’analyse est bien conforme à celle que fournit l’expérience. En effet, en prenant pour C x la valeur 0, 45 relative à une sphère, on obtient : Re =

8 × 9, 80 × 0, 6 × 10 −3(1 000/1, 3 − 1) 3 × 0, 45

vl =

1/2

≈ 5, 2 m . s−1

III . 4 . — Traînée et portance d’une aile d’avion a) Plaque mince rectangulaire inclinée Une plaque mince rectangulaire, faisant un angle g avec la direction de la vitesse v d’un fluide en mouvement, constitue une représentation approchée mais efficace d’une aile d’avion ou d’une voile de bateau. Sur la figure 31.10a, on a représenté la déformation des lignes de courant ; ces lignes sont proches les unes des autres au point haut H , et au contraire espacées au point bas B . Il en résulte que la vitesse du fluide est plus élevée le long des lignes de courant dans le voisinage de H que le long des lignes voisines de B . On en déduit, en appliquant le théorème de Bernoulli, que la pression de l’air est plus élevée en B qu’en H , d’où la portance orientée selon la verticale ascendante. Il en résulte que la force résultant des actions du fluide a deux composantes : l’une, la traînée T , orientée selon la vitesse du fluide, l’autre, la portance N , perpendiculaire à cette vitesse. Ces deux composantes ont pour expressions respectives : r fv 2 rf v2 S et N = C y S 2 2 rf étant la masse volumique du fluide, S la surface de la projection de la plaque dans un plan perpendiculaire à v , C x le facteur de traînée et C y le facteur de portance. T = Cx

y H  c Dunod – Toute reproduction non autorisée est un délit

v

N

°

y R v

T B

°

H

x

x B b)

a) F IG . 31.10.

b) Aile d’avion L’aile d’avion diffère sensiblement de la plaque précédente par son profil en fuseau et incurvé, comme le montre la figure 31.10b, ce qui permet d’obtenir un facteur C x très faible. Cependant, l’analyse en termes de traînée et de portance orientée selon la verticale ascendante est analogue à la précédente. Pour une aile donnée, le rapport des facteurs C y/C x , qui interviennent dans les expressions de la traînée et de la portance, ne dépend que de l’angle g que fait la droite HB reliant les extrémités avec la direction du mouvement. On l’appelle la finesse de l’aile et la courbe donnant C y en fonction de Cx ,

566

31. Fluides visqueux

pour différentes valeurs de g, est sa polaire (Fig. 31.11). Au cours d’un vol normal pour lequel g est faible, Cx ≈ 0, 05 et Cy ≈ 0, 8, ce qui correspond à une finesse de 16. Cy 9◦

0;8

15 ◦

3◦ 0;4 0 −0;4

− 3◦ 0;1

0;2

−9 ◦

Cx

F IG . 31.11.

F IG . 31.12.

III . 5 . — Solide en mouvement accéléré dans un fluide Si le corps immergé est accéléré par rapport au référentiel lié au fluide, apparaît une force supplémentaire exercée par ce dernier, différente de la poussée d’Archimède et de la force de viscosité, mais en relation avec la couche de liquide qui l’entoure. C’est ce que montre l’expérience suivante dans laquelle on se propose de déterminer l’accélération initiale d’une balle de ping-pong maintenue immergée dans un récipient rempli d’eau, grâce à un fil (Fig. 31.12). Lorsque le fil cesse d’exercer une tension, on constate que l’accélération initiale a i n’a pas la valeur attendue, mais une valeur bien plus faible. En effet, écrivons l’équation du mouvement, sans tenir compte de cette force supplémentaire : m

dv r = mg − mf g − av = mg 1 − f dt r

− av

r et rf étant les masses volumiques de la balle et du fluide respectivement. On en déduit, à l’instant initial où v = 0 , l’accélération suivante : ai = −g (r f /r − 1) . Comme m = 2, 4 g et D = 2R = 4 cm , r ≈ 71, 6 kg.m−3 , alors que rf ≈ 1 000 kg.m −3 , l’expression donne : ai ≈ −13g. Or l’expérience, réalisée avec soin, montre que l’ascension de la balle suit une loi horaire parabolique d’accélération a i ≈ 1, 5g . On l’interprète en considérant une force supplémentaire proportionnelle à l’accélération de balle et à la masse du fluide de remplacement mr = rf : F (a) = −m am r

dv dt

m a étant un facteur qui vaut 1/2 pour l’objet considéré de forme sphérique. La quantité mamr homogène à une masse, est souvent appelée la masse ajoutée. Finalement l’équation du mouvement est la suivante : dv mr d v m = mg − mf g − av − dt 2 dt d’où l’accélération initiale telle que : m+

mr 2

ai = m g 1 −

rf r

et

ai = −g

rf /r − 1 ≈ −1, 6 g 1 + rf /(2r)

en bon accord avec la valeur trouvée expérimentalement qui vaut approximativement −1, 5 g .

567

Fluides visqueux

CONCLUSION Retenons les points essentiels. (1) Une portion de fluide exerce sur une autre, à travers la surface qui sépare les deux portions, une force de frottement, proportionnelle à la vitesse de glissement. La viscosité h est le coefficient caractéristique de cette force que l’on interprète comme un transfert de quantité de mouvement. Ce transfert fait apparaître la viscosité cinématique : n = h/r où r est la masse volumique du fluide. L’eau est un liquide très peu visqueux (h = 0, 001 Pa.s), contrairement aux huiles dont la viscosité est de l’ordre de 1 Pa.s. (2) Le nombre de Reynolds Re = rvL/h , sans dimension, v et L étant une vitesse et une longueur caractéristiques, permet de distinguer les régimes laminaires, pour lesquels Re est faible, des régimes turbulents caractérisés par des valeurs fortes de Re. (3) La formule de Poiseuille résume les lois relatives à l’écoulement stationnaire, incompressible, d’un fluide visqueux dans une conduite cylindrique horizontale : p D 4 p1 − p2 r qm = L 128 h (4) Si Re est faible, la force de frottement qui s’exerce sur une bille, de rayon r, satisfait à la loi linéaire de Stokes : F = −av avec a = 6phr.

 c Dunod – Toute reproduction non autorisée est un délit

(5) Si Re est fort, la force de frottement est proportionnelle au carré de la vitesse. Dans certaines conditions largement exploitées en aérodynamisme, l’action du fluide visqueux, de masse volumique r f , fait apparaître, outre la traînée qui est une force de frottement visqueux opposée au mouvement, une force normale à la direction du mouvement et pouvant compenser le poids, appelée la portance : r fv 2 rf v2 S S et N = C y T = Cx 2 2 (6) Même si, en dehors d’une couche limite, située dans le voisinage des obstacles, le fluide réel se comporte comme s’il n’était pas visqueux, ce qui justifie l’étude préalable des fluides non visqueux, la prise en compte de la viscosité est indispensable, non seulement pour un traitement approfondi des problèmes en mécanique des fluides, mais aussi pour comprendre le comportement réel des fluides et connaître les forces exercées par les fluides sur les corps en mouvement.

EXERCICES ET PROBLÈMES P31– 1. Fluide entre une plaque fixe et une plaque parallèle mobile Un fluide, situé entre deux plaques parallèles, distantes de h, est mis en mouvement par viscosité, en déplaçant la plaque supérieure à la vitesse Uex. Son champ de vitesse est : v = (Uy/h) e x . 1. Calculer div v et le vecteur tourbillon. 2. Trouver la fonction de courant C. P31– 2. Viscosimètre de Couette On considère le viscosimètre de Couette, constitué de deux cylindres de même axe vertical Oz et de même hauteur h (Fig. 31.7). Le premier C, intérieur, de rayon R, est maintenu fixe, alors que le second Ce , extérieur, de rayon Re > R, est mobile autour de son axe Oz, avec une vitesse angulaire

568

31. Fluides visqueux

uniforme V e . La vitesse des particules du fluide, qui remplit l’espace entre les cylindres, passe donc de 0 à VeR e lorsque la coordonnée radiale r varie entre R et Re . Les effets de bord sont négligés. 1. Montrer que le champ des vitesses en coordonnées polaires dans le fluide (r, u) est de la forme : 1 d(rv u) = Cte v = v u eu avec r dr 2. En déduire l’expression suivante de v u : r − R 2/r Re − R 2/Re V étant une vitesse que l’on calculera en fonction de R e et Ve . vu = V

3. Calculer la quantité (d v u/ d r) r=R en fonction de Ve et u = R/R e . Comparer sa valeur à Ve /(1 − u). Sachant que Re = 5 cm, pour quelles valeurs de R, l’erreur, faite sur la mesure de la viscosité, en admettant cette valeur approchée de (d v u/ d r)r=R , est-elle inférieure à 1/100 ? P31– 3. Liquide visqueux en rotation Un récipient cylindrique, de diamètre 2R et contenant un liquide visqueux tel que l’huile, est mis en rotation uniforme autour de son axe de révolution avec une vitesse angulaire V. 1. Trouver, à partir de l’équation de Navier, le champ de vitesse v = v u(r)e u . 2. Montrer que : r(v · grad)v = grad

v2 2

+ rot v × v = −r V 2e r

3. Quel est le champ de pression p(r, z) ? En déduire la forme de la surface du liquide. P31– 4. Viscosité de l’huile de ricin On veut déterminer la viscosité de l’huile de ricin, extrait d’une plante oléagineuse de masse volumique 970 kg . m−3, en mesurant la vitesse limite de chute d’une bille de verre (masse volumique 2 600 kg . m−3, diamètre 1 mm). L’expérience donne une vitesse limite de 0, 75 mm . s −1. 1. Écrire l’équation du mouvement à laquelle satisfait la bille. Trouver l’évolution de la vitesse, sachant qu’initialement la vitesse de chute est nulle. 2. Déduire de la vitesse limite la valeur de la viscosité. P31– 5. Traînée d’une balle de fusil Une balle de fusil, de calibre 8 mm, de masse 10 g, présente un facteur C x = 0, 40 grâce à un profil en ogive à l’avant et effilé à l’arrière. Comparer la traînée de cette balle à son poids, sachant que sa vitesse est de 500 m . s−1 et la masse volumique de l’air 1, 3 kg . m −3 . P31– 6. Freinage d’un avion par un parachute Un avion de masse 4 t est freiné à l’atterrissage, à l’aide d’un parachute de diamètre D = 4 m. Sa vitesse, au moment de l’ouverture du parachute, était v 0 = 50 m . s−1 . 1. Trouver la vitesse v(t).

569

Fluides visqueux

2. Le facteur C x du parachute vaut 1, 5 et la masse volumique de l’air est 1, 3 kg . m −3 . Quelle doit-être la distance d à parcourir pour que la vitesse soit réduite à 5 m . s −1 ? P31– 7. Propulsion d’un bateau à voile On assimile la voile d’un petit bateau à une surface plane S v placée dans l’écoulement d’air que constitue le vent soufflant dans la voile. Le bateau se déplace suivant un axe Ox à vitesse constante par rapport au référentiel terrestre (Fig. 31.13). 1. Établir la relation entre S v , l’angle f que fait la voile avec la vitesse v1 , de norme v, du vent et la section S a de la veine d’air qui arrive sur la voile. On admet que l’action de la voile consiste seulement à dévier de 2f le tube de courant d’air qui aurait traversé S v en l’absence de voile. 2. Montrer que le facteur de traînée C x a pour expression : Cx = 4 sin 2 f sin(a − f), a étant l’angle que fait la direction du vent avec l’axe Ox : a = (Ox, −v1 ). y

O

Vent

y

v1 α

x

A b

φ

x

φ Sv



A Voile

F IG . 31.13.

F IG . 31.14.

 c Dunod – Toute reproduction non autorisée est un délit

P31– 8. Anémomètre à coupelles Un anémomètre est constitué principalement d’un ensemble de quatre coupelles formant un solide en rotation autour d’un axe vertical Oz avec la vitesse angulaire V. La figure 31.14 représente schématiquement un tel solide avec ses quatre coupelles identiques en forme de demi-sphères creuses, de rayon R, dont les centres géométriques sont situés à la même distance b = 5 cm de l’axe de rotation. La liaison pivot qui maintient l’axe de l’anémomètre vertical est parfaite. En première approximation, on peut supposer que les actions qu’exerce, sur l’anémomètre, l’air environnant, de vitesse u = u e x, sont torsoriellement équivalentes aux deux vecteurs liés suivants : {A, F} =

rS rS  C x(u − bV)2 ex et {A  , F } = Cx (u + bV)2 ex 2 2

r étant la masse volumique du fluide et S = pR 2 ; Cx = 1, 42 et C x = 0, 38 sont les facteurs C x liés à la géométrie des coupelles et à la forme concave ou convexe qu’elles présentent au vent. Les points A et A  ont pour coordonnées respectives dans R : (0, b, 0) et (0, −b, 0). On constate que la vitesse de rotation V acquise par l’anémomètre est constante. Quelle est, en fonction du rapport g = C x /C x , b et V, la vitesse du vent ? Calculer cette vitesse en km . h−1, si l’anémomètre tourne à la vitesse angulaire de 30 tr . s −1 .

570

31. Fluides visqueux

P31– 9. Petit animal en mouvement dans l’eau Un petit animal, de masse m , qu’on assimile à une sphère de rayon r = 1 mm , se déplace dans l’eau ( r = 1 000 kg.m−3 et h ≈ 1, 1 × 10−3 Pa.s ), avec une vitesse égale à 30 mm.s −1 . Sa masse volumique est voisine de celle de l’eau. 1. Calculer le nombre de Reynolds Re . 2. Établir l’équation suivante du mouvement : d v/ d t = v/t + F/m, F étant une force occasionnelle exercée sur l’animal et t une grandeur que l’on exprimera en fonction de r , h et r . 3. Alors que l’animal a une vitesse v 0 , on exerce sur lui une force constante F0 afin de le stopper. a) Établir la relation entre la durée Dt nécessaire, la distance Dx qu’il parcourt avant de s’arrêter et vF = F0 t/m . b) Montrer que Dt est donné par l’expression : Dt = t ln (1 + v 0 /vF) . c) Sachant que Dt ≈ 0, 6 ms , calculer F 0 et Dx .

32 Ondes mécaniques dans un milieu continu Dans ce chapitre, nous nous proposons d’étudier la propagation des ondes mécaniques dans un milieu matériel continu, notamment l’air. On sait que, contrairement aux ondes électromagnétiques, les ondes mécaniques ont besoin d’un support matériel pour se propager. Si la fréquence des ondes f est comprise entre 20 Hz et 20 kHz, le domaine spectral est celui de l’acoustique : l’onde mécanique correspondante est le son ; en marge de ce domaine, ces ondes sont appelées les infrasons pour f < 20 Hz et les ultrasons pour f > 20 kHz. Nous ne développerons pas ici les généralités sur les ondes, notamment l’établissement de l’équation de propagation le long d’une corde tendue et sa résolution étudiées par ailleurs (cf. Optique). Nous supposerons connues les expressions des ondes monochromatiques planes ou sphériques ainsi que leur importance dans la représentation harmonique d’une onde quelconque par un ensemble d’ondes monochromatiques (cf. Électromagnétisme). En revanche, nous soulignerons les spécificités de la propagation des ondes mécaniques dans des milieux fluides.

 c Dunod – Toute reproduction non autorisée est un délit

I . — PROPAGATION D’ONDES MÉCANIQUES Lorsqu’un corps vibre en un point d’un milieu matériel, cette vibration est transmise aux autres points, de proche en proche, du fait du contact des différents points du milieu. Il en résulte une propagation de la vibration sous forme d’une onde. Cette propagation de proche en proche dans un milieu continu implique des déplacements des particules du milieu, lesquels sont à l’origine de déformations et par conséquent de modifications des variables du milieu telles que la pression. L’élongation de la vibration, notée C, dépend alors du temps t par l’argument t − x/v, x étant une variable spatiale et v la vitesse de propagation de l’onde, appelée aussi célérité (cf. Optique) ; cette vitesse dépend notamment des coefficients de compressibilité du fluide dont il faut rappeler la définition. I . 1 . — Coefficients de compressibilité d’un fluide On sait que les variables macroscopiques qui caractérisent l’état d’équilibre thermodynamique d’un fluide sont reliées par son équation d’état (cf. Thermodynamique) : f (p, , T) = 0, p étant la pression, le volume et T la température absolue.

572

32. Ondes mécaniques dans un milieu continu

On définit généralement deux coefficients de compressibilité, l’un isotherme et l’autre isentropique (cf. Thermodynamique) : 1 ∂ 1 ∂ kT = − et k S = − ∂p T ∂p S Pour un gaz parfait, l’équation d’état d’une masse m de gaz est : p = nRT = mrT ou p = rrT, n étant le nombre de moles et r la masse volumique m/ . En outre, un tel gaz, en évolution isentropique, satisfait à l’équation de Laplace p g = Cte, g étant le rapport des capacités thermiques à pression et volume constants. Les coefficients de compressibilité valent dans ce cas : 1 −nRT 1 1 1 dp d et kS = − puisque =0 kT = − − = = +g 2 p p gp gp p I . 2 . — Ondes longitudinales dans un milieu homogène unidimensionnel Dans un fluide au repos par rapport au référentiel du laboratoire R = Oxyz, imposons à une paroi plane rigide un mouvement oscillant de faible amplitude, suivant une direction Ox (Fig. 32.1). On constate que le déplacement de la plaque se transmet dans tout le milieu, suivant la direction du mouvement, avec la même fréquence, mais avec un certain retard ; on dit qu’il y a propagation du déplacement d’une section de fluide. Nous nous proposons d’établir l’équation de propagation, dans l’hypothèse d’un milieu infini transmettant le mouvement de la plaque, en négligeant les effets dus aux dimensions latérales de la plaque. Désignons par C(x, t) le déplacement, à l’instant t, des particules de la section AB, par rapport à leur position de repos A0 B0, d’abscisse x ; C(x + d x, t) est, au même instant t, le déplacement des particules de la section CD par rapport à sa position de repos C 0D0 , d’abscisse x + d x. L’épaisseur de la tranche cylindrique ABCD est d x au repos et d x + C(x + d x, t) − C(x, t) à l’instant t. Sa masse est aussi celle de la tranche A 0B 0C 0D 0 , soit rS d x, si S désigne la surface d’une section et r la masse volumique du fluide. dx x O

Bo S

B

x + dx Co C

A

Do

x

Ao

Ψ(x, t)

D Ψ(x + dx,t)

F IG . 32.1.

a) Équation différentielle du mouvement L’équation du mouvement de la tranche cylindrique ABCD est obtenue en appliquant le théorème de la quantité de mouvement à la tranche cylindrique ABCD. Les forces à considérer sont, outre les forces occasionnelles Foc dont la pesanteur, les forces de pression qui s’exercent sur les bases AB et CD. On a donc, en projetant selon Ox : ∂ 2C(x, t) = SpAB − Sp CD + F oc ∂t2 Il convient alors d’exprimer la différence de pression pCD − pAB en fonction de C(x, t). Notons tout d’abord que le déplacement du fluide, à un instant, s’accompagne d’une dilatation qui vaut : rS d x

S[C(x + d x) + (x + d x) − C(x) − x] = S(d C + d x) = S d x 1 +

∂C ∂x

573

Ondes mécaniques dans un milieu continu d’où la dilatation relative de la tranche : d

=

∂C S(d C + d x) − S d x = ∂x Sdx

Or, une dilatation relative du fluide implique une surpression P = p − p e , pe étant la pression à l’équilibre : kS = −

1

∂ ∂p

donne

P = p − pe = −

1 kS

d

=−

1 ∂C soit kS ∂x

P=−

1 kS

∂C ∂x

On a donc, entre A0B0 et AB, au même instant t, la différence de pression suivante : pAB = pe (x, t) −

1 kS

∂C ∂x

x,t

De même, entre C0D 0 et CD, la différence de pression est, à l’instant t : pCD = pe (x + d x, t) −

1 kS

∂C ∂x

x+d x,t

On en déduit la différence des pressions pCD − pAB : pCD − pAB = p e(x + d x, t) − pe (x, t) −

1 kS

soit pe(x + d x, t) − pe (x, t) −

1 kS

∂C ∂x

x+d x,t

∂C ∂x



x,t

∂2 C dx ∂x2

L’équation du mouvement s’écrit donc :

 c Dunod – Toute reproduction non autorisée est un délit

rS d x

S ∂2C ( x + d x , t ) + Sp = −Sp (x , t ) + e e kS ∂t2

∂2 C ∂x2

d x + Foc

Comme, en l’absence de mouvement, on a : 0 = −Sp e (x + d x, t) + Spe (x, t) + Foc, il vient, en simplifiant et en négligeant les faibles variations de r autour de sa valeur à l’équilibre r e : re S d x

S ∂2 C ∂ 2C dx = k S ∂x2 ∂t 2

soit

∂ 2C ∂ 2C k = r e S ∂x2 ∂t 2

Notons qu’en dérivant partiellement cette équation par rapport à x et en la multipliant par −1/k S, on obtient une équation analogue relative à la surpression P ; de même, en dérivant partiellement l’équation par rapport au temps, on trouve une autre équation analogue relative à la vitesse de déplacement u = ∂C/∂ t suivant la direction de propagation : ∂ 2P ∂ 2P = r k e S ∂x2 ∂t2

et

∂2 u ∂2 u = r k e S ∂x2 ∂t 2

574

32. Ondes mécaniques dans un milieu continu

b) Nature du mouvement L’équation différentielle précédente est caractéristique d’un phénomène ondulatoire. Cependant, contrairement à la propagation le long d’une corde où l’onde est transversale, l’onde est ici longitudinale car le déplacement C(x, t) se produit selon la direction de propagation. La solution générale de cette équation s’écrit (cf. Optique) : C(x, t) = C + t −

x cs

+ C− t +

x cs



cs =

1 re k S

1/2

est la vitesse de propagation des ondes mécaniques dans le milieu. Le premier terme représente une onde se propageant dans le sens des x croissants alors que le second représente une propagation dans le sens des x décroissants. Une solution simple est l’onde monochromatique, plane, progressive, d’expression complexe : C(x, t) = A exp −iv t −

x cs

= A exp[−i(vt − kx)]

où A est l’amplitude réelle, v la pulsation ou fréquence angulaire et k = v/cs le nombre d’onde. On introduit souvent l’amplitude complexe c = A exp(ikx), la fréquence f = v/(2p), la période T = 2p/v et la longueur d’onde l = 2p/k : f=

1 v = 2p T

l=

2p 2pcs cs = c sT = = v n k

Ce type de solution est important en raison de la possibilité de considérer toute onde comme une superposition linéaire d’ondes monochromatiques planes (cf. Optique). Exprimons, dans le cas d’une onde monochromatique plane, l’expression complexe de la surpression : 1 P = p − pe = − kS

∂C ∂x

=

ikA exp[−i(vt − kx)] = ir eAvcs exp[−i(vt − kx)] kS

d’où l’amplitude Pm de la surpression : P m = re Avcs . I . 3 . — Ordres de grandeur de la vitesse de propagation des ondes acoustiques Notons tout d’abord que les ondes acoustiques ne se propagent pas dans le vide, ce que l’expérience, représentée sur la figure 32.2, confirme aisément : le son émis par un haut-parleur, placé sous une cloche à vide, s’atténue lorsqu’on fait le vide jusqu’à disparaître totalement ; en rétablissant la pression, le son redevient perceptible. En plaçant un microphone à l’intérieur de la cloche, on constaterait la même atténuation. Manomètre

Haut-parleur Support

Cloche a` vide Pompe F IG . 32.2.

GBF

575

Ondes mécaniques dans un milieu continu a) Vitesse du son dans les gaz

Dans les gaz, les déplacements des couches de fluide sont si rapides que le gaz subit des transformations isentropiques (cf. Thermodynamique). On a donc, si le gaz est assimilé à un gaz parfait : re k S =

re m M = = gp gpV gRT

On en déduit la vitesse cs du son dans l’air : cs =

gp re

1/2

soit

cs =

gRT M

1/2

On voit que cs ne dépend pas de la pression et augmente avec la température comme T 1/2. Dans l’air sec à T = 273 K, on trouve, puisque g = 1, 40 : c s ≈ 331, 4 m . s −1, soit 1 193 km . h−1. Pour T = 289 K, c s ≈ 340, 5 m . s−1 .

Ce résultat concorde bien avec celui que l’on obtient expérimentalement. Les premières mesures précises furent effectuées en 1822 par Laplace qui, le premier, eut l’idée d’utiliser dans cette étude le coefficient de compressibilité isentropique 1/(gp) ; avant lui, l’expression de la vitesse de propagation, établie par Newton, ne comportait pas le facteur g, ce qui donnait une valeur théorique différente de la valeur expérimentale, d’environ 20 %. Les mesures consistaient à déterminer, sur une distance de 18, 6 km, la durée qui séparait la perception de la lueur de l’explosion d’un coup de canon de celle du son émis. Les résultats trouvés furent 340, 8 m . s−1 à 289 K et 330 m . s−1 à 273 K. On peut mesurer une telle vitesse en produisant des ondes stationnaires dans un tube. Actuellement, les mesures sont réalisées avec des ultrasons. Remarques : (1) Cette vitesse de propagation dans un gaz est du même ordre de grandeur que la vitesse la plus probable des molécules d’un gaz parfait (cf. Thermodynamique). (2) L’influence du vent sur la mesure de la vitesse du son à l’air libre, bien que faible, n’est pas négligeable. On s’en affranchit en calculant la moyenne des valeurs obtenues dans les deux sens opposés.

 c Dunod – Toute reproduction non autorisée est un délit

b) Vitesse du son dans les liquides Dans les liquides, on peut considérer que r e et k sont des constantes. Pour l’eau, à 281 K, on a : k S = 4, 9 × 10−10 Pa−1

re = 1 000 kg . m−3

cs = 1 428 m . s−1

d’où

La mesure de la vitesse de propagation du son dans l’eau du lac de Genève, faite en 1827, en déterminant la durée mise par le son d’une cloche pour parcourir une distance de 14 km, a donné 1 435 m . s −1. Les mesures actuelles sont faites avec des ultrasons. c) Vitesse du son dans les solides La théorie précédente peut être appliquée aux solides isotropes. Le coefficient de compressibilité s’écrit dans ce cas, où l’on introduit la longueur l = /s du barreau et la tension mécanique (force par unité de surface) f s = −p que l’on exerce sur lui : kS = −

1

∂ ∂p

= S

1 l

∂l ∂f

= S

1 E

576

32. Ondes mécaniques dans un milieu continu

E étant le module d’Young. Dans le cas de l’acier : E = 0, 2 × 10 12 Pa d’où

r e = 8 000 kg . m−3

cs =

E re

1/2

= 5 000 m . s −1

Un ressort en acier peut être aussi le siège d’ondes mécaniques de déformation. Si l’on désigne par r l la masse linéique du fil d’acier, s sa section et K sa raideur, on a, pour ce type d’onde : cs =

E re

1/2

=

sE rl

1/2

=

Kl rl

1/2

puisque

E=l

∂f ∂l

= S

l s

∂F ∂l

= S

lK s

Concrètement, pour K = 60 N . m−1 , l = 1, 5 m et r l = 0, 225 kg . m −1 , la vitesse de propagation de ces ondes mécaniques de déformation vaut cs = 20 m . s−1 . I . 4 . — Propagation d’ondes longitudinales tridimensionnelles dans un milieu homogène a) Cas tridimensionnel À trois dimensions, l’équation différentielle à laquelle satisfait la propagation d’ondes longitudinales, dans un milieu continu homogène, peut être obtenue en généralisant le cas unidimensionnel : ∂ 2 C(r, t) ∂2 C(r, t) ∂ 2C(r, t) ∂2 C(r, t) + + = r k e S ∂x2 ∂y2 ∂z 2 ∂t 2 ce qui s’écrit, de façon condensée : DC(r, t) = re kS

∂ 2C(r, t) ∂t 2

où r représente le vecteur position dans l’espace, de composantes cartésiennes x, y, z. Cette équation admet des solutions monochromatiques planes de la forme : C(r, t) = A exp[−i(vt − k · r)] où k est le vecteur d’onde. Si la source qui produit la perturbation acoustique dans le milieu homogène est ponctuelle, les solutions monochromatiques sont sphériques (cf. Optique) : C (r, t) =

A exp[−i(vt − kr)] r

où r est la norme du vecteur position compté à partir du point où se trouve la source. b) Obtention de l’équation d’onde à partir de l’équation d’Euler On peut retrouver l’équation différentielle des ondes à partir de l’équation d’Euler (cf. chapitre 30) : 1 ∂v + (v · ∇)v = − grad p + g r ∂t Plaçons-nous dans l’hypothèse des petits mouvements du fluide, autour d’une position moyenne, ce qui implique que l’équation d’Euler soit réduite à son approximation linéaire ; précisément, on néglige le terme non linéaire (v · ∇)v.

577

Ondes mécaniques dans un milieu continu

La relation entre la surpression P = p − p e , avec P  p e et la variation de masse volumique ε = r − re , avec ε  r e , est obtenue à l’aide de r d = d m = Cte. En effet, on a alors : dr d + r

= 0 d’où kS = −

L’équation d’Euler devient :

1

∂ ∂p

= S

1 r

∂r ∂p

S



1 ε re P

et donc

ε = kS reP

∂v 1 =− grad(pe + P) + g ∂t (r e + ε)

ce qui donne, en simplifiant, puisque grad p e = r e g à l’équilibre : re

∂v = − grad P ∂t

soit

∂ div(re v) = −DP ∂t

en prenant la divergence des deux membres de l’équation. Comme div(re v) = −∂r/∂t = −∂ε/∂t, d’après l’équation-bilan de la masse, il vient : ∂2 ε = DP ∂t2

d’où

DP = kSre

∂ 2P ∂t 2

et donc

DC = kSre

∂2 C ∂t2

en remplaçant ε par kS re P et en utilisant la relation P = −(1/kS )(∂ C/∂x).

II . — ASPECT ÉNERGÉTIQUE La propagation de l’énergie d’une onde acoustique le long d’un milieu continu unidimensionnel peut être étudiée de la même façon que pour une corde (cf. Optique). II . 1 . — Énergie cinétique massique d’un élément

 c Dunod – Toute reproduction non autorisée est un délit

Considérons la propagation d’une onde acoustique dans un milieu continu, de masse volumique r, le long d’un axe Ox et de section droite S (Fig. 32.1). L’énergie cinétique massique d’un élément du milieu, compris entre les abscisses x et x + d x, a pour expression : ek =

1 2

∂C ∂t

2

d’où x

d ek = d t

∂C ∂t

x

∂2 C ∂t2

x

la variation de ek au cours de la durée élémentaire d t. II . 2 . — Énergie massique reçue par un élément de milieu Évaluons la puissance élémentaire reçue dP par l’élément de milieu continu, de longueur d x , par l’intermédiaire des forces de pression latérales : dP = S [p(x) − p e ] S dP = − kS

∂ C(x, t) ∂t

x

S ∂C ∂C + ∂x ∂t x,t kS

puisque p = −(1/kS)(∂ C/∂x).

− S[p(x + d x) − p e] ∂C ∂C ∂x ∂t

x+d x,t

∂ C(x + d x, t) ∂t

x+d x

578

32. Ondes mécaniques dans un milieu continu Comme C est une fonction de (t − x/c s), ∂C/∂x = −(1/c s)∂ C/∂t. Il en résulte que : S dP = − k Sc s

soit dP = 2S d x

1 k Sc s

∂C ∂t ∂C ∂t

x

2

∂C − ∂t x,t ×

∂ ∂x

∂C ∂t

2

S dx ∂ =− kS cs ∂x

x+d x,t

x

= −2S d x

1 k S c2s

∂C ∂t

∂C ∂t

x

2

x,t

∂2C ∂t 2

x

kS c2s

puisque (∂C/∂t)x est aussi une fonction de (t − x/cs). En remplaçant par r, on trouve que la puissance massique pm reçue par l’élément, à travers les forces de pression qui s’exercent latéralement, a pour expression : dP ∂C =2 pm = rS d x ∂t

∂2 C ∂t 2

x

x

Finalement, en multipliant par la durée élémentaire d t, on obtient un travail massique reçu égal au double de la variation de l’énergie cinétique massique : pm d t = 2 d ek II . 3 . — Énergie potentielle d’un élément de milieu L’expression de l’énergie potentielle massique, due au travail des forces de pression, peut être déduite du bilan d’énergie mécanique massique, effectué sur l’élément de milieu continu d’épaisseur d x, entre deux instants voisins t et t + d t : d(e k + ep) = p m d t d’où d(ek + ep) = 2 d ek puisqu’on a supposé qu’aucune énergie mécanique n’était perdue par frottement. Il en résulte que d ek = d ep soit, en choisissant convenablement l’origine de l’énergie potentielle : e k = ep Retenons donc que l’énergie cinétique massique et l’énergie potentielle massique de la tranche de gaz sont égales à chaque instant. II . 4 . — Intensité acoustique Calculons la puissance mécanique moyenne reçue par l’élément de milieu d x de la part de l’élément voisin situé à sa gauche, si la propagation a lieu dans le sens gauche-droite. On a : P = SP

∂C ∂t

=−

S kS

∂C ∂x

∂C ∂C S = k Scs ∂t ∂t

2 x

= rSc s

∂C ∂t

2

>0 x

puisque P = −(1/kS)∂ C/∂x et ∂ C/∂x = −(1/cs)∂ C/∂t. Ainsi, le transfert de puissance et donc d’énergie par unité de temps, qui accompagne la propagation d’une onde le long du tuyau, se fait dans le sens de la propagation. Dans le cas d’une onde monochromatique, pour laquelle le déplacement s’écrit C = A cos(vt − f), cette puissance vaut, à l’instant t : P = rS c s A2 v2 sin 2(vt − f)

Calculons sa valeur moyenne sur une durée de détection Td grande devant la période T = 2p/v du phénomène. Il vient : 1 P= Td

Td 0

rSc sA2 v2 Pdt = Td

Td 0

sin 2 (vt − f) d t

579

Ondes mécaniques dans un milieu continu Comme l’intégrale vaut Td/2 , on obtient :

rSA2v2 cs 2 Cette puissance acoustique, qui est l’énergie mécanique moyenne transportée par l’onde, pendant l’unité de temps, à travers la surface S, est donc proportionnelle au carré de l’amplitude réelle, au carré de la fréquence f = v/(2p) et au produit rcs. En divisant par S , on obtient la puissance acoustique par unité de surface ou intensité acoustique I : P=

I=

rA 2v2 cs 2

Cette quantité, que l’on exprime en W . m−2 , est un courant volumique d’énergie acoustique. Les grandes puissances acoustiques sont obtenues dans un liquide avec des ultrasons. Ordres de grandeur : i) Pour un son de fréquence f = 1 000 Hz, d’amplitude 100 mm , se propageant dans l’air à la vitesse cs, l’intensité acoustique vaut : I=

1, 293 × 10 −8 × 4p2 × 106 rA2 v2 × 340 = 86, 5 W . m −2 cs = 2 2

Si la source acoustique a une surface de 0, 04 m2, la puissance acoustique est de 3, 46 W, ce qui est suffisant pour l’audition dans une grande salle. ii) Pour des ultrasons, de fréquence f = 100 kHz, d’amplitude 0, 1 mm , se propageant dans l’eau, I vaut : 1 000 × 10−14 × 4p2 × 10 10 rA2 v2 × 1 400 = 2 763 W . m−2 I= cs = 2 2 II . 5 . — Relation entre la surpression acoustique et l’intensité acoustique Dans le cas d’une onde monochromatique plane, on a vu que l’amplitude de la surpression et l’intensité acoustique avaient pour expressions respectives :

 c Dunod – Toute reproduction non autorisée est un délit

Pm = rAvcs

et I =

rA2 v 2 cs 2

On en déduit I = P2m /(pc √ s) et la relation suivante entre l’intensité I et l’amplitude efficace de la surpression Pef = Pm/ 2 et I : I=

P2ef rc s

II . 6 . — Pression de radiation La pression de radiation est la pression exercée par une onde acoustique sur un élément de surface normal à la direction de propagation. On sait que la pression est apparue dans l’équation de Bernoulli comme une énergie par unité de volume. Par conséquent, à l’énergie acoustique volumique rA 2v 2/2 correspond la pression de radiation prad suivante : prad =

rA2 v2 I P = = ef2 2 cs rcs

580

32. Ondes mécaniques dans un milieu continu

Ordre de grandeur : Dans l’exemple précédent sur les ultrasons dans l’eau, la pression de radiation vaut prad = I /cs = 2 Pa, ce qui est faible mais peut être mis en évidence expérimentalement à l’aide d’une balance de torsion. II . 7 . — Atténuation du son. Loi de Beer L’analyse énergétique précédente a montré que la propagation du son produisait un transfert d’énergie. Ce transfert, qui est proportionnel au carré de la fréquence, s’accompagne nécessairement d’une atténuation due à la viscosité du fluide. Cette atténuation de l’énergie est proportionnelle au carré de la vitesse de déplacement du fluide (∂C/∂t)2 et donc au carré de la fréquence ; elle est plus forte pour les grandes fréquences. Il en résulte que les sons graves portent plus loin que les sons aigus, ce qui explique la faible hauteur du son émis par les sirènes d’alerte à grande distance. L’atténuation de l’intensité acoustique I satisfait à une loi de Beer, analogue à celle de l’optique (cf. Optique). À la traversée d’une épaisseur élémentaire d x de fluide, l’opposé de la variation d I est proportionnelle à I et à d x. Si on introduit le coefficient d’absorption acoustique m, on a : d I = −mI d x En intégrant et en tenant compte de la valeur de I en x = 0, on obtient : I = I (0) exp(−mx)

III . — RÉFLEXION ET TRANSMISSION DES ONDES MÉCANIQUES Jusqu’à maintenant, nous avons considéré une onde mécanique se propageant dans un milieu infini. Lorsqu’une telle onde rencontre un obstacle ou une surface de séparation entre deux milieux différents, elle est généralement partiellement réfléchie et partiellement transmise. Nous nous proposons de calculer les facteurs de réflexion et de transmission en amplitude et en énergie correspondants. III . 1 . — Réflexion et transmission à la surface de séparation de deux milieux fluides infinis Considérons une onde monochromatique plane se propageant le long d’un axe Ox, dans le sens des x croissants. En O, l’onde rencontre la surface séparant deux fluides, ce qui provoque une réflexion partielle de l’onde (Fig. 32.3).

x

O Milieu 1

Milieu 2 S F IG . 32.3.

Les conditions aux limites dans ce cas sont au nombre de deux. D’une part les déplacements de particules de fluides sont identiques de part et d’autre de la surface de séparation S, située à l’origine des coordonnées. D’autre part, puisque la masse de cet élément peut être rendue infiniment faible, la force de pression totale qui s’exerce sur un élément de volume cylindrique, dont les surfaces de base

581

Ondes mécaniques dans un milieu continu

sont situées de part et d’autre de S, est nulle. Il en résulte l’égalité des pressions acoustiques. On a donc : 1 ∂C1 1 ∂C2 = C 1(0, t) = C 2 (0, t) et p1(0, t) = p 2 (0, t) soit k1 ∂x x=0 k2 ∂x x=0 Dans le milieu 1 (x < 0), le déplacement C(x, t) de la particule de fluide s’écrit, en notation complexe : C1(x, t) = A1 exp[−i(vt − k 1x)] + B1 exp[−i(vt + k 1x)]

Le premier terme représentant l’onde acoustique incidente se propageant à la vitesse acoustique c1 = v/k1 , dans la direction des x croissants, avec la pulsation v et le vecteur d’onde k1, alors que le second représente l’onde réfléchie de même pulsation v et de même nombre d’onde k1 . Il est alors naturel d’introduire le facteur de réflexion en amplitude complexe : r1 = B1 /A1 . Dans le milieu 2 (x > 0), C(x, t) s’écrit aussi : C2(x, t) = A2 exp[−i(vt − k 2x)] + B2 exp[−i(vt + k 2x)] où le premier terme représente l’onde acoustique transmise se propageant à la vitesse acoustique c2 = v/k2 , dans la direction des x croissants, avec la pulsation v et le vecteur d’onde k 2. Le second n’a aucune signification si aucun obstacle physique n’est présent sur l’axe au-delà de x = 0. On a donc B2 = 0 et on pose A 2 = t 2A1 où t 2 représente le facteur de transmission. Il en résulte que : C 1(x, t) = A 1 [exp(ik1x) + r 1 exp(−ik 1x)] exp(−ivt)

et C 2(x, t) = A1t2 exp(ik2 x) exp(−ivt)

Les conditions aux limites en x = 0 donnent respectivement : A1(1 + r1) = A 1t2

et iA1

k1 k2 (1 − r1) = iA1 t 2 k1 k2

soit :

r2 c2 k2 k1 c1 k1 t2 = t2 = t2 r1 c1 k1 k2 c2 k2 puisque k2 /k1 = c1/c 2 , rc1 = 1/(k 1c1) et rc2 = 1/(k2c 2). La résolution de ce système d’équations algébriques est aisée ; on trouve : 1 + r 1 = t2

 c Dunod – Toute reproduction non autorisée est un délit

r1 =

et 1 − r1 =

r1 c1 − r2 c2 r1 c1 + r2 c2

et

t2 =

2r1 c1 r 1 c1 + r2 c2

On en déduit les facteurs de réflexion et de transmission en puissance : R = |r 1|2 =

(r1 c1 − r2c2 )2 (r1 c1 + r2c2 )2

et

T=

4r1c 1 r2 c2 r2c 2 |t2| 2 = r1c 1 (r1 c1 + r2c2 ) 2

Notons que :

4r21c21 r2c2 mais T = |t2 |2 2 r1c1 (r1 c1 + r2c 2) car l’intensité acoustique est proportionnelle au carré de l’amplitude et au produit rcs. On a évidemment, en l’absence d’absorption, R + T = 1. T=  |t 2| 2 =

Remarque : Les formules précédentes rappellent celles que l’on établit pour les ondes lumineuses (cf. Optique). Nous donnerons plus loin d’autres expressions de ces formules en fonction des impédances acoustiques.

582

32. Ondes mécaniques dans un milieu continu Exemples : i) Si le milieu incident est de l’air et le milieu émergent de l’eau, on a : r2c 2 ≈ 1 000 × 1450 ≈ 1, 45 × 106 SI

r 1c1 ≈ 1, 3 × 340 ≈ 442 SI

d’où

r1 ≈ −1

et

t2 ≈ 0

Ainsi, un son émis dans l’air n’est pratiquement pas perçu dans l’eau. Il est réfléchi par la surface de séparation en subissant un déphasage de p. De même, une explosion violente dans l’eau ne provoque que très peu de perturbation dans l’air. ii) Si le milieu incident est de l’air et le milieu émergent de l’hydrogène : r 1c1 ≈ 1, 3 × 340 ≈ 442 SI puisque : c2 =

gRT M

On en déduit : r1 ≈

1/2

=

r 2 c2 ≈

2 × 10−3 × 1260 = 113 SI 22, 4 × 10−3

1, 4 × 8, 314 × 273 0, 002

1/2

= 1 260 m . s−1

442 − 113 2 × 442 ≈ 0, 6 et t 2 ≈ ≈ 1, 6 442 + 113 442 + 113

III . 2 . — Réflexion sur une paroi rigide Considérons une onde se propageant selon l’axe Ox dans le sens des x croissants et rencontrant à la distance L de l’origine O une paroi rigide fixe (Fig. 32.4a). Les particules de fluide en contact avec cette paroi sont donc au repos. En raison de cet obstacle, se superposent l’onde incidente et une onde réfléchie : C(x, t) = Ci (x, t) + Cr (x, t) = Ai exp[−i(vt − kx)] + Ar exp{−i[vt − k(2L − x)]}

puisque l’onde réfléchie qui atteint le point A a parcouru la distance 2L − x. On peut mettre C(x, t) sous une autre forme, en introduisant notamment le facteur de réflexion r 1 : C(x, t) = A i exp[−i(vt − kL)] × exp[ik(x − L)] + r1 exp[−ik(x − L)] a) Onde stationnaire Nous avons vu que le facteur de réflexion r 1 valait approximativement −1 si le milieu 1 était un gaz et le milieu 2 un liquide ou un solide. Il en résulte que C(x, t) a pour expression : C(x, t) = Ai exp[−i(vt − kL)] × exp[ik(x − L)] − exp[−ik(x − L)]

= 2iAi sin[k(x − L)] exp[−i(vt − kL)] = 2A i sin[k(L − x)] exp −i vt − kL +

p 2

On voit que les dépendances spatiale et temporelle sont séparées : il n’y a pas de propagation mais seulement une vibration dont l’amplitude dépend de la position du point considéré. On dit que C(x, t) représente une onde stationnaire. b) Nœuds et ventres de déplacement D’après ce qui précède, le déplacement des particules de fluide est nul si la condition suivante est réalisée : 2p l k(L − x) = m p soit (L − x) = m p et x = L−m 2 l m étant un entier. Ces points définissent des nœuds de déplacement. Notons que la distance qui sépare deux nœuds de déplacement consécutifs vaut : Dx = l/2.

583

Ondes mécaniques dans un milieu continu

Les nœuds de déplacement sont séparés par des points en lesquels le déplacement est maximal ; on dit que ces points sont des ventres de déplacement. c) Nœuds et ventres de pression Comme la surpression P(x, t) = p(x, t) − p e est reliée au déplacement C(x, t) par une relation de dérivation, il vient : P(x, t) = −

1 kS

∂C ∂x

t

= 2kA i cos[k(L − x)] exp −i vt − kL +

p 2

Les nœuds de pression se produisent aux points tels que : k(L − x) = (2m + 1)

p 2

soit

x = L− m+

1 2

l 2

Ils correspondent donc aux ventres de déplacement. De la même façon, les ventres de pression coïncident avec les nœuds de déplacement. Paroi rigide fixe O

L A(x)

Paroi rigide mobile

Paroi rigide fixe O

x

A(x)

x

L a)

b) F IG . 32.4.

d) Résonance d’ondes stationnaires On impose une nouvelle condition, en plaçant une paroi mobile au point O ; ce dernier devient un ventre de déplacement (Fig. 32.4b). Il en résulte :

 c Dunod – Toute reproduction non autorisée est un délit

L− m+

1 2

l = 0 soit L = 2

m+

1 2

l 2

ou

f=

1 cs m+ 2L 2

en introduisant la fréquence f = v/(2p) = c s/l. Ainsi, en raison des conditions aux limites, une sélection des fréquences de vibration des ondes stationnaires s’opère ; on dit qu’il y a résonance pour certains modes propres de vibration. III . 3 . — Réflexion sur une extrémité ouverte Supposons que le point d’abscisse L ne soit pas astreint à un déplacement nul, comme dans un tuyau fermé, mais à une surpression nulle car en contact avec l’air libre. Là aussi, on interprète bien les phénomènes acoustiques en admettant la superposition de deux ondes dans le tuyau, l’onde incidente et une onde réfléchie : C(x, t) = Ci (x, t) + Cr (x, t) = Ai exp[−i(vt − kx)] + Ar exp{−i[vt − k(2L − x)]} = A i exp[−i(vt − kL)] × exp[ik(x − L)] + r1 exp[−ik(x − L)]

584

32. Ondes mécaniques dans un milieu continu

a) Onde stationnaire Le facteur de réflexion à cette extrémité ouverte vaut 1, puisque les deux milieux sont identiques. Il en résulte que C(x, t) a pour expression : C(x, t) = A i exp[−i(vt − kL)] × exp[ik(x − L)] + exp[−ik(x − L)] = 2Ai cos[k(x − L)] exp[−i(vt − kL)]

Là aussi, C(x, t) représente une onde stationnaire. b) Nœuds et ventres

Le déplacement des particules de fluide est nul (nœuds de déplacement) pour : k(L − x) = (2m + 1)

p 2

soit

2p p (L − x) = (2m + 1) 2 l

et

x = L − (2m + 1)

l 4

La distance qui sépare deux nœuds de déplacement consécutifs est : Dx = l/2. Ces nœuds de déplacement sont séparés par des ventres pour lesquels x = L − ml/2. Comme la surpression P(x, t) s’obtient en dérivant le déplacement C(x, t) par rapport à x, les nœuds de pression coïncident avec les ventres de déplacement et les ventres de pression avec les nœuds de déplacement. c) Résonance d’ondes stationnaires Comme précédemment, imposons en outre au point O d’être un ventre de déplacement. Cette nouvelle condition se traduit par : L−m

l l = 0 soit L = m 2 2

ou

f =m

cs 2L

Là aussi, en raison des conditions aux limites, une sélection des fréquences de vibration des ondes stationnaires s’opère ; il y a résonance pour certains modes propres de vibration. III . 4 . — Impédance acoustique caractéristique Rappelons que nous avons défini l’impédance offerte par un système mécanique oscillant, soumis à une force sinusoïdale, par le rapport de l’amplitude complexe de la force excitatrice sur l’amplitude complexe de la vitesse (cf. chapitre 11) : F Z= m Vm De façon analogue, on définit l’impédance d’un dipôle électrique par le rapport de l’amplitude complexe de la tension à ses bornes sur l’amplitude complexe de l’intensité qui le traverse : Z=

Vm Im

Ce concept d’impédance se généralise au cas des ondes acoustiques. a) Définition Considérons une onde acoustique monochromatique plane se propageant selon la direction Ox. Une tranche du milieu vibre avec une élongation : C (x, t) = A exp[−i(vt − kx)]

585

Ondes mécaniques dans un milieu continu

La vitesse u et la surpression P associées ont pour expressions complexes respectives : 1 dV 1 ∂C ∂C ik u= = −ivC et P = − =− =− C kS V kS ∂x kS ∂t On appelle impédance acoustique caractéristique du milieu, relativement à la propagation d’ondes acoustiques monochromatiques planes, le rapport de la surpression complexe sur la vitesse complexe : Zc =

P k = vkS u

d’où

Zc =

1 = rcs c sk S

Comme kS est homogène à l’inverse d’une pression, les dimensions de Z c sont celles d’une impédance mécanique par unité de surface ; on l’exprime en Pa . s . m −1 dans SI. On introduit aussi l’impédance acoustique Z a, rapport de l’impédance acoustique caractéristique Z a par la surface S sur laquelle s’exerce la surpression P : Za =

Zc S

Ce concept est très utile dans l’étude des tuyaux sonores. Remarques : (1) L’impédance acoustique caractéristique Zc = rc s représente la masse du milieu continu atteinte par l’onde pendant l’unité de temps et par unité de surface. En effet : Srcs d t dm Zc = rcs = = S dt S dt

(2) L’admittance accoustique caractéristique du milieu est l’inverse de l’impédance : Yc = u/r = 1/Zc .

b) Ordres de grandeur i) Pour l’air, à 273 K, Z c = rc s = 1, 293 × 331, 4 = 428 Pa . s . m−1 . Lorsque la température change, Zc varie comme T−1/2 puisque r = p0/(RT ) est proportionnelle à 1/T et cs à T 1/2 . ii) Pour l’eau, à 281 K, Z c = rc s = 1 000 × 1 428 = 1, 428 × 10 6 Pa . s . m−1

 c Dunod – Toute reproduction non autorisée est un délit

iii) Pour un muscle, à 310 K, Z c = rc s = 1 040 × 1 580 = 1, 64 × 106 Pa . s . m −1. iv) Pour un os, à 310 K, Z c = rcs = 1 900 × 4 000 = 7, 6 × 106 Pa . s . m−1 .

v) Pour l’acier, à température ambiante, Z c = rcs = 8 000 × 5 000 = 40 × 106 Pa . s . m−1. c) Autre écriture des facteurs de réflexion et de transmission On exprime souvent, en fonction des impédances acoustiques caractéristiques, les facteurs de réflexion et de transmission d’une onde franchissant la surface qui sépare deux milieux différents : r1 =

Z c,1 − Zc,2 Z c,1 + Zc,2

et

t2 =

2Zc,1 Z c,1 + Zc,2

d’où les facteurs de réflexion et de transmission en intensité : R = |r1| 2 =

(Z c,1 − Zc,2 ) 2 (Z c,1 + Zc,2 ) 2

et

T=

4Z c,1Zc,2 Zc,2 |t 2|2 = Zc,1 (Zc,1 + Zc,2) 2

Notons que r1 = 0 et donc R = 0 si Z c,2 = Zc,1 : il n’y a pas dans ce cas d’onde réfléchie ; on dit qu’il y a adaptation d’impédance.

586

32. Ondes mécaniques dans un milieu continu

Exemple : Justifions l’intérêt du stéthoscope (en grec stethos signifie poitrine), inventé par le médecin français R. Laennec au début du XIXe siècle, pour détecter les ondes sonores internes (battements de cœur, etc.). La comparaison des valeurs de facteur T, obtenues avec le couple muscle-air et le couple muscle-stéthoscope, donne : Tm,a =

4 × 1, 64 × 106 × 428 ≈ 2, 4 × 10 −6 (1, 64 × 106 + 428) 2

et Tm,s =

4 × 1, 64 × 106 × 1, 5 × 106 ≈ 0, 998 (1, 64 × 10 6 + 1, 5 × 106 )2

car, dans le cas du stéthoscope, Z c = 1, 5 × 10 6 Pa . s . m−1 . III . 5 . — Application des ondes stationnaires à la mesure des vitesses de propagation a) Ondes stationnaires dans un tube Un haut-parleur, placé à l’extrémité d’un tube en verre, d’environ 1, 5 m de long, produit dans ce tube des ondes stationnaires dont on peut détecter les nœuds de pression à l’aide d’un microphone de petite taille (Fig. 32.5). On néglige l’influence de la dimension transversale du tuyau, ce qui revient à admettre que son diamètre D est très grand devant la longueur d’onde l des ondes acoustiques. En déplaçant le micro le long de l’axe, on mesure la distance séparant deux maxima de pression ; on en déduit la longueur d’onde l et la vitesse de propagation cs . Par exemple, avec une onde de fréquence f = 1, 7 kHz, on a mesuré une distance qui sépare six maxima de pression égale à 50 cm. On a donc : 5×

l = 0, 5 m d’où 2

l = 0, 2 m et

cs = l × f = 340 m . s −1

Tube en verre l

GBF

Tige Haut-parleur

Micro

O

F IG . 32.5.

F IG . 32.6.

b) Tube de Kundt Dans un tube analogue au précédent, on réalise des ondes stationnaires de l’air en faisant vibrer un piston placé à l’extrémité d’une tige métallique. On crée dans cette dernière, fixée en un point O, des vibrations longitudinales par frottement (Fig. 32.6). Le tube, fermé à l’autre extrémité, contient une poudre fine (lycopode) qui, en raison des ondes stationnaires, se rassemble en tas aux nœuds de déplacement ; en effet, la poudre n’est dispersée qu’aux ventres de déplacement. La détermination de la longueur d’onde la de ces ondes dans l’air permet de trouver la fréquence f de la vibration, connaissant (a) la vitesse de propagation cs du son dans l’air. Or la tige, de longueur l, est aussi le siège d’ondes stationnaires avec un nœud en son milieu O. Par conséquent, si ls désigne la longueur d’onde dans la tige et c(s t) la vitesse de propagation du son dans la tige, on a : l=

ls 2

d’où

c (s t) = l s × f = c(s a)

2l ls = c (s a) la la

Cette expérience fut réalisée pour la première fois par le physicien allemand du XIX e siècle A. Kundt.

587

Ondes mécaniques dans un milieu continu

IV . — ACOUSTIQUE PHYSIOLOGIQUE L’acoustique physiologique prend en compte la détection des ondes acoustiques par l’oreille. IV . 1 . — Loi de Fechner L’expérience montre que la sensation sonore est sensiblement proportionnelle au logarithme de l’intensité acoustique I de l’onde. Ce résultat constitue la loi de Fechner énoncée par le physiologiste et philosophe allemand G. Fechner. En général, on préfère considérer la quantité suivante sans dimension, appelée niveau d’intensité acoustique , que l’on exprime en décibel dB (du nom du physicien américain A. Bell) : L I = 10 lg

I I0



I0 = 10 −12 W . m −2

est une intensité de référence correspondant au seuil d’audibilité par l’oreille humaine d’un son sinusoïdal de fréquence f = 1 kHz. Notons que le logarithme est décimal et que le seuil d’audibilité correspond à une surpression efficace qui vaut dans l’air : Ps = (I 0Zc )1/2 = (I 0 rcs)1/2 ≈ 20 × 10−6 Pa = 20 mPa Un tel seuil correspond à une amplitude de vibration de l’ordre de 10 pm seulement : A=

2I0 rv2 cs

1/2

=

2 × 10−12 1, 293 × 4p 2 × 105 × 340

1/2

≈ 10 pm

IV . 2 . — Diagramme de Fletcher La sensibilité de l’oreille dépend donc fortement de la fréquence f . On constate en effet, par exemple à l’aide d’un générateur basse fréquence, qu’un signal de 2 kHz produit un sifflement sonore, alors qu’un signal de même amplitude, mais de fréquence 50 Hz, est inaudible.

 c Dunod – Toute reproduction non autorisée est un délit

En outre, en faisant varier l’intensité du signal de fréquence déterminée, la sensation sonore passe d’un seuil de perception où l’on entend à peine le signal à un seuil de douleur insupportable. À 1 kHz, le seuil d’audibilité est pris conventionnellement égal à 0 dB. Les deux courbes donnant 10 lg(I /I 0), en fonction de la fréquence f , au seuil de perception et au seuil de douleur, définissent l’aire audible (Fig. 32.7). On voit que la zone très sensible de l’oreille se situe entre 500 Hz et 5 000 Hz, et que le maximum de sensibilité se trouve à environ 3 000 Hz. L’écart entre les deux seuils atteint 120 dB dans la zone sensible et n’est plus que de 70 dB aux fréquences extrêmes 20 Hz et 20 kHz. Le tracé du réseau de courbes relatif à des sensations sonores intermédiaires, entre les deux seuils, forme le diagramme de Fletcher, du nom du physiologiste H. Fletcher. Pour définir une sensation auditive intermédiaire, on utilise un son de référence à 1 kHz dont on compare la sensation auditive à celle produite par le signal considéré ; on fait alors varier l’intensité de ce dernier afin de parvenir à l’égalité des sensations auditives. Si le niveau d’intensité est mesuré en dB à 1 kHz, la sensation auditive s’exprime en phone. Ce procédé permet de comparer des sensations sonores caractéristiques : ainsi, alors que le calme à la campagne correspond à 20 dB, une voiture a un niveau sonore de 50 dB, une rue bruyante de 70 dB ; une moto en accélération a un niveau sonore de 90 dB, comme un concert de rock à une distance de 500 m, alors que celui d’un avion dans un aéroport est de 120 dB !

588

32. Ondes mécaniques dans un milieu continu

10 lg

I I0 Seuil de douleur

120 100 80 60

Aire audible

40 20 0

Seuil d'audibilité 0,05

0,5

10

f (kHz)

F IG . 32.7.

IV . 3 . — Hauteur et timbre des sons Un son nous paraît d’autant plus aigu que sa fréquence est élevée. On appelle hauteur d’un son la perception sonore de sa fréquence, laquelle est déterminée par les vibrations des molécules d’air ; on explique ainsi que, si l’on remplace l’air par l’hélium plus léger, les sont émis soient plus aigus. Dans l’air, cette hauteur n’est perceptible que pour une fréquence comprise entre 20 Hz et 20 kHz. Des sources sonores de natures différentes émettent des sons, de même intensité et de même hauteur, qui diffèrent généralement par la présence d’ondes sinusoïdales dites harmoniques ; ces dernières, de fréquences égales à un multiple entier de f 0, accompagnent le son fondamental de fréquence f 0. Le timbre est la sensation sonore que donnent ces harmoniques.

V . — PROPRIÉTÉS ONDULATOIRES DES ONDES ACOUSTIQUES On peut mettre en évidence, avec les ondes acoustiques, tous les phénomènes caractéristiques des ondes : diffraction, interférence (cf. Optique), effet Doppler (cf. Relativité et invariance), groupes d’ondes (cf. Électromagnétisme), etc. V . 1 . — Diffraction Comme en optique, on appelle diffraction d’une onde acoustique le phénomène d’éparpillement de l’énergie acoustique que l’on observe lorsqu’un diaphragme limite l’étendue de cette onde. Le point de départ de l’analyse est le principe d’Huygens-Fresnel (cf. Optique). La diffraction est importante si la longueur d’onde l n’est pas négligeable devant la taille D caractéristique du diaphragme. Donnons simplement un résultat utile : dans l’approximation de Fraunhofer de la diffraction, c’est-à-dire à grande distance du diaphragme (cf. Optique), la majeure partie de l’énergie acoustique, diffractée par un diaphragme circulaire de diamètre D, se trouve localisée dans un cône

589

Ondes mécaniques dans un milieu continu d’angle au sommet (Fig. 32.8a) : u ≈ 1, 22

l D

Si la longueur d’onde est négligeable devant le diamètre du diaphragme, on se trouve dans l’approximation de l’acoustique géométrique (cf. Optique). Le rayon acoustique est alors donné par l’équivalent du principe de Fermat que l’on écrit sous la forme suivante, dt = 0 avec t =

ds cs

d s étant l’élément différentielle curviligne et cs la vitesse du son dans le milieu considéré. Autrement dit, la durée t du trajet suivi par l’onde acoustique est stationnaire le long du rayon acoustique. L’inverse de la vitesse de phase c s peut donc jouer le rôle d’indice en acoustique. Haut-parleur

GBF S

Haut-parleur

A1

A2

θ

Déplacement de la coulisse

P

Oscilloscope

Microphone a)

b) F IG . 32.8.

 c Dunod – Toute reproduction non autorisée est un délit

V . 2 . — Interférence de deux ondes Lorsqu’on superpose deux ondes monochromatiques, isochrones, issues d’une même source primaire, qui ont parcouru des chemins différents avec des durées différentes, on constate que la sensation sonore résultant de cette superposition peut varier entre deux niveaux sonores suivant la différence des durées de propagation. Le niveau le plus faible devient nul lorsque les ondes sont de même amplitude. On met en évidence cette interférence avec le trombone à coulisse de Kœnig (Fig. 32.8b). Cet appareil est constitué de deux tubes SA1P et SA 2P dans lesquels le son, issu de la source S (diapason ou haut-parleur), se propage avant d’atteindre le point où se trouve le détecteur P (oreille ou microphone). En déplaçant la coulisse, on constate des maxima et des minima ; lorsqu’on déplace la coulisse de d, on observe m minima distants de l tel que : l = 2d = (m − 1)l

d’où

l=

cs 2d = f m−1

et c s = lf =

Par exemple, si d = 36 cm et m = 5, on en déduit l = 18 cm.

2d f m−1

V . 3 . — Effet Doppler en mécanique L’effet Doppler, du nom du physicien autrichien C. Doppler, est la modification de la fréquence d’une onde acoustique, si cette onde est reçue par un récepteur en mouvement, ou si elle est émise par une source en mouvement.

590

32. Ondes mécaniques dans un milieu continu

a) Effet Doppler avec la source immobile et le récepteur mobile Considérons une source située en O et émettant une onde acoustique monochromatique plane de pulsation v et de vecteur d’onde k, dans le référentiel R = Oxyz (Fig. 32.9). Un récepteur, lié au référentiel R en translation rectiligne uniforme par rapport à R, se déplace à la vitesse v e selon l’axe Ox. Comme les coordonnées d’un point A se transforment selon les formules de transformation de Galilée, x = x + ve t , y = y  , z = z  , la phase de l’onde sinusoïdale s’écrit : vt − k · r = vt − k cos u x − k sin u y = (v − kve cos u) t − k cos u x  − k sin u y ve

y

y



R

R Source immobile

Récepteur mobile θ

O

x

O

x

F IG . 32.9.

La phase est invariante par changement de référentiel puisqu’elle permet d’interpréter les phénomènes d’interférence. En l’identifiant à la phase dans R , d’expression (v t − k  cos u  x − k  sin u  y ), on trouve : k  cos u = k cos u k  sin u  = k sin u v = v − kv e cos u On en déduit k = k et u = u . Comme k = v/c s, il en résulte, en indiçant par r et s ce qui est relatif au récepteur et à la source et en introduisant les fréquences correspondantes f r et fs : fr = fs 1 −

v r cos ur cs

On voit que seule la vitesse radiale du récepteur, vr cos u r , suivant le sens de propagation de l’onde, importe. i) Si le récepteur s’éloigne (cos u r > 0), f r < fs ; ii) s’il se rapproche (cos u r < 0), f r > f s . Exemple : Si v r = 20 m.s −1 et u r = 0, on trouve, dans l’air : fr − fs vr 20 1 =− =− =− 340 17 fs cs Notons que la longueur d’onde de l’effet Doppler en mécanique est invariante par changement de référentiel, puisque ks = kr : 2p 2p = ls lr

donne

ls = lr

ce qui est conforme à

fs fr = cs cs − v r cos ur

issu de la relation principale. Ce résultat diffère essentiellement de l’effet Doppler-Fizeau établi dans le cas de la lumière en raison de l’invariance de la vitesse de la lumière dans le vide (cf. Relativité et invariance).

591

Ondes mécaniques dans un milieu continu b) Effet Doppler avec la source mobile et le récepteur immobile

En procédant comme précédemment dans la configuration représentée sur la figure 32.10a, la phase de l’onde sinusoïdale reçue par le récepteur a pour expression : vt − k · r = vt − k cos u x − k sin u y = (v − kve cos u) t − k cos u x  − k sin u y

En identifiant cette expression avec celle de la phase de l’onde dans le référentiel R lié à la source, (v t − k  cos u x  − k sin u y ), on trouve, k = k , u = u. Comme k = v/cs , on en déduit : ve cos u v = v − kv e cos u = v 1 − cs On a donc, avec des notations explicites : fr =

fs 1 − v s cos us /cs

On voit ici aussi que seule la vitesse radiale de la source, v s cos u s , suivant le sens de propagation de l’onde, importe : i) si la source se rapproche (cos u s > 0), f r > f s, ii) si elle s’éloigne (cos u s < 0), f r < f s. Exemple : le son émis par un véhicule est perçu plus aigu lorsque la source se rapproche et plus grave lorsqu’elle s’éloigne (Fig. 32.10b). Ici aussi, que la longueur d’onde est invariante : l r = ls .

ve

y

y

R

Source mobile O



fr

R

fs

Récepteur immobile 

θ x

x

O

 c Dunod – Toute reproduction non autorisée est un délit

O a)

x

b) F IG . 32.10.

c) Effet Doppler avec la source mobile et le récepteur mobile La combinaison des deux formules précédentes permet d’établir une formule générale. En passant par le référentiel R lié au milieu, l’air par exemple, dans lequel un son de fréquence fm se propage, on obtient, en désignant par vs cos us et vr cos ur les vitesses radiales, suivant le sens de propagation de l’onde, de la source et du récepteur par rapport à R : d’où :

fr = f m 1 −

v r cos ur cs fr = f s

et f m =

fs 1 − vs cos us/c s

1 − v r cos u r/cs 1 − v s cos u s/cs

592

32. Ondes mécaniques dans un milieu continu

d) Application à la vélocimétrie à ultrasons Une application importante de l’effet Doppler acoustique est fournie par le vélocimètre à ultrasons ; ce dernier est constitué d’une sonde comportant une source S d’ultrasons et un détecteur D , tous deux fonctionnant par effet piézoélectrique. La source envoie une onde sur l’objet en mouvement dont on veut déterminer la vitesse v m ; ce dernier réfléchit cette onde vers le détecteur. La comparaison de la fréquence d’émission fe et de la fréquence de réception f d permet d’obtenir v m . Entre f e et la fréquence fm de l’onde atteignant l’objet, on a la relation suivante, puisque la source est fixe : vm f m = fe 1 − cs Entre fm et la fréquence fd de l’onde atteignant le détecteur, on a, la source étant mobile et la vitesse radiale selon la direction de propagation égale à −vm : 1 1 − v m/cs d’où f d = fe fd = f m 1 + v m/cs 1 + v m/cs On en déduit le décalage spectral Df = fe − f d : 1 − vm/c s 2vm Df Df = f e − fe fe d’où vm = cs = cs + vm 1 + vm/c s 2fe − Df Cette technique permet par exemple de déterminer la vitesse de déplacement vm de globules sanguins : fe = 40 kHz

c s = 1 500 m . s−1

Df = 4 Hz

donnent

vm = 7, 5 cm . s−1

V . 4 . — Onde quasi monochromatique. Vitesse de groupe Les ondes réelles ne sont pas monochromatiques mais polychromatiques, c’est-à-dire constituées de plusieurs ondes monochromatiques. En général, ces ondes sont regroupées autour d’une fréquence moyenne f0 : on dit qu’elles sont quasi monochromatiques si la largeur spectrale totale à mi-hauteur est très inférieure à f0 (cf. Optique). Rappelons que la vitesse de phase est définie pour une onde monochromatique par : v w = v/k, v étant la pulsation et k le nombre d’onde (cf. Électromagnétisme). Dans le cas où coexistent plusieurs ondes, on introduit la vitesse de phase associée à la fréquence moyenne f0 , mais aussi une vitesse de groupe que l’on définit par : vg =

dv dk

Pour connaître la signification de vg, considérons le groupe d’ondes suivant constitué simplement de deux ondes monochromatiques, de même amplitude A et de pulsations voisines : Dv Dv et v2 = v0 + avec Dv = v 2 − v1  v0 v1 = v0 − 2 2 Il vient, si l’on choisit l’axe Ox selon la direction de propagation : v1 v2 C(x, t) = A cos(v1 t − k1x) + A cos(v2t − k 2 x) avec ≈ = vw puisque v1 ≈ v2 k1 k2 La fonction d’onde C(x, t) s’écrit autrement : v1 − v 2 v1 + v2 k 1 − k2 k + k2 C(x, t) = 2A cos x × cos t− t− 1 x 2 2 2 2 soit, en posant Dk = k 2 − k1 et k 0 = (k1 + k2)/2 : Dv Dk C(x, t) = 2A cos x cos(v0 t − k 0x) t− 2 2

593

Ondes mécaniques dans un milieu continu

Ainsi, le groupe d’ondes se comporte comme une onde dont l’amplitude est modulée avec la vitesse de groupe vg = Dv/Dk. Sur la figure 32.11, on a représenté la dépendance temporelle C(0, t) au point origine x = 0 ; la période de la modulation d’amplitude est 2p/Dv. – (0,t)

2π/Φ ω

0

t

F IG . 32.11.

De façon générale, on a, puisque v = v w k : vg =

dv d d vw = (vw k) = vw + k dk dk dk

Pour un milieu non dispersif, vw ne dépend pas de k et vg = v w. C’est le cas de la propagation des ondes acoustiques dans un milieu matériel où la vitesse de phase vw = c s = 1/(rkS) 1/2 est indépendante de k. Remarque : L’analyse de Fourier permet une étude approfondie de la structure d’un train d’ondes (cf. Optique). V . 5 . — Ultrasons

 c Dunod – Toute reproduction non autorisée est un délit

On produit généralement des ultrasons en faisant vibrer une lame cristalline de quartz que l’on soumet à une tension électrique alternative de fréquence f ∼ 1 MHz (Fig. 32.12). C’est l’effet piézoélectrique inverse. Si cette fréquence coïncide avec l’une des fréquences propres de la lame de quartz, laquelle dépend de son épaisseur, l’amplitude des oscillations mécaniques est très importante et provoque la propagation d’ondes ultrasonores dans l’air environnant. Une valeur typique de cette fréquence est 1 MHz obtenue pour une épaisseur d’environ 5 mm. En réalité, cette épaisseur est bien plus grande que celle de la lame de quartz ; c’est l’épaisseur d’un ensemble constitué d’une lame mince de quartz flanquée de deux lames épaisses d’acier ; P. Langevin a montré en effet qu’un tel ensemble vibrait à la fréquence définie par l’épaisseur totale. Générateur de tension

Acier

Quartz

Acier F IG . 32.12.

Beaucoup d’animaux, insectes, chauves-souris, dauphins, baleines, émettent de tels sons pour s’orienter ; les fréquences sont généralement comprises entre 20 kHz et 200 kHz. La faiblesse de la longueur d’onde des ultrasons leur confère des propriétés intéressantes. On peut en effet se placer dans l’approximation géométrique de l’acoustique, qui est l’équivalent de l’optique

594

32. Ondes mécaniques dans un milieu continu

géométrique, et utiliser les lois habituelles de la réflexion des ondes sonores. C’est ainsi que le sonar permet de sonder les fonds marins en envoyant un signal et en déterminant la durée qui sépare l’instant de sa détection après réflexion de l’instant d’émission. L’échographie, largement utilisée en médecine, fonctionne de la même façon. Cette faible longueur d’onde est à la base de l’efficacité de l’imagerie ultrasonore, puisque la dimension minimale détectée atteint la valeur de la longueur d’onde : l ∼ 350 mm. On utilise aussi les ultrasons pour produire des réseaux optiques de phase et donc pour moduler des faisceaux lasers (cf. Optique). Enfin, les ultrasons ont un avenir prometteur dans le domaine de l’audition des sons ; une technique récente consiste à reproduire des sons, de fréquences comprises entre 100 Hz et 20 kHz, en formant des battements entre des signaux d’ultrasons de fréquences voisines. On pourrait ainsi éviter certains défauts des enceintes acoustiques à haut-parleurs.

CONCLUSION Retenons les points essentiels. (1) Dans un milieu continu, les ondes mécaniques longitudinales, de faible amplitude et de fréquences comprises entre 20 Hz et 20 kHz, sont des ondes acoustiques. Leur vitesse de propagation c s s’exprime en fonction de la masse volumique r et du coefficient de compressibilité isentropique : cs =

1 rkS

1/2

Pour l’air, cs ≈ 330 m . s −1, alors que pour l’eau c s ≈ 1 500 m . s −1 et pour l’acier cs ≈ 5 000 m . s−1 . Notons que, le long d’un ressort en acier, la vitesse de propagation d’une onde mécanique de déformation est de l’ordre de 20 m . s−1 . (2) L’intensité acoustique d’une onde sonore monochromique, de pulsation v, se mesure en dB selon : I rA 2v2 cs 10 lg avec I0 = 10−12 W . m−2 et I = 2 I0 (3) L’impédance acoustique caractéristique d’une colonne de fluide, Z c = rc s , permet d’exprimer les facteurs de réflexion et de transmission en intensité d’une onde acoustique monochromatique plane, à la surface de séparation entre deux milieux : R=

(Z c,1 − Zc,2 )2 (Z c,1 + Zc,2 )2

et T =

4Zc,1 Z c,2 (Zc,1 + Z c,2) 2

(4) Si deux ondes acoustiques se propageant en sens inverse se superposent, il n’y a pas de propagation de l’ensemble ; on dit que l’onde résultante est stationnaire. On mesure généralement la vitesse de propagation des ondes acoustiques en réalisant des ondes stationnaires. (5) On retrouve en acoustique les propriétés ondulatoires habituelles : diffraction, interférence, effet Doppler, bien connues en optique. Cependant, pour l’effet Doppler, deux distinctions essentielles doivent être soulignées : en acoustique, la longueur d’onde est invariante, alors qu’elle ne l’est pas en optique, en raison de l’invariance de la vitesse de lumière dans le vide ; en outre, on doit considérer trois référentiels (source, récepteur, milieu), alors qu’en optique deux suffisent (source, récepteur). (6) Enfin les ultrasons donnent lieu à beaucoup d’applications en raison de la faiblesse de la longueur d’onde devant les dimensions des systèmes étudiés : sonar, échographie, imagerie acoustique, acousto-optique.

595

Ondes mécaniques dans un milieu continu

EXERCICES ET PROBLÈMES P32– 1. Vitesse de propagation du son dans l’air. Influence d’un gradient de température 1. Calculer la durée t h de propagation du son, dans l’air à 288 K, provenant d’une explosion produite à une distance de D = 10 km dans un plan horizontal. 2. Quelle est la durée t v , si la source de l’explosion se trouve à la même distance mais sur la verticale ? La température de l’air varie selon : T = T0 − Bz avec T 0 = 288 K et B = 6, 5 K . km −1 . P32– 2. Résonateur d’Helmholtz On considère une sphère creuse, de volume intérieur V 0 = 1 L, munie d’un ajutage horizontal de section s = 5 cm2 et de longueur l = 5 cm. L’air contenu dans l’ajutage est animé d’un mouvement d’ensemble oscillant de faible amplitude ; parallèlement, l’air enfermé dans la cavité évolue de façon adiabatique réversible, sa pression ayant même valeur en tout point. La pression extérieure est la pression atmosphérique p0 = 1 bar, la masse volumique de l’air de l’ajutage est r = 1, 3 kg . m −3 et le rapport des capacités thermiques à pression et volume constants vaut g = 1, 4. Calculer la fréquence d’oscillation de ce système appelé résonateur d’Helmholtz. Comparer la longueur d’onde de l’onde acoustique qui se propage aux dimensions de la sphère. P32– 3. Isolation acoustique Les impédances acoustiques caractéristiques de l’air, de la pierre et du verre valent respectivement, en unités SI : Zc,1 = 425, Zc,2 = 7, 5 × 10 6 et Zc,3 = 10 × 106 . Comparer les capacités d’isolation acoustique de la pierre et du double vitrage. P32– 4. Vitesse de phase et vitesse de groupe dans un pavillon exponentiel

 c Dunod – Toute reproduction non autorisée est un délit

Dans un pavillon acoustique, dont la section droite S dépend de la coordonnée axiale x selon l’équation S (x) = S0 exp(ax), une solution d’onde, de forme complexe C = A exp[−i(vt − kx)] conduit à la relation de dispersion suivante reliant le nombre d’onde k et la pulsation v : ia 1 k= ± 2 2

4v2 −a + 2 cs

1/2

2

cs étant la vitesse de propagation du son dans le milieu en l’absence de pavillon. Quelles sont les vitesses de phase et de groupe dans le pavillon ? P32– 5. Tunnel acoustique à gradient de célérité dans les océans À une certaine profondeur z m , la vitesse cs du son dans l’eau des océans varie faiblement autour de z m selon la loi quadratique : cs = v m 1 +

(z − z m )2 2r2



v m = 1 500 m . s−1

et r = 2 000 m . s−1

Dans l’approximation géométrique, la trajectoire des rayons acoustiques est donnée par l’équation différentielle suivante : d(u/cs ) 1 = grad ds cs

596

32. Ondes mécaniques dans un milieu continu

u étant le vecteur unitaire porté par la tangente à la trajectoire. 1. Trouver l’invariant de la trajectoire reliant c s à l’angle que fait sa tangente avec la verticale. Calculer cet invariant dans le cas où la source sonore est située à la profondeur zm et émet un rayon sonore sous l’angle im = 85◦ par rapport à la verticale descendante (Fig. 32.13). 2. Montrer que l’équation différentielle de la trajectoire est approximativement : d2 z a(z − zm ) + =0 d x2 r 2 sin 2 im a étant une constante que l’on déterminera. En déduire la trajectoire et ses caractéristiques. O x

zm

O

im z

y

Océan

S

F IG . 32.13.

h

x

P F IG . 32.14.

P32– 6. Effet Doppler Deux véhicules A 1 et A2 se croisent sur une route rectiligne avec les vitesses respectives constantes v1 = 72 km . h−1 et v 2 = 90 km . h −1 . Peu de temps avant le croisement, les conducteurs se reconnaissent et se saluent par un coup d’avertisseurs sonores, de fréquences respectives f1 = 1 kHz et f2 = 1, 5 kHz. On suppose qu’il n’y a pas de vent et que la vitesse du son dans l’air est c s = 340 m . s−1. 1. Calculer les fréquences des sons reçus par les passagers de A 2 avant et après le croisement. 2. Même question pour les passagers de A1 . P32– 7. Onde de choc Une onde de choc est la perturbation acoustique que l’on ressent, en raison de l’accumulation des effets, lorsqu’une source sonore se déplace dans un milieu avec une vitesse supérieure à celle du son. 1. Une source sonore S (avion) se déplace dans un plan avec une vitesse constante v supérieure à la vitesse cs du son dans l’air. Établir, en coordonnées polaires (r, u), l’équation de la trajectoire réalisant une onde de choc à l’origine O. 2. La source sonore a un mouvement rectiligne suivant un axe Ox et une vitesse variable v le long de cet axe pour x < 0 (Fig. 32.14). On considère un point P de coordonnées (0, −h). Comment doit varier v en fonction de la coordonnée x < 0 pour qu’il y ait une onde de choc en P ? P32– 8. Pression de radiation d’un faisceau d’ultrasons Un faisceau parallèle d’ultrasons, d’intensité acoustique 4 kW . m −2, se propage dans l’air à la vitesse de 340 m . s−1. 1. Quelle est la pression de radiation de ce faisceau ? 2. Trouver l’accélération acquise par des gouttelettes d’eau en suspension dans l’air sous l’action de cette pression de radiation ? Le diamètre des gouttelettes est de 0, 2 mm.

597

Ondes mécaniques dans un milieu continu P32– 9. Vélocimétrie par ultrasons

On détermine la vitesse v de déplacement des globules sanguins en utilisant une sonde à ultrasons constituée de deux cristaux piézoélectriques, l’un A pour l’émission et l’autre B pour la réception de l’onde réfléchie par les globules. La vitesse v des globules est faible devant la vitesse de propagation des ultrasons cs ≈ 340 m . s −1 ; en outre l’angle que fait la direction de propagation avec la vitesse des globules est petit. La combinaison du faisceau incident, de fréquence 5, 3 MHz, et du faisceau reçu après réflexion par les globules donne des battements dont la fréquence Df = 1, 4 kHz est égale à la différence des fréquences des deux signaux. En déduire la vitesse des globules sanguins. P32– 10. Ouverture d’un faisceau ultrasonore La fréquence des vibrations d’un générateur d’ondes ultrasonores à quartz est reliée à la vitesse du son cq dans le quartz et à l’épaisseur e de la lame par l’équation f = c q /(2e). Les ondes émises, utilisées pour explorer le relief sous marin, sont diffractées par une ouverture circulaire de diamètre D = 40 cm. En admettant l’approximation de Fraunhofer satisfaite, calculer l’ouverture angulaire du faisceau, sachant que la vitesse de propagation de ces ondes dans l’eau est ce = 1 500 m . s −1 , cq = 5 000 m . s−1 et e = 6 cm. P32– 11. Déplacement du tympan au seuil d’audibilité Sachant que l’intensité acoustique de l’oreille au seuil d’audibilité est en moyenne de 10 −12 W . m−2 , à 1 kHz, évaluer l’amplitude du déplacement correspondant du tympan. P32– 12. Solitons de torsion et d’antitorsion le long d’une tige métallique

 c Dunod – Toute reproduction non autorisée est un délit

Un système mécanique est constitué d’un ensemble de N pendules simples, rigides, identiques (masse m = 10 g, longueur l = 6 cm), dont les points d’attache sont disposés régulièrement, avec une période spatiale a = 1, 2 cm, le long d’une tige en acier (Fig. 32.15). Chaque segment de la tige exerce sur les pendules, plaçés en ses extrémités, des couples de torsion de la forme −Cu, C = 0, 09 N . m . rad−1 étant leur constante de torsion et u l’angle dont tourne la tige autour de son axe Ox horizontal. On suppose que la liaison des pendules sur l’axe est parfaite. O n−1

g

a θn−1

On

On+1 x

θn

An−1 An

θn+1 An+1

F IG . 32.15.

1. a) En appliquant le théorème du moment cinétique au pendule A n , en On, en projection suivant l’axe Ox, montrer que l’équation différentielle à laquelle satisfait l’angle u n de déviation du pendule, par rapport à la verticale, s’écrit : ¨ un = −v 20 sin u n − v21(2un − u n+1 − un−1 ) v 0 et v1 étant deux coefficients que l’on exprimera en fonction des caractéristiques du système et que l’on calculera numériquement.

598

32. Ondes mécaniques dans un milieu continu

b) Que devient l’équation différentielle en u en l’absence de torsion de la tige ? Quelle est la nature du mouvement de chaque pendule dans l’approximation linéaire ? Calculer la période commune des pendules. 2. a) Sachant que la distance a entre deux pendules consécutifs est suffisamment faible, développer u n+1 et un−1 , autour de la valeur u = un , jusqu’à l’ordre 2. Montrer que l’équation différentielle du mouvement en u devient : 1 ∂ 2u ∂ 2u = k20 sin u − ∂x2 v 20 ∂t2 v0 et k 0 étant deux coefficients que l’on exprimera en fonction des caractéristiques et que l’on calculera numériquement. b) Que devient l’équation précédente en l’absence des pendules ? Trouver sa solution. En déduire la valeur de la vitesse de propagation de l’onde de torsion le long de la tige en acier. Comparer cette vitesse à l’ordre de grandeur de la vitesse de propagation des ondes mécaniques longitudinales dans l’acier. 3. On se place dans l’approximation linéaire de l’équation différentielle en u établie en 2 . a). a) Montrer que cette équation admet une solution de la forme u = A exp[−i(vt − kx)], t étant la variable temporelle et x la variable spatiale le long de la tige. En déduire la relation de dispersion v(k) ? Quelle est la nature précise de la courbe donnant v/v0 en fonction de k/k 0 ? Calculer la valeur de la pulsation de coupure. b) Établir la relation entre la vitesse de groupe et la vitesse de phase. 4. On appelle soliton de torsion ou d’antitorsion le phénomène physique représenté par la solution u(s) de l’équation différentielle non linéaire précédente : v u(s) x tan = exp(±gs) avec s = v0 t − g = (1 − b 2)−1/2 et b = 4 v v0 v étant une constante. Le signe + dans l’argument de l’exponentielle représente le soliton de torsion, et le signe − le soliton d’antitorsion. a) Donner la signification physique de v (dimension, interprétation, valeur limite).

b) Calculer les valeurs de u(s) et de sa dérivée u  = d u/ d s pour s = −∞, s = 0 et s = ∞.

c) Représenter graphiquement la solution u s (s) qui correspond au soliton de torsion statique caractérisé par v = 0. Même question pour le soliton de torsion ultra rapide caractérisé par v ≈ v0 . d) Si l’on calcule l’énergie mécanique du système, on trouve : E = 8g

ml2 v0v 0 a

En mettant cette énergie sous la forme E = gms v20 , déterminer la masse m s du soliton et calculer sa valeur. En déduire l’expression de sa quantité de mouvement en fonction de m s, g et v. Commenter.

Annexe 1 Les coniques

I . — DÉFINITION Une conique est l’ensemble des points M d’un plan tels que le rapport des distances à un point F et à une droite D soit constant : FM = Cte = e HM

Le nombre positif e est l’excentricité, F un foyer et D une directrice de la conique (Fig. A1.1a). On peut aussi définir les coniques géométriquement par l’intersection d’un cône par un plan, d’où leur nom (Fig. A1.1b). On introduit généralement la distance p, appelée paramètre de la conique, et on écrit la distance de F à D sous la forme : p e

Parabole

Cercle

y H

M r w

H0 D

Hyperbole

 c Dunod – Toute reproduction non autorisée est un délit

FH 0 =

F

p=e

Axe focal x

a)

Ellipse

b) F IG . A1.1.

600

Annexe 1.

II . — ÉQUATION POLAIRE En coordonnées polaires, r = FM et w = (Fx, FM), l’équation FM = e HM s’écrit aussi : p p p + r cos w d’où r = soit aussi r = r=e 1 − e cos w 1 + e cos(w − p) e Notons que l’angle w − p est l’angle (−Fx, FM) que fait le rayon vecteur FM avec l’axe −Fx. i) Si e < 1, la conique est une ellipse ; r min  r  rmax avec : p p et r max = rmin = 1+e 1−e ii) Si e > 1, la conique est une hyperbole ; r min  r < ∞ avec : p rmin = 1+e iii) Dans le cas limite où e = 1, la conique est une parabole ; r min  r < ∞ avec : p r min = 2 Remarque : La signification de p est évidente : p = r pour w = p/2. En outre, si e = 0, la conique se réduit à un cercle.

III . — ÉQUATION CARTÉSIENNE Il est naturel d’adopter le système d’axes H 0 xy. Exprimons, en fonction des coordonnées x et y, la relation FM2 = e2HM 2. Il vient : 2px p 2 p2 (E) x− + y2 = e2x2 soit x2 (1 − e 2) − + y2 + 2 = 0 e e e C’est une équation du deuxième degré en x et en y. III . 1 . — Parabole La parabole est la conique d’excentricité égale à 1. Dans ce cas, l’équation (E) précédente se réduit à y 2 = 2p (x − p/2) . En introduisant le nouveau système d’axes OXY défini par X = x − p/2 et Y = y (Fig. A1.2), l’équation de la parabole se met sous la forme canonique suivante : Y2 = 2pX y Y H

M p

H0 O p

w

F

F IG . A1.2.

x;X

601

Les coniques III . 2 . — Ellipse et hyperbole

Pour e = 1, il est judicieux de multiplier les deux membres de l’équation (E) par (1 − e 2)/p2 . Il vient : 2 2 (1 − e 2)2 1 1 − e2 p 2 (1 − e ) − + =0 x − + y e(1 − e2 ) p2 e2 p2 e2 Dans le nouveau système d’axes OXY (Fig. A1.3 et A1.4) tel que : X=x−

p e(1 − e 2 )

et Y = y

l’équation (E) devient : X 2(1 − e2 )2 Y 2(1 − e 2) + =1 p2 p2 Les anciennes coordonnées de l’origine O sont alors : x O =

p et yO = 0. e(1 − e 2)

a) Ellipse Pour l’ellipse, définie par une excentricité inférieure à 1, l’équation (E) se met sous la forme canonique suivante : X2 Y2 + =1 a2 b2

avec

a=

p 1 − e2

et

b=

p (1 − e2 )1/2

Comme e < 1, l’origine O se trouve sur H0 x à droite de F (Fig. A1.3). La distance c entre les points O et F vaut :

 c Dunod – Toute reproduction non autorisée est un délit

c = OF = H0 O − H 0F =

p p pe = ea d’où − = e(1 − e 2) 1 − e2 e

e=

c a

En outre, puisque b = a(1− e2 )1/2 on a la relation entre a, b et c : c2 = e 2a2 = a 2−a 2(1−e2) = a2 −b2 , soit : a 2 = c 2 + b2 Y y

y

Y

H p=e H0

H

M p F

w O

F  H0 x;X

 F

M p

H0 H0

F

O

p=e

D

D F IG . A1.3.

F IG . A1.4.

w x;X

602

Annexe 1.

b) Hyperbole Pour l’hyperbole, définie par une excentricité supérieure à 1, l’équation canonique de l’hyperbole se met sous la forme canonique : X2 Y2 =1 − a2 b2

avec

a=

e2

p −1

et

b=

(e2

p − 1)1/2

Comme e > 1, l’origine O se trouve sur H0x à gauche de D : x O < 0 (Fig. A1.4). La distance c entre les points O et F vaut : c = OF = H0 O + H 0F =

e(e2

p p pe + = 2 = ea d’où − 1) e e −1

e=

c a

En outre, puisque b = a(e2 − 1) 1/2 , on a la relation : c 2 = e2a2 = a 2 + a2 (e2 − 1) = a2 + b2, soit : a 2 = c 2 − b2

IV . — PROPRIÉTÉS FONDAMENTALES DES CONIQUES IV . 1 . — Parabole Dans le cas de la parabole (Fig. A1.2), on a, puisque e = 1 : MF = MH La parabole est l’ensemble des points M situés à égale distance d’un point F et d’une droite D . IV . 2 . — Ellipse et hyperbole Notant ε = ±1, les équations canoniques de l’ellipse et de l’hyperbole peuvent être rassemblées selon : X2 Y2 + = 1 et a2 = c 2 + εb 2 a2 εb2 Il en résulte que : X2 Y2 c 2X2 2 2 2 2 − 1 = − soit X + Y + c = a + a2 a 2 − c2 a2 2 2 en multipliant les deux membres de l’équation par (a − c ). En y ajoutant ±2cX , on obtient les deux équations suivantes : cX 2 cX 2 (X + c) 2 + Y 2 = a + et (X − c) 2 + Y 2 = a − a a a) Ellipse Dans le système d’axes OXY, dans lequel F a pour coordonnées (−c, 0) (Fig. A1.3), FM a pour composantes (X + c, Y ). Par conséquent, la première des deux équations précédentes s’écrit : cX FM = a + a 2

2

Notant F le point symétrique de F par rapport à O, la seconde équation s’écrit, de façon analogue : F M 2 = a −

cX a

2

603

Les coniques Or |X |/a  1 et c < a entraînent c|X |/a < a. Par conséquent : FM = a +

cX a

F M = a −

cX a

et

FM + FM = 2a

L’ellipse est l’ensemble des points M tels que la somme des distances à deux points F et F  est constante. b) Hyperbole Dans le système d’axes OXY, dans lequel F a pour coordonnées (c, 0) (Fig. A1.4), FM a pour composantes (X − c, Y ). On peut donc écrire, comme précédemment : FM 2 = a −

cX a

2

cX et F M = a + a 

2

2

F  étant le symétrique de F par rapport à O. Or, c étant supérieur à a, on a : X a

2

=1+

Y2 > 1 d’où c2 − a2

c|X| >c>a a

Deux cas se présentent donc suivant la position du point M. 1er cas : X > 0. Comme cX/a > a, on a : FM =

cX −a a

F M =

cX +a a

et

F  M − FM = 2a

Le point M décrit la branche droite de l’hyperbole (Fig. A1.4). 2e cas : X < 0. Comme −cX /a > a, on a a + cX /a < 0, d’où : FM = a −

cX a

F M = −

cX −a a

et

FM − F M = 2a

Le point M décrit la branche gauche de l’hyperbole (Fig. A1.4). Ainsi :

 c Dunod – Toute reproduction non autorisée est un délit

|FM − F  M| = 2a L’hyperbole est l’ensemble des points M tels que la différence des distances à deux points F et F  est constante. Lorsque l’abscisse X de M tend vers ±∞, l’équation de l’hyperbole donne : Y2 X2 X2 = 2 −1≈ 2 b2 a a Par conséquent, l’hyperbole admet deux asymptotes : Y = bX/a et Y = −bX/a qui se coupent en O.

Annexe 2 Dérivées et différentielles I . — DÉRIVÉES I . 1 . — Dérivée d’une fonction a) Définition On dit qu’une fonction réelle f , définie dans un intervalle des réels R, est dérivable, en un point x0 de cet intervalle et admet pour dérivée f  (x 0 ), si : lim

x→x0

f (x) − f (x0) x − x0

= f (x0 )

df (x0 ) dx

notée

ou

df dx

x0

b) Interprétation géométrique Traçons le graphe de f (Fig. A2.1). Le rapport Df /Dx, au point d’abscisse x 0 , représente la tangente de l’angle u que fait la sécante M0 M avec l’axe Ox du graphe : Df HM = = tan u Dx M 0H Lorsque M se rapproche de M 0, la position limite de la sécante devient la tangente à la courbe en M 0 . L’angle u prend la valeur a : f  (x 0) =

df (x0 ) = tan a dx

f ( x) M Δf ®

M0 0

Δx x0 F IG . A2.1.

µ

H x

605

Dérivées et différentielles

La fonction f  est appelée la fonction dérivée de f . Elle peut, elle aussi, admettre une fonction dérivée et ainsi de suite. On note ces dérivées successives : f

. . . f (n)

f 

d2 f d x2

df dx

ou bien

...

...

dn f d xn

...

I . 2 . — Dérivées partielles Soit f (x, y, z) une fonction réelle, définie dans une partie de R 3 contenant le point (x0 , y0, z 0). Pour (x0 , y 0) donnés, la fonction f (x, y0, z 0) est une fonction g de la seule variable x ; si elle est dérivable en x0 , sa dérivée s’appelle la dérivée partielle de f par rapport à x au point (x 0, y0, z 0). On la note : ∂f fx(x0 , y0, z0) ou (x0 ) ∂x De même, on définit fy et fz . Les fonctions telles que f x (x, y, z) sont les fonctions dérivées partielles. Elles peuvent admettre à leur tour des fonctions dérivées et ainsi de suite. Ces dérivées successives, qui peuvent être croisées, sont notées comme suit : fx

fx

fxy

. . . f x(n)

...

ou bien

∂f ∂x

∂2f ∂x2

∂ 2f ∂ x∂ y

...

∂(n) f ∂xn

...

I . 3 . — Dérivée d’une fonction composée a) Fonction d’une seule variable Soient u une fonction dérivable dans un intervalle I de R et f une fonction réelle définie dans l’intervalle u(I ). Si u est dérivable en x0 et f dérivable en u(x0 ), alors F = f ◦ u est dérivable en x0 et : F (x0 ) = f  [u(x 0)] u(x 0) La fonction F  (x0) = f  [u(x)] u (x) est appelée la fonction dérivée composée de F. Exemple : La fonction dérivée par rapport à t de a cos u(t) est : −a sin u(t)(d u/ d t). b) Fonction de plusieurs variables

 c Dunod – Toute reproduction non autorisée est un délit

Soit f une fonction réelle, dérivable dans une partie U de R 3. Soient u, v, w des fonctions réelles dérivables dans un intervalle I de R, telles que [u(x), v(x), w(x)] ∈ U. Dans ces conditions, la fonction composée F(x) = f [u(x), v(x), w(x)] admet comme dérivée : F (x) =

∂f  ∂f  ∂f  v (x) + w (x) u (x) + ∂v ∂w ∂u

I . 4 . — Dérivée logarithmique L’emploi de la dérivée logarithmique donne une forme particulièrement simple aux résultats sur la dérivée d’un produit ou d’un quotient. En effet : P = uvw

entraîne

ln |P| = ln |u| + ln |v | + ln |w|

d’où

v w P u = + + v w P u

De même : Q=

u v

entraîne

ln |Q| = ln |u| − ln |v|

d’où

Q u v  − = Q u v

606

Annexe 2.

I . 5 . — Dérivée d’un vecteur par rapport à un paramètre t a) Définition On appelle dérivée du vecteur A, par rapport au paramètre t, en t 0, le vecteur : lim

t→t 0

A(t) − A(t0) t − t0

notée

dA (t 0) dt

Ses composantes dans une base B = (e x , ey, ez ) sont les dérivées, par rapport à t, des composantes de A dans cette base. Ainsi la dérivée de A = Ax ex + Ay e y + Az ez est le vecteur : dA dt

= B

d Ax d Ay d Az ex + ey + ez dt dt dt

ce que l’on écrit souvent B

d Ax/ d t d Ay/ d t d Az / d t

b) Dérivée d’un produit scalaire de deux vecteurs Comme A · B = A x B x + A yB y + A z Bz , il vient : d d Ax d Bx (A · B) = + ··· B x + Ax dt dt dt

d’où

dA dB d (A · B) = ·B+A· dt dt dt

Exemple : Dérivons le carré de la norme du vecteur position r : r2 = r2 = r · r

donne

2r

dr dr = 2r · dt dt

soit

r

dr dr =r· dt dt

Si r2 = Cte, c’est le cas des vecteurs unitaires, r · d r/ d t = 0 : le vecteur dérivée est normal à r. c) Dérivée d’un produit vectoriel Dérivons le vecteur C = A × B, relativement à la base B. Avec des notations évidentes, (d C/ d t)B s’écrit : dC dt Or :

= B

d d d (A yB z − AzB y ) ex + (Az B x − Ax Bz) e y + (Ax By − AyB x) e z dt dt dt

d (A yB z − AzB y) = dt

d Ay dt

Bz −

d Az dt

By + Ay

d Bz dt

− Az

d By dt

En procédant de la même façon pour les deux autres composantes et en ordonnant, on obtient : d (A × B) dt

= B

dA dt

B

×B + A×

dB dt

B

Remarque : Les définitions précédentes peuvent être étendues aux fonctions à valeurs complexes en considérant les fonctions réelles Re{f } et Im{f }.

607

Dérivées et différentielles

II . — DIFFÉRENTIELLES II . 1 . — Définition On appelle différentielle d’une fonction f , qui admet des dérivées partielles dans une partie de R 3 , l’application linéaire définie sur l’espace vectoriel E3 par : d f (U) = f x u1 + fy u2 + fz u3

U étant le vecteur de composantes (u1 , u2, u 3). Les quantités u1, u 2, u 3 sont respectivement les valeurs que prennent les différentielles d x, d y et d z sur le vecteur U ; en effet, si f (x, y, z) = x, on a : fx = 1

fy = 0

fz = 0

et

Il en résulte que d x(U) = u1. De même pour u2 et u3. Nous retiendrons donc l’écriture : d f = f x d x + fy d y + fz d z En physique, on utilise le plus souvent la propriété suivante : la valeur de d f fournit une valeur, approchée à l’ordre deux près, de la variation Df de f pour un déplacement de composantes Dx, Dy, Dz, qui est d’autant plus précise que Dx, Dy et Dz sont faibles. C’est ce qu’illustre à une dimension le graphe f (x) de la figure A2.2. f ( x) M Δf M0

df dx

0

x0

x

F IG . A2.2.

II . 2 . — Exemples Sur le tableau A2.1, on a rassemblé les différentielles de quelques fonctions courantes.

 c Dunod – Toute reproduction non autorisée est un délit

Fonctions 2

Différentielles

f (x) = 3x + 5x + 4 f (x) = 1/x f (x) = ln x f (x) = exp (ax) f (x) = a cos x

d f = (6x + 5) d x d f = − d x/x2 d f = d x/x

d f = a exp(ax) d x d f = −a sin x d x

f (x) = a sin x

d f = a cos x d x

f (x) = tan x f (x, y) = x + y

d f = d x/ cos2 x df = dx +dy

f (x, y) = xy

df = ydx + xdy

f (x, y) = x/y

d f = (y d x − x d y)/y2 TAB . A2.1.

608

Annexe 2.

II . 3 . — Différentielle logarithmique La différentielle logarithmique s’obtient directement à partir de la dérivée logarithmique d’un produit ou d’un quotient et de la définition de la différentielle : donne

P = uvw

P u v w = + + v w P u

et

dP du dv dw = + + v w P u

De même : u v

Q=

donne

Q u v  − = v Q u

et

dQ du dv − = v Q u

II . 4 . — Gradient d’une fonction a) Définition Soit f une fonction réelle, définie et continûment dérivable dans une partie de R 3 . Sa différentielle d f s’écrit : d f = fu d u + fv d v + fw d w Notons d r le vecteur déplacement élémentaire : d r = d l u eu + d lv e v + d l we w où eu , e v, e w est la base orthonormée d’un système de coordonnées locales (Fig. A2.3). On appelle gradient de la fonction f , noté grad f , le vecteur défini par : Par conséquent :

d f = grad f · d r = (grad f ) u d l u + (grad f )v d lv + (grad f )w d l w

(grad f )u =

∂f ∂lu

(grad f ) v =

∂f ∂lv

(grad f )w =

∂f ∂l w

z H d lw θ O ϕ x

d lu r

d lv grad f

y

ρ

f 2 > f1 f1

P F IG . A2.3.

F IG . A2.4.

b) Propriété fondamentale du gradient Un déplacement élémentaire d r sur la surface définie par f (u, v, w) = Cte n’entraîne aucune variation de f . Comme d’autre part cette variation s’identifie à la différentielle d f , on a : d f = grad f · d r = 0

Il en résulte que le vecteur gradient est normal à la surface f (u, v, w) = Cte. Si le déplacement élémentaire se fait dans la direction et le sens de grad f , d f est positif. Ainsi grad f est orienté suivant les valeurs croissantes de f (Fig. A2.4).

609

Dérivées et différentielles

III . — SYSTÈMES DE COORDONNÉES Pour préciser l’écriture des lois de la mécanique, et plus généralement celles de la physique, il est utile de connaître les expressions des éléments différentiels, de longueur d r, de surface d S et de volume d , dans les principaux systèmes de coordonnées utilisés. En effet, on sait que l’on peut repérer la position d’un point dans l’espace en utilisant différents systèmes de coordonnées. Ce sont les conditions particulières du problème à traiter qui permettent de choisir celui qui sera occasionnellement le plus adapté : symétrie, expressions des champs, etc. Nous explicitons ici les éléments différentiels, d r, d S et d , dans les trois systèmes de coordonnées les plus utilisés : cartésiennes, cylindriques et sphériques. III . 1 . — Coordonnées cartésiennes a) Expression des éléments de longueur, de surface et de volume Un déplacement élémentaire MM  = d r a pour composantes d x, d y et d z dans la base (e x, ey , ez ) des coordonnées cartésiennes (Fig. A2.5). On écrit donc : d r = d x ex + d y e y + d z ez Les trois éléments de surface sont, respectivement pour MAA B, MBB C et MCC  A : d x d y, d y d z et d z d x. Quant au volume du parallélépipède, construit sur les trois déplacements élémentaires de base, il a pour expression d = d x d y d z. b) Expression du gradient En coordonnées cartésiennes (x, y, z), d l u = d x, d l v = d y et d lw = d z. Par conséquent : (grad f )x =

 c Dunod – Toute reproduction non autorisée est un délit

z

∂f ∂x

C dy C

O

A



A

∂f ∂y

(grad f ) z =

dx

B

 C M B A M A

C

dz B

y

O ϕ

x

P

∂f ∂z

z

B

M

M

(grad f ) y =

x

F IG . A2.5.

ρ ρ dϕ P dρ

y

F IG . A2.6.

III . 2 . — Coordonnées cylindriques a) Expression des éléments de longueur, de surface et de volume On peut repérer la position d’un point M dans l’espace à l’aide des coordonnées cylindriques (r, w, z) représentées sur la figure A2.6 : r est la distance OP, P étant la projection de M dans le

610

Annexe 2.

plan Oxy, w est l’angle que fait OP avec l’axe Ox. On a les relations suivantes : x = r cos w

et y = r sin w

Les trois vecteurs (er , ew, ez ) forment, au point M, une base orthonormée locale de l’espace euclidien adaptée à l’étude des problèmes à symétrie cylindrique. Dans cette base, le déplacement élémentaire s’écrit naturellement : d r = MM = d r er + r d w e w + d z e z Les trois éléments de surface s’écrivent, respectivement pour MAAB, MBB  C et MAC C : r d r d w, r d w d z et d r d z. Quant à l’élément de volume quasi parallélépipédique que l’on construit sur les trois déplacements élémentaires de base, il a pour expression : d = r d r d w d z. b) Expression du gradient En coordonnées cylindriques, l’expression ci-dessus de d r montre que : d l u = d r, d l v = r d w et d lw = d z. Donc : (grad f ) r =

∂f ∂r

(grad f )w =

1 ∂f r ∂w

(grad f ) z =

∂f ∂z

III . 3 . — Coordonnées sphériques a) Expression des éléments de longueur, de surface et de volume On repère aussi la position d’un point M dans l’espace à l’aide des coordonnées sphériques (r, u, w), r étant la norme de OM et u l’angle que fait OM avec l’axe Oz (Fig. A2.7). On a les relations suivantes : x = r sin u cos w y = r sin u cos w et z = r cos u C

z

A rdθ dr A M  M C B B r

θ y

O ϕ x r sin θ d ϕ F IG . A2.7.

Les trois vecteurs er , eu, ew forment, au point M, une base orthonormée locale de l’espace euclidien adaptée à l’étude des problèmes à symétrie sphérique. Dans cette base, le déplacement élémentaire s’écrit naturellement : d r = MM  = d r e r + r d u eu + r sin u d w ew Les trois éléments de surface sont, respectivement pour MAA B, MAC C et MBB C : r d r d u, r sin u d r d w et r2 sin u d u d w. Quant à l’élément de volume quasi parallélépipédique, que l’on construit sur les trois déplacements élémentaires de base, il a pour expression : d = r 2 sin u d r d u d w.

611

Dérivées et différentielles

Notons que, dans le cas d’un système à symétrie sphérique, on peut adopter, comme élément différentiel, le volume compris entre deux sphères concentriques, de rayons voisins r et r + d r. Les intégrales suivant w, entre 0 et 2p, et suivant u, entre 0 et p, valant respectivement 2p et 2, cet élément différentiel a pour valeur 4pr2 d r. b) Expression du gradient L’expression ci-dessus de d r montre que : d l u = d r, d l v = r d u et d lw = r sin u d w. Donc : (grad f )r =

∂f ∂r

1 ∂f r ∂u

(grad f ) u =

(grad f )w =

1 ∂f r sin u ∂w

Remarque : Il ne faut pas confondre  d r, norme de d r, et d r variation élémentaire de la norme r. c) Exemples de calcul i) Soit la fonction f (x, y, z) = x 2 + 2xy + yz. Le vecteur gradient s’obtient aisément : grad f = 2(x + y) ex + (2x + z) e y + y e z Au point de coordonnées (1, 3, 2), ses composantes valent respectivement : 8, 4, 3. ii) Si f est une fonction radiale, c’est-à-dire une fonction uniquement de r, on a : grad f = (d f / d r) er On obtient ainsi les expressions de grad r, grad(1/r) et grad(1/r 3 ) : gradr = er

grad

1 r

r 1 = − 2 er = − 3 r r

grad

1 r3

=−

3r 3 er = − 5 4 r r

IV . — FORMES DIFFÉRENTIELLES

 c Dunod – Toute reproduction non autorisée est un délit

IV . 1 . — Définition Soient P(u, v, w), Q(u, v, w) et R(u, v, w) trois fonctions des variables (u, v, w). La quantité : dC = P d u + Q d v + R d w est une forme différentielle de degré 1. A priori, cette quantité n’est pas la différentielle d’une certaine fonction f puisque P, Q et R ne sont pas nécessairement des dérivées partielles, d’où la notation d distincte de d . Un exemple de forme différentielle est donné par la circulation élémentaire (ou travail élémentaire) d’un champ de vecteur quelconque A : dC = A · d r = Au d lu + A v d lv + A w d lw La forme différentielle dC n’est une différentielle exacte que si P, Q et R sont les dérivées d’une même fonction f respectivement par rapport à l’une des trois variables u, v, w, les deux autres étant maintenues constantes : P=

∂f ∂u

Q= v,w

∂f ∂v

R= u,w

∂f ∂w

u,v

612

Annexe 2.

En l’absence d’ambiguité, on omet généralement d’expliciter les variables maintenues constantes. Si les conditions précédentes sont satisfaites, alors : dC = P d u + Q d v + R d w = d f Il en résulte que, sous réserve de la continuité des dérivées partielles : ∂R ∂Q − =0 ∂v ∂w

∂P ∂R − =0 ∂w ∂u

∂Q ∂P − =0 ∂u ∂v

IV . 2 . — Exemple Considérons la forme différentielle suivante : (8xy − 3z 2 ) d x + (4x2 + 3z2) d y − 6z(x − y) d z C’est une différentielle totale exacte, c’est-à-dire la différentielle d’une certaine fonction, car : ∂[−6z(x − y)] ∂(4x2 + 3z2) = = 6z ∂y ∂z

∂(8xy − 3z 2) ∂[−6z(x − y)] = = −6z ∂z ∂x

et ∂(4x2 + 3z2) ∂(8xy − 3z2 ) = 8x = ∂x ∂y On calcule la fonction f associée à l’aide des trois équations suivantes : 8xy − 3z 2 =

∂f ∂x

4x 2 + 3z2 =

∂f ∂y

− 6z(x − y) =

∂f ∂z

En intégrant la première équation par rapport à x, on trouve : f (x, y, z) = 4x2 y − 3z2x + g(y, z) À l’aide de la deuxième équation, on peut écrire : 4x2 + 3z 2 = 4x2 +

∂g ∂y

d’où

∂g = 3z2 ∂y

et g = 3z2 y + h(z)

En tenant compte de la troisième équation, il vient : −6z(x − y) = −6zx + 6zy +

dh dz

d’où

h(z) = Cte

Finalement : f (x, y, z) = 4x 2y − 3z2 (x − y) + Cte

Annexe 3 Équations différentielles Les lois de la physique, et donc celles de la mécanique, se traduisent le plus souvent par des équations reliant des fonctions dépendant d’une ou plusieurs variables à leurs dérivées première et seconde par rapport à ces variables. Ces équations sont appelées des équations différentielles. Parmi elles, les équations linéaires jouent un rôle important en raison de leur simplicité.

I . — ÉQUATIONS DIFFÉRENTIELLES LINÉAIRES Les équations différentielles que l’on rencontre le plus souvent dans les problèmes simples de mécanique sont linéaires, c’est-à-dire que toute combinaison linéaire de solutions est encore solution de l’équation. C’est le cas pour les équations suivantes, respectivement du premier et du deuxième ordre : dv v + =a dt t

et

d2 x + lx = b d t2

 c Dunod – Toute reproduction non autorisée est un délit

t, a, l et b étant des constantes. Les expressions ci-dessus sont linéaires par rapport aux fonctions v, x et leurs dérivées. Remarque : Si v a les dimensions d’une vitesse et t représente le temps, t est homogène à une durée et la constante a à une accélération. Si x a la dimension d’une longueur, l est homogène à l’inverse d’une durée au carré. Ces deux équations différentielles peuvent se ramener aisément à des équations sans second membre. En effet, posant V = v − at et X = x − b/l, on obtient respectivement : dV d2 X V + =0 et + lX = 0 dt d t2 t Il suffit donc de résoudre les équations sans second membre et d’ajouter les solutions particulières suivantes : v = Cte = at et x = Cte = b/l respectivement. I . 1 . — Équation différentielle du premier ordre L’intégration de cette équation s’effectue sans difficulté. En effet, cherchons une solution de la forme Cte × exp(rt). L’équation différentielle : dV V + =0 dt t

donne

r+

1 exp(rt) = 0 t

614

Annexe 3.

d’où l’équation caractéristique r + 1/t = 0. On en déduit : V (t) = Cte × exp −

t t

et v(t) = Cte × exp −

t + at t

Pour calculer la constante, il suffit de connaître v à un instant particulier. Par exemple, si, pour t = 0, v = v0 , alors (Fig. A3.1) : t v(t) = (v0 − at) exp − + at t v v0 a¿ 0

¿

t F IG . A3.1.

I . 2 . — Équation différentielle linéaire du deuxième ordre sans second membre Cherchons des solutions de la forme X = Cte × exp(rt). L’équation différentielle : d2 X + lX = 0 d t2

(r 2 + l) exp(rt) = 0

donne

d’où l’équation caractéristique r2 + l = 0. 1er cas : l > 0 Posant l = v20 , on obtient les deux solutions imaginaires pures r = jv 0 et r = −jv0. La solution générale est une combinaison linéaire de ces deux solutions. Donc : X(t) = C+ exp(jv0 t) + C − exp(−jv0 t)

ou

X (t) = A cos(v0t) + B sin(v 0 t)

ce qui s’écrit aussi X (t) = C cos(v0t + w). Les constantes C + , C− , A, B, C et w sont déterminées ˙ par des valeurs particulières de X et X. x + v20 x = b admet comme solution : Exemple : L’équation ¨ x(t) = X (t) +

b v 20

avec

X(t) = C + exp(jv0 t) + C− exp(−jv 0t)

Si, initialement, X = 0 et X˙ = v0, il vient : X (0) = C + + C− = 0 et X˙ (0) = jv 0(C + − C − ) = v0 . On en déduit C+ = −C − = v0 /(2jv0). Ainsi : X(t) = 2jC+ sin(v0 t) =

v0 sin(v0 t) v0

2e cas : l < 0 Posant l = −a2, on obtient les deux solutions réelles r = a et r = −a. La solution générale X (t), combinaison de ces deux solutions, peut donc s’écrire : X(t) = C + exp(at) + C− exp(−at)

ou

X (t) = A cosh(at) + B sinh(at)

615

Équations différentielles

˙ Par exemple, supposons que Les constantes sont déterminées par des valeurs particulières de X et X. ˙ pour t = 0, on ait X = 0 et X = v0 . Il vient : X(0) = C+ + C − = 0 et X˙ (0) = a(C + − C− ) = v 0 . On en déduit C+ = −C − = v0 /(2a). Finalement : X(t) =

v0 sinh(at) a

3e cas : l = 0 Dans ce cas limite, on a : d2 X = 0 d’où d t2

dX = Cte = v 0 et X(t) = v 0 t dt

si les conditions initiales sont les mêmes que précédemment.

II . — ÉQUATION DIFFÉRENTIELLE NON LINÉAIRE Les équations différentielles non linéaires interviennent de plus en plus en physique moderne car elles décrivent des phénomènes intéressants (cf. chapitres 10, 11 et 24). En raison de la complexité des solutions, ces équations différentielles sont de nos jours étudiées par des moyens informatiques. Un exemple d’équation différentielle non linéaire, est fourni par le mouvement d’un point matériel soumis à son poids et à une force de frottement visqueux proportionnelle au carré de la vitesse : v2 dv =a 1− 2 dt v1 Les quantités a et v1 ont respectivement les dimensions d’une accélération et d’une vitesse. En séparant v et t, cette équation différentielle s’écrit : dv dv dv = a dt = + 2 2 1 − v /v 1 2 (1 + v/v1 ) 2 (1 − v/v1 ) d’où, en intégrant, si v = 0 à t = 0 :

 c Dunod – Toute reproduction non autorisée est un délit

ln 1 +

v v1

− ln 1 −

v v1

= 2a t

On en déduit l’expression de v(t) : v(t) = v1

1 − exp(−2at) = v1 tanh(at) 1 + exp(−2at)

On voit que v1 est la limite vers laquelle tend vers v lorsque t augmente infiniment.

Annexe 4 Flux et circulation de vecteur Les notions de flux d’un vecteur à travers une surface et de circulation d’un vecteur le long d’une courbe sont essentielles pour exprimer les lois de la physique, notamment celles de la mécanique des fluides et de l’électromagnétisme.

I . — FLUX D’UN CHAMP DE VECTEUR I . 1 . — Orientation d’une surface Orienter une surface ouverte consiste à distinguer conventionnellement ses deux faces en choisissant un sens positif sur sa normale. Pour cela, on trace sur cette surface une courbe fermée sans point double, entourant le pied de la normale, et on l’oriente. Le sens positif de la normale est donné par la règle dite du tire-bouchon ou du bonhomme d’Ampère (Fig. A4.1a). Tout élément de surface, d’aire d S, est alors caractérisé par le vecteur n d S, parfois noté d S, n étant le vecteur unitaire normal à l’élément de surface et orienté conformément à cette règle. Dans le cas d’une surface fermée, la distinction entre les deux faces n’est plus ambiguë (Fig. A4.1b) : l’une est intérieure alors que l’autre est extérieure. On convient généralement d’orienter vers l’extérieur la normale en chaque point d’une surface fermée. n dS n

dS

a)

b) F IG . A4.1.

I . 2 . — Flux d’un champ de vecteur Considérons une surface quelconque S orientée et un champ de vecteur A. Par définition, on appelle flux de A à travers S l’intégrale suivante : F=

S

A ·n dS

617

Flux et circulation de vecteur Remarquons que dF = A · n d S peut se mettre sous la forme :

dF = P d y d z + Q d z d x + R d x d y

C’est donc une forme différentielle de degré 2 dans R 3. I . 3 . — Angle solide La notion d’angle solide est une généralisation de celle d’angle plan. Rappelons que, sur un ensemble de cercles concentriques d’origine O et de rayons R, R, etc., le rapport de la longueur d’un arc de cercle, embrassé par deux demi-droites passant par O, sur son rayon est indépendant du rayon (Fig. A4.2a) ; ce rapport sans dimension est égal à l’angle u entre les deux demi-droites, exprimé en radians : u=

AB A B =  R R

B B µ O

Ω A

A

Σ

Σ

O

a)

b) F IG . A4.2.

a) Définition Considérons plusieurs sphères concentriques de centre O et un cône de sommet O (Fig. A4.2b). On appelle angle solide V le rapport invariant de l’aire de la calotte sphérique S, découpée sur une sphère par la surface du cône, sur le carré de son rayon :

 c Dunod – Toute reproduction non autorisée est un délit

V=

S S = 2 R2 R

Ce nombre sans dimension, est mesuré en stéradians. Pour une sphère entière V = 4p et pour une demi-sphère V = 2p. b) Angle solide élémentaire Le plus souvent, la surface définissant l’angle solide n’est pas sphérique, aussi est-il nécessaire d’exprimer l’angle solide élémentaire en fonction de la surface élémentaire interceptée d S. Considérons le cône élémentaire de la figure A4.3. D’après ce qui précède, l’angle solide élémentaire s’écrit : r·n d S d S cos u = = 3 dS 2 2 r r r d S étant la surface élémentaire découpée par le cône sur la sphère de centre O et de rayon r. Ainsi, l’angle solide est le flux du vecteur r/r3 à travers la surface interceptée : dV =

V= S

r · n dS = r3

où er désigne le vecteur unitaire porté par r.

S

er · n dS r2

618

Annexe 4.

L’angle solide s’exprime simplement en coordonnées sphériques par d V = sin u d u d w, car l’élément de surface sur une sphère de rayon r est r 2 sin u d u d w (cf. annexe 2). On retrouve bien, pour une sphère centrée en O : 2p

V=

0

p

dw

0

sin u d u = 4p θ

dΣ n r O

α

O

θ dS

F IG . A4.3.

a

C

ρ

F IG . A4.4.

c) Applications i) Angles solides définis par tout l’espace, un demi-espace, un quadrant et un disque Calculons l’angle solide sous lequel est vue, d’un point O, la portion de l’espace délimitée, en coordonnées sphériques, par u 1  u  u 2 et w 1  w  w 2 : u2

w2

V= u1

w1

sin u d u d w = (cos u 1 − cos u2 )(w2 − w1)

Si u1 = 0 et w 1 = 0, on obtient : – pour tout l’espace (u2 = p et w2 = 2p) : V = 4p, – pour un demi-espace (u 2 = p/2 et w2 = 2p) : V = 2p, – pour un quadrant (u 2 = p/2 et w2 = p/2) : V = p/2, – pour un disque (Fig. A4.4) (u 2 = u et w2 = 2p) : V = 2p(1 − cos u). ii) Angle solide défini par une surface fermée ne contenant pas l’origine Considérons le cône élémentaire issu de l’origine O qui découpe, sur une surface fermée, deux surfaces élémentaires orientées n1 d S1 et n2 d S 2, conformément à l’orientation d’une surface fermée. D’après ce qui précède, les angles solides définis par les surfaces n1 d S1 = −n1 d S 1 et n2 d S2 sont égaux puisque relatifs au même cône (Fig. A4.5). Par conséquent : d V2 = d V1 = − d V 1

et

d V1 + d V 2 = 0

Il en est de même pour tous les cônes élémentaires. Il en résulte que l’angle solide défini par une surface fermée, ne contenant pas l’origine O , est nul. n1 O

dS1

S

dS 2

d

n2

n1

A

V

F IG . A4.5.

n

F IG . A4.6.

I . 4 . — Divergence d’un vecteur. Formule d’Ostrogradsky La formule d’Ostrogradsky peut être considérée comme une relation de définition de la divergence d’un champ de vecteur. Elle relie le flux d’un champ de vecteur A, à travers une surface fermée S, à l’intégrale de sa divergence dans le volume délimité par S (Fig. A4.6) : S

A· n dS =

div A d

619

Flux et circulation de vecteur a) Expression de la divergence d’un champ de vecteur en coordonnées cartésiennes

Soit un champ de vecteur A, de composantes A x , Ay, A z dans une base cartésienne. Le flux de ce champ à travers une surface entourant un volume élémentaire d = d x d y d z, construit au point M(x, y, z), se met sous la forme (Fig. A2.5) : d F = Ax (x + d x, y, z) d y d z − A x(x, y, z) d y d z

+ Ay (x, y + d y, z) d z d x − A y(x, y, z) d z d x + Az (x, y, z + d z) d x d y − Az (x, y, z) d x d y

ce qui s’écrit aussi : dF =

∂Ax ∂A y ∂Az dx dy dz + dx dy dz + dx dy dz = ∂x ∂y ∂z

∂Ax ∂Ay ∂A z + + ∂x ∂y ∂z

d

Il en résulte que la divergence d’un champ de vecteur a pour expression en coordonnées cartésiennes : div A =

∂Ax ∂Ay ∂A z + + ∂x ∂y ∂z

b) Expression de la divergence d’un champ de vecteur en coordonnées cylindriques En coordonnées cylindriques, la divergence de A(r, w, z) a pour expression : div A =

1 ∂(rAr) 1 ∂Aw ∂Az + + r ∂r r ∂w ∂z

Pour le montrer, exprimons le flux du vecteur A à travers la surface fermée d S entourant le volume élémentaire d , en coordonnées cylindriques, (Fig. A2.6). On a : d F = Ar (r + d r, w, z)r d w d z − Ar(r, w, z)r d w d z

 c Dunod – Toute reproduction non autorisée est un délit

+ Aw(r, w + d w, z) d r d z − A w (r, w, z) d r d z + Az(r, w, z + d z) d r d z − A z(r, w, z) d r d z

ce qui s’écrit aussi : dF =

1 ∂(rA r) 1 ∂A w ∂A z ∂(rAr ) ∂Aw ∂Az + + dr dw dz+ dr dw dz+ rdr dw dz = d r ∂r r ∂w ∂z ∂z ∂r ∂w

Il en résulte, en identifiant le coefficient de d à div A, l’expression de la divergence en coordonnées cylindriques : 1 ∂(rAr) 1 ∂Aw ∂Az + + div A = r ∂r r ∂w ∂z c) Expression de la divergence d’un champ de vecteur en coordonnées sphériques En coordonnées sphériques, la divergence de A(r, u, w) s’écrit : div A =

∂ ∂ 1 ∂ 2 1 1 r Ar + Aw sin uAu + r2 ∂r r sin u ∂u r sin u ∂w

620

Annexe 4.

Montrons-le en exprimant le flux du vecteur A à travers la surface fermée d S entourant le volume élémentaire d , en coordonnées sphériques, (Fig. A2.7). On a : d F = Ar (r + d r, u, w)r2 sin u d u d w − Ar(r, u, w) r 2 sin u d u d w + Au (r, u + d u, w)r sin u d u d w − A u(r, u, w) r sin u d u d w + Aw (r, u, w + d w)r d r d u − A w(r, u, w)r d r d u

On obtient : dF = soit, puisque d

∂(sin u Au ) ∂(r2 A r) ∂Aw d r sin u d u d w + r dr dw + rdr du ∂u ∂w ∂r

= r2 sin u d r d u d w : dF =

1 ∂(r2Ar ) 1 ∂(sin u Au ) 1 ∂A w d + + r sin u r sin u ∂w r 2 ∂r ∂u

Il en résulte, en identifiant le coefficient de d à div A, l’expression de la divergence en coordonnées sphériques : 1 ∂(r2Ar ) 1 ∂(sin u Au ) 1 ∂Aw + + div A = 2 ∂u r sin u r sin u ∂w r ∂r d) Exemples (1) Calcul de div r div r = div(x ex + y ey + z e z) =

∂x ∂y ∂z + + =3 ∂x ∂y ∂z

On peut retrouver ainsi le volume d’une sphère, de rayon R. En effet, on a, d’après la formule d’Ostrogradsky : 1 1 1 = div r d = r ·n dS 3d = 3 3 3 S ce qui donne, pour une sphère, puisque n = er et r = R e r sur la sphère : =

1 3

S

R er · er d S =

R 3

S

dS =

4 R 4pR2 = pR3 3 3

(2) Calcul de div(r/r) = div(e r) div

r 1 ∂ 2 = 2 (r 2) = r r ∂r r

(3) Calcul de div(K/r), K étant un vecteur constant div

Kx e x + K ye y + K ze z ∂ Kx ∂ K + · · · = Kx = = div r r ∂x r ∂x ∂ (1/r) ∂r x = Kx + · · · = −K x 3 + · · · ∂r ∂x r

En regroupant les trois termes analogues, on trouve : div (K/r) = − K · r/r 3 .

1 + ··· r

621

Flux et circulation de vecteur

II . — CIRCULATION D’UN CHAMP DE VECTEUR II . 1 . — Définition La circulation d’un champ de vecteur A le long d’un contour fermé C est l’intégrale suivante : C

A·dr

soit

(Ax d x + Ay d y + A z d z) C

en explicitant le produit scalaire en coordonnées cartésiennes. II . 2 . — Formule de Stokes La formule de Stokes peut être considérée comme une relation de définition du rotationnel d’un champ de vecteur. Elle relie la circulation d’un champ de vecteurs A le long d’une courbe fermée C au flux de son rotationnel à travers une surface ouverte S qui s’appuie sur C (Fig. A4.7a) : C

A · dr =

S

rot A · n d S

Notons que si A peut se mettre sous la forme A = grad f , alors rot A = 0. La réciproque n’est vraie (rot A = 0 entraîne A = grad f ) que si certaines conditions sur le domaine U de l’espace affine sont satisfaites ; on montre que U doit être simplement connexe. Le tore est un exemple de domaine non simplement connexe. z M

S C

n

Q

N

P

O

dS

y

rot A

A

x a)

b)

 c Dunod – Toute reproduction non autorisée est un délit

F IG . A4.7.

a) Expression du rotationnel d’un champ de vecteur en coordonnées cartésiennes Soit un champ de vecteur A, de composantes A x , Ay , Az dans une base cartésienne, et un contour élémentaire rectangulaire MNPQ contenu dans un plan parallèle au plan Oxy et construit au point M(x, y, z) (Fig. A4.7b). En appliquant la relation de définition, on obtient, pour la composante suivant Oz : (rot A)z d x d y = Ax(x, y, z) d x + A y (x + d x, y, z) d y − Ax (x + d x, y + d y, z) d x − A y(x, y + d y, z) d y d’où :

∂Ay ∂Ax d x d y et dx dy + ∂y ∂x On déduit les autres composantes par permutation circulaire : (rot A) z d x d y = −

rot A =

∂Az ∂Ay − ∂y ∂z

ex +

∂Ax ∂Az − ∂z ∂x

(rot A) z =

ey +

∂Ay ∂A x − ∂x ∂y

∂Ay ∂Ax − ∂x ∂y

ez

622

Annexe 4. Exemple : Considérons le champ de vecteur A défini par : A = (8xy − 3z 2) e x + (4x 2 + 3z2) e y − 6z(x − y) e z

Le calcul donne rot A = 0 puisque : ∂[−6z(x − y)] ∂(4x 2 + 3z2 ) − = 6z − 6z = 0 ∂y ∂z et

∂(8xy − 3z2 ) ∂[−6z(x − y)] − = −6z − (−6z) = 0 ∂z ∂x

∂(4x 2 + 3z 2) ∂(8xy − 3z2) = 8x − 8x = 0 − ∂x ∂y

b) Expression du rotationnel d’un champ de vecteur en coordonnées cylindriques Calculons par exemple (rot A) z. Pour cela considérons un élément de surface MNPQ orthogonal à ez (Fig. A4.8). La circulation de A sur le contour MNPQ comporte quatre termes : N M Q P

P

A · d r = Ar (r, w, z) d r

N

A · d r = −Ar (r, w + d w, z) d r

A · d r = Aw(r + d r, w, z)(r + d r) d w

M Q

A · d r = −Aw(r, w, z)r d w

Puisque A · d r s’écrit : [Aw (r + d r, w, z)(r + d r) d w − Aw(r, w, z)r d w] − [A r (r, w + d w, z) d r − Ar (r, w, z)r d r] on trouve, d’après la définition : 1 r dr dw

(rot A)z = lim

dr→0 dw→0

MNPQM

A·dr

soit

(rot A) z =

1 ∂ 1 ∂Ar (rAw ) − r ∂r r ∂w

On calculerait de la même façon : (rot A)w =

∂Ar ∂Az − ∂r ∂z

et (rot A)r =

er

z

z

Q

M

dϕ M O

1 ∂Az ∂Aw − r ∂w ∂z

Q ρ dϕ



P θ

dρ N

O ϕ

y x

x F IG . A4.8.

P

N

y dϕ

F IG . A4.9.

623

Flux et circulation de vecteur c) Expression du rotationnel d’un champ de vecteur en coordonnées sphériques

Pour calculer la première composante (rot A) r = (rot A) · e r, considérons un élément de surface MNPQ orthogonal à er (Fig. A4.9). La circulation de A le long du contour MNPQ comporte quatre termes : N M

P

A · d r = Au(r, u, w)r d u

N

Q P

A · d r = Aw (r, u + d u, w)r sin(u + d u) d w

M

A · d r = −Au(r, u, w + d w)r d u

Q

A · d r = −A w(r, u, w) r sin u d w

Puisque A · d r s’écrit : [Aw(r, u + d u, w)r sin(u + d u) d w − A w(r, u, w)r sin u d w]−[A u (r, u, w + d w)r d u − Au (r, u, w)r d u] On a, d’après la définition : (rot A)r = lim

du→0 dw→0

1 du dw

r2 sin u

MNPQM

A·dr

soit (rot A) r =

∂ 1 1 ∂Au (A w sin u)− r sin u ∂u r sin u ∂w

On démontre, de façon analogue, que : (rot A)u =

1 ∂Ar 1 ∂(rAw ) − r sin u ∂w r ∂r

et (rot A)w =

1 ∂(rAu ) 1 ∂Ar − r ∂r r ∂u

III . — OPÉRATEURS DIFFÉRENTIELS DU SECOND ORDRE III . 1 . — Rotationnel d’un gradient Établissons que le rotationnel du gradient d’une fonction est toujours nul. Calculons par exemple la composante suivant Ox :

 c Dunod – Toute reproduction non autorisée est un délit

(rot grad f ) x =

∂ ∂ ∂ 2f ∂ 2f − (grad f ) z − (grad f ) y = =0 ∂y ∂z ∂ y∂ z ∂ z∂ y

d’où

rot grad f = 0

III . 2 . — Divergence du rotationnel Montrons que la divergence d’un rotationnel est toujours nulle : div rot A =

∂ ∂x

∂ ∂Az ∂Ay − + ∂y ∂z ∂y

∂ ∂Ax ∂A − z + ∂z ∂x ∂z

∂Ay ∂Ax − ∂x ∂y

d’où

div rot A = 0

III . 3 . — Laplacien Par définition, le laplacien d’une fonction scalaire f des coordonnées de l’espace est la divergence du vecteur grad f : Df = div grad f Par extension, on définit le laplacien d’un vecteur A par le vecteur dont les composantes sont les laplaciens des composantes Ax , A y, A z.

624

Annexe 4.

a) Laplacien en coordonnées cartésiennes À partir des expressions du gradient et de la divergence en coordonnées cartésiennes, on obtient : Df =

∂ ∂f ∂x ∂x

+

∂f ∂y

∂ ∂y

+

∂ ∂z

∂f ∂z

soit

Df =

∂2 f ∂ 2f ∂ 2f + + ∂x 2 ∂y 2 ∂z2

b) Laplacien en coordonnées cylindriques De même, à partir de la définition, explicitée en coordonnées cylindriques r, w, z, on obtient : Df =

∂f 1 ∂ r ∂r r ∂r

1 ∂ + r ∂w

∂ 1 ∂f + ∂z ∂w r

∂f ∂z

∂f 1 ∂ r = ∂r r ∂r

∂2f 1 ∂ 2f + 2 2 + 2 ∂z r ∂w

c) Laplacien en coordonnées sphériques Comme Df = div(grad f ), il vient, en utilisant les expressions de la divergence et du gradient en coordonnées sphériques r, u, w : Df =

1 ∂ r2 ∂r

r2

∂f ∂r

+

1 ∂f ∂ sin u 2 r sin u ∂u ∂u

+

∂2 f r2 sin2 u ∂w2 1

III . 4 . — Rotationnel du rotationnel d’un vecteur Calculons rot rot A, précisément sa composante suivant Ox. Il vient : (rot rot A) x = =

∂ ∂ ∂ (rot A) z − (rot A)y = ∂y ∂z ∂y ∂ ∂x

∂Ay ∂Az ∂A x − + + ∂y ∂z ∂x

∂Ay ∂A x − ∂x ∂y



∂ ∂z

∂ 2A x ∂ 2Ax ∂ 2 Ax + + ∂x2 ∂y2 ∂z2

∂Ax ∂Az − ∂z ∂x =

∂ (div A) − DAx ∂x

En généralisant ce résultat aux deux autres composantes, on trouve : rot rot A = grad div A − DA

IV . — RELATIONS D’ANALYSE VECTORIELLE IV . 1 . — Opérateur nabla On appelle opérateur nabla, noté ∇, l’opérateur différentiel que l’on définit, en coordonnées cartésiennes, par : ∂ ∂ ∂ ∇ = ex + ey + ez ∂x ∂y ∂z Cet opérateur permet d’exprimer formellement les opérateurs différentiels, gradient, divergence et rotationnel, selon : grad f = ∇f div A = ∇ · A rot A = ∇ × A

625

Flux et circulation de vecteur car : ∇f = ex ∇·A =

ex

∂ ∂ ∂ + ey + ez ∂x ∂y ∂z

∂f ∂f ∂f + ey + ez ∂x ∂y ∂z · (ex Ax + e y Ay + e z Az ) =

et (∇ × A)z = ∇x Ay − ∇y Ax =

∂Ax ∂Ay ∂Az + + ∂x ∂y ∂z

∂A y ∂Ax − ∂x ∂y

On peut alors retrouver symboliquement les opérateurs du second ordre : rot grad f = ∇ × ∇f = 0

div rot A = ∇ · (∇ × A) = 0

div grad f = ∇ · ∇f = ∇ 2f = Df

et : rot rot A = ∇ × (∇ × A) = ∇(∇ · A) − ∇2A = grad div A − DA IV . 2 . — Formules d’analyse vectorielle a) grad (f1 f2) = f 1 grad f2 + f2 grad f1 En effet : grad(f 1f2) = ∇(f 1f2 ) = f1 ∇f2 + f2 ∇f1 = f1 grad f2 + f2 grad f 1 Cette formule permet de calculer grad(p · r), p étant un vecteur uniforme ; en effet, développons le produit scalaire : grad(p · r) = grad(p x x + pyy + p z z) = pxex + x grad p x + · · · = p On en déduit aisément, si l est une constante réelle, grad(lf ) = l grad f . b) div(f A) = f div A + grad f · A En effet :

 c Dunod – Toute reproduction non autorisée est un délit

div(f A) =

∂ ∂ ∂ (fA x) + (fAy) + (fA z) ∂x ∂y ∂z

soit : div(f A) =

∂f ∂f ∂f Ax + Ay + Az + f ∂x ∂y ∂z

∂Ax ∂A y ∂Az + + ∂x ∂y ∂z

= A · grad f + f div A

Cas particulier : Si f = l (constante réelle), alors div(lA) = l div A. c) rot (f A) = f rot A + grad f × A

Calculons par exemple la composante suivant Ox de rot(f A) :

[rot(f A)] x =

∂ ∂ ∂A z ∂Ay − (fAz ) − (fAy ) = f ∂y ∂z ∂y ∂z

+ Az

∂f ∂f − A y = f (rot A) x + (grad f × A)x ∂y ∂z

En généralisant aux deux autres composantes, on trouve la formule annoncée. Dans le cas où f = l (constante réelle), rot(lA) = l rot A.

626

Annexe 4.

d) div(A × B) = B · rot A − A · rot B

Établissons cette relation : ∂ ∂ ∂ div(A × B) = (A y Bz − AzB y ) + (A zB x − AxB z) + (Ax By − Ay Bx) ∂x ∂y ∂z ∂Ay ∂Az ∂A y ∂Ax ∂Az ∂Ax − − − + By + Bz ∂y ∂z ∂z ∂x ∂x ∂y ∂B z ∂By ∂Bx ∂Bz ∂By ∂Bx − Ax − Ay − Az − − − ∂y ∂z ∂z ∂x ∂x ∂y

= Bx

Par conséquent : div(A × B) = B x (rot A)x + By (rot A)y + Bz (rot A)z − A x(rot B)x − A y (rot B)y − Az(rot B) z soit : div(A × B) = B · rot A − A · rot B Remarque : L’opérateur nabla doit être employé avec précaution. Certaines formules vectorielles ne sont pas obtenues simplement ; par exemple, grad(A · B) n’est pas égal à B div A + A div B, mais à : grad(A · B) = (A · grad)B + (B · grad)A + A × rot B + B × rot A

où (A · grad) = (A · ∇) = A x ∂/∂x + Ay ∂/∂y + A z ∂/∂z. IV . 3 . — Intégrales vectorielles Proposons-nous d’établir les deux intégrales vectorielles suivantes : grad f d

= S

fn dS

rot A d

et

= S

(n × A) d S

Pour cela, introduisons le champ de vecteur uniforme quelconque K (div K = 0 et rot K = 0). Dans le premier cas, on a, puisque div(f K) = f div K + K · grad f = K · grad f : K·

grad f d

K · grad f d

=

div(f K) d

=

En appliquant la formule d’Ostrogradsky, on obtient : K · grad f d

= S

fK · n dS = K ·

S

fn dS

d’où la première formule, puisque K est un vecteur uniforme quelconque. Dans le second cas, il vient, puisque div(A × K) = K · rot A − A · rot K = K · rot A : K · rot A d

=

div(A × K) d

= S

(A × K) · n d S

en appliquant la formule d’Ostrogradsky. Il en résulte : K·

rot A d

=K·

S

(n × A) d S

Comme cette équation est vraie, quel que soit K, la seconde formule en découle.

Annexe 5 Simulation en mécanique La simulation numérique présente un intérêt considérable en mécanique en raison de la complexité des équations du mouvement d’un système : ces dernières impliquent parfois des non linéarités et plusieurs degrés de liberté. C’est Poincaré qui montra, au début du XX e siècle, que les équations du mouvement non linéaires et/ou impliquant au moins deux degrés de liberté étaient plus répandues qu’on ne le pensait avant lui. Dans ce contexte d’une mécanique enrichie par la grande variété des systèmes non linéaires, l’ordinateur est un outil précieux pour connaître la dynamique d’un système mécanique. De nos jours, il est indispensable pour le développement de la science, dans les laboratoires universitaires, mais aussi pour son enseignement puisqu’il permet d’éviter des calculs fastidieux et ainsi rendre la transmission plus agréable et donc plus efficace. Les simulations numériques ont été réalisées avec le logiciel M ATLAB pour deux raisons : d’une part ce logiciel est très répandu dans l’enseignement et la recherche, d’autre part il permet une écriture à la fois simple et proche de celle utilisée en physique. Le lecteur pourra télécharger sur son ordinateur l’ensemble des algorithmes présentés dans cette annexe à l’adresse http://www-loa.univ-lille1.fr/~pujol. On commence par analyser les changements d’échelle en physique en s’appuyant sur l’exemple d’une planète dont on modifie les paramètres pertinents, comme la masse ou la distance qui la sépare de son étoile. On étudie ensuite l’interaction forte entre nucléons à partir de la discussion qualitative que permet la conservation de l’énergie mécanique pour un tel système à un seul degré de liberté. On analyse aussi l’influence de la force de frottement visqueux (non linéaire) qu’exerce l’atmosphère sur le mouvement d’un satellite terrestre. On poursuit avec le tracé des portraits de phase d’un oscillateur à un seul degré de liberté soumis à divers types de force de frottement. Enfin, on montre les trajectoires variées de la Machine d’Atwood Dansante (MAD), laquelle se présente comme un pendule non linéaire à deux degrés de liberté qui évolue dans le champ de pesanteur ; ce système présente, sous certaines conditions, un comportement chaotique.

I . — SIMILITUDE ET LOI D’ÉCHELLE Pour illustrer les changements d’échelle en mécanique, on étudie la modification de la période de révolution T d’une planète, en orbite autour de son étoile, lorsqu’on change les paramètres pertinents comme les masses ou les distances. Naturellement, le système Soleil-Terre est la référence : la distance moyenne entre ces deux astres est d’environ 149, 6 × 106 km, T ≈ 12 mois et MS ≈ 1, 99 × 10 30 kg est la masse du Soleil.

628

Annexe 5

Sur le plan informatique, il est plus intéressant, pour calculer T, de résoudre les équations générales du mouvement qui tiennent compte de la variation de la distance r entre la planète et son étoile, au cours du mouvement, en prenant comme conditions initiales celles qui correspondent à un mou˙ vement circulaire : r˙ (0) = 0 et v(0) = r 0u(0) = (GMS /r0 )1/2 , où G est la constante de Newton et r0 = r(0). En outre, on adopte dans toutes les simulations u(0) = 0. Établissons tout d’abord les équations générales du mouvement. I . 1 . — Équations du mouvement d’une planète Le mouvement orbital d’une planète autour du Soleil, par rapport au référentiel héliocentrique, est donné par les équations suivantes (cf. chapitre 12) : ¨r − r ˙u2 = −G

MS r2

r¨ u + 2r˙u˙ = 0

et

Remarque : Si r est constant (˙r = ¨ r = 0), on retrouve les équations du mouvement circulaire : u˙ = −GM S/r 3 = Cte et donc ¨ u = 0. I . 2 . — Changement d’échelle Effectuons un changement d’échelle en multipliant la masse par un facteur a, les distances par un facteur b et le temps par un facteur g ; évidemment, ces trois facteurs sont positifs : M S → aM S

r → br

t → gt

On obtient alors : MS bg−2 ¨r − r ˙u2 = ab−2 −G 2 r soit ¨r − r u˙ 2 = ag 2b−3 −G

MS r2

et

et

bg −2 r¨ u + 2 ˙ru˙ = 0

ru¨ + 2 ˙ru˙ = 0

On voit que les équations du mouvement sont invariantes par le changement d’échelle si ag2 b−3 = 1. Ainsi, en choisissant une valeur pour deux de ces paramètres, celle du troisième est nécessairement fixée. I . 3 . — Résolution numérique et calcul de la période a) Résolution numérique des équations du mouvement Numériquement, les équations du mouvement, qui font intervenir deux degrés de liberté r et u, sont introduites dans la fonction odegravitation et résolues par la fonction gravitation qui utilise la méthode ode45 de MATLAB. La solution apparaît dans les variables numériques t et z, où la première est le temps t et la seconde regroupe, dans l’ordre suivant, les variables numériques ˙ ˙ u et u. r, dr, theta et dtheta, qui représentent respectivement les variables physiques r, r, ˙ Les conditions initiales r(0), r˙ (0), u(0) et u(0) sont données, respectivement, par les variables numériques r0, dr0, theta0 et dtheta0 que l’on a regroupées, dans cet ordre, dans la variable numérique z0.

Simulation en mécanique

629

Remarques : 1) Il est nécessaire que les deux fonctions, odegravitation et gravitation soient écrites comme indiqué, c’est-à-dire de façon imbriquée. En effet, si ce n’était pas le cas, les paramètres alpha, beta et gamma, qui sont essentiels pour la fonction odegravitation, ne seraient pas connus par cette dernière au moment de son appel par ode45. 2) L’instruction gamma=beta ˆ (3/2) se justifie par l’écriture des termes de dérivée tem˙ ¨ ˙ ¨ porelle r, u et u dans les équations du mouvement. En effet, lorsqu’on change r, d’échelle de distance, on doit tenir compte de l’invariance des équations du mouvement (ag 2b −3 = 1). Si a = 1, on reconnaît en g 2 b−3 = 1 une autre écriture de la troisième loi de Kepler (cf. chapitre 12). function [r dr theta dtheta]=gravitation(t,z0,alpha,beta) gamma=alpha*beta^(3/2); [t,z]=ode45(@odegravitation,t,z0); r=z(:,1); theta=z(:,3); dr=z(:,2); dtheta=z(:,4); clear z function dz=odegravitation(t,z) G = 6.67E-11; % Constante de Newton MS = 1.99E+30; % Masse du Soleil dz=zeros(size(z)); % Initialisation r =z(1); theta =z(3); dr=z(2); dtheta=z(4); %% Equations du mouvement d2r = r.*(dtheta.^2)-(G*MS./(r.^2))*alpha*gamma^2/beta^3; d2theta = -2*dr.*dtheta./r; dz(1)=dr; dz(3)=dtheta; dz(2)=d2r; dz(4)=d2theta; end end

 c Dunod – Toute reproduction non autorisée est un délit

b) Calcul numérique de la période Une fois les équations du mouvement résolues, le calcul de la période de révolution T de la planète s’effectue en déterminant l’instant t pour lequel u = 2p, ce qui correspond à une révolution complète. Comme numériquement on ne passe jamais rigoureusement par 2p, on estime la valeur de T de la façon suivante : i) on repère, dans la variable t, les deux instants successifs pour lesquels theta encadre 2p ; ii) on affine la précision en interpolant entre ces deux instants, à l’aide de la fonction polyfit de MATLAB ; iii) on estime la valeur de l’interpolation en 2p avec la fonction polyval de MATLAB, ce qui nous donne une bonne approximation de T, que l’on exprime en mois. C’est la fonction perioderevolution qui permet de calculer T. Retrouvons la valeur bien connue d’environ 12 mois pour la Terre autour du Soleil ; pour cela écrivons : G = 6.67E-11; MS = 1.99E+30; t = linspace(0,365*86400,365); r0 = 149.6E+09;dtheta0 = sqrt(G*MS/r0^3); alpha=1; beta=1; z0 = [beta*r0 0 0 sqrt(alpha/beta^3)*dtheta0];

630

Annexe 5

[r dr theta dtheta]=gravitation(t,z0,alpha,beta); T=perioderevolution(t,theta); figure(1) graphetheta;hold all

on obtient bien T=12.0078. Pour tracer l’évolution de u, on utilise la commande graphetheta, qui fournit la figure A5.1. 6.28

40

Terre−Soleil Proxima du Centaure Sirius

4.72

30 T (mois)

θ (rad)

Terre−Soleil Proxima du Centaure Sirius −1/2 α

35

3.14

25 20 15

1.57

10 0 0

2

4

6 t (mois)

8

10

12

F IG . A5.1. — Évolution de l’angle polaire du centre de la Terre autour du Soleil, de Proxima du Centaure ou de Sirius

5 0

0.5

1

α

1.5

2

2.5

F IG . A5.2. — Tracé de la période de révolution T en fonction de la valeur de a

I . 4 . — Et si la Terre tournait autour d’une autre étoile ? Comparons la période de révolution de la Terre, en orbite autour du Soleil, à celle que l’on obtiendrait si la Terre était en orbite autour d’une autre étoile, à une même distance r 0 ≈ 149, 6 × 106 km. Il suffit de modifier la masse, les distances et les durées étant inchangées : a = 1 et b = g = 1. Illustrons par deux exemples. i) La Terre est en orbite autour de l’étoile Proxima du Centaure, située dans la constellation du Centaure à 4, 22 al (année-lumière) de nous, de masse M = 0, 123 MS (a = 0, 123). ii) La Terre est en orbite autour de l’étoile Sirius, dans la constellation du Grand Chien à 8, 6 al de nous, de masse M = 2, 02 MS (a = 2, 02). On résout alors deux fois les équations du mouvement et on complète la figure A5.1 selon : alpha=0.123; beta=1; z0 = [beta*r0 0 0 sqrt(alpha/beta^3)*dtheta0]; [r dr theta dtheta]=gravitation(t,z0,alpha,beta); T=perioderevolution(t,theta); graphetheta;hold all alpha=2.02; beta=1; z0 = [beta*r0 0 0 sqrt(alpha/beta^3)*dtheta0]; [r dr theta dtheta]=gravitation(t,z0,alpha,beta); T=perioderevolution(t,theta); graphetheta; h=legend(’Terre-Soleil’,’Proxima du Centaure’,’Sirius’,’Location’, ’best’); set(h,’Fontsize’,11)

631

Simulation en mécanique

On trouve pour les périodes, respectivement T = 34.2381 et T = 8.4486, soit environ 34,2 mois et 8,5 mois. La période de révolution décroît lorsqu’on augmente la masse de l’étoile, ce que l’on constate sur la figure A5.1. Précisément, T varie comme a−1/2 , ce qui était prévisible, car l’invariance des équations du mouvement par ce changement d’échelle impose : ag 2b−3 = 1 soit g = a−1/2

puisque

b=1

C’est ce que montre la figure A5.2 obtenue selon la procédure suivante : figure(2) plot(1,12.0078,’ko’,0.123,34.2381,’ks’,2.02,8.4486,’kd’); xlabel(’$\alpha$’,’Interpreter’,’Latex’,’Fontsize’,14); ylabel(’$T$ (mois)’,’Interpreter’,’Latex’,’Fontsize’,14); set(gca,’Fontsize’,14) grid on;box on;hold on alpha=linspace(0.1,2.5,100); T=(2*pi*r0^(3/2)/(G*MS)^(1/2))/86400/(365/12)*alpha.^(-1/2); plot(alpha,T,’k-’); h=legend(’Terre-Soleil’,’Proxima du Centaure’,’Sirius’, ’\alpha^{-1/2}’); set(h,’Fontsize’,12)

I . 5 . — Période de révolution des planètes du système solaire Calculons la période de révolution des planètes du système solaire, la Terre servant ici aussi de référence. Dans ce cas, la masse est inchangée puisqu’on conserve la même étoile, le Soleil, mais la distance à ce dernier change d’une planète à une autre. Ainsi, a = 1 et b = 1. Pour les sept autres planètes du système solaire, Mercure, Vénus, Mars, Jupiter, Saturne, Uranus et Neptune, on a respectivement : b = 0, 38

b = 0, 72

b = 1, 52

b = 5, 20

b = 9, 54

b = 19, 22 et

b = 30, 06

6.28

6.28

4.72

4.71 θ (rad)

θ (rad)

 c Dunod – Toute reproduction non autorisée est un délit

C’est en écrivant SystSol que l’on obtient le résultat et l’évolution de u. Pour plus de clarté, on a distingué, dans les figures, les planètes telluriques, Mercure, Vénus, la Terre et Mars (Fig. A5.3), des planètes gazeuses que sont Jupiter, Saturne, Uranus et Neptune (Fig. A5.4).

3.14 Terre Mercure Venus Mars

1.57

0 0

2

4

6 t (mois)

8

10

Jupiter Saturne Uranus Neptune

3.14

1.57

12

F IG . A5.3. — Évolution de l’angle polaire des planètes telluriques du système solaire

0 0

2

4

6 t (mois)

8

10

F IG . A5.4. — Évolution de l’angle polaire des planètes gazeuses du système solaire

12

632

Annexe 5

Ainsi, Mercure et Vénus ont une période de révolution plus courte que celle de la Terre, dans les rapports 4 et 1,5 environ, respectivement T ≈ 2, 9 mois et T ≈ 7, 4 mois. Les autres planètes ont une période plus longue : Mars parcourt un peu plus de la moitié de son orbite en une année terrestre, puisque T = 22, 6 mois, alors que Jupiter, Saturne, Uranus et Neptune en font moins que le huitième, respectivement : T = 142, 4 mois, T = 353, 4 mois, T = 1009 mois et T = 1 970 mois, On retrouve bien l’accroissement de la période lorsqu’on augmente la distance : T varie comme b , conformément à ce qu’impose l’invariance des équations du mouvement par changement d’échelle : a = 1 et ag 2 b−3 = 1 impliquent g = b 3/2 3/2

C’est ce que montre la figure A5.5 obtenue de la façon suivante : figure(5) for k=1:length(beta) plot(beta(k),T(k),’ko’); hold(’on’); hold on end xlabel(’$\beta$’,’Interpreter’,’Latex’,’Fontsize’,14); ylabel(’$T$ (mois)’,’Interpreter’,’Latex’,’Fontsize’,14); set(gca,’Fontsize’,14) grid on;box on; beta=linspace(0.1,35,100); T=(2*pi*r0^(3/2))/sqrt(G*MS)/86400/(365/12)*beta.^(3/2); plot(beta,T,’k-’); h=legend(’Planetes’,’\beta^{3/2}’,’location’,’best’); set(h,’Fontsize’,11) 2500

6.28

Planetes β

3/2

2000

1500

θ (rad)

T (mois)

4.71

1000

51 Pegasi b HD 149026 b HD 4308 b

1.57

500

0 0

3.14

10

20 β

30

40

F IG . A5.5. — Période de révolution T d’une planète autour du Soleil en fonction de b

0 0

5

10

15 t (jours)

20

25

30

F IG . A5.6. — Évolution de l’angle polaire de trois exoplanètes

I . 6 . — Couple exoétoile - exoplanète Pour un couple exoétoile - exoplanète, les facteurs a, b et g sont tous différents de l’unité. Calculons, par exemple, la période de révolution des trois exoplanètes suivantes : i) La planète 51 Pegasi b qui orbite autour de l’étoile 51 Pegasi, située dans la constellation de Pégase à environ 51 al de nous. Cette planète est à une distance r de son étoile qui est égale à 0, 05 fois la distance Terre-Soleil ; la masse M de l’étoile est 1, 11 fois celle du Soleil : b = 0, 05 et a = 1, 11.

633

Simulation en mécanique

ii) La planète HD149026 b qui orbite autour de l’étoile HD149026, située à 78, 9 pc (parsec) de nous. Elle est à une distance r de son étoile qui est égale à 0, 042 fois la distance Terre-Soleil ; la masse M de l’étoile est 1, 30 fois celle du Soleil : b = 0, 042 et a = 1, 30. iii) La planète HD4308 b qui orbite autour de l’étoile HD4308, située à 21, 9 pc de nous. Elle est à une distance r de son étoile qui est égale à 0, 114 fois la distance Terre-Soleil ; la masse M de l’étoile est 0, 83 fois celle du Soleil : b = 0, 114 et a = 0, 83. La commande Exoplanete donne les périodes de révolution de ces exoplanètes, ainsi que la figure A5.6. On trouve des périodes de révolution proches de 4, 3 et 15 jours respectivement. Remarque : Comme la période de révolution est, pour ces exoplanètes, de l’ordre de quelques jours, les abscisses de la figure A5.6 sont exprimées en jours. Pour cela, l’utilisateur doit intervenir dans la fonction graphetheta, d’une part en supprimant les commentaires des lignes de commande correspondantes et, d’autre part, en neutralisant par des commentaires celles relatives à l’expression des durées en mois. On vérifie bien que T est proportionnel à 2

ag b

−3

=1

b3/a

implique

1/2

:

g=

b3 a

1/2

II . — INTERACTION FORTE ENTRE NUCLÉONS L’interaction forte, qui est la plus intense des quatre interactions fondamentales, est celle qui explique la cohésion entre les nucléons (protons et neutrons) formant les noyaux atomiques ; on la décrit généralement par l’énergie potentielle radiale suivante : K r Ep = exp − r a où K et a sont deux constantes. La première est négative en raison du caractère attractif de l’interaction, et homogène au produit d’une énergie par une distance ; la seconde, qui a la dimension d’une longueur, est la portée de l’interaction.

 c Dunod – Toute reproduction non autorisée est un délit

Le système formé de deux nucléons étant conservatif, son énergie mécanique E m , somme de son énergie cinétique E k et de son énergie potentielle Ep , est une constante. On se propose d’analyser le comportement d’un tel système à partir de la discussion qualitative que permet la conservation de l’énergie mécanique pour un système à un seul degré de liberté. II . 1 . — Énergie potentielle effective On sait que l’évolution de la variable radiale r est caractérisée par une énergie potentielle effective, reliée à Ep , d’expression (cf. chapitres 12 et 13) :

L2 K r L2 − = + exp 2mr2 2mr2 r a où L est la valeur constante du moment cinétique, dans le référentiel du centre de masse du système, et m la masse réduite directement reliée aux masses des corps en interaction. Si on introduit la longueur p 1 = L 2 /(m|K |), la distance réduite R = r/p 1 et la portée réduite A = a/p1 , on obtient : E p,ef = Ep +

Ep,ef =

|K | p1 r p2 R 1 − exp − + 12 = E 0 − exp − p1 r a R A 2r

+

1 2R2



E0 =

|K | p1

634

Annexe 5

d’où, l’énergie potentielle effective réduite, notée Er pour simplifier l’écriture informatique : Er =

R Ep,ef 1 1 = − exp − + 2 R A E0 2R

On calcule cette énergie potentielle effective réduite à l’aide de la fonction energie. De même, le calcul de la dérivée de l’énergie potentielle effective réduite par rapport à R, notée dErdR, laquelle est essentielle pour l’analyse la fonction Er (R), s’effectue à l’aide de la fonction denergiedR. II . 2 . — Graphes de l’énergie potentielle effective réduite et de sa dérivée Sur la figure A5.7, on a représenté les graphes de E r et d E r/ d R, en fonction de R, pour trois valeurs très différentes de la portée réduite A. a) A = 10 000 La portée de l’interaction est très grande. Rappelons que les interactions newtonienne ou coulombienne ont une portée infinie (A = ∞). On obtient alors le graphe en définissant aussi un intervalle de valeurs pour R (tracé logarithmique), et en utilisant la commande grapheforte : A = 10000;R = logspace(-2,2,100); figure(1) grapheforte(A,R,energie(R,A));axis([0.1 max(R) -1 1]); figure(2) grapheforte(A,R,denergiedR(R,A));axis([0.1 max(R) -1 1]);

L’énergie potentielle effective réduite passe par un minimum négatif qui vaut −0, 5 pour R ≈ 1 et s’annule à l’infini (Fig. A5.7a) ; la dérivée, elle, passe par un maximum et s’annule à grande distance (Fig. A5.7b). Remarque : En raison de la présence d’une exponentielle dans l’expression de E r , et donc aussi de d E/ d R, il peut être nécessaire d’agrandir sur la figure obtenue. Pour cela, on redéfinit les limites des axes en écrivant, comme pour les figures A5.7a et A5.7b par exemple, axis([min(R) max(R) -1 1]). Une fois la figure obtenue, l’utilisateur peut utiliser la commande axis comme il le souhaite. b) A = 2 Dans ce cas, un minimum apparaît encore pour R ≈ 1, mais la faible portée de l’interaction induit une valeur maximale positive de l’énergie potentielle effective réduite quand R ≈ 7. Le minimum vaut environ −0, 1, alors que le maximum avoisine 0 (Fig. A5.7c). Concernant la dérivée, le maximum est plus prononcé et il apparaît un minimum pour R ≈ 9 (Fig. A5.7d). A = 2; figure(3) grapheforte(A,R,energie(R,A));axis([0.1 max(R) -0.2 0.2]); figure(4) grapheforte(A,R,denergiedR(R,A));axis([1 max(R) -0.02 0.08]);

Déterminons les deux valeurs de R pour lesquelles E r = 0. Pour cela, on utilise la fonction fzero de MATLAB ; on lui donne, dans l’ordre, la variable numérique R, l’énergie réduite E et une valeur proche de la valeur de R recherchée. On a donc, pour les deux valeurs de R, respectivement R1 et R2 :

635

Simulation en mécanique

0.5

0.5 dE r/dR

1

Er

1

0

−0.5

−1 −1 10

0

−0.5

10

0

R

10

1

10

−1 −1 10

2

0

1

10

a)

10

R

2

10

b) 0.08

0.2 0.15

0.06 0.1

dEr /dR

Er

0.05 0

−0.05

0.04

0.02

−0.1

0 −0.15 −0.2 −1 10

0

10

10

1

−0.02 0 10

2

10

R

1

10

d) 0

8

−2

6

−4

dEr /dR

10

Er

 c Dunod – Toute reproduction non autorisée est un délit

c)

4

−6

−8

2

0 −1 10

2

10 R

0

10

1

R

e)

10

10

2

−10 −1 10

0

1

10

10

2

10

R

f)

F IG . A5.7. — Graphes de Er et d Er/ d R pour A = 10 000 (a et b), A = 2 (c et d) et A = 0, 01 (e et f)

636

Annexe 5

R1 = fzero(@(R) energie(R,A),0.7); [R1,energie(R1,A)] ans = 0.7148 0.0000 R2 = fzero(@(R) energie(R,A),4); [R2,energie(R2,A)] ans = 4.3066 0.0000

De la même manière, recherchons les deux valeurs finies de R qui réalisent d E r/ d R = 0, c’està-dire celles qui donnent les deux extréma de l’énergie potentielle effective réduite : R1 = fzero(@(R) denergiedR(R,A),1); ans = 1.1228 0 -0.1114 R2 = fzero(@(R) denergiedR(R,A),5); ans = 6.7938 0 0.0059

[R1,denergiedR(R1,A),energie(R1,A)] [R2,denergiedR(R2,A),energie(R2,A)]

c) A = 0, 01 La portée est dans ce cas si faible qu’il n’y a plus de valeur minimale non nulle (Fig. A5.7e). Quant à la dérivée, elle augmente continûment jusqu’à une valeur maximale nulle (Fig. A5.7f). A = 0.01; figure(5) grapheforte(A,R,energie(R,A));axis([0.1 max(R) 0 10]); figure(6) grapheforte(A,R,denergiedR(R,A));axis([0.1 max(R) -10 0]);

II . 3 . — Influence de la portée Il est instructif de connaître l’influence du terme de portée A sur les deux valeurs de R qui réalisent l’annulation de Er et sur les deux autres qui annulent d E r / d R. a) Valeurs de R qui annulent Er

Pour déterminer la première valeur de R pour laquelle E r = 0, on procède comme suit : i) on part de la valeur A = 10 4 pour laquelle R 1 ≈ 0, 01 d’après la figure A5.7a ;

ii) on estime R1 de proche en proche, en diminuant A de 1% à chaque itération. On utilise pour cela la commande while associée à la condition flag==1 : si, à l’itération numéro k, fzero ne trouve pas une valeur de R1 qui annule Er, alors la variable numérique flag prend une valeur différente de 1 et le processus itératif s’arrête. iii) On représente finalement le graphe R 1(A). Ces instructions se trouvent dans la fonction portee ; en écrivant [A,Rsol,Esol]=portee(A,val,R), où Rsol, Esol et val représentent respectivement R1, E r(R 1 ) et la valeur approchée de R 1 , puis en tapant figure(7) ;grapheportee, on obtient le tracé de la figure A5.8 qui représente R 1 en fonction de A. On obtient la seconde valeur R 2 qui annule Er en procédant de la même façon : on part de la valeur A = 2 pour laquelle R 2 est de l’ordre de 4 d’après la figure A5.7c ; on estime R 2 de proche en proche en augmentant A de 1% à chaque itération et en imposant une limite supérieure A = 20 ; on écrit pour cela [A,Rsol,Esol]=portee(A,val,R) ; on obtient le tracé graphique avec figure(8) ;grapheportee ;hold on ; on complète le graphe vers les faibles valeurs de A en recommençant le procédé mais, cette fois, en diminuant A de 1% à chaque itération, d’où [A,Rsol,Esol]=portee(A,val,R) ;grapheportee. La figure A5.9 représente le graphe correspondant R2 (A).

637

Simulation en mécanique

1.3

120

1.2 100

1.1 80 R2

R1

1 0.9 0.8

60 40

0.7 20

0.6 0.5 0 10

1

10

2

10 A

10

3

10

0 0 10

4

F IG . A5.8. — Valeurs de R 1 qui annulent Er en fonction de A

10 A

1

10

2

F IG . A5.9. — Valeurs de R 2 qui annulent Er en fonction de A

II . 4 . — Valeurs de R qui annulent d Er / d R

La procédure de détermination des valeurs de R qui annulent la dérivée d E r / d R est analogue à la précédente. On écrit, en partant de A = 104, la valeur pour laquelle R1 est de l’ordre de 1 d’après la figure A5.7b : [A,Rsol,Esol]=portee(10000,1,R); figure(9) grapheportee

Le graphe R 1 (A) est représenté sur la figure A5.10. Pour R 2, on écrit en partant de A = 2, la valeur pour laquelle R 2 est de l’ordre de 7 d’après la figure A5.7d, en suivant un processus itératif croissant puis un second décroissant : [A,Rsol,Esol]=portee(2,7,R); figure(10) grapheportee hold on [A,Rsol,Esol]=portee(2,7,R); grapheportee

150

2

1.5

100 1

R2

R1

 c Dunod – Toute reproduction non autorisée est un délit

Le graphe R 2 (A) est représenté sur la figure A5.11.

0.5

50 0

−0.5 0 10

1

10

2

10 A

3

10

4

10

F IG . A5.10. — Valeurs de R1 qui annulent d Er / d R en fonction de A

0 0 10

10 A

1

2

10

F IG . A5.11. — Valeurs de R2 qui annulent d E r / d R en fonction de A

638

Annexe 5

III . — SATELLITES DANS L’ATMOSPHÈRE Le mouvement d’un satellite terrestre est affecté par la force de frottement visqueux qu’exerce l’atmosphère sur lui ; précisément, il se rapproche de la Terre et sa vitesse augmente (cf. chapitre 12). Analysons cette influence à l’aide de l’outil informatique. III . 1 . — Analyse quantitative du problème Deux types de force de frottement visqueux F f sont souvent utilisés (cf. chapitres 4 et 31) : le premier, proportionnel à la vitesse, F f ,S = −a v, dit de Stokes, convient si la vitesse est suffisamment faible ; le second, proportionnel au carré de la vitesse, Ff ,V = −bv 2 v/v, dit de Venturi, est adapté pour des vitesses élevées ; aussi est-ce cette force que nous considèrerons dans cette illustration informatique. Appliqué au satellite (masse m), en mouvement orbital autour de la Terre, à une distance r de son centre, le théorème de l’énergie mécanique donne : d Em = P (nc) dt

avec

mv2 GMT m GMT m − =− Em = Ek + Ep = 2 2r r

et P (nc) = Ff · v

G étant la constante de Newton et MT la masse de la Terre. III . 2 . — Force de frottement de Venturi avec b constant Si la force de frottement est de type Venturi, l’équation du mouvement s’écrit : d dt



GMT m 2r

= −bv 3 ≈ −b

3/2

GM T r

car v2 ≈ GM T /r (cf. chapitre 12), ce qui donne, en simplifiant, avec une bonne approximation : d dt

1 r

2b = GMT m

GMT r

3/2

Le coefficient b est relié à la masse volumique r de l’air atmosphérique et à la surface S du satellite normale à sa vitesse par l’équation (cf. chapitre 31) : b = C x rS/2 où Cx est un facteur de forme qui dépend de l’écoulement de l’air autour du satellite. Cette équation différentielle non linéaire s’écrit, en introduisant la variable u = 1/r : du = k u3/2 dt

en posant k =

2b(GMT )1/2 m

soit

du = k dt u 3/2

Il vient alors, par intégration, si b est une constante bi et donc k = ki : −2u −1/2 = −2r 1/2 = k i t + Cte

soit

r1/2 = −

ki t + r1i /2 2

où ri est la valeur prise par r à l’instant pris comme origine.

avec

ki =

2b i(GMT )1/2 m

639

Simulation en mécanique III . 3 . — Force de frottement de Venturi avec b variable a) Équation du mouvement

L’analyse concrète de l’influence de l’atmosphère sur le mouvement d’un satellite montre que l’hypothèse d’une valeur constante de b, et donc de k, n’est pas satisfaisante, car r varie avec r selon une exponentielle décroissante : r = ri exp −ai (r − r i) . Ainsi : k = ki exp −ai (r − ri )

avec

ki =

2b i (GMT ) 1/2 ri SC x(GM T ) 1/2 = m m

L’équation différentielle à résoudre est donc la suivante : d dt

1 r

= k i exp −a i (r − ri ) r −3/2

soit

dr = −ki exp −ai (r − r i ) r 1/2 dt

b) Résolution numérique Plaçons-nous dans le cas concret du satellite SPOT, de masse 1, 8 tonne, d’altitude initiale 832 km, pour lequel SCx = 34 m2 . En outre, les coefficients a i et ri valent respectivement 1, 5×10 −5 m −1 et 8, 55 × 10−15 kg.m −3. Toutes ces valeurs sont utilisées par la fonction satellite qui met en œuvre l’équation différentielle du premier ordre précédente et retourne, à chaque instant t, dans la variable dr, la valeur de la dérivée d r/ d t, function dr=satellite(t,r) G=6.67E-11;Mt=6E24;Rt=6400E3;SCx=34;m=1800;Hi=832E3; ri=Rt+Hi;alphai=1.5E-5;rhoi=8.55E-15; kappai=rhoi*SCx*sqrt(G*Mt)/m; dr = -kappai*exp(-alphai*(r-ri)).*sqrt(r);

 c Dunod – Toute reproduction non autorisée est un délit

L’intégration numérique de l’équation précédente s’effectue en utilisant la fonction ode45 de MATLAB qui renvoie dans la variable r les valeurs de r aux instants t contenus dans la variable t. Le code est le suivant : Rt = 6400E3;Hi = 832E3;ri = Rt+Hi; t = linspace(0,100*86400,1000); option = odeset(’RelTol’,1E-6,’AbsTol’,1E-6); [t,r] = ode45(’satellite’,t,ri,option);

c) Évolution de l’altitude du satellite Sur la figure A5.12, on a représenté l’évolution de l’altitude Dh = r − r i , le temps étant exprimé en jour. La comparaison avec le résultat obtenu analytiquement en supposant b = bi montre que 1/2 l’écart entre les deux représentations, précisément la différence D = (−ki t/2 + r i )2 − r entre la 1/2 coordonnée radiale (−ki t/2 + ri )2 relative à b constant et celle obtenue avec b variable, est faible : D est de l’ordre de quelques centimètres (Fig. A5.13). La schématisation avec b variant selon une loi exponentielle décroissante fournit cependant une valeur proche de la valeur expérimentale. Les deux figures ont été obtenues avec graphesatellite.

640

Annexe 5 0

−10 0.04

−20

D (m)

Φ h (m)

−30 −40 −50

0.02

−60 −70 −80

0

20

40 60 t (jour)

80

100

F IG . A5.12. — Perte d’altitude du satellite SPOT

0 0

20

40 60 t (jour)

80

100

F IG . A5.13. — Écart par rapport au cas où b serait constant

IV . — PORTRAIT DE PHASE D’OSCILLATEURS Le portrait de phase permet de représenter graphiquement l’état mécanique d’un système, c’est-àdire à la fois sa position et sa quantité de mouvement. La simulation numérique permet évidemment de tracer des portraits de phase de façon précise. C’est ce que nous allons montrer sur l’exemple d’un oscillateur à un seul degré de liberté, x(t), soumis à divers types de force de frottement F f : le frottement solide, indépendant de la vitesse de l’oscillateur, le frottement visqueux de type Stokes, proportionnel à la vitesse, et le frottement visqueux de type Venturi, proportionnel au carré de la vitesse (cf. simulation IV). IV . 1 . — Équation du mouvement L’équation du mouvement d’un oscillateur amorti se met sous la forme : d 2 x Ff − + v20 x = 0 d t2 m où m est la masse de l’oscillateur, v0 sa pulsation propre et F f la projection algébrique sur l’axe du mouvement du vecteur force de frottement. Avec MATLAB, on peut résoudre cette équation différentielle du second ordre par rapport au temps en l’écrivant sous la forme du système suivant. Ainsi, on écrit : dx = vx dt

Ff d vx − v 20 x = m dt

Dans la suite, c’est la commande ode45 de MATLAB qui fournira la solution de ce système. IV . 2 . — Frottement solide La force de frottement solide est indépendante de la vitesse, de sorte que F f est une constante. Son rôle devient cependant de plus en plus important au fur et à mesure que la vitesse s’affaiblit : c’est pourquoi l’oscillateur peut s’arrêter au voisinage de sa position d’équilibre sans l’atteindre. On gère l’arrêt éventuel du mouvement en imposant une vitesse nulle lorsque la force de rappel est inférieure à la force de frottement et la vitesse proche de zéro ; les dérivées temporelles de la position et de la vitesse sont alors mises à zéro, d’où le test if dans la fonction odeFrotSolide qui contient les

Simulation en mécanique

641

équations du mouvement : la variable numérique sol rassemble, elle, les variables numériques x et vx qui représentent respectivement x et v x, et la variable numérique solp rassemble, elle, leurs dérivées respectives. La solution des équations du mouvement par la méthode ode45 est stockée dans la variable numérique sol, dont on extrait la position x et la vitesse vx. Les conditions initiales x = 1 et v x = 0, en unité arbitraire, sont représentées par la variable numérique CI. La masse m de l’oscillateur est prise égale à l’unité, d’où Ff /m = 0, 04 ; on peut cependant modifier la valeur de m. La résolution est obtenue en donnant d’abord des valeurs aux constantes m, |F f | et v 20, puis en définissant les conditions initiales et l’intervalle temporel sur lequel on résout l’équation du mouvement, enfin en appelant la fonction Frottement : m=1;Ff=0.04;omega02=1;CI = [1 0]; t=linspace(0,50,1000); [t,x,vx]=Frottement(t,m,Ff,omega02,CI);

On doit d’abord répondre à la fonction Frottement qui réclame le type de frottement. On choisit ensuite le frottement solide en écrivant 0 ; cette fonction Frottement appelle alors la fonction FrotSolide qui s’écrit : function [t,x,vx]=FrotSolide(t,m,Ff,omega02,CI) [t,sol] = ode45(@odeFrotSolide,t,CI); vx = sol(:,2); x = sol(:,1);

 c Dunod – Toute reproduction non autorisée est un délit

function solp=odeFrotSolide(t,sol) solp=zeros(size(sol)); solp(1)=sol(2); solp(2)=-omega02*sol(1)-(Ff/m)*sign(sol(2)); if (abs(omega02*sol(1)) T s, puisque le point M considéré de la Terre doit parcourir l’arc supplémentaire M M pour se trouver à nouveau au zénith. L’angle a correspondant est égal à l’angle b défini par l’arc TT  décrit simultanément par le centre de la Terre : T − Ts T a = b avec a = 2p m et b = 2p m Ts Ta On en déduit : T m − Ts Tm Tm Tm2 soit Ts = et T m − Ts = ≈ 4 = 1 + T Ts Ta Ta + T m m /Ta

T M Soleil

Terre

M  b a

F IG . S3.2.

T

L

Lune

M

M 

L

α

M

β

M Terre

F IG . S3.3.

661

et problèmes S3– 8. Lunaison synodique et lunaison sidérale

À l’aide de la figure S3.3, on voit que Tn > Tl, puisque le point M considéré de la Terre doit parcourir l’arc supplémentaire M M . L’angle a correspondant est égal à l’angle b défini par l’arc TT décrit simultanément par le centre de la Terre : Tn − Tl Tn a = b avec a = 2p et b = 2p Tl Ta On en déduit : Tn − Tl Tn = Tl Ta

soit T n =

Ta Tl T a − Tl

et T n − Tl =

Tl2 ≈ 2, 2 j Ta − T l

d’où Tn = 29, 5 j

S3– 9. Mouvement d’un point sur un disque lié à une tige 1. Par rapport à R, le point O 1 décrit un cercle de centre O et de rayon l. Donc : vO1 /R = w˙ ez × OO1 = w˙ e z × (le x  ) = lw˙ ey  Par rapport à R, A décrit aussi un cercle mais de centre O1 et de rayon r. Donc : v A/R  = u˙ e z × OA = u˙ e z × (r cos u ex  + r sin u ey )

Il en résulte dans la base de R : v A/R  = ru˙ (−r cos u ex + cos u e y  ). 2. D’après la composition des vitesses, il vient :

vA/R = v A/R + v e avec

v e = vO1 + VR /R × O1 A

˙ z × O 1A soit : On en déduit : v A/R = vA/R + v O1 + w˙ e z × O1A = lw˙ ey  + (w˙ + u)e vA/R = −r(w˙ + u˙ ) sin u ex + [r(w˙ + u˙ ) cos u + lw] ˙ e y 3. Pour u = 0 et u = p, on a respectivement : ˙ + lw] (v A/R )u=0 = [r(w˙ + u) ˙ e y

˙ + lw] et (vA/R ) u=p = [−r(w˙ + u) ˙ e y

Comme vO1 /R = lw˙ ey  , cette vitesse est la demi-somme des deux vitesses précédentes. 4. Si la vitesse de A est nulle lorsque u = p, alors : ˙ = lw˙ r(w˙ + u)

soit (l − r )w˙ = ru˙

 c Dunod – Toute reproduction non autorisée est un délit

Si w˙ = 2 tr . min−1, alors u˙ = l − r w/ ˙ r = 60 tr . min −1. La vitesse angulaire du disque par rapport à R est la somme des vitesses angulaires w˙ ez et u˙ ez. En effet : V D/R = VD/R + V R /R = u˙ e z + w˙ ez = 80 tr . min−1

Chapitre 4 S4– 1. Mouvement d’une fusée D’après la loi fondamentale, on a : md v/ d t = mG + Fp + R. En projetant suivant la normale à la trajectoire, on obtient : mv 2 mG 2v2 = mG sin(30◦ ) = d’où R = = 3 000 km G R 2

662

Solutions des exercices

S4– 2. Influence de la taille d’un athlète au lancé du poids 1. Dans un système d’axes Oxy où Oy est la verticale ascendante, il est bien que connu que : g x = (v 0 cos a) t et y = − t 2 + (v0 sin a) t + h 2 d’où y = 0 si : −

gx2 + x tan a + h = 0 2v20 cos2 a

soit



gx 2 2 + x sin a cos a + h cos a = 0 2v 20

pourvu que a = p/2. Différentions x par rapport à a ; on obtient : − d’où :

gx d x + d x sin a cos a + x d a (cos 2 a − sin 2 a) − 2h cos a sin a d a = 0 v 20

dx 2h sin a cos a − x(cos2 a − sin 2 a) =0 = da −gx/v20 + sin a cos a

si

x = xm = 2h

sin a cos a cos 2 a − sin2 a

2. En injectant cette expression dans l’équation du deuxième degré, on trouve : 2gh 1 −1 = 2 tan2 a v0

d’où

tan 2 a m =

1 1 + 2gh/v20

3. La distance x m s’écrit aussi : xm = 2h

1/ tan a v 20 = g 1/ tan2 a − 1

1+

2gh v02

1/ 2

Ainsi xm augmente avec h. Pour h = 2, 2 m, xm = 20, 66 m ; pour h = 2, 5 m, xm = 20, 93 m. S4– 3. Expérience de Millikan 1. Comme md v/ d t = mg − 6phr v donne d v/ d t + v/t = g, il vient, en projetant suivant la verticale descendante Oz : dv v t + = g d’où v = gt + Cte × exp − t t dt

= gt 1 − exp −

t t

en intégrant et en tenant compte des conditions initiales : v = 0 à t = 0. Ainsi, la vitesse tend vers une vitesse limite vl : 1/ 2 2rgr3 9hv l mg mg vl = gt = = soit r = et r = ≈ 2m m 6phr 6phvl 9hv l 2rg 2. L’effet d’une force constante telle que qE est de modifier la vitesse limite vl : v l = v l + qE/(6phr ). 3. Le mouvement dû à E est ascendant : v l = −vlez . On a donc, en projetant la relation vectorielle précédente : 6phr(v l + v l ) qE d’où q = −vl = vl − 6phr E Le tableau S4.1 rassemble les différentes valeurs de vl + vl et de la charge correspondante. On en déduit : −19

Dq 1 = (8 − 11, 2) × 10

−19

= −3, 2 × 10

Dq 3 = (12, 8 − 9, 6) × 10−19 = 3, 2 × 10−19 C

La charge élémentaire est donc e = 1, 6 × 10−19 C.

C et

−19

Dq2 = (9, 6 − 8) × 10

= 1, 6 × 10

−19

C

Dq4 = (14, 4 − 12, 8) × 10 −19 = 1, 6 × 10 −19 C

663

et problèmes v + v q × 1019 C

0,662 11,2

0,472 8

0,567 9,6

0,755 12,8

0,850 14,4

TAB . S4.1. S4– 4. Mouvement d’un point soumis à une force résistante en v2 1. La loi fondamentale de la dynamique, explicitée selon la verticale descendante, donne : m

rSv2 dv = mg − C x dt 2

d’où

v2 dv =g 1− 2 dt v1

avec

v1 =

1/ 2

2mg C xrS

2. L’équation précédente n’est pas linéaire. Pour l’intégrer, écrivons-la sous la forme : dv dv = 2g d t + 1 − v/v 1 1 + v/v1 Par conséquent, puisqu’à t = 0, v = 0 : v = v1

d’où

ln

1 + v/v 1 1 − v/v 1

=

2gt v1

gt 1 − exp (−2gt/v1) exp(gt/v 1) − exp(−gt/v 1) = v1 = v 1 tanh v1 1 + exp (−2gt/v1) exp(gt/v 1) + exp(−gt/v 1)

Pour t faible, v = gt ; lorsque t → ∞, v tend vers une vitesse limite égale à v1 (Fig. S4.1).

v v1

v

v0

0 v 1 /g

0

t

F IG . S4.1.

t F IG . S4.2.

S4– 5. Mouvement d’un point soumis à une force résistante en v1/2

 c Dunod – Toute reproduction non autorisée est un délit

1. D’après la loi fondamentale projetée suivant l’axe horizontal du mouvement, il vient : b dv dv = −b v1/2 d’où = − dt m dt v 1/ 2 m 2. Il en résulte, en intégrant cette équation différentielle non linéaire, que : v=

1/ 2

v0



bt 2m

2

1/ 2

car initialement v = v0. Le graphe v (t) est une parabole (Fig. S4.2) ; v s’annule t1 = 2mv0 /b.

Chapitre 5 S5– 1. Forces et énergies potentielles Calculons le travail de ces forces : dW1 = F 1 · d r = ay d x

Comme W1 dépend du chemin défini par y(x), la force F1 ne dérive pas d’une énergie potentielle.

664

Solutions des exercices La force F 2 dérive de l’énergie potentielle Ep,2 = −(ax 2/2 + by 2/2) + Cte, puisque : dW2 = F2 · d r = ax d x + by d y = − d[−(ax 2/2 + by2 /2)]

Il en est de même de F3 : d’où Ep,3 = −axy + Cte

dW3 = F3 · d r = a(y d x + x d y) = − d(−axy)

S5– 2. Conservation de l’énergie dans un puit d’énergie potentielle radiale 1. On a : E k + Ep = E m = Cte avec Ep défini comme suit : dW = F · d r =

K K −K e · dr = n dr = −d n r (1 − n)n−1 r r

Il en résulte l’expression suivante de Em : Em =

= − d Ep

avec

Ep =

−K + Cte (1 − n)r n−1

1 −K + Cte = Cte mv 2 + 2 (1 − n)r n−1

2. Pour r = 0, v est fini ; donc n < 1. En choisissant la constante de l’énergie potentielle nulle, la conservation de Em donne : 1 −K = mv 20 n−1 2 (1 − n)r 0 ce qui exige que K < 0 : la force est donc attractive.

S5– 3. Mouvement d’une particule dans un champ de force en r−3 1. On obtient E p à partir du travail élémentaire : dW = F · d r = K

dr K er · dr = K 3 = −d 3 2 2r r r

d’où E p =

K K + Cte = 2 2r2 2r

si Ep = 0 pour r infini.

2. Comme la force est centrale : MO = 0 d’où LO = OA × mv = Cte. Ainsi, OA et v restent perpendiculaires à LO qui est une constante vectorielle : le mouvement est donc plan. En coordonnées polaires, moment cinétique et énergie cinétique s’écrivent : L O = OA × mv = r e r × (r˙ er + rw˙ e w) = mr2 w˙ ez

et Ek =

1 2 1 mv = m(˙r2 + r2 w˙ 2 ) 2 2

3. Les équations du mouvement sont obtenues à partir de la conservation de L et de l’énergie Em : L = mr2 w˙ ez = L e z et Em = E k + Ep = Cte puisque la force est conservative. On en déduit : 1 2 1 2 Em = m ˙r + mr 2 2 avec : Ep,ef (r) = E p (r) +

1 L2 = 2 2 2mr 2r

L2 m 2r 4

L2 +K m

4. On a, dans ce cas, E m = m˙r2 /2 et ˙r = 2Em/m r˙ 0 =

2E m m

1/ 2

+

1/ 2

1 2 K = m ˙r + Ep,ef (r) 2r2 2 =

K r2

et K  =

= Cte = r˙ 0 . Ainsi :

et r =

2Em m

1/ 2

t + r0

1 2

L2 +K m

665

et problèmes

5. Dans le cas où K  = 0, il vient, en introduisant s = r2 , on obtient Ep,ef = K/s, d’où en différentiant 2rr˙ = s˙. On en déduit : ˙r =

˙s ˙s = 1/ 2 2r 2s

et Em =

1 ˙s2 K + m 2 4s s

ou s E m =

m s˙2  +K 8

En dérivant, on obtient : ˙sEm = m ˙s¨ s/4 soit ¨s = (4Em /m) = Cte. Il en résulte : ˙s =

4E m 4Em t + Cte = t + 2r0r˙ 0 m m

2

et s = r =

2E m 2 2 t + 2r 0˙r0 t + r0 puisque m

6. D’après ce qui précède, si ˙r0 = 0, r 2 − r20 = 2E mt 2 /m. i) Si E m < 0 : r 2Em 2 r2 + t = r20 soit m r0

2

+

r = r0 à t = 0

2Em 2 t =1 mr 20

C’est une ellipse ; notons que r = 0 pour t = t = r0 /(2E m/m) 1/2 (Fig. S5.1a). ii) Si Em > 0 :

r2 −

2Em 2 t = r20 m

soit

r r0

2



2Em 2 t =1 mr 20

C’est une hyperbole (Fig. S5.1b).

r r0

r

Em < 0

v vl

Em > 0

r0 0

¿

vl/2

0

t

0

t

a)

x

b) F IG . S5.1.

F IG . S5.2.

 c Dunod – Toute reproduction non autorisée est un délit

S5– 4. Mouvement d’une particule soumise à une force de puissance constante 1. Comme la puissance de la force motrice Fm est constante, on a Fm = P/v. La loi fondamentale donne alors : dv dv 1 dv P P 2 m d’où = − bmv = = − bv dt dx v v dt mv2 Il en résulte que : dx =

mv 2 d v P − bmv3

d’où x =

1 mv 2 d v =− 3b P − bmv 3

du 1 3 =− ln |P − bmv | + Cte 3b u

en posant u = P − bmv3, ce qui donne d u = −3bmv2 d v. Comme à t = 0, v = 0, il vient : 0=−

1 ln P + Cte 3b

et x =

P bmv 3 1 1 ln ln 1 − = − 3b 3b P P − bmv 3

avec 1/v3l = bm/P . Lorsque v tend vers vl , x tend vers l’infini (Fig. S5.2). 2. Lorsque b tend vers 0, x tend vers :

−1 3b

−bmv3 P

=

mv3 P

3. b = P/(mvl3) ≈ 0, 001 SI et x = −1/(3b) × ln (1 − 1/8) = 44, 5 m.

=−

v3 1 ln 1 − 3 3b vl

666

Solutions des exercices

S5– 5. Analyse énergétique d’un oscillateur harmonique dans le champ de pesanteur terrestre 1. Le système étant conservatif, on a : Ek + Ep = Em = Cte avec Ek =

m˙x2 2

et Ep = Epg + Epe = −mgx +

K (x − l0 )2 + Cte 2

puisque l’axe Ox est orienté suivant la verticale descendante. 2. Faisons apparaître dans l’énergie potentielle la longueur l1 à l’équilibre : K (l 1 − l0 ) = mg. Il vient, en introduisant l’allongement X = x − l1 : Ep

1 K [X + (l1 − l 0 )]2 + Cte 2 1 1 = −mgX − mgl 1 + KX 2 + K (l1 − l 0) 2 + KX (l1 − l0 ) + Cte 2 2 1 K 2 = KX + X [−mg + K (l1 − l0)] − mgl 1 + (l 1 − l0 )2 + Cte 2 2 = −mgX − mgl 1 +

Finalement Ep = KX 2 /2 + Cte . S5– 6. Aspect énergétique de la force de frottement de Stokes 1. Le théorème de l’énergie mécanique appliqué au mouvement de la particule donne D(Ek + Ep ) = −D 2. a) Dans le cas de la chute de la particule dans le champ de pesanteur terrestre, la particule atteint une vitesse limite. En effet, d’après la loi fondamentale, on a : vx dv d vx + m =g = mg − av soit t dt dt en explicitant selon la verticale descendante Ox et en introduisant la durée caractéristique t = m/ a : L’intégration de cette équation différentielle donne (cf. chapitre 4 et annexe 4) : vx = Cte exp −

t t + gt = gt 1 − exp − t t

La vitesse limite est donc : vl = gt = 1, 2 m . s −1 , puisque t = m/ a = 4, 5 ms. Cette valeur de t montre que la vitesse limite est atteinte rapidement, puisqu’au bout de 0, 1 ms, la vitesse vx vaut : vx = vl avec une précision de 2 × 10−10 . b) La perte d’énergie de la gouttelette d’eau est égale à la variation de l’énergie potentielle −mgx, puisque la vitesse est pratiquement constante : D = −DEp = Ep,i − Ep,f = 0 − (−mgh) = mgh = 8, 8 × 10−12 J. S5– 7. Fronde de longueur variable 1. a) Pour connaître la tension du fil, appliquons, par rapport au référentiel R du laboratoire, la loi fondamentale de la dynamique : maA = mg + T avec aA = −l0w˙ 2 er Comme w˙ = 2p × 3 = 31, 4 rad.s −1 , il vient ma 0 = 0, 1 × 1 × 31, 4 2 ≈ 98, 5 N alors que mg ≈ 0, 98 N. Ainsi le poids peut être négligé devant la tension du fil.

b) La vitesse, le moment cinétique en O et l’énergie cinétique de A valent respectivement : v0 = l0 w˙ = 1 × 31, 4 = 31, 4 m.s−1 et

LO = mv0l0 = 0, 1 × 31, 4 × 1 = 3, 14 kg.m 2.s−1

1 0, 1 × 31, 42 2 = 49, 3 J Ek,0 = mv 0 = 2 2

667

et problèmes 2. a) Appliquons le théorème du moment cinétique en O , fixe dans R : d LO = OA × T = 0 dt

Ainsi, le moment cinétique se conserve : mv1 l 1 = mv0 l0 . Quant à l’énergie cinétique, elle augmente puisque la fronde reçoit du travail de la part de l’opérateur : d Ek = dWop > 0, dWop étant le travail de la force de traction exercée par l’opérateur. b) La vitesse de A et son énergie cinétique deviennent, si la longueur du fil est réduite de moitié : v1 = v0

l0 = 2v0 = 62, 8 m.s−1 l1

et Ek =

1 2 1 2 l2 mv 1 = mv 0 02 = 4E k,0 = 197, 2 J 2 2 l1

On en déduit le travail Wop de la force de traction : Wop = Ek,1 − Ek,0 = 3Ek,0 = 147, 9 J.

On peut retrouver ce résultat en calculant le travail de la force de traction. On a, en effet, si L est la longueur du fil inextensible AE : f

W= i

f

Fop · d OE =

m i

v2 d(L − l) = − l

l 0 /2

mv20

l0

l 20 mv20 dl = 3 2 l

l20 l2

l0 /2 l0

= −3Ek,0

Chapitre 6 S6– 1. Force de gravitation exercée par une sphère Déterminons le champ de gravitation à l’aide du théorème de Gauss appliqué à une surface sphérique concentrique à la sphère, de rayon r. On trouve : i) pour r < R : 2 4pr Gr = 0

ii) pour r > R :

d’où

 c Dunod – Toute reproduction non autorisée est un délit

2

Gr = 0

4pr Gr = −4pGM∗ d’où Gr = −

GM ∗ r2

F=0

et

et F = −

GM ∗ m ∗ er r2

S6– 2. Force de gravitation exercée par un disque Comme F = m ∗G, il vient, en projetant : Gz = G · e z = G

r ∗s

r d S · e z = Gr∗s r3

r · ez dS r3

et

Gx = G · e x = Gr∗s

x dS r3

Or la première intégrale fait apparaître l’angle solide sous lequel on voit le disque du point A (cf. annexe 3) et la seconde est nulle du fait de la symétrie. Il en résulte, puisque r∗s = M∗ /(pR 2) : F=

Gm∗ M ∗ Gm∗M ∗ 2 p( 1 − cos a) e 2(1 − cos a) e z = z pR 2 R2

668

Solutions des exercices

S6– 3. Comparaison de différents champs de gravitation Le champ de gravitation en un point de la surface d’un astre peut être exprimé en fonction du champ de gravitation terrestre : G=

GM ∗ m∗ × = G T R2 r2

avec

GT = G

M ∗T = 9, 77 m . s−2 R 2T

m=

M∗ MT∗

et r =

R RT

On obtient, en unités SI ( m . s−2 ) : G S = GT ×

0, 334 × 106 = 272 109, 42

G M = GT ×

0, 11 = 3, 8 0, 533 2

GL = G T × GJ = GT ×

0, 0122 = 1, 6 0, 2722

318 = 24, 8 11, 22

G V = GT × G Sa = GT ×

0, 82 = 8, 9 0, 9492

94 = 10, 3 9, 452

S6– 4. Point équigravitationnel entre la Lune et la Terre Le champ de gravitation G résultant est la somme des deux contributions lunaire G L et terrestre G T . Il est nul au point E tel que : eL GM ∗T GML∗ M∗T e T G = GT + G L = 0 d’où = − = − e e soit T L TE 2 LE 2 LE2 M∗L TE 2 Il en résulte que les points T , E et L sont alignés et que TE /LE = 9. Le point équigravitationnel est donc situé sur la droite TL à 38 400 km de la Lune. S6– 5. Champ de gravitation dans une cavité creusée dans une boule Le champ de gravitation G p créé par la sphère pleine est la somme du champ recherché G et du champ G c produit par la distribution sphérique remplissant la cavité : r∗ r r ∗ r et Gc = (−4pG) 3 3 ∗ obtenus en appliquant le théorème de Gauss, r étant la masse grave volumique, r = OA et r = OA. Il en résulte que : r∗ r∗ G = (−4pG) (r − r ) = (−4pG) OO 3 3 Ainsi, le champ dans la cavité est uniforme. Gp = G + G c

avec

G p = (−4pG)

S6– 6. Énergies potentielles de gravitation Terre-Lune et Terre-Soleil L’énergie potentielle de gravitation a pour expression : Ep = K/ r, avec K = −Gm∗1 m ∗2 , si E p = 0 pour r infini. On a donc pour les systèmes Terre-Lune et Terre-Soleil respectivement : Ep = −G

M ∗T M∗L 29 = −0, 77 × 10 J TL

et

Ep = −G

M∗T MS∗ 33 = −5, 32 × 10 J TS

S6– 7. Variation de l’énergie potentielle de gravitation en fonction de l’altitude On sait que Ep = K /r + Cte. Si l’origine est prise pour r = RT , RT étant le rayon de la Terre, il vient : 0 = K /RT + Cte. Il en résulte, si r ≈ RT : Ep = K

1 1 − r RT

≈−

K GM T∗m ∗ z= z = m∗ G z 2 RT R 2T

en posant G =

GM ∗T R2T

669

et problèmes S6– 8. Moment des forces gravitationnelles au centre de la Terre 1. On a, par définition : F T →K = M ∗KG = MK∗(G r er + G u e u) et : M ∗T ,T →K = TK × M ∗K(G r er + G u e u ) = TK × M∗K Gu eu =

3G M∗ (I3 − I1 ) K3 sin(2u) e u r 2

2. Le moment en T des actions qu’exerce K sur la Terre est l’opposé du précédent. Par conséquent : 3G M∗K (I3 − I1 ) 3 sin(2u) eu 2 r d’où le moment des forces gravitationnelles exercées par le Soleil et la Lune : M∗T ,K →T = −M∗T ,T →K = −

∗ ∗ 3G MS M + 3L sin(2u) eu (I3 − I 1 ) 3 2 rS rL Le rapport des contributions de la Lune et du Soleil est défini par le rapport des masses et le cube du rapport des distances. C’est lui qui intervient dans les marées ; il vaut 2, 18 à l’avantage de la Lune (cf. chapitre 7).



S6– 9. Énergie gravitationnelle d’une boule homogène 1. Le champ de gravitation produit par une telle distribution a pour expression (cf. chapitre 6) : r r G in = −GM∗ 3 et G ex = −GM ∗ 3 R r Comme, du fait de la symétrie, l’élément de volume peut être pris égal à 4pr2 d r, l’intégration donne : R

GM ∗2 Ep = − 8p Comme

M∗S

≈ 2 × 10

30

0



r2 4pr2 d r + R6

R

4pr2 dr r4

=−

GM ∗2 2

1 1 + 5R R

=−

3 GM ∗2 5 R

9

kg et RS ≈ 0, 7 × 10 m, E p = −0, 228 × 1042 J.

2. On a : −

3G M∗2 = −M ∗c2 5 R

d’où R 0 =

5 3

GM ∗ c2

3

≈ 2, 5 × 10 m

S6– 10. Énergie gravitationnelle d’une sphère homogène

 c Dunod – Toute reproduction non autorisée est un délit

1. Le potentiel gravitationnel produit par la sphère s’obtient à partir du champ. On a, à l’intérieur et à l’extérieur, respectivement : Gr = 0 = −

dF dr

d’où F = Cte et

Gr = −

dF GM ∗ =− dr r2

d’où F = −

GM ∗ + Cte r

Comme F est nul à l’infini cette constante est nulle et le potentiel à l’intérieur vaut −GM∗ /R. 2. Dans le cas d’une distribution de masse surfacique, l’énergie gravitationnelle a pour expression : Ep =

1 2

1 sF d S = F 2 S

S

s dS =

1 ∗ GM∗2 −10 M F=− = −3, 335 × 10 J. 2 2R

puisque le potentiel est uniforme et que l’intégrale représente la masse totale M∗ de la distribution. S6– 11. Détermination de certaines caractéristiques physiques de la Terre 1. Appliquons le théorème de Gauss à travers une sphère de rayon r ≈ R : 4 −4pR2G0 = −4pG pR3 r∗0 3

d’où r ∗0 =

3G 0 = 5, 515 × 10 3 kg . m −3 2Gl

et M ∗ =

G0 l2 ≈ 6 × 1024 kg 4p2 G

670

Solutions des exercices 2. a) La masse totale s’écrit : 4pR3 ∗ 4pr32 ∗ 4p 3 (R − r32)r∗1 r0 = r2 + 3 3 3

d’où r ∗0 = r∗1 +

r2 R

3

(r∗2 − r∗1 )

b) Appliquons à nouveau le théorème de Gauss à une surface sphérique de rayon r compris entre r2 et r 1. Il vient : 4p r 2r32 + (r3 − r 32)r ∗1 −4pr2 G = (−4pG) 3

d’où G =

4pG ∗ 1 r 1 r + r 32 (r∗2 − r∗1 ) 2 3 r

On en déduit, en différentiant et en faisant r ≈ R : dG ≈

4pG ∗ r2 r1 − 2 3 R

3

(r∗2 − r ∗1) dr = 2G 0

3r∗1 −1 2r∗0

dr R

c) La période du pendule simple étant T0 = 2p (l0 /G) 1/2 , sa variation relative est : 1 DG DT0 =− 2 G T0

d’où r∗1 =

2 ∗ DT 0 R r0 1 − 3 T0 dr

Comme DT0/T0 = −1/(24 × 3 600) et dr = −h = 195 m, on trouve r ∗1 = (2r ∗0 /3) × 0, 62 = 0, 413 et r ∗0 = 2 289 × 103 kg . m−3 . d) D’après 2 . a), il vient, puisque R = l/(2p) = 6 366 km, r2 = R − H = 5 066 km : r ∗2 = r∗1 +

R R−H

3

(r ∗0 − r∗1 ) = 8, 67 × 10 3 kg . m −3

S6– 12. Oscillations dans le champ de gravitation à l’intérieur d’une boule 1. La force de gravitation à laquelle est soumise la particule A a pour expression : GMT F = mG avec G = − 3 r RT 2. La force précédente est une force de rappel proportionnelle r. Elle rappelle celle exercée par un ressort. La loi fondamentale de la dynamique appliquée à A donne donc : r=− m¨

GmMT r d’où ¨ r + v20 r = 0 R3T

avec

v 20 =

GMT R 3T

On en déduit la période propre : T0 = 2p

R3T GMT

1/ 2

= 2p

1/ 2

RT G0

en introduisant G0 =

GM T R2T

le champ de gravitation G0 à la surface de la Terre. On trouve : T0 = 5 077 s = 84, 6 min. S6– 13. Paradoxe sur le système Soleil-Terre-Lune 1. Les forces de gravitation qu’exercent le Soleil et la Terre sur la Lune sont respectivement : FS→L = G

M SM L = 439, 4 × 1018 N SL2

et

F T →L = G

MT ML = 199, 48 × 1018 N TL2

d’où un rapport égal à 2,2 à l’avantage du Soleil, ce qui semble en première analyse paradoxal, car la Lune reste dans le voisinage de la Terre. On lève aisément ce paradoxe en notant que la Terre est, comme la Lune, attirée par le Soleil. Ce qui importe dans la séparation des deux astres, Terre et Lune, c’est la différence des champs de gravitation solaires aux points où se trouvent la Terre et la Lune.

671

et problèmes 2. Calculons le champ de gravitation qu’exerce la Terre en L : GM T −3 = 2, 71 × 10 m . s−2 TL 2 Quant à la différence des champs de gravitation solaire en T et en L, sa valeur maximale est : GT =

GMS GMS GM S − ≈ TL SL2 ST 2 ST 3 puisque SL = ST + TL avec TL  ST. On en déduit la valeur suivante de cette différence : GS (L) − GS (T) =

GS (L) − GS(T ) ≈

GMS −7 TL = 3, 46 × 10 m . s−2 ST 3

ce qui est négligeable devant GT .

Chapitre 7 S7– 1. Angle entre G et g Dans le triangle OAB, la relation des sinus donne (Fig. S7.1) : sin a V 2T R cos l

=

sin l g

d’où

sin a =

V2T R sin(2l) et 2g

a ≈ 6 sin(2l)

La valeur maximale est donc obtenue pour l ≈ 45◦. À Toulouse (l = 43◦ 35 ), a = 5, 99  ≈ 6 . z

O

V2T R cos ¸ VT

® G A

¸

¸

O

y Nord

g B

x

Est

F IG . S7.1.

F IG . S7.2.

 c Dunod – Toute reproduction non autorisée est un délit

S7– 2. Influence de la force de Coriolis sur une balle de fusil 1. La loi fondamentale approchée ma = mg donne les résultats bien connus : y = v0t

x=0

z=−

gt2 2

d’où z = −

g y 2 v0

2

Comme y/v0 = 0, 1 s, z ≈ −5 cm.

2. La force de Coriolis −2mVT × v dévie la particule vers l’est (Fig. S7.2). 3.

m

¨x ¨y = ¨z

0 0 − 2mVT −mg

0 cos l × sin l

˙x ˙y ˙z

d’où

4. Comme ¨x ≈ 2VT v0 sin l : ˙x ≈ (2V T v 0 sin l) t

d’où x ≈ (VT v0 sin l) t

2

x = −2V T (˙z cos l − ˙y sin l) ¨ ¨ y = −2V T ˙x sin l ¨ z = −g + 2V T x˙ cos l

et x(d ) ≈ VT sin l

d2 ≈ 0, 5 mm v0

672

Solutions des exercices

S7– 3. Analyse quantitative du mouvement d’un pendule de Foucault 1. La loi fondamentale de la mécanique dans le référentiel terrestre local R s’écrit rigoureusement : ma = mg + FT − 2mVT × v En projetant suivant les trois axes, il vient, z étant la verticale ascendante, Ox étant orienté vers l’est : m

¨ x ¨ y = ¨z

x − 2mV T y z−l

0 FT 0 − l −mg

0 sin u × cos u

˙x ˙y ˙z

u étant la colatitude. 2. Faisant z ≈ 0, z˙ ≈ 0 et ¨z ≈ 0 dans les équations précédentes, on obtient FT ≈ mg et les deux équations recherchées : ¨ ¨ x − 2VT cos u ˙x + v20 x = 0 et y + 2VT cos u y˙ + v20 y = 0 ce qui donne, en notation complexe :

¨ z + 2jVT cos uz˙ + v20 z = 0

avec

z = x + jy

3. Cherchons des solutions de la forme exp(rt) , r étant complexe. On obtient l’équation caractéristique : r 2 + 2jVT cos u r + v20 = 0 de racines r = −jVT cos u ± j V2T cos 2 u + v20

1/ 2

≈ −jVT cos u ± jv0

puisque VT  v0 . Par conséquent, z peut s’écrire : z = exp(−jVT cos u t )[A cos(v 0 t) + B sin(v0 t)] soit, en tenant compte des conditions initiales : z = x m et z˙ = 0 : z = xm exp(−jV T cos u t) cos(v0 t) + j

VT cos u sin(v0t) v0

4. Le terme multiplicatif exp(−jV T cos u t) représente la rotation du plan horizontal Oxy autour de l’axe vertical Oz avec une vitesse de rotation égale à (−VT cos u). Dans le plan tournant la trajectoire est une ellipse très aplatie puisque VT  v0 (Fig. S7.3). La durée d’un tour complet de l’ellipse, dans le sens nord-est-sud-ouest, est : 2p 2p = T= VT cos u VT sin l ◦ ◦  Pour l = 90 (pôles) , Tp = 24 h ; pour l = 43 35 (Toulouse), TT = 34, 86 h ; pour l = 0◦ (équateur) T e = ∞. Dans l’hémisphère sud, la période est négative : l’ellipse tourne en sens inverse.

y (Nord)

y

x (Est) (VT cos u)t

Ouest Sud F IG . S7.3.

x

673

et problèmes S7– 4. Mouvement d’une masselotte dans un véhicule accéléré 1. Comme v = ( ˙x2 + y˙2)1/2 et ˙y = f  ˙x, il vient : v = ˙x(1 + f 2 )1/2

f v (1 + f2 )1/2

et ˙y =

L’accélération suivant l’axe vertical s’obtient en dérivant cette dernière expression : ¨ y=

d v 2 f  f v ˙ x = d x (1 + f 2 )1/2 (1 + f 2) 2

2. Dans R1 , non galiléen, mais en translation, on doit ajouter la force d’inertie −mae . Par conséquent : ma 1 = T + mg − ma e

y 1 = −K (y1 − l0 ) − mg − donne m¨

mv2 f (1 + f 2)1/2 

3. Comme A est immobile dans R 1, il vient : T = −mg + ma e

soit, en projection T = −K (y1 − l0 ) = mg +

mv 2 f  (1 + f 2 )2

Pour une bosse f  < 0, |T | < mg, alors que pour un creux f  > 0, |T | > mg. Le module |T | représente le poids apparent. 4. Si f (x) = h 1 − x2 /l2 , alors f  = −2hx/l 2 , f  = −2h/l2 . On en déduit : T S = mg 1 −

2hv 2 gl2

d’où TS = 0 pour v = l

g 2h

1/ 2

Cette vitesse réalise donc l’impesanteur. S7– 5. Pendule de Schüler 1. Écrivons vectoriellement le théorème du moment cinétique pour le pendule, au point de suspension O : d LO   = O A × m(g − ae ) + O A × (−maC ) dt Si le pendule reste immobile par rapport à R, cette équation donne : 0 = O A × m(g − ae ) avec 

1/ 2

Il en résulte que v = (g/R)

g = −g e z et a e = v2 R ez

.

 c Dunod – Toute reproduction non autorisée est un délit

2 . Dans R, la période du pendule ainsi constitué vaut : T=

2p R = 2p g v

1/ 2

= 5 072 s

soit T = 84, 5 min

S7– 6. Influence de la force de Coriolis terrestre sur le lit d’un fleuve 1. Sous sa forme vectorielle rigoureuse, la loi fondamentale de la dynamique appliquée à un point A , de masse m , en mouvement horizontal par rapport à R non galiléen, s’écrit : m a = m g − 2m V T × v + R

La force de Coriolis est orientée vers l’est, précisément selon l’axe Ox . 2. En explicitant l’équation précédente dans la base cartésienne de R , on obtient, en introduisant la colatitude u : ¨ 0 0 x x˙ Rx m

¨ y =m ¨ z

0 −g

− 2mVT

sin u ×

cos u

y˙ +

Ry



Rz

674

Solutions des exercices

d’où les trois équations différentielles suivantes : m¨x = −2mVT (˙z sin u − y˙ cos u) + Rx

y = −2mV T ˙x cos u + Ry m¨

Comme le mouvement de A dans R est uniforme selon l’axe Oy , on a : ¨ x=0

x˙ = 0

y˙ = v 0

¨ y=0

z = −mg + 2mV T ˙x sin u + Rz m¨

¨x = 0

x˙ = 0

d’où : 0 = 2mVT v0 cos u + Rx

0 = Ry

0 = −mg + Rz

3. On déduit de ce qui précède les expressions suivantes de Rx et Rz : Rx = −2mVT v 0 cos u

et Rz = mg

La réaction du sol n’est donc pas verticale mais inclinée d’un angle a tel que : tan a =

−2VT v0 cos u −2VT v0 sin l Rx −5 = = ≈ −2 × 10 Rz g g

l = p/2 − u étant la latitude du lieu. Ainsi, la réaction est dirigée vers l’ouest, ce qui indique que la force de Coriolis agit davantage sur la rive droite que sur la rive gauche. On en déduit la dénivellation h suivante entre les deux rives : h = L|a| ≈ 6 mm

z

R a

Fleuve

(Ouest) Rive gauche

x (Est) Rive droite

Oy L F IG . S7.4.

S7– 7. Chute des corps et relativité générale 1. a) Selon le principe d’équivalence de la RG, la masse grave est égale à la masse inerte. Par conséquent, la loi fondamentale de la dynamique donne, en considérant que le référentiel terrestre n’est qu’une approximation d’un référentiel galiléen : m a = m g − 2m V T × v où VT est la vitesse de rotation de la Terre ; sa valeur est V T ≈ 7, 29 × 10−5 rad.s−1 . b) On sait que (cf. Leçon 26) :

g = G T(A) + V2T HA + G a(A) − G a(T )

où GT est le champ de gravitation terrestre, Ga celui produit par les autres astres que la Terre et H la projection de A sur l’axe de rotation de la Terre. La verticale descendante a pour direction et sens celle et celui de g . c) On retrouve aisément l’équation caractéristique de la chute libre, telle qu’elle a été analysée successivement par Galilée et Newton, en négligeant la force de Coriolis terrestre : ma = mg

d’où a = g

2

Ainsi vz (t) = gt et z(t) = (1/2)gt . La durée de chute la plus grande est celle de l’expérience de Reich : t=

2HR g

1/ 2

= 5, 7 s

675

et problèmes

d) Comparons l’équation précédente à celle que l’on obtiendrait en soumettant une bille, de même masse, mais de charge électrique q , à un champ électrique constant E : q ma = qE d’où a = E m Il n’y a dans ce cas aucune simplification possible. La singularité fondamentale de la force de gravitation, à l’origine de la théorie de la RG élaborée par Einstein et achevée en 1916, réside précisément dans l’égalité de la masse grave et de la masse inerte. La force de gravitation est proportionnelle à la masse des corps comme les forces d’inertie. En RG, ce n’est pas une vraie force. 2. a) Réécrivons la loi fondamentale, en tenant compte de la force de Coriolis. Il vient en simplifiant : a = g + 2v × VT

Le terme de Coriolis est perpendiculaire à la vitesse verticale de chute et au vecteur orienté du sud vers le nord. Il est, en outre, orienté vers l’est du lieu que l’on soit dans l’hémisphère nord ou dans l’hémisphère sud. La déviation est maximale à l’équateur où v et VT sont perpendiculaires et minimale (nulle) aux pôles où v et V T sont colinéaires. b) En effectuant le produit vectoriel on obtient l’accélération suivante : a = g + 2gt VT cos l ex e x étant le vecteur unitaire porté par la direction et le sens de l’est, au lieu considéré. c) Suivant l’axe Ox , la composante de l’accélération de A est : a x = 2gV T cos l t

2

d’où v x = gVT cos l t + Cte = gVT cos l t

2

en intégrant par rapport au temps et en tenant compte de la condition initiale vx = 0 pour t = 0 . Quant à la coordonnée suivant l’axe des x , on l’obtient en intégrant, par rapport au temps, cette dernière expression : x = gV T cos l

t3 t3 + Cte = gVT cos l 3 3

car la constante est nulle, x étant nul à t = 0 . d) Pour établir des expressions approchées de v x et x , après une chute de la hauteur H , il suffit de remplacer, dans les expressions précédentes, le temps t par sa valeur approchée issue de H ≈ gt2/2 . On trouve effectivement : 1/ 2 2H (2H /g)3/2 8H 3 V T cos l = 2HVT cos l et x(H ) = gV T cos l v x(H ) ≈ gV T cos l = 3 9g g

 c Dunod – Toute reproduction non autorisée est un délit

e) La déviation est concrètement mesurée en comparant le point de chute et l’extrémité d’un long fil à plomb. Les valeurs de la déviation prédites par la théorie, avec VT = 7, 3 × 10−5 rad.s −1 sont, respectivement : 1, 08 cm

0, 870 cm

2, 76 cm

0, 179 cm

0, 816 cm

ce qui donne les écarts relatifs (xexp. − xthe. )/xthe. suivants : 0, 759

0, 034

0, 014

− 0, 162

− 0, 228

3. Appliquons la loi fondamentale à la bille par rapport au référentiel R , non galiléen. Comme la force d’inertie de Coriolis est nulle et que la force d’inertie d’entraînement est −mae = mg , il vient : ma = mg − mae = 0 Ce référentiel non galiléen R est qualifié d’inertiel, car on peut y réaliser le principe de l’inertie : un corps abandonné garde son état de repos ou sa vitesse en norme et en direction. Einstein l’a qualifié de naturel car, dans ce référentiel, la gravitation est neutralisée par la force d’inertie d’entraînement. Dans R  en translation rectiligne accélérée par rapport à R , il n’y a pas de déviation, car la cabine subit elle-même la déviation vers l’est.

676

Solutions des exercices

S7– 8. Effet Eötvös 1. Le poids terrestre d’un corps est défini expérimentalement par la force opposée à la tension T qui le maintient en équilibre par rapport au référentiel terrestre R . On a alors, en négligeant les termes de marées : g = G + V 2T HA

où G est le champ de gravitation terrestre, VT est la vitesse de rotation de la Terre par rapport au référentiel géocentrique Rg , HA le vecteur défini par le lieu A et sa projection H sur l’axe de rotation de la Terre. 2. a) Dans R b = Txbyb z 0 , la relation prend une forme analogue à la précédente, mais avec la nouvelle vitesse angulaire Vb,e qui est celle de Rb par rapport au référentiel géocentrique : v b,e v b,e Vb,e = VT + = VT + KB RT cos l car KB = RT cos l . On en déduit le champ de pesanteur dans Rb : g b = G + V 2b,eKB. b) Exprimons la différence gb,e − g b,o :

gb,e − gb,o = V 2b,e KB − V 2b,oKB = (Vb,e + Vb,o )(Vb,e − V b,o)KB = 4V T vb,e er

On en déduit aisément la projection de cette différence vectorielle sur la verticale du lieu : (g b,e − g b,o) · e z = 4VT vb,e e r · ez = 4V T vb,e cos l Comme vb,e = 25 m.s −1 et l = 45◦ , l’effet a pour valeur : 4VT vb,e cos l = 4 × 7, 3 × 10

−5

−4

× 25 × 0, 707 ≈ 5 × 10

soit de l’ordre de 0, 05 %

c) Ces résultats sont identiques dans les deux hémisphères puisqu’intervient uniquement le vecteur radial er . Si le bateau se dirigeait vers le nord ou vers le sud, on n’observerait aucune variation de g , car pour de tels déplacements la vitesse angulaire de Rb serait égale à VT . d) La force de Coriolis terrestre appliquée à un point du bateau en mouvement vaut, par unité de masse de ce point : FCor,e F Cor,o FCor,e F Cor,o = −2VT × v b,e et = −2V T × vb,o d’où = −4VT × vb,e = 4VT vb,e er − mb mb mb mb On trouve précisément la même expression, ce qui n’est pas surprenant car, dans le premier cas, on considère le référentiel lié au bateau dans lequel on y définit le champ de pesanteur, alors que, dans le second, on considère, par rapport au référentiel terrestre, le mouvement d’un point du bateau.

Chapitre 8 S8– 1. Réfraction et réflexion en optique corpusculaire 1. Comme le champ électrique est dirigé suivant l’axe des z : p˙ x = 0

d’où

px = Cte et p 1 sin i 1 = p sin i 1/ 2

1/ 2

En outre, la conservation de l’énergie donne : p2/(2m) = eV ; il en résulte que : V1 sin i 1 = V 2 sin i2 . Notons l’analogie avec la loi de Snell-Descartes en optique géométrique (cf. Optique) : l’indice n est proportionnel à V1/2 . 2. L’égalité V2 = V1 sin2 i1 implique i 2 = p/2. Si V2 < V1 sin2 i 1, la relation de réfraction n’a plus de sens car sin i2 qui vaut V 1 sin 2 i1/V 2 devrait être supérieur à 1. L’électron n’atteint pas G2 : c’est l’analogue de la réflexion.

677

et problèmes S8– 2. Focalisation électrique dans un condensateur cylindrique 1. a) Les équations du mouvement de l’électron sont, en coordonnées polaires : m(¨r − rw˙ 2 ) = −eE

et L = mr2w˙ = Cte

En appliquant le théorème de Gauss à une surface cylindrique concentrique avec les électrodes de longueur l, on trouve, compte tenu des symétries (cf. Électromagnétisme) : 2prlE =

r ll ε0

d’où E =

Cte er r

b) Si la trajectoire est circulaire (r = r 0) : ¨ r = 0 et mr 0w˙ 20 = eE0. 2. Comme L = mr2 w˙ = mr 0v0 cos ε, on a : r 2w˙ = r 20 w˙ 0 d’où w˙ = w˙ 0 r20 /r 2 ≈ w˙ 0 (1 − 2Dr /r 0 ) . 3. L’équation ¨r − rw˙ 2 = −eE/m donne :

4Dr Dr eE 0 Dr 2 2 1− d’où D¨ r + 2w˙ 0 Dr = 0 w˙ 0 1 − =− r0 r0 m r0 √ √ On en déduit : Dr = b sin 2w˙ 0t : les trajectoires se recoupent (Dr = 0) pour w˙ 0T = p/ 2 = 127 ◦. D¨r − r0 1 +

S8– 3. Canon de microscope électronique 1. En appliquant la conservation de l’énergie, on obtient : 0 + (−e)(−V a) =

m ev 2 +0 2

d’où

v=

2eVa me

1/ 2

2. Comme m ec 2 = 511 keV : v = c

2eVa m e c2

1/ 2

2V a (kV ) 511

=

1/ 2

Pour Va = 5 kV, v/c ≈ 0, 14 ; pour Va = 100 kV, v/c ≈ 0, 625. Dans le second cas, la vitesse n’est pas négligeable devant la vitesse de la lumière ; la relativité est indispensable (cf. Relativité et invariance). S8– 4. Filtre de Wien

 c Dunod – Toute reproduction non autorisée est un délit

1. La loi fondamentale ma = q(E + v × B) s’explicite selon : m

¨ x ¨ y =q ¨ z

0 0 +q E

x˙ y˙ × z˙

B 0 0

d’où

x=0 (1) ¨ ¨ ˙ (2) y = vc z ¨z = a 1 − vcy˙ (3)

avec vc = qB/m et a 1 = qE/m. On en déduit : x˙ = Cte = 0 et x = Cte = 0. Posant y = y + jz, il vient en effectuant (2) + j(3) : y = −jvc ˙y + ja 1 . Par conséquent : y˙ = Cte × exp(−jvc t) +

a1 = vc

en raison des conditions initiales. Ainsi : a1 [1 − cos (v c t)] ˙y = vc

a1 vc

et z˙ =

[1 − exp(−jvt )] a1 sin(vc t) vc

En intégrant et en tenant compte des conditions initiales, on obtient y et z : a1 a1 y = 2 [vc t − sin(vc t)] et z = 2 [1 − cos(vc t)] vc vc La trajectoire est donc une cycloïde (cf. chapitre 2).

678

Solutions des exercices

2. Dans ce cas où le mouvement de particule est rectiligne et uniforme, la loi fondamentale de la dynamique donne : E ma = q(E + v × B) d’où E = −v × B = −v 0 × B et v0 = B Si v = v0 , ma = q(v − v0 ) × B = 0. À l’aide d’une fente placée sur l’axe Oy, on peut sélectionner les électrons dont la vitesse est v0 ; les autres sont déviés et donc arrêtés par la fente. S8– 5. Précession de Larmor Écrivons la loi fondamentale de la dynamique pour la particule A, dans le référentiel non galiléen R tournant autour de B et tel que sa projection P dans Oxy soit fixe dans R : ma A/R = qvA/R × B − mae − maC Projetons cette équation vectorielle dans la base de R : r ¨ 0 ¨z

= vc

0 0 1

r˙ rw˙ × z˙



−rw˙ 2 r¨ w 0

− 2

0 0 × w˙

r˙ 0 ˙z

d’où

(1) r ¨ = vc rw˙ + r w˙ 2 0 = −vc r˙ − rw ¨ − 2r˙ w˙ (2) z=0 (3) ¨

L’équation (2) entraîne : 0 = −v c r˙ −

1 d 2 (r w) ˙ r dt

soit

r2 d 2 (r w˙ ) = −v crr˙ et r 2 w˙ = −vc + Cte dt 2

en intégrant. Comme la particule se trouve en O pour t = 0, Cte = 0 et w˙ = −vc/2. La vitesse angulaire vL = −vc /2 est la pulsation de Larmor. En remplaçant w˙ par vL dans (1), on obtient : r ¨ + v2Lr = 0, ce qui traduit une oscillation de r de pulsation vL. S8– 6. Trajectoires trochoïdales d’électrons 1. Les valeurs des grandeurs demandées sont : eE eB 14 9 a1 = − vc = − = −1, 76 × 10 m . s −2 = −3, 52 × 10 rad . s−1 me me

et

E 4 = 5 × 10 m . s−1 B

2. Les équations paramétriques du mouvement ont été établies en cours. Introduisant t, elles s’écrivent : z=−

a1 t+ v2c

v c v0 − 1 sin t a1

et x = −

a1 v 2c

vc v 0 − 1 (1 − cos t) a1

Calculons les constantes : −

a1 Em e = 14, 2 mm = eB2 v2c

et

a1 E = = 5 × 104 m . s −1 B vc

Il vient : z = 14, 2(t − 0, 2 sin t) et x = −2, 84(1 − cos t). L’allure de la trajectoire est représentée sur la figure S8.1a. 3. Pour v0 = 8 × 10 4 m . s−1, on obtient : z = 14, 2(t + 0, 6 sin t) et x = 8, 52(1 − cos t). L’allure de la trajectoire est représentée sur la figure S8.1b. Pour v 0 = 12 × 104 m . s−1 , on trouve : z = 14, 2(t + 1, 4 sin t) et x = 19, 88(1 − cos t). L’allure de la trajectoire est représentée sur la figure S8.1c.

679

et problèmes x 0

z x

x

z

0

a)

z

0

b)

c)

F IG . S8.1.

Chapitre 9 S9– 1. Mouvement d’une masselotte guidée sur une courbe circulaire 1. L’énergie potentielle de pesanteur a pour expression : Ep = mgyA + Cte = mgl(1 − cos u) + Cte = mgl(1 − cos u)

puisque Ep = 0 pour u = 0. Comme il n’y a pas de frottement, l’énergie mécanique se conserve : E m = mgh =

1 2 mv + mgl(1 − cos u) 2

1/ 2

d’où v = [2gh − 2gl(1 − cos u)]

2. La réaction est donnée par la loi fondamentale de la dynamique ma = mg + R, ce qui donne en projetant : mv 2 = R − mg cos u l

d’où R = 2mg

h 3 − 1 + cos u l 2

3. La condition R > 0 entraîne mgh > mgl −(3/2)mgl cos u soit Em > Ep avec E p = Ep −(1/2)mgl cos u. À l’aide de la condition (1), Em  Ep(u) car Ek  0, et cette deuxième condition (2), E m  Ep, la discussion peut être menée graphiquement (Fig. S9.1) : i) Si 0 < mgh  mgl, le mouvement est oscillatoire. ii) Si mgl < mgh < 5mgl/2, le mobile quitte le guide. iii) Si mgh  5mgl/2, le mouvement est révolutif.

 c Dunod – Toute reproduction non autorisée est un délit

4. h = 3l : le mouvement est donc révolutif. En B (u = 0), v = (6gl)1/2 et R = 7mg. En T (u = p), v = (2gl) 1/2 et R = mg.

Énergie (2)

(1) 0 - mg l / 2

5mgl / 2 2mgl mgl µ

F IG . S9.1. S9– 2. Mouvement d’un corpuscule sur une courbe hélicoïdale 1. a) Projetons suivant la tangente à la trajectoire la loi fondamentale de la dynamique ma = mg + R : m

d2 s = mg · et = −mg(ez · e t) d t2

680

Solutions des exercices

Comme d x = −R sin u d u, d y = R cos u d u et d z = −R d u, il vient, compte tenu des conditions initiales : √ √ √ 2 2 2 2 2 1/ 2 d s = d u(R sin u + R cos u + R ) = R 2 d u d’où s = R 2u + Cte = R 2u On en déduit : d x/ d s 1 e t d y/ d s = √ R 2 R d z/ d s

d x/ d u d y/ d u d z/ d u

1 =√ 2

− sin u + cos u −1

√ (ez · e t ) = −1/ 2

et

m

mg d2 s = √ d t2 2

On obtient en intégrant et en tenant compte des conditions initiales : g gt ds = √ t + Cte = √ dt 2 2

gt 2 gt 2 et s = √ + Cte = √ 2 2 2 2

Ainsi, u = gt2 /(4R). On aurait pu établir ce résultat en appliquant le théorème de l’énergie mécanique, ce qui est commode car il n’y a pas de frottement : 1 m(˙x2 + ˙y2 + ˙z2 ) + mgz = Em 2 Il en résulte, en dérivant et en simplifiant par u˙ : 2mR¨ u − mg = 0

soit

soit m u˙ 2 R2 + mgR(2p − u) = E m

g ¨ u= 2R

g u˙ = t 2R

et u =

gt 2 4R

b) Au bout de la durée tB , l’angle u a varié de 2p. Donc : √ √ gt 2B √ = R 2Du = 2pR 2 2 2

d’où

2. a) a = at e t + a n en avec : at =

tB =

d2 s d = d t2 dt

8pR g

1/ 2

et vB =

ds dt

B

gt B = √ = (4pgR) 1/2 2

√ ds ˙ d ˙ (R 2u) u = du dt

√ Ainsi at = R 2¨ u. Notons que et = j. Comme v et a s’explicitent selon : u sin u + u˙ 2 cos u) ¨x = −R (¨ x˙ = −Ru˙ sin u 2 et ¨y = R(¨ y˙ = Ru˙ cos u u cos u − u˙ sin u) ˙ = −Ru˙ u z = −R¨ R z R ¨

u 2 + u˙ 4 ) et an = Ru˙ 2 puisque a2t = 2R 2 ¨ u2. on en déduit a2 = R2(2 ¨ √ √ Remarquons que a t = a · et = 2R¨ u et a n = a · en, avec e n tel que : u/ 2 = R 2¨ d et d et en du × = = r ds du ds

s’explicite selon

1 1 √ √ R 2 2

− cos u 1 − sin u = 2R −0

− cos u − sin u 0

avec en = − cos u ex − sin u e y . Finalement, on trouve r = 2R et an = (a · en ) = R ˙u2 . 2. b) La loi fondamentale, m a = m g + R où R = T e t + N ne n+ N beb avec e b = et × en , donne en projection : ˙2 0 √ Ru Nn √ ¨ + T (< 0) m R 2u = mg/ 2 √ Nb 0 −mg/ 2

Les quantités T et N = (N2n + N 2b)1/2 sont reliées |T | = mN soit T = −mN , puisque T < 0 . 2. c) L’équation du mouvement est donc :

Elle s’écrit aussi :

√ mg mg m2 g2 1/ 2 2 2 4 u = √ − m(Nn + N b) = √ − m m R u˙ + mR 2 ¨ 2 2 2 g g2 s˙4 ¨ + 2 s = √ −m 2 4R 2

1/ 2

1/ 2

681

et problèmes

√ Pout t = 0, s˙ = 0, ¨ s = g(1 − m)/ 2 ; si m < 1, ¨s  0, ce que l’on suppose, ˙s augmente et ¨ s diminue jusqu’à ce que : √ 1/ 2 g 2 g2 s˙ 4 1 = 0 d’où (s˙max )4 = 2R 2g2 −1 −m ¨s = + 2 m2 2 2 4R √ Ainsi u˙ tend vers la limite : u˙ max = s˙max /(R 2). S9– 3. Écart maximal d’un pendule sphérique avec la verticale 1. Comme la tension du fil ne travaille pas, l’énergie mécanique se conserve : Em = Ek + Ep =

mv 20 − mgl cos u0 2

2. a) Projetons suivant la verticale Oz le théorème du moment cinétique en O : d LO d LOz = OA × m g donne = 0 et L Oz = Cte dt dt b) On a OA = l er et vA = l u˙ e r + l sin uw˙ ew. c) À l’instant initial, vA est parallèle à ew. Lorsque u est maximal, u˙ = 0 ; par conséquent, dans ce cas aussi, vA et e w sont parallèles : donne mv 0l sin u 0 = mvml sin u m

(L Oz )u0 = (LOz )u m

3. Si u m = p/2, v0 = vm/ sin u 0 . Or mv20 /2 − mgl cos u0 = mvm2 /2. Donc : v0 = (2gl/ cos u 0)1/2 . S9– 4. Masselotte en contact sans frottement avec une demi-sphère en rotation uniforme 1. Exprimons dans la base de R les différents vecteurs : Il vient, respectivement : 0 0 V

 c Dunod – Toute reproduction non autorisée est un délit

R

×

ve = V × OA

x −yV y = xV z R 0

R

ae = V × (V × OA) et

0 0 V

×

−yV = xV 0

R

−V2x −V2y 0

a C = 2V × v

et

0 0 V

×

x˙ y˙ = z˙

−V˙y −V˙x 0

2. Comme R n’est pas galiléen, le bilan des forces est : m g, R (normale à S ), −m ae = m V 2 HA et la force de Coriolis −m aC = −2m V × v. Dans ces expressions, H est la projection de A sur l’axe Oz0 ; R peut s’écrire −Rr/r0 car R est dirigée suivant r dans le sens de −r (pas de frottement). La loi fondamentale ma = m g + R + m V 2 HA − 2m V × v se projette selon : 0 ¨x R − m ¨y = mg 0 r0 ¨z −1

d’où :

R 2 x + mV x + 2mVy˙ r0 R ¨ − V2 y = 0 y + 2V x˙ + mr0 m¨x = −

soit

x y + mV 2 z

x y − 2m 0

−V˙y −V˙x 0

R R 2 − V2 x = 0 m¨ y = − y + m V y − 2mVx˙ mr0 r0 R R ¨z + m¨z = −mg − z z+g= 0 r0 mr 0 ¨ x − 2Vy˙ +

3. a) L’énergie potentielle de pesanteur est : E p = mgz + Cte = mgz puisque Ep = 0 pour x = 0. b) La force d’inertie d’entraînement dérive aussi d’une énergie potentielle : −m ae · d r = mV 2(x d x + y d y) =

mV2 mV 2 2 d(x2 + y 2) = − d − (x + y 2) 2 2

682

Solutions des exercices

Donc, si on prend Ep,c = 0 pour z = 0 : E p,c = −

mV2 2 mV 2 2 mV2 2 mV 2 2 mV 2z2 (x + y 2) + Cte = − (r 0 − z2 ) + Cte = z − r0 + Cte = 2 2 2 2 2

c) L’énergie potentielle totale est donc : E p, = mgz + mV2 z2/2. 4. a) Le graphe f (u) est une parabole : f (u) = u2 +

2v 20 2v 2 u = u u + 20 2 V V

et f (u) = 2u +

2v20 = V2

2u +

v 20 V2

On en déduit : f

z r0

=

z2 z2 2v 2 z 2g z 2 + 20 = 2 + 2 = 2 r0 r0 mV2r20 V r0 V r0

mgz +

mV2 z2 2

d’où Ep =

mV2 r 20 f 2

z r0

b) L’énergie mécanique dans R se conserve car P  = 0 : Ek + E p = Em = Cte d’où E k = Em − Ep  0. i) Si E m  0, zmin  z  0, la bille évolue entre z min et z = 0. La valeur z > 0 est exclue puisqu’elle ne respecte pas le contact. ii) Si −mv 20r02/2 < Em < 0, alors z1  z  z 2 ; la bille évolue entre deux plans horizontaux parallèles. c) z = Cte pour f (u) minimal. D’où u m = −v20/V 2 et zm = −r 0 v 20/V2 = −0, 4 r0 . 5. À la position d’équilibre, on a :

F = 0 (Fig. S9.2) :

m g + R + m V 2 HA = 0

d’où R = −m(g + V2 HA) = −m g a

Comme tan u = V2 HA/g = V 2 r0/g sin u, il vient : V2 1 = 2 v cos u

d’où



v2 ze = 02 r0 V

et z e = −r 0

v20 = −0, 4 r0 V2

Le résultat ze = z m était prévisible puisque l’équilibre est un cas particulier du mouvement stationnaire en z1 : c’est le cas où, en plus de z = Cte, on a x = Cte et y = Cte.

p/2 O H

µe

R A

0

m Ω2 HA

mg

B Wc

W

-p / 2

F IG . S9.2.

F IG . S9.3.

S9– 5. Mouvement sans frottement d’une masselotte sur une tige en mouvement 1. La loi fondamentale de la dynamique par rapport à R s’écrit, maA/R = mg + R, ce qui donne, en explicitant en coordonnées polaires, c’est-à-dire dans la base tournante (er, e w) associée à la tige : 2 m(¨ r − r V ) = −mg sin(Vt ) et

m(2˙rV) = −mg cos(Vt ) + R

La première équation est l’équation différentielle du mouvement, la seconde donne l’expression de R : R = [mg cos(Vt) + 2mV˙r] ew 2. Le théorème de l’énergie mécanique par rapport à R donne : d(Ek + Ep ) = Pnc dt

avec

P nc = R · v A/R = RrV = 0

683

et problèmes

puisque la vitesse de A par rapport à R a deux composantes, l’une radiale r˙ et l’autre orthoradiale r v. On retrouve l’équation différentielle en r en remplaçant R par son expression et en explicitant Ek et E p : Ek =

1 m(˙r 2 + r2 V2) et 2

Ep = −mg · OA = mgy = mgr sin(Vt)

3. Le théorème de l’énergie mécanique Em par rapport au référentiel tournant R donne lui :

d(Ek + Ep ) nc = 0 puisque P = R · v A/R = 0 dt la vitesse vA/R  n’ayant qu’une composante radiale. On retrouve l’équation du mouvement en r en explicitant les énergies cinétique et potentielle : 1 2 1    m r˙ et Ep = Ep,g + Ep,c = mgr sin(Vt) − mV2 r2 2 2 En effet, dans R, il faut prendre en compte aussi l’énergie potentielle centrifuge, la force de Coriolis ne travaillant jamais. E k =

Chapitre 10 S10– 1. Composition des raideurs de ressorts 1. Appliquons la loi fondamentale à la masselotte A, de coordonnée x suivant la verticale descendante : m¨ x = −K1 (x − l 1) − K 2(x − l2 ) + mg soit

m¨ x + (K1 + K2 )x = K1 l 1 + K 2l2 + mg

On en déduit que la pulsation de l’oscillateur est : v0 = (K /m)1/2 avec K = K1 + K 2. Ainsi, en parallèle, les raideurs s’ajoutent (comme les capacités des condensateurs). 2. La loi fondamentale appliquée à la masselotte A, de coordonnée x, suivant la verticale descendante et au point I, de masse négligeable, situé entre les deux ressorts, donne respectivement : m¨ x = −K2 (x − x I − l 2) + mg et 0 = +K2 (x − xI − l 2 ) − K 1 (xI − l1 )

Il en résulte, en remplaçant dans la première équation l’expression de xI tirée de la seconde : m¨ x = −K2 x −

K2 x + K 1l1 − K2l 2 − l2 K1 + K2

+ mg

soit  c Dunod – Toute reproduction non autorisée est un délit

m¨ x+

K1 K2 x = mg + K2 K1 + K2

K1 l 1 − K2l2 + l2 K 1 + K2

= mg +

K 1K 2 (l1 + l2 ). K1 + K 2

On en déduit que la pulsation de l’oscillateur est : v0 = (K /m) 1/2 avec K tel que 1/K = 1/K1 + 1/K2. En série, les inverses des raideurs s’ajoutent (comme les capacités des condensateurs). S10– 2. Pendule simple et élastique 1. Appliquons la loi fondamentale de la dynamique au pendule, en projetant sur un axe horizontal et en adoptant comme origine des abscisses la position d’équilibre. Il vient, dans le cas où u est petit : 2K g x = −Kx − Kx − mgu soit ¨x + x=0 + m¨ m l puisque les variables linéique et angulaire sont reliées par : x = lu. On en déduit la période T0 : 2p 2K g 2 avec v 0 = + T0 = v0 m l 2. Le facteur de qualité vaut : Q = v 0te = v0 × 10te,c = 5 puisque Qc = v 0t e,c = 1/2.

684

Solutions des exercices

S10– 3. Oscillations d’un pendule élastique 1. a) Appliquons la loi fondamentale de la dynamique au point A par rapport au référentiel galiléen lié à la tige. Il vient : m¨ r e r = −K (r − l0 ) er + mg + R soit m¨r = −K (r − l0) + mg cos u en projection suivant la tige. On en déduit la position d’équilibre re = l 0 + mg cos u 0 /K . En posant R = r − re , on obtient l’équation caractéristique d’une oscillation harmonique : ¨ R + v20 R = 0

avec

K m

v20 =

d’où T 0 =

2p = 0, 44 s v0

b) La force de frottement est de type visqueux : R = Rm exp −

On en déduit : L=

R0 1 ln n R 50

=

1 ln 3 = 0, 022 50

te =

t cos(va t + f) 2te

T T0 = 10 s ≈ 2L 2L

et Q = v0 te =

te 10 = 22, 7 = 0, 44 T0

2. a) On a :

1 2 1 2 Ep = mgz = mgr cos u0 E p = K (r − l 0 ) m˙r 2 2 On sait que la force d’inertie d’entraînement centrifuge à vitesse angulaire constante dérive de l’énergie potentielle : Ek =

Ep = −

mV 2HA2 mV 2r2 sin2 u0 =− 2 2

b) Appliquons le théorème de l’énergie dans le référentiel tournant où le système est conservatif : Ek + Ep = E m

soit

1 2 1 sin2 u 0 m˙r + mgr cos u0 + K (r − l 0)2 − mV 2r2 = Em 2 2 2

En dérivant, on obtient : m¨r + mg cos u0 + K (r − l0) − mV 2 r sin2 u20 = 0 ce qui s’écrit ¨ r + (v 20 − V 2 sin2 u 0)r = v 20l 0 − g cos u 0 c) On en déduit la position d’équilibre en faisant r = Cte : re =

l0 − g cos u0 /v20 1 − (V2 /v20 ) sin 2 u0

La pulsation v0 du mouvement oscillant est telle que :

v0 2 = v 20 − V 2 sin 2 u 0 = 0, 19v20

d’où T0 =

T0 2p ≈1s =  v0 0, 436

S10– 4. Accéléromètre à pendule élastique 1. Dans le référentiel non galiléen R , en translation par rapport au référentiel terrestre, la loi fondamentale appliquée au point A de l’oscillateur s’écrit, en projection suivant l’axe Ox du mouvement :    m¨ x = −a˙x − K (x − l 0 ) − mae

2. Au bout d’une période, la masselotte s’immobilise sur une position xe telle que, si l’on pose v20 = K /m : ae   x e = l0 + 2 soit ae = v20(x e − l0) v0 Un tel système permet donc de déterminer l’accélération ae du véhicule connaissant l’allongement et v 0.

685

et problèmes S10– 5. Influence de l’amplitude sur la période des oscillations d’un pendule simple 1. La conservation de l’énergie appliquée à un pendule simple s’écrit : 1 2˙2 ml u − mgl cos u = −mgl cos u0 2 ce qui donne, en dérivant et en simplifiant : 2¨ + mgl sin u = 0 ml u

2 ¨ u + v0 u = 0

soit

2

en posant

v0 =

g l

si u est petit. On en déduit la période propre : T0 = 2p(l/g)1/2 . 1/ 2

2. Comme d u/ d t = ± [4p/T0 (cos u − cos u0)] pour expression : u0

dt = 4

T(u0 ) =

0

dt du

d u = T0 J

avec

= ±(T0/p)[sin 2 (u0 /2) − sin2 (u/2)]1/2 , la période a 1 p

J=

u0

d u sin

u0 2

2

0

2

− sin

u 2

−1/2

3. Comme sin(u/2) = g sin f, il vient, en différenciant : cos(u/2) d u = 2g cos f d f. On en déduit : J=

p/2

2 p

0

df = cos(u/2)

2 p

p/2 0

Ainsi : g2 J = 1+ p

p/2 0

2

sin f d f + · · · ≈ 1 +

p/2

df 2 ≈ 2 2 −1 / 2 p (1 − g sin f)

g2 +··· 4

df 1 + 0

d’où T (u0) ≈ T 0

1+

sin 2 u0 /2 4

g2 2 sin f + · · · 2

≈ T0

1+

u20 16

S10– 6. Pendule élastique à deux positions d’équilibre stables 1. Par définition, la force de rappel qu’exerce le ressort sur la masselotte A a pour expression : F = −K (FA − l0 ) e r = −K[(h 2 + x2)1/2 − l 0 ] er

On en déduit la travail élémentaire :

x FA

dW = −K (FA − l0 ) er · e x d x = −K (FA − l0 )

d x = −Kx 1 −

l0 dx x 2) 1/ 2

(h2 +

et l’énergie potentielle associée définie par : dW = − d Ep . En effet, dW s’écrit :

 c Dunod – Toute reproduction non autorisée est un délit

dW = −Kx 1 −

l0 d x = − d Ep 2 (h + x2)1/2

avec

Ep = K

x2 − Kl 0(h 2 + x 2 )1/2 + Cte 2

La constante additive de Ep est obtenue en faisant Ep (x) = 0 pour x = 0 : Ep = 0 = −Kl 0 (h2 ) 1/2 + Cte

d’où E p =

1 2 Kx − Kl0(h2 + x 2 )1/2 + Kl 0 h 2

2. Si h = 0, on a :

1 2 (x − l0 )2 Kl 2 − 0 Kx − Kl0x = K 2 2 2 ce qui est caractéristique, à une constante additive près Kl20/2, de l’énergie potentielle d’un oscillateur harmonique. La fréquence d’oscillation est : v0 1 K 1/ 2 = f0 = = 10 Hz 2p 2p m Ep =

3. La fonction E p(x) est symétrique. La dérivée, qui s’identifie au signe près à la force, est nulle, si : Kx 1 −

l0 =0 (h2 + x 2)1/2

soit

x=0

ou

h 2 + x 2 = l20

686

Solutions des exercices

En dérivant une autre fois, on a le signe de la dérivée seconde par rapport à x : K 1−

(h2

l0 + Kl 0 x2 (h2 + x2 )−3/2 2 1 / 2 +x )

qui vaut K (1 − l0/h) pour x = 0 et Kl0 x2(h2 + x2 )−3/2 = K (1 − h2 /l20 ) pour x tel que h2 + x 2 = l 20. On distingue alors trois cas (Fig. S10.1) : i) l 0 < h ; la seule position d’équilibre est x = 0. Comme K (1 − l0/h) > 0, c’est une position d’équilibre stable. ii) l 0 = h ; la seule position d’équilibre est x = 0. C’est une position d’équilibre indifférent. iii) l0 > h ; il existe trois positions d’équilibre : – la position x = 0 correspond à un équilibre instable puisque K (1 − l0 /h) < 0, – x = ±(l20 − h 2 )1/2 sont des positions d’équilibre stable puisque K (1 − h2 /l 20) > 0.

Le théorème de l’énergie donne, autour d’une position d’équilibre stable, puisque le système est conservatif : d2 E p d x2

1 2 mx˙ + 2

e

x2 = Em 2

d’où les pulsations caractéristiques pour l0 < h et l0 > h respectivement : v1 =

K 1/ 2 l0 1− m h

1/ 2

= v0 1 −

l0 h

1/ 2

et v 2 =

K 1/ 2 h2 1− 2 m l0

= v0 1 −

h2 l 20

1/ 2

4. Pour h = 10 cm et l 0 = 15 cm, la position d’équilibre stable et la fréquence des oscillations autour de cette position sont respectivement : xe = ±(l 20 − h2 )1/2 = ±11, 8 cm et f 2 = f 0 1 −

E p (x )

h2 l20

1/ 2

= 4, 28 Hz

l0 < h l0 = h l0 > h

.

0

.

x

F IG . S10.1. S10– 7. Pendule élastique bidimensionnel à quatre ressorts 1. En raison de l’origine choisie et de la symétrie du problème, l’énergie potentielle ne comporte pas de terme constant ni de terme linéaire en x ou y, d’où la forme Ep = a(x2 + y 2). Pour déterminer a, il suffit de se placer dans le cas particulier d’une oscillation suivant l’axe des x : on sait que les raideurs s’ajoutent. Par conséquent, il vient : 1 E p = (2K )x 2 = ax2 d’où en identifiant a = K et Ep = K (x 2 + y 2 ) 2 2. Explicitons la loi fondamentale de la dynamique ma = F. Il vient : ∂Ep ∂Ep m¨ x=− = −2Kx et m¨y = − = −2Ky ∂x ∂y Le système oscille, avec la fréquence f0 = (2K /m)1/2 /(2p), dans les deux directions perpendiculaires.

687

et problèmes 3. Le mouvement est une combinaison de deux mouvements sinusoïdaux de même pulsation : x = xm cos(v0t + fx ) et

y = ym cos(v0 t + fy )

On sait que la trajectoire de A est une ellipse inscrite dans le rectangle de côtés 2xm et 2y m. La structure de l’ellipse est définie par le déphasage f = fx − fy . Si f = 0 ou p, l’ellipse se réduit à une droite. S10– 8. Pulsation d’un plasma. Plasmon 1. Si les électrons se déplacent de x, des charges excédentaires +e et −e apparaissent aux extrémités. Le champ électrique à l’intérieur du plasma est analogue à celui existant entre les armatures du condensateur ainsi formé. On sait que (cf. Électromagnétisme) : s 1 ne eSx ne ex = = E= ε0 ε0 ε0 S 2. Le mouvement d’un électron du plasma satisfait à l’équation : me ¨ x=−

e2 ne x ε0

d’où

¨ x + v2p x = 0

avec

vp =

n ee 2 m e ε0

1/ 2

=

4pne q 2e me

1/ 2

3. Pour Al, n e = 3NAr/A = 1, 8 × 1029 m−3 , f p = 3, 82 × 1015 Hz et vp = 15, 75 eV. S10– 9. Potentiel de Morse 1. Cherchons le minimum de E p(r) en dérivant cette fonction par rapport à r :

dEp = −2aE0 {exp[−2a(r − r 0)] − exp[−a(r − r 0)]} = 0 pour r = r0 dr On en déduit Ep (r0 ) = −E0 . Cette position d’équilibre correspond à un minimum, car : d2 Ep,ef dr2

0

= −2aE0 {(−2a) exp[−2a(r − r 0)] + a exp[−a(r − r0)]} 0 = 2a2 E0 > 0

Ainsi, r0 est la position d’équilibre, E0 est l’opposée de la valeur minimale de Ep (r) et K = 2a 2E0 directement relié à la raideur du ressort équivalent dans l’approximation quadratique de l’énergie potentielle. 2. Effectuons le développement de Ep (r) : Ep,ef (r) = Ep,ef (r0 ) +

 c Dunod – Toute reproduction non autorisée est un délit

soit :

(r − r 0) 2 2!

d2 E p,ef d r2

E p,ef (r) = −E0 + K

avec : A=

d3 Ep,ef d r3

0

(r − r 0 ) 3 3!

d3 Ep,ef d r3

+... 0

(r − r0 )2 (r − r 0)3 +A 2 6

2

0

+

2

= −2aE0 (4a ) exp[−2a(r − r 0 )] − a exp[−a(r − r0 )]

3

0

= −6a E 0

Le calcul donne K = 2a2E0 = 2 325,4 N . m−1 et A = 2,19 × 10 24 J . m−3 . S10– 10. Période du botafumeiro On sait que la période T a pour expression : T = T0 (1 + u 2m/8) 1/2 avec T 0 = 2p(l/g)1/2 . Le calcul donne : T 0 = 2p

l g

1/ 2

= 9, 10 s

T1 = T0 1 +

132p 2 1802 × 8

1/ 2

= 9, 13 s et T2 = T0 1 +

822 p2 1802 × 8

1/ 2

= 10, 2 s

La concordance avec les valeurs expérimentales est plus satisfaisante pour la faible amplitude, comme le prévoit l’analyse.

688

Solutions des exercices

Chapitre 11 S11– 1. Pendule élastique horizontal soumis à l’action d’une force sinusoïdale 1. La loi fondamentale de la dynamique donne, avec les notations habituelles : ma = m g + R − K (x − x S − l 0 ) ex − a v d’où

x = −K (x − xS − l 0) − a˙x m¨

en projetant sur l’axe horizontal Ox. Comme xS = dm cos(vt + fe ), l’équation canonique à laquelle satisfait X = x − l0 est : X˙ + v 20X = v 20d m cos(vt + fe) X¨ + te 2. L’oscillateur étant faiblement amorti, la période d’oscillation libre est voisine de la période propre. On a donc : T T0 t 2p 0, 5 ≈ 0, 5 s te = = 5 s et Q = v0 te = 2p e ≈ 63 ≈ = T0 = 1 / 2 (K / m ) T0 2L 2L 0, 1 3. L’amplitude de la vitesse est maximale pour f 0 = 2 Hz ; Z se réduit alors à a = mte = 0, 5 SI. S11– 2. Oscillations forcées d’une particule chargée dans un champ électrique sinusoïdal 1. La loi fondamentale appliquée à la charge donne : ma = −K r + qE d’où, en projection suivant l’axe Ox : qE 0 cos(vt) m¨ x + Kx = qE0 cos(vt) et ¨x + v20 x = m 2. La solution est une combinaison linéaire de la solution générale de l’équation homogène et d’une solution particulière de l’équation complète : x = A cos(v0t + f) +

qE0 cos(vt) m(−v 2 + v 20)

soit x =

qE0 [cos(vt) − cos(v0t)] m(−v2 + v 20 )

compte tenu des conditions initiales x(0) = 0 et ˙x(0) = 0. Notons qu’en l’absence de terme d’amortissement le premier terme ne peut être omis. 3. Si v ≈ v0 , alors : x≈

qE0 sin[(v − v 0 )t/2] sin(vt) (v − v 0) mv

et E ≈

1 2 1 q2E20 sin2[(v − v 0)t/2] Kx m = mv20 x2m = 2 2 2m (v − v0 )2

S11– 3. Pendule élastique sur une tige en mouvement horizontal Appliquons la loi fondamentale par rapport au référentiel non galiléen R lié à la tige horizontale : ma = mg + R + T + Ff − maO

˙ ¨  + X + v20X  = v2 dm cos(vt) ce qui donne X te

en projection horizontale suivant l’axe Tx ; on a posé X = x  − l0 avec x  = TA, v20 = K /m et te = m/a. Le poids et la réaction se compensent verticalement. La méthode complexe de résolution de cette équation différentielle donne :  v X˙  2 2 2 ¨ + v20 X = v2e dm exp(jvt ) d’où l’on déduit X + −v + j + v0 X m = v dm te te On trouve, en introduisant Q = mv0 /a et u = v/v0 : Xm =

Q 1 dm ≈ Qdm [1/u2 + Q2 (1 − 1/u2 )] 1/2 [1/u 2 + Q 2(1 − 1/u2)] 1/2

689

et problèmes Xm est maximal pour u ≈ 1 et vaut Qdm . L’amplitude est divisée par Q2 u −

2

1 u

=1

soit u =

√ 2 pour :

1 1 1 ± ≈ 1± u Q Q

Il en résulte que u2 − u1 ≈ 2/Q soit Dv = v2 − v 1 ≈ 2v0/Q = 2/t e et Dv × te ≈ 2. S11– 4. Isolement de vibration 1. L’équation différentielle à laquelle satisfait la coordonnée verticale x, comptée à partir de la position de repos, s’écrit si xS désigne la coordonnée donnant la position du support à partir de sa position de repos : M¨ x = −M v 20(x − x S ) − a˙x + mV2r cos(Vt ) 2. La recherche d’une solution forcée, de la forme x = x m cos(Vt + fx ) donne, sachant que |xS|  x m : 2

2

2

xm (−MV + Mv + jV) = mV r

soit x m =

(m/M )V2 r |v20 − V2 |

puisqu’on néglige l’amortissement. Comme la force qui s’exerce sur le support est K (x − xS ) ≈ Kx, le module du facteur d’amplification est : 1 Kxm = H(V) = 2 |1 − V2 /v20| mV r Pour que H(V) soit faible, il faut que v0  V : H(V) = 0, 001 pour v0 ≈ V/31, 6. On réalise cette condition en intercalant entre la machine tournante et son support des ressorts très souples ou du caoutchouc (K faible). S11– 5. Force sinusoïdale sur un oscillateur harmonique de pulsation différente L’équation différentielle du mouvement est : ¨ x + v20 x =

Fm cos(vt) m

soit ¨ x + v20 x = 40 cos(vt )

La solution s’écrit : x(t) = A cos(v0t) + B sin(v 0t) + C cos(vt + fx ). L’amplitude C de la solution particulière s’obtient aisément sans utiliser la méthode complexe, puisqu’il n’y a pas de terme dissipatif. Il suffit d’injecter cette solution dans l’équation générale : C(−v2 + v20 ) cos(vt + f x ) = 40 cos(vt ) d’où

C=

40 ≈ 5 cm 4p | − 36 + 16| 2

et

fx = p

 c Dunod – Toute reproduction non autorisée est un délit

Pour trouver A et B, il suffit de tenir compte des conditions initiales dans l’expression générale : x(t) = A cos(v0 t) + B sin(v 0t) − C cos(vt ) On trouve : 0 = A − D et 0 = Bv0. Il en résulte que x(t) = 5[cos(v0 t) − cos(vt)] = 5[cos(8pt) − cos(12pt)]. S11– 6. Force sinusoïdale sur un oscillateur harmonique de même pulsation 1. L’équation différentielle du mouvement selon la verticale descendante, comptée à partir de la position d’équilibre est : F x = −Kx + F (t) d’où ¨ m¨ x + v20x = cos(vt) m 1/ 2 −1 −1 avec v0 = (K /m) = v = 4 rad . s et F /m = 12 N . kg . 2. Comme v = v 0 , la solution particulière de l’équation différentielle a la forme singulière donnée. Il vient, en dérivant : ˙x = [A cos(4t ) + B sin(4t )] + 4t[−A sin(4t) + B cos(4t )] et ¨x = −16x + 8[−A sin(4t ) + B cos(4t )]

690

Solutions des exercices

En injectant la forme donnée de x (t) dans l’équation différentielle, on obtient : 8[−A sin(4t ) + B cos(4t)] = 12 cos(4t)

d’où A = 0

et

B=

3 2

Ainsi, x(t) = 12t cos(4t) est une solution particulière. 3. La solution générale s’écrit donc : x(t) = C cos(4t) + D sin(4t) + 12t cos(4t). Les conditions initiales donnent : x(0) = C = 0 et x˙(0) = 4D − 48 = 0. Finalement (Fig. S11.1) : x(t) = 12 sin(4t ) + 12t cos(4t).

x

0

t

F IG . S11.1. S11– 7. Diffusion de la lumière 1. L’application de la loi fondamentale à un électron donne l’équation suivante suivant la direction Ox du champ électrique E : x˙ eE 0 2 ¨x + cos(vt) + v0x = − te me 2. La recherche de solution de la forme exp(jvt) donne : xm = d’où : H=

me [(v 20

eE 0 − + v 2/t 2e ] 1/2 v2) 2

tan f x =

v/te v20 − v 2

KX m Q Q = = 2 2 2]1/2 2(u 2 eE0 [u + Q u[1 + Q (u − 1/u) − 1)2 ]1/2

3. Comme Q = v0 te = 10, H passe par un maximum pour u ≈ 1. Pour u ≈ 0, 2, H varie peu puisque : dH du

u=0,2

=

−Qu[1 + 2Q 2(u2 − 1)] ≈ 0, 039 [u2 + Q2(u2 − 1)2 ]3/2

4. La puissance rayonnée P est proportionnelle au carré de l’accélération et donc à v2H 2 . Comme H est pratiquement indépendant de v, P varie comme v4 , ce qui privilégie les radiations bleues du spectre de la lumière solaire. S11– 8. Capteur de vibration 1. Les caractéristiques v 0 et te de cet oscillateur valent respectivement : K 1/ 2 1 = 10 rad . s−1 et te = 2t e,c = = 0, 1 m v0 On en déduit le facteur de qualité : Q = 1 . v0 =

puisque vet e,c = 0, 5

2. L’équation différentielle à laquelle satisfait le mouvement de la masselotte par rapport au capteur est (cf. chapitre 11) : ˙ ¨ + X + v 20X  = v2dm cos(vt) X te

691

et problèmes

3. Cherchons une solution particulière de l’équation différentielle complexe associée de la forme : X  = X m exp(jvt ). Il vient, en substituant : −v2 + v20 + j

v  X m = v2 dm te

soit X m =

u −ju dm = Qdm 1/u − u + j/Q 1 + jQ(u − 1/u)

On en déduit :  Xm =

[1

+ Q 2(u

u Qdm − 1/u) 2] 1/2

et fx = − arctan Q u −

1 u



p p = −w − 2 2

Le maximum de Xm s’obtient en annulant la dérivée de la fonction : f (u) =

1 1 + Q2 1 − 2 u2 u

2

Qdm Xm

car

2

=

1 1 u2 = 2 + Q2 1 − 2 1 + Q 2(u − 1/u)2 u u

2

Il vient :

df 2 1 1 −1/2 = 0 soit u = 1 − = 3 −1 + 2Q2 1 − 2 du 2Q 2 u u √ Pour Q = 1, on trouve u = 2 = 1, 414 et Xm = 1, 155d m. Sur la figure S11.2a, on a représenté le graphe X m(u) pour Q = 10. Lorsque u tend vers l’infini, Xm tend vers dm . 4. L’amplitude complexe de la vitesse se déduit aisément de celle de l’élongation : v m = jvXm = v0 d’où (Fig. S11.2b) : vm = v0

u2 Qdm 1 + jQ(u − 1/u)

u2 Qdm [1 + Q (u − 1/u)2 ]1/2 2

et fv = f x +

X m

p = −w 2

vm0

dm

0

u

1

0

1

 c Dunod – Toute reproduction non autorisée est un délit

a)

u

b) F IG . S11.2.

S11– 9. Pendule simple excité par des oscillations horizontales 1. On obtient l’équation différentielle du mouvement en appliquant le théorème du moment cinétique au point I , mobile dans R , par lequel passe, à chaque instant, la réaction qu’exerce le guide sur A : d LI + vI × mvA = IA × mg dt dans laquelle le second membre de l’équation s’explicite aisément selon −mgl sin u ez . Pour exprimer vA , et donc LI = IA × mv A , explicitons OA = OI + IA dans la base B cartésienne de R . Il vient, pour OA et vA respectivement : l cos u −lu˙ sin u y I + l sin u

B

0

˙yI + lu˙ cos u

d’où en dérivant

B

0

692

Solutions des exercices

Il en résulte, respectivement pour le moment cinétique LI et le terme complémentaire : 2 ˙ u + sin2 u) + ml˙yI cos u] ez = (ml 2u˙ + mly˙ I cos u) ez LI = [ml2 u(cos

et v I × mvA = −mly˙ Iu˙ sin u e z

Par conséquent, en effectuant et en simplifiant par ml2 , on trouve l’équation différentielle suivante : ˙yI y¨I g ¨ u − 2 ˙u sin u + cos u = − sin u l l l

où v0 =

g l

1/ 2

= 4, 95 rad.s−1

est la pulsation propre d’un pendule simple qui oscille faiblement autour de sa position d’équilibre (cf. Leçon 7). 2. Si les mouvements sont petits (sin u ≈ u , cos u ≈ 1) , l’équation précédente donne, en négligeant le terme en y˙ I ˙u qui est un infiniment petit d’ordre deux : hm ¨ sin(vt) u + v 20u = −v2 l

car ¨ y I = −v2 h m sin(vt)

Cette équation rappelle celle caractéristique d’un phénomène de résonance (cf. Leçon 15). 3. La solution de l’équation différentielle précédente est la somme de la solution libre ul (t) (sans second membre) et de la solution particulière up (t) : u(t) = ul(t) + u p(t)

avec

ul (t) = Al cos(v0 t + f) et

up(t) = A p sin(vt)

Al et A p étant deux constantes réelles. Cette solution particulière satisfait à l’équation : (−v 2 + v 20)A p sin(vt) = −v2

hm sin(vt) l

Il en résulte : u(t) = A l cos(v0 t + f) + Comme initialement u = 0 et u˙ = 0 , on a respectivement : A l cos f = 0

d’où

f=

p 2

Finalement : u(t) =

et

− v0 Al + v

d’où A p =

v2 hm v20 − v 2 l

hm v 2 sin(vt) l(v20 − v2)

hmv2 =0 l(v20 − v2 )

d’où

Al =

h mv2 v v0 l(v20 − v 2 )

hm v2 v sin(vt) − sin(v0 t) l(v 20 − v 2 ) v0

Le mouvement est donc la superposition de deux mouvements sinusoïdaux d’amplitudes et de fréquences différentes. 4. a) Pour v/v 0  1 : u(t) ≈

hm l

v v0

2

sin(vt) ≈ 0

2 Si f = 0, 01 Hz , alors que f0 = v0 /(2p) = 0, 788 Hz , le facteur d’amplification vaut : H = (v/ v0) = 0, 00016. Le système se comporte comme un étouffeur de vibration.

b) Pour v/v 0  1 :

hm v sin(v0t) l v0 Si f = 100 Hz , alors que f0 = 788 Hz , le facteur d’amplification vaut : H = v/v0 = 126, 9. Le système se comporte comme un amplificateur de l’amplitude des oscillations du guide. u(t) ≈

693

et problèmes

Chapitre 12 S12– 1. Oscillateur bidimensionnel 1. La force s’obtient selon : F = − grad Ep (r) = −d E p/ d r e r = −Kr er = −Kr.

Elle est caratéristique d’un oscillateur. On voit en outre qu’elle est centrale, ce qui implique que le moment cinétique de la particule à l’origine O soit une constante vectorielle. Le mouvement est donc plan. 2. Les lois de conservation du mouvement de la particule, en fonction des coordonnées polaires (r, w), sont celles fournies par la constance du moment cinétique LO et celle de l’énergie mécanique E m. On obtient : 2

mr w˙ = L et

1 2 2 2 2 m(˙r + r w˙ ) + Kr = E m 2

La valeur du moment cinétique se calcule aisément à l’aide des valeurs initiales : LO = mr0 × v0 = mr0 v0 sin a 0 e z

d’où L = 0, 1 × 1, 2 × 10 × 0, 5 = 0, 6 SI

3. L’énergie potentielle effective s’écrit, en fonction de r : L2 1 1, 8 = 20r 2 + 2 E p,ef = Kr 2 + 2 2mr 2 r On voit que Ep,ef tend vers l’infini pour r = 0 et r = ∞. S’il existe un extrémum, c’est un minimum. Pour le déterminer, dérivons cette fonction par rapport à r : d E p,ef L2 = Kr − 3 = 0 pour dr mr

r=

L2 mK

1/ 2

= 0, 3 m

Sur la figure S12.1 on a représenté avec soin le graphe Ep,ef (r). 4. Les valeurs minimale et maximale de r qui correspondent à une énergie mécanique déterminée Em sont données par l’équation de l’énergie dans laquelle on fait r˙ = 0 :

 c Dunod – Toute reproduction non autorisée est un délit

1 2 Em 2 L2 L2 4 d’où r − 2 =0 = E + Kr + r m 2 2mr2 K mK La résolution donne : r2 =

Em K

1± 1−

KL 2 mEm2

1/ 2

On trouve numériquement : rmax = 2, 23 m et r min = 0, 132 m.

Ep, ef

r

0

F IG . S12.1.

694

Solutions des exercices

S12– 2. Périodes des planètes du système solaire Comme T 2/a3 = Cte, on en déduit : T /(1 an) = (a/aT )3/2 avec a T ≈ 1, 49 × 1011 m. D’où le tableau S12.1 :

T (année)

Me

Ve

Ma

Ju

Sa

Ur

0,243

0,61

1,87

11,86

29,56

84,1

TAB . S12.1. S12– 3. Vitesses d’évasion vl,T

1/ 2 = vl,T (M ∗ /R∗ )1/2 avec : M ∗ = M /MT , Le calcul donne : vl ≈ (2GM/R) 1/ 2 −1 = (2GMT /R T ) = 11, 2 km . s . On en déduit le tableau S12.2.

vl( km . s −1)

Lu

Ve

Ma

Ju

2,35

10,3

5

60,3

R ∗ = R/R T

et

TAB . S12.2. S12– 4. Satellite Arabsat, Spot et satellite d’observation à haute résolution 1. a) Comme d LT / d t = 0 , le vecteur LT = TS × mvS = Cte ; le mouvement est donc plan. Comme, L = mr2 w˙ = Cte et r = Cte impliquent w˙ = Cte, le mouvement est aussi uniforme. 1/ 2 b) La relation mv 2/r = GmM T /r2 , MT étant la masse de la Terre, donne : v = (GMT /r) . En outre :

T=

2pr 2pr3/2 = v (GMT )1/2

d’où

4p2 T2 = Cte = r3 GMT

c) Les différentes énergies s’écrivent : 1 GmMT K GmMT E k = mv 2 = Ep = − = − r r 2 2r d) Au sol : Ep = −

GmMT R

et Em = Ek + Ep = −

GmMT 2r

1 2 1 et E k = mv sol = mV2 r2 cos2 l 2 2

2. a) On a, puisque vS/R = 0 : vS/R T = vS/R + VR/RT × TS = V R/RT × TS

Ainsi, vS est perpendiculaire à V ; la trajectoire de S est dans le plan perpendiculaire à V et passant par T (plan équatorial). Comme TS · d TS/ d t = d(TS)2 / d t = 0, la distance TS est constante et le mouvement circulaire : vs = r sV =

GMT rs

1/ 2

3

2

d’où rs V = GMT

On trouve rs = 42 219 km, vs = rs V = 2prs /T = 3, 079 km . s−1. b) L’énergie à fournir est : DE m = −

1 GmM T GmMT − mV2 R2T cos 2 l − 2r 2 RT

=m

GM T RT



RT V 2R 2T cos2 l +1 − 2r 2

Ainsi, DEm est plus faible près de l’équateur. 3. a) La vitesse de satellisation est : v=

GMT RT

1/ 2

RT r

1/ 2

= 7, 9 × 103 1 +

h R

On trouve : v = 7, 43 × 103 m . s−1 et T = 6 616 s soit 1 h 42 .

−1/2

et T =

2p(R T + h) v

695

et problèmes

b) Le nombre de points sur une ligne est : (9 × 106 )1/2 = 3 000. Quant à la longueur d’un côté, elle vaut : 832 × tan 4◦ ≈ 58, 084 km. On en déduit la taille d’un point 58 084/3 000 ≈ 19, 4 m. 4. a) Dans le cas d’un satellite d’observation, on a : v=

GM T r

1/ 2

= 7, 791 km . s−1

d’où T =

2p(RT + h) = 5 306 s v

soit 1 h 28

b) Le théorème de l’énergie mécanique donne : D(Ek + Ep ) = dW(nc) < 0, d’où Em diminue et r aussi puisque Em = −GmMT /(2r). Il en résulte que Ek augmente (Ek = −Em ). Cet effet paradoxal est dû à l’influence de E p. c) En différentiant, le théorème de l’énergie mécanique permet d’écrire : bv 2 GmM T 2pr Dr = − h 2r 2

d’où Dr = Dh = −

4pb r3 v2 4pb 2 4pbR2 = −28, 6 m =− r ≈− h GmM T hm hm

S12– 5. Orbite de transfert d’Hohmann 1. L’orbite basse doit être dans le plan équatorial car le mouvement est plan et l’orbite haute géostationnaire contenue dans le plan équatorial (cf. chapitre 12). 2. Sur la trajectoire elliptique de transfert, l’énergie n’est fonction que du demi-grand axe de l’ellipse, d’où : Ep GM T GMT =− =− 2a m R 1 + R2 La durée du séjour du satellite sur l’orbite de transfert est T/2 avec T tel que (3e loi de Kepler) : 4p2 T2 = [(R1 + R2 )/2]3 GMT

2

3

soit T = (R1 + R 2)

p2 2GM T

S12– 6. Excentricité d’une trajectoire de satellite Le vecteur excentricité, qui est une constante du mouvement dans le problème de Kepler, a pour expression : h=

mv0r0 Lz v 0 + 1 ew = v + ew = − K GmM T

1−

v20 v2s

ew

 c Dunod – Toute reproduction non autorisée est un délit

puisque v2s = G(MT + m)/r0, m = M T m/(M T + m) et v0 est porté par ew . Si v0 = vs, l’excentricité est nulle, ce qui était prévisible puisque la trajectoire est alors circulaire. Pour v0 = 0, 5vs , e = 0, 75 ; pour v0 = 2v s , e = 3. S12– 7. Mouvement hyperbolique d’un satellite artificiel 1. On a :

GMT 1/2 mv 20 GM T m mv 2 = 7, 8 km . s −1 et E m = − = − 0 = −6, 08 × 1010 J r0 r0 2 2 2. a) L = Cte = r0 × mv0 définit le plan de la trajectoire : 1 GMT m = 7, 44 × 1010 J L = mr0v 0 = 1, 842 × 10 14 J . s et E m = mv12 − 2 r1 v0 =

b) Comme E m > 0, la trajectoire est une hyperbole (cf. chapitre 12) ; p est le paramètre de la conique et e > 1 son excentricité. c) D’après ce qui précède : p=

L2 7 = 2, 12 × 10 m GMT m 2

et r 0 =

p 1+e

d’où e =

p − 1 = 2, 22 r0

696

Solutions des exercices

S12– 8. Trajectoire de Télécom 2A 1. Les vitesses de satellisation vp et va , respectivement pour h p = 300 km et ha = 35 980 km, se calculent aisément : 1/ 2 1/ 2 GM T GM T = 7, 73 km . s−1 et v a = = 3, 07 km . s−1 vp = R T + hp R T + ha 2. La trajectoire géostationnaire doit être dans le plan équatorial (cf. chapitre 12). Il en est donc de même pour l’orbite de transfert et l’orbite circulaire initiale. 3. a) La vitesse vp que doit avoir le satellite au périgée pour qu’il prenne l’orbite de transfert est telle que : 2

mv  p GMT m GMT m − =− rp rp + ra 2

2

d’où v p = 2GM T

On en déduit :

1/ 2

GMT rp

Dv p = v p − vp =

2ra rp + r a

1 1 − rp rp + r a

=

GM T rp

2ra r p + ra

1/ 2

− 1 = 2, 43 km . s−1

b) La vitesse va qu’a le satellite à l’apogée, avant de prendre l’orbite géostationnaire, est telle que : mv 2a GM T m GM T m − = Em,a = − 2 ra ra + rp On en déduit :

d’où v2a = 2GMT

GMT ra

Dva = va − v a =

1/ 2



1 1 − ra rp + ra

=

2rp r p + ra

GM T ra

1/ 2

2ra rp + ra

+ 1 = 1, 46 km . s−1

S12– 9. Satellite COBE (COsmic Background Explorer) La période s’obtient aisément à l’aide de la troisième loi de Kepler relative aux satellites : T 2 a3 =

4p2 GM T

4p2 a3 GM T

donne T =

1/ 2

= 103 ×

4 × p2 × 0, 1 × 7, 33 6 × 6, 67

soit 1 h 43

= 6 195 s

S12– 10. Masse volumique de Jupiter On a, d’après la troisième loi de Kepler : 4p2 T2 = a3 GM

d’où

MJ = MT

aG aL

3

TL TG

2

et

rJ = rT

aG/JG aL /TL

3

TL TG

2

En tenant compte des valeurs numériques, on obtient : rJ = rT

15 60

3

3, 934 1, 03

2

= 0, 228

d’où r J = 5 520 × 0, 228 = 1 260 kg . m−3

S12– 11. Durée de passage d’une comète dans l’environnement terrestre 1. On a, pour la Terre et pour la comète, respectivement, puisque les excentricités sont e ≈ 0 et e = 1 : r = pT =

L2T GMT2 MS

avec LT = MT r2w˙ et Lc = Mc r2 w. ˙

et r =

pc = 1 + cos w

L 2c GM c2 M S

1 1 + cos w

697

et problèmes 2. Les points d’intersection des deux trajectoires s’obtiennent en égalant les équations polaires : pc 1 + cos w0

r = pT =

d’où

cos w0 =

pc −1 pT

3. La durée du passage s’obtient en intégrant l’équation de conservation du moment cinétique de la comète : T=

Mc p2c Lc

w0 −w0

1 Mc p2c d w = (1 + cos w)2 Lc

w0

1 + tan

2

0

dw 2 cos2 (w/2)

w 2

puisque (1 + cos w)2 = 4 cos4 (w/2). Or : d tan Il en résulte que : T=

Mc p2c Lc

w0

w 2

1 + tan

2

0

=

w 2

w 1 1 + tan2 2 2

d tan

w 2

=

dw =

dw 2 cos2 (w/2)

w0 Mc p2c tan 2 Lc

1+

1 2 w0 tan 3 2

S12– 12. Mise sur orbite elliptique d’un satellite 1. On sait que l’équation de la trajectoire est (cf. chapitre 12) : r=

p avec 1 + e cos(w − w0 )

p=

L2 Gm2 MT

e= 1+

2pEm GmMT

1/ 2

et w0 = 0

puisque, pour w = 0, r = p/(1 + e cos w0 ) = rmin = p/(1 + e). 2. Comme : L = Cte = mr 0 v0 cos a 0 avec il vient : p=

v0 =

L2 = r 0 cos2 a 0 Gm2M T

et r min =

GM T r0

e= 1+

p r 0 cos 2 a0 = 1+e 1 + | sin a0 |

1/ 2

et E m = 2pEm GmMT rmax =

mv 20 GmMT GmM T − =− r 2 2r0 0

1/ 2

= 1−

p r0

1/ 2

= | sin a0 |

p r0 cos2 a0 = 1 −e 1 − | sin a0 |

 c Dunod – Toute reproduction non autorisée est un délit

On en déduit : a = (rmin + r max)/2 = r0 ; on sait en effet que a est défini par la seule énergie qui ne dépend que de r 0. S12– 13. Comète Hale-Bopp 1. Le Soleil exerce sur la comète la force centrale (−GMS m HB/r 2) er = (−GMS mHB/r 3) SH. Le moment cinétique en S est donc une constante vectorielle, ce qui implique un mouvement plan. 2. On obtient l’énergie potentielle de la comète en exprimant le travail élémentaire de la force de gravitation : dW = −

GM Sm HB GMS mHB GM S m HB GMS m HB rdr = − dr = d SH · d SH = − 3 3 2 r r r r

= − d Ep

avec, puisque Ep = 0 pour r infini : Ep = −

GM S mHB GM Sm HB + Cte = − r r

3. a) La trajectoire est une ellipse dont l’un des foyers est occupé par S ; le périhélie P correspond à w = 0, l’aphélie A à w = p.

698

Solutions des exercices

b) On a : p rmin = 1 +e

rmax =

p 1 −e

d’où 2a = rmin + rmax =

2p p p + = 1+e 1−e 1 − e2

2

et p = a(1 − e )

4. a) L’application de la troisième loi de Kepler entre la comète et la Terre donne : ac = aT

Tc TT

2/ 3

b) On en déduit :

= 1, 49 × 1011 × 24002/3 = 2, 67 × 1013 m

p = a(1 − e 2) = 2, 66 × 10 11 m rmax =

p = 5, 30 × 10 13 m 1−e

rmin =

p = 1, 33 × 1011 m 1+e

et TC = TS − rmin = 0, 16 × 1011 m

5. a) La composante radiale de la vitesse de H au périhélie et à l’aphélie est nulle puisque en ces points r est minimal ou maximal. La vitesse minimale de la comète s’obtient à l’aide de la loi des aires au périhélie et à l’aphélie. On a : 1, 33 rmin = 200 000 × = 501, 9 km . h va rmax = vprmin d’où va = v p 530 rmax 6. L’énergie mécanique de la comète peut être calculée en n’importe quel point de la trajectoire par exemple à l’aphélie : 2 m HBvmin GMS mHB = 1, 939 × 1016 − 5 × 1018 ≈ −5 × 1018 J Em = − 2 rmax

Chapitre 13 S13– 1. Système de trois points matériels 1. La quantité de mouvement du système s’obtient en effectuant la somme des quantités de mouvement des particules : P = 0, 2 × (2 e x + 2t e z) + 0, 3 × (e x + 3 ey ) + 0, 5 × (2t ex − e y + 2 ez ) soit P = (0, 7 + t )ex + 1, 4 ey + (1 + 0, 4t)ez . Comme la masse M vaut 1 kg, la vitesse du centre de masse est : P = (0, 7 + t)ex + 1, 4 e y + (1 + 0, 4t)ez M Le système n’est donc pas isolé. Pour obtenir la somme des forces extérieures, il suffit de dériver P par rapport au temps. On trouve : Fex = ex + 0, 4 ez . vC =

2. Le moment cinétique du système en O s’obtient en sommant les moments cinétiques de chaque particule : ce qui donne :

LO = OA1 × m 1v1 + OA2 × m2 v 2 + OA3 × m 3v3

LO = 0, 2 × −6t ex − 2t 2 ey + 6 ez + 0, 3 × (12 ex − 4 e y + 3 ez ) + 0, 5 × −ex + 2t(t − 1) ey + t 2 ez En effectuant, on trouve LO = (−1, 2t + 3, 1)e x + (0, 6t2 − t − 1, 2)ey + (2, 1 + 0, 5t2 )ez . Le moment des forces extérieures s’en déduit par dérivation : MO,ex = −1, 2 e x + (1, 2t − 1) ey + t e z. 3. L’énergie cinétique du système s’obtient de la même façon par sommation :

Ek =

1 1 1 1 0, 2 × (4 + 4t2) + 0, 3 × (1 + 9) + 0, 5 × (4t 2 + 1 + 4) = 1, 4t2 + 3, 15 m 1 v21 + m 2v22 + m 3v 23 = 2 2 2 2

On en déduit la puissance de toutes les forces, extérieures et intérieures, qui s’exercent sur le système en dérivant Ek et le travail en calculant sa variation entre les dates 0 et 1 s : P = 2, 8 t et W = DEk = 1, 4 J.

699

et problèmes S13– 2. Conservation de l’énergie et invariance temporelle

1. D’après la définition de l’énergie potentielle, on a : dW1 = F 1 ·d r1 = − d Ep et dW 2 = F2 ·d r 2 = − d E p, d’où : ∂Ep ∂Ep F1 = − ex + · · · = − grad1 E p et F 2 = − e x + · · · = − grad2 E p ∂x 1 ∂x 2 2. Appliquons le théorème de l’énergie cinétique et le théorème de la quantité de mouvement au système. On a, respectivement : d Ek = d Ek,1 + d E k,2 = dW1 + dW2 = − grad 1 E p · d r1 − grad 2 Ep · d r 2

et

d P = d P 1 + d P2 = (F 1 + F 2) d t

3. A priori : ∂Ep ∂Ep ∂Ep ∂E p + +··· + +··· = + grad 1 Ep · d r 1 + grad2 E p · d r2 ∂t ∂x 1 ∂x2 ∂t

dEp =

Comme ∂Ep/∂t = 0, on en déduit : d Ep = grad1 E p · d r 1 + grad2 E p · d r 2 = − d Ek

d’où

d(Ek + Ep ) = 0 c’est-à-dire Em = Ek + Ep = Cte

S13– 3. Atome de positronium 1. Le référentiel du centre de masse R∗ associé au système est en translation par rapport à R et tel que P = Mv∗C = 0 ; le centre de masse est donc fixe et peut être pris comme origine de R∗ . Ce dernier est galiléen car le système est isolé. ∗

2. a) En faisant travailler les deux forces intérieures qui s’exercent sur A1 et A2 , on trouve l’énergie potentielle suivante, si l’origine de l’énergie potentielle étant prise à l’infini (cf. chapitre 13) : Ep = K /r avec K = −e2 /(4pε 0). Le signe négatif de K traduit le caractère attractif de l’interaction. b) Pour r = 106 pm, l’énergie potentielle vaut : E p = −9 × 109 ×

1, 6 × 10−19 e2 = −9 × 10 9 = −13, 6 eV 106 × 10−12 r

c) L’énergie potentielle d’interaction gravitationnelle des deux particules vaut :

 c Dunod – Toute reproduction non autorisée est un délit

E p = −G‘,

m 2e (0, 91 × 10−30 ) 2 = −6, 67 × 10−11 × = −3, 26 × 10−40 eV 106 × 10−12 × 1, 6 × 10 −19 r

Cette énergie est donc négligeable devant la précédente.

3. a) La masse réduite vaut me /2. Comme la force à laquelle est soumise A est centrale, son moment est nul et le moment cinétique constant ; on en déduit que le mouvement de A est plan. b) On déduit les mouvements de A 1 et A 2 dans R∗ de celui de A par des homothéties : CA1 =

m2 CA CA = m 1 + m2 2

et CA2 = −

m1 CA CA = − m1 + m 2 2

4. a) En appliquant la loi fondamentale de la dynamique à A dans R∗ , on obtient : 1 mvA2 |K | |K | = 2 d’où Ek∗ = mv2A = 2 2r r r

avec

|K | =

e2 4pε0

b) On en déduit l’énergie totale du positronium dans R∗ : Em∗ = Ek∗ + E p =

|K | |K | |K | − =− 2r 2r r

d’où Em∗ = −Ek∗ =

Ep = −6, 8 eV 2

700

Solutions des exercices

c) D’après le théorème de l’énergie, le rayonnement provoque une diminution de E ∗m . Compte tenu de la relation entre les différentes énergies, Ep diminue d’une quantité double et E∗k (r) augmente d’une même quantité. Cette augmentation surprend car dans la plupart des problèmes, où l’énergie potentielle ne joue pas de rôle majeur, la perte d’énergie entraîne celle de l’énergie cinétique. C’est le cas lorsqu’on freine un véhicule. On voit que le rayonnement implique une diminution continuelle de l’énergie, et donc la chute de l’électron sur le positron. Le système est donc instable, d’où les hypothèses de Bohr signes précurseurs de la théorie de la quantique indispensable à l’échelle microscopique. S13– 4. Vibrations de la molécule HCl 1. On sait que, dans le référentiel du centre de masse R ∗, l’énergie et le moment cinétique sont des constantes : 1 2 m˙r + Ep,ef = E E = Cte et L = Cte d’où 2 2. Un développement de Taylor de l’énergie potentielle effective autour de r = r0 donne : E p,ef ≈ E0 +

1 K (r − r0 )2 2

d’où F = − grad Ep,ef = −K (r − r0 ) er

On en déduit, en dérivant l’équation de conservation de l’énergie Em , l’équation d’un mouvement oscillatoire : m¨r + K (r − r 0) = 0

soit

¨ R + v20 R = 0

avec

R = r − r0

2

et v 0 =

K m

3. On trouve K = mv20 = 4p 2f02m1 m 2/(m 1 + m2) ≈ 471 N . m −1. S13– 5. Interaction nucléaire entre neutron et proton 1. La force étant attractive la constante d’interaction doit être négative. Comme la force correspondante dérive d’une énergie potentielle qui ne dépend que de r, elle est centrale et a pour expression : K d Ep F=− er = 2 dr r

1+

r r exp − a a

er

Notons que l’on retrouve l’interaction de type newtonien en faisant a infini ; aussi dit-on que l’interaction gravitationnelle a une portée infinie. 2. On sait que (cf. chapitre 13) : L∗2 2mr2 Les valeurs de r pour lesquelles E p,ef = 0 satisfont à l’équation : Ep,ef = Ep +

K r L∗2 exp − =0 + 2mr2 r a La valeur rm pour laquelle d Ep,ef / d r = 0 est déterminée par : d E p,ef K =− 2 dr r

1+

r r L2 exp − − 3 =0 a a mr

d’où

K 1+

rm rm exp − a a

+

L2 =0 mrm

3. Le graphe de E p,ef = Ep + L2 /(2mr2 ) présente un minimum pour r = r 0 et un maximum pour r = r 1. D’où, les différents états suivants : i) Si E > E1 avec E 1 = E p,ef (r1 ) , l’état est libre.

ii) Si 0 < E < E1 , l’état est lié ou libre. iii) Si E 0 < E < 0 avec E0 = Ep,ef (r0 ), l’état est lié.

701

et problèmes 4. Lorsque r = r m, l’énergie E ∗ a pour expression : E∗ =

K rm exp − rm a

+

L2 K rm K rm rm − = exp − 1+ exp − rm a a a 2mr2m 2rm

soit E∗ = −32, 75 MeV. Comme ˙r = 0 en ce point, Ek∗ se réduit à : E ∗k =

m 2 2 L∗2 K rm 1+ rm w˙ = 2 = − 2 2mrm 2rm a

exp −

rm a

=

K rm rm 1− exp − a a 2rm

= 49, 1 MeV

Il en résulte que Ep = E ∗ − Ek∗ = −81, 85 MeV. Quant au moment cinétique dans R∗ , il vaut, puisque m ≈ mn /2 : ∗2 2 ∗ −34 J.s L = m nrm Ek = 1, 14 × 10 S13– 6. Interaction harmonique entre deux corps 1. La force d’interaction est une force de rappel : F = (−∂ Ep /∂r) er = −Kr e r.

2. Pour L ∗ = 0, le graphe E p,ef(r) = Ep (r) est une parabole. Pour une énergie donnée, le mouvement en r est oscillatoire entre r = 0 et r = rmax . Pour L ∗ =  0, le graphe Ep,ef (r) = E p(r)+L ∗2 /(2mr 2) est un puits de potentiel qui passe par un minimum E 0 lorsque r = r0 . Le mouvement est oscillatoire entre deux valeurs de part et d’autre de r0. 3. La trajectoire est circulaire si r = Cte = r0 et donc si l’énergie est égale au minimum E0 de l’énergie potentielle. Comme mv2 /r0 = Kr 0 d’après la loi fondamentale, il vient : E = E k + Ep =

Kr02 Kr 2 + 0 = Kr 20 2 2

1/ 2 2 et L∗ = mvr0 = (Km) r0

S13– 7. Effondrement gravitationnel du Soleil 1. L’application du théorème de l’énergie au Soleil S, dans le référentiel du centre de masse, donne, si l’on désigne par P r la puissance rayonnée et par ST la distance entre le Soleil et la Terre : dEm dEp 4pST 2 16pR2 3 24 = = Pr = −1 × 10 3 × = −10 × = −265 × 10 W puisque DSu 2 DS dt dt

u=

2R . ST

2. Comme Ep = −(3/5)GM2 /R ≈ −2, 29 × 1041 J (cf. chapitre 6), la durée de vie du Soleil serait :

 c Dunod – Toute reproduction non autorisée est un délit

tS =

Ep = 0, 864 × 1015 s d E p/ d t

soit

tS = 2, 36 × 107 années = 23, 6 Man

ce qui est beaucoup plus faible que l’âge admis pour le Soleil qui est de 5 × 109 années (5 Gan). On en conclut que l’effondrement gravitationnel ne permet pas d’expliquer l’âge du Soleil. Le physicien américain H. Bethe, d’origine allemande, a montré que ce sont les réactions de fusion de noyaux au sein du Soleil qui permettent d’estimer correctement l’âge du Soleil. S13– 8. Effet de marée dans un satellite 1. Dans les triangles TCA1 et TCA2 , on a les relations : r 21 = r20 + Par conséquent :

l2 − r0 l cos u 4

1 1 = (r02 + l 2/4 − r 0 l cos u)−1/2 = r1 r0

1+

l cos u l2 − 2 r0 4r0

et de même pour 1/r2 : 1 1 ≈ r2 r0

et r22 = r20 +

1−

−1/2

l2 + r 0 l cos u 4



1 r0

l2 l cos u 3l2 cos 2 u − + 2 r0 8r0 8r20

1−

l cos u 3l 2 cos2 u l2 + + 2 r0 8r0 8r20

702

Solutions des exercices

On en déduit Ep :

1 1 + r1 r2

E p = E p(r 1 ) + Ep (r2 ) = −GMT m

=−

GM T m l2 2 − 2(1 − 3 cos 2 u) r0 4r0

2. On obtient les positions d’équilibre en dérivant Ep par rapport à u : d Ep 2 GMT m = 3l sin(2u) du 4r30

Ainsi, les positions d’équilibre sont u0 = 0, u 2 = p/2 et u 3 = p. Pour étudier leur stabilité, calculons la dérivée seconde : d2 Ep 2 GM T m = 3l cos(2u) d u2 2r30

d2 Ep d u2

ce qui donne

d2 Ep d u2

>0 0

0 p

Les positions 0 et p sont donc stables alors que la position p/2 est instable : le satellite s’oriente selon la direction de la Terre. 3. Appliquons la conservation de l’énergie mécanique : Em = Ek + Ep = Cte. Il vient : 2×

m(l2/4) ˙u2 GMT m l2 2 − 2 (1 − 3 cos2 u) = Cte d’où − 2 r0 4r 0

ml2 u˙ 2 GM T m − 3l2 cos2 u = Cte 4 4r30

Il vient, en dérivant et en simplifiant par u˙ : 3GMT ¨ u+ cos u sin u = 0 r30 Au voisinage de u = 0, cette équation se réduit à : ¨ u + v20 u = 0

v 20 =

avec

3GM T r30

et T 0 =

Au voisinage de u = p, on a, en posant ε = u − p :

3GMT cos ε sin ε = 0 soit, puisque ε ≈ 0 r 30

ε¨ +

r30 3GM T

2p = 2p v0

ε¨ +

1/ 2

3GM T ε≈0 r03

On obtient donc la même pulsation T0 , ce qui était prévisible. S13– 9. Masse volumique de l’Univers 1. Par analogie, on remplace 1/(4pε 0) par −G et les charges par des masses. On trouve (cf. chapitre 6) :

3 GM 2 5 R Le signe négatif exprime le caractère attractif de l’interaction. Ep = −

2. L’énergie cinétique de la distribution s’écrit : Ek =

1 2

Effectuons cette dernière sommation : 2

i

R

2

rr d V = r

mi ri = V

2

mi v 2i = i

H2 2 R

2

r 4pr d r = 4pr 0

i

4

r d r = 4pr 0

puisque M = 4prR3/3. On en déduit : Ek = aMH2 R2 3. Comme E = 0, il vient :

3 3G M 2 MH 2R2 − =0 10 5 R

mi r2i

d’où

avec

r5 5

R

= 4pr 0

3 R5 2 = MR 5 5

a = 3/10.

4pR 3r H 2R 3 = GM = G 2 3

et r =

3H 2 8pG

703

et problèmes On trouve : r = 9, 2 × 10−27 kg . m −3 , soit, comme r = nvm p : nv =

r 9, 2 × 10−27 = 5 m−3 = 1, 67 × 10 −27 mp

S13– 10. Galaxie spirale 1. La loi fondamentale de la dynamique appliquée au point A s’écrit : m

v2 GMn m = r r2

d’où M n =

v2 r G

2. On en déduit : v 2r Mn 10 = 9, 5 × 10 = MS GMS

puisque 1 pc =

1, 49 × 1011 16 ≈ 3 × 10 m 5 × 10−6

3. Comme le modèle simplifié prévoit une variation de v en fonction de r selon r−1/2 , ce qui est nettement en désaccord avec les faits, il est nécessaire de revenir sur la distribution de masse qui, probablement, n’est pas concentrée au centre de la galaxie. S13– 11. Ordre de grandeur de la température du Soleil 1. D’après la définition du viriel, on a : 2

=−

i

F i · ri = −

ij

Fj→i · r i + F i→j · r j = −

ij

Fj→i · rij

puisque rij = ri − r j et Fi→j + F j→i = 0. En remplaçant la force Fj→i par Krij/r 3ij , il vient : 2

=−

ij

Krij · r ij = − r3ij

ij

K = −Ep rij

1 d’où Ek = − E p 2

2. L’énergie cinétique moyenne interne du Soleil et l’énergie potentielle moyenne interne ont pour expressions respectives (cf. Thermodynamique) : Ek = N

3k BT M 3k BT = m 2 2 p

et

Ep = −

3 GM 2 5 R

 c Dunod – Toute reproduction non autorisée est un délit

En introduisant RS = (2GM/c 2) ≈ 3 km, on trouve : k BT =

RS mp c2 10R

soit k BT =

3 × 103 × 938 × 106 = 402 eV 10 × 0, 7 × 109

On en déduit, en divisant par kB = 1, 38 × 10−23 SI : T ≈ 4, 7 MK, à comparer avec la température de la surface du Soleil qui est 5 700 K. S13– 12. Viriel associé à l’énergie potentielle Ep = Krn que :

Comme E p = K r n, le terme F · r vaut −nKr n, d’où : E k = −F · r/2 = nKrn /2 = nEp /2. Il en résulte

2 n Ek = 1 + Ep 2 n Pour n = 2 (oscillations harmoniques), on trouve que E = 2Ek = 2Ep. E = Ek + Ep = 1 +

Pour n = −1 (problème de Kepler), on trouve bien : E = −Ek = Ep /2. Pour n = −6 (forces de van der Waals), E = 2Ek /3 = −2E p .

704

Solutions des exercices

S13– 13. Système de deux corpuscules liés par un fil inextensible 1. Les forces extérieures au système sont le poids qui s’exerce sur chaque corpuscule, la réaction du plan sur A 1 et la réaction des bords du trou sur le fil. Le moment en O de cette dernière réaction qui passe par O est donc nul. Quant aux moments des autres forces, ils sont horizontaux. Il en résulte, d’après le théorème du moment cinétique, en projection suivant l’axe vertical : d LO · ez = MO · ez = 0 dt

d’où

L O,z = LO · e z = 0

Or, le moment cinétique de A2 est nul et celui de A1 vertical : OA 2 × m2 v 2 = 0 et OA1 × m1v 1 = m1 r2 w˙ ez. Il en résulte que m1 r2w˙ = L = Cte. 2. Calculons le travail élémentaire des forces intérieures en un point du fil. Pour cela, considérons deux points voisins du fil A1 et A 2 : dW = T2→1 · d OA 1 + T1→2 · d OA2 = T2→1 d A 2 A1 = T2→1 d(A2 A 1 )

puisque la tension du fil est colinéaire à A2A 1. Le fil étant inextensible, l’élément A 2 A1 a une longueur constante. Le travail de ces forces intérieures est donc nul. Il en est de même pour tous les éléments du fil. Comme l’autre force non conservative R a, elle aussi, un travail nul, en l’absence de frottement, l’énergie mécanique du système se conserve : 1 1 Ek + Ep,ex = E m = Cte avec E k = m1 (r˙2 + r 2w˙ 2 ) + m2z˙22 et Ep = m2gz 2 2 2 Or, la coordonnée z2 du corpuscule A2 , selon la verticale ascendante, vaut z2 = −(l − r ). Par conséquent : 1 1 m 1( ˙r2 + r2w˙ 2) + m 2r˙ 2 + m 2g(r − l) = E m et finalement 2 2

m1 + m 2 2 L2 r˙ + + m2g(r − l) = E m 2 2m 1 r2

3. Pour que A2 ne traverse pas le trou, il faut que r˙ = 0 pour r < l. Comme : m1 + m 2 2 L2 m 1 lv0 m 1v02 m 2gl L = m et E − = r˙ + + m 2g(r − l) = E m avec 1 r0v 0 = m 2 2 2m1 r 2 2 2 il vient : m1 + m2 2 L2 m1v 20 m 1 l2 v20 ˙r = Em − − m − m 2g(r − l) = 2 g(r − l/2) − 2 2m 1 r2 2 8r2 2 En imposant l’inégalité ˙r  0, pour r = l, on satisfait à la condition cherchée : m1 v20 l m 1 v02 0 − m2g − 2 2 8

soit

v0 

4m 2gl 3m1

1/ 2

= 5, 1 m.s−1

S13– 14. Système de deux masselottes en interaction élastique et en chute libre 1. a) Le théorème du centre de masse au système donne, par rapport au référentiel R du laboratoire : 2maC = mg + mg

d’où aC = g

Le mouvement de C est donc uniformément accéléré, d’accélération g . La relation demandée est : m 1 a 1 + m 2a 2 a = a1 + a2 = 2g car aC = 2 m 1 + m2 b) L’énergie mécanique du système se conserve car les seules forces qui travaillent (le poids) sont conservatives. On en déduit Em = Ek + E p = Cte , avec : m 2 (x˙ 1 + x˙ 22) et 2 d’où l’équation différentielle suivante : Ek =

Ep = −mg . OA 1 − mg . OA2 +

K (x2 − x 1 − l0 )2 2

m 2 K 2 2 (x˙ 1 + x˙2 ) − mgx1 − mgx 2 + (x2 − x 1 − l 0) = Cte 2 2

705

et problèmes 2. a) Par rapport à R , la loi fondamentale de la dynamique, appliquée à chaque masselotte donne : m1

d2 r 1 = F 2→1 + m1 g et d t2

m2

d2 r2 = −F2→1 + m2 g dt2

d’où, en divisant la première équation par m1 , la seocnde par m2 et en effectuant la différence : d2 r1 F2→1 = m d t2

avec

r = r1 − r 2

et m =

m1 m 2 m = 2 m 1 + m2

b) Dans le référentiel du centre de masse, les mouvements de A1 et A2 se déduisent de celui d’une particule fictive A , de masse m = m/2 , soumise de la part de C à la force exercée par le ressort (cf. Leçon 25) : K mx˙2 + (x − l0 )2 = Cte soit en dérivant par rapport au temps X¨ + v20 X = 0 2 2 en posant : X = x − l 0 = x2 − x 1 − l 0

et v0 =

K m

1/ 2

=

2K m

1/ 2

=

2 × 400 0, 2

1/ 2

≈ 63, 2 rad.s−1

La fréquence propre d’oscillation est donc f0 ≈ 10 Hz . On en déduit l’expression suivante de X : X (t ) = Xm cos(2pf0 t + f) Xm et f étant déterminés par les conditions initiales : X0 = Xm cos f

et X˙ = −v 0 sin f = 0

d’où

f=0

et X0 = Xm = 0, 01 m

Ainsi, X (t ) = Xm cos(v0 t) . b) Comme : CA2 CA 1 CA 2 − CA 1 =− = m1 m2 m1 + m2

donne x 2 − xC =

m1 x x= 2 m1 + m 2

et x1 − x C = −

m2 x x=− 2 m 1 + m2

pour m1 = m2 , on trouve le mouvement de A1 et A 2 , dans R , selon : x1 = x C −

1 1 9, 8t 2 x = gt2 − Xm cos(v 0t) + l0 = − 0, 005 cos(63, 2 t) + 0, 1 2 2 2 2

x2 = x C +

x 1 1 9, 8t 2 + 0, 005 cos(63, 2 t) + 0, 1 = gt2 + Xm cos(v 0t) + l0 = 2 2 2 2

 c Dunod – Toute reproduction non autorisée est un délit

et :

3. Pour obtenir les accélérations des masselottes, appliquons la loi fondamentale à chacune d’elle, à l’équilibre, en tenant compte de la tension du fil : ma1 = 0 = T + mg + K (x2,e − x 1,e − l 0 ) ex

et ma 2 = 0 = mg − K (x2,e − x 1,e − l0 ) ex

ce qui donne en projection : 0 = T x + mg + K (x 2,e − x 1,e − l0 ) et

0 = mg − K (x2,e − x1,e − l 0 )

Ainsi, à l’instant initial, on a : mg = K (x2,e − x1,e − l 0 ) d’où

ma1,x = Tx + 2mg et

ma2,x = mg − K (x2,e − x 1,e − l0 ) = 0

Si on annule Tx en coupant le fil, on trouve respectivement a 1,x = 2g et a2,x = 0 . Notons que les accélérations ne sont pas continues lorsqu’on coupe le fil, car une force a été brutalement annulée.

706

Solutions des exercices

Chapitre 14 S14– 1. Spectrométrie neutronique 1. La collision neutron-noyau d’hélium étant élastique, on a les relations de conservation suivantes : 2

p 2 2m2

Q = Ek2 =

  p1 = p 1 + p 2 et

p21 p = 1 + Q avec 2m 1 2m1

Q = E2k =

p2 2m 2

Par conséquent : Q=

p21 m2 p1 m2 (p − p2 )2 cos u2 (2m2 Q) 1/2 d’où Q 1 + − 1 =− Q+ 2m 1 2m1 m1 m1 m1

On en déduit : Q

1/ 2

1+

m2 m1

=

p1 1/ 2 (2m2 ) cos u2 m1

soit Q =

4m 1 m2

=

p1 cos u 2 (2m 2Q) 1/2 m1

2

2

(m1 + m2)

cos u2 Ek1

2. a) D’après l’expression de Q1/2 , on a : cos u2  0 d’où 0  u2  p/2. b) Pour u 2 = 0 : Q = Qmax =

4m1 m2 Ek,1 (m 1 + m2)2

et Ek,1 = E k,1 − Qmax = Ek,1

m 1 − m2 m 1 + m2

2

On en déduit : p1 = (2m 1 Ek,1 )1/2 = p 1 



m 1 − m2 m 1 + m2

puisque m2 > m1



et p2 = (2m 2Q max)

1/ 2

= p1

2m2 m1 + m 2

Comme p2 − p1 = p1 , u 1 = p et u 2 = 0.

c) Q min = 0 pour u2 = p/2 ; Ek,1 = E k,1 , d’où p 1 = p1 . On en déduit u1 = 0.  3. Si m1 = m2 : Q = E k,1 cos2 u2 et E k,1 = E k,1 − Q = Ek,1 sin2 u2.

Les lois de conservation donnent : p 1 = p 1 + p2 et p 21 = p 21 + p22 . Il en résulte que p1 et p 2 sont perpendiculaires. S14– 2. Ralentissement de neutrons 1. On sait que (cf. chapitre 14) : Q max =

4mn ma Ek (m n + m a)2

d’où qmax =

4h Q max = Ek (1 + h)2

Ek étant l’énergie cinétique du neutron incident. La courbe q max(h) passe par 0, par un maximum pour h = 1 et tend vers 0 pour h infini (Fig. S14.1) : d qmax / d h = (1 − h2 )/(1 + h) 2 2. Le nombre moyen n de collisions est tel que : n

0, 82 =

Ek,f E k,i

avec

Ek,f =

3kBT = 38, 8 meV 2

d’où n =

E k,f /Ek,i ≈ 95 ln(0, 82)

707

et problèmes

Qmax 1

p1

C

p2



θ1

A 0

C 1 p2 θ* B

O

θ 2

θ 2

η

1 F IG . S14.1.

F IG . S14.2.

S14– 3. Angle maximal de diffusion dans une collision élastique En utilisant le cercle de rayon mv du diagramme de collision élastique (cf. chapitre 14), on voit que, pour m1 > m2 , le point figuratif A est en dehors du cercle (Fig. S14.2) : à une valeur u1 de l’angle de diffusion de la particule incidente , correspondent deux points sur le cercle, C et C, et donc deux valeurs de u2 et deux valeurs de l’énergie cinétique Ek,2 . S14– 4. Énergie transférée au cours d’une collision inélastique 1. Les lois de conservation de la quantité de mouvement et de l’énergie cinétique donnent : p1 = p 1 + P

p 2 p21 P2 + U2 = 1 + + U2 2m 1 2m 1 2m 2

et

avec

U2 = U2 + DE n

2. Exprimons P 2 : 2

2

2

2

  P = p 1 + p 1 − 2p 1p1 cos u ≈ 2p1 1 − m1

DE n 2m 1DEn 2 − 1− p1 p21

1/ 2

2 2

2

cos u ≈ p1 u + m1

DEn2 p21

ce qui donne, si u est petit P2 ≈ p 21u 2 + m1DE 2n/p21 . 3. D’après ce qui précède :

 c Dunod – Toute reproduction non autorisée est un délit

Q min =

P2min DEn2 = 2me 2me v 21

pour u = 0

et

Qmax ≈

2p 21 me

pour u = p

S14– 5. Énergie seuil d’une collision inélastique ∗ ∗ 1. Dans R∗, E k∗ = E  k + DEn . Comme, dans R∗, E k peut être nul, Ek∗  DE n . Par conséquent :

1 2

m 1m 2 m 1 + m2

v21 =

m2 Ek,1  DE n m 1 + m2

soit E k,1 

m1 + m2 DEn m2

On trouve : Esk,1 ≈ (13/12) × 300 = 325 eV. 2. D’après les lois de conservation : p1 = p1 + p 2 2

  + Ek,2 + U1 + U2 et Ek,1 + U1 + U 2 = Ek,1

avec

U 1 = U1

2

Comme p21 = p 1 + p 2 + 2p 1p 2 cos u, il vient, puisque E k,1 = p21/(2m 1 ) : DEn =

m2 m2    − 1 Ek,2 + 2 cos u E k,1E k,2 m1 m1

1/ 2

et U2 = U2 + DEn

708

Solutions des exercices

Chapitre 15 S15– 1. Diffusion d’un proton par un noyau lourd 1. La quantité de mouvement du système n’est pas conservée puisque le noyau est immobile et que la quantité de mouvement de A1 change en norme et en direction : p2 = 0 et p 1 = Cte Ce résultat n’est pas surprenant puisqu’une force extérieure maintient A2 immobile. En réalité, c’est un problème mal posé : l’hypothèse « A2 immobile » est artificielle : A2 a un petit mouvement autour du centre de masse de l’ensemble, tel que p2 = −p1 à tout instant dans le référentiel du centre de masse R∗ (cf. chapitre 13). 2. La conservation du moment cinétique et celle de l’énergie donnent respectivement : m 1v 0 b = m 1v s s et

m 1 v02 m 1 vs2 K = + 2 2 s

avec

K=

Ze2 4pε 0

3. On déduit des relations précédentes : Em

1−

b2 s2

=

K s

2 soit s −

K s − b2 = 0 Em

La résolution de cette équation est immédiate, en excluant la solution négative sans aucun sens physique : s=

r0 + 2

b2 +

r20 4

1/ 2

= 1, 74 pm

puisque r 0 =

K = 1, 166 pm Em

S15– 2. Diffusion d’un noyau d’hélium par un noyau de cuivre Les charges des noyaux d’hélium et de cuivre valent respectivement : q1 = 2e et q 2 = 29e. Par conséquent : K=

q 1q 2 e2 = 58 = 1, 34 × 10 −26 J . m 4pε 0 4pε 0

Comme tan(x/2) = |K |/(mv20 b), avec m ≈ 4 × (64/68) mp , il vient : b=

mv20

1, 34 × 10−26 |K | = = 2, 1 pm tan(x/2) 4 × (64/68)mp c2 × (1/300)

S15– 3. Diffusion de particules a par une feuille d’argent Le paramètre d’impact est donné, en fonction de l’angle de diffusion, par l’expression suivante : b=

r0 2 tan(x/2)

avec

r0 =

|K | = 0, 135 fm Em

On trouve respectivement : b = 3, 87 fm, b = 0, 382 fm et b = 0, 737 fm. S15– 4. Sections efficaces associées à différentes excitations

Le nombre de centre diffuseurs est respectivement 3na et 2na , na = NA r/M étant le nombre d’atomes par unité de volume, r la masse volumique et M la masse molaire de l’aluminium. Les sections efficaces totales sont reliées aux libres parcours moyens par : 1 1 sp = et sK = 3na p 2n a K Le calcul donne : sp = 176, 3 × 103 barn et sK = 830, 5 barn (1 barn = 10 −28 m 2 ).

709

et problèmes S15– 5. Expressions de la section efficace différentielle de Rutherford

1. Puisque m e  ma , m ≈ me et x ≈ u1 . La section efficace différentielle de Rutherford s’écrit donc : ds dV

K 2me v20

=

2

1 = 4 sin (x/2)

2

K 4E 1

1 sin (u 1/2) 4

Le rapport recherché est donc : 2

dN / d t K = ds = 4E 1 Jn

2

K 4E 1

1 2p sin u 1 d u 1 = 4 sin u1 /2

1 d V1 sin (u 1/2) 4

2. Dans le cas où les deux particules en interaction sont des électrons : m = me /2 et x = 2u1. Donc : ds dV

d’où : ds =

K 2E1

2

=

K me v20

2

1 sin4(x/2)

1 K 4p sin(2u1 ) d u1 = E1 sin4 u1

2

cos u1 d V1 sin3 u 1

S15– 6. Détection de particules a diffusées par de l’aluminium 1. Le paramètre d’impact b s’obtient selon : r0 |K | x = avec r0 = = 3 pm Em 2 2b puisque Z1 = 2 et Z2 = 13. On en déduit, puisque x = p/2, b = r0 /2 = 1, 5 pm et la distance minimale : tan

s=

r0 + 2

b2 +

r20 4

1/ 2

= b(1 +

√ 2) = 3, 62 pm

2. Le flux des particules non diffusées, lorsqu’elles atteignent le détecteur, a pour expression : F nd =

d Nnd = nv,d Jn dt

ds dV

sd

V avec

ds dV

sd

=

Il vient donc :

m s2 A Jn 4 D2 Amn −1 Le calcul donne : Fnd = 1 090 atomes . s . F nd =

puisque nv,d =

s2 4

et V =

A D2

m Am n

 c Dunod – Toute reproduction non autorisée est un délit

S15– 7. Détection d’électrons diffusés par une feuille métallique Le nombre d’électrons d I n diffusés par les électrons atomiques et reçus par le détecteur, pendant l’unité de temps, est proportionnel au courant particulaire du faisceau Jn , à la section efficace élémentaire d s et au nombre de centres diffuseurs de la cible : ds K 2 1 d I n = Jn d s n d e S = Jn nd e S 2p sin x d x = J n nd e S 4p sin(2u1) d u1 dV 2E 1 sin4 u 1 1 2prw K 2 cos u 1 K 2 = Jn nd e S d V1 = J nn deS 3 E1 E1 sin u 1 sin3 u1 L2 puisque d V1 = 2p sin u1 d u1 avec sin u1 = r/L et cos u1 d u 1 = d r /L = w/L. S15– 8. Atténuation d’un faisceau d’électrons L’atténuation du faisceau d’électrons se manifeste par une variation du flux de particules, en fonction de l’épaisseur x de matériau traversée, qui est de la forme : F(x) = F(0) exp(−nvs tx) avec

st = pR 2

710

Solutions des exercices

dans le modèle de sphère dure. On a donc : 2 s t = pR =

1 F(0) ln nvx 1 F(x 1 )

d’où R = 1, 1 pm

On en déduit st = pR2 = 38 400 barn et  = (nvst )−1 = 3, 25 mm S15– 9. Diffusion et rayonnement d’une particule chargée 1. On sait que ce problème de Kepler est caractérisé par les trois lois de conservation suivantes : L = Cte

E = Cte

et R = v × L + K er = Cte

2. Comme a = K er /(mr2 ), il vient : K d LO da = OA × mt = OA × mt 3 dt dt mr

|K |t dr 3 dr − r =− 3 L dt r dt mr

Si la trajectoire est circulaire, L = mr2u˙ = mrv = (|K |mr) 1/2 car mv 2/r = K /r2 . On obtient donc : 1 2

|K |m r

1/ 2

dr − 2 |K |t |K |t 1/ 2 2 soit r d r = dt = − 3 (|K |mr ) dt mr m

et

3

r =

−6|K |tt 3 + r0 m

en intégrant et en tenant compte des conditions initiales.

Chapitre 16 S16– 1. Mise en rotation d’un plateau 1. Exprimons l’égalité des vitesses des deux points I1 et I 2 en contact, appartenant respectivement au plateau et au disque : v 1 = Vp × OI1 = RVp e v

d’où : Vp = V0 − Vr /R.

v2 = vC + V × CI 2 = V0 × CO1 + V × CI2 = (RV0 − rV) e v

2. La vitesse du point H s’obtient à partir du champ des vitesses du disque : v H = (RV0 + rV) e v = R(2V0 − V p ) ev Si le plateau est bloqué (Vp = 0), V0 = rV/R ; par conséquent : vH = 2RV0 e v . S16– 2. Roulements à billes Exprimons l’égalité des vitesses des points des solides en contact : i) Pour C 1 et C3, v 1 = v 3 donne V1 r1 ev = (vA − r 3 V3) e v

ii) Pour C 2 et C 3, v 2 = v3 donne V2r2 e v = (vA + r 3V3 ) ev.

On en déduit, en éliminant vA , V2 r2 − V1 r1 = 2r3V3 = (r 2 − r1 )V3 et : V3 =

V 2r 2 − V1 r 1 r 2 − r1

711

et problèmes S16– 3. Mise en rotation d’un disque à partir d’un axe incliné 1. En I, les vitesses des points I 1 et I 2 appartenant respectivement au disque et au cône sont égales : vI1 = Vd × OI = −V dd e x 2. Vd /V c = 0, 5

si

vI2 = Vc × OI = −V c d sin g e x d’où Vd = V c sin g

g = 30 ◦.

S16– 4. Cône roulant sans glisser sur un plan ˙ ez . 1. Comme u = p/2 − g = Cte, V = c˙ e z + f

2. En raison du roulement sans glissement v I2 = vI 1 = 0 avec vI2 = V × OI, I étant un point de la génératrice en contact. Ainsi V est porté par OI = OI eu, d’où : c˙ Vz = c˙ + f˙ sin g = 0 et V = V · e u = f˙ cos g = − tan g S16– 5. Entraînement d’un plateau circulaire ˙ ez et V T /R = u˙ e z. Par conséquent : 1. V P /R = c ˙ 1 et + u˙ e z et V 2/R = V 2/T + VT /R = f˙ 2 e t + u˙ e z V 1/R = V1/T + VT /R = f ˙ 1 et et V 2/R = f ˙ 2 et . Si on bloque la tige (u˙ = 0) , V 1/R = f ˙ = lu˙ − r ˙f1. 2. En I 1, point de contact de D 1 avec P, la condition de roulement sans glissement donne : lc ˙ ˙ ˙ De même en I2 , on a : −lc = −l u − rf 2 , ce que l’on obtient en changeant l en −l dans l’équation précédente. ˙ 2 = 0. En ajoutant ces deux équations, on trouve : f˙ 1 + f ˙ 1 = −rf ˙ 2 et : 3. Si on bloque le plateau (c˙ = 0), il vient lu˙ = r f ˙ 1 e t + r ez ˙ 2 et + ˙uez = −f˙ 1 et − r ez V 1/R = f˙ 1e t + ˙uez = f et V 2/R = f l l Ces vecteurs de rotation sont portés par les droites HI1 et HI2 . S16– 6. Roulement sans glissement d’un solide-satellite sur un solide-couronne Au point de contact, l’égalité des vitesses des points de la couronne et du solide-satellite se traduit par : 0 = vC + VS/R × CI avec

 c Dunod – Toute reproduction non autorisée est un délit

Il en résulte que :

vC = (R − r ) ˙u eu

˙ 0 = (R − r )u˙ + r (u˙ + f) d’où

˙ ez et VS/R = VS/OC + VOC/R = (u˙ + f) ˙ ˙u = − f 4

et u = −

f 4

S16– 7. Base et roulante dans le mouvement d’un disque sur un axe 1. Le C.I.R est le point I tel que 0 = v C + IC × V. En explicitant dans R = Oxyz, il vient : v0 0 0

+

xC − x r−y × 0

0 0 = −V

0 0 0

d’où x = xC

et y = r −

v0 V

La base est donc la droite y = r − v0 /V. Pour obtenir la roulante, explicitons la relation vectorielle dans R = Cx y  z : v0 cos f −v 0 sin f + 0

−x  −y 0

×

0 0 −V

=

0 0 0

d’où v0 cos f = −y V et

Comme x2 + y2 = (v0 /V)2, la roulante est le cercle de centre C et de rayon v0/V.

− v 0 sin f = xV

712

Solutions des exercices

2. S’il n’y a pas de glissement, v0 = rV. Donc y = 0 et x 2 + y2 = r2. La base est alors l’axe Ox et la roulante est le cercle périphérique du cylindre. S16– 8. Mise en rotation de deux disques coaxiaux Les conditions de roulement sans glissement de la roue sur les disques, obtenues en égalant les vitesses des ˙ 2 = bV + rV s. En ajoutant ces deux équations, points en contact, donnent respectivement : bf˙ 1 = bV− r V s et b f on voit que la somme des vitesses angulaires des disques est une constante égale à 2V. Ce problème est analogue à celui du différentiel. S16– 9. Bille roulant sans glisser sur un rail diédrique 1. Les conditions de roulement sans glissement de la bille sur les parois du rail s’écrivent : vC + V × CI = 0

et v C + V × CJ = 0

d’où V × (CI − CJ) = V × JI = 0

√ Il en résulte que V et JI sont parallèles. On en déduit que : vC = V × IC d’où vC = Vr 2/2.

Chapitre 17 S17– 1. Moments d’inertie principaux d’un cerceau homogène 1. La base orthonormée formée par le vecteur unitaire porté par l’axe de révolution Oz et par deux axes perpendiculaires est principale d’inertie en raison de la symétrie matérielle. Le moment d’inertie par rapport à l’axe de révolution est : IOz = m i (x 2i + y 2i ) = miR2 = MR 2 i

i 2

On en déduit : I Ox = IOy = IOxy + IOz/2 = MR /2 puisque IOxy =

i

m i z 2i = 0.

2. Le moment d’inertie du cerceau par rapport à tout diamètre est MR2/2. D’après le théorème d’Huygens, le moment d’inertie par rapport à l’axe D est donc : ID = MR 2/2 + MR2 = 3MR2/2. S17– 2. Moment d’inertie d’une tige homogène 1. La relation de définition du moment d’inertie par rapport à un axe donne en désignant par E l’extrémité considérée de la tige (Fig. S17.1) : mi Hi A 2i =

IED = i

m i EA2i sin2 u = IEz sin2 u = i

2

Ml2 sin2 u 3

puisque IEz = Ml /3 est le moment d’inertie de la tige par rapport à un axe perpendiculaire passant par E .

Φ θ Hi

Ai

O E F IG . S17.1.

713

et problèmes

2. La base orthonormée formée par l’axe Oz de la tige et par deux axes perpendiculaires est principale d’inertie en raison des symétries matérielles. Comme IOz = i m i (x2i + y 2i ) = 0, il vient : I Ox = I Oy = IOxy +

l/2

I Oz = IOxy 2

avec

z2 rl d z = rl

IOxy = −l/2

l3 Ml 2 = 12 12

Si l’origine de l’axe est prise à l’extrémité E de la tige, on trouve, en utilisant le théorème d’Huygens : IEx = IEy = Ml2/12 + M (l/2) 2 = Ml2 /3. Le vecteur unitaire eD défini par l’axe D et l’opérateur d’inertie [IE] s’explicitent respectivement selon : sin u 0 cos u

R

Ml 2 3

et

1 1 0

R

2

En effectuant IED = e D · [IE ] eD , on trouve bien IED = (Ml /3) sin2 u. S17– 3. Centre d’inertie et moments d’inertie principaux d’un demi-cercle homogène 1. Le centre d’inertie C se trouve sur l’axe de symétrie. En faisant tourner le demi-cercle autour de son diamètre, on obtient grâce au premier théorème de Guldin : 2p × OC × pR = 4pR2 d’où OC = 2R/p, O étant le centre du cercle générateur. 2. Le repère R est principal d’inertie ; on sait que IOz = MR2, IOx = I Oy = MR2 /2 comme pour un cerceau. L’opérateur d’inertie [I ]E s’explicite donc selon : 1 Ml2 1 3 0 R 3. Le moment d’inertie par rapport à l’axe Sz s’obtient aisément à l’aide du théorème d’Huygens selon : I Sz = IOz + MR 2 = 2MR 2 S17– 4. Moments d’inertie principaux d’un quart de disque homogène 1. À l’aide du théorème de Guldin (cf. chapitre 17), on trouve aisément que xC = y C = 4R/(3p).

 c Dunod – Toute reproduction non autorisée est un délit

2. Le moment d’inertie par rapport à l’axe de symétrie a même expression que le moment d’inertie d’un disque complet par rapport à l’un de ses diamètres. Par conséquent IOX = MR2 /4. 3. Comme pour un disque complet, les moments d’inertie principaux sont : IOz = MR 2 /2, Oz étant l’axe de révolution du disque générateur, et IOX = I OY = MR2 /4, OX étant l’axe de symétrie du quart de disque et OY l’axe perpendiculaire à OX. L’opérateur principal d’inertie [I ]E s’explicite donc selon : MR2 4

R

1 1 2

Cependant, le produit d’inertie Ixy n’est pas nul car R n’est pas repère principal d’inertie. Calculons Ixy en utilisant les coordonnées polaires x = r cos u et y = r sin u : R

I xy =

r sxy d S = r s

soit : Ixy

R4 = rs 4

0

p/2 0

sin2 u 2

R

r2 cos u sin u r d r d u = rs 0

p/2

= 0

rs R 4 MR2 = 8 2p

p/2

r3 d r

puisque M =

cos u sin u d u 0

rs pR2 4

714

Solutions des exercices

S17– 5. Centre d’inertie et moments principaux d’inertie d’un demi-disque homogène 1. Le centre de masse se trouve sur l’axe de symétrie Oy. D’après le deuxième théorème de Guldin, on a : 2p × y C ×

4pR3 pR2 = 2 3

4R 3p

d’où y C =

2. Les moments principaux d’inertie sont ceux du disque plein : IOz = MR2 /2 et I Ox = IOy = MR 2 /4, Oz étant l’axe de révolution du disque plein générateur et Oy étant l’axe de symétrie. On en déduit la matrice principale d’inertie qui est la même que pour un quart de disque. 3. Le moment d’inertie cherché est celui du disque générateur par rapport à tout diamètre ; c’est donc MR2 /4. S17– 6. Centre d’inertie et moments d’inertie d’une plaque carrée homogène 1. L’opérateur d’inertie [I ] O d’une plaque carrée, en son centre O, a pour expression (cf. chapitre 17) : Ma2 12

R

1 1 2

2. On en déduit que IOu = eu · [I ]O e u avec e u = cos u e x + sin u e y . Donc : IOu =

Ma2 12

et ISu =

2

p Ma2 a + m √ sin u − 12 4 2

=

Ma2 4

4 − sin(2u) 3

S17– 7. Centre d’inertie et moments d’inertie d’une demi-boule homogène 1. Le centre de masse est situé sur l’axe de symétrie Oz et tel que : zdm

MzC =

avec

M=r

2pR 3 3

et

d m = rp(R2 − z2 ) d z

Il en résulte que : r

2pR3 zC = rp 3

R 0

(R 2 − z2) z d z = rp R 2

z2 z4 − 2 4

R

= 0

rpR4 4

d’où zC =

3R 8

2. Le moment d’inertie par rapport à tout diamètre se déduit de celui relatif à la sphère entière : IOx = IOy = IOz =

2 2MR2 IO = 3 5

R

car IO =

R

r2 d m =

0

r2r 4pr 2 d r =

0

3MR2 5

S17– 8. Moments d’inertie principaux d’un double panneau solaire 1. Par symétrie, R = Oxyz est repère principal central d’inertie. Les moments principaux d’inertie s’obtiennent aisément à partir des résultats bien connus et du théorème d’Huygens : I1 = 2 ×

M 2

c2 c + a+ 12 2

2

=M

c2 2 + a + ac 3

I2 = 2 ×

M 2

b2 12

=

Mb2 12

On en déduit : I 3 = I1 + I2 = M c2/3 + a 2 + ac + b 2/12 . 2. Pour le moment cinétique d’un double panneau solaire, on a : L O/R0 = [I] OV /R0

avec

V/R 0 = V cos u ez + V sin u ey

d’où LO/R0 = I2 V sin u e y + I3 V cos u ez .

715

et problèmes S17– 9. Moment d’inertie principaux d’une antenne semi-cylindrique concave

1. Le repère R = Oxyz est principal d’inertie. D’autre part, en raison des symétries, les moments d’inertie sont ceux relatifs à un cylindre creux entier, la masse étant celle du demi-cylindre : IOx = MR2

R2 l2 + 2 12

IOy = M

IOz =

R2 l2 + 2 12

2. Pour le moment cinétique d’un radar demi-cylindrique concave, l’axe Oz étant axe principal d’inertie, on a : LO/R0 = [I]O V/R 0 = IOzV/R 0 e z . S17– 10. Centre d’inertie et moments d’inertie principaux d’un cône creux 1. La surface d’un cône est : dS

S= On en déduit : S = 2p

avec

R 1 h cos g

d S = 2pr

h

z dz = 0

dz cos g

et z =

pR 2 z h cos g

h 0

= pR

rh R

h cos g

2. Par symétrie, C se trouve sur l’axe de révolution Oz et tel que : zrs d S

rs SzC =

2pR pRh zC = cos g h cos g

ce qui donne

h

z2 d z =

0

2pRh2 3 cos g

et zC =

2h 3

3. Comme Oxyz est trièdre principal d’inertie, IOx = I Oy = IOz /2 + IOxy : r 2r s d S =

I Oz =

z2 rs d S =

IOxy = 2

2p R3 rs 3 cos g h

2

h

z3 d z =

0

2pR rs h cos g

h

r sphR3 MR 2 Mh2 tan 2 g = = 2 cos g 2 2

z3 d z = rs

0

pRh 3 Mh 2 = 2 cos g 2

2

On en déduit : I Ox = Mh tan g/4 + Mh /2. S17– 11. Moments d’inertie principaux d’un tore creux homogène

 c Dunod – Toute reproduction non autorisée est un délit

1. Commençons par calculer la masse du tore creux : 2p

M=

dm = 0

2prs R(a + R cos u) d u = rs × 2pR × a2p

Le moment d’inertie par rapport à l’axe de révolution est : d m(a + R cos u)2 =

I3 =

2prs R(a + R cos u)3 d u

soit : 2

I3 = 2prs R 0

(a3 + 3a2 R cos u + 3aR2 cos2 u + R3 cos3 u) d u = 2pr sR a 3 × 2p +

3aR2 × 2p 2

On trouve donc : I3 = M a 2 + 3R 2/2 . En outre : 2p

2p

2pr s R(a + R cos u)R2 sin2 u d u = 2prs R3

IOxy = 0 3

(a sin 2 u + R cos u sin 2 u) d u 0

2

soit : IOxy = 2prs R (pa) = MR /2. On trouve donc : I1 = I2 = M a 2/2 + 5R2 /4 .

716

Solutions des exercices 2. La masse du tore plein est : 2p

R

2

dm =

M=

0

r2p(a + r cos u)r d r d u = r × 2pa × pR

0

Le moment d’inertie I3 est : d m(a + R cos u)2 =

I3 =

2pr(a + r cos u) 3r d r d u

En développant, il vient, les termes en cos u et cos3 u donnant des contributions nulles : I3 =

R

2pr(a 3 + 3ar2 cos2 u) r d r d u =

2pr(2pa3 + 3par2 ) r d r = r2ap2 R2 a 2 +

0 2

3 2 R 4

2

On trouve donc : I3 = M a + 3R /4 . Pour obtenir I 1 = I 2, il suffit de calculer I Oxy : R

z2 d m =

IOxy =

0

soit :

R

IOxy = r2p On trouve donc :

r2p(a + r cos u)r2 sin 2 u d r r d u

0

2p

r3 d r

0

2p

R

2

(a + r cos u) sin u d u = r2p 0

r 3 d r (ap ) = r 2p 2 a

0 2

R4 MR2 = 4 2

2

I1 = I 2 = M a /2 + 5R /4 .

S17– 12. Moments d’inertie d’un anémomètre à coupelles Le moment d’inertie de chaque demi-sphère par rapport à l’axe de rotation passant par l’axe O1 z, parallèle à Oz et passant par le centre O1 de chaque sphère génératrice, est : I1 = 2I O1 /3 = 2mR2 /3. On sait en effet que le moment d’inertie d’une demi-sphère a même expression que celui de la sphère. En appliquant le théorème d’Huygens et en ajoutant les quatre contributions, on obtient : I Oz = 4

2mR 2 2 + mb 3

S17– 13. Moment d’inertie d’un pendule de torsion On trouve aisément en utilisant l’additivité des moments d’inertie et le théorème d’Huygens : I Oz = 4I 1 avec

I1 =

2MR2 mb2 + + M (R + b) 2 3 5

S17– 14. Moments d’inertie de la molécule d’ammoniac 1. Le moment d’inertie I 1 est le moment par rapport à l’axe de symétrie Ox de la molécule passant par l’azote, le centre de masse C de la molécule et le centre de masse CH des atomes H . Si on désigne par b le côté du triangle équilatéral formé par les trois atomes H , on a : h 2

b = 2a sin d’où : I1 = 4mHa 2 sin2

h 2

mi (OHi )2 = 3mH OH 2

I1 =

avec

i

−27

≈ 4 × 1, 67 × 10

× (101, 4 × 10

−12 2

2 OH = b cos 3

) × sin 2

107 2

p 6

b = √ 3 −47

≈ 4, 438 × 10

kg.m2

717

et problèmes

2. Pour calculer I⊥, c’est-à-dire le moment d’inertie par rapport à l’axe Cy (Fig. 17.26), C étant le centre de masse de la molécule, utilisons les propriétés de symétrie : I1 + ICyz où ICyz = mN (CN ) 2 + 3m H(CCH )2 2 est le moment d’inertie par rapport au plan Oyz . Comme mNCN + 3m HCC H = 0 , il vient : I 2 = I3 =

CCH −CN NCH + = 3m H mN m N + 3mH Il en résulte : ICyz = (NCH ) 2

9m2H m N + 3m H m 2N = m(NC H )2 (mN + 3mH)2

où :

4 3m Hm N 2 h sin et m = m 3 2 N + 3mH est la masse réduite entre l’atome d’azote et l’ensemble des trois atomes d’hydrogène. Finalement : (NCH )2 = a2 1 −

2

I2 = I3 = 2m Ha sin

h 4 2 2 h + ma 1 − sin 2 3 2

2

2

= ma

≈ 4, 242 × (1 − 0, 5236 × 0, 646) ≈ 2, 806 × 10

−47

4 mH −2 m 3

1−

sin

h 2

2

kg.m 2

S17– 15. Surface et fil en forme de triangle 1. Désignons par A , B  et C  les milieux des segments BC , AC et AB respectivement (Fig. 17.27a). Comme les aires des deux ensembles de triangles que l’on forme en traçant des parallèles à BC sont égales deux à deux (même hauteur et bases égales), les masses de ces derniers sont égales : Gs se trouve donc sur la médiane AA . Il en est de même pour les autres médianes, d’où Gs situé au point de concours des médianes. 2. a) Les trois côtés du fil triangulaire étant homogènes, on peut, pour déterminer le centre de masse Gl , les remplacer par leurs mileux A , B  , C  affectés respectivement des coefficients rl a/2 , rl b/2 et r lc/2 (Fig. 17.27b). b) L’équation vectorielle à laquelle satisfait G l est : 0 = r l a Gl A + r lb G l B + r l cGl C 





soit aussi 0 = a G lA + b G lB + c GlC 





c) Si on multiplie vectoriellement à droite cette dernière équation par GA , on trouve : 

d’où bc × GlJ  = cb × Gl I 

 c Dunod – Toute reproduction non autorisée est un délit

0 = b × aire//Gl B ,GlA + c × aire//Gl A,GlC 

G lJ  et G lI  étant les distances de Gl aux droites A B et A C . Comme on obtient des résultats analogues en   multipliant vectoriellement par G lB et Gl C , on a, si Gl H  est la distance de G l à B C  : 





G lH = Gl I = G lJ = r



d) D’après la relation précédente, le centre de masse Gl coïncide avec le centre du cercle inscrit au triangle A B  C ; c’est aussi le point d’intersection des bissectrices du triangle A B C  . Son rayon r est tel que l’aire S du triangle soit égale à la somme de trois contributions : S  = airetri.B  C Gl + airetri.A C Gl + airetri.A B Gl =

ar  b r  c r p r + + = 2 2 2 2

p = a + b + c étant le périmètre. e) En fonction de a  , b et de l’angle g que font entre eux C A  et C B , l’aire a pour expression : 2 2 2  S = ab sin g /2 avec c = a + b − 2a b cos g. Il en résulte : 2

S =

2

2

2

2

2

a b a b a b 2 (1 − cos g) = (1 + cos g)(1 − cos g ) = 16 16 16

2

2

1+

2

a  + b − c 2a b

2

2

1−

2

a + b  − c  2ab

2

718

Solutions des exercices

soit : 2

S =

1 1 2 2 2 2 2 2   2 2   (2ab + a + b − c  )(2a b − a  − b + c  ) = [(a + b )2 − c ][c − (a − b )2 ] 16 16

Ainsi : 2

S =

1  p            (a + b + c )(a + b − c )(c + a − b )(c − a + b ) = 16 2

p  −a 2

p  −b 2

p  −c 2

Dans les trois cas, p = 24 cm , d’où la masse : −6

M = rsp = 8 900 × 25 × 10

× 0, 48 = 0, 2225 × 0, 48 ≈ 0, 1068 kg

Quant à S et p , on trouve, pout chaque cas, respectivement : S  ≈ 21, 34 cm2

S = 24 cm2

 r ≈ 1, 78 cm

soit M = 106, 8 g

S  ≈ 27, 71 cm 2

 r = 2 cm

 r ≈ 2, 31 cm

3. Le moment d’inertie du fil triangulaire par rapport à l’axe G lz normal à son plan passant par Gl est la somme des contributions des trois côtés : I Glz = I C ,c + IB,b + IA,a + mc(G l C )2 + mb (G lB )2 + ma(Gl A)2 avec IC  ,c = m c c2 /12 = rsc3/12 , I B ,b = rsb 3 /12 et IA ,a = ra3/12 , puisque mc = rsc , m b = rsb et ma = rsa . Les termes supplémentaires, dus à l’application du théorème d’Huygens, font apparaître les quantités (GlA  )2 , (G lB ) 2 et (Gl C)2 . L’expression du premier s’obtient selon : 2

  2

(G l A ) 2 = Gl I + b − C I 



avec

Gl I  = r

et

g r = tan   2 CI

Il en résulte :

1 2b r  2 − + b tan2 (g/2) tan(g /2) Les autres quantités analogues s’obtiennent par simple permutation : (G l A )2 = r 

(G l B) 2 = r

2

1+

1 tan

2(a

2

/2)

2

+ c −

1+

2c r  tan(a /2)

(G lC )2 = r 

2

1+

1 2 (b

tan

2

/2)

+ a −

2ar tan(b /2)

Pour a = 10, 5 cm , b = 16, 5 cm et c = 21 cm , on trouve, sachant que S ≈ 21, 34 cm2 : sin a =

2S 2 × 21, 34 ≈ 0, 493 ≈ 8, 25 × 10, 5 b c 

2S 2 × 21, 34 ≈ 0, 774 ≈   5, 25 × 10, 5 ac 2S 2 × 21, 34  sin g =   ≈ ≈ 0, 985 ab 8, 25 × 5, 25 sin b =

d’où

tan

d’où

tan

d’où

a 2

b 2 g tan 2

= 0, 263 = 0, 474 = 1, 187

l’angle g étant supérieur à p/2 . En effectuant, on trouve : Gl A ≈ 6, 98 cm, G l B ≈ 5, 3 cm et  Gl C ≈ 2, 32 cm. On en déduit : IGlz = rl

c3 b3 a3 + + + c(GlC ) 2 + b(Gl B ) 2 + a(Gl A  )2 12 12 12

avec rl = 0, 2225 kg.m −1 = 2, 225 g.cm−1 . Le calcul donne : IGl z ≈ 518, 6 g.cm2 . S17– 16. Quantité de mouvement et moment cinétique d’un ensemble tige et disque 1. La quantité de mouvement et le moment cinétique ont pour expressions respectives : P=l

m ml2 ˙ + m u˙ e u et L O = u e z + OC × mvC + 0 = 2 3

les points A et C ayant même vitesse lu˙ e u.

ml2 + ml 2 − mlr sin u u˙ e z 3

719

et problèmes 2. Dans ce cas où la tige et le disque sont soudés, on a, puisque vA = l ˙u eu et v C = (l − r )u˙ eu : m P = l u˙ + (l − r )mu˙ eu 2

LO =

et

ml2 ˙ mr 2 + m(l − r) 2 ˙u ez u ez + 3 2

S17– 17. Quantité de mouvement et moment cinétique de deux tiges homogènes articulées Comme OC 1 et OC2 s’explicitent selon : (l 1/2) cos u 1 (l 1/2) sin u1 0

R

l 1 cos u 1 + (l 2/2) cos u2 l 1 sin u 1 + (l2/2) sin u2 0

et R

les vitesses v1 et v 2 des centres de masse sont respectivement :

R

−(l1 /2)u˙ 1 sin u1 +(l1 /2)u˙ 1 cos u 1 0

et R

−l1 u˙ 1 sin u 1 − (l2 /2)u˙ 2 sin u2 l 1u˙ 1 cos u1 + (l2/2)u˙ 2 cos u 2 0

On en déduit la quantité de mouvement et le moment cinétique : P = m1 v1 + m 2v 2 soit : LO = ez

L O = LO,1 + L O,2 = I Oz,1 V 1 + IC 2 z,2 V2 + OC2 × m2v2

m 1l 21 ˙ m 2l 22 ˙ l2 l1 l2 ˙ 2 u1 + u 2 + m2 l1 u˙ 1 + 2 ˙u2 + (u1 + u˙ 2) cos(u1 − u 2) 3 12 4 2

S17– 18. Quantité de mouvement et moment cinétique d’une plaque triangulaire a La quantité de mouvement P s’écrit : MV × OC avec OC = √ e x et V = c˙ ez + u˙ e u . 3  Comme la base de R est principale d’inertie et : √ √ 3 1 1 3 eu = e  − ey ez = sin u ew + cos u ez  ew = e x + e 2 x 2 2 2 y la quantité de mouvement P et le moment cinétique LO = [I] OV s’explicitent respectivement selon : √ √ ˙ +c ˙ sin u/2 3u/2 0 a 3 √ Ma ˙ cos u ˙ ˙ c = √ M 0 −u/2 + c 3 sin u/2 × √ 3 ˙ ˙ −c ˙ 3 sin u/2 R  c cos u R 0 R u/2  c Dunod – Toute reproduction non autorisée est un délit

et

R

√ ˙ +c ˙ I 1 ( 3u/2 √ sin u/2) ˙ I 2 (−u/2 + 3c˙ sin u/2) ˙ cos u I3 c

avec I3 = I 1 + I 2. Calculons I1 et I2 : I1 =

2

2

y rs d x d y = 2rs

d’où : 2r I1 = √ 3 3

y dy

√ a 3/ 2 0

x2 d x

√ a 3 −x 2 √ a 3 −x 2

avec

=

√ 3 x y= 3

Ma2 24

Quant à I2 , il vaut : I2 =

2

rs x d x d y = rs

√ 2rs 3 x d x(2y) = 3

a

2

0



3/ 2

x3 d x =

3 Ma 2 8

720

Solutions des exercices

S17– 19. Moment cinétique d’une barre tournant autour d’un axe 1. On a : LO = OA i × m iv i = OAi × m iVOA i sin u0 e u0 = (V sin u0 ) i m iOA2i e u0 , e u0 étant le i i vecteur unitaire perpendiculaire à la barre et contenu dans le plan qu’elle fait avec D. Comme la sommation entre −l/2 et l/2 donne Ml2 /12, on obtient : Ml2 V sin u 0 e u0 L O= 12 2. Les deux vecteurs L et V ne sont pas colinéaires sauf pour u0 = 0, u 0 = p/2 et u0 = p ; dans ces cas, l’axe de rotation est axe principal d’inertie.

Chapitre 18 S18– 1. Conservation de la quantité de mouvement L’ensemble est pseudo-isolé et initialement au repos : 0 = m1(v1 + v 2) + m2v 2 si v 1 est la vitesse de l’homme par rapport au radeau. Donc : m1 m1 v2 = − v  et Dr2 = − Dr 1 m 1 + m2 1 m1 + m2 Le déplacement du radeau est donc indépendant de la vitesse avec laquelle l’homme se déplace sur le radeau. S18– 2. Conservation du moment cinétique 1. Les moments cinétiques du disque et de l’homme s’écrivent respectivement : MR2 ˙ ˙ ez f ez et L hO = OH × mvh = m(v h + Rf) 2 vh = vee u désignant la vitesse de l’homme sur les bords du plateau par rapport à ce dernier. ˙ LD O = I f ez =

h 2. Comme l’ensemble est pseudo-isolé et initialement au repos, on a : LO = LD O + L O = 0. Il en résulte, en  explicitant, puisque Dt = pR/vh :

v h MR2 ˙ m  ˙ d’où f˙ = − f = −mR(vh + R f) 2 m + M /2 R

˙ =− et Df = fDt

p 1 + (M /2m)

S18– 3. Cerceau lesté roulant sans glisser sur une droite Au point géométrique de contact I, mobile, par lequel passe la réaction, le théorème du moment cinétique s’écrit : d LI + v I × P = IA × mg avec P = M v K + mvA dt Les composantes de vK , v A et P dans R sont respectivement :

R

Comme vI = vK et

−Ru˙ 0 0

R

−Ru˙ (1 − cos u) R u˙ sin u 0

R

−MR˙u − mRu˙ (1 − cos u) mRu˙ sin u 0

LI = LCI + LAI = [2MR 2u˙ + 2mR 2 u˙ (1 − cos u)] ez, il en résulte que : 2 2 2 2R ¨ u [M + m(1 − cos u)] + mR u˙ = −mgR sin u

2. Si u petit, l’équation précédente devient : 2 2MR ¨ u + mgRu ≈ 0

2 soit ¨ u + v0 u ≈ 0

avec

2

v0 =

mg 2MR

et T0 = 2p

2MR mg

1/ 2

721

et problèmes S18– 4. Mouvement du centre de masse d’une poulie mobile

Pour la poulie mobile, les théorèmes de la quantité de mouvement et du moment cinétique en son centre C 1 donnent : d L1 m1 a1 = −m 1g + T1 + T et = C1 A × T + C1 A1 × T1 dt où T1 est la tension exercée par le brin gauche et T la tension exercée par le brin droit. On obtient en explicitant, u1 en raison du roulement sans glissement du fil sur la poulie : sachant que a1 = r1 ¨ et

m1 a1 = −m1g + T1 + T

m1 r 21¨ u1 = r 1(T − T1 )

Pour la masselotte A, la loi fondamentale ma = mg − T donne, en projetant, m¨ x = mg − T , x étant compté ¨ positivement suivant un axe vertical descendant. Comme x = 2a1 , on trouve l’accélération a1 en éliminant les tensions : 2m − m 1 2h − 1 a1 = g= g 4m + 2m1 4h + 2 Pour h = 0, 5, l’accélération est nulle ; pour h  1, elle vaut −g/2 ; pour h  1, elle est égale à g/2. S18– 5. Rupture d’une cheminée d’usine 1 . a) Le théorème de la quantité de mouvement donne maC = mg + R, soit en projection dans R : 2 −(l/2)u˙ m (l/2)¨ u 0

mg cos u −mg sin u + 0

=

Rr Ru Rz

Quant au théorème du moment cinétique d LO / d t = OC × mg, il s’explicite selon : ml 2 ¨ l mgl u e z = e r × mg ex = − sin u e z 3 2 2

b) En multipliant l’équation précédente par u˙ et en intégrant, on obtient, compte tenu des conditions initiales : 3g ¨ u u˙ = − sin u u˙ soit 2l

u˙ 2 3g = cos u + Cte 2 2l

et

˙u 2 = 3g (1 + cos u) l

c) D’après le théorème du centre de masse, on a : R=

l 2 −mg cos u − m u˙ 2

er + mg sin u + m

u l¨ 2

5 mg e u = − mg cos u − 3 2 2

er +

mg sin u e u 4

 c Dunod – Toute reproduction non autorisée est un délit

Ainsi a = 5/2. 2. a) Comme L K = LO + KO × m 1vC = L O + m 1vC × OK, il vient : LK =

m1 r2 ˙ r 1 mr 3 ˙ u ez + m1 u˙ e u × r er = − u ez 3 2 6 l

puisque m1 = mr/l. En identifiant, on trouve g = 1/6. b) De l’équation : d LK + vK × m1 vC1 = G2→1,K + KC1 × m1 g + KO × R dt on déduit, puisque vK et v C1 sont parallèles : G2→1,K = −

mr 3 ¨ r mr u ez + e r × g e x + r er × R = 6l 2 l



mr 3 ¨ mgr2 mgr sin u + sin u ez u+ 6l 2l 4

ce qui donne : G2→1,K = mg sin u e z

r r2 r3 − + 2 4 2l 4l

=

mgl sin u r r 1− 4 l l

2

ez

d’où l =

1 4

722

Solutions des exercices c) Comme f (u) = u(1 − u)2,

df = (1 − u)2 + 2(1 − u)(−1)u = (1 − u)(1 − 3u) du

La dérivée est donc nulle pour u = 1 et u = 1/3. Ainsi, la norme de G2→1,K est maximale pour u = 1/3, d’où la rupture probable au tiers de la longueur en cas d’incident. S18– 6. Mouvement d’une barre en contact sans frottement avec deux plans perpendiculaires 1. Le centre de masse se trouve à la distance l/2 de l’origine O ; il décrit donc le quart de cercle de centre O et de rayon l/2. Le théorème du centre de masse s’écrit, puisque les forces sont le poids mg et les réactions RA et R B : ce qui donne en projetant ma x = RB

ma = mg + RA + RB

et may = −mg + R A

Exprimons les coordonnées de C, sa vitesse vC et son accélération aC en fonction de a :

R

l cos a l sin a 0

l 2

R

2

−a˙ sin a a˙ cos a 0

l 2

R

−¨ a sin a − a˙ cos a l¨ a cos a − l a˙ 2 sin a 0

Il en résulte les deux équations suivantes : ml (−a ¨ sin a − a˙ 2 cos a) = RB 2

et

ml (¨ a cos a − a˙ 2 sin a) = −mg + R A 2

2. Au point I où se coupent les réactions, leurs moments sont nuls. Il en résulte que : d LI l + vI × vC = IC × mg = mg cos a ez dt 2 avec vI parallèle à vC . Quant à L I il vaut, en notant que l’angle (Ox, AB) = (p − a) et que la vitesse de rotation est d(p − a)/ d t = −a˙ : LI = LC + IC × P = LC − OC × P =

ml2 ml 2 ml2 (−a)e ˙ z − a˙ e z = − a˙ ez 12 4 3

L’équation du mouvement est donc : −

ml2 l a ¨ = mg cos a 3 2

soit a ¨=−

3g cos a 2l

3. Si l’on multiplie l’équation différentielle précédente par a, ˙ on obtient une intégrale première. En effet : a ¨ a˙ = −

3g cos a a˙ donne 2l

3g a˙ 2 = − sin a + Cte 2 2l

soit

2 a˙ = −

3g (sin a 0 − sin a) l

en tenant compte des conditions initiales. Cette intégrale première est celle de l’énergie mécanique (cf. Exercice P20 . 8). 4. Il y a rupture du contact en B si : RB = −

ml (¨ a sin a + a˙ 2 cos a) = 0 2

soit

cos a −

3g sin a + a˙ 2 2l

=0

Le contact cesse pour a = p/2, ce qui était prévisible et sans intérêt, et pour a = a1 tel que : a˙ 21 = −

3g sin a 1 soit 2l



3g 3g (sin a 0 − sin a 1 ) = − sin a1 l 2l

d’après l’intégrale première. Finalement : sin a1 = (2/3) sin a 0 .

723

et problèmes S18– 7. Mouvement d’une chaîne

1. Appliquons le théorème du centre de masse à la chaîne. La seule force extérieure est le poids de la partie pendante, de longueur OB = x. Comme elle vaut gMx/L, il vient : x Ma C = Mg avec a C = aB L puisque la distance entre A et B est L/2. On en déduit l’équation différentielle suivante : ¨x = g

x L

2 soit ¨ x−a x =0

en posant

a=

g L

1/ 2

2. La solution est bien connue (cf. annexe 3) : x(t) = C + exp(at) + C− exp(−at ) avec

Ainsi, C+ = C− = l/2 et :

x (t) = On en déduit : t=

x(0) = l = C+ + C−

et ˙x(0) = 0 = a(C+ − C − )

l [exp(at) + exp(−at)] = l cosh(at ) 2

L 1 argcosh l a

=

1 5, 3 argcosh(100) = = 1, 7 s 3, 13 3, 13

S18– 8. La mécanique du football 1. a) Le moment d’inertie I du ballon est celui d’une sphère creuse de masse M . On calcule aisément le moment d’inertie par rapport à un diamètre à l’aide du moment d’inertie par rappport au centre de masse C : 2 I = IC 3

avec

mi (x2i + y 2i + z 2i )

IC = i

En raison de l’égalité des trois moments d’inertie par rapport aux différents diamètres, il vient : IC =

1 2

mi(x 2i + y2i ) + i

m i (y2i + z 2i ) + i

m i (y2i + z2i ) = i

3 I 2

Il suffit donc de calculer IC : mi (x 2i + y 2i + z2i ) =

IC =

 c Dunod – Toute reproduction non autorisée est un délit

i

m ir2 = Mr2 i

2 2 d’où I = Mr 3

b) On trouve : I = (2/3)Mr 2 = (2/3) × 0, 420 × 0, 112 ≈ 3, 34 × 10−3 kg.m2 . c) Admettant que l’air enfermé dans le ballon est un gaz parfait, ce qui est une hypothèse très raisonnable à 300 K, il vient : n=

2 × 10 5 × 4p × (0, 11)3 /3 pV ≈ 0, 45 = 8, 314 × 300 RT

d’où M air = 0, 45 × Mm = 0, 45 × 29 ≈ 13 g

ce qui est négligeable devant la masse du ballon.

2. a) Les vitesses des points supérieur et inférieur du ballon, valent, en raison du champ des vitesses d’un solide : vA = vC + AC × V , d’où, pour V = 5 tr.s−1 : et :

v S = vC + SC × V d’où

v S = v C + rV = 33, 5 m.s−1

v I = v C + IC × V d’où

vI = vC − rV = 26, 5 m.s −1

b) La valeur de V pour laquelle la vitesse du point inférieur du ballon est nulle est telle que : vI = vC − rV = 0

d’où

V=

30 vC = 272, 72 rad.s−1 = 0, 11 r

soit

272, 72 = 43, 4 tr.s−1 2p

724

Solutions des exercices

Cette condition est nécessaire si le ballon roule sans glisser sur la pelouse ; notons que, dans la pratique, les joueurs font rouler sans glisser le ballon, avec une vitesse de translation et une vitesse angulaire V bien plus faibles, afin de suivre de près le ballon. 3. a) Le référentiel R , lié au terrain de jeu, est supposé galiléen, pour les deux raisons suivantes :

i) la force d’inertie d’entraînement terrestre est contenue dans le poids, puisque ce dernier est une combinaison vectorielle de la gravitation et de cette force d’inertie, ii) la force d’inertie de Coriolis terrestre est très faible, à moins que les vitesses considérées soient de l’ordre de 700 m.s−1 , qui est la vitesse du balle de fusil, ce qui est exclu dans le cas considéré. Dans ces conditions, le référentiel lié au terrain de football peut être considéré comme une excellente approximation d’un référentiel galiléen. b) Les forces qui s’exercent sur le ballon sont uniquement le poids, la force exercée par l’air étant négligée. Le théorème de la quantité de mouvement et le théorème du moment cinétique au centre de masse C , par rapport à R s’écrivent donc : dP d LC = Ma C = Mg et =0 dt dt puisqu’on peut considérer que le poids s’exerce en C . On en déduit : aC = g et LC = Cte. On sait qu’en un point mobile O , le théorème du moment cinétique s’applique en prenant en compte d’un terme complémentaire : d LO  = dt

MO  + v O  × MvC

d’où

d LC = dt

MC

puisque vC × Mv C = 0

c) En intégrant l’équation du mouvement a C = g , on obtient : vC = gt + v i ce qui prouve bien que vC , et donc le mouvement, est contenu dans le plan formé par g et vi . Il en résulte qu’en l’absence de joueur modifiant la trajectoire du ballon, le tireur de corner ne peut pas envoyer le ballon directement dans la cage adverse. d) Les équations différentielles du mouvement auxquelles satisfait le mouvement de C dans le plan vertical défini par g et vi sont, selon l’axe horizontal Oy et l’axe vertical ascendant Oz : ¨ y = 0 et ¨z = −g ce qui ˙ ˙ , a y = v z = −gt + v étant l’angle que fait vi avec le plan du terrain. donne, par intégration : i cos ai et i sin a i i Ce mouvement est bien connu. La trajectoire est une parabole. Comme la vitesse angulaire initiale est nulle, le moment cinétique est nul, ainsi que la vitesse angulaire car LC = IV . 4a) La quantité m = 2raV b/M n’a pas de dimension physique, puisque c’est le rapport de deux masses. Elle vaut : m=

2 × r a Vb 2 × 1, 3 × 4p × 0, 113 = 0, 0345 = 3 × 0, 42 M

b) En tenant compte de la force de Magnus, les théorèmes du centre de masse et du moment cinétique s’écrivent respectivement : MaC = Mg + 2rVb V × v C et LC = Cte puisque la pesanteur et la force de Magnus s’exercent en C .

c) Comme la force de Magnus est perpendiculaire à vC , le centre de masse C n’évolue plus dans le plan, défini par g et vi . La trajectoire est courbe et le ballon peut entrer dans la cage adverse sans intervention d’un autre joueur, encore faut-il que le vecteur V soit convenablement orienté. Dans le cas de la figure, il faut que Vz soit négatif. d) L’équation vectorielle à laquelle satisfait la vitesse vC du centre de masse C du ballon, peut s’écrire aussi : aC,h + a C,v = g + m V × (vC,h + vC,v ) car aC = aC,h + a C,v

et v C = vC,h + v C,v

725

et problèmes

les indices v et h étant relatifs aux contributions verticale et horizontale. Comme V est vertical, il vient, en simplifiant : a C,h + aC,v = m V × vC,h + g, soit, en séparant contributions verticale et horizontale :

d v C,v d v C ,h = VM × vC,h avec VM = m V = g et a C,h = dt dt L’équation du mouvement dans le plan horizontal est caractéristique d’un mouvement circulaire dont le sens de courbure est donné par le vecteur VM , de même direction et de même sens que V . a C,v =

Chapitre 19 S19– 1. Mise en mouvement d’un bloc sur un plan incliné Le théorème du centre de masse, appliqué à l’ensemble, donne : (m1 + m2 )¨x = m1g − m2 g sin a − mm2 g cos a

puisque la masse 2 glisse avec le facteur de frottement m. Donc : ¨ x=

h − sin a − m cos a g = 0, 11 m.s−2 1+h

S19– 2. Contact de deux solides avec frottement 1. Le théorème de la quantité de mouvement appliqué aux deux solides donne : m1a1 = T1 et m 2a 2 = F(t ) − T1

où T1, qui représente l’action transversale de la planche sur la barre, satisfait à la relation |T1 |  mm 1g. 2. À la limite du glissement de la planche sur la barre, on a a 1 = a2 et |T1| = mm 1 g. Donc : a1 = mg

a2 =

at0 mm1 g − m2 m2

d’où t0 =

m(m 1 + m2 )g a

 c Dunod – Toute reproduction non autorisée est un délit

3. Dans la phase de non-glissement, le système forme un seul solide de masse m1 + m2 ; on en déduit alors : F a1 = a2 = m 1 + m2 Dans la phase de glissement, a1 = mg et a2 = (at − mm1g)/m 2 . S19– 3. Cylindre sur un tapis roulant incliné 1. La vitesse de glissement du cylindre 1 sur le tapis roulant 2 est : v1/2 = v1 − v 2 = v C + AC × V − v0 = ( ˙xC + ru˙ − v0 ) ex 2. Les théorèmes généraux ma = mg + R et d LC / d t = CI × R donnent, en projection : ¨ x C = −g sin a +

Rt m

0 = −g cos a +

Rn m

et

mr2 ¨ u = rR t 2

3. La dérivée par rapport au temps de la vitesse de glissement a pour expression : 3R t m Comme |Rt | = mRn = mmg cos a, on obtient, dans la phase initiale de glissement (vg < 0, Rt > 0) : v˙ g = ¨ x C + r¨ u = −g sin a + v˙g = 3g cos a m −

tan a 3

726

Solutions des exercices

4. Si m > tan a/3, vg augmente à partir de −v0 ; il y a d’abord glissement puis non-glissement. Si m < tan a/3, vg diminue à partir de −v0 ; il y a glissement uniquement. S19– 4. Sphère sur un plan horizontal 1. Les théorèmes de la quantité de mouvement et du moment cinétique s’écrivent : ma C = mg + R et

d LC 2mr 2 d V = = CI × R = CI × R t car CI × Rn = 0 dt 5 dt

2. Comme vg = v 1 − v2 = vC + IC × V − 0, on en déduit : d v g / d t = aC + IC × d V/ d t. En tenant compte des équations du mouvement, on obtient : d vg R R Rn 7Rt 5 5 − + = g+ [CI × (CI × R)] = g + + Rt = g + dt 2m 2m m 2mr 2 m m car [CI × (CI × R)] = −r2 Rt . Or vg est contenu dans le plan horizontal et aC · ez = 0. Donc :

2 Rn d vg 7Rt 1 d vg = = = 0 et vg · · vg < 0 m dt 2 dt 2m Il en résulte que la norme de vg ne peut que diminuer. Le mouvement se termine donc toujours par une phase sans glissement.

g+

S19– 5. Expérience de Timochenko : oscillations d’une barre reposant sur deux galets 1. Le théorème du centre de masse, appliqué à la masselotte A, donne, si Ft représente la tension qu’exerce vers le haut le fil : x = mg − F t ma A = mg + Ft soit, en projetant m¨ Quant aux théorèmes généraux appliqués à la barre seule, ils s’écrivent : MaC = Mg + R 1 + R2 + F

et

d LC = 0 = CO 2 × R2 + CO1 × R1 dt

la barre n’ayant qu’un mouvement de translation, la poulie étant de masse négligeable (F = Ft ) et la direction de F passant par C. En projetant, on trouve : M¨ x = T 1 + Ft

0 = −Mg + N1 + N2

−x −h × 0

et

0 N2 0

+

l−x × −h 0

T1 N1 0

=

0 0 0

ce qui donne, en effectuant, N1 l − x(N1 + N2 ) + hT1 = 0.

2. Comme la barre commence par glisser (vb/G1 = 0 − v A1 = 0), il vient : |T1 | = m|N1 | soit

Comme N1 (l − hm) − Mgx = 0, il vient : N1 =

Mgx l − hm

− T1 = mN 1 puisque T 1 < 0 et T 1 = −m

Mgx l − hm

3. On déduit de ce qui précède, en éliminant Ft , l’équation différentielle à laquelle satisfait x : (M + m ) ¨ x+

mMg x = mg l − hm

soit

¨ + v 20X = 0 X

avec

X = x − x e et v 20 =

mMg (M − m)(l − hm)

Le mouvement est donc oscillatoire autour de la valeur xe = m/mM (l − hm) : x = xe + A cos (v 0t + f) de période T 0 = 2p/v 0. De la mesure de la période, on peut déduire la valeur du facteur de frottement m.

727

et problèmes S19– 6. Cylindre sur un plan horizontal

La vitesse de glissement du cylindre sur le plan a pour expression : vg = v1 − v 2 où v1 est la vitesse du point du cylindre et v2 la vitesse du point du plan, en contact au point I. Comme v2 = 0, il vient : ˙ ex v 1/2 = v1 = v C + IC × V = (˙x − r u)

Les théorèmes généraux appliqués au cylindre donnent : maC = mg + R et d LC / d t = CI × R d’où, en explicitant : m

¨x 0 =m 0

0 0 + −g

Rx Ry Rz

et

0 u˙ 0

mr 2 2

0 0 −r

=

Rx × 0 = mg

0 −rR 0

Il en résulte les équations suivantes : u mr ¨ = −Rx 2 Le mouvement commence par une phase de glissement puisqu’initialement vg = −rV0 ex = 0. Comme vg = x˙− r˙u, il vient : m¨ x = Rx

Ry = 0

d vg 2Rx 3R x Rx = ¨x − r ¨ u= + = dt m m m

R z = mg et

d’où vg

dvg 3R x = vg < 0 dt m

et

d dt

v2g 2

0, d’où : |R t | = m |Rn| donne Rx = mmg. On en déduit que : d vg = 3mg et v g = 3mgt + Cte = 3mgt − rV 0 dt

compte tenu des conditions initiales. Cette phase s’achève à l’instant t1 = rV0 /(3mg). On en déduit : x = mg m¨ x = Rx = mmg d’où ¨

et

˙x = mgt + Cte = mgt

car à l’instant initial le centre de masse est immobile. En intégrant, on trouve : mgt2 mgt2 + Cte = 2 2 puisque x = 0 pour t = 0. La distance parcourue jusqu’à l’instant t1 est donc : x=

 c Dunod – Toute reproduction non autorisée est un délit

x1 =

mgt 21 mg r2V 20 r 2V 20 = = 2 2 9m2 g 2 18mg

Dans la deuxième phase, on a : x˙ = a ˙u et ¨x = a¨ u d’où Rx /m = −2R x /m et Rx = 0. Donc : ¨ x=0

¨ u=0

x˙ = Cte = mgt1 =

rV0 3

et

V0 x˙ u˙ = Cte = = a 3

S19– 7. Influence du freinage sur le mouvement propre d’un véhicule 1. La relation algébrique entre x˙ C et u˙ est facile à établir : x˙C = −ru˙ (cf. chapitre 16).

2. Les théorèmes du centre de masse et du théorème du moment cinétique au centre de masse s’écrivent respectivement : d L∗C 0 = mg + R 1 + R2 − ma C et = CI 1 × R1 + CI2 × R2 dt R 1 et R2 désignant les réactions exercées par le sol sur les roues du véhicule, le contact étant ponctuel. Il vient, en explicitant : 0 = T1 + T2 − M¨ xC et 0 = −Mg + N 1 + N2

728

Solutions des exercices

Comme L∗C est le moment cinétique des roues, il vient : 0 0 = u 2I0 ¨

−b 1 × h 0

T1 N1 0

+

−b1 −h 0

×

T2 N2 0

u = −b1 N1 + b2 N2 + h(T 1 + T2 ) soit −b1N1 + b 2 N2 = − (Mh + 2I0 /r) ¨xC. On a donc l’équation : 2I0 ¨

3. On déduit, de cette dernière équation et de l’équation N1 + N2 = Mg, les expressions de N1 et N 2 : N1 =

Mgb 2 + ¨xC (Mh + 2I0 /r) b1 + b2

et N 2 =

Mgb 1 − ¨xC(Mh + 2I 0 /r) b 1 + b2

4. On voit que, si le véhicule accélère (x¨ C > 0), N1 > N 2 : le véhicule exerce une pression plus forte à xC < 0), N 1 < N 2 : le véhicule exerce une pression plus forte à l’arrière en se cabrant. En revanche, s’il freine (¨ l’avant en piquant du nez. S19– 8. Différentes phases du mouvement d’un disque D’après les résultats établis au chapitre 19, trois cas doivent être envisagés suivant la valeur initiale de la vitesse de glissement vg = (v0 − rV0 ) ex . 1. v 0 > r0 V0 . Initialement, on a donc : v g > 0. Le mouvement comporte deux phases, la première avec glissement et la seconde sans glissement. Phase 1. vg > 0 d’où R t < 0 et R t = −mmg. Il en résulte que : ¨x = −mg et u¨ = −2mg/r

Ainsi, les mouvements de translation et de rotation sont uniformément décélérés : 2mg ˙x = −mg t + v0 et u˙ = − t − V0 r Cette phase se termine lorsque vg = 0, à l’instant t1 tel que : v 0 − rV 0 x˙ + ru˙ = −3mgt 1 + v 0 − rV 0 = 0 d’où t 1 = 3mg

˙ La réaction R est a priori contenue dans le cône de frottement. Elle ne sera Phase 2. v g = 0 d’où x˙ = −r u. déterminée qu’une fois le mouvement connu. Compte tenu de l’absence de glissement, les équations du mouvement donnent : ¨ = 0 = 3R t d’où ¨x = 0 et u ¨=0 ¨ x + ru m Les mouvements sont donc uniformes et la réaction R est normale à Ox. 2. Initialement, v0 = r 0V 0 d’où vg = 0. Là, il n’y a qu’une seule phase analogue à la deuxième phase précédente. 3. Initialement, v 0 < r0V 0 d’où v g,x < 0. Comme |v g| ne peut que diminuer, vg va augmenter jusqu’à s’annuler. On observera là aussi deux phases, l’une avec glissement et l’autre sans glissement. ¨ = 2mg/r Phase 1. vg < 0 d’où R t > 0 et R t = mmg. On en déduit : x¨ = mg et u Les mouvements de translation et de rotation sont donc uniformément accélérés : ˙ = 2mg t − V 0 ˙x = mgt + v0 et u r Cette phase se termine à l’instant t3 tel que : vg = (3mgt + v 0 − rV 0) e x = 0 soit t 3 = Phase 2 Elle est analogue à la phase 2 de non-glissement du 1er cas. Sur la figure S19.1, on a résumé l’ensemble de ces résultats.

rV0 − v 0 . 3mg

729

et problèmes

v0

v0 > rV0

. x

v0 O

vg Rt

. rµ

- r V0

v0 vg

O

t

v0 = rV0

x. Rt . rµ

- rV 0

a)

O

t

v 0 < rV 0

. x Rt vg

t . rµ

- rV 0

b)

c)

F IG . S19.1.

Chapitre 20 S20– 1. Oscillations d’un volant roulant sans glisser sur un guide circulaire 1. La condition de roulement sans glissement est facile à établir en écrivant que la vitesse du point du volant en contact avec le guide est nulle (cf. chapitre 16) : ˙ =0 (R − r ) u˙ + r (u˙ + f)

d’où

˙ =0 Ru˙ + r f

2. En traduisant la conservation de l’énergie mécanique car, en l’absence de glissement, la puissance de la réaction est nulle, on obtient Em = Ek + Ep = Cte avec : Ep = −mg(R − r ) cos u, si E p = 0 pour u = p/2, et : Ek =

2

1 2 1 ˙ 2 = 1 m(R − r )2 ˙u2 + 1 I u˙2 1 − R mv + I(u˙ + f) 2 C 2 2 2 r

3. En dérivant, on trouve : I u + mg sin u = 0 + m (R − r) ¨ r2

2 soit ¨ u + v 0u = 0

=

en posant

1˙2 I 2 u (R − r ) m + 2 2 r

2

v0 =

On en déduit la période T0 = 2p/v0 .

mg (m + I /r2 ) (R − r)

4. Le moment cinétique au point mobile I de contact a pour expression : ˙ e z = (I + mr 2 )u˙ 1 − R L I = (I + mr 2 )(u˙ + f) r

ez

 c Dunod – Toute reproduction non autorisée est un délit

puisque I est le C.I.R. Le théorème du moment cinétique en I restitue l’équation du mouvement : d LI + mvI × vC = IC × mg dt

soit (I + mr2) 1 −

R r

¨ u = mgr sin u

vI et vC étant parallèles. S20– 2. Pendule composé formé d’une tige et d’un disque articulés 1. Les liaisons étant parfaites, la conservation de l’énergie mécanique fournit une première équation : Em = Ek + E p = Cte avec E k =

Mr2 2 1 ml2 ˙ 2 Ml2u˙ 2 u + + V 2 3 2 4

et Ep = −

mgl cos u − Mgl cos u 2

en prenant comme origine de l’énergie potentielle sa valeur pour u = p/2. Une seconde équation de conservation est donnée par la projection du moment cinétique du disque sur l’axe de rotation. La liaison en C étant parfaite, on a : Mr2 LCz = V = Cte d’où V = Cte = V0 2

730

Solutions des exercices 2. D’après ce qui précède, il vient : ˙u2 l2 2

m + M − gl cos u 3

m u + V2 u ≈ 0 + M = Cte soit ¨ 2

avec

2

V =

M + m/2 g M + m/3 l

Si l’ensemble formait un solide, on aurait u˙ = V et par conséquent : Em =

1 ˙2 u 2

ml 2 Mr2 + Ml2 + 3 2

avec : 2

V =

− gl cos u

m + M = Cte d’où 2

M + m/2 g < V2 M + m/3 + Mr2 /2l2 l

2 ¨ u + V u = 0

T > T

et donc

S20– 3. Conservation de l’énergie d’un cerceau lesté On a E m = E k + Ep = Cte, avec : Ek =

1 1 2 2 2 2 2 2 2 (Mr + Mr )˙u + mv A = Mr u˙ + mr u˙ (1 − cos u) 2 2

et Ep = Mgr + mgr (1 − cos u)

Donc :

r2u˙ 2 [M + m(1 − cos u)] + mgr (1 − cos u) + Mgr = Cte ce qui donne, en dérivant et en simplifiant par u˙ : 2r2 ¨ u[M + m(1 − cos u)] + mr 2 ˙u 2 sin u + mgr sin u = 0 En représentant graphiquement Ep (r), on montre aisément qu’il existe deux mouvements possibles : le mouvement est révolutif si Em  (M + 2m)gr ; il est oscillatoire si Mgr < Em < (M + 2m)gr. S20– 4. Système articulé de deux barres 1. À l’aide du théorème de Kœnig relatif à l’énergie cinétique de chaque barre, on trouve : Ek = 2 ×

ml2 ˙ 2 ml2 ˙ 2 u + u 12 4

1 2

=

ml2 ˙ 2 u 3

2. La conservation de l’énergie mécanique donne, puisque : Ep = 2 × (mgl/2) sin u si Ep = 0 pour u = 0 : Em =

ml 2 ˙ 2 u + mgl sin u = Cte 3

3. On a DEk + DEp = 0 avec : DEp = −mgh et DEk =

ml 2u˙ 2 3 1/ 2

puisque vA = (lu˙ cos u)u=0 = v0 . Il en résulte que : v0 = (3gh)

u˙ 0

= 0

mv 20 3

.

S20– 5. Mouvement d’une plaque homogène de forme carrée 1. En appliquant le théorème d’Huygens, on obtient : Ek =

1 ˙2 I Oxu avec 2

√ I Ox = I Cx + m a 2

puisque ICx = m[(2a)2 + (2a) 2 ]/12 = 2ma2 /3.

2

=

2 2 8 ma + 2ma2 = ma2 3 3

√ 2. L’énergie potentielle de pesanteur a pour expression Ep,1 = mgz C = −mgzC = −mga 2 cos u, si Ep1 = 0 pour u = p/2.

731

et problèmes 3. Quant à l’énergie potentielle centrifuge, elle vaut : E p,2 = −

V2 2

(x 2 + y 2 ) d m = −

√ Comme I Oz = ICz + m(a 2 sin u)2 avec ICz = ma2 /3, il vient :

V 2 IOz 2

1 + sin2 u Ep,2 = −ma2 V2 6 4. Les positions d’équilibre stable sont obtenues à partir de l’énergie potentielle totale Ep = E p,1 + Ep,2, par d Ep / d u = 0 et d 2 Ep / d u2 > 0. On a donc : √ √ d Ep g 2 V 2c 2 2 = ma sin u g 2 − 2aV cos u = 0 pour u = np et cos ue = 2 en posant : Vc = V du 2a On a aussi : √ d2 E p 2 2 2 = mga 2 cos u − 2ma V (2 cos u − 1) 2 du d’où : √ d2 E p d2 E p g2 2 4 n 2 2 2 2 V V 2 (− 1 ) − 2ma et = 2ma 1 − = mga a V >0 d u 2 u=np d u 2 ue 2 L’équilibre u = np est instable si n est impair et stable si n est pair, pourvu que V < Vc . Quant l’équilibre u = ue , √ il est stable quand il existe, c’est-à-dire si V > Vc . L’angle ue = 60◦ correspond à un équilibre stable si V = Vc 2. S20– 6. Pendule elliptique 1. Appliquons le théorème du centre de masse à l’ensemble en projection suivant l’axe horizontal Ox. Comme toutes les forces extérieures sont verticales (poids et réaction sans frottement exercée par l’axe Ox), il vient, compte tenu des conditions initiales : m¨ xC = 0

d’où

˙xC = Cte = 0

et

xC = Cte = x0

2. La conservation de l’énergie de ce système conservatif donne Em = Ek + Ep = Cte avec :

mv2C (I − ml2 )u˙ 2 et Ep = mgyC = −mgl cos u + Cte = mgl(1 − cos u) + 2 2 en adoptant comme origine de Ep sa valeur pour u = 0. Notons que l’expression de Ek découle directement de l’application du théorème de Kœnig ; comme OC = x0 ex + (−l cos u) e y, on a :  c Dunod – Toute reproduction non autorisée est un délit

Ek =

v C = lu˙ sin u ey

Ek =

Iu˙ 2 u et − ml2 u˙ 2 cos2 2 2

I ˙u2 − ml 2 u˙ 2 cos2 u + mgl(1 − cos u) = Cte 2

3. La période des petits mouvements d’oscillation s’obtient en ne conservant que les termes quadratiques de l’équation du mouvement : u˙ 2 u2 (I − ml 2) + mgl = Cte 2 2 ce qui donne en dérivant : 1/ 2 mgl 2p ¨ u + v20 u = 0 avec v0 = = 2 I − ml T0 4. Les coordonnées du point B sont, puisque OB = OO1 + O1B :

x − x0 l−b Cette équation de trajectoire est celle d’une ellipse, d’où le nom du pendule. x = x0 − (l − b) sin u

et y = −b sin u d’où

2

+

y b

2

=1

732

Solutions des exercices

S20– 7. Mise en rotation d’un disque par la chute d’une masse 1. Si l’on désigne par f˙ la vitesse de rotation du disque et par z˙ la vitesse de chute de la masselotte A, les conditions de roulement sans glissement s’établissent aisément : lu˙ = bf˙ et bf˙ = − z˙. Comme ces équations sont intégrables, elles réduisent le nombre de degrés de liberté à 1. 2. Les liaisons étant parfaites, la conservation de l’énergie mécanique donne : E m = E k + Ep = Cte avec : Ek =

I1 u˙ 2 I2f˙ 2 m˙z2 l2 u˙2 + + = I 1 + 2 (I 2 + mr 2) 2 2 2 2 b

r Ep = mgzA + Cte = −mg lu b

et

en adoptant comme origine celle correspondant à u = 0. Il en résulte l’équation du mouvement : ˙u2 l2 rl 2 I 1 + 2 (I2 + mr ) − mg u = Cte 2 b b

soit en dérivant ¨ u=

mgrl/b I 1 + (l 2 /b2 )(I 2 + mr 2 )

Le mouvement du plateau est donc à accélération angulaire constante. S20– 8. Étude de la chute d’une barre par l’énergie La conservation de l’énergie de ce système conservatif donne Em = E k + Ep = Cte avec :

mv 2C ICa˙ 2 ml2 a˙ 2 ml 2a˙ 2 ml 2a˙ 2 l et Ep = mgyC + Cte = mg sin a + = + = 2 2 8 24 6 2 en adoptant comme origine des énergies potentielles celle correspondant à a = 0. On trouve donc : Ek =

3g 3g a˙ 2 + sin a = Cte soit en dérivant a ¨ = − cos a l 6 2l S20– 9. Cylindre sur un véhicule accéléré Appliquons le théorème de l’énergie dans le référentiel non galiléen R du véhicule. En raison du roulement sans glissement, du déplacement horizontal et du mouvement de translation de R, la seule force qui travaille est la force d’inertie d’entraînement : 2 ˙ 2 d m x˙ Iu mr2 2 2 2 = (−ma0 · x˙  ex ) avec I = et ˙x = r u˙  + dt 2 2 2 puisqu’il y a roulement sans glissement. Il en résulte : d dt

2 2 m ˙x m x˙ + 2 4

2

= −ma 0 ˙x 

d(3m ˙x /4) 2 = −ma0 x˙  et ¨x = − a0 dt 3

soit

en simplifiant. Le mouvement dans R du centre de masse du cylindre est donc uniformément accéléré. Comme le véhicule et le cylindre sont initialement au repos, le rapport des longueurs est dans le rapport des accélérations : L = 3 l/ 2 . S20– 10. Mouvement d’un secteur circulaire dans un référentiel tournant 1. Le centre de masse se trouve sur l’axe de symétrie à une distance de B telle que : R

MzC =

zdm

avec

rs d S =

M=

a

rs r d r d w 0

−a

en utilisant les coordonnées polaires dans le plan du secteur. Il vient, en effectuant les intégrations : M = rs

r2 2

R

w 0

a −a

= r sR 2a

733

et problèmes et zC =

1 r sR 2a

R

a

2

r sr cos w d r d w = 0

−a

1 R2a

R

r3 3

a

sin w

−a

0

=

2R sin a 3 a

2. Calculons le moment d’inertie I 3 par rapport à Bz1 : I3 =

y2 rs r d r d w = rs

R

r3 d r sin 2 w d w = rs 0

soit : I 3 = rs

R4 4

a −a

w−

sin(2w) = rs 2

R4 4

a−

a

r3 d r

−a

[1 − cos (2w)] d w R2 4

sin(2a) =M 2

1−

sin(2a) 2a

Le moment d’inertie par rapport à l’axe perpendiculaire Bx1 se calcule aisément car c’est celui d’un disque complet : I1 = MR2 /2. On en déduit alors I2 selon : I 2 = I1 − I 3 = M

sin(2a) R2 1+ 4 2a

L’énergie cinétique du système dans le référentiel tournant R = Ox yz lié à la tige vaut : E k = I1u˙ 2/2. 3. L’énergie potentielle de pesanteur est, si l’origine est prise pour u = p/2 : E p = mgzC + Cte = −

2mgR sin a cos u a 3

Dans R non galiléen, l’énergie potentielle centrifuge a pour expression : Ep,c = −V2 IOz . Le calcul de IOz est assez laborieux. Proposons une méthode directe : I Oz =

HA2r sr d r d w

avec

HA 2 = l2 + r2 sin2 (w + u)

puisque OA = OB + BA = lex  + r sin(w + u)ey − r cos(w + u)ez . On a donc : IOz =

2

soit : I Oz = Ml 2 + rs  c Dunod – Toute reproduction non autorisée est un délit

2

2

2

3

[l + r sin (w + u)]r s r d r d w = Ml + rs R4 4

r dr

2

sin (w + u) d w

w sin 2(w + u) R2 MR 2 − − = Ml 2 + M [2 sin(2a) cos(2u)] 2 4 4 16a

Ainsi, Ep,c peut se mettre sous la forme : E p,c = −

V2 2

Ml2 + M

R2 MR2 − 4 4

sin(2a) 2 (1 − 2 sin u) 2a

=−

V2 Ml2 + I 3 + (I2 − I3) sin 2 u 2

4. La période des petits mouvements est obtenue en faisant u petit dans l’équation de conservation de l’énergie mécanique dans R : Em = E k + Ep = Cte soit

2mgR sin a I 1u˙ 2 − a 2 3

1−

u2 2



V2 Ml2 + I3 + (I2 − I 3)u2 = Cte 2

En dérivant par rapport au temps et en simplifiant, on trouve : 4g sin a ¨ u+ u − V2 3R a

I 2 − I3 I1

u=0

734

Solutions des exercices

¨ = 0 soit 4. a) Cas d’un disque (a = p et I2 = I 3) : il n’y a pas d’oscillation, ce qui était prévisible : u u˙ = Cte. b) Cas d’un demi-disque (a = p/2 et I2 = I 3) : le terme centrifuge n’a pas d’influence ; la période d’oscillation est déterminée par la seule pesanteur : 8g ¨ u+ u=0 3pR

d’où

T0 = 2p

3pR 8g

1/ 2

c) Cas d’un quart de disque (a = p/4) : le terme centrifuge a une influence : √ V2 8g 2 ¨ u=0 − u+ p 3pR Si la vitesse angulaire V est suffisamment faible, on observera une oscillation autour de u = 0. Dans le cas contraire, il n’y a pas d’oscillations. S20– 11. Bille sphérique creuse dans un guide diédrique en rotation 1. Dans R les conditions de roulement sans glissement de la bille sur les parois du rail s’écrivent :

vC + V × CI = 0 et vC + V × CJ = 0 d’où V × (CI − CJ) = V × JI = 0 √ ˙ Le moment Il en résulte que V et JI sont parallèles. On en déduit : vC = V × IC d’où vC = V r/ 2 = Ru. 2 d’inertie de la bille creuse par rapport à un axe passant par son centre est : ICz = 2mr /3 (cf. chapitre 17). Son énergie cinétique est donc : Ek =

2 2 1 1 1 mr 2V 1 2mr2 V 7 7 2 2 + = mv C + ICz V2 = mr2 V = mR2u˙ 2 2 2 2 2 2 3 12 6

2. Dans R  , l’énergie potentielle est la somme de l’énergie potentielle de pesanteur et de l’énergie potentielle centrifuge qui s’écrivent, à une constante additive près : V2r V2r 2 2 I Oz = − (ICz + mR sin u) 2 2 On en déduit l’énergie mécanique Em dans R. Cette dernière se conserve car, dans R , la puissance des actions de contact est nulle ; en effet, en raison du roulement sans glissement la vitesse des points de la bille en contact avec le guide est nulle : E p,g = mgyC = mgR(1 − cos u) et

E m =

E p,c = −

V2r 7 2˙ 2 mR u + mgR(1 − cos u) − 6 2

mR2 sin2 u +

2mr2 3

= Cte

3. La discussion du mouvement peut être menée à partir du graphe de la fonction énergie potentielle : E p(u) = mgR(1 − cos u) −

mV2r 2

R2 sin2 u +

2r2 3

Les positions d’équilibre sont celles qui réalisent un extrémum de Ep (u) : d Ep 2 = mR sin u(g − V r R cos u) = 0 du

pour

sin u = 0

et

cos u =

g RV 2r

La stabilité des positions d’équilibre, u = 0, u = p et u3 = arccos(g/RV2r ) est étudiée à partir de la dérivée seconde : d 2 Ep 2 2 2 2 = mR cos u(g − Vr R cos u) + mR Vr sin u du2 d’où : d2 Ep d u2

0

= mR(g − V 2r R)

d2 Ep d u2

g/R.

735

et problèmes

4. Dans R, la vitesse de rotation angulaire de la bille est : V = V + V r , d’où vC = (V  + V r ) × IC. On en déduit l’énergie cinétique Ek = mv2C /2 + ICz V2 /2 : Ek =

m 2

r2 V 2 + R 2 V2r sin2 u 2

+

mr 2 V2r 1 2mr 2  2 7 (mR 2 sin2 u) + (V + V2r ) = mR2 u˙ 2 + 2 3 6 2 3

L’énergie potentielle de pesanteur est : Ep = mgR(1 − cos u) si on adopte comme origine des énergies potentielles celle correspondant à u = 0. On en déduit l’énergie mécanique dans R : Em =

V2r 7 2˙2 mR u + 6 2

2 mR2 sin2 u + mr2 3

+ mgR(1 − cos u)

Cette énergie n’est pas constante car la puissance des actions de contact n’est pas nulle dans le référentiel du laboratoire R ; dans le langage de la mécanique analytique, la bille a un mouvement guidé non naturel (cf. chapitre 24). S20– 12. Machine d’Atwood dansante 1. Machine d’Atwood historique a) La condition de roulement sans glissement d’une poulie sur le fil s’exprime en écrivant que les vitesses des points N et K appartenant respectivement à la poulie et au fil sont égales. Si w désigne l’angle de rotation de la poulie orienté de A ver B , et z la coordonnée selon la verticale, il vient : vN = Rw˙ et

v K = z˙ e t d’où R w˙ = ˙z

En intégrant, il vient z = Rw + Cte . A priori, le nombre de degré de liberté du système est la coordonnée z de B , l’angle w de rotation de la poulie et la coordonnée verticale de A , zA . Or ces trois variables sont reliés par deux équations, la condition précédente de roulement sans glissement et l’inextensibilité du fil : L = Cte = zA + pR + D + pR + z

d’où zA = −z + Cte

L étant la longueur du fil. Le nombre de degrés de liberté du système, c’est-à-dire le nombre de paramètres indépendants qui permettent de connaître la position du système, est donc 1 . b) L’énergie potentielle de pesanteur, a pour expression : Ep = −mg · OA − Mg · OB + Cte = −mg(−z + Cte) − Mgz + Cte = −(M − m)gz + Cte

 c Dunod – Toute reproduction non autorisée est un délit

Quant à l’énergie cinétique, on l’obtient selon : Ek =

1 1 1 1 1 1 2I p mv 2 + Mv 2B + 2 × Ip V 2 = m˙z2 + M˙z2 + Ip w˙ 2 = ˙z2 m + M + 2 2 A 2 2 2 2 2 R

c) L’expression « la liaison entre les poulies et leurs axes est parfaite » signifie que la puissance des actions de contact des axes sur les poulies est nulle. Quant à la puissance des forces de frottement de contact qu’exercent les poulies sur le fil, elle est nulle en raison du roulement sans glissement du fil sur les poulies. Pour trouver la nature du mouvement, il suffit d’appliquer le théorème de l’énergie, sachant que les forces non conservatives ne travaillent pas : E = E k + Ep =

1 2 2Ip ˙z m + M + 2 2 R

− (M − m)gz + Cte = Cte

ce qui donne, en dérivant par rapport au temps : ˙z¨z m + M +

2I p − (M − m)g z˙ = 0 R2

Ainsi, le mouvement est uniformément accéléré.

soit

¨ z=

M−m g m + M + 2Ip /R 2

736

Solutions des exercices

d) L’intérêt pédagogique de cette machine est d’obtenir un mouvement uniformément accéléré dans le champ de pesanteur, dont l’accélération soit différente de g . e) Pour m = 120 g , M = 360 g , Ip = 7 × 10 −6 kg.m2 et R = 2, 5 cm , on trouve, sachant que le champ de pesanteur vaut g = 9, 8043 m.s−2 : ¨z =

0, 36 − 0, 12 × 9, 8043 = 4, 6835 m.s −2 0, 12 + 0, 36 + 2 × 1, 12 × 10−2

2. Machine d’Atwood dansante ou SAM (Swinging Atwood Machine) Comme précédemment, la masselotte B a un mouvement rectiligne vertical, mais A peut aussi osciller. a) Avec deux poulies, la masselotte A peut osciller sans pertubation. b) Avec les notations précédentes, la condition de roulement sans glissement du fil sur les poulies est inchangée : Rw˙ = z˙ soit z = Rw + Cte. L’angle u d’écartement du pendule A par rapport à la verticale est un autre degré de liberté. Exprimons l’inextensibilité du fil, de longueur L : p pR −u +D+ + z d’où r − Ru + z = Cte 2 2 Ainsi, on compte a priori quatre variables, z , w , r et u , reliées par deux équations. Le nombre de degré de liberté du système est donc deux, par exemple r et u . c) L’énergie potentielle s’écrit comme précédemment, à une constante additive près : L = Cte = r + R

E p = −mg · OA − Mg · OB avec

OB = z ez et OA = OQ + QA

Comme OQ = −R cos u ey − R sin u ez et QA = −r sin u ey + r cos u ez , il vient :

Ep = mg(R sin u − r cos u) − Mgz = mg(R sin u − r cos u) + Mg(r − Ru)

Quant à l’énergie cinétique, on l’obtient selon : Ek = et : On trouve :

1 2 1 1 mv A + Mv2B + Ip w˙ 2 2 2 2

avec

2 2 ˙ 2 vB = z˙ = (r˙ − Ru)

w˙ 2 =

˙z R

2

=

˙ 2 (r˙ − Ru) 2 R

v2A = (Ru˙ sin u − r u˙ cos u − ˙r sin u)2 + (−R u˙ cos u − r u˙ sin u + ˙r cos u)2

1 1 2Ip Mt (Ru˙ − ˙r) 2 + mr2 ˙u2 avec Mt = M + m + 2 R 2 2 d) La conservation de l’énergie mécanique de ce système conservatif donne : Ek =

2 ˙2 1 ˙ 2 + mr u + mg(R sin u − r cos u) + Mg(r − Ru) = Cte Mt (r˙ − R u) 2 2

e) L’application du théorème du moment cinétique à la masselotte A , au point mobile I de contact du fil avec la poulie 1, permet d’éliminer la réaction inconnue qu’exerce la poulie sur le fil. Comme I est mobile, il vient : d LI + vI × mvA = IA × mg avec dt Il en résulte, puisque IA × mg = −mgr sin u ex :

LI = IA × mvA = mr 2u˙ e x

˙ d(mr 2 u) − mRr ˙r ˙u = −mgr sin u dt

et v I = Ru e t

soit r¨ u + 2˙r u˙ − R r˙ u˙ = −g sin u

f) On retrouve l’équation du mouvement de la machine d’Atwood historique, en faisant dans les équations précédentes u = 0 et u˙ = 0 . La seconde équation se réduit à 0 = 0 . Quant à la première, elle donne : 1 Mt ˙r2 + (M − m)gr = Cte avec r + z = Cte et 2

Mt = M + m

737

et problèmes S20– 13. Pendule pesant inversé sur un chariot en mouvement 1. Le théorème du centre de masse C appliqué au seul véhicule donne : d P v = Mg − R + R + Fp ce qui donne, en projetant selon des deux axes Ox et Oz , en tenant compte de X = 0 : M

¨ 0 0 x X Fp − +  + =M ¨ z −g Z Z 0

On en déduit : et

M¨x = −X + Fp

− Z + Z  = Mg

2. Le théorème du centre de masse appliqué au pendule donne : d p/ d t = mg + R d’où, en projetant selon des deux axes Ox et Oz : 0 ¨ x X + m ¨ =m z −g Z Comme OA = OO + O A , il vient : OA = On en déduit :

x + l sin u = l cos u

d’où p =

m ˙x + lu˙ cos u −lu˙ sin u

u cos u − lu˙ 2 sin u 0 mx¨ + l¨ X + =m 2 ¨ ˙ −g Z −lu sin u − lu sin u

Ainsi, on trouve les deux équations différentielles suivantes : 2 u cos u − ml u˙ sin u = X m¨ x + ml¨

et

2

− ml( ¨ u sin u + u˙ sin u) = −mg + Z

3. Appliquons le théorème du moment cinétique au pendule au point mobile O de R : d LO  + mvO × v A/R = O A × m g dt le moment des actions de contact en O étant perpendiculaire à l’axe de rotation, puisque la liaison est supposée parfaite. En efectuant les produits vectoriels de l’équation vectorielle précédente, on obtient : O A × m g = mgl sin u ey  c Dunod – Toute reproduction non autorisée est un délit



˙ x × (x˙ + lu˙ cos u) e x − l ˙u sin u ez = ml˙xu˙ sin u e y et m vO × vA/R = mxe

Quant au moment cinétique en O , il a pour expression : L O = OA × p = (l sin u e x + l cos u e z) × [(mx˙ + lu˙ cos u) ex + −lu˙ sin u e z] = (ml2 u˙ + ml˙x cos u) ey Il en résulte, en dérivant et en simplifiant : l¨ u + ¨x cos u = g sin u. Dans le cas des petits mouvements ( cos u ≈ 1 et sin u ≈ u), l’équation précédente se réduit à : l¨ u +¨ x = gu. 4. D’après ce qui précède, on a, dans le cas des petits mouvements, les trois équations suivantes u+ ¨ l¨ x = gu

M¨ x = −X + Fp

u=X et m¨x + ml¨

ce qui donne, en ajoutant ces deux dernières équations : u + ¨x = gu l¨

et (M + m)¨ x + ml¨ u = Fp

On en déduit, en introduisant v20 = g/l : M¨x = Fp − mv20 lu et

Ml¨ u = −F p + (m + M )v20lu

738

Solutions des exercices 5. Les résultats précédents peuvent être condensés selon : 0 q˙ 1 0 p˙ 1 = q˙ 2 0 0 p˙ 2

1/M 0 0 0

0 −mv20 0 (m + M )v20

0 0 1/M 0

0 q1 1 p1 + Fp q2 0 −1 p2

Pour l = 0, 4 m , v0 = 4, 95 rad.s−1 .

Chapitre 21 S21– 1. Pendule balistique 1. Comme la liaison pivot en O est parfaite, le moment cinétique suivant l’axe de rotation se conserve : (OK × mv + 0)z = (I V + m OK × vK )z Il en résulte que : V=

soit mv h = I V + mOK2 V

mv h mv h = I + mOK2 I + mh 2 / cos2 a

2. La quantité de mouvement du système a pour expression P = M vC + mvK = Ml V e x + mOKV e t , d’où les composantes : P x = Ml V + mOKV cos a = (Ml + mh)V

et

Py = −mOKV sin a = −m Vh tan a

3. En appliquant le théorème du centre de masse, on trouve l’expression de la variation de la quantité de mouvement : DPt = P − mv

qui s’explicite selon R

(Ml + mh) V − mv −mv h tan a

La réaction est nulle si (Ml + mh)V − mv = 0 et tan a = 0, DPt = 0. On en déduit que : a=0

et (Ml + mh)V = mv = I +

mh2 cos2 a

V = I + mh2 h

V h

soit I = Ml h

L’énergie cinétique de l’ensemble a alors pour expression : Ek =

1 2 1 1 1 1 m 2 v2 h2 mh2 1 2 IV + mv K2 = IV2 + mOK 2 V2 = I + mh 2 V 2 = mv = 2 2 2 2 2 2 2(I + mh ) I + mh2 2

La conservation de l’énergie mécanique dans le mouvement ultérieur donne : DEk + DEp = 0

soit

0 − E k − Mgl(cos um − 1) − mgh(cos um − 1) = 0

Par conséquent : mh2 mv 2 = (Ml + mh) g(1 − cos um ) et I + mh2 2

g v =2 h

1/ 2

um sin 2

I + mh 2 mh

1/ 2

739

et problèmes S21– 2. Coup horizontal sur une bille de billard 1. Les théorèmes généraux de la mécanique des chocs donnent : DP = P  − 0 = Pr

et DL C = L  − 0 = CH × Pr

si Pr désigne la quantité de mouvement reçue par l’intermédiaire de la queue de billard. Il en résulte que : mvC = P

2 2  r mr V = CH × P 5

r

d’où V  = −

5(h − r) vC ez r 2r

2. Le mouvement ultérieur satisfait aux équations : 2mr 2 d V d vC = R + mg et = CI × R dt 5 dt où R est la réaction exercée par le tapis sur la boule. Comme le contact est permanent, le mouvement de C est horizontal. D’où, puisque R = Rn + Rt : m

ma C = R t et

2mr2 d V = CI × R t 5 dt

Plusieurs cas doivent être considérés suivant la valeur de la vitesse de glissement vg : vg < 0, vg = 0 et vg > 0 (cf. chapitre 19). Cette vitesse a pour expression : vg = v 2 − v1 = v 2 = vC + IC × V , si v2 est la vitesse du point de la boule qui coïncide avec le point géométrique. Ainsi :   vg = vC −

5(h − r) 5h −r IC × ez = 1 − 2r2 2 r

 vC =

7r − 5h  vC 2r

i) h > 7r/5, vg < 0 ; donc, d’après les lois de Coulomb, Rt > 0 et R t = mRn = mmg. La boule glisse d’abord avec une accélération, puis roule sans glisser. ii) h = 7r/5, v g = 0 ; la boule roule sans glisser. iii) h < 7R/ 5, v g > 0 ; donc, Rt < 0 et Rt = −mR n = −mmg. La boule glisse d’abord avec une accélération négative, puis roule sans glisser. S21– 3. Balle superélastique 1. Appliquons les théorèmes généraux de la mécanique des chocs à la balle : 2mr2 r (V − V) = CI × P 5 On a donc trois équations, deux pour la quantité de mouvement dans le plan Oxy et une pour le moment cinétique suivant l’axe des z : 

 c Dunod – Toute reproduction non autorisée est un délit

DP = m(v − v) = P

r

et DL C =

2mr 2  (V − V) = r Prx 5 Le nombre d’inconnues est cinq (deux pour v, une pour V et deux pour P r). Deux équations supplémentaires sont donc nécessaires. m(vx − v x ) = P rx

m(v y − vy ) = Pry

et

2. Comme DEk ≈ 0 :

1 2 1 2mr 2 2 1 1 2mr2 2 2 V ≈ mv  + V mv + 2 2 5 2 2 5 En outre, vy = −vy puisque la valeur absolue de la composante normale de la vitesse se conserve. On en déduit : 2 2 2 2 2 2 2 2 v − v = v x − vx = − r (V  − V ) soit encore 5

Il en résulte que : vx + vx = −r(V  + V), ce qui donne : 2v x = −r(V  + V) −

2 2 (vx − vx )(vx + v x ) = − r (V − V)(V  + V) 5

2r(V − V) r = (−7V − 3V) 5 5

740

Solutions des exercices

Par conséquent :  V =−

5(2vx − 3rV/5) 3 10 vx = − V− 7r 7 7 r

3. Si V = 0 et v x > 0, alors V = −

et v x =

3 4 vx − rV 7 7

10 vx 3 et v x = − vx . 7 r 7

S21– 4. Collision inélastique parfaite de deux disques en rotation 1. Le moment cinétique de l’ensemble en projection sur l’axe de rotation se conserve car les frottements ne concernent que les efforts intérieurs : I1 V1 + I2 V2 = (I1 + I2) V

d’où V =

I1V 1 + I 2V2 I1 + I 2

1 2 1 I1 I2 (V 1 − V2 )2 . IV − (I1 V21 + I 2 V22 ) = − 2 2 2(I 1 + I2 ) Ce résultat est analogue à celui d’une collision inélastique entre deux particules : les moments d’inertie remplacent les masses et les vitesses angulaires remplacent les vitesses. 2. DEk =

S21– 5. Chute protégée par une corde 1. La percussion, entre l’instant t i où la hauteur de chute est h et l’instant tf où cette hauteur de chute est h + x , a pour expression : (r)

P

= DP = m(vf − vi ) = −vh

puisque vf = 0 et vi = vh

avec vh tel que, d’après la conservation de l’énergie mécanique : mv2h = mgh soit v h = (2gh)1/2 ez 2 e z définissant la verticale descendante. Il en résulte : P

(r)

= −m(2gh) 1/2 ez

avec

m(2gh)1/2 = 75 × (2 × 9, 81 × 50) 1/2 = 2, 35 × 103 kg.m.s−1

2. Il vient, d’après la conservation de l’énergie mécanique, entre les deux valeurs de la coordonnée verticale ascendante, z = 0 et z = h + zm : d’où 0 = 0 − mg(h + z m) +

(Ek + Ep,g + Ep,e )0 = (Ek + Ep,g + Ep,e )h+zm

1 Kz2m 2

Il en résulte l’équation du deuxième degré en zm : 2

z m − 2l0 zm − 2l 0 h = 0

avec

l0 =

mg K

La solution acceptable est la suivante : 2

1/ 2

zm = l0 + (l0 + 2l 0h)

= l0 1 + 1 +

2h l0

1/ 2

=

2hK mg 1+ 1+ K mg

1/ 2

b) La force maximale vaut : Fm = Kz m = mg 1 + 1 + On en déduit la raideur K selon : mg (r − 1)2 − 1 K= 2h

2hK mg

1/ 2

qui vaut K =

d’où r =

2hK Fm = 1+ 1+ mg mg

75 × 9, 81 × 195 ≈ 1435 N.m−1 2 × 50

1/ 2

pour r = 15

741

et problèmes 3. a) Il vient, en introduisant E , s et le facteur de chute fc = h/l , dans l’expression de F m : Fm = mg 1 + 1 +

2Esh mgl

1/ 2

= mg 1 + 1 +

2Es fc mg

1/ 2

b) On obtient E selon :

1 435 × 100 Kl ≈ 1, 82 GPa = (3, 14/4) × 10 −4 s c) On obtient une estimation de la durée du choc, en considérant l’équation différentielle du mouvement d’allongement de la corde sous l’action de la charge ; on a, en effet, selon l’axe vertical ascendant, dans le voisinage de la la position d’équilibre : E=

K 1/ 2 = 4, 37 rad.s−1 m dont la solution est une sinusoïde de période T = 2p/v0 . La durée du choc vaut donc : d’où ¨ z + v 20 z = 0

m ¨z = −Kz

t=

T ≈ 0, 36 s 4

d’où F =

avec

v0 =

2, 35 × 103 P(r) ≈ 6, 5 kN ≈ t 0, 36

et Fm = r mg = 15 × 75 × 9, 81 ≈ 11 kN

S21– 6. Pendule balistique industriel 1. a) L’énergie potentielle de pesanteur E p de l’ensemble, chariot et double portique, a pour expression, à une constante additive près sans intérêt : l E p = −Mc gl cos a − 2Mp g cos a = −(M cl + M pl)g cos a 2 b) Quant à l’énergie cinétique de l’ensemble, elle s’écrit : 1 1 Mp l2 2 2 Ek = E k,c + E k,p = M c(la) a˙ = ˙ + ×2 2 2 3

M cl2 Mp l2 + 2 3

2



la vitesse du chariot étant égale à celle la˙ des extrémités du portique, en raison de son mouvement de translation. 2. a) L’équation reliant l’angle initial d’inclinaison ai à la vitesse vf du chariot à l’instant de l’impact s’obtient à partir de la conservation de l’énergie mécanique, laquelle est réalisée car les liaisons sont supposées parfaites et les seules forces qui travaillent sont conservatives : (Ek + Ep) i = (Ek + Ep )f

 c Dunod – Toute reproduction non autorisée est un délit

car Ek,i = 0. Il vient, sachant que a f = 0 :

soit Ep,i = Ek,f + Ep,f

Mcl2 Mp l2 + 2 3

−(Mc l + Mp l)g cos ai =

a˙ 2f − (Mc l + Mp l)g

On en déduit, puisque vf2 = l2a˙ 2f : ai sin = 2

vf2 2gl

1/ 2

M c/2 + M p/3 Mc + Mp

On trouve, en effectuant pour vf = 20 km.h−1 = 5, 55 m.s−1 : ai sin = 0, 67 × 0, 655 = 0, 438 2

1/ 2

d’où a i ≈ 52◦

b) Si toute la masse du pendule avait été concentrée au centre de masse du chariot, on aurait obtenu le résultat habituel pour un pendule simple après simplification de la masse : sin

ai 2

=

v2f 2gl

1/ 2

≈ 0, 67

d’où a i ≈ 84, 1◦

742

Solutions des exercices

Chapitre 22 S22– 1. Phase balistique du lancement d’une fusée 1. La fusée est un système ouvert dont l’équation du mouvement est (cf. chapitre 22) : q m u = M (t)g + R − M (t) a

soit, puisque R ≈ 0,

a=g−

q mu M (t)

Or M (t ) = 2M − qmt = 2M (1 − t/T ) avec qm = 2M /T et u = 2gT. Par conséquent, en projetant suivant la verticale ascendante, on obtient : ¨ z = −g +

2g 1 − t/ T

pour 0 < t
0 et yB > 0 car ces composantes sont opposées au glissement naissant vers le bas. On en déduit que : x + r = (h + 2r ms )X B/P. En outre : P = Y A + YB = −ms XA + ms XB = 2ms X B

d’où x + r =

h + 2r m s 2ms

soit x =

h 2ms

S23– 5. Homme sur une échelle double Traduisons l’équilibre du système en annulant le torseur des forces qui s’exercent sur l’ensemble en O, ainsi que le torseur des forces qui s’exercent sur la tige OB en O. On a pour l’ensemble : R A + RB + 2Mg + mg = 0 et OA × RA + OB × R B + OH × mg = 0

et pour OB :

R B + Ra→b + Mg = 0 et

OC2 × Mg + OB × R B = 0

En projetant les deux premières équations vectorielles dans la base (ex , e y , ez), on obtient : XA + XB = 0, Y A + Y B− (2M + m)g = 0 et : −l sin a −l cos a × 0

XA YA 0

+

l sin a −l cos a × 0

XB YB + 0

−x sin a −x cos a × 0

0 −mg = 0

0 0 0

Il en résulte que :

 c Dunod – Toute reproduction non autorisée est un délit

−lYA sin a + lX A cos a + lY B sin a + lX B cos a + mgx sin a = 0 puisque XA + X B = 0. On en déduit que : g gx g x 1− YB = (2M + m) − m = Mg + m 2 2l 2 l Annulons le moment des actions qui s’exercent en O sur OB : l sin a/2 −l cos a/2 × 0

0 −Mg + 0

En posant u = m(1 − x/l)/M , on obtient : X A = −XB = tan a −

Mg + YB 2

=

et YA = (2M + m)g − YB

l sin a −l cos a × 0

Mg tan a 2

1+

soit l(YB − YA) + mgx = 0

m M

0 0 0

XB YB = 0

1−

x l

=

Mg tan a (1 + u) 2

et

mg x u 1− = Mg 1 + l 2 2 La discussion est facilitée à l’aide d’un plan cartésien dans lequel l’abscisse est u et l’ordonnée les forces en unité Mg (Fig. S23.1). Dans le cas où m = M , on a, en coordonnées réduites : YB = Mg +

xA =

tan a XA (1 + u) = −x B = 2 Mg

yB =

YB u = 1+ 2 Mg

et yA =

YA u = 3 − yB = 2 − 2 Mg

746

Solutions des exercices

Pour que l’équilibre soit possible, il faut que la droite |xA/ms| coupe les droites représentant yA et yB dans l’intervalle [0, 1] pour u. Plusieurs cas se présentent : (1)

tan a/(2ms ) > 2 soit tan a > 4m s :

pas d’équilibre possible

(2)

1 < tan a/(2ms) < 2 soit 2ms < tan a < 4ms :

les points A et B glissent

(3)

3/4 < tan a/(2ms) < 1 soit 3ms /2 < tan a < 2m s : le point B glisse avant A

(4)

0 < tan a/(2ms) < 3/4 ou 0 < tan a < 3ms /2 :

pas de glissement.

(1) 3

| xA |

(2)

m yA

2

(3)

yB 1

−1

(4) 0

1

u

F IG . S23.1. S23– 6. Stabilité de l’équilibre d’un disque L’énergie potentielle du système est, à une constante additive près : 1 K (z − l0 )2 + mgl cos u avec z − l0 = ru 2 puisque le fil ne glisse pas sur la poulie. Donc : Ep = 12 Kr2u 2 + mgl cos u. On en déduit : Ep = E pe + Epg =

d Ep = Kr2 u − mgl sin u = 0 du

si

Kr 2 sin u d 2 Ep sin u et − cos u = = Kr2 − mgl cos u = mgl mgl u u d u2

L’équilibre est stable si d2 E p/ d u 2 > 0 soit si tan u > u. S23– 7. Équilibre d’un pendule double pesant (méthode des travaux virtuels) Le travail virtuel de l’ensemble des forces qui s’exercent sur le système doit être nul : dv W = mg dv x 1 + mg d v x2 + F d vy = 0. Comme : x 1 = l2 cos u1 , x 2 = l cos u 1 + l/2 cos u 2 et y = l sin u1 + l sin u 2, il vient : l dv x1 = − sin u 1 dvu 1 2

d vx 2 = −l sin u 1 d vu 1 −

l sin u2 d v u2 2

et

dv y = l cos u1 dv u1 + l cos u2 d v u2

Par conséquent : 3mgl mgl sin u1 + Fl cos u1 dv u1 + Fl cos u2 − sin u 2 d v u2 = 0 2 2 On en déduit : tan u1 = 2F/(3mg) et tan u2 = 2F/(mg), d’où u2 > u1 . d vW = −

S23– 8. Équilibre de divers solides géométriques 1. À l’équilibre, le centre de masse se trouve sur la verticale passant par le point de suspension. L’angle a est donc tel que : 2 4 1 3 a soit tan a 1 = tan a2 = tan a3 = et tan a4 = R p 3p 2 8 ◦ ◦ ◦ ◦ On en déduit les valeurs suivantes a1 ≈ 32, 5 , a 2 ≈ 23 , a 3 ≈ 26, 6 et a4 ≈ 20, 55 ; R n’a aucune influence. tan a =

747

et problèmes

2. Pour déterminer la période des petites oscillations, écrivons l’équation du mouvement en appliquant par exemple le théorème du moment cinétique au point de suspension O , en projection selon l’axe de rotation horizontal Oz . Il vient : IOz w ¨ ez = OC × mg = −mglc sin w e z où IOz est le moment d’inertie du solide par rapport à Oz , w l’angle que fait OC avec la verticale descendante et lc la distance OC . L’équation des petites oscillations est donc : w ¨ + v20 w ≈ 0

avec

v0 =

mgl c IOz

1/ 2

Or, d’une part lc = R/ cos a, d’autre part IOz a même expression que pour les figures géométriques génératrices : IOz = mMR2 avec, respectivement : m1 = 1, m2 = 1/2, m3 = 2/3, m4 = 2/5. Ainsi : 1/ 2 1/ 2 2p Rm cos a g v0 = d’où T0 = = 2p v0 mR cos a g Le résultat dépend donc de R , contrairement à la position d’équilibre. S23– 9. Équilibre d’un solide en contact avec un plan horizontal 1. L’équilibre est stable si le solide oscille autour de la position d’équilibre, lorsqu’on l’écarte légèrement de cette position, théoriquement si la dérivée seconde de son énergie potentielle de pesanteur par rapport à l’écart angulaire u est positive : d2 Ep d u2

>0 e

avec

Ep = −mg · IC = −mg · (IK + KC)

K étant le centre de courbure de la demi-sphère de contact. Si on désigne par z la mesure algébrique KC , il vient : d2 Ep = (−mgz cos u)e = −mgz > 0 Ep = mg(R + z cos u) d’où d u2 e Ainsi la condition de stabilité est z = zC − R < 0 soit z C < R .

2. Dans le cas considéré, exprimons z en fonction de R et h . Il vient, en désignant par s la masse surfacique uniforme du solide, le centre de masse de la demi-sphère se trouvant au milieu du rayon : z=

h 2 − R2 h−R s2pRh × h/2 + s2pR 2 × (−R/2) = = 2 s2pRh + s2pR 2(h + R) 2

 c Dunod – Toute reproduction non autorisée est un délit

Il en résulte la condition suivante d’équilibre stable : (h − R)/2 < 0 soit h < R ce qui était prévisible puisque les centres de masse des portions sont situés au milieu de ces portions. S23– 10. Équilibre d’une masselotte et point de Fermat d’un triangle 1. Pour établir la condition d’équilibre, écrivons que la somme des tensions de fil qui s’exercent sur la masselotte A est nulle : T1 + T 2 + T3 = 0 soit T1 e1 + T2 e 2 + T3 e3 = 0 en introduisant les vecteurs unitaires définis par vv1 , vv2 et vv 3 respectivement. Ces tensions sont directement reliées aux poids, puisque les trois masses étant en équilibre, on a, la tension ne variant pas le long des fils mi g − T i = 0 . Il en résulte : m 1 g e 1 + m 2g e 2 + m 3g e 3 = 0 d’où m1 e 1 + m 2 e 2 + m3 e 3 = 0 a) Si la masse m 3 est très inférieure au deux autres masses, il vient m 1 e1 + m2 e 2 ≈ 0 . Le point A se trouve pratiquement sur la droite A1 A2 , précisément au centre de masse de A1 et A 2 . b) Si la masse m 3 est très supérieure aux deux autres masses, alors l’approximation m3 e3 ≈ 0 n’a pas de sens physique. On lève la difficulté en introduisant dans le bilan des forces la force supplémentaire de réaction en raison du contact de A avec A3 .

748

Solutions des exercices 2. Puisque m 1 = m2 = m 3 , le point de Fermat est tel que : e1 + e2 + e3 = 0.

En A , les trois vecteurs unitaires forment entre eux des angles égaux à 2p/3 . b) Pour que A existe, il faut que de A3 on puisse couper, par des droites, les directions des vecteurs e 1 et e2 , ce qui suppose que l’angle Aˆ3 soit inférieur à 2p/3 . 3. a) L’énergie potentielle d’un tel système, en fonction des cotes zi = A i Mi des points Mi , a pour expression : Ep = −mg(z1 + z2 + z3 ) = −mg zi i

b) Le minimum de E p , qui réalise l’équilibre, est aussi celui pour lequel la somme des cotes z est i i maximale. Comme on a, entre les cotes zi et les longueurs des fils li = AMi , la relation li = AAi + zi , il vient : zi = i

i

li −

AAi i

Ainsi ce maximum de la somme zi entraîne un minimum de i AAi . Le point de Fermat d’un triangle est donc le point tel que la somme des distances à ces trois points soit minimale. 4. On sait qu’un quadrilatère admet un cercle circonscrit si la somme des angles en deux sommets opposés est égale à p . C’est le cas pour A1B2A3 A puisque ˆ A = 2p/3 . Comme ce résultat est valable pour les B2 = p/3 et ˆ deux autres côtés, A est le point d’intersection des trois cercles circonscrits.

Chapitre 24 S24– 1. Pendule elliptique constitué de deux masselottes 1. L’énergie cinétique a pour expression : Ek =

l2u˙ 2 1 1 1 ml2 ˙ 2 1 2 1 2 m1 v 21 + m2 v22 + u + mv C = M1x˙ + M2lx˙ u˙ cos u + M3 2 2 2 12 2 2 2

avec M1 = m 1 + m2 + m, M 2 = m2 + m/2 et M 3 = m 2 + m/3. Quant à l’énergie potentielle, elle vaut, à une constante additive près : Ep = −M2 gl cos u. On en déduit les moments conjugués à l’aide du lagrangien L = Ek − E p : ∂L ∂L px = = M1 ˙x + M 2 lu˙ cos u et pu = = M2 l˙x cos u + M3l 2 u˙ ˙ ∂ x˙ u ∂ 2. Comme le lagrangien ne dépend pas de x, on a, d’après les équations de Lagrange : p˙ x =

∂L =0 ∂x

d’où

p x = M1 x˙ + M2 l u˙ cos u = Cte = 0

En outre, comme le temps n’intervient pas explicitement, l’hamiltonien est une constante égale à l’énergie mécanique : Em = Ek + Ep = Cte. Il en résulte que : M 3l 2u˙ 2 1 M 1x˙ 2 + M 2lx˙ u˙ cos u + − M 2gl cos u = Em 2 2 On en déduit, en tenant compte de la première équation du mouvement : M3 −

M22 cos 2 u M1

l2 u˙ 2 − M 2gl cos u = Em 2

749

et problèmes 3. Dans le cas des petits angles, l’équation précédente donne : M3 −

M22 M1

l2 ˙u2 2 + M 2 glu = Cte soit 2

T0 =

2p ≈ 2p v0

2 ¨ u + v0 u ≈ 0

avec

2

v0 ≈

g l

Par conséquent : l g

1/ 2

M2 M3 − M22 /M 1

(m 1 + m2 + m)(m2 + m/3) − (m 2 + m/2)2 (m 1 + m2 + m)(m 2 + m/2)

1/ 2

S24– 2. Pendule simple excité par déplacement horizontal 1. Afin d’établir l’expression du lagrangien, exprimons les coordonnées du point A. Comme OA =1 vv + O1A, on a :

R

x1 + l sin u −l cos u 0

d’où en dérivant R

˙x 1 + l ˙u cos u l u˙ sin u 0

les composantes de la vitesse de A. On en déduit les énergies cinétique et potentielle : 1 1 E k/R = mv 2 = m(˙x12 + l 2 u˙ 2 + 2x˙ 1 l u˙ cos u) 2 2 à une constante additive près. Le moment conjugué est donc :

et E p = −mgl cos u

∂L = ml 2u˙ + mx˙ 1 l cos u = ml2 ˙u − mlv dm sin(vt) cos u ˙ ∂u Par conséquent, l’équation de Lagrange correspondante est ˙pu = ∂L/∂u soit : pu =

ce qui donne :

2 ml ¨ u − mlvdm[v cos(vt ) cos u − u˙ sin(vt) sin u] = −ml sin u − mx˙ 1 l ˙u sin u

˙ m 2vud x˙ 1 u˙ dm sin(vt) + sin u = v2 cos u cos(vt ) en posant u¨ + v 20 1 + g g l

v 20 =

g l

2. Dans le cas des petits mouvements (u petit), l’équation précédente linéarisée se réduit à : dm ¨ u + v20 u ≈ v2 cos(vt) l On reconnaît l’équation caractéristique d’un oscillateur excité par déplacement (cf. chapitre 11).  c Dunod – Toute reproduction non autorisée est un délit

S24– 3. Oscillateur sur une poulie 1. Exprimons d’abord l’énergie cinétique et l’énergie potentielle de ce système : d(l − x) 1 1 1 E k = m x˙ 2 + m( x˙ + y) ˙ 2 + (2m) 2 2 2 dt

2

1 = 2m ˙x2 + m ˙y2 + m˙x y˙ 2

l étant la longueur du fil inextensible, et : 1 2 1 2 Ep = −mgx − mg(x + y + l0 ) − 2mg(l − x − pr ) + Ky = −mgy + Ky 2 2 à une constante additive près. On en déduit le lagrangien et les moments conjugués : L = Ek − E p

px =

∂L = m(4x˙ + ˙y) et ∂ ˙x

py =

∂L = m( ˙x + ˙y) ∂ y˙

2. L’hamiltonien est ici la somme de l’énergie cinétique et de l’énergie potentielle : H = Ek + Ep avec : Ek =

p2x + p 2y p2 − y 6m 2m

puisque

x˙ =

px − p y 3m

et y˙ =

4py − px 3m

750

Solutions des exercices

On trouve donc : H=

p2x + 4p2y px py 1 − − mgy + Ky2 6m 3m 2

3. Les équations canoniques donnent : p˙x = −

∂H =0 ∂x

d’où

p x = m(4x˙ + y˙ ) = Cte et

p˙ y = −

∂H ∂y

soit m(¨ x+¨ y) = mg − Ky

On en déduit : 4¨ y = 0 et ¨ x +¨ y + v 21y = 4g/3 avec v21 = 4K/(3m). La solution en y est une oscillation de pulsation v1 d’expression, compte tenu des conditions initiales : mg mgv 1 [1 + cos (v 1t)] et y˙ = − sin(v1 t) y= K K Il en résulte, puisque 4˙x + y˙ = 0 : y˙ mgv1 mg mg x˙ = − = sin(v1 t) et x = − cos(v1t) + Cte = [1 − cos (v 1t)] 4 4K 4K 4K S24– 4. Petites oscillations d’un cerceau lesté 1. Comme Ek et E p valent respectivement (cf. Exercices du chapitre 20) : E k = Mr2 u˙ 2 + mr u˙ 2(1 − cos u) 2

et Ep = Mgr + mgr (1 − cos u)

˙2

il vient : L = Ek − Ep = r u [M + m(1 − cos u)] − mgr(1 − cos u) − Mgr, d’où : d dt

2 2 2 2r [M + m(1 − cos u)]u˙ = −mgr sin u + mr sin u ˙u

Il en résulte : r2¨ u [M + m(1 − cos u)] = −mgr sin u − mr2 ˙u 2 sin u. 2. Comme H = Ek + Ep = r2 u˙ 2 [M + m(1 − cos u)] + mgr (1 − cos u) + Mgr, l’équation canonique suivant u est : ∂H = −mgr sin u + mr2 ˙u2 sin u p˙ u = − ∂u On retrouve bien l’équation différentielle déjà établie (cf. chapitre 20). S24– 5. Équations canoniques du problème de Kepler 1. Le lagrangien a pour expression : L = Ek − Ep = m(r˙2 + r2 w˙ 2 ) −

K r

∂L = m˙r ∂˙r

d’où pr =

et p w =

∂L = mr2 w˙ ∂ w˙

Par conséquent : H = Ek + Ep = p2r /(2m) + p 2w/(2mr 2 ) + K /r . 2. Les équations du mouvement sont :

∂H K p2 = 2 − u3 et p˙ w = 0 soit pw = Cte ∂r r mr Comme pw est le moment cinétique, on reconnaît les équations du problème de Kepler (cf. chapitre 12). p˙ r = m¨ r=−

S24– 6. Pendule double 1. Le lagrangien a pour expression : L = Ek − Ep

avec

Ek =

1 2 1 mvA + mvB2 2 2

et E p = −mgl cos u1 − mgl(cos u1 + cos u 2 )

Or v2A = l2 u˙ 21 et vB s’obtient en dérivant les composantes de OB = OA + AB : l R

cos u1 + cos u2 sin u 1 + sin u2 0

donne l R

−u˙ 1 sin u 1 − u˙ 2 sin u 2 u˙ 1 cos u 1 + ˙u2 cos u2 0

751

et problèmes Il en résulte que : L = ml2 2u˙ 21 + u˙ 22 + 2 u˙ 1u˙ 2 cos(u 1 − u2) /2 + mgl cos u 1 + mgl(cos u1 + cos u 2 ). On en déduit les moments conjugués : pu1 =

∂L = ml2 2 u˙ 1 + ˙u2 cos(u1 − u 2) ∂ u˙ 1

et

p u2 =

∂L = ml2 ˙u2 + u˙ 1 cos(u1 − u2 ) ∂ u˙ 2

ainsi que les équations du mouvement ˙pu 1 = ∂L/∂u1 et p˙ u 2 = ∂L/∂u 2. En effectuant on trouve : 2 ml2 2¨ u1 + ¨ u2 cos(u1 − u 2) − u˙ 2 sin(u1 − u 2)( u˙ 1 − ˙u 2) = −2mgl sin u1 − ml u˙ 1u˙ 2 sin(u1 − u 2 )

et

2 ml2 ¨ u2 + ¨ u1 cos(u1 − u 2) − u˙ 1 sin(u1 − u 2)( u˙ 1 − ˙u 2) = −mgl sin u2 + ml ˙u1 u˙ 2 sin(u1 − u 2 )

2. Dans le cas des petits angles, on obtient le système linéarisé suivant : u 1 + l¨ u2 = −2gu1 2l¨

u2 + l¨ u 1 = −gu2 et l ¨

S24– 7. Machines d’Atwood simple et double 1. On désigne par x 1 et x2 les coordonnées des deux masselottes suivant la verticale ascendante à partir du sol par exemple. Le fil étant inextensible, on a la relation : x1 + x 2 + l = Cte. Comme : Ek =

1 1 m 1x˙ 21 + m 2 ˙x22 2 2

et Ep = m1gx 1 + m2gx2 + Cte

le lagrangien s’écrit : L=

(m 1 + m2 ) x˙ 21 + g(m1 − m2)x 1 + Cte 2

d’où

d [(m1 + m2) x˙ 1] = g(m 1 − m2). dt

x1 = g(m1 − m2)/(m1 + m 2). Ainsi le mouvement est uniformément accéléré : ¨

2. Le système a deux degrés de liberté puisque les fils sont inextensibles. On a L = Ek − Ep avec : Ek =

1 1 1 2 2 2 m 3 x˙ 3 + m 4 ˙x4 + m 2 ˙x2 2 2 2

et Ep = m 2gx 2 + m3 gx3 + m 4 gx4

Comme les fils sont inextensibles : x3 + x4 + l2 − pr 2 = 2x 1, d’où x˙ 3 + ˙x4 = 2˙x 1 = −2˙x2 . Donc : Ek =

1 1 1 1 1 m3x˙23 + m 4 (−2x˙ 22 − x˙ 3 )2 + m 2 ˙x22 = x˙ 22 (m2 + 4m 4 ) + x˙ 23(m 3 + m 4) + 2m 4x˙ 2 ˙x3 2 2 2 2 2

 c Dunod – Toute reproduction non autorisée est un délit

et Ep = m2 gx 2 + m3 gx3 + m 4g(−2x2 − x 3) + Cte. On en déduit : p2 =

∂L = (m 2 + 4m4 )x˙ 2 + 2m4 ˙x3 ∂ x˙ 2

et p 3 =

∂L = (m 3 + m4)x˙ 3 + 2m4 x˙ 2 . ∂ x˙ 3

Les équations différentielles du mouvement sont donc : (m 2 + 4m4 ) ¨ x2 + 2m4 ¨ x3 = −m2 g + 2m4 g et

2 m 4¨ x2 + (m3 + m 4 )¨ x3 = −m3 g + m 4g

En combinant ces deux équations, on trouve : ¨ x2 =

4m3 m4 − m2 (m 3 + m4) (−m2 + 2m4 )(m3 + m4 ) − 2(−m3 + m4)m 4 g= g 2 + 4m ) − 4m 4m3 m4 + m2 (m 3 + m4) (m 2 4)(m3 + m4 4

S24– 8. Modélisation de la molécule de dioxyde de carbone 1. Considérant les trois coordonnées indépendantes, x1, x2 et x3 , le lagrangien a pour expression L = Ek − E p avec : m1 x˙ 21 m2 x˙ 22 m1 x˙ 23 K (x 1 − x 2) 2 K (x 3 − x 2)2 et E p = Ek = + + + 2 2 2 2 2

752

Solutions des exercices

Par conséquent : p1 =

∂L = m 1¨ x1 = −K (x1 − x 2 ) ∂˙x1

et p3 =

p2 =

∂L = m 2¨ x2 = K (x1 − x 2) + K (x 3 − x 2) ∂ x˙ 2

∂L = m1 ¨ x3 = −K (x 3 − x 2 ) ∂ x˙3

2. La quantité de mouvement du système dans R ∗ est nulle. Donc : m 1x˙1 + m 2 ˙x 2 + m 1x˙3 = 0

d’où

m 1x1 + m2 x 2 + m1x 3 = Cte = 0

car les atomes de cette molécule peuvent être simultanément au repos dans ce référentiel, x1 = x2 = x3 = 0. 3. Comme x 1 , x2 et x3 sont reliés, le système précédent est identique au suivant : m 1¨x1 + K (x1 − x 2 ) = 0

et m 2¨ x 2 + 2Kx 2 − K (x1 + x 3 ) = 0

soit m2¨ x2 + Kx2 (2 + m2 /m1) = 0

puisque (x1 + x3) = −m2x 2/m1 . S24– 9. Système de poulies

1. Le lagrangien de l’ensemble est : L = Ek − Ep avec :

1 1 1 Ep = −m1gx 1 − m2gx2 − m3 gx3 m 1 ˙x21 + m 2x˙ 22 + m 3x˙23 2 2 2 et x1 + 2x 3 + x2 + 3pr = Cte puisque le fil est inextensible. Par conséquent : Ek =

1 2 1 2 m3 m3 L = x˙ 1 m 1 + + ˙x2 m 2 + 2 4 2 4

+ x˙ 1x˙ 2

m3 m3 + gx1 m1 − 4 2

+ gx2 m2 −

m3 2

+ Cte

2. Les moments conjugués sont : ∂L ∂L m3 m3 m3 m3 ˙x 2 et p 2 = = m1 + x˙ 1 + = m2 + x˙1 + x˙ 1 4 4 4 4 ∂ ˙x 1 ∂ ˙x2 On en déduit les équations du mouvement : m3 m3 m3 m3 m3 m3 et m2 + ¨ ¨ ¨ ¨ m1 + x1 + x 2 = g m1 − x2 + x1 = g m 2 − 4 4 2 4 4 2 Par conséquent : 4m 1m 2 − m3 (3m2 − m1) 4m 1m 2 − m 3(3m1 − m 2) a1 = g a2 = g 4m1 m2 + m3 (m1 + m2 ) 4m 1 m2 + m3(m1 + m2 ) et 4m 1m 2 − m3(m 1 + m 2) a1 + a2 a3 = − = −g 2 4m 1m 2 + m3(m 1 + m 2) p1 =

S24– 10. Lagrangien et énergie d’un système articulé en rotation uniforme 1. Dans le référentiel tournant R  = Oxy  z, l’énergie cinétique des deux barres inférieures qui ont un point fixe est : Ek,1 = 2 × (1/2)(ml 2/3)u˙ 2 . Celle des deux barres supérieures s’obtient en appliquant le théorème de Kœnig. Comme OK et vK/R  ont pour composantes respectives :

R

0 (l/2) sin u (3l/2) cos u

et R

0 (l/2) u˙ cos u −(3l/2)u˙ sin u

L’énergie cinétique totale dans R est donc : Ek =

u˙ 2 2 ml × 2

on trouve E k,2 = 2 ×

u˙ 2 2 2 1 1 + + + 4 sin 2 u = ml × 3 6 2 2

1 2

4 4ml2 + 4 sin 2 u = 3 3

mv2K +

u˙ 2 2

ml 2 ˙ 2 u 12

1 + 3 sin 2 u

753

et problèmes

2. L’énergie potentielle du système dans le référentiel tournant R est la somme de trois contributions : pesanteur, force élastique et force centrifuge : Ep,g = 4mgzC = 4mgl cos u

E p,e =

K K (BF − l0 )2 = (L − 2l cos u − l0 )2 = 2mgl(2 − cos u)2 2 2

et E p,c = −IOz

V2 = −4 × 2

ml2 sin2 u 3

V2 2

Par conséquent : E p = mgl 4 cos u + 8 + 2 cos 2 u − 8 cos u −

2lV 2 sin2 u 3g

= mgl −4 cos u + 10 − 2 sin 2 u 1 +

lV 2 3g

soit :

V2 l sin2 u 1+ + 6mgl 2 3g Comme Ep est défini à une constante additive près, on peut abandonner le terme additif constant sans intérêt 6mgl. Ainsi : V20 = 3g/l. On en déduit le lagrangien et l’énergie constante du système dans R : E p = 4mgl 1 − cos u −

L  = Ek − Ep

et E  = Ek + Ep

S24– 11. Bille creuse dans un guide diédrique On a déjà établi les expressions de Ek et Ep par rapport au référentiel du laboratoire R (cf. Exercices du chapitre 20). On en déduit aisément le lagrangien : 7 2 2 V2 L = E k − Ep = mR u˙ + 6 2

2 2 2 2 mR sin u + mr 3

− mgR(1 − cos u)

On en déduit le moment conjugué en u : pu =

∂L 7 = mR2u˙ et 3 ∂u˙

soit :

∂L 7 u= mR2¨ = mR2 V2 sin u cos u − mgR sin u ∂u 3

RV2 cos u 7 mR2u¨ + mgR sin u 1 − g 3

=0

S24– 12. Invariant d’un pendule simple

 c Dunod – Toute reproduction non autorisée est un délit

1. Les expressions de E k et E p sont les suivantes : Ek =

mv2 ml2w˙ 2 = 2 2

et Ep = mgl(1 − cos w) ≈ mgl

w2 2

2. En l’absence de frottement, l’énergie mécanique se conserve, d’après le théorème de l’énergie : w2 ml2 w˙ 2 + mgl = Cte 2 2 En dérivant par rapport au temps l’équation de conservation de l’énergie, écrite en fonction de la seule variable de configuration w , on trouve évidemment : w ¨ + v20 w = 0 avec v20 = g/l. La pulsation v0 , la fréquence f0 et la période T0 valent respectivement : Em = Ek + Ep =

v0 =

g l

1/ 2

= 6, 26 rad.s−1

f0 =

v0 ≈ 0, 98 Hz et 2p

T0 =

3. a) Le moment cinétique du pendule a pour expression en O : L O = OA × mv = ml er × lw˙ ew = ml2 w˙ ez = Lz ez

1 ≈1s f0

754

Solutions des exercices b) Dans l’espace des phases (w, Lz) , le mouvement satisfait à l’équation : L 2z w2 =1 + 2E 2ml m 2Em /(mgl)

qui est celle d’une ellipse dont les axes sont a = [2Em/(mgl)] 1/2 et b = (2ml2 E m)1/2 . c) L’aire de cette ellipse vaut : 1/ 2 2Em (2ml2Em )1/2 = E mT0 A = pab = p mgl 4. a) Pour exprimer la variation élémentaire d’énergie lorsque l varie de d l , dérivons l’expression de Em dans l’espace des phases, par rapport à l . Il vient : d Em L2 mgu2 = − u3 + dl 2 ml

avec :

d’où

d Em L2 mglu 2 = − u2 + d l/ l 2 ml

dl d T0 l g −2 = 0 puisque = Cte = 2 4p 2 l T0 T0 Remplaçons le second membre par sa valeur moyenne. On y reconnaît le double de Ek , changé de signe, et Ep ; la valeur moyenne du premier terme est −Em alors que celle du second est E m/2 . Il en résulte : d Em Em =− 2 d T0 /T0 2

d’où

dEm dT0 =0 + Em T0

b) On en déduit que EmT0 = Cte . Ainsi, lorsqu’on tire lentement sur le fil, l’énergie et la période ne se conservent pas, mais leur produit, qui a la dimension physique d’une aire dans l’espace des phases, c’est-à-dire celle d’une action, d’un moment cinétique ou de la constante de Planck, se conserve, lui. c) Comme Em T0 se conserve, on a, puisque Em = mglu2m /2 , um étant l’amplitude angulaire du pendule : mglu2m l × 2p 2 g

1/ 2

3/ 4

= Cte d’où u ml

um,f = u m ,i

= Cte et

li lf

3/ 4

3/ 4

=2

≈ 1, 68

d) Le qualificatif adiabatique utilisé en thermodynamique signifie sans échange thermique avec le milieu extérieur ; c’est le cas ici puisque les frottements sont exclus, d’une part ceux de l’oscillateur initial, d’autre part ceux qu’introduirait la transformation si la variation de la longueur du fil n’était pas très lente.

Chapitre 25 S25– 1. Mise en rotation d’une plaque à l’aide d’un projectile 1. Comme l’énergie se conserve, on a : Em = E k + Ep = Cte avec E k =

1 Ma2 ˙ 2 u 2 3

et E p = −

Mga cos u 2

2. Pour que la plaque fasse un demi-tour, il faut que : Em  (Ep )max =

Mga 2

soit

Ma2 u˙ 20 Mga Mga  − 6 2 2

6g ce qui entraîne u˙20  a

Pour obtenir u˙ 0 , il suffit d’appliquer la conservation du moment cinétique selon l’axe de rotation au système projectile-plaque : a mv = 2

Ma2 ma2 + 3 4

On trouve : v  515, 3 m . s−1.

˙u0 d’où

u˙ 0 =

6mv (4M + 3m)a

et v 

M + 3m/4 m

8 ga 3

1/ 2

755

et problèmes S25– 2. Rotation autour d’un axe incliné par rapport à la verticale

En projection suivant l’axe Oz, le théorème du moment cinétique donne, puisque la liaison pivot est parfaite : ma2 ˙ a d LOz = (OC × mg) · e z avec L Oz = u et (OC × mg) · ez = −mg sin a sin u dt 3 2 En effet : OC = (a/2) cos u ex + (b/2) cos u ey et g = g sin a ex − g cos a ez . Dans le cas des petits mouvements, on trouve : 1/ 2 3g sin a 2a ¨ et T0 = 2p u + v 20u = 0 avec v 20 = 2a 3g sin a Si a = 0, il n’y a pas d’oscillation. On retrouve la période bien connue du pendule pesant en faisant a = p/2. S25– 3. Réactions sur l’axe d’un disque en rotation Appliquons le théorème de la quantité de mouvement en projection horizontale et le théorème du moment cinétique en projection suivant l’axe de rotation : u = 0 = F1d1 + F2d2 −mrV 2 = F 1 + F2 et IOz ¨ F1 et F2 étant les forces horizontales qui s’exercent sur l’axe. Par conséquent les expressions recherchées sont :

d2 >0 d2 − d1 On trouve F1 = −592 N et F2 = 237 N. −F1 = mrV2

et

− F 2 = −mrV

2

d1

. I 3(I 3 + mR 2 ) i) Cerceau I1 = mR2 /2 et I3 = mR2 : f˙ 20 > g/(4R) ε¨ + v 20 ε = 0

ii) Disque

I1 = mR2 /2

et

en posant v20 =

˙ 20 > g/(3R) f I1 =  0 : f˙ 20 doit être très grand.

I 3 = mR2 /2 :

iii) Roue avec axe matériel I3 ≈ 0

et

˙ 0 = 0 satisfont aussi à une solution stationnaire. Les équations (1) et (3) du 4. Les valeurs u 0 = p/2 et f ˙ : mouvement deviennent, si l’on néglige toutes les vitesses angulaires devant |c| ˙ 0 cos u + f) ˙ + I1c ˙ 20 ε = mgR ε (I1 + mR 2 ) ε¨ + (I3 + mR 2) c˙ 0(c d ˙ ˙ = mR2c˙ 0 ε˙ soit (I 3 + mR 2)(c˙ 0 cos u + f) ˙ = mR 2f˙ 0 ε. (c0 cos u + f) (I3 + mR 2 ) dt En reportant cette équation dans (1), on trouve : 2 2˙2 ˙2 (I 1 + mR ) ε¨ + (mR c 0 + I1 c0 − mgR) ε = 0

soit

2

ε¨ + v 0 ε = 0

2

avec

v0 =

˙ 20 − mgR (I1 + mR2)c I1 + mR 2

˙ 20 > mgR/(I1 + mR2). Ainsi, le mouvement est stable pour c i) Cerceau c˙ 20 > 2g/(3R) ˙ 20 > 4g/(5R) ii) Disque c iii) Roue avec axe matériel : la condition de stabilité est facile à réaliser.

 c Dunod – Toute reproduction non autorisée est un délit

S26– 8. Couple cabreur ou piqueur d’un avion L’avion, qui possède un moment cinétique orienté selon sa vitesse de translation, est soumis, lorsqu’il aborde un virage, de la part des parties tournantes, à un couple gyroscopique d’expression : ˙ ez × ˙c ez = I3 f ˙ c ˙ eu M C = LC × V D/R = I3 f ˙ c ˙ < 0, l’avion se cabre. Si I3 f˙ c˙ > 0, l’avion pique et si I 3 f S26– 9. Équations de Lagrange et équations canoniques du mouvement d’une toupie 1. L’énergie cinétique et l’énergie potentielle de pesanteur d’une toupie ont pour expression, avec les notations habituelles (cf. chapitre 26) : Ek =

1 ˙2 ˙ 2 sin2 u + I3V2z  ) et (I1 u + I1 c 2

Ep = mgl cos u avec

˙ Vz = c˙ cos u + f

On en déduit le lagrangien : L = E k − Ep =

1 ˙2 (I1u + I1 c˙ 2 sin 2 u + I 3V 2z ) − mgl cos u 2

762

Solutions des exercices

et les moments conjugués : pc =

∂L ˙ sin 2 u + I3 (c˙ cos u + f˙ ) cos u = I 1c ˙ ∂c

pu =

∂L ∂L ˙ cos u + f) ˙ = I1 u˙ et p f = = I3 (c ∂ u˙ ∂ f˙

Les équations de Lagrange sont donc : d pc ∂L =0 = dt ∂c d pu ∂L = dt ∂u

soit

2 p c = I1c˙ sin u + pf cos u = Cte

˙ 2 sin u cos u) + I 3V z  c ˙ sin u = mgl sin u soit I 1(¨ u− c

d pf ∂L = =0 ∂f dt

soit

˙ = Cte p f = I3(c˙ cos u + f)

2. Exprimons l’hamiltonien, c’est-à-dire l’énergie à l’aide des variables c, u, f et des moments conjugués pc , p u, pf : H = Ek + Ep =

2 2 2 1 ˙2 ˙ 2 sin 2 u + I3 V2z ) + mgl cos u = p u + (p c − p f cos u) + pf + mgl cos u (I1 u + I1 c 2 2I 1 2I3 2I 1 sin2 u

On en déduit les équations canoniques : d pc ∂H =− =0 dt ∂c d pu ∂H =− dt ∂u

soit

2 ˙ cos u + f˙ ) cos u = Cte I1c˙ sin u + I 3(c

˙ 2 sin u cos u − I3 V zc˙ sin u + mgl sin u soit I1 ¨ u = I1 c

∂H d pf =0 =− ∂f dt

soit

˙ = Cte pf = I3 (c˙ cos u + f)

Chapitre 27 S27– 1. Système de poulie et ressorts 1. a) À l’équilibre, on a, d’après les théorèmes généraux : m1 g 2 b) Désignons par X1 la coordonnée de A 1 à partir d’une origine prise sur le bâti fixe. En I, la vitesse du fil est ˙ En J , la vitesse est donc X˙J = X˙ 1 + Ru˙ = 2Ru˙ = 2 X ˙ 1. nulle : 0 = X˙ 1 − Ru. c) Les équations différentielles du mouvement sont obtenues en appliquant le théorème du centre de masse et le théorème du moment cinétique à la poulie. Ils donnent en explicitant : 0 = m 1 g − TI − T J

0 = R(TI − T J) d’où

m 1¨ X1 = m1 g − T I − TJ

Il en résulte : m1 +

I R2

T I = TJ =

et I ¨ u = R (T I − T J )

¨ X1 = m1g − 2TJ = m1 g − 2K1(X J − l0) = m1 g − 2K1 (2X 1 − Cte)

d’où :

4K1 m1 + I /R2 (X1 ) e étant la position de repos. Ainsi, le système oscille autour de cette position avec la période T1 = 2p/v 1 . ¨x1 + v 21x 1 = 0

avec

x1 = X 1 − (X 1)e

2 et v 1 =

763

et problèmes

2. a) L’équation à laquelle satisfait x 1 est l’équation différentielle précédente dans laquelle on ajoute la force K2(x2 − x 1 ) qu’exerce le second pendule, x2 étant l’écart de A 2 par rapport à la position d’équilibre. Quant à la seconde, on l’obtient en appliquant la loi fondamentale. On a donc : I m 1 + 2 ¨x1 = −4K 1x1 + K2 (x2 − x 1 ) et m 2¨ x2 = −K2 (x2 − x 1) R b) On trouve les pulsations propres du système par la méthode habituelle, à partir de l’équation du deuxième degré issue de la recherche de solutions en exp(jVt). Comme le système d’équations s’écrit aussi : ¨ x1 + v 20(x1 − x2) +

3v20 x1 = 0 2

¨ x 2 + v 20(x 2 − x1) = 0

et

l’équation caractéristique est : V4 −

7v20 2 3v4 V + 0 = 0 de solutions 2 2

v0 Va = √ 2

et Vb =

√ 3v 0

S27– 2. Système de deux barres couplées par un ressort 1. Le théorème du moment cinétique appliqué en O1 à la barre (1) donne : u1 = − I¨

mgl l l l sin u1 + K b − sin u 1 + sin u2 − l 0 2 2 2 2

Comme, à l’équilibre, u1 = 0, u2 = 0 et b = l0 , les termes constants s’annulent. L’équation du mouvement dans l’approximation des petits angles en résulte : u1 = − I¨

mgl Kl2 u1 − (u1 − u 2) avec 2 4

I=

ml2 3

De même, en appliquant ce théorème en O2 à la barre (2), on obtient : u2 = − I¨

mgl Kl 2 u2− (u 2 − u 1) 2 4

Les équations précédentes s’écrivent donc, en posant v20 = g/l et v 2c = K /m : 3v 20 3v2 ¨ u1 + u1 + c (u 1 − u2 ) = 0 2 4

3v20 3v 2c u2 + et u¨2 + (u2 − u 1) = 0 2 4

 c Dunod – Toute reproduction non autorisée est un délit

2. Pour trouver les modes normaux de vibration, cherchons des solutions en exp(jVt ). On obtient l’équation caractéristique suivante : −V2 +

3v 20 3v2c + 2 4

2



9v4c =0 16

d’où

V2a =

3 2 v0 2

et V2b =

3 2 (v0 + v2c ) 2

On en déduit : u1 = Aa cos(Va t + f a ) + Ab cos(V bt + f b) et u 2 = A a cos(Va + f a) − A b cos(Vbt + f b ) ˙ 2 = 0, u1 = 0 et u2 = u 0, on trouve : fa = 0, fb = 0, Compte tenu des conditions initiales, u˙ 1 = u A a = −A b = u 0 /2, d’où : u1 =

u0 [cos(V at) − cos(V bt)] 2

et u 2 =

u0 [cos(V a t) + cos(Vbt)] 2

S27– 3. Pendule de Wilberforce 1. L’énergie potentielle a pour expression : Ep =

Kx2 Cu 2 + + lxu 2 2

d’où Fx = −

∂E p = −Kx − lu ∂x

et Gu = −

∂Ep = Cu − lx ∂u

764

Solutions des exercices 2. D’après le théorème du centre de masse et le théorème du moment cinétique, on a, respectivement : m¨ x = −Kx − lu

u = −C u − lx et I ¨

3. En cherchant des solutions de forme complexe : x = A 1 exp(jVt ) et u = A2 exp(jVt ), on trouve les pulsations propres : l 1/ 2 l 1/ 2 Va = v20 + et V b = v 20 − mR mR S27– 4. Vibrations forcées d’une molécule ionique 1 . Les équations différentielles du mouvement sont : m1 ¨ u1 = −K (u1 − u2 ) − eE0 cos(vt)

et m2 ¨ u2 = −K (u2 − u1) − eE 0 cos(vt)

2 . En cherchant des solutions de forme complexe : u1 = q 1 exp(jvt ) et u2 = q 2 exp(jvt ), on obtient : q1 =

eE0 m2 [−K (m 1 + m2 ) + m1 m 2v 2 ]

et q 2 =

−eEm1 [−K (m 1 + m2) + m1 m2 v2]

On en déduit : q 1/q 2 = −m 2/m1 . Les mouvements de 1 et 2 sont donc en opposition de phase. S27– 5. Pendule double 1. Établissons les deux équations différentielles en appliquant le théorème du moment cinétique en O à l’ensemble, puis au second pendule en A1 mobile. Pour cela, calculons le moment cinétique en O de l’ensemble : LO = OA1 × mv 1 + OA2 × mv 2

avec OA1 = l(cos u1 ex + sin u 1 e y) et OA2 = l(cos u 1 + cos u 2 ) ex + l(sin u1 + sin u2 ) ey . On en déduit les vitesses : v1 = lu˙ 1(− sin u1 ex + cos u 1 ey ) et En effectuant, on obtient, suivant Oz :

v2 = l(−u˙1 sin u1 − u˙ 2 sin u2 ) ex + l(u˙ 1 cos u1 + ˙u2 cos u 2) e y

L O = ml2 2 u˙ 1 + u˙ 2 + (u˙ 1 + u˙ 2 ) cos(u1 − u2) ez Comme : on trouve :

d LO = OA 1 × mg + OA2 × mg = −mgl(2 sin u1 + sin u2)ez dt

u1 + ¨ u2 + +(¨ u1 + ¨ u2 ) cos(u1 − u 2 ) + ( ˙u1 + u˙ 2)(u˙ 2 − u˙ 1) sin(u1 − u2 ) = −mgl(2 sin u1 + sin u 2) ml 2 2 ¨ Le théorème du moment cinétique appliqué au second pendule en A1 s’écrit : d L A1 + mv1 × v2 = A1 A2 × mg = −mgl sin u 2 e z dt avec LA1 = A1A2 × mv2 = ml2 ˙u2 + u˙ 1 cos(u 1 − u 2) ez . En effectuant, on trouve : ml

2

2 ¨ u2 + ¨ u 1 cos(u1 − u 2) − u˙ 1 sin(u1 − u2 ) = −mgl sin u2

Dans le cas des petits angles, les équations précédentes se réduisent à : u1 + 2l¨ u 2 + +g(2u1 + u2 ) = 0 3l ¨

u 2 + l¨ u 1 + gu 2 = 0 et l¨

soit aussi, en remplaçant la première équation par la différence avec la seconde : u1 + l l¨

¨ u2 + gu 1 = 0 2

et

u2 + l ¨ u 1 + gu2 = 0 l¨

765

et problèmes

2. Cherchons des solutions de la forme : u1 = A 1 exp(jVt ) et u 2 = A 2 exp(jVt ). On obtient, en posant v21 = g/l : V2 2 2 2 (−V2 + v 21)A 1 − A2 = 0 et − V A1 + (−V + v1 )A2 = 0 2 d’où l’équation du deuxième degré : √ √ g g V4 − 4v21 V2 + 2v41 = 0 de racines V2a = (2 − 2) et V 2b = (2 + 2) l l S27– 6. Modes normaux de vibration longitudinale de la molécule CO2 1. Dans R∗ supposé galiléen (la molécule est isolée), les équations différentielles du mouvement sont : m 2¨x2 = −K (x 2 − x1 ) − K (x2 − x 3) et

m 1¨x1 = −K (x 1 − x 2 )

m 1¨x3 = −K (x3 − x 2 )

2 . Cherchons des solutions de la forme : x1 = A 1 exp(jVt ), x2 = A2 exp(jVt ) et x3 = A 3 exp(jVt ). Il vient : (−m 1V 2 + K )A1 − KA2 = 0

2 − KA1 + (−m 2V + 2K )A2 − KA 3 = 0

2 − KA2 + (−m1 V + K )A3 = 0

En annulant le déterminant des coefficients, on obtient : (K − m1 V2)[(−m2 V2 + 2K)(−m 1 V2 + K ) − K2 ] + K (−K) (−m1 V2 + K) = 0 soit : V2 (K − m1 V2)[m1m 2 V2 − K (m 1 + m2 )] = 0 On en déduit les solutions suivantes en V2 : V 2a = 0, V2b = K /m 1 et V2g = K (2m1 + m2 )/(m1 m2 ). 3. Pour V = Vb : A 2 = 0, A1 = −A3 ; le carbone est fixe et les atomes d’oxygène vibrent en opposition de phase avec une même amplitude. Pour V = Vg : A1 =

KA 2 m A = − 2 2 = −A3 −K (2m1 + m2 )/m2 + K 2m1

Les atomes d’oxygène sont en phase entre eux et en opposition par rapport au carbone. Le rapport de pulsations propres vaut : 2m1 + m2 1/2 32 + 12 1/2 Vg = 1, 91 = = 12 m2 Vb  c Dunod – Toute reproduction non autorisée est un délit

4. Les fréquences f b et f g sont respectivement : fb =

1 2p

KNA M1

1/ 2

14

= 0, 425 × 10 Hz et f g =

1 KNA (M1 + M2 ) 2p M1 M2

1/ 2

= 0, 815 × 10

14

Hz

S27– 7. Pendule élastique et pendule simple couplés 1. La quantité de mouvement a pour expression : P = m 1v1 + m 2v 2

avec

v1 = x˙ e x et v 2 = v1 + u˙ e z × A1 A2 = (˙x + lu˙ cos u)ex + l u˙ sin u ey

Quant à l’énergie cinétique et l’énergie potentielle du système, elles valent respectivement : Ek =

1 1 (m1 v21 + m2 v22 ) = (m 1 ˙x2 + m2 x2 + m2 l2 u˙ 2 + m2 l˙xu˙ cos u) 2 2

à une constante additive près.

et Ep =

1 2 Kx − m2 gl cos u 2

766

Solutions des exercices 2. Le théorème de la quantité de mouvement pour l’ensemble, donne, suivant la direction horizontale Ox : d[(m1 + m2 )˙x + m2lu˙ cos u] = −Kx soit dt

2

x + m 2l¨ (m1 + m2 )¨ u cos u − m2 l u˙ sin u = −Kx

Quant au théorème du moment cinétique, appliqué à A2 , au point mobile A1, il s’écrit : d L A1 + v1 × m2v2 = A1 A2 × mg avec dt

LA 1 = A1 A2 × m 2v 2

ce qui s’explicite selon : l sin u −l cos u × 0

(x˙ + lu˙ cos u) = lu˙ sin u 0

0 0 m 2 l2 u˙ + m 2l x˙ cos u

On en déduit l’équation suivante en u : m2l2 ¨ x cos u + m 2 lx˙ ˙u sin u = −m2 gl sin u. u + m2 l¨ 3. Comme le lagrangien a pour expression : L = Ek − Ep = il vient : px =

1 1 (m1 x˙2 + m2 x2 + m 2l 2u˙ 2 + m2 l˙xu˙ cos u) − Kx 2 + m 2 gl cos u 2 2

∂L = (m1 + m2 )˙x + m2 l u˙ cos u ∂ ˙x

et pu =

∂L = m2 l u˙ + m2 lx˙ cos u ∂u˙

d’où :

∂L ∂L m 2l˙x ˙u sin u d px d pu = −Kx et = = =− − m2 gl sin u dt dt 2 ∂u ∂x On retrouve ainsi les équations précédentes. 4. a) Dans l’approximation des petits mouvements, on obtient, en tenant compte des relations entre les masses, les deux équations différentielles suivantes : l u=0 ¨x + v20 x + ¨ 4

et l ¨ x=0 u + v20 u + ¨

La recherche de solutions complexes, de la forme : x = A exp(jVt) et u = B exp(jVt), donne l’équation aux valeurs propres suivante : V4 −

8 2 2 4 4 v0 V + v 0 = 0 de solutions 3 3

V2a =

2 2 v0 3

et V 2b = 2v 20

b) On en déduit les coefficients C1 = 2/l et C 2 = −2/l. En tenant compte des conditions initiales, on trouve finalement : x=

lu0 [cos(Va t) − cos(Vb t)] 4

et u =

u0 [cos(V a t) + cos(V bt)] 2

c) Les coordonnées normales s’obtiennent aisément en combinant x et u : x+

lu lu 0 cos(Va t) = 2 2

et

−x+

lu lu0 cos(Vb t) = 2 2

S27– 8. Étouffeur de vibration Les équations différentielles du mouvement s’obtiennent en appliquant le théorème du centre de masse aux deux solides. En appelant x1 et x 2 les coordonnées verticales ascendantes, comptées à partir du repos, il vient : m1 ¨ x 1 = −K1x 1 − a ˙x1 − K2 (x1 − x2 ) + F (t)

et m 2¨x2 = −K2 (x2 − x 1 )

767

et problèmes

La recherche de solutions harmoniques forcées, de la forme x1 = A1 exp(jvt ) et x2 = A2 exp(jvt ), donne les équations algébriques suivantes : [−m1 v 2 + (K1 + K2) + jva]A1 + K2A2 = F m

et (−m2 v 2 + K2 )A 2 − K2 A1 = 0

On en déduit que A1 = 0 si m 2 = K 2/v 2. S27– 9. Chaîne de trois oscillateurs mécaniques identiques Comme les masselottes fictives 0 et 4 aux extrémités sont fixes, il vient (cf. chapitre 27) : k=k

p (N + 1)d

soit

kd = k

p 4

puisque N = 3

On en déduit l’expression générale de la pulsation propre : Vk = 2v 0 sin k

p 2(N + 1)

= 2v0 sin k

p 8

avec

v0 =

K m

1/ 2

Ainsi les pulsations propres sont : V a = 2v0 sin

p 8

Vb = 2v0 sin

p 4

et V g = 2v0 sin

3p 8

Chapitre 28 S28– 1. Accéléromètre à liquide La surface du liquide est normale au champ de pesanteur apparent g a = g − a 0 , a0 étant l’accélération d’entraînement du véhicule. D’où : tan a = h/d = a0/g = 0, 57 et a 0 = 5, 6 m . s−2.

 c Dunod – Toute reproduction non autorisée est un délit

S28– 2. Eau dans une cuve rectangulaire 1. L’équation de la statique des fluides dans R donne : p(x , z) = −rax −rgz+Cte. Les surfaces équibares sont donc des plans inclinés d’équation : z = −ax/g + Cte. Comme la valeur maximale de la hauteur d’eau est h, on a l’équation suivante de la surface de l’eau : a  z =− x +h g 2. Si l’une des faces verticales de la cuve n’est plus en contact avec le liquide, z = 0 sur cette face. Par conséquent : a z = 0 = − b + h avec h = 2h0 = 0, 3 m g puisque le volume de fluide est le même : b2h 0 = b2 h/2. La valeur de a correspondante est : 2h0 0, 3 a= g= g = 2g b 0, 15

768

Solutions des exercices

S28– 3. Liquide en rotation uniforme 1. On sait que (cf. chapitre 28) : z=

V2 r2 + z0 2g

z 0 étant la hauteur du sommet du paraboloïde. On détermine z0 en traduisant la conservation du volume occupé par l’eau, puisque les liquides sont incompressibles : R

pR2 h e =

R

0

V2 r2 + z0 2g

r

2prz d r = 2p 0

d r = 2p

V2 R4 R2 + 2pz0 2g 4 2

Ainsi z0 = he − V 2R2 /(4g). On en déduit que z 0 = 0 pour V = 2(ghe )1/2 /R ≈ 40 rad . s−1 , soit 6, 4 tr . s−1. 2. L’eau déborde du flacon si z > h, c’est-à-dire : V 2 R2 V2R2 + he − > h soit V > V s avec 2g 4g

Vs =

2 [g(h − he )]1/2 ≈ 79 rad . s−1 R

3. La pression p au fond du flacon vaut : p = p a + rgz 0 = pa + rgh e − rV2 R 2/4 ≈ p0 + 364 Pa. S28– 4. Baromètre à deux liquides 1. La pression dans l’extrémité supérieure du tube effilé étant négligeable, on a : pa r − m h 1 = 10, 3 cm pa − 0 = rm gh1 + rggh2 d’où h2 = rg g rg 2. Si p 0 varie de Dp0 , les trois niveaux (dans la cuve, à l’interface et à l’extrémité supérieure du tube) changent. Désignons par z1 , z2 et z 3 les cotes de ces niveaux. On a h 2 = z 2 − z1 et h1 = z1 − z3 , d’où : rm Dp0 − Dh 2 = Dz2 − Dz1 et Dh1 = Dz1 − Dz 3 et Dz 2 − Dz1 = (Dz1 − Dz3 ) r gg rg Les liquides étant incompressibles, on a les relations S1 Dz1 = S2 Dz2 et S1 Dz1 = −S3Dz 3. En remplaçant, on obtient : Dz2 1 −

S2 S1

=

rm Dp0 Dz2 − rg g rg

S2 S2 + S1 S3

soit Dz2 =

Dp0 /g r g(1 − S2 /S1 ) + r m (S2 /S 1 + S2 /S3)

Concrètement : Dz2 = 3, 7 cm. Avec un baromètre à mercure ordinaire, on aurait obtenu : Dz =

10 3 Dp = = 7, 5 mm rb g 13 600 × 9, 81

L’intérêt du baromètre à deux liquides est sa grande sensibilité. S28– 5. Manomètre à deux liquides 1. On obtient le rapport des hauteurs de liquide au-dessus du ménisque à l’aide de l’équation suivante : re ghe = ragh a

d’où

ra he = 0, 975 = ra ha

769

et problèmes

2. Désignons par z e , za et zm les hauteurs, au-dessus du tube horizontal, des surfaces eau-air, aniline-air et eau-aniline. La relation précédente devient : r e g(ze − z m ) = ra g(za − z m) d’où

Dp + r eg(Dz e − Dz m) = r ag(Dza − Dzm)

Comme les liquides sont incompressibles, on a, en outre : S1 Dze = sDzm et S 2 Dza = −sDzm . Il vient donc : Dzm (re − r a) −

s s re + ra DS1 DS2

=

Dp g

d’où

Dz m 1/g =− − r + r Dp ra e e s/S1 + ra s/S2

On trouve : Dzm /Dp = −1, 8 × 10 −3 m . Pa−1. Avec un baromètre à mercure ordinaire, on aurait obtenu : Dzm 1/g 1 = −7, 5 × 10−6 m . Pa −1 =− =− Dp rm 13 600 × 9, 81 La sensibilité a donc été multipliée par 240, cela grâce à l’utilisation de deux liquides de masses volumiques voisines et du rapport faible des sections s/S1 et s/S2. S28– 6. Tube en U en rotation uniforme 1. On obtient la densité de l’huile en exprimant de deux façons la pression à l’interface I entre l’huile et l’eau : rh e = = 0, 8 p I = p a + rh gh = pa + re ge d’où d = h re 2. Lorsque le tube en U tourne uniformément, l’équation des isobares dans l’eau est donnée par : p = −r egz + r e

V2 r2 + Cte 2

r étant la distance du point considéré à l’axe de rotation. Appliquée au point I à l’interface huile-eau, et au point E à l’interface air-eau, cette équation donne : pE − pI = −re ge + re

V2 2 (r E − r2I ) 2

 c Dunod – Toute reproduction non autorisée est un délit

Or pE − p I = pH − p I = −r hgh = −r e ge. Par conséquent : 

e = e+ a) Pour r I = 0 : e = e +

V2 2 (r − r2I ) 2g E

V2 2 4p2 2 l = 0, 10 + 0, 15 = 0, 145 m 2g 2g

b) Pour r I = rE , les hauteurs restent inchangées, ce qui était prévisible puisque les mêmes quantités d’eau dans la partie horizontale du tube sont concernées par la force d’inertie centrifuge. S28– 7. Tube coudé en rotation uniforme 1. Le liquide monte dans la partie verticale du tube coudé car l’air, contenu dans la branche horizontale, est soumis à la force d’inertie centrifuge, ce qui provoque une aspiration du liquide. 2. L’équation de la statique des fluides en référentiel non galiléen, appliquée à l’unité de volume de fluide dans la branche horizontale, donne, en projection horizontale : ∂p/∂r = rV2r.

770

Solutions des exercices

Or, l’air étant à température ordinaire un gaz parfait de masse molaire M = 29 g . mol−1 , on a : pV =

m RT M

d’où r =

m pM = V RT

Il vient donc, l’altitude étant fixée : dp pM 2 Vr = dr RT L’intégration donne aisément : p = Cte exp

dp M 2 V rdr = p RT

soit

r2 r 20

avec

ro =

2RT MV2

On détermine la constante à l’aide de la valeur de la pression qui vaut pa si r = l : p 0 = Cte × exp

l2 r 20

d’où p = pa exp

(r 2 − l 2 ) MV2(r 2 − l2 ) = p a exp 2 2RT r0

Ce résultat rappelle le nivellement barométrique, l’énergie potentielle de pesanteur Mgz étant remplacée par l’énergie potentielle centrifuge −MV2 r2 /2. 3. La hauteur h de l’eau dans la branche verticale s’obtient selon : r egh = p a − pI , I étant un point de l’interface liquide-air dans le tube. Comme pI égale sensiblement la pression dans l’air du tube pour r = 0, on trouve : MV 2l2 pa h = h 0 1 − exp − avec h0 = = 10, 33 m re g 2RT Comme l’argument de l’exponentielle est très petit, on peut développer ce terme : MV2l2 ≈ 1, 83 × 10−3 2RT

d’où h ≈ 10, 33 × 1, 83 × 10−3 ≈ 19 mm

S28– 8. Variation de pression dans la troposphère Comme d p/ d z = −rg et p = rrT = rr (T 0 − Bz), il vient : dp g dz =− p r(T0 − Bz)

ce qui donne en intégrant

ln

p g ln(T0 − Bz) = Cte rB

On a donc :

Bz n g avec n = T0 rB p(0) = Cte × T0n étant la pression à z = 0. On trouve p(0) = 101, 3 kPa, n ≈ 5, 26 et T0 /B = 44, 33 km. p = Cte × (T0 − Bz)n = p(0) 1 −

S28– 9. Statique d’un aréomètre Les conditions d’immobilité de l’aréomètre dans l’eau (masse volumique re ) et dans le liquide (masse volumique r) se traduisent respectivement par : 0 = −mg + re g( Par conséquent : re g(

+ S xe ) = rg[

+ Sx e ) et

0 = −mg + rg[

+ S (xe + x)] d’où : d=

r = re

+ Sxe + S (xe + x)

+ S (xe + x)]

771

et problèmes S28– 10. Statique d’un cube en bois partiellement immergé Si le cube est immobile, on a, F étant la poussée exercée par le liquide (masse volumique r e ) : 0 = mg + F

soit

0 = mg − re a 2hg

en projetant suivant la verticale descendante. La position d’équilibre est donc, r étant la masse volumique du bois : h=

rb m = a = 0, 5 m ra2 r

Le mouvement autour de cette position d’équilibre satisfait à l’équation différentielle issue de l’application du théorème du centre de masse, dans laquelle z désigne le déplacement suivant la verticale descendante : m¨ z = −rga 2 z soit ¨ z + v 20z = 0

en posant

v20 =

ra 2 g rg = rba m

On en déduit la période d’oscillation T0 = 2p/v 0 ≈ 1, 4 s. S28– 11. Statique d’un tronc d’arbre flottant sur l’eau Le tronc d’arbre de masse volumique r a inférieure à celle (re ) de l’eau flotte. Il est immobile sous l’action de son poids et de la poussée exercée par l’eau : 0 = mg + Fp . On a donc, en projetant suivant la verticale ascendante : u − sin(2u) 2 2 0 = −ra pR lg + r eSilg où Si = R 2 désigne la surface de la section immergée et u l’angle qui définit l’immersion : h = R(1 − cos u). Comme h = 0, 87 m, u = 82, 5 ◦ . On en déduit ra selon : ra =

re p 1 000 × 2 [2u − sin(2u)] = × u − sin(2u) = 417 kg . m−3 2p 2p 180

S28– 12. Statique d’une bille entre deux liquides

 c Dunod – Toute reproduction non autorisée est un délit

1. L’immersion est totale si : 4prs R3 4prl R3 − 0 3 3

soit a =

rs  1. rl

Il y a demi-immersion si : 4prs R3 2prl R3 − =0 3 3 2. La sphère est immobile si 4pr sR3 /3 − r l √

R i

= 2p

dx R− X

0

R2 −x2

i

R− X

rs = 0, 5 rl

= 0. Le volume immergé

R

r dr = p

soit

2

2

2

d x(R − x ) = p R x −

i

x3 3

On en déduit l’équation : 4r sR3 = X 2(3R − X )r l soit 3R − X = Il en résulte, en identifiant que : b = 3R et c = 4aR3 .

4aR3 X2

vaut donc : R

= pX R− X

2

R−

X 3

772

Solutions des exercices 3. Si F est la poussée d’Archimède, le théorème du centre de masse C s’écrit : 4 pr s R 3a C = F avec 3

aC = ¨ x ex

et F = −rl D

ig e x

si x désigne le déplacement du centre de masse à partir de sa position de repos. Comme D

i

= p(2XR − X2 )DX = pX (2R − X )x, il vient :

4p 3 r sR ¨x = −rl xpX (2R − X )g 3

2 d’où ¨x + v0 x = 0

avec

2

v0 =

3gX(2R − X ) 4aR3

La sphère oscille donc avec la période T0 = 2p/v0 . S28– 13. Statique d’une sphère flottant sur un liquide La masse volumique de la bille étant comprise entre les deux masses volumiques des liquides, la bille s’immobilise entre les deux liquides de telle sorte que l’on ait, si x désigne la fraction de volume immergée dans l’eau : mg + Fe + F h = 0 avec m = rb Fe = r e x g ez F h = r h(1 − x ) g e z Il vient, en projetant suivant la verticale ascendante : −rbg

+ xr eg

+ (1 − x)r hg

=0

d’où

x=

r b − rh = 0, 4 r e − rh

S28– 14. Extraction d’un liquide d’un récipient cylindrique L’eau ne s’écoulera plus par le robinet lorsque sa pression au bas du récipient sera égale à la pression atmosphérique. Par conséquent, si I est un point de l’interface eau-air dans le récipient et Sx le volume de l’eau qui s’est écoulé, on a : pI + r eg(l0 − x) = p0

I étant un point de l’interface eau-air dans le récipient. Or, l’air étant un gaz parfait, on a aussi la relation suivante : pI S(h − l0 + x) = p 0 S(h − l0 ). En éliminant p I entre ces deux équations, on obtient : p0 − reg(l 0 − x) =

p 0(h − l0 ) h − l0 + x

soit x 2 + x

p0 + h − 2l0 re g

− l0(h − l0 ) = 0

Numériquement, on obtient l’équation du second degré : x2 + 9, 4x − 0, 2 = 0. La seule solution physiquement acceptable est x = 2, 1 cm, ce qui correspond au volume : = Sx = (pD2/4)x = 10, 55 L. S28– 15. Barrage à profil parabolique En omettant la contribution atmosphérique, on sait que les composantes horizontale et verticale de la somme des forces de pression sont données par les expressions suivantes : 0

Fh = −h0

0

(−rgz) a d z et

Fv = −h 0

b

(−rgz) a d y = −rgah0

0

y b

2

− 1 dy

Le calcul donne F h = rgah20 /2 = 10, 6 MN et Fv = 2rgah0 b/3 = 4, 7 MN. S28– 16. Barrage en béton à profil triangulaire Le barrage est soumis aux forces de pression de l’eau Fp , à son poids M g et à la réaction R = Rtey + R n e z qu’exerce le sol sur lui. Il est en équilibre si : F p + Mg + R = 0 avec

|R t|  m|Rn |

773

et problèmes On a donc les deux équations suivantes, relatives aux projections selon les vecteurs ey et ez : Fp,y + R t = 0

et F p,z − Mg + Rn = 0

avec

Il en résulte :

|Fp,y |  ms |Mg − Fp,z |

HLl g 2 La projection selon Oy de la somme des forces de pression due à l’eau, a pour expression, en enlevant la contribution de la pression atmosphérique qui agit sur les deux faces : Mg = r b

h

Fp,y = 0

(p − p a) l d z avec

p = −re gz + Cte

Comme p = pa pour z = h, il vient : h

p = p a + re g(h − z) d’où Fp,y = re gl

0

(h − z) d z = r e gl hz −

z2 2

h

= 0

re glh2 2

Quant à la projection selon Oz de la somme des forces de pression due à l’eau, elle s’écrit : h

Fp,z = reg 0

On a donc :

(h − z) (−l d y) = −

reh 2  ms rb HL + r e h2 L/H

L r egl H

soit L 

h 0

(h − z) d z = −

L re glh2 H 2

1 = 36, 7 m m s(r b /re )H /h2 + 1/H

S28– 17. Verre plein ou vide sur un flotteur 1. Avec les notations de la figure 28.25, écrivons la conservation de la masse et donc du volume, l’eau étant incompressible : S(z1 − h1) + (S − s)h 1 + ah = S (z2 − h 2) + (S − s)h2

d’où S(z2 − z 1 ) = s(h2 − h 1 ) + ah

2. Traduisons le repos du système dans les deux configurations, en utilisant le théorème d’Archimède : (M + re ah)g = r e sh 1g et

Mg = r esh 2 g d’où

s(h1 − h2 ) = ah

3. La comparaison de ces deux équations donne :

 c Dunod – Toute reproduction non autorisée est un délit

S(z2 − z1 ) = s(h2 − h1 ) + ah = 0

Le niveau dans le cristallisoir ne varie donc pas. Une variante de ce problème est la fusion d’un glaçon flottant dans un verre d’eau plein ; le niveau de l’eau ne varie pas au cours de la fusion. S28– 18. Eau dans un entonnoir retourné En raison de la symétrie autour de l’axe vertical, la résultante des forces de pression Rl qu’exerce latéralement l’eau sur les parois de l’entonnoir est verticale. Si cette force est suffisante, le cône est soulevé ; on a alors à la limite : Mg + Rl = 0, car le contact cessant la réaction exercée par le plan est nulle. Pour calculer R l, exprimons la pression p en un point de cote z et introduisons l’angle g du cône : R h Notons que la contribution de la pression atmosphérique est nulle puisqu’elle s’exerce sur toute la surface qui délimite le système. Il vient donc : p = pa + rg(zm − z) et

R l = ez

p d S sin g = rg sin g ez

tan g =

(z m − z) × 2pr

dz cos g

774

Solutions des exercices

Comme r = R − z tan g, Rl s’écrit : zm

Rl = 2prg tan g e z

(zm − z)(R − z tan g) d z = 2prg tan g ez

0

[zm R − z(R + zm tan g) + z2 tan g] d z

Le calcul de l’intégrale donne : z2mR −

z2m z3m tan g z2m = (R + z m tan g) + 2 3 2

R−

zm tan g 3

On en déduit la masse de l’entonnoir : M = pr

R 2 zm zmR 1 − h 3h

= pr

R2 2 zm zm h − 2 h 3

= 367, 5 g

S28– 19. Eau dans une cloche hémisphérique La cloche se soulève si la hauteur hm est telle que le poids de la cloche est inférieure à la composante verticale des forces de pression. Cette dernière s’écrit : F p,z =

p d S × sin u avec

p = p a + r e g(h − z)

et

d S = 2p(R cos u)R d u

Le facteur sin u provient de la projection sur l’axe vertical. Par conséquent, en omettant la contribution de la pression pa qui agit de part et d’autre de la paroi du récipient, il vient : uh

Fp,z = 0

re g(h − R sin u) sin u 2p(R cos u)R d u

avec

Ainsi : uh

2

Fp,z = 2pR r e g 0

2

(h − R sin u) sin u d(sin u) = 2pR re g h

sin uh =

h R

sin2 u sin3 u −R 2 3

uh 0

On trouve donc : F p,z = 2pR 2r e g h

sin2 uh sin3 uh −R 2 3

= 2pR2r e g h

h2 /R 2 h3/R 3 −R 2 3

=

pre gh3 3

La cloche se soulève si : pr e gh3  Mg soit h  h m 3

avec

hm = (3Mpre )1/3

Pour M = 2 kg, hm = 12, 4 cm. Notons que hm  R, ce qui implique une valeur minimale de R pour une masse donnée. S28– 20. Effet paradoxal de la force centrifuge Si V = 0, la direction prise par la ficelle est celle de la verticale ascendante (celle de −g) puisque la balle est moins dense que l’eau. Pour V = 0, dans le référentiel qui tourne uniformément, le champ de pesanteur habituel est remplacé par le champ de pesanteur apparent ga = g + V2 HA, H étant la projection de A suivant l’axe de rotation. La direction de la ficelle est donc celle de −ga : la balle se rapproche de l’axe de rotation !

775

et problèmes

Chapitre 29 S29– 1. Lignes de courant dans un fluide Les lignes de courant sont données par les équations : dx dy = 2x 2y

dx dz =− z 2x

Il en résulte que : ln x = ln y + ln C1 et ln x = −2 ln z + ln C2, d’où l’équation des lignes de courant : x = C1 y et x =

C2 z2

Pour la ligne de courant qui passe par le point A, les valeurs des constantes sont les suivantes : C1 = 2 et C2 = 2. S29– 2. Profil gaussien de vitesse d’un fluide dans une conduite On a :

2r2 v(r0 ) = exp − 2 0 v0 r0 /4

et

r0

qv = 0

v0 exp −

2r2 w2

= exp(−8) ≈ 0, 33 × 10−3 −w 2 2

2pr d r = pv 0

[exp(−8) − 1] ≈

pv0r 20 v0 S = 8 4

On en déduit la vitesse moyenne : vm = qv /S = v0 /4. S29– 3. Profil parabolique de vitesse d’un liquide dans une conduite 1. La vitesse n’étant pas uniforme, le débit-masse de liquide est donné par l’intégrale : qm = r S

v dS = r

r0 0

r r0

v0 1 −

2

2pr d r = 2prv0

r2 r4 − 2 2 4r0

r0

2 v0

= rpr0 0

2

La vitesse moyenne vm, définie par q m = rpr20 vm , vaut donc vm = v0 /2.

 c Dunod – Toute reproduction non autorisée est un délit

2. Le débit d’énergie cinétique est : dEk = dt

S

r0

rv 2 v d S = pr 2

v03

0

r2 1− 2 r0

3

r d r = prv30 −

r20 2

0

u3 d u =

1

rv 30 2 pr 0 8

Le débit d’énergie cinétique, dans le cas d’une vitesse uniforme égale à la vitesse moyenne v0 /2, est : d Ek dt

v0 2

=r vm

2

v0 2 rv 30 2 pr0 = pr0 d’où 2 16

d Ek / d t =2 (d Ek / d t) vm

S29– 4. Accélération d’une particule de fluide dans un champ de vitesse radial On sait que : ∂v + (v · ∇)v soit a = (v · ∇)v ∂t puisque le champ est stationnaire. En coordonnées polaires, on a : a=

a = vr

∂ ∂r

vr er =

1 r2



2 r3

er = −

2 r5

er = −2 m . s −2

776

Solutions des exercices

S29– 5. Variation de masse volumique d’un fluide Calculons div v : div v = On en déduit :

1 ∂ (rv r) 1 ∂v u 4 sin u − = 0− r ∂r r ∂u r

1 Dr 4 sin u = 0, 01 = − div v = r r Dt

d’où

Dr = 1 kg . m−3 . s −1 Dt

S29– 6. Écoulement incompressible d’air dans une conduite Le débit-masse est : qm = rSv. Comme : r=

3 × 105 8, 32 p R = = = 3, 56 kg . m −3 puisque r = ≈ 287 J . K−1 . kg −1 rT M 287 × 293 0, 029

on trouve : qm = 3, 56 × p × 0, 01 × 3 = 0, 336 kg . s−1 .

S29– 7. Vitesses d’un liquide dans une conduite de section variable Les vitesses v 1 et v2 en amont et en aval sont reliées par l’équation : qv = S1 v 1 = S 2 v2, le pétrole liquide étant incompressible. Par conséquent : v1 =

qv 0, 1 ≈ 5, 66 m . s −1 = p × 0, 152 /4 S1

et v 2 =

qv 4 × 0, 1 ≈ 4 × 5, 66 = 22, 64 m . s−1 = p × 0, 075 2/4 S2

S29– 8. Vecteur rotation d’un solide et vorticité Considérons un cylindre tournant autour de son axe fixe avec une vitesse angulaire Vs. La vitesse d’un point du périmètre au point A de coordonnées (x, y) est : v = Vs × OA = −Vs y e x + Vs x ey . On en déduit la vorticité V = rot v/2 : Vx =

1 2

∂vz ∂v − y ∂y ∂z

=0

1 2

Vy =

∂vx ∂vz − ∂z ∂x

=0

Vz =

1 2

∂v y ∂vx − ∂x ∂y

= Vs

Ainsi, V = Vs d’où le vecteur tourbillon. S29– 9. Champ de vitesse d’un fluide Calculons div v :

1 Dr ∂vx ∂v y ∂v z + + = 0 d’où =0 r Dt ∂x ∂y ∂z Le fluide est donc en écoulement incompressible. Le vecteur V vaut (1/2) rot v. Comme : div v =

(rot v)x =

∂vz ∂vy − ∂y ∂z

= 2(y − z)

et de même pour les autres composantes : V = (y − z) ex + (z − x) ey + (x − y) ez. S29– 10. Champ de vitesse dans une tornade Comme la vitesse est continue : v c (r c) = ve(rc ), d’où K = Vt r2c . Pour calculer le vecteur tourbillon V, rappelons que : C

v · dr =

S

rot v · n d S = 2

S

V · n dS

Appliquons cette équation au cercle horizontal de rayon r et de centre le point O de l’axe de la tornade : Vt r × 2pr = 2Vpr2

d’où V = Vt

777

et problèmes S29– 11. Superposition de deux champs de vitesse irrotationnels Le champ de vitesse se déduit aisément du potentiel résultant : d’où v = − grad F = −10(3 e x + 4 ey )

F = F1 + F 2 = 10(3x + 4y) Les lignes de champ s’obtiennent selon : dx dy = 3 4

ce qui donne y =

4 x + Cte 3

Les lignes de champ sont donc des droites de pente 4/3 dans le plan du mouvement. Ce champ de vitesse est uniforme et vaut 10(9 + 16)1/2 = 50 m . s −1. S29– 12. Superposition d’un écoulement uniforme et d’un écoulement radial divergent 1. En superposant les fonctions potentiel et courant de ces deux écoulements incompressibles, on trouve : F = −v0 r cos u − K ln r

et

C = −v0 r sin u − K u

avec

K>0

l’écoulement radial étant divergent. On en déduit l’expression du champ de vitesse : v r = v0 cos u +

K r

et v u = −v0 sin u

2. La ligne de courant, qui coïncide avec l’axe Ox pour r infini, est caractérisée par u = p puisqu’on a alors vr = −v0 et v u = 0. Donc C = −Kp. On en déduit l’équation de cette ligne : −Kp = −v 0 r sin u − K u

soit r sin u =

K (p − u ) v0

ce qui s’écrit r sin u = r0 a

en posant a = p − u et r0 = K/v 0. Pour 0 < a < p, r(sin a/a) = r 0 : r est l’inverse de la fonction bien connue sin a/a (cf. Optique). Notons que pour a = 0, r = r0 . On obtient les points d’arrêt à l’aide des équations suivantes traduisant l’annulation de la vitesse (Fig. S29.1) : vr = v0 cos u + K /r = 0

et

vu = −v0 sin u = 0

Il en résulte que u = 0 ou u = p. La première solution (u = 0) est exclue car elle implique v0 + K /r = 0, alors que K > 0. Pour u = p, −v0 + K /r = 0 admet la solution r = r0 = v0 /K .

 c Dunod – Toute reproduction non autorisée est un délit

y

A0

a r0 0

u

x

F IG . S29.1.

S29– 13. Écoulement dans un dièdre droit Le potentiel complexe W (z) = Kz2 /2 donne : F(x, y) = Re

K 2 (x + jy) 2

=

K 2 2 (x − y ) et 2

C(x, y) = Im

K 2 (x + jy) 2

= Kxy

Les lignes équipotentielles sont donc les hyperboles x2 − y2 = Cte et les lignes de courant les hyperboles orthogonales xy = Cte. On en déduit : vx = −Ax et vy = Ay. L’origine O est donc un point d’arrêt.

778

Solutions des exercices

Chapitre 30 S30– 1. Ondes de gravité On sait que la vitesse de propagation v des ondes de gravité est v = (gH)1/2 = 1, 57 m . s−1 d’où : l = vT = 1, 57 × 4 = 6, 26 m

et t =

2L 2 × 40 = 51 s = v 1, 57

S30– 2. Oscillations d’un liquide dans un tube en U 1. La force F qu’il faut exercer est telle que pa + F /s = pa + rgh d’où F = srgh = 0, 196 N. 2. Le théorème du centre de masse, appliqué à la colonne de liquide de longueur l, donne, puisque la dénivellation est 2x lorsque le déplacement du centre de masse est x : rSl ¨ x = −2rgS x

d’où

¨ x + v20 x = 0

2

avec

v0 =

2g l

et T 0 =

2p l = 2p v0 2g

1/ 2

= 0, 63 s

S30– 3. Jet d’eau de Genève Comme q m = r(pd 2/4)v, v = 52, 6 m . s−1. D’après l’équation de Bernoulli : 1 2 rv = pa + rgh 2 On en déduit la hauteur réelle et la puissance nécessaire : pa +

hr = rh = r

v2 = 106 m et 2g

P=

dm dt

v2 2

q m v2 = 692 kW 2

=

S30– 4. Durée de vidange d’un réservoir La relation entre la hauteur d z et la durée élémentaire d t est immédiate : d t = − d z/v avec vS = s du fait de la conservation du débit-volume pour ce fluide en écoulement incompressible. D’après l’équation de Bernoulli, qui s’écrit r( 2 − v2 )/2 = rgz avec v  , il vient : H

t= 0



S s

dz

≈−

S s

H

0

dz S = s (2gz)1/2

2H g

1/ 2

= 2, 7 s

S30– 5. Puissance fournie par une turbine hydraulique avec diffuseur 1. Comme la surface de l’étendue du barrage est très grande, on peut supposer que la vitesse de l’eau sur la surface est négligeable et donc la hauteur hb constante. Appliquons la relation de Bernoulli entre un point A de la surface de l’eau du barrage et la sortie S de la turbine. Il vient, en régime stationnaire : vs2 pa + + ghs r 2

− 0+

pa + gh b r

=w

d’où le travail massique que fournit la turbine au milieu extérieur : wf = −w = g hb − hs − v 2s /2 . Concrètement : vs = qv /(pD2 /4) = 7, 64 m . s−1 et w f = 9, 81 65 − 7, 642 /2 = 351 J . kg −1, d’où la puissance totale fournie Pf = 351 × 6 000 = 2, 1 MW. 2. Avec le diffuseur, l’application de la relation de Bernoulli entre A et D, de cote −h d, donne : wf ,d = −w =

0+

pa + ghb r

− 0+

Le gain est donc significatif : 687/351 ≈ 2.

pa + rgh d + g(−hd ) = ghb = 9, 81 × 70 = 687 J . kg −1 r

779

et problèmes S30– 6. Puissance fournie par une pompe de distribution d’essence

La relation généralisée de Bernoulli, appliquée entre la surface libre de l’essence dans la cuve et la sortie, donne : rv s2 qv + p a + rghs − qv (0 + pa + rgha ) = P 2 La puissance réelle de la pompe est donc : Pr =

q r P = v r r

vs2 + gh 2

avec

vs =

qv pD2/4

Comme vs ≈ 2, 8 m . s−1, Pr = (2 × 10−3 × 750/0, 7) × 2, 82 /2 + 9, 80 × 3, 2 ≈ 75, 7 W. S30– 7. Écoulement d’eau dans un canal de section variable Écrivons la conservation du débit volumique (liquide incompressible) et la relation de Bernoulli sur une ligne de courant qui coïncide avec la surface libre de l’eau : l 1 h1 v 1 = l 2 h2 v2

et

v21 v2 pa pa + + gh1 = 2 + + + gh 2 r r 2 2

puisque les pressions sont égales à la pression atmosphérique. Il en résulte : v21 l2 h2 + g(h 1 − h2 ) = 12 1 2 2 2 l2 h 2

soit

On en déduit l2 et v 2 :

l2 l1

2

=

(h1 /h2)2v 21 /2 v 21 /2 + g(h1 − h 2 )

l 1h 1v 1 ≈ 1, 9 m . s−1 l2 h2 Noter que le canal est moins large et moins profond dans la section 2 que dans la section 1. l2 ≈ 1, 04 m

et v2 =

S30– 8. Lance à eau

 c Dunod – Toute reproduction non autorisée est un délit

On a, d’après le théorème d’Euler appliqué à la surface fermée de contrôle Sc entourant l’extrémité et projeté suivant la direction horizontale : qm(vs − ve ) = −ps S s + p eSe + F x

où qm = rq v = 50 kg . s−1 est le débit-masse, ps = 0 la pression manométrique à la sortie, pe la pression manométrique à l’entrée et Fx la force horizontale que l’on doit exercer pour maintenir la lance immobile. On sait que : v 2e v 2s pe avec rve Se = rve Ss = qm + = r 2 2 On en déduit : ve = 10 m . s−1 , vs = 50 m . s−1, p e = 12 bar, qm(v s − ve ) = 2 kN, p e Se = 6 kN et F x = −4 kN. Il faut donc retenir la lance en exerçant une force horizontale de 4 kN dans le sens de l’expulsion de l’eau. S30– 9. Déplacement d’un chariot à l’aide d’un jet Dans le référentiel R  , non galiléen, lié au chariot, on peut écrire, pour une surface S entourant le chariot : d P   + q m(vs − v e) = M g + R − M a dt

avec M = Mc + rsr u, qm = rs(V − v ) et (vs − v e) x = (V − v)(cos u − 1), les forces de pression atmosphérique ayant une somme nulle. Dans R , la quantité de mouvement du chariot est nulle, alors que celle du jet, suivant x, vaut : u

 Px,jet = 0

rsR(V − v ) cos a d a = rsR(V − v) (sin a)u0 = rsR(V − v ) sin u

780

Solutions des exercices

Par conséquent : d Px,jet / d t = −rsRa sin u, a étant l’accélération. Il en résulte : 2

−rsRa sin u + rs(V − v) (cos u − 1) = −(Mc + rsRu)a

d’où a =

rs(V − v)2 (cos u − 1) rsR sin u − (Mc + rsr u)

On voit que a = 0 si u = p/2 ou si V = v, ce qui était prévisible. S30– 10. Jet d’eau sur une aube D’après le théorème d’Euler appliquée sur la surface S, en pointillés, entourant l’aube, on a : qm (vs − v e ) = F avec

qm = rs(V − v )

vs = Ves − v

ve = V − v

On en déduit : F = rs(V − v )V (es − e x ), d’où les composantes : Fx = rs(V − v )V (− cos u − 1) et

Fy = rs(V − v )V sin u

On trouve Fx = −180 N et F y = 104 N. S30– 11. Réaction de jet Le théorème du centre de masse appliqué au système ouvert constitué par le réservoir perforé donne : qmu = mg + R

soit horizontalement q mu = Rt

R étant la force occasionnelle qu’il faut exercer pour maintenir le réservoir au repos. La force à exercer est donc opposée à u et de valeur Rt = 10 N. S30– 12. Force nécessaire pour maintenir immobile une plaque soumise à l’action d’un jet d’eau La force qu’exerce le jet, en l’absence de frottement, a pour expression (cf. chapitre 30) : p R = rq vv sin u ex = rsv 2 sin u e x = rsv 2 ex puisque u = 2 La force F qu’il vaut exercer pour maintenir la plaque immobile est donc l’opposée : F = 90 N. S30– 13. Conduite en forme de coude de section variable Pour trouver la force qui s’exerce sur la conduite, il suffit d’appliquer le théorème d’Euler, relatif à la quantité de mouvement, au contenu d’une surface de contrôle, de bases les sections transversales et de surface latérale la surface intérieure de la conduite. Il vient : qm(v s − ve ) = mg + Fp,e + Fp,s − R si R est la force qu’exerce le fluide sur la conduite, mg le poids et Fp,e, F p,s les forces de pression à l’entrée et à la sortie. En explicitant dans un plan horizontal, on obtient : R = −qm (vs − ve ) + Fp,e + Fp,s , ce qui donne, en projetant selon les axes x et y : Rx = −q m(v s cos u − v e) + p es e − ps ss cos u

et Ry = −qm vs sin u − ps ss sin u

Pour trouver la pression à la sortie, il suffit d’appliquer la relation de Bernoulli et la conservation du débit-volume : pe +

rv 2e rv 2 = ps + s 2 2

r 2 s2 et s eve = ss v s d’où ps = p e + v e 1 − e2 2 ss

On en déduit : Rx = 9 kN et R y = 1, 732 kN.

5

5

= 5 × 10 − 3 × 10 = 2 bar

781

et problèmes S30– 14. Équilibre d’une plaque soumise à l’action d’un jet d’eau

Sur une surface de contrôle Sc entourant la plaque, le théorème d’Euler, relatif au moment cinétique en O, donne : qm (OAs × vs − OAe × ve ) = OC × mg avec OAs × vs = 0 puisque l’eau s’écoule le long de la plaque après impact. Il en résulte, en explicitant : a −q m vd cos a = −mg sin a 2

d’où

tan a =

2rsv 2 d = 0, 57 mga

et a = 29, 7◦

S30– 15. Action d’un jet sur un auget d’une turbine Pelton Appliquons le théorème d’Euler, relatif à la quantité de mouvement, à la surface de contrôle entourant le fluide. Il vient, en régime stationnaire : qm,1 vs,1 + q m,2vs,2 − q m(v − u) = −R si R est la force qu’exerce le jet sur l’auget ; vs,1 et v s,2 désignent les vitesses du jet à la sortie par rapport à l’auget. Or, la conservation de la masse donne : qm,1 + q m,2 = qm soit qv,1 = qv,2 = qv /2, en raison de la symétrie et de l’incompressibilité du fluide. On en déduit R : R = rq v (v − u) −

1 (vs,1 + vs,2 ) 2

avec

v s,1 = v s,2 = v − u

d’après la relation de Bernoulli et l’égalité des pressions. Il en résulte : R = rq v (v − u)(1 − cos a) ex = r

pD2 v (v − u)(1 − cos a) ex 4

Comme v = (2gh)1/2 = 153, 4 m . s−1 : R = (1 000p × 0, 04/4) × 153, 4 × 133, 4 × (1 + 1/2) = 0, 96 MN. S30– 16. Anémomètre à laser

 c Dunod – Toute reproduction non autorisée est un délit

La durée nécessaire à une particule pour parcourir la distance qui sépare deux franges brillantes consécutives est : TD = i/v = l/(2v sin a/2). Le signal lumineux diffusé a donc une fréquence égale à : fD =

f Dl 2v sin a/2 d’où v = = 12 cm . s−1 l 2 sin a/2

Chapitre 31 S31– 1. Fluide entre une plaque fixe et une seconde plaque parallèle mobile 1. Comme div v = ∂v x /∂x = 0, fluide est incompressible ; on peut donc définir une fonction de courant. En outre : U ∂y U 1 V = rot v = − ez = − ez 2 2h ∂y 2h Ainsi V = 0 : le champ de vitesse ne dérive pas d’un potentiel.

782

Solutions des exercices 2. La fonction de courant C s’obtient en intégrant les équations : vx =

∂C Uy =− h ∂y

et v y = 0 =

∂C ∂x

d’où C =

Uy 2 + f (x) avec 2h

∂C df = =0 dx ∂x

Finalement, on a, C étant une constante additive : C = Uy2/(2h) + C. S31– 2. Viscosimètre de Couette 1. Compte tenu du mouvement orthoradial imposé au fluide par Ce et de la symétrie cylindrique, la vitesse du fluide n’a qu’une composante orthoradiale qui ne dépend que de r. L’équation de Navier-Stokes s’écrit, en régime stationnaire : r(v · grad )v = − grad p + rg + hDv ce qui donne, en projection orthoradiale : r vu

1 ∂v u r ∂u

=−

∂p d 1 d +h (rv u) dr r dr r∂u

Puisque vu et p ne dépendent que de r , on obtient : 0=h

d dr

1 d (rvu ) r dr

d’où

1 d(rv u ) = Cte r dr

2. Il vient, en intégrant l’équation précédente, si C1 désigne la constante : d(rvu) = C1r dr

et r vu =

Cr 2 + C2 2

On détermine aisément C2 à l’aide de la relation : Rvu =

C1 R2 + C2 = 0 2

d’où

C2 =

C 1R 2 2

et vu =

C1 2

r−

R2 r

Comme vu = V eR e pour r = Re , on a : Ve Re =

C 2

Re −

R2 Re

d’où

Ve R e C1 = 2 R e − R2 /R e

3. Calculons le gradient : d vu = V eR e dr Pour r = R :

et vu = V e R e

1 + R 2 /r2 Re − R2 /R e

r − R2 /r Re − R2 /R e

et V = V eR e

R

Ve Ve 1 + R 2/r 2 2 × = =2 2 2 1−u 1+u 1−u Re − R /Re R R On détermine la viscosité à l’aide du moment des forces de viscosité qui vaut : d vu dr

= Ve Re

2

G O = h2pR h2 ×

Ve Ve 2 hhu 2 × e ez 2 z = 4pRe 1−u 1 − u2

L’erreur faite sur la mesure de la viscosité, en remplaçant 2/(1 + u) par 1, sera inférieure à e = 0, 01 si : 2 1−u − 1  e soit ε  1+u 1+u

ou u 

1 −ε 0,99 ≈ 0,98 = 1 +ε 1,01

Ainsi, il faut que l’on ait 0, 98 Re < R < Re et donc 4, 9 cm < R < 5 cm.

783

et problèmes S31– 3. Liquide visqueux en rotation

1. Compte tenu du mouvement orthoradial imposé au fluide par le récipient et de la symétrie cylindrique, la vitesse du fluide n’a qu’une composante orthoradiale vu qui ne dépend que de r. L’équation de Navier-Stokes s’écrit, en régime stationnaire : r(v · grad )v = − grad p + rg + hDv avec p(r, z). Les trois projections, suivant r, u et z, de cette équation sont : r vu

1 ∂vu r ∂u

=−

∂p ∂r

r vu

1 ∂v u r ∂u

=0=h

d 1 d (rv u ) dr r dr

0=−

∂p − rg dz

La deuxième équation donne : 1 d(rvu ) r2 C1 r C 2 + = C1 d’où rvu = C1 + C 2 et v u = 2 2 r dr r Comme la vitesse ne doit pas être infinie au centre, C2 = 0. En outre : C1 R 2

(v u)r=R = RV =

ce qui implique C 1 = 2V et finalement vu = rV

2. On a, puisque vr = 0 et vz = 0 (cf. annexe 4) : v2 r2 V2 = 2 2

et

rot v =

1 r

rot v =

1 ∂ (rvu) e z = 2V ez r ∂r

∂vz ∂vu − r∂u ∂z

1 ∂v r ∂vz eu+ − ∂z ∂r r

er +

1 ∂vr ∂ (rv u) − ∂r r ∂u

soit : et

rot v × v = −2rV2 er

Par conséquent (v · grad)v = −rV2 er

 c Dunod – Toute reproduction non autorisée est un délit

3. On obtient le champ de pression en intégrant les deux autres équations différentielles : −rrV2 = −

∂p ∂r

0=−

∂p − rg dz

Il vient, en intégrant d’abord la deuxième équation et en injectant sa solution dans la première : p = −rgz + C1 (r)

Par conséquent : C1 (r) = r

V 2r 2 + C2 2

et

2 − rrV = −

d C1 dr

et finalement p = −rgz + r

V2r2 + C2 2

On détermine la constante C2 en introduisant la pression atmosphérique p0 pour r = 0 et z = z0 : p0 = rgz 0 + C2

d’où p = −rg(z − z0) + r

V2 r 2 + p0 2

L’équation de la surface libre du liquide s’obtient en faisant p = p0 : z = z 0 + V2r 2/(2g).

ez

784

Solutions des exercices

S31– 4. Viscosité de l’huile de ricin 1. Le bilan des forces, poids, poussée d’Archimède, traînée, permet d’écrire, selon un axe vertical descendant : m

dv  = mg − m g − T dt

avec

m=

4pr3 r b 3

ce qui s’écrit : dv v + = a0 t dt

avec

t=

 m =

2r2 r b 9h

4pr 3rh 3

et T = 6phr v

et a 0 = g 1 −

rh rb

La solution de cette équation différentielle est, puisque v = 0 à l’instant intitial : v = Cte × exp −

t t

+ a0 t soit

v = a0 t 1 − exp −

t t

2. Il résulte de ce qui précède que la vitesse limite vl vaut a0t : vl = a0 t = g(rb − rh)

2r 2 9h

d’où h =

2g(r b − rh )r2 = 1, 16 Pa . s 9vl

S31– 5. Traînée d’une balle de fusil Le calcul donne :

T = Cx S

rv 2 = 3, 26 N 2

mg = 0, 098 N

d’où

T ≈ 33 mg

S31– 6. Freinage d’un avion par un parachute 1. Appliquons le théorème du centre de masse à l’avion. Il vient md v/ d t = mg + Ff , ce qui donne en projection horizontale : m

rSv 2 dv = −bv 2 d’où = −Cx dt 2

b dv = − dt 2 v m

et

b 1 1 = t+ v v0 m

en intégrant et tenant compte de la vitesse initiale v0 . 2. Intégrons une nouvelle fois pour obtenir la position : v=

v0 1 + bv0t/m

donne x = v 0

dt bv0t m + Cte = ln 1 + b 1 + bv 0 t/m m

puisque l’origine des abscisses est prise à t = 0. Concrètement b = rS Cx/2 = 12, 25 m/b = 326, 5 m. La durée au bout de laquelle la vitesse vaut 5 m . s−1 est t ≈ 59 s, d’où : d = 326, 5 ln 10 ≈ 753 m. Le parachute ne suffit donc pas pour ralentir l’avion sur une distance faible ; aussi un freinage mécanique doit-il être utilisé simultanément. S31– 7. Propulsion d’un bateau à voile 1. On voit, à l’aide de la figure 31.13 que Sa = Sv sin f.

2. Pour obtenir le facteur C x , appliquons le théorème d’Euler : q m (v2 − v1) = F + mg soit

qm (v 2,x − v1,x ) = F x

en projection horizontale, F étant la force qu’exerce la voile sur le vent. Par conséquent, la force qu’exerce horizontalement le vent sur la voile est, puisque qm = rSa v = rS vv sin f : −F x = rSv v sin f(v1,x − v 2,x).

785

et problèmes

Or la différence des vecteurs vitesses sécrit : v1 − v2 = 2v sin f e, e étant le vecteur unitaire normal à la voile. Ce vecteur fait, avec l’axe Ox, l’angle a + p − f + p/2 = a − f − p/2. Il en résulte que : −Fx = rS v 2v 2 sin2 f cos a − f −

p 2

=

rSvv 2 2 4 sin f sin(a − f) 2

2

d’où Cx = 4 sin f sin(a − f)

S31– 8. Anémomètre à coupelles Comme la vitesse de rotation de l’anémomètre est constante, le moment par rapport à l’axe des actions exercées par le vent est nul : rSb 2 2  [C x (u − bV) − Cx (u + bV) ] = 0 2

d’où

u + bV = (u − bV)g

1/ 2

et u =

g1/2 + 1 bV g1/2 − 1

Comme g = 3, 74 , on trouve u = 29, 6 m . s−1, soit 106, 6 km . h−1 . S31– 9. Petit animal en mouvement dans l’eau 1. Le nombre de Reynolds se calcule aisément : Re =

rvr 1 000 × 3 × 10−5 × 10−6 ≈ 3 × 10 −2 = h 1, 1 × 10−6

2. Comme la masse volumique de l’animal est voisine de celle de l’eau, son poids est compensé par la poussée d’Archimède. En outre, Re  1 implique une force de frottement de type Stokes FS = −av avec a = 6phr . Il en résulte : dv dv F a =− v+ m = −av + F soit dt dt m m avec t = m/a = 2r2 r/(9h) ≈ 2 × 10−12 × 103 /(9 × 1, 1 × 10−3) ≈ 0, 2 ms . 3. a) Pour établir la relation demandée entre Dt et Dx , intégrons l’équation différentielle. Il vient, en projetant selon l’axe Ox du mouvement, la force F0 = −F0 ex étant opposée au mouvement : 0

 c Dunod – Toute reproduction non autorisée est un délit

V0

dv = −

1 t

Dx 0

dx −

F0 m

dt

d’où

− v0 = −

Dx F0 − Dt m t

Il en résulte que Dx = v0 t − vFDt.

b) L’équation différentielle du mouvement donne : dt = −

dv dv = −t v + vF v/t + F0 /m

d’où en intégrant Dt = −t ln {v + vF} 0v = t ln 1 + 0

c) Comme Dt ≈ 0, 6 ms et t ≈ 0, 2 ms , il vient : 3 = ln 1 +

v0 vF

soit vF =

v0 ≈ 1, 57 mm.s −1 exp(3) − 1

Ainsi F0 = mvF /t = av F = 6phrvF ≈ 33 fN et Dx = v0 t − vFDt ≈ (30 × 0, 2 − 1, 57 × 0, 6) × 10−12 ≈ 5 pm

v0 vF

786

Solutions des exercices

Chapitre 32 S32– 1. Vitesse de propagation du son dans l’air. Influence d’un gradient de température 1. On sait que :

1 7 288 gp gRT = = = × 8, 314 × M rk r 5 0, 029 et th = D/cs = 10 000/340 = 29, 4 s. c 2s =

d’où cs = 340 m . s −1

2. Comme la température varie avec l’altitude z, calculons la durée élémentaire d t : dt = −

dz =− cs

1 gRT0 /M

1/ 2

dz 1 dz 1 =− = 1 / 2 1 / 2 (T /T 0 ) cs,0 (1 − Bz/T0 ) c s,0

T0 B

du u 1/ 2

en posant u = 1 − Bz/T0, ce qui donne en intégrant : tv =

1 c s,0

2

T0 B

1−

Bz T0

1/ 2 0

= D

BD 2T0 1− 1− Bcs,0 T0

1/ 2

= 31, 3 s

L’écart relatif est donc (th − t v)/th = −0, 006 5. S32– 2. Résonateur d’Helmholtz Appliquons le théorème de la quantité de mouvement à un élément de fluide de l’ajutage, de longueur l. Dans le mouvement autour de la position de repos, seules les forces de pression interviennent. On a donc, suivant la direction horizontale Ox, en désigant par x le déplacement de l’élément : rsl

d2 x = s(p − p a ) avec dt2

p − pa sx +g = 0 pa V0

d’après la loi de Laplace pV g = Cte à laquelle satisfait l’air supposé parfait et diatomique (cf. Thermodynamique). On a donc : 1/ 2 d2 x d2 x gs2pa gspa 2 + v = rsl 2 + x = 0 d’où avec v0 0x = 0 V0 rlV0 dt d t2 On en déduit la fréquence d’oscillation et la longueur d’onde de l’onde acoustique qui se propage : v 340 cs f0 = 0 = 165 Hz l= = 2, 06 m ≈ 2p 165 f0 Cette longueur est très grande devant le rayon de la sphère qui vaut : R = [3V0 /(4p)]1/3 ≈ 6, 2 cm. S32– 3. Isolation acoustique On sait que le facteur de transmission en intensité acoustique s’écrit simplement en fonction des impédances acoustiques : 4Zc,1 Zc,2 T 1,2 = (Zc,1 + Zc,2 ) 2 Dans le cas de la pierre, on a, puisqu’il existe deux interfaces : Tp = Ta2,b =

4Z c,1 Zc,2 (Zc,1 + Z c,2 )2

2

= 58 × 10−9

d’où 10 lg Tp = −72, 3 dB

Dans le cas du double vitrage, il y a quatre interfaces. Par conséquent : Tv = T4a,v =

4Zc,1 Zc,3 (Zc,1 + Zc,3) 2

4 −15

≈ 10

L’atténuation est donc bien meilleure avec le double vitrage.

d’où 10 lg T v ≈ −150 dB

787

et problèmes

S32– 4. Vitesse de phase et vitesse de groupe dans un pavillon exponentiel L’onde acoustique qui se propage dans le pavillon a pour expression (cf. Optique) : C = A exp −

ax 2

exp −iv t −

x vw

2

où v w =

v2 c2s = v 2 /c2s − a2 /4 1 − v2c /v2

avec vc = acs /2. La vitesse de phase dans le pavillon est donc : vw = c s/(1− v2c /v 2)1/2 . On en déduit la vitesse de groupe vg = d v/ d k : k2 =

v2 a2 − 2 cs 4

donne en différentiant vg =

k a 2v 2 dv = c2s = cs 1 − v dk 4v2

1/ 2

= cs 1 −

v 2c v2

1/ 2

On en déduit la relation : vw vg = c 2s. S32– 5. Tunnel acoustique à gradient de célérité dans les océans 1. Comme la vitesse de phase c s du signal sonore ne varie pas suivant la direction horizontale, l’équation de la trajectoire donne en projection horizontale : d dx dx = 0 d’où = Cte = C soit d s cs d s csds ce qui rappelle la loi de Snell-Descartes (cf. Optique).

sin i = C avec cs

C=

sin im = 0, 66 × 10 −3 s . m −1 vm

2. En projection suivant l’axe vertical descendant Oz, on a, puisque u · ez = d z/ d s : d ds

dz cs d s

=

d(1/cs) dz

d’où

d dz C dx dx

dx 1 d(z − z m )2 ≈− 2 ds 2r vm dz

Il en résulte, C = sin im/vm étant une constante : C2 vm

d2 z z − zm ≈− 2 2 r vm dx

et

d2 z z − zm + 2 2 ≈0 2 dx r sin im

Ainsi, a = 1. La trajectoire est donc une sinusoïde, de période p = 2pr sin im = 12, 5 km, à la profondeur z m. S32– 6. Effet Doppler 1. La fréquence percue par les passagers de A2 avant le croisement a pour expression (cf. chapitre 32) : 1 + v 2 /cs 1 − v 1 /cs puisque, avant le croisement, la source A1 se rapproche à la vitesse v 1 et le récepteur se rapproche à la vitesse v2. Comme v1 = 20 m . s−1 et v2 = 25 m . s −1, on trouve :  c Dunod – Toute reproduction non autorisée est un délit

fr,av = f 1

fr,av = 1 ×

1 + 25/340 = 1, 14 kHz 1 − 20/340

Pour obtenir l’expression analogue après le croisement, il suffit de changer les signes de v1 et v2 : 1 − v2/cs 1 − 25/340 f r,ap = f 1 = 1× = 0, 875 kHz 1 + v1/cs 1 + 20/340 2. Le raisonnement est analogue ; il suffit de permuter les indices 1 et 2 : fr,av = f2

1 + v1 /cs 1 + 20/340 = 1, 5 × = 1, 71 kHz 1 − v2 /cs 1 − 25/340

puisque, avant le croisement, la source A2 se rapproche à la vitesse v2 et le récepteur se rapproche à la vitesse v1 . Pour obtenir l’expression analogue après le croisement, il suffit de changer les signes de v1 et v2 : fr,ap = f1

1 − v1/c s 1 − 20/340 = 1, 5 × = 1, 315 kHz 1 + v2/c s 1 + 25/340

788

Solutions des exercices

S32– 7. Onde de choc 1. Écrivons que, pendant la durée élémentaire d t, alors que la source se déplace de S à S , l’onde acoustique parcourt la distance SO − SO = − d r ( d r < 0) : SS SO − S O = v cs

dl dr =− v cs

soit

avec

2 2 2 2 d l = dr + r du

On a donc, en introduisant le rapport m = v/cs (vitesse en mach) : 1+

r2 d u2 = m2 dr2

d’où

1 dr 1 =− 2 r du (m − 1) 1/2

dr